Barbu_TEOREME_FUNDAMENTALE(2008).pdf

500
8 Cătălin Barbu TEOREME FUNDAMENTALE din GEOMETRIA TRIUNGHIULUI

Transcript of Barbu_TEOREME_FUNDAMENTALE(2008).pdf

Page 1: Barbu_TEOREME_FUNDAMENTALE(2008).pdf

8

Cătălin Barbu

TEOREME FUNDAMENTALE din

GEOMETRIA TRIUNGHIULUI

Page 2: Barbu_TEOREME_FUNDAMENTALE(2008).pdf

9

CAPITOLUL I

PUNCTE, DREPTE ŞI CERCURI REMARCABILE

ASOCIATE UNUI TRIUNGHI

I.1. Centrul de greutate al unui triunghi

„Drumul de o mie de mile începe cu primul pas.”- Lao Tse1

Punctul de concurenţă al medianelor unui triunghi ABC se numeşte centrul de greutate al triunghiului ABC (se notează de obicei cu G) . Centrul de greutate al unui triunghi este un punct interior triunghiului.

1) Centrul de greutate al unui triunghi se află pe fiecare mediană la o treime de mijlocul laturii opuse corespunzătoare şi la două treimi de vârful corespunzător. Demonstraţie: Fie

a b cM M M triunghiul median al

triunghiului ABC. Teorema lui Menelaus aplicată triunghiului

aAM C şi transversalei

bBGM ne dă:

1b a

a b

M C BMGA

GM M A BC⋅ ⋅ = sau 2

aGA GM= , deci

1

3a aGM AM= şi

2

3 aGA AM= .

2) Distanţele de la centrul de greutate al unui triunghi la vârfurile triunghiului sunt

egale cu: 2 2 2 2 2 2 2 2 21 1 12( ) , 2( ) , 2( )

3 3 3+ − + − + −b c a a c b b a c .

Demonstraţie. Deoarece 2

3 aGA AM= , rezultă 2 2 2 2 2 22 2 1 1

2( ) 2( )3 3 2 3

= = ⋅ + − = + −a

GA AM b c a b c a ,

unde am utilizat teorema medianei. Analog, 2 2 212( )

3= + −GB a c b şi

2 2 212( ) .

3= + −GC a b c

3) Distanţele de la centrul de greutate al unui triunghi la laturile triunghiului sunt egale

cu: 1 1 1

, ,3 3 3a b ch h h , unde , ,

a b ch h h sunt lungimile înălţimilor triunghiului ABC.

Demonstraţie. Fie a

G şi a

H proiecţiile punctelor G, respectiv A pe BC. Din asemănarea

triunghiurilor a a

GG M şi a a

AH M rezultă 1

3= =a a

a a

GG GM

h AM, deci

1.

3=

a aGG h

1 Lao Tse (sec IV î.e.n.) – filosof chinez, figură centrală în religia taoistă

A

B C aM

bM cM

G

Fig. 1

Page 3: Barbu_TEOREME_FUNDAMENTALE(2008).pdf

10

4) Centrul de greutate al unui triunghi aparţine dreptei lui Euler a triunghiului. Demonstraţie. Vezi „Dreapta lui Euler”. 5) Consecinţă: Centrul de greutate, centrul cercului circumscris, şi ortocentrul unui

triunghi ABC sunt puncte coliniare şi 1 1

2 3= =GO HG OH .

Demonstraţie. Vezi „Dreapta lui Euler”.

6) Dacă L este punctul lui Longchamps al unui triunghi ABC, atunci 1

3=GO OL .

Demonstraţie. Vezi „Punctul lui Longchamps”.

7) Dacă G este centrul de greutate al unui triunghi ABC, atunci2 2 2

2 2

9

a b cOG R

+ += − .

Demonstraţie. Vezi „Teorema lui Leibniz”. 8) Pentru orice punct M din planul triunghiului ABC este adevărată relaţia:

3

MA MB MCMG

+ +=

uuur uuur uuuuruuuur

.

Demonstraţie. Din teorema medianei scrisă vectorial avem: 2a

MM MB MC= +uuuuur uuur uuuur

. Din

2a

GA

GM= rezultă

2

1 2 3a

MA MM MA MB MCMG

+ + += =

+

uuur uuuuur uuur uuur uuuuruuuur

(1).

Consecinţe:

2.1) Dacă M G≡ relaţia (1) devine: 0GA GB GC+ + =uuur uuur uuur r

.

2.2) Dacă M O≡ relaţia (1) devine: 3OA OB OC OG OH+ + = =uuur uuur uuur uuur uuuur

(relaţia lui Sylvester).

2.3) Dacă M A≡ relaţia (1) devine: .3

AB ACAG

+=

uuur uuuruuur

9) Coordonatele baricentrice absolute ale centrului de greutate al unui triunghi ABC

sunt 1 1 1, ,

3 3 3G

.

Demonstraţia rezultă din proprietatea precedentă.

10) Afixul centrului de greutate al unui triunghi ABC este egal cu .3

+ += A B C

G

z z zz

Demonstraţie analoagă cu cea din teorema (8).

11) Fie G centrul de greutate al triunghiului ABC. Pentru orice punct M din planul

triunghiului ABC este adevărată relaţia: 2 2 2

2 2 2 23 .3

AB BC CAMA MB MC MG

+ ++ + = +

Demonstraţie. Vezi „Teorema lui Leibniz”.

Page 4: Barbu_TEOREME_FUNDAMENTALE(2008).pdf

11

12) În orice triunghi ABC este adevărată relaţia: 2 2 2

2 2 2

3

+ ++ + =

AB BC CAGA GB GC .

Demonstraţia rezultă din teorema precedentă pentru .≡M G

13) În orice triunghi ABC este adevărată relaţia: 4 4 4

4 4 4

9

+ ++ + =

a b cGA GB GC .

Demonstraţie. Ridicând la pătrat relaţia 2 2 2 21[2( ) ]

9= + −AG b c a rezultă:

4 4 4 2 2 4 2 2 21[4( 2 ) 4 ( )],

81= + + + − +AG b c b c a a b c de unde:

4 4 44 4 4

9

+ ++ + =

a b cGA GB GC .

14) Centrul de greutate al unui triunghi ABC aparţine dreptei lui Nagel corespunzătoare. Demonstraţie. Vezi „Dreapta lui Nagel”. 15) O dreaptă d, care nu este paralelă cu BC şi trece centrul de greutate G al triunghiului ABC , intersectează laturile AB şi AC în punctele M respectiv N. Atunci:

1.BM CN

MA NA+ =

Demonstraţie. Fie a

M mijlocul laturii BC şi

fie D, E, F, L proiecţiile punctelor , ,a

B M C

respectiv A pe dreapta d (Fig. 2). Avem:

, 2 , 22

+= = =a a a

BD CFM E GA GM AL M E

(deoarece triunghiurile ALG şi a

M EG sunt

asemenea) , de unde rezultă că: .AL BD CF= + Din asemănarea triunghiurilor

BDM şi ALM precum şi a triunghiurilor

CFN şi ALN rezultă: BM BD

MA LA= şi

=CN CF

NA LA, deci: 1.

BM CN BD CF LA

MA NA LA LA LA+ = + = = Dacă d BC , problema este banală.

16) Fie P un punct în interiorul triunghiului .ABC Prin punctul P ducem paralelele ,PL PM şi PN la laturile BC, AC respectiv AB ( , ,L AB M BC N AC∈ ∈ ∈ ). Dacă ariile triunghiurilor BPL, CPM şi APN sunt egale, atunci P este centrul de greutate al triunghiului .ABC

Demonstraţie. Fie 'L PL AC= ∩ (Fig. 3).

Atunci, [ ] [ ] [ ]' .BPL CPM CPLA A A= = Cum

' ,LL BC rezultă că înălţimile din B şi C ale triunghiurilor BPL şi respectiv 'CPL sunt egale şi deci ',PL PL= adică P aparţine

A

B C

G D E

F

aM

L M

N d

Fig. 2

A

B C M

N

P L

M'

N'

L '

Fig. 3

Page 5: Barbu_TEOREME_FUNDAMENTALE(2008).pdf

12

medianei ce pleacă din A. Analog, se arată că punctul P aparţine şi celorlalte mediane, deci P este centrul de greutate al triunghiului .ABC Reciproca este evident adevărată.

I.2. Centrul cercului circumscris unui triunghi

„Fie să-mi clipească vecinice, abstracte, Din culoarea minţii, ca din prea vechi acte. Eptagon cu vârfuri stelelor la fel.

Şapte semne, puse ciclic.” – Ion Barbu2

Punctul de intersecţie al mediatoarelor unui triunghi ABC se numeşte centrul

cercului circumscris triunghiului ABC (se notează de obicei cu O). Raza acestui cerc se numeşte raza cercului circumscris triunghiului ABC (se notează de obicei cu R).

Observaţii: i) Centrul cercului circumscris unui triunghi ascuţitunghic se află în interiorul triunghiului (Fig. 4 ). ii) Centrul cercului circumscris unui triunghi dreptunghic se află la mijlocul ipotenuzei. Raza acestui cerc are lungimea jumătate din lungimea ipotenuzei (Fig. 5 ). iii) Centrul cercului circumscris unui triunghi obtuzunghic se află în exteriorul triunghiului (Fig. 6). 1) Triunghiul podar al centrului cercului circumscris unui triunghi ABC este triunghiul median al acestuia. 2) Fie ' ' 'A B C triunghiul pedal al centrului cercului circumscris triunghiului ABC. Atunci,

' sin 2,

' sin 2

A B C

A C B=

' sin 2

' sin 2

B C A

B A C= şi

' sin 2.

' sin 2

C A B

C B A=

2 Ion Barbu (1895-1961) – matematician român, profesor la Universitatea din Bucureşti, contribuţii în algebră şi

geometrie

A

B C

O

Fig. 4

aM

bM cM A

B C

O

Fig. 5

aM

cM

A

B C

O

Fig. 6

aM bM cM

A

B C

O

Fig. 7

aM

bM cM

A'

B'

C'

A"

B" C"

Page 6: Barbu_TEOREME_FUNDAMENTALE(2008).pdf

13

Demonstraţie. Avem: ( ') ( )m BAA m BAO= = 1[180 2 ( )]

2m C°− = 90 ( )m C° −

şi ( ') ( ) 90 ( )m CAA m CAO m B= = °− (Fig. 7). Din teorema sinusurilor aplicată în

triunghiurile 'ABA şi 'ACA rezultă: ' '

sin( / 2 ) sin

A B AA

C Bπ=

−, respectiv

' '

sin( / 2 ) sin

A C AA

B Cπ=

−, de unde

' sin cos sin 2

' sin cos sin 2

A B C C C

A C B B B= = . Analog se arată că

' sin 2

' sin 2

B C A

B A C= şi

' sin 2.

' sin 2

C A B

C B A=

3) Fie ' ' 'A B C triunghiul pedal al centrului cercului circumscris triunghiului ABC.

Atunci, sin 2 sin 2 sin 2 sin 2 sin 2 sin 2

, , .' sin 2 ' sin 2 ' sin 2

AO B C BO C A CO A B

OA A OB B OC C

+ + += = =

Demonstraţie. Din teorema lui Van-Aubel rezultă: ' ' sin 2 sin 2

' ' ' sin 2

AO AB AC B C

OA B C C B A

+= + = .

Analog se arată celelalte egalităţi. 4) Fie O centrul cercului circumscris unui triunghi ABC. Pentru orice punct M din

planul triunghiului este adevărată egalitatea: sin2 sin2 sin2

sin2 sin2 sin2

A MA B MB C MCMO

A B C

⋅ + ⋅ + ⋅=

+ +

uuur uuur uuuuruuuur

.

Demonstraţie. Din sin 2 sin 2

' sin 2

AO B C

OA A

+= rezultă

sin 2 sin 2'

sin 2sin 2 sin 2

1sin 2

B CMA MA

AMOB C

A

++

=+

+

uuur uuuur

uuuur (1), iar

din ' sin 2

,' sin 2

A B C

A C B= rezultă

sin 2sin 2 sin 2sin 2'

sin 2 sin 2 sin 21sin 2

CMB MC

B MB C MCBMAC B C

B

+ ⋅ + ⋅= =

++

uuur uuuuruuur uuuur

uuuur (2). Din

relaţiile (1) şi (2) rezultă concluzia.

Observaţie: Ţinând cont de identitatea 2

2sin 2 sin 2 sin 2 4sin sin sin

SA B C A B C

R+ + = = ,

unde S reprezintă aria triunghiului ABC, egalitatea demonstrată anterior devine: 2

(sin 2 sin 2 sin 2 ).2

RMO A MA B MB C MC

S= ⋅ + ⋅ + ⋅

uuuur uuur uuur uuuur

Cazuri particulare:

1) Dacă M O≡ obţinem: sin 2 sin 2 sin 2 0.A OA B OB C OC⋅ + ⋅ + ⋅ =uuur uuur uuur ur

2) Dacă M G≡ obţinem: sin 2 sin 2 sin 2

sin 2 sin 2 sin 2

A GA B GB C GCGO

A B C

⋅ + ⋅ + ⋅=

+ +

uuur uuur uuuruuur

.

3) Dacă M A≡ obţinem sin 2 sin 2

sin 2 sin 2 sin 2

B AB C ACAO

A B C

⋅ + ⋅=

+ +

uuur uuuruuur

.

Page 7: Barbu_TEOREME_FUNDAMENTALE(2008).pdf

14

5) Coordonatele baricentrice absolute ale centrului cercului circumscris unui triunghi

ABC sunt: 2 2 2

sin 2 , sin 2 , sin 22 2 2

R R RO A B C

S S S

.

Demonstraţia rezultă din proprietatea precedentă. 6) Fie , ,

A B Cz z z afixele vârfurilor unui triunghi ABC. Afixul centrului cercului

circumscris triunghiului ABC este egal cu sin 2 sin 2 sin 2

sin 2 sin 2 sin 2A B C

O

A z B z C zz

A B C

⋅ + ⋅ + ⋅=

+ +.

Demonstraţia rezultă din proprietatea (3). 7) Coordonatele unghiulare ale centrului cercului circumscris unui triunghi

ascuţitunghic ABC sunt egale cu: ( ) 2 ( ), ( ) 2 ( )m BOC m A m COA m B= = şi

( ) 2 ( ).m AOB m C= Demonstraţia este evidentă deoarece – de exemplu – BOC este unghi la centru, deci are

măsura egală cu măsura arcului BC .

8) Raza cercului circumscris unui triunghi oarecare este egală cu 4

=abc

RS

, unde a, b, c

sunt lungimile laturilor triunghiului şi S este aria acestuia. Demonstraţie. Vezi „Aria unui triunghi”. 9) Consecinţă: Raza cercului circumscris unui triunghi echilateral de latură l este

3

3

lR = .

10) Distanţele de la centrul cercului circumscris unui triunghi ascuţitunghic ABC la

laturile triunghiului sunt egale cu , , .2 2 2⋅ ⋅ ⋅

a b cctgA ctgB ctgC

Demonstraţie. Avem cos cos2sin 2

= = = ⋅a

a aOM R A A ctgA

A. Analog,

2= ⋅b

bOM ctgB şi

.2

= ⋅c

cOM ctgC

11) Centrul cercului circumscris unui triunghi aparţine dreptei lui Euler a triunghiului. Demonstraţie. Vezi „Dreapta lui Euler”. Consecinţe: 12.1 - Centrul cercului circumscris, centrul de greutate şi ortocentrul unui triunghi ABC sunt puncte coliniare. 12.2 - 3OH OG OL= = şi 2HG OG= (vezi „Dreapta lui Euler”şi „Punctul lui Longchamps (L)”). 13) Dacă G este centrul de greutate al unui triunghi ABC,

atunci2 2 2

2 2

9

a b cOG R

+ += − .

Demonstraţie. Vezi „Teorema lui Leibniz”.

Page 8: Barbu_TEOREME_FUNDAMENTALE(2008).pdf

15

14) Dacă H este ortocentrul triunghiului ABC, atunci 2 2 2 2 29 ( ).OH R a b c= − + + Demonstraţie. Vezi „Teorema lui Leibniz”. 15) Dacă H este ortocentrul triunghiului ABC, atunci 2 2 (1 8cos cos cos ).= −OH R A B C Demonstraţie. Puterea punctului H faţă de cercul circumscris triunghiului ABC este egală

cu: 2 2 22= ⋅ = −H aP AH HH R OH sau 2 22 cos 4 cos cos⋅ = −R A R B C R OH , 2 2 28 cos cos cos = −R A B C R OH de unde rezultă concluzia.

16) Consecinţă: Din teoremele (13) şi (14) rezultă

2 2 2 28 (1 cos cos cos ).+ + = +a b c R A B C

17) Dacă I este centrul cercului înscris în triunghiul ABC, atunci 2 2 2= −OI R Rr . Demonstraţie. Vezi „Cercul înscris”. 18) Dacă

aI este centrul cercului A - exînscris în triunghiul ABC, atunci

2 2 2a a

OI R Rr= + . Demonstraţie. Vezi „Cercuri exînscrise”. 19) Dacă K este punctul lui Lemoine al triunghiului ABC, atunci

2 2 22 2

2 2 2 2

3

( )

a b cOK R

a b c= −

+ +.

Demonstraţie. Egalitatea 2 2 2 2 2 2 2 2 2

22 2 2 2 2 2 2

3

( )

a MA b MB c MC a b cMK

a b c a b c

+ += −

+ + + + (vezi

„Teorema lui Van-Aubel”) pentru M O≡ , devine 2 2 2

2 22 2 2 2

3

( )

a b cOK R

a b c= −

+ +.

20) Într-un triunghi ABC, distanţa de la punctul lui Nagel (N) la centrul cercului circumscris (O) este egală cu diferenţa dintre raza acestui cerc şi diametrul cercului înscris ( 2ON R r= − ). Demonstraţie. Vezi „Punctul lui Nagel”. 21) Pentru un triunghi ABC, fie O centrul cercului circumscris, H ortocentrul său, I centrul cercului înscris triunghiului, N punctul lui Nagel al triunghiului ABC . Segmentele HI şi ON sunt congruente. Demonstraţie. Vezi „Punctul lui Nagel”. 22) Centrul cercului circumscris şi ortocentrul unui triunghi sunt puncte izogonal conjugate. Demonstraţie. Vezi „Drepte izogonale”.

23) Într-un triunghi ABC este adevărată egalitatea: OH OA OB OC= + +uuuur uuur uuur uuur

(relaţia lui Sylvester). Demonstraţie. Vezi „Centrul de greutate al unui triunghi”.

Page 9: Barbu_TEOREME_FUNDAMENTALE(2008).pdf

16

I.3. Cercul înscris într-un triunghi

„Matematica nu deţine adevărul absolut, ci doar frumuseţea supremă – o frumuseţe rece şi austeră, ca a o sculptură având o puritate sublimă capabilă de a atinge perfecţiunea.” – Bertrand Russell3

1) Bisectoarele interioare ale unui triunghi sunt concurente. Demonstraţie: Fie triunghiul ABC şi

', ', 'A B C picioarele bisectoarelor unghiurilor

, ,A B C iar ' '= ∩I BB CC (Fig. 8). Fie

, ,a b c

C C C proiecţiile punctului I pe laturile

, , .BC CA AB Din congruenţa triunghiurilor

aBC I cu

cBC I , respectiv

aCC I cu

bCC I

rezultă că ≡a c

C I C I şi ≡a b

C I C I , de unde

rezultă ≡c b

C I C I adică punctul I aparţine şi

bisectoarei 'AA .

Observaţii:

1) Deoarece punctul I se află la egală distanţă faţă de laturile triunghiului ABC , el este centrul unui cerc tangent interior laturilor triunghiului ABC - cercul se numeşte cercul înscris în triunghiul ABC . 2) Punctul I de concurenţă al bisectoarelor interioare unghiurilor triunghiului ABC se numeşte centrul cercului înscris în triunghiul ABC . 3) Raza cercului înscris în triunghiul ABC o vom nota cu r. 4) Triunghiul

a b cC C C ale cărui vârfuri sunt punctele de tangenţă dintre laturile triunghiului

şi cercul înscris se numeşte triunghiul de contact al triunghiului ABC . 2) Distanţele de la centrul cercului înscris într-un triunghi la laturile triunghiului sunt egale cu raza cercului înscris în acest triunghi. 3) Distanţele de la centrul cercului înscris într-un triunghi la vârfurile triunghiului sunt

egale cu , ,sin sin

2 2

r r

A Brespectiv .

sin2

r

C

Demonstraţie. Din triunghiul c

AIC rezultă sin

2

=r

AIA

; analog sin

2

=r

BIB

şi sin

2

=r

CIC

.

4) Fie I centrul cercului înscris în triunghiul ABC. Atunci, 4 sin sin2 2

= ⋅B C

AI R .

3 Bertrand Russell (1872 - 1970) – filosof, logician şi matematician englez, laureat al Premiului Nobel pentru literatură

A

B C

Fig. 8

A'

A"

B"

C"

Cc

Cb

Ca

I O

r

Page 10: Barbu_TEOREME_FUNDAMENTALE(2008).pdf

17

Demonstraţie. Avem 4 sin sin sin

2 2 2 4 sin sin .2 2sin sin

2 2

= = =

A B CR

r B CAI R

A A Analog,

4 sin sin2 2

= ⋅C A

BI R şi 4 sin sin2 2

= ⋅A B

CI R .

5) Dacă I este centrul cercului înscris în triunghiul ABC , atunci

1( ) 90 ( )

2= °+ m BIC m BAC ,

1( ) 90 ( )

2= °+ m AIB m ACB şi

1( ) 90 ( )

2= °+ m CIA m ABC .

Demonstraţie. ( ) ( ') ( ' ) [ ( ) ( )] [ ( ) ( )]= + = + + + m BIC m BIA m A IC m BAI m ABI m CAI m ICA

[ ]1 1( ) ( ) ( ) ( ) 90 ( )

2 2= + + = °+ m BIC m BAC m ABC m ACB m BAC . Analog se

determină şi măsurile celorlalte două unghiuri. 6) Fie ABC un triunghi de laturi a,b,c, I centrul cercului înscris în triunghi şi M un

punct din planul triunghiului. Atunci: ( )aMA bMB cMC a b c MI+ + = + +uuur uuur uuuur uuur

.

Demonstraţie. Din teorema bisectoarei rezultă '

'

BA c

A C b= , de unde '

bMB cMCMA

b c

+=

+

uuur uuuuruuuur

.

Teorema lui Menelaus aplicată triunghiului 'AA C şi transversalei 'BIB dă:

' '1

' '

AI A B CB

IA BC B A⋅ ⋅ = , de unde rezultă că

'

AI b c

IA a

+= . Atunci,

1

b cMA MD

aMA bMB cMCaMIb c a b c

a

++ + +

= =+ + ++

uuur uuuuruuur uuur uuuur

uuur.

7) Coordonatele baricentrice ale centrului cercului circumscris triunghiului ABC sunt:

, ,2 2 2

a b c

p p p

.

Demonstraţia rezultă din proprietatea precedentă. 8) Fie , ,

A B Cz z z sunt afixele vârfurilor A,B,C

ale triunghiului ABC de laturi a,b,c. Afixul centrului cercului înscris este egal cu

A B C

I

az bz czz

a b c

+ +=

+ +.

Demonstraţie. Alegem un sistem cartezian cu originea în punctul O, centrul cercului circumscris triunghiului ABC (Fig. 9). Din

teorema bisectoarei avem: '

'

c BA

b A C= sau

'c BA

b c BC=

+, deci '

acBA

b c=

+, de unde

A

B C A'

B' I

Fig. 9

Page 11: Barbu_TEOREME_FUNDAMENTALE(2008).pdf

18

rezultă că '

1

B C

A

cz z

bzc

b

+=

+. Teorema bisectoarei aplicată în triunghiul 'ABA pentru

bisectoarea BI ne dă: ' '

AB IA

BA IA= sau

'

IA b c

IA a

+= , deci

'

1

A AA B C

I

b cz z

az bz czazb c a b c

a

++ ⋅ + +

= =+ + ++

.

9) Dacă

a b cC C C este triunghiul de contact al triunghiului ABC atunci

, ,= = − = = − = = −b c a c a b

AC AC p a BC BC p b CC CC p c , unde a,b,c sunt lungimile

laturilor , ,BC AC BA , iar 2

+ +=a b c

p .

Demonstraţie. Evident , ,= = = = = =b c a c a b

AC AC x BC BC y CC CC z , de unde rezultă că

2( ) 2+ + = + + =x y z a b c p , deci = + +p x y z . Cum ,+ = + =y z a z x b rezultă

, ,= − − − = −x p a y p b z p c .

10) Dacă r este raza cercului înscris în triunghiul ABC, atunci:

2 2 2

− − −= = =p a p b p c

rA B C

ctg ctg ctg

.

Demonstraţie. Din triunghiul dreptunghic b

AIC rezultă 2

−=

A p actg

r. Analog se obţin şi

celelalte două egalităţi.

11) Consecinţă: .2 2 2

= ⋅ ⋅ ⋅A B C

p r ctg ctg ctg

Demonstraţie. Avem:

2 2 2 2 2 2 − + − + − = + + = ⋅ ⋅ ⋅

A B C A B Cp a p b p c r ctg ctg ctg r ctg ctg ctg .

12) Dacă O este centrul cercului circumscris triunghiului ABC , R raza cercului circumscris triunghiului ABC şi r raza cercului înscris în acest triunghi, atunci

2 2 2= −IO R Rr .

Demonstraţie. Fie "A al doilea punct de intersecţie dintre dreapta AI şi cercul circumscris triunghiului ABC (Fig.8). Utilizând puterea punctului I faţă de cercul circumscris

triunghiului ABC , avem: 2 2" ( )( )⋅ = + − = −AI IA R IO R IO R IO , adică 2 2 "= − ⋅IO R AI IA

(1).Din triunghiul c

AIC rezultă sin

2

=r

AIA

(2). Avem : ( ") ( ) ( )m BIA m IAB m IBA= + =

[ ]1( ) ( )

2m A m B+ şi

1 1( ") ( ) ( ") ( ) ( " )

2 2m IBA m B m CBA m B m A AC= + = + =

Page 12: Barbu_TEOREME_FUNDAMENTALE(2008).pdf

19

1 1( ) ( ).

2 2m B m A+ Din teorema sinusurilor în triunghiul "ABA rezultă " 2 sin

2=

ABA R ,

adică " 2 sin2

=A

IA R (3). Din relaţiile (1), (2) şi (3) rezultă 2 2 2= −IO R Rr .

Observaţii :

1) 2 2 2= −OI R Rr se numeşte relaţia lui Euler. 2) Deoarece 2 0≥OI rezultă 2 2≥R Rr , adică 2≥R r ( inegalitatea lui Euler ). 3) Egalitatea 2=R r are loc numai pentru triunghiul echilateral.

4) Relaţia lui Euler poate fi scrisă şi astfel: 1+ =− +r r

R OI R OI.

13) Fie I centrul cercului înscris şi G centrul de greutate al triunghiului ABC . Atunci:

9

516 222 rRrp

IG+−

= (unde p este semiperimetrul triunghiului ABC ; R şi r sunt

razele cercului circumscris, respectiv a cercului înscris în triunghiul ABC).

Demonstraţie. Din relaţia cba

abccMCbMBaMAMI

++

−++=

2222 (vezi „Relaţia lui Van-

Aubel”) pentru GM ≡ rezultă: cba

cGCbGBaGAGI

++

++=

2222 . Dar

2 2 2 22 2 2( )

3 9

+ − = =

a

b c aGA m şi analoagele dau:

9

516 222 rRrp

GI+−

= , unde am

utilizat şi relaţiile 2 2 2 2 22( 4 )+ + = − −a b c p Rr r , 22 4 rRrpacbcab ++=++ şi

Rrpabc 4= .

14) Fie I centrul cercului înscris şi H ortocentrul triunghiului ABC . Atunci,

2 22 2= −h

IH r r R (unde hr este raza cercului înscris în triunghiul ortic al triunghiului

ABC). Demonstraţie. Fie

a b cC C C şi

a b cH H H triunghiurile de contact, respectiv ortic

corespunzătoare triunghiului ABC (Fig. 10). Deoarece AI şi BI sunt bisectoarele unghiurilor BAC, respectiv ABC rezultă

(1)sin / 2

=r

AIA

şi (2)2

=a

BBC rctg . Avem:

cos , cos .2

= = − = −a a a a a

BBH c B C H BH BC c B rctg

Din triunghiul a a

IC H rezultă 2

2 2 cos (3)2

= + −

a

BIH r c B rctg . Dar 2 cosAH R A=

(4) şi 2 cos cosa

HH R B C= (5) Relaţia lui

Stewart aplicată în triunghiul a

AIH ne dă:

A

B C

Fig. 10

aH Ca

I H

Page 13: Barbu_TEOREME_FUNDAMENTALE(2008).pdf

20

2 2 2 (6).a a a a a

AI HH IH AH IH AH HH AH AH⋅ + ⋅ − ⋅ = ⋅ ⋅ Din relaţiile (1) - (6) rezultă 2 22 2 .= −

hIH r r R , unde 2 cos cos cos=

hr R A B C (vezi „Triunghiul ortic”).

15) Într-un triunghi ABC, centrul cercului înscris (I), centrul de greutate (G) şi punctul

lui Nagel ( N ) sunt coliniare şi 2

GNIG = .

Demonstraţie. Vezi „Punctul lui Nagel”. 16) Într-un triunghi ABC, fie I centrul cercului înscris triunghiului, O centrul cercului circumscris, H ortocentrul şi N punctul lui Nagel al triunghiului ABC. Atunci,

2

HNIO = şi .HN OI

Demonstraţie. Vezi „Punctul lui Nagel”. 17) Dreptele IH şi

pS O sunt paralele şi 2

pIH S O= (unde

pS este punctul lui Spieker

al triunghiului ABC). Demonstraţie. Vezi „ Punctul lui Spieker”.

18) Centrul cercului înscris în triunghiul median al triunghiului ABC, este mijlocul segmentului IN ( unde N este punctul lui Nagel al triunghiului ABC) . Demonstraţie. Vezi „Punctul lui Nagel”. 19) Punctele , , ,

pI G S N sunt coliniare şi 12 6 4 3 2 .= = = =

p pGS GI IS NG NI

Demonstraţie. Vezi „Punctul lui Nagel”. 20) Centrul cercului înscris într-un triunghi aparţine dreptei lui Nagel corespunzătoare triunghiului. Demonstraţie. Vezi „Punctul lui Nagel”. 21) Centrul cercului înscris în triunghiul ABC este punctul lui Nagel al triunghiului median Demonstraţie. Vezi „Punctul lui Nagel”. 22) În triunghiul ABC fie ( )∈M AB şi ( ).∈N AC Dreapta MN trece prin centrul

cercului înscris în triunghiul ABC dacă şi numai dacă aNA

NCc

MA

MBb =⋅+⋅ .

Demonstraţie. Vezi „Relaţia lui Van-Aubel”. 23) Fie I centrul cercului înscris în triunghiul ABC şi ',',' CBA punctele de intersecţie dintre bisectoarele AI , BI respectiv CI cu cercul circumscris triunghiului ABC .

Ortocentrul triunghiului ''' CBA este punctul I .

Demonstraţie. Fie ' ' '= ID AA B C . Avem ( ' ') ( ' ')+ =m AA C m DC A

( ') [ ( ' ) ( ')]+ + =m ACC m B BC m CAA 1[ ( ) ( ) ( )] 90

2+ + = °m C m B m A , de unde rezultă că

( ' ') 90= °m A DC , deci ''' CAAA ⊥ deci I este ortocentrul triunghiului ''' CBA .

Page 14: Barbu_TEOREME_FUNDAMENTALE(2008).pdf

21

24) Fie I centrul cercului înscris în triunghiul ABC şi ',',' CBA punctele de intersecţie dintre bisectoarele AI , BI respectiv CI cu mediatoarele corespunzătoare. Atunci,

' 'A B este mediatoarea segmentului IC, ' 'B C este mediatoarea segmentului IA şi ' 'C A este mediatoarea segmentului IB. Demonstraţie: Punctele ',',' CBA sunt punctele de intersecţie dintre bisectoarele AI , BI respectiv CI cu cercul circumscris triunghiului ABC . Deoarece 'A este centrul cercului circumscris triunghiului BIC (vezi „Cercuri exînscrise”) rezultă concluzia. 25) Măsura unghiului determinat de bisectoarea interioară unghiului A a triunghiului

ABC şi înălţimea din A este egală cu 1

( ) ( )2

− m B m C .

Demonstraţie. Fie a

H piciorul înălţimii din A şi 'A piciorul

bisectoarei din A (Fig. 11). Considerăm cazul în care ' ( )∈

aA H C , cazul în care ' ( )∈

aA H B tratându-se analog.

Din 1 1( ') ( ) ( ) ( ) [90 ( )]

2 2m H AA m A m H AB m A m Ba a= − = − − =

1 1( ) ( ( ) ( ) ( )) ( )

2 2m A m A m B m C m B

− + + − rezultă

1( ') ( ) ( ) .

2 = − am H AA m B m C

26) Proiecţiile vârfului A al triunghiului ABC pe cele patru bisectoare ale unghiurilor B şi C sunt coliniare. Demonstraţie. Fie P, Q şi R, S proiecţiile vârfului A pe bisectoarele exterioare, respectiv interioare ale vârfurilor B şi C. Patrulaterele PBRA şi CQAS sunt dreptunghiuri, deci PR trece prin M, mijlocul lui AB şi SQ trece prin N, mijlocul laturii AC (Fig. 12). Deoarece

≡ ≡ MBR MRB RBC rezultă că MR BC , deci R aparţine dreptei MN. Analog se arată că ∈S MN , deci punctele P,Q,R,S sunt coliniare.

C

A

B A' Ha

Fig. 11

P

Fig. 12

A

B C

Q S R

M N

Page 15: Barbu_TEOREME_FUNDAMENTALE(2008).pdf

22

I.4. Ortocentrul unui triunghi

„Matematica nu exclude poezia şi reciproc; matematica reunită cu poezia poate oferi un orizont mult mai vast pe care ochiul şi sufletul omenesc să-l cuprindă

şi să-l apropie. Înţelegând şi savurând poezia am pătruns în templul armoniei, precum atunci când am înţeles matematica am pătruns în templul certitudinii.”

Ion Barbu4 Punctul de intersecţie al înălţimilor unui triunghi se numeşte ortocentrul triunghiului (H). Dacă triunghiul ABC este ascuţitunghic , ortocentrul se află în interiorul triunghiului (Fig.

13). Dacă triunghiul ABC este dreptunghic ( ) 90m BAC = ° , ortocentrul triunghiului este punctul A (Fig. 14). Dacă triunghiul ABC este obtuzunghic , ortocentrul se afla în exteriorul triunghiului ABC (Fig. 15).

1) Fie H ortocentrul unui triunghi nedreptunghic ABC şi a b c

H H H triunghiul său ortic.

Sunt adevărate egalităţile: , , .a b c

a b c

BH CH AHtgC tgA tgB

H C tgB H A tgC H B tgA= = =

Demonstraţie. Din triunghiurile dreptunghice a

BH A şi a

CH A rezultă a

a

AHBH

tgB= şi

a

a

AHCH

tgC= de unde a

a

BH tgC

H C tgB= . Analog se arată şi celelalte egalităţi.

2) Fie H ortocentrul unui triunghi nedreptunghic ABC şi

a b cH H H triunghiul său ortic.

Sunt adevărate egalităţile: cos cos cos

, ,cos cos cos cos cos cos

a b c

AH A BH B CH C

HH B C HH C A HH A B= = =

⋅ ⋅ ⋅.

Demonstraţie. Din teorema lui Van-Aubel rezultă cos

cos cosb c

a b c

AH AHAH tgC tgB A

HH H C H B tgA tgA B C= + = + =

⋅.

4 Ion Barbu (1895-1961) – matematician român, profesor la Universitatea din Bucureşti, contribuţii în algebră şi geometrie

C A H≡ B

C

Ha

H

A

B C

Hb

Hc

Ha

H

A

B

Hb

Ha

Hc

Fig. 13 Fig. 14 Fig. 15

Page 16: Barbu_TEOREME_FUNDAMENTALE(2008).pdf

23

3) Pentru orice punct M din planul unui triunghi nedreptunghic ABC este adevărată

egalitatea: .tgA tgB tgC

MH MA MB MCtgA tgB tgC tgA tgB tgC tgA tgB tgC

= + ++ + + + + +

uuuur uuur uuur uuuur

Demonstraţie. Din a

AH tgC tgB

HH tgA

+= şi a

a

BH tgC

H C tgB= avem:

1

a

tgC tgBMA MH

tgAMH

tgC tgB

tgA

++

= =+

+

uuur uuuuur

uuuur ( ) atgA MA tgC tgB MH

tgA tgB tgC

⋅ + + ⋅

+ +

uuur uuuuur

şi 1

a

tgCMB MC

tgBMH

tgC

tgB

+= =

+

uuur uuuur

uuuuur

tgB MB tgC MC

tgB tgC

⋅ + ⋅+

uuur uuuur

, de unde rezultă concluzia.

4) Coordonatele baricentrice absolute ale ortocentrului H al unui triunghi ascuţitunghic

ABC sunt , , .tgA tgB tgC

HtgA tgB tgC tgA tgB tgC tgA tgB tgC

+ + + + + +

Demonstraţia rezultă din proprietatea precedentă. Observaţie: Deoarece tgA tgB tgC tgA tgB tgC+ + = ⋅ ⋅ rezultă

( , , ).H ctgBctgC ctgCctgA ctgActgB

5) Fie , ,

A B Cz z z afixele vârfurilor triunghiului ABC. Afixul ortocentrului H al

triunghiului ABC este egal cu

.H A B C

tgA tgB tgCz z z z

tgA tgB tgC tgA tgB tgC tgA tgB tgC= + +

+ + + + + +

Demonstraţia rezultă din proprietatea (3). 6) Coordonatele unghiulare ale ortocentrului unui triunghi ascuţitunghic ABC sunt

egale cu: ( ) 180 ( )m BHC m A= °− , ( ) 180 ( )m CHA m B= ° − , ( ) 180 ( ).m AHB m C= ° −

Demonstraţie. Avem: ( ) ( ) 180 ( )b c

m BHC m H HH m A= = ° − (deoarece patrulaterul

c bAH HH este inscriptibil). Analog, ( ) 180 ( )m CHA m B= ° − şi ( ) 180 ( ).m AHB m C= ° −

7) Distanţele de la ortocentrul unui triunghi ABC la vârfurile acestuia sunt egale cu 2 cos ,2 cos ,2 cos .R A R B R C

Demonstraţie. Deoarece patrulaterul a c

BH HH este inscriptibil rezultă ( ) ( )=c

m H HA m B ,

atunci sin sin= = c

c

H AH HA B

AH, de unde

cos2 cos

sin sin= = =cAH b A

AH R AB B

. Analog se

arată că 2 cos=BH R B şi 2 cos .=CH R C

8) Consecinţă: 2( ).AH BH CH R r+ + = +

Page 17: Barbu_TEOREME_FUNDAMENTALE(2008).pdf

24

Demonstraţie. Avem: 2 (cos cos cos ) 2 1 4sin sin sin2 2 2

A B CAH BH CH R A B C R

+ + = + + = +

;

dar 4sin sin sin2 2 2

r A B C

R= , deci 2( ).AH BH CH R r+ + = +

9) Distanţele de la ortocentrul unui triunghi ABC la laturile acestuia sunt egale cu 2 cos cos ,2 cos cos ,2 cos cos .R B C R C A R A B Demonstraţie. Din triunghiul dreptunghic

aBHH rezultă

cos 2 cos cos= =a

HH BH C R B C . Analog, 2 cos cos=b

HH R C A şi

2 cos cos .=c

HH R A B

10) Ortocentrul H al triunghiului ABC aparţine dreptei lui Euler a triunghiului ABC . Demonstraţie. Vezi „Dreapta lui Euler “ . 11) Dacă H este ortocentrul triunghiului ABC, O centrul cercului circumscris triunghiului şi G centrul de greutate al triunghiului ABC, atunci 2HG GO= şi

3 .HO GO= Demonstraţie. Vezi „Dreapta lui Euler “ . 12) Dacă H este ortocentrul triunghiului ABC şi O centrul cercului circumscris acestui triunghi, atunci 2 2 (1 8cos cos cos ).HO R A B C= −

Demonstraţie. Vezi „Centrul cercului circumscris unui triunghi”. 13) Ortocentrul H, centrul cercului circumscris O şi punctul lui Longchamps L sunt coliniare şi HO OL≡ şi 2 .LH OH= Demonstraţie. Vezi „Punctului lui Longchamps”.

14) Consecinţă: Dacă G este centrul de greutate al unui triunghi ABC, atunci 3

.4

HO LG= .

15) Într-un triunghi ABC fie H ortocentrul său, O centrul cercului circumscris, I centrul cercului înscris triunghiului, N punctul lui Nagel al triunghiului ABC. Atunci,

2=HN OI şi .HN OI

Demonstraţie. Vezi „Punctul lui Nagel”. 16) Consecinţă: Segmentele HI şi ON sunt congruente. Demonstraţie. Vezi „Punctul lui Nagel”. 17) Într-un triunghi ABC fie H ortocentrul său, O centrul cercului circumscris, I centrul cercului înscris triunghiului,

pS punctul lui Spieker al triunghiului ABC.

Dreptele IH şi pS O sunt paralele şi 2 pHI S O= .

Demonstraţie. Vezi „Punctul lui Nagel”. 18) Ortocentrul H al unui triunghi ABC şi centrul cercului circumscris O al triunghiului ABC sunt puncte izogonal conjugate . Demonstraţie. Vezi „Puncte izogonale “.

Page 18: Barbu_TEOREME_FUNDAMENTALE(2008).pdf

25

19) Simetricul ortocentrului H al triunghiului ABC faţă de mijlocul unei laturi se află pe cercul circumscris triunghiului. Demonstraţie. Fie

aM mijlocul laturii BC şi 'A

punctul diametral opus lui A (Fig. 16). Deoarece BH ⊥ AC şi 'A C AC⊥ rezultă 'BH CA . Analog, rezultă 'BH CA , deci patrulaterul

'BHCA este paralelogram, deci simetricul lui H faţă de

aM este situat pe cercul circumscris

triunghiului ABC . 20) Simetricul ortocentrului H al triunghiului ABC faţă de una din laturile triunghiului se află pe cercul circumscris triunghiului . Demonstraţie: Fie 1A punctul de intersecţie dintre

înălţimea a

AH şi cercul circumscris triunghiului

ABC. Deoarece ( ) 90 ( )a

m HBH m BCA= ° − =

190 ( )m BA A° − =

1( )a

m A BH rezultă că înălţimea a

BH este şi bisectoarea unghiului 1HBA ,

adică triunghiul 1HBA este isoscel , deci 1a aHH H A= .

Observaţie: Fie 1B şi 1C simetricele ortocentrului H faţă de laturile AC , respectiv AB.

Triunghiul 1 1 1A B C se numeşte triunghiul circumpedal al ortocentrului triunghiului ABC. 21) Fie H ortocentrul unui triunghi dat ABC şi C cercul circumscris triunghiului

ABC . Să se arate că în cercul C se pot înscrie o infinitate de triunghiuri care să-l aibă

pe H ortocentru. Demonstraţie: Fie ortocentrul H este situat în interiorul cercului C (Fig. 17). Prin H ducem o coardă oarecare MN şi fie 'N simetricul segmentului HN . Prin 'N ducem coarda ''BA perpendiculară pe MN . Deoarece

''' BAMN ⊥ şi 'N este simetricul lui H faţă de ''BA rezultă că H este ortocentrul triunghiului ''BMA . Cum coarda MN a fost considerată arbitrară, rezultă că sunt o infinitate de triunghiuri care au punctul H drept ortocentru. Dacă H este situat în extreriorul cercului, atunci una din perpendicularele duse prin mijlocul sementelor HM sau HN intersectează cercul C după coarda ''BA .

22) Fie DCBA ,,, patru puncte conciclice . Ortocentrele triunghiurilor DABCDABCDABC ,,, sunt vârfurile unui patrulater congruent şi invers omotetic cu ABCD .

Demonstraţie. Fie 4321 ,,, HHHH ortocentrele

CDABCDABC ,, respectiv DAB, 'A mijlocul laturii BC şi O centrul cercului circumscris triunghiului ABC (Fig. 18). Avem: '221 OADHAH == şi 21 || DHAH

A

B C H

M N

A'

B'

N'

Fig. 17

A

B C 1H

4H 3H D

A'

2H

Fig. 18

O

Ha

Hb

A

Fig. 16

B

Hc

C

H

A' 1A

aM

1B

1C

Page 19: Barbu_TEOREME_FUNDAMENTALE(2008).pdf

26

(deoarece BCAH ⊥1 şi BCDH ⊥2 ) deci patrulaterul DHAH 21 este paralelogram, de

unde rezultă 21HHAD ≡ si 21|| HHAD . Analog se arată că 32HHAB ≡ , 43HHBC ≡ ,

14HHCD ≡ şi 32|| HHAB , 43|| HHBC , 14|| HHCD . Deoarece patrulaterele ABCD şi

4321 HHHH cu laturile egale şi paralele două câte două rezultă că ele sunt congruente şi

omotetice, omotetia fiind invers deoarece centrul de susţinere se află între vârfurile omoloage.

23) Laturile unui triunghi determină de două transversale ortogonale, care trec prin ortocentru, segmente propoţionale. Demonstraţie. Fie ''' CBA −− şi """ CBA −− cele două transversale ortogonale şi H ortocentrul triunghiului ABC (Fig. 19). Triunghiurile HBA' şi "HAB , respectiv "BHA şi HAB ' sunt asemenea, având laturile perpendiculare două câte două, de unde:

AH

BA

AB

BH

HB

HA '

""

'== şi

HB

HA

AH

BA

AB

BH

'

""

'== , iar de aici

rezultă "

'

"

'

AB

AB

BA

BA= (1). Din teorema lui Menelaus

aplicată în triunghiurile ''BCA şi "" ACB tăiate de

transversala AB rezultă: 1'''

'''=⋅⋅

AB

AC

AC

BC

BC

BA şi 1

"""

"""=⋅⋅

AB

AC

AC

BC

BC

BA (2). Împărţind relaţia

(2) rezultă: 1'

"

""

""

''

''

"

'=⋅⋅⋅

AB

AB

BC

AC

AC

BC

BA

BA (2). Din relaţiile (1) şi (2) rezultă

""

''

""

''

CB

CB

CA

CA= .

Analog se arată că ""

''

""

''

AB

AB

CA

CA= , de unde

""

''

""

''

""

''

AB

AB

CB

CB

CA

CA== .

24) Fie H ortocentrul unui triunghi ABC. Cercurile circumscrise triunghiurilor BCH, ACH şi ABH sunt congruente cu cercul circumscris triunghiului ABC. Demonstraţie. Fie 'A al patrulea vârf al paralelogramului

'ABA C (Fig. 20). Evident triunghiurile ABC şi 'A CB sunt congruente, deci cercurile circumscrise acestor două triunghiuri

sunt congruente. Deoarece ( ) ( )b c

m BHC m H HH= =

180 ( ')m A° − rezultă că patrulaterul 'BHCA este inscriptibil,

deci cercul circumscris triunghiului BCH este tocmai cercul circumscris triunghiului 'A CB . Analog se arată pentru triunghiurile ACH şi ABH.

25) Dacă L este proiecţia ortocentrului triunghiului ABC pe mediana aAM şi 1L este

simetricul lui L faţă de aM , atunci 1L aparţine cercului circumscris triunghiului ABC.

Demonstraţie. Fie aH piciorul înălţimii duse din A pe BC . Avem 1a aLM M L= .

Deoarece patrulaterul a aHH M L este inscriptibil, din puterea punctului A faţă de cercul

circumscris acestui patrulater rezultă: ( )a a a aAM AM M L AH AH− = ⋅ . Dar

A

B C

H C'

A" A'

B'

B"

Fig. 19

C"

A

B C

H

A'

Fig. 20

bH

cH

Page 20: Barbu_TEOREME_FUNDAMENTALE(2008).pdf

27

2 cosAH R A= ⋅ ⋅ , 2 2 2

cos2

b c aA

bc

+ −= ,

2

2a

S b cAH

a R

⋅ ⋅= =

rezultă: 2 2 2

( )2a a a a

b c aAH AH AM AM M L

+ −⋅ = = − (1)

(Fig. 21). Fie 'L AL= I C (C fiind cercul

circumscris triunghiului ABC). Analog, 2

'4a a a a

aA M M L B M M C⋅ = ⋅ = . Dar

2 2 22 2( )

4a

b c aAM

+ −= , de unde rezultă că

2 2 2

( ')2a a a

b c aAM AM M L

+ −− = (2). Din relaţiile (1)

şi (2) rezultă 'a aM L M L≡ , deci 1 'a aM L M L= sau

1 'L L≡ , de unde rezultă concluzia.

26) Fie M un punct situat pe cercul circumscris al unui triunghi ABC . Ortocentrul H al triunghiului ABC aparţine dreptei lui Steiner corespunzătoare triunghiului. Demonstraţie. Vezi „Dreapta lui Steiner”. 27) Dreptele lui Steiner ale simetricelor ortocentrului H al triunghiului ABC faţă de laturile triunghiului sunt paralele cu laturile triunghiului ortic al triunghiului .ABC

Demonstraţie. Vezi „Dreapta lui Steiner”. 28) Triunghiul ce are vârfurile oricare trei puncte dintre centrele cercurilor tritangente, are drept ortocentru pe cel de-al patrulea punct din cele de mai sus. Demonstraţie. Vezi „Cercuri exînscrise”. 29) Fie o dreptă d ce conţine ortocentrul H al triunghiului ABC. Simetricele dreptei d faţă de laturile triunghiului ABC sunt concurente într-un punct de pe cercul circumscris triunghiului. Demonstraţie. Vezi „Punctul antisteiner”.

Ha

Hb

A

Fig. 21

B

Hc

C

H

1L

aM

L

Page 21: Barbu_TEOREME_FUNDAMENTALE(2008).pdf

28

I.5. Punctul lui Gergonne5 „Dacă natura n-ar fi atât de minunată nici n-ar merita să o cunoaştem, iar viaţa n-ar merita să fie trăită. Am în vedere nu frumuseţea care îţi sare în ochi, ci acea frumuseţe profundă care se dezvoltă în armonia componentelor sale şi este accesibilă numai raţiunii. Frumuseţea intelectuală oferă satisfacţie prin sine însăşi.” – Henri Poincaré6 1) Într-un triunghi ABC dreptele care unesc vârfurile triunghiului cu punctele de contact ale cercului înscris cu laturile opuse sunt concurente. Demonstraţie. Fie , ,

a b cC C C punctele de

tangenţă dintre cercul înscris în triunghiul ABC şi laturile BC, AC respectiv AB (Fig. 22). Cum ,

a cBC BC=

a bCC CC= şi

,b c

AC AC= avem: 1a b c

a b c

C B C C C A

C C C A C B⋅ ⋅ = , iar din

reciproca teoremei lui Ceva rezultă că dreptele ,

a bAC BC şi

cCC sunt concurente.

Punctul Γ de concurenţă al dreptelor ,

a bAC BC

şi c

CC se numeşte punctul lui Gergonne. 2) Dacă (Γ ) este punctul lui Gergonne al triunghiului ABC, iar

a b cC C C triunghiul său

de contact, atunci ( )

( )( )a

A a p a

C p b p c

Γ −=

Γ − −,

( )

( )( )b

B b p b

C p c p a

Γ −=

Γ − −,

( )

( )( )c

C c p c

C p a p b

Γ −=

Γ − −.

Demonstraţie. Din teorema lui Van-Aubel rezultă c b

c c b

AC ACA p a p a

C C B C C p b p c

Γ − −= + = + =

Γ − −

( )

( )( )

a p a

p b p c

−− −

. Analog se demonstrează şi celelalte două egalităţi.

3) Dacă (Γ ) este punctul lui Gergonne al triunghiului ABC, atunci pentru orice punct M din planul triunghiului ABC este adevărată egalitatea:

1 1 1 1M MA MB MC

s p a p b p c

Γ = + + − − −

uuuur uuur uuur uuuur, unde

1 1 1s

p a p b p c= + +

− − −.

Demonstraţie. Din ( )

( )( )a

A a p a

C p b p c

Γ −=

Γ − − rezultă

( )

( )( )( )

1( )( )

a

a p aMA MC

p b p cM

a p a

p b p c

−+

− −Γ =

−+

− −

uuur uuuuur

uuuur (1), dar

a

a

BC p b

C C p c

−=

− de unde

( ) ( )

1a

p bMB MC

p c MB p b MCp cMC

p b a

p c

−+

− + −−= =

−+

uuur uuuuruuur uuuur

uuuuur (2). Din relaţiile

(1) şi (2) rezultă concluzia.

5 Joseph Gegonne (1771-1859) – matematician francez, fondator al revistei Annales de Mathématiques în 1810 6 Henri Poincaré (1854 -1912) – matamatician şi fizician francez, contribuţii importante în toate ramurile matematicii

A

B C aC

bC

cC

Γ

Fig. 22

Page 22: Barbu_TEOREME_FUNDAMENTALE(2008).pdf

29

4) Coordonatele baricentrice relative ale punctului lui Gergonne sunt 1 1 1

, ,p a p b p c

Γ − − −

.

Demonstraţia rezultă din proprietatea precedentă. 5) Fie , ,

A B Cz z z sunt afixele vârfurilor A,B,C ale triunghiului ABC de laturi a,b,c. Afixul

punctului lui Gergonne corespunzător triunghiului ABC este egal cu 1 1 1

1 1 1

A B Cz z z

p a p b p cz

p a p b p c

Γ

+ +− − −

=+ +

− − −

.

Demonstraţia rezultă din proprietatea (2). 6) Punctul lui Gergonne (Γ ) al triunghiului ABC este punctul simedian al triunghiului de contact al triunghiului ABC .

Demonstraţie. Fie a b c

C C C triunghiul de contact al triunghiului ABC şi

a b c

AC BC CCΓ = ∩ ∩ . Deoarece simediana dintr-un vârf al unui triunghi conţine punctul

de intersecţie al tangentelor la cercul circumscris duse în celelalte două vârfuri ale triunghiului (vezi „Simediane”), rezultă că

aC A ,

bC B şi

cC C sunt simediane în triunghiul

a b cC C C , deci punctul lor de intersecţie Γ , este punctul lui Lemoine al triunghiului de

contact a b c

C C C .

7) Punctele lui Gergonne ( Γ ) şi Nagel ( )N ale triunghiului ABC sunt puncte izotomice. Demonstraţie. Fie

a b cC C C triunghiul de contact al triunghiului ABC şi

aD ,

bE ,

cF

punctele de tangenţă ale cercurilor exînscrise cu laturile BC , CA , respectiv AB . Deoarece

a aBD CC p c= = − , rezultă că punctele

aD şi

aC sunt simetrice faţă de mijlocul laturii

BC . Analog, punctele bE şi

bC , respectiv

cF şi

cC sunt simetrice faţă de mijloacele

laturilor AC , respectiv AB . Deci punctele de concurenţă ale dreptelor ( , , )a b c

AC BC CC şi

( , , )a b c

AD BE CF – adică punctul lui Gergonne, respectiv punctul lui Nagel – sunt

izotomice. 8) Fie ABC un triunghi neisoscel,

a b cC C C triunghiul

său de contact, 'b c

A C C BC= ∩ , 'a c

B C C AC= ∩ ,

'a b

C C C AB= ∩ . Punctele 'A , 'B , 'C sunt coliniare. Demonstraţie. Teorema lui Menelaus aplicată în triunghiul ABC (Fig. 23) pentru transversalele ( ', , )

c bA C C ,

( ', , )c a

B C C , respectiv ( ', , )a b

C C C dă:

'1

'c b

c b

C A C CA B

A C C B C A⋅ ⋅ = ,

'1

'a c

a c

C B C AB C

B A C C C B⋅ ⋅ = şi

'1

'a b

a b

C B C CC A

C B C C C A⋅ ⋅ = , de unde rezultă:

A

B

C

A'

B'

C'

aC

bC cC

Fig. 23

Page 23: Barbu_TEOREME_FUNDAMENTALE(2008).pdf

30

2' ' '

1' ' '

a b c

a b c

C C C A C BA B B C C A

A C B A C B C B C C C A

⋅ ⋅ = ⋅ ⋅ =

. Atunci, din reciproca teoremei lui Menelaus

rezultă că punctele 'A , 'B , 'C sunt coliniare. Observaţie: Dreapta ce conţine punctele 'A , 'B , 'C se numeşte dreapta lui Gergonne. 9) Dreapta lui Gergonne a triunghiului ABC este polara triliniară a punctului lui Gergonne. Demonstraţia rezultă din proprietatea precedentă, triunghiurile

a b cC C C şi ABC fiind

omologice, punctul lui Gergonne fiind polul triliniar. 10) Punctul lui Gergonne, punctul lui Nagel şi retrocentrul unui triunghi sunt coliniare. Demonstraţie. Vezi „Retrocentrul unui triunghi”. Teorema lui Poncelet

11) Dreptele care unesc mijloacele laturilor unui triunghi ABC cu respectiv mijloacele cevienelor corespunzătoare punctului lui Gergonne sunt concurente în centrul cercului înscris în triunghiul ABC .

Demonstraţie. Fie a

M mijlocul laturii BC , a

C şi a

D

proiecţiile centrului cercului înscris I, respectiv a punctului

aI centrului cercului A–exînscris pe BC , iar P punctul

diametral opus lui a

C în cercul înscris în triunghiul ABC

(Fig. 24). Prin omotetia de centru A şi raport aAI

AI ,

punctul P se transformă în punctul a

D , deci punctele A ,

P şi a

D sunt coliniare. Cum a

IM este linie mijlocie în

triunghiul a a

C PD rezultă a a

IM AD , deci a

IM trece şi

prin mijlocul segmentului a

AC . Analog se arată că şi

celelalte două segmente ce unesc mijloacele laturilor triunghiului cu mijloacele cevienelor punctului lui Gergonne trec prin I. 12) Punctul lui Gergonne Γ al triunghiului ABC este propriul său punct ciclocevian. Demonstraţie. vezi „Puncte cicloceviene”.

13) Punctul lui Gergonne al triunghiului ortic corespunzător unui triunghi ABC este punctul de întâlnire al dreptelor ce unesc picioarele înălţimilor triunghiului ABC cu proiecţiile ortocentrului pe laturile triunghiului ortic. Demonstraţie. Înălţimile

aAH ,

bBH ,

cCH

sunt bisectoarele unghiurilor triunghiului ortic

a b cH H H (vezi „Triunghiul ortic”), deci H

este centrul cercului înscris în triunghiul ortic,

A

Fig. 24

B

C aC aD

aI

P

Γ I

aM

A

B C

H

aH

bH

cH

Fig. 25

1H

2H 3H

Page 24: Barbu_TEOREME_FUNDAMENTALE(2008).pdf

31

iar 1H , 2H , 3H - proiecţiile lui H pe laturile triunghiului ortic – sunt punctele de contact

ale cercului înscris cu laturile triunghiului ortic, deci dreptele 1aH H , 2b

H H şi 3cH H sunt

concurente în punctul lui Gergonne al triunghiului ortic a b c

H H H (Fig. 25).

14) Dreptele care unesc vârfurile unui triunghi ABC cu punctele de contact dintre un cerc exînscris şi dreptele AB, BC, CA sunt concurente. Demonstraţie. Fie 1 1 1, ,A B C punctele de contact dintre cercul A -

exînscris şi dreptele BC, CA respectiv AB (Fig. 26). Cum

1 1 1 1,AB AC BA BC= = şi 1 1CA CB= rezultă: 1 1 1

1 1 1

1⋅ ⋅ =A B BC C A

AC B A C B şi

din reciproca teoremei lui Ceva rezultă că dreptele 1 1,AA BB şi

1CC sunt concurente într-un punct Γa. Analog, se obţin punctele

Γb şi .Γ

c Punctele , ,Γ Γ Γ

a b c se numesc adjunctele punctului

Gergonne. 15) Punctele adjuncte ale lui Gergonne

aΓ ,

bΓ ,

cΓ sunt pe

cevienele punctului lui Nagel. Demonstraţia rezultă din construcţie. 16) Cevienele punctelor adjuncte ale lui Gergonne sunt concurente în punctul lui Nagel. Demonstraţie. Evident, datorită construcţiei. 17) Triunghiul ABC şi triunghiul

a b cΓ Γ Γ sunt omologice, centrul de omologie fiind

punctul lui Nagel ( )N al triunghiului ABC .

Demonstraţia rezultă din proprietăţile precedente.

18) Triunghiul ABC şi triunghiul a b c

N N N , ale cărui vârfuri sunt adjunctele punctului

lui Nagel al triunghiului ABC sunt omologice, centrul de omologie fiind punctul lui Gergonne ( )Γ al triunghiului ABC .

Demonstraţia rezultă din cele de mai de sus. 19) Coordonatele baricentrice ale adjunctelor punctului Gergonne sunt:

1 1 1, , ,

Γ + + − − + − + − aa b c a b c a b c

1 1 1, , ,

Γ − − + + + − − b

a b c a b c a b c

1 1 1, , .

Γ − − − + − + + ca b c a b c a b c

Demonstraţia este analoagă cu cea din teorema (3). 20) Triunghiul a b cN N N - având vârfurile în punctele adjuncte ale lui Nagel - şi

triunghiul a b cΓ Γ Γ - având vârfurile în punctele adjuncte ale lui Gergonne - sunt

omologice, centrul de omologie aparţinând dreptei NΓ (unde N este punctul lui Nagel şi Γ este punctul lui Gergonne al triunghiului ABC ). Demonstraţie. Triunghiul ABC este omologic cu triunghiul a b cN N N , iar

a c bBN CNΓ = ∩ , b c aAN CNΓ = ∩ , c b aAN BNΓ = ∩ . Conform teoremei lui

Voronèse (vezi „Triunghiuri omologice”), triunghiurile a b cN N N şi a b cΓ Γ Γ sunt

1C 1B

1A

Γa

aI

A

B C

Fig. 26

Page 25: Barbu_TEOREME_FUNDAMENTALE(2008).pdf

32

omologice, centrul de omologie aparţinând dreptei ce uneşte centrele de omologie ale triunghiului ABC cu triunghiurile a b cN N N , respectiv a b cΓ Γ Γ - adică dreptei NΓ .

21) Fie T punctul de contact dintre tangenta dusă din punctul aM – mijlocul laturii BC

a triunghiului ABC - (diferită de BC ) la cercul A –exînscris. Punctul T aparţine cevienei AΓ (Γ fiind punctul lui Gergonne al triunghiului ABC ). Demonstraţie. Fie aD punctul de contact al cercului A –

exînscris cu latura BC , aC punctul de contact al cercului

înscris cu latura BC şi Q punctul diametral opus lui aD în

cercul A –exînscris (Fig. 27). Punctele A , aC şi Q sunt

coliniare (vezi „Punctul lui Nagel”) . Fie 1T primul punct de

intersecţie dintre ceviana AΓ cu cercul A –exînscris.

Deoarece 1

1( ) ( ) 90

2= = °a am D TQ m D Q , rezultă

1( ) 90= °a am C T D , iar cum punctele aC şi aD sunt

izotomice, rezultă că aTM este mediană în triunghiul

dreptunghic 1a aC T D , deci 1 1 1a a a a aM T D M D T TQD≡ ≡ , de

unde: ( ) 90= °a am M TI , adică 1T T≡ .

22) Într-un triunghi, paralelele duse prin mijloacele laturilor la cevienele punctului lui Gergonne ale vârfurilor opuse trec prin centrele cercurilor exînscrise respective. Demonstraţie. Fie aM mijlocul laturii BC a triunghiului ABC şi Q punctul diametral opus

lui aD în cercul A – exînscris, iar aC punctul de contact al cercului înscris cu latura BC.

Punctele ,A aC şi Q sunt coliniare (vezi „Punctul lui Nagel”). Atunci a aM I este linie

mijlocie în triunghiul ,a aD C Q adică ,a a aM I C Q deci .a aA M IΓ

23) Fie 1 1 1, ,A B C proiecţiile centrului cercului A – exînscris ( aI ) al triunghiului ABC pe

mediatoarele corespunzătoare laturilor triunghiului ABC. Triunghiurile 1 1 1A B C şi ABC

sunt omologice, centrul de omologie fiind punctul lui Gergonne ( Γ ) al triunghiului ABC.

Demonstraţie. În notaţiile proprietăţii precedente, fie 1A punctul de intersecţie dintre

mediatoarea segmentului BC cu dreapta AQ. Deoarece aM este şi mijlocul segmentului

a aC D , iar aQD BC⊥ rezultă că 1A este mijlocul ipotenuzei aC Q a triunghiului ,a aC D Q

deci patrulaterul 1a a aM D I A este dreptunghi. Am arătat că proiecţia lui aI pe mediatoarea

laturii BC aparţine cevienei din A a punctului lui Gergonne. Analog, se arată că 1B şi 1C

aparţin cevienelor din B respectiv C ale punctului lui Gergonne, adică triunghiurile 1 1 1A B C

şi ABC sunt omologice, centrul de omologie fiind punctul lui Gergonne al triunghiului ABC.

A

Fig. 27

B

C aC

aD

aI

Q

Γ aM

1A T

Page 26: Barbu_TEOREME_FUNDAMENTALE(2008).pdf

33

24) În triunghiul ABC, fie ( )∈U AB şi ( ).∈V AC Punctul lui Gergonne (Γ ) al triunghiului ABC aparţine dreptei MN dacă şi numai dacă:

1 1 1,⋅ + ⋅ =

− − −UB VC

UA p b VA p c p aunde a, b, c sunt lungimile laturilor BC, CA respectiv AB,

iar p este semiperimetrul triunghiului ABC. Demonstraţie. Fie a b cC C C triunghiul de contact al

triunghiului ABC (Fig.28). Deoarece dreapta UV trece prin punctul lui Gergonne atunci

(1)Γ

⋅ + ⋅ =a a a

a

C C BC CUB VC

UA BC VA BC AC (vezi „Relaţia lui

Van-Aubel”). Dar, ( )

( )( )a

A a p a

C p b p c

Γ −=

Γ − − iar

, ,a aC C p c BC p b= − = − relaţia (1) devenind:

( ) ( )( ) ( )

− ⋅ −⋅ − + ⋅ − =

−UB VC p c p b

p c p bUA VA p a

, de unde

rezultă 1 1 1

.⋅ + ⋅ =− − −

UB VC

UA p b VA p c p a

25) În triunghiul ABC, fie ( )∈U AB şi ( ).∈V AC Punctul lui Gergonne (Γ ) al triunghiului ABC aparţine dreptei MN dacă şi numai dacă :

.2 2 2

⋅ + ⋅ =UB B VC C A

tg tg tgUA VA

Demonstraţie. Din proprietatea (24) avem: 1 ( )( ) 1 ( )( )UB p a p c VC p a p b

UA b ab VA c ab

− − − −⋅ + ⋅ =

1 ( )( )p b p c

a bc

− −. Ţinând cont de 2 ( )( )

sin2

− −=

A p b p c

bc şi de relaţiile analoage precum şi

de teorema sinusurilor obţinem:

2 2 2sin sin sin2 2 2

sin sin sin⋅ + ⋅ =

B C A

UB VC

UA B VA C A, deci

.2 2 2

⋅ + ⋅ =UB B VC C A

tg tg tgUA VA

26) Într-un triunghi ABC, punctul lui Gergonne (Γ ), punctul lui Nagel (N) şi centrul antibisector (Z) sunt coliniare. Demonstraţie. Fie ( )∈U AB şi ( )∈V AC astfel încât Γ şi N aparţin dreptei UV. Deoarece

UVΓ∈ , atunci (1)2 2 2

UB B VC C Atg tg tg

UA VA⋅ + ⋅ = şi din N UV∈ rezultă:

(2)2 2 2

⋅ + ⋅ =UB B VC C A

ctg ctg ctgUA VA

(vezi „Punctul lui Nagel”). Din relaţiile (1) şi (2)

rezultă:

2 2 2 2 2 2sin cos sin cos sin cos2 2 2 2 2 2

sin cos sin cos sin cos2 2 2 2 2 2

+ + +⋅ + ⋅ =

B B C C A A

UB VC

B B C C A AUA VA

A

B C aC

bC

cC

Γ

Fig. 28

U

V

Page 27: Barbu_TEOREME_FUNDAMENTALE(2008).pdf

34

adică 1 1 1

sin sin sin⋅ + ⋅ =

UB VC

UA B VA C A şi de aici

1 1 1⋅ + ⋅ =

UB VC

UA b VA c a (vezi „Centrul

antibisector”) relaţie care arată că punctul .Z UV∈ 27) Fie a, b, c lungimile laturilor BC, CA, respectiv AB ale triunghiului ABC. Dacă dreapta ce uneşte punctul lui Gergonne ( )Γ cu punctul lui Nagel (N) al triunghiului

ABC este paralelă cu latura AB, atunci 2 2+

=+

a bc

a b.

Demonstraţie. Fie , ,Γ Γ Γa b c şi , ,a b cN N N picioarele

cevienelor determinate de punctul lui Gergonne, respectiv punctul lui Nagel cu laturile triunghiului ABC (Fig. 29). Din teorema lui Van – Aubel rezultă:

ap

cp

bp

cp

A

C

B

CC

b

b

a

a

c −

−+

−=

Γ

Γ+

Γ

Γ=

ΓΓΓ

(1) şi

cp

ap

cp

bp

AN

CN

BN

CN

NN

CN

b

b

a

a

c −

−+

−=+= (2). Deoarece

ABN ||Γ rezultă Γ=

ΓΓc c

C CN

NN (3). Din relaţiile (1), (2) şi

(3) rezultă 2 2+

=+

a bc

a b.

28) Dacă [ ] [ ] [ ] [ ], , ,

ABC BC AC ABA A A AΓ Γ Γ sunt ariile triunghiului ABC, ,BC ACΓ Γ , respectiv

,ABΓ unde Γ este punctul lui Gergonne al triunghiului ABC, atunci:

[ ]

[ ]

[ ]

[ ]

[ ]

[ ]

.ABC ABC ABC a b c

BC AC AB

A A A r r r

A A A rΓ Γ Γ

+ ++ + =

Demonstraţie. Fie a b c

C C C triunghiul de contact al

triunghiului ABC (Fig. 30). Dacă ,2

a b cp

+ += atunci

, ,b c c a

AC AC p a BC BC p b= = − = = − .a b

CC CC p c= = − Dacă

"A şi 'Γ sunt proiecţiile punctelor A şi Γ pe latura BC

obţinem: [ ]

[ ]

" ".

' '

ABC a

aBC

A ACAA BC AA

A CΓ

⋅= = =

ΓΓ ΓΓ Γ Din relaţia lui

Van Aubel avem: b c

a a a

r rA p a p a

C p b p c r r

Γ − −= + = +

Γ − − de unde

[ ]

[ ]

1 1ABC b c a b c

a a aBC

A r r r r rA

A C r rΓ

+ + +Γ= + = + =

Γ şi analog

[ ]

[ ]

,ABC a b c

bBC

A r r r

A rΓ

+ +=

[ ]

[ ]

ABC a b c

cBC

A r r r

A rΓ

+ += prin sumarea relaţiilor precedente şi ţinând seama că

1 1 1 1

a b cr r r r= + +

rezultă concluzia.

A

B C

bN

aΓ aN

N

Fig. 29

Γ cΓ

A

B C aC

bC cC

Γ

Fig. 30

A" 'Γ

Page 28: Barbu_TEOREME_FUNDAMENTALE(2008).pdf

35

29) Dacă două ceviene Gergonne ale unui triunghi ABC au lungimile egale, atunci triunghiul este isoscel. Demonstraţie. Soluţia 1. Fie

bBC şi

cCC cevienele Gergonne congruente (Fig. 31).

Aplicând teorema cosinusului în triunghiurile b

ABC , respectiv c

ACC rezultă:

2 2 2( ) 2 ( ) cosb

BC c p a c p a A= + − − − , 2 2 2( ) 2 ( ) cosc

CC b p a b p a A= + − − − , unde

2

+ +=a b c

p şi de aici rezultă prin egalare că: 2 22( )( )cos ( ) 0− − − − =b c p a A b c egalitate

echivalentă cu 2 2 2( )( )

( ) ( ) 02

− + + + −− − + =

a b c b c ab c b c

bc. Egalitatea

2 2 2( )( )( ) 0

2

− + + + −− + =

a b c b c ab c

bc, echivalentă cu 2 2 2( ) ( ) ( ) 0a b c a b c a b c a b c+ − + + − + + − = nu

poate avea loc datorită inegalităţii triunghiului, de unde rezultă că =b c , adică triunghiul ABC este isoscel. Soluţia 2. Fie

b cP BC CC= ∩ . Construim paralelogramul

c bBC QC (Fig. 32). Dacă

≠AB AC , fie <AB AC ( <c b ), de unde − > −p c p b şi ( ) ( )b c

m C BC m C CB> , deci

CP BP> . Dacă c b

CC BC≡ rezultă c b

C P PC< (1). În triunghiurile ABC şi c

AC C

avem: b c

AC AC p a= = − , c b

CC BC≡ şi am presupus că <AB AC . Urmează că :

( ) ( )b c

m AC B m AC C< şi ( ) ( )b c

m BC C m BC C> de unde rezultă că ( ) ( )b c

m PC C m PC B>

(2). Din triunghiurile c

BC P şi b

PC C cu c b

BPC C PC≡ şi relaţia (2) rezultă:

( ) ( )c b

m C BP m PCC> (3). Cum triunghiul c

C PQ este isoscel rezultă: c c

C QC C CQ≡ de

unde ( ) ( ) ( ) ( ).c b b b b

m C QC m C QC m PCC m C CQ+ = + Utilizând relaţia (3) rezultă

( ) ( )b b

m C QC m C CQ> şi de aici b b

C C C Q< , adică − < −p c p b de unde <b c , ceea ce

contrazice presupunerea facută. Analog se tratează cazul în care >b c . Atunci =b c , deci triunghiul ABC este isoscel.

B

cC

aC

p a− p a−

A

p b− p c−

p c− p b−

bC

C

Fig. 31

A

Q

C

cC bC

B

P

Fig. 32

Page 29: Barbu_TEOREME_FUNDAMENTALE(2008).pdf

36

30) Bisectoarele interioare ale unghiurilor B şi C ale triunghiului ABC întâlnesc ceviana Gergonne AD în E, respectiv F. Dacă =BE CF , atunci triunghiul ABC este isoscel. Demonstraţie. Presupunem ≠AB AC , fie

<AB AC Atunci, ,> − < −b c p b p c şi

( )∈E FD (Fig. 33). Din ( ) ( )<m ABC m ACB

rezultă ( ) ( ) ( )> >m EBC m FCD m ECB , deci

CE BE> , adică CE CF> (4) (deoarece

BE CF≡ ). Cum ( ) ( ) 90m ADC m EDC= > °

rezultă că ( )m FEC =

( ) ( ) 90m EDC m ECD+ > ° şi ( ) 90m EFC < ° ,

de unde CE CF< ceea ce contrazice (4). Analog, dacă AB AC> se ajunge la o contradicţie. Urmează că AB AC≡ .

I.6. Punctul lui Nagel. Dreapta lui Nagel7

„Se poate vorbi de un umanism modern, de un sistem complet de cunoştinţe capabil să formeze omul, bazat însă pe matematică? Sunt convins că da.”

Ion Barbu8

Fie , ,a b cτ τ τ punctele de contact dintre cercurile A – exînscris, B – exînscris,

respectiv C – exînscris cu laturile BC, CA, respectiv AB ale triunghiului ABC (Fig. 34). Teorema lui Nagel

1) Dreptele , ,a b cA B Cτ τ τ sunt concurente. Demonstraţie. Soluţia 1. Fie a, b, c lungimile laturilor triunghiului ABC şi p semiperimetrul său. Fie =

aB xτ şi .=

aC yτ Atunci, x y a+ = şi x c y b+ = + de unde

x p c= − şi ,y p b= − deci .−

=−

a

a

B p c

C p b

ττ

Analog, −

=−

b

b

C p a

A p c

ττ

şi ,−

=−

c

c

A p b

B p a

ττ

de unde

rezultă 1⋅ ⋅ =a b c

a b c

B C A

C A B

τ τ ττ τ τ

şi din reciproca teoremei lui Ceva rezultă că dreptele

, ,a b cA B Cτ τ τ sunt concurente.

7 Christian von Nagel (1803-1882) – matematician german, contribuţii în geometria triunghiului 8 Ion Barbu (1895-1961) – matematician român, profesor la Universitatea din Bucureşti, contribuţii în algebră şi

geometrie

D C

A

p-c p-b

F

E

B

Fig. 33

Page 30: Barbu_TEOREME_FUNDAMENTALE(2008).pdf

37

Soluţia 2. Din −

=−

a

a

B p c

C p b

ττ

rezultă că afixul punctului aτ este

( ) ( )

1a

B C

B C

p cz z

p b z p c zp bz

p c a

p b

τ

−+ ⋅

− + −−= =

−+

şi analog se obţin relaţii similare pentru

punctele bτ şi

cτ . Dacă ( )∈

aP Aτ şi =

a

APk

Pτ atunci

1

+=

+aA

P

z kzz

k

τ de unde rezultă:

1 1( ) ( ) ( )

1

= − + − + − + −

P A B C

k kz p a z p b z p c z

k p a a a. Se obţine o formă simetrică pentru

relaţia din paranteză dacă 1

=−

k

p a a, adică pentru =

−a

kp a

şi fie N punctul

corespunzător acestei valori a lui k. Obţinem un punct ce va avea afixul 1[( ) ( ) ( ) ].= − + − + −N A B Cz p a z p b z p c z

p Simetria relaţiei precedente arată că punctul N

aparţine şi dreptelor b

Bτ , respectiv c

Cτ .

A

B

C

Ia

Ib

Ic

N

Fig. 34

bτ cτ

Page 31: Barbu_TEOREME_FUNDAMENTALE(2008).pdf

38

Observaţii:

1) Punctul de concurenţă al dreptelor , ,a b cA B Cτ τ τ se numeşte punctul lui Nagel. 2) Afixul punctul lui Nagel (N) al triunghiului ABC este dat de:

1[( ) ( ) ( ) ].= − + − + −N A B Cz p a z p b z p c z

p

2) Triunghiul a b cτ τ τ se numeşte triunghiul lui Nagel (sau triunghiul cotangentic).

3) Triunghiul cotangentic a b cτ τ τ este triunghiul cevian al punctului lui Nagel.

2) Dacă N este punctul lui Nagel al triunghiului ABC, atunci pentru orice punct M din planul triunghiului este adevărată egalitatea:

1[( ) ( ) ( ) ].MN p a MA p b MB p c MC

p= − + − + −

uuuur uuur uuur uuuur

Demonstraţia rezultă din precedenta. 3) Coordonatele baricentrice absolute ale punctului lui Nagel sunt

, ,p a p b p c

Na b c

− − −

.

Demonstraţia rezultă din proprietatea de mai sus. 4) Punctul lui Nagel este centrul de omologie dintre triunghiul neisoscel ABC şi triunghiul său cotangentic

a b cτ τ τ .

Demonstraţie. Triunghiurile ABC şi a b cτ τ τ sunt omologice, centrul de omologie fiind

punctul lui Nagel al triunghiului ABC (ca o consecinţă a teoremei lui Desargues). 5) Într-un triunghi ABC, punctul lui Nagel ( N ), centrul de greutate (G) şi centrul cercului înscris (I) sunt coliniare şi 2 .GN GI=

Demonstraţie. Soluţia 1. Fie 'A piciorul bisectoarei din A (Fig. 35). Din teorema bisectoarei rezultă

'

'=

A B c

A C b şi de aici: '

+=

+

a cA B

b c. Teorema

bisectoarei aplicată în triunghiul 'ABA ne dă:

(1)' '

+= =

IA c b c

IA A B a de unde:

';

' 2

IA a

AA p= Dacă

aM este mijlocul segmentului BC iar

aτ şi

punctele de tangenţă al cercurilor A – exînscris şi B – exînscris cu latura BC respectiv AC, atunci

( )( ) , ' ,

2 2 2 2( )

− −= − − = = − =

+ +a a a

a b c a ac a b cM p b A M

b c b cτ

de unde (2).+

=aM b c

MA a

τ Din relaţiile (1) şi (2) rezultă

a aIM Aτ şi de

aici: (3).'

=a

a

IM IA

A AAτ Fie .= ∩

aG AM IN Cum IM AN rezultă

(4).= =a a

GA AN GN

GM IM GI Din relaţiile (3) şi (4) rezultă

' (5).

'= ⋅

a a

GA NA AA

GM A IAτ Teorema

A

B C

N G I

aM A'

Fig. 35

Page 32: Barbu_TEOREME_FUNDAMENTALE(2008).pdf

39

lui Menelaus aplicată în triunghiul a

A Cτ şi transversala , ,bN Bτ ne dă:

1⋅ ⋅ =b a

b a

A NBC

C B NA

τ ττ τ

, de unde 1−

⋅ ⋅ =− −

aNp c a

p a p c NA

τ şi de aici

−=a

N p a

NA a

τ, adică

=a

a

N a

A p

ττ

. Atunci, relaţia (5) devine 2

2= ⋅ =a

GA a p

GM p a, de unde 2 (6)=

aGA GM , adică

G este centrul de greutate al triunghiului ABC, deci punctele N, G şi I sunt coliniare. Din relaţiile (4) şi (6) rezultă 2 .=GN GI Soluţia 2. Afixele centrului de greutate G al centrului cercului înscris I sunt şi al punctului

lui Nagel sunt: ,3 2

+ + + += =A B C A B C

G I

z z z az bz czz z

p respectiv

1[( ) ( ) ( ) ].= − + − + −N A B Cz p a z p b z p c z

p Atunci,

1

2

−= ∈

−G I

N G

z z

z z deci punctele G, I şi

N sunt coliniare şi 2 ,− = −N G G Iz z z z adică: 2 .=NG GI

Observaţie: Dreapta IN se numeşte dreapta lui Nagel. 6) Într-un triunghi ABC fie O centrul cercului circumscris, H ortocentrul său, I centrul cercului înscris triunghiului, N punctul lui Nagel al triunghiului ABC. Atunci

2=HN OI şi .HN OI

Demonstraţie. Soluţia 1. Fie H ortocentrul triunghiului .ABC Atunci, 2HG GO= (dreapta lui Euler) şi 2 .=NG GI Din asemănarea triunghiurilor OGI şi HGN (deoarece

≡ NGH OGI şi =GH GO

GN GI) rezultă că HN OI şi 2 .=HN OI

Soluţia 2. Alegem un reper cartezian cu originea în O, centrul cercului circumscris

triunghiului ABC. Atunci, = + +H A B Cz z z z şi 2

−= − ∈

−N H

I O

z z

z z adică NH OI şi

2 ,− = ⋅ −N H I Oz z z z adică 2 .=NH OI

7) Consecinţă: Într-un triunghi ABC fie O centrul cercului circumscris, H ortocentrul său, I centrul cercului înscris triunghiului, N punctul lui Nagel al triunghiului ABC. Segmentele HI şi ON sunt congruente. Demonstraţie. Din trapezul isoscel HNOI rezultă

.≡HI ON 8) În triunghiul ABC fie O centrul cercului circumscris, I centrul cercului înscris, N punctul lui Nagel şi 9O centrul cercului lui Euler. Dreapta care uneşte mijloacele

segmentelor NI şi NO conţine punctul 9O .

Demonstraţie. În triunghiul ,NOI dreapta (d) care uneşte mijloacele laturilor NO şi NI este paralelă cu dreapta OI, deci paralelă şi cu HN (Fig. 36). În triunghiul ,NOH

dreapta d fiind paralelă cu NH rezultă că trece şi prin mijlocul lui OH, adică prin 9O -

centrul cercului lui Euler al triunghiului .ABC

H

I

G

O

N

9O

Fig. 36

Page 33: Barbu_TEOREME_FUNDAMENTALE(2008).pdf

40

9) Punctul lui Spieker, centrul cercului înscris în triunghiul median al triunghiului ABC, este mijlocul segmentului .IN Demonstraţie. Soluţia 1. Fie

a b cM M M triunghiul median al triunghiului ABC (Fig. 37).

Dacă 2 ,IG x= atunci ,conform aplicaţiei precedente, 4 ,=GN x 6 .=IN x Fie PS mijlocul

segmentului .IN Avem: 3= =P p

IS x S N şi .P

GS x=

Deoarece P

G IS∈ şi 2 2P

IG x GS= = , iar 2a

AG GM=

rezultă 2a P

AG IG

GM GS= = şi cum

P aIGA S GM≡ avem

că triunghiurile AGI şi a P

M GS sunt asemenea, deci

.P a

AI S M Dar ,a b a c

AB M M AC M M iar AI este

bisectoarea unghiului ,BAC deci şi a pM S este

bisectoarea unghiului .c a b

M M M Analog se arată că

punctul PS aparţine bisectoarelor unghiurilor

a b cM M M şi ,

b c aM M M deci punctul lui Spieker

(PS ) este mijlocul segmentului .IN

Soluţia 2. Atunci , ,2 2 2b c c a a b

a b cM M M M M M= = = şi '

2

pp = unde p este

semiperimetrul triunghiului median. Atunci afixul centrului cercului înscris în triunghiul

median este: 1 1

2 ' 2 2 2 2 ' 2 2 2 2 2 2p a b c

B C C A A BS M M M

z z z z z za b c a b cz z z z

p p

+ + + = ⋅ + ⋅ + ⋅ = ⋅ + ⋅ + ⋅ =

adică : pS

z =1

[( ) ( ) ( ) ] ( )4

+ + + + + ∗A B Cb c z c a z a b zp

. Dacă S este mijlocul segmentului

aIN , atunci rezultă

1 1 1( ) [( ) ( ) ( ) ]

2 2 2I N

S A B C A B C

z zz az bz cz p a z p b z p c z

p p

+ = = + + + − + − + − =

[ ]1( ) ( ) ( )

4 A B Cb c z c a z a b zp

+ + + + + (**). Din relaţiile (*) şi (**) rezultă =pS Sz z adică

.PS S≡

10) Consecinţă : Dreptele IH şi

pS O sunt paralele şi 2

pHI S O= .

Cercul înscris în triunghiul median - C ( , / 2)PS r - se numeşte cercul lui Spieker.

11) Punctele , , ,

pI G S N sunt coliniare şi 12 6 4 3 2 .= = = =

p pGS GI IS NG NI

Demonstraţie. O primă soluţie rezultă din proprietăţile precedente. Coliniaritatea punctelor o mai putem demonstra şi prin utilizarea coordonatele baricentrice. Astfel,

(1,1,1), ( , , ), ( , , ), ( , , ).− − − + + +G I a b c N p a p b p c S b c c a a b Deoarece

1 1 1

0

a b c

p a p b p c

=

− − −

şi

1 1 1

0

a b c

b c c a a b

=

+ + +

rezultă că punctele G, I, N respectiv G, I, S

sunt coliniare, adică punctele G, I, N, S sunt coliniare.

A

B C

I G

pS

N

aM

bM cM

Fig. 37

Page 34: Barbu_TEOREME_FUNDAMENTALE(2008).pdf

41

Observaţie: Punctele , , ,p

I G S N aparţin dreptei lui Nagel.

12) Punctele lui Nagel şi Gergonne ale unui triunghi sunt izotomic conjugate. Demonstraţie. Fie

a b cC C C triunghiul de contact al triunghiului ABC,

eG - punctul lui

Gergonne şi , ,a b cτ τ τ punctele de tangenţă ale cercurilor exînscrise cu segmentele BC, CA

respectiv AB. Deoarece = = −a a

B CC p cτ rezultă că punctele aτ şi

aC sunt simetrice faţă

de mijlocul laturii BC. Analog, punctele bτ şi

bC , respectiv

cτ şi

cC sunt simetrice faţă de

mijloacele laturilor AC, respectiv AB. Cum dreptele , ,a b cA B Cτ τ τ sunt concurente în

punctul lui Nagel al triunghiului ABC, rezultă că punctele lui Nagel şi Gergonne sunt izotomic conjugate.

13) Centrul cercului înscris în triunghiul ABC este punctul lui Nagel al triunghiului median. Demonstraţie. Soluţia 1. Fie

a b cM M M şi

a b cC C C triunghiurile

median, respectiv de contact ale

triunghiului ABC , ' ,= ∩aC AN BC

= ∩b c

P AN M M şi

' = ∩a b cP M I M M (Fig. 38).

Deoarece '

a aAC M I rezultă

'

a aAC M P (vezi „Cercuri

exînscrise”) şi punctul aM este

mijlocul segmentului 'a a

C C atunci

dreapta aM I va conţine mijlocul

segmentului aAC - punct ce aparţine

liniei mijlocii b cM M - deci punctul '.P Cum 'PP este linie mijlocie în triunghiul 'a a

AC C

, deci mediana aAM trece şi prin mijlocul segmentului ',PP deci şi prin mijlocul

segmentului b cM M , adică punctele P şi 'P sunt izotomice. Conform proprietăţii

precedente rezultă concluzia. Soluţia 2. Fie 'N punctul lui Nagel al triunghiului median .a b cM M M Atunci, afixul lui

'N este: '

1 1[( ' ') ( ' ') ( ' ') ] ( ) ,' 2

= − + − + − = + + =a b cN M M M A B C Iz p a z p b z p c z az bz cz z

p pde

unde rezultă că ' .≡N I 14) Punctul lui Nagel al triunghiului ABC este centrul cercului înscris în triunghiul anticomplementar al său. Demonstraţie. Se aplică proprietatea precedentă luând triunghiul median în rolul triunghiului ABC. 15) Într-un triunghi ABC, distanţa de la punctul lui Nagel (N) la centrul cercului circumscris (O) este egală cu diferenţa dintre raza acestui cerc şi diametrul cercului înscris.

A

B C

P P '

N I pS

aM

bM cM

'aC Fig. 38

aC

Page 35: Barbu_TEOREME_FUNDAMENTALE(2008).pdf

42

Demonstraţie. Cercul circumscris triunghiului ABC este cercul lui Euler al triunghiului anticomplementar ' ' 'A B C având raza R. Cercul înscris în triunghiul ' ' 'A B C are centrul în punctul lui Nagel – conform proprietăţii precedente – deci, aceste două cercuri sunt tangente în 'ϕ - punctul lui Feuerbach al triunghiului anticomplementar. Astfel,

' ' 2ON O N R rϕ ϕ= − = − (r fiind raza cercului înscris în triunghiul ABC).

Observaţie: Analog, se arată că 2 , 2a a b bON R r ON R r= + = + şi 2c cON R r= + (unde

, ,a b cN N N sunt punctele adjuncte ale lui Nagel, iar , ,a b cr r r lungimile razelor cercurilor

exînscrise triunghiului ABC ). 16) Punctele lui Nagel (N), Bevan (V) şi Longchamps (L) sunt coliniare, iar .≡NV VL Demonstraţie. Vezi „Punctul lui Bevan”. 17) Punctul lui Nagel (N), punctul lui Gergonne (Γ ), centrul antibisector (Z) şi retrocentrul (R) unui triunghi ABC sunt coliniare. Demonstraţie. Vezi „Punctul lui Gergonne” şi „Retrocentrul unui triunghi”. 18) Fie a b cC C C triunghiul de contact al triunghiului ABC şi

( , , ), ( , , ),a a a b b bD E F D E F ( , , )c c cD E F punctele de tangenţă dintre cercurile A, B, C – exînscrise corespunzătoare triunghiului ABC cu laturile BC, CA respectiv AB. Dreptele

, ,a c bAC BE CF sunt concurente. Demonstraţie. Fie a, b, c lungimile laturilor BC, CA, respectiv AB şi p semiperimetrul triunghiului ABC. Avem

, , ,a a bBC p b CC p c AF p c= − = − = − , ,b c cBF p CE p AE p b= = = − (vezi „Cercuri

exînscrise”) de unde 1,a b c

a b c

BC F A E C

CC F B E A⋅ ⋅ = adică dreptele ,a cAC BE şi bCF sunt concurente

într-un punct aN .

Observaţii:

1) Analog, cevienele ,bBC aCF şi cAD sunt concurente într-un punct bN iar cevienele

, ,c bCC AD aBE sunt concurente într-un punct .cN

2) Punctele , ,a b cN N N se numesc punctele adjuncte ale punctului lui Nagel.

19) Triunghiul a b cN N N - având vârfurile în punctele adjuncte ale lui Nagel - şi

triunghiul a b cΓ Γ Γ - având vârfurile în punctele adjuncte ale lui Gergonne - sunt

omologice, centrul de omologie aparţinând dreptei NΓ (unde N este punctul lui Nagel şi Γ este punctul lui Gergonne al triunghiului ABC ).

Demonstraţie. Vezi „Punctul lui Gegonne”. 20) Într-un triunghi cevienele punctului lui Nagel (N) trec prin punctele diametral opuse punctelor de contact ale cercului înscris, iar cevienele punctului lui Gergonne (Γ ) trec prin punctele diametral opuse ale punctelor de contact ale cercurilor exînscrise respective. Demonstraţie. Fie P şi Q punctele diametral opuse lui aC şi aD în cercurile înscris

respectiv A – exînscris (Fig. 39). Din aPC BC⊥ şi aQD BC⊥ rezultă .a aPC QD

Page 36: Barbu_TEOREME_FUNDAMENTALE(2008).pdf

43

Deoarece cercul înscris şi cel A – exînscris sunt omotetice, omotetia fiind de centru A şi

raport ,ar

r iar a a aI D r

IP r= rezultă că punctele aD şi P se corespund prin omotetia

considerată, deci .P AN∈ Analog, .Q A∈ Γ

21) Fie 1 1 1, ,A B C proiecţiile centrului cercului înscris (I) al triunghiului ABC pe

mediatoarele laturilor triunghiului. Triunghiurile ABC şi 1 1 1A B C sunt omologice, centrul de omologie fiind punctul lui Nagel. Demonstraţie. Fie P punctul diametral opus punctului de contact aC dintre cercul înscris în triunghiul ABC şi latura

BC (Fig. 39). Punctele A, P şi aD (punctul de tangenţă

dintre cercul A – exînscris şi BC) sunt coliniare. Deoarece triunghiul a aPC D este dreptunghic iar I este mijlocul

catetei aC P , atunci 1'IA este linie mijlocie în a aPC D

(unde 1'A este mijlocul ipotenuzei aPD ), deci şi 1' aA M va

fi linie mijlocie de unde rezultă că 1' aA M este mediatoarea

segmentului ,a aC D deci 1 1' .A A≡ Analog, se arată că 1B şi

1C aparţin cevienelor punctului lui Nagel. Deci

triunghiurile 1 1 1A B C şi ABC sunt omologice, centrul de

omologie fiind punctul lui Nagel (N). 22) Într-un triunghi, paralelele duse prin mijlocul laturilor la cevienele punctului lui Nagel sunt concurente în centrul cercului înscris. Demonstraţie. Fie aM mijlocul laturii BC şi P punctul diametral opus lui aC în cercul

înscris în triunghiul ABC (Fig. 39). Din proprietatea precedentă rezultă că punctele A, P şi

aD sunt coliniare. Atunci, aIM este linie mijlocie în triunghiul ,a aPC D deci ,a aPD IM

adică .a aAD IM

23) Punctul lui Nagel al triunghiului ABC aparţine cercului lui Fuhrmann corespunzător triunghiului ABC. Demonstraţie.vezi „Triunghiul lui Fuhrmann”. 24) Dreptele ce unesc punctul lui Nagel al unui triunghi ABC cu vârfurile acestuia conţin punctele de contact dintre cercul lui Spiecker şi triunghiul median al triunghiului ABC.

Demonstraţie. Fie a b cM M M

triunghiul median al triunghiului ABC, P punctul de contact dintre cercul lui Spiecker al triunghiului ABC şi latura b cM M , ,= ∩D AN BC

1 = ∩P AP BC (Fig. 40). Deoarece

laturile triunghiului median au

A

Fig. 39

B

C aC aD

aI

P

Q

N

Γ I 1A

aM

A

B C

P

N I pS

aM

bM cM

Fig. 40

D

Page 37: Barbu_TEOREME_FUNDAMENTALE(2008).pdf

44

lungimile / 2, / 2, / 2a b c şi P este un punct de contact al cercului înscris în triunghiul

a b cM M M rezultă (1)2 2c

p c BDM P

−= = (D fiind punctul de tangenţă dintre cercul A –

exînscris cu latura BC). Dar cM P este linie mijlocie în triunghiul 1,ABP deci

1 (2).2c

BPM P = Din (1) şi (2) rezultă 1≡BD BP şi cum 1, [ ]∈D P BC rezultă 1,D P≡ deci

.P AN∈ 25) Fie , ,A B CQ Q Q mijloacele segmentelor AN, BN, respectiv CN (unde N este punctul lui

Nagel al triunghiului ABC). Triunghiul A B CQ Q Q este circumscris cercului Spieker al triunghiului ABC. Demonstraţie. Prin omotetia de centru N şi raport 1/2 triunghiul A B CQ Q Q se transformă în

triunghiul ABC, deci şi cercul înscris în triunghiul A B CQ Q Q se transformă în cercul înscris

în triunghiul ABC, iar 1

2NS NI=uuur uur

(unde S şi I sunt centrele cercurilor înscrise în

triunghiurile A B CQ Q Q şi ABC), relaţie ce arată că S este punctul lui Spieker al triunghiului

ABC. Observaţie: Fie I şi O centrele cercurilor înscris, respectiv circumscris ale unui triunghi

ABC, ' , ,⊥ ∈MM AI M AB ' , '∈ ∈M AC I MM . Cercul tangent în M şi 'M laturilor AB,

respectiv AC este tangent şi cercului circumscris triunghiului ABC.

Demonstraţie. Deoarece mediatoarele 'OB şi 'OC sunt perpendiculare pe AC respectiv AB (unde

', '∈B C C(O,R)), rezultă ' 'aOB O M şi

' ,aOC O M unde aO este centrul crecului tangent

laturilor AB şi AC în M respectiv 'M (Fig. 41). Atunci, triunghiurile ' 'OB C şi 'aO M M sunt

omotetice; fie P centrul acestei omotetii - ' ' '.= ∩P C M B M Deoarece hexagonul

' 'PC CABB are cinci vârfuri pe cercul circumscris triunghiului ABC şi că laturile sale se intersecteză în punctele coliniare M, I şi 'M rezultă – din teorema lui Pascal – că punctul P aparţine cercului circumscris triunghiului ABC. Deoarece 'OC OP≡ rezultă ,a aO M O P≡ deci P aparţine şi cercului

tangent laturilor AB şi AC având centrul în ,aO

deci cercul C ( , )a aO O P este tangent cercului C(O,R ) în P.

26) Fie 'MM ( , )∈ ∈M AB N AC perpendiculara în I – centrul cercului exînscris în triunghiul ABC – pe bisectoarea AI. Dacă P este punctul de tangenţă dintre cercul circumscris triunghiului ABC şi cercul tangent în M şi 'M laturilor AB şi AC, atunci AP este izogonala cevianei aAD a punctului lui Nagel.

Demonstraţie. Fie 1 (∈B AC şi 1 (∈C AB astfel încât 1AB AB≡ şi 1 .AC AC≡ Dreapta

1 1BC este a doua tangentă comună cercurilor înscrise în triunghiul ABC şi A – exînscris;

A

B

C

P

M

M' I

B'

C'

O aO

Fig. 41

Page 38: Barbu_TEOREME_FUNDAMENTALE(2008).pdf

45

aC şi aD punctele de tangenţă dintre cercul înscris

şi A – exînscris cu dreapta BC (Fig. 42). Atunci simetricele lor '

aC şi '

aD faţă de bisectoarea

unghiului A vor fi punctele de tangenţă dintre 1 1BC

cu cercurile înscris, respectiv A – exînscris, ceea ce arată că '

aAD şi '

aAC sunt izogonalele

dreptelor ,aAD respectiv .aAC Arătăm că punctul '∈a

P AD Prin inversiunea de centru A şi raport

AB AC⋅ punctul 'a

D se transformă în punctul P

deci dreapta 'a

PD trece prin punctul A.

27) Fie ', ', 'A B C punctele de intersecţie dintre bisectoarele interioare ale unghiurilor unui triunghi ABC şi cercul circumscris triunghiului ABC, iar a b cC C C triunghiul de

contact corespunzător triunghiului ABC. Dreptele ' , ' , 'a b cA C B C C C sunt concurente în izogonalul punctului lui Nagel. Demonstraţie. Fie " ' '= ∩A AB B C (Fig. 43).

Atunci, 1

( " ') [ ( ) ( )]2

= + m AA B m B m C , de

unde rezultă că ( " ') ( " ')+ = m AA B m A AA

1[ ( ) ( ) ( )] 90

2+ + = ° m B m C m A , deci

' ',AA BB⊥ adică bisectoarea 'AA a unghiului A este înălţime în triunghiul ' ' '.A B C Analog , se arată că 'BB şi 'CC sunt înălţimi în triunghiul

' ' '.A B C Deoarece a bAI C C⊥ rezultă

' '.c bC C C B Analog ' ', ' ',a c a bC C A C C C A B

deci triunghiurile a b cC C C şi ' ' 'A B C sunt

omotetice. Şi fie ' ' ' '= ∩ ∩a b cN A C B C C C

centrul acestei omotetii. Conform problemei precedente punctul '= ∩P AN C(O,R) aparţine izogonalei cevienei aAD a punctului lui

Nagel. Deoarece segmentele paralele ' ', b cB C C C şi ' ( ' , ')⊥ ∈MM MM AI I MM sunt

omotetice, centrele de omotetie vor fi coliniare. Cum aP este centrul de omotetie al

segmentelor ' 'B C şi 'MM iar A centrul de omotetie al segmentelor 'MM şi b cC C rezultă

că 'N centrul de omotetie dintre ' 'B C şi b cC C aparţin dreptei ,aAP adică izogonalei

cevianei aAD a punctului lui Nagel. Analog, se arată că punctul 'N aparţine şi

izogonalelor cevianelor bBE şi cCF ale punctului lui Nagel al triunghiului ABC.

A

B C

I M

M'

P

1B

1C

aI

aC 'aC

aD

'aD

Fig. 42

A

B C

P A'

B'

C'

M

M' I

A"

aC

bC

cC

Fig. 43

N'

Page 39: Barbu_TEOREME_FUNDAMENTALE(2008).pdf

46

28) Izogonalul punctului lui Nagel aparţine dreptei ce uneşte centrul cercului înscris (I) cu centrul cercului circumscris (O) al unui triunghi. Demonstraţie. Deoarece triunghiul de contact a b cC C C şi triunghiul circumpedal ' ' 'A B C

al lui I sunt omotetice (conform proprietăţii precedente), punctul ' ' ' '= ∩ ∩a b cN A C B C C C - izogonalul punctului lui Nagel – fiind centrul de omotetie

(conform proprietăţii precedente), rezultă că centrele cercurilor circumscrise triunghiurilor

a b cC C C şi ABC – adică I şi O – se corespund prin această omotetie de centru 'N - adică

punctele ',N I şi O sunt coliniare. 29) Fie O centrul cercului circumscris triunghiului ABC, a b cH H H triunghiul ortic al tri-

unghiului ABC, '= I b cA AO H H , '= I a cB BO H H , ' .= I b aC CO H H Dreptele

' , ' , 'a b cA H B H C H sunt concurente în punctul lui Nagel al triunghiului ortic a b cH H H .

Demonstraţie. Deoarece A, B, C sunt centrele cercurilor exînscrise corespunzătoare triunghiului ortic

a b cH H H , iar AO, BO, CO sunt

perpendiculare pe , ,b c c aH H H H respectiv

a bH H (vezi „Triunghiul ortic”), rezultă că

', ', 'A B C sunt punctele de tangenţă dintre cercurile exînscrise triunghiului ortic cu laturile acestuia, deci dreptele ' ,aA H

' ,bB H ' cC H sunt concurente în punctul

lui Nagel al triunghiului ortic a b cH H H .

30) În triunghiul ABC, fie ( ), ( ).∈ ∈U AB V AC Dacă dreapta UV trece prin punctul lui

Nagel (N) al triunghiului ABC, atunci ( ) ( ) .⋅ − + ⋅ − = −UB VC

p b p c p aUA VA

Demonstraţie. Fie , ,a b cτ τ τ punctele de tangenţă

ale cercurilor exînscrise cu laturile BC, CA respectiv AB (Fig. 45). Deoarece

N UV∈ atunci ⋅ + ⋅ =a a aC B NUB VC

UA BC VA BC AN

τ τ τ şi

(1)−

=aN p a

AN a

τ (vezi „Relaţia lui Van –

Aubel”). Atunci: ,UB p b VC p c p a

UA a VA a a

− − −⋅ + ⋅ =

de unde rezultă concluzia.

A

B C

N

Fig. 45

U V

A

B' B C

H

aH

bH

cH

Fig. 44

A'

O N'

Page 40: Barbu_TEOREME_FUNDAMENTALE(2008).pdf

47

31) În triunghiul ABC, fie ( ), ( ).∈ ∈U AB V AC Dacă dreapta UV trece prin punctul lui

Nagel (N) al triunghiului ABC atunci: .2 2 2

UB B VC C Actg ctg ctg

UA VA⋅ + ⋅ =

Demonstraţie. Din proprietatea precedentă rezultă următoarea egalitate: 1 ( ) 1 ( ) 1 ( )

.− − −

⋅ ⋅ + ⋅ ⋅ = ⋅UB p p b VC p p c p p a

UA b ac VA c ab a bc Utilizând relaţia 2( )

cos−

=p p a

Abc

şi

analoagele precum şi teorema sinusurilor avem:

2 2 2cos cos cos2 2 2

sin sin sin

B C A

UB VC

UA B VA C A⋅ + ⋅ =

relaţie echivalentă cu .2 2 2

UB B VC C Actg ctg ctg

UA VA⋅ + ⋅ =

32) Dacă punctul lui Nagel al unui triunghi ABC aparţine cercului înscris în triunghiul ABC, atunci suma a două laturi ale triunghiului este egală cu triplul celei de a treia. Demonstraţie.

Fie , ,a a aH C N proiecţiile punctelor A, I, respectiv N pe latura BC. Din asemănarea

triunghiurilor a aNNτ şi a aAHτ rezultă 2aa a

a

N p aNN h r

A a

ττ

−= ⋅ = ⋅ ,

2 2 2 ( )( )

2a

a a a a

a

N a b c p a b c p aN H p b

A a p a

ττ τ

τ + − − − −

= ⋅ = − + ⋅ =

şi a aC b cτ = − . Deoarece

punctul lui Nagel aparţine cercului înscris în triunghiul ABC rezultă 2 2 2( )a a a ar IC NN C N= − + egalitate echivalentă cu :

2 22 2

2 2

(2 ) ( ) (2 )a b c b c a pr r

a a

− − − −= ⋅ + , de unde rezultă concluzia.

A

B C

N

aN

I

aC aH aτ

Fig. 46

Page 41: Barbu_TEOREME_FUNDAMENTALE(2008).pdf

48

I.7. Punctul lui Longchamps 9 „Matematica şi arta izvorăsc din partea cea mai curată a sufletului omenesc, numai că arta este expresia pură a sentimentului, pe când matematica este expresia cristalină a raţiunii pure.” – Immanuel Kant10

Simetricul ortocentrului H al unui triunghi faţă de centrul cercului circumscris O al unui triunghi ABC se numeşte punctul lui Longchamps (L). 1) Ortocentrul H, centrul cercului circumscris O şi punctul lui Longchamps L sunt coliniare şi HO OL≡ şi 2 .LH OH= Demonstraţia rezultă din definiţia punctului lui Longchamps. 2) Consecinţă: Dacă G este centrul de greutate al unui triunghi ABC, atunci

4.

3LG OH=

3) Punctul lui Longchamps al triunghiului ABC aparţine dreptei lui Euler a triunghiului ABC. Demonstraţia rezultă din definiţia punctului lui Longchamps. 4) Punctul lui Longchamps al unui triunghi ABC este ortocentrul triunghiului anticomplementar al triunghiului ABC. Demonstraţie. Fie H ortocentrul triunghiului ABC (Fig. 47) şi 1L ortocentrul triunghiului

anticomplementar ' ' 'A B C , 1 'D A L BC= ∩ ,

aH AH BC= ∩ şi 1O mijlocul segmentului

1L H . Din congruenţa triunghiurilor 'BDA şi

aAH C rezultă aBD CH≡ (1). Fie

'O proiecţia lui 1O pe BC. Cum 1O este

mijlocul segmentului 1L H rezultă că 'O este

mijlocul segmentului aDH , adică

' ' aDO O H≡ (2).Din relaţiile (1) şi (2) rezultă

că ' 'BO O C≡ , adică 1 'OO este mediatoarea

laturii BC. Analog se arată că 1O aparţine şi

mediatoarelor laturilor AC, respectiv AB, adică 1O coincide cu O -centrul cercului

circumscris triunghiului ABC, iar cum 1L este simetricul lui H faţă de O rezultă că 1L

coincide cu L- punctul lui Longchamps al triunghiului ABC. 5) Punctul lui Longchamps (L), punctul lui Bevan (V) şi punctul lui Nagel (N) al triunghiului ABC sunt coliniare şi .NV VL≡ Demonstraţie. Vezi „Punctul lui Bevan”.

9G. de Longchamps (1842-1906) – matematician francez, contribuţii în geometrie 10Immanuel Kant (1724-1804) – filosof german

A'

B' C'

C B

A

aH

bH

D

H

Fig. 47

O L

O'

Page 42: Barbu_TEOREME_FUNDAMENTALE(2008).pdf

49

6) Coordonatele baricentrice ale punctului lui Longchamps al unui triunghi ABC sunt: 2 2 2

sin2 , sin2 , sin2R R R

L A ctgBctgC B ctgCctgA C ctgActgBS S S

− − −

, unde prin S am notat aria

triunghiului ABC

Demonstraţie. Deorece O este mijlocul segmentului HL , iar 2 2 2

sin2 , sin2 , sin22 2 2

R R RO A B C

S S S

şi ( , , )H ctgBctgC ctgCctgA ctgActgB , rezultă concluzia.

7) Punctul lui Longchamps aparţine dreptei lui Soddy corespunzătoare triunghiului ABC.

Demonstraţie. Coordonatele baricentrice ale punctului lui Longchamps verifică ecuaţia dreptei lui Soddy 2 2 2( ) ( ) ( ) ( ) ( ) ( ) 0p a c b x p b a c y p c b a z− − + − − + − − = (vezi „Punctele Soddy”),

deci L aparţine acestei drepte.

8) Consecinţă: Dreapta Soddy intersectează dreapta lui Euler în punctul lui Longchamps al triunghiului. Demonstraţia este evidentă întrucât punctul lui Longchamps aparţine ambelor drepte. 9) Triunghiul podar al punctului lui Longchamps al unui triunghi ABC este triunghiul cevian al retrocentrului triunghiului ABC. Demonstraţie. Vezi „Retrocentrul unui triunghi”. Simetricul punctului lui Longchamps faţă de centrul cercului înscris în triunghiul ABC se numeşte punctul lui Longuet – Higgins.

10) Fie CA cercul având centrul în vârful A al triunghiului ABC şi raza de lungime egală

cu cea a laturii opuse BC; analog se definesc şi cercurile CB şi CC. Axele radicale ale

perechilor de cercuri considerate sunt concurente în punctul lui Longchamps al triunghiului ABC. Demonstraţie. Fie ' ' 'A B C triunghiul anticomplementar al triunghiului ABC şi "A piciorul înălţimii din 'A pe ' 'B C (Fig.48). Fie M al doilea punct de intersecţie

dintre cercurile CA şi CC., iar

' " ' .L A A B M= ∩ Deoarece axa

radicală 'B M este perpendiculară pe linia centrelor AC, iar

' 'AC A C (AC fiind linie mijlocie în triunghiul ' ' 'A B C ), rezultă ' ' ',B M A C⊥ deci 'B M este dreapta suport a înălţimii din

'B a triunghiului anticomplementar ' ' '.A B C Atunci L punctul de intersecţie dintre înălţimile ' "A A şi 'B M

A'

B' C'

C B

A

M

AC

CC

L

Fig. 48

Page 43: Barbu_TEOREME_FUNDAMENTALE(2008).pdf

50

este ortocentrul triunghiului ' ' ',A B C deci L este punctul lui Longchamps (vezi

proprietatea 3). Analog axele radicale ale cercurilor CA şi CB respectiv CB şi CC. trec tot

prin L, deci L este centrul radical al cercurilor CA, CB , CC.

I.8. Punctul lui Spieker11

„Matematicile pun în joc puteri sufleteşti care nu sunt cu mult diferite de cele solicitate de poezie şi arte.” – Ion Barbu12

Centrul cercului înscris în triunghiul median al unui triunghi ABC se numeşte punctul lui Spieker ( pS ) al triunghiului ABC (Fig. 49).

Cercul înscris în triunghiul median se numeşte cercul lui Spieker. 1) Raza cercului lui Spieker ( sr ) are lungimea jumătate din lungimea razei cercului înscris în triunghiul ABC.

Demonstraţie. Semiperimetrul triunghiului median a b cM M M este '2

pp = , iar

[ ] [ ]1

4a b cM M M ABCA A= de unde rezultă 1

'4sr p r p⋅ = ⋅ ⋅ , adică

1

2 4s

pr rp⋅ = ⋅ , deci

2s

rr = .

2) Punctul lui Spieker aparţine dreptei lui Nagel. Demonstraţie. Vezi „ Punctul lui Nagel”. 3) Punctul lui Spieker, centrul cercului înscris în triunghiul median al triunghiului ABC, este mijlocul segmentului .IN

Demonstraţie: vezi „ Punctul lui Nagel”.

11 Theodor Spieker (1828-1908) – matematician german, contribuţii în geometrie 12 Ion Barbu (1895-1961) – matematician român, profesor la Universitatea din Bucureşti, contribuţii în algebră şi

geometrie

A

B C

N I pS

aM

bM cM

Fig. 49

AQ

BQ CQ

P

Page 44: Barbu_TEOREME_FUNDAMENTALE(2008).pdf

51

4) Punctele , , ,p

I G S N sunt coliniare şi 12 6 4 3 2 .= = = =p p

GS GI IS NG NI

Demonstraţie. Vezi „ Punctul lui Nagel”.

5) Pentru orice punct M din planul unui triunghi ABC este adevărată relaţia :

(2 ) (2 ) (2 )

4p

MA p a MB p b MC p cMS

p

⋅ − + ⋅ − + ⋅ −=

uuur uuur uuuuruuuur

.

Demonstraţie. Din 2p

GI

S G= rezultă că pentru orice punct M din

planul triunghiului ABC este adevărată relaţia :

2

3pMI MS

MG+

=

uuur uuuuruuuur

sau 3

2p

MG MIMS

−= =

uuuur uuuruuuur

33 2

2

MA MB MC aMA bMB cMC

p

+ + + +⋅ −

=

uuur uuur uuuur uuur uuur uuuur

, de unde rezultă

concluzia (Fig. 50). 6) Coordonatele baricentrice absolute ale punctului lui Spieker sunt :

2 2 2, , .

4 4 4p

p a p b p cS

p p p

− − −

Demonstraţia rezultă din teorema precedentă. 7) Afixul punctului lui Spieker al unui triunghi ABC este egal cu :

(2 ) (2 ) (2 )

4p

A B CS

z p a z p b z p cz

p

⋅ − + ⋅ − + ⋅ −= .

Demonstraţia este analoagă celei din teorema (5). 8) Simetricul centrului cercului circumscris unui triunghi ABC faţă de punctul lui Spieker al triunghiului ABC este punctul lui Furhmann. Demonstraţie. Considerăm un reper complex având originea în centrul cercului circumscris triunghiului ABC (Fig. 51).

Atunci, 0,Oz = (2 ) (2 ) (2 )

2A B C

F

z p a z p b z p cz

p

− + − + −=

de unde rezultă că 2 p

F OS

z zz

+= .

9) Fie , ,A B CQ Q Q mijloacele segmentelor AN, BN respectiv CN (unde N este punctul lui

Nagel al triunghiului ABC). Triunghiul A B CQ Q Q este circumscris cercului Spieker al triunghiului ABC. Demonstraţie. Vezi „ Punctul lui Nagel”. 10) Consecinţă: Punctul lui Spieker al triunghiului ABC este centrul cercului înscris în triunghiul său median a b cM M M şi al triunghiului A B CQ Q Q ale cărui vârfuri sunt mijloacele segmentelor AN, BN respectiv CN (unde N este punctul lui Nagel al triunghiului ABC).

M

pS I G

Fig. 50

G

F H N

pS

O I Fig. 51

Page 45: Barbu_TEOREME_FUNDAMENTALE(2008).pdf

52

11) Punctele de contact dintre cercul lui Spieker şi laturile triunghiului A B CQ Q Q sunt

mijloacele segmentelor , ,a b cNC NC NC , unde , ,a b cC C C sunt punctele de contact dintre

cercul înscris în triunghiul ABC şi laturile acestuia, iar N punctul lui Nagel al triunghiului ABC. Demonstraţie. Prin omotetia de centru N şi raport ½ , punctul de contact dintre cercul lui Spieker şi latura B CQ Q se transformă în aC , deci acest punct este mijlocul segmentului

.aNC

12) Fie , ,A B CQ Q Q mijloacele segmentelor AN, BN respectiv CN (unde N este punctul lui

Nagel al triunghiului ABC). Triunghiurile median a b cM M M şi A B CQ Q Q sunt congruente. Demonstraţie.

Deoarece ( / 2)a b A B

M M Q Q c≡ = , ( / 2)b c B C

M M Q Q a≡ = şi ( / 2)c a C A

M M Q Q b≡ =

rezultă că a b c A B CM M M Q Q Q≡ .

13) Dreptele IH şi

pS O sunt paralele şi 2

pHI S O= .

Demonstraţie. Vezi „ Punctul lui Nagel”. 14) Punctele de contact dintre cercul lui Spieker al triunghiului ABC cu laturile triunghiului median sunt intersecţiile acestor laturi cu , ,AN BN CN , unde N este punctul Nagel al triunghiului ABC .

Demonstraţie. Fie P punctul de tangenţă dintre latura b c

M M cu cercul lui Spieker al

triunghiului ABC şi D AN BC= ∩ . Ţinând cont că lngimile laturilor triunghiului medial

a b cM M M sunt , ,2 2 2

a b c şi că P este un punct de contact al cercului înscris în triunghiul

a b cM M M rezultă 2 2c

p c BDM P

−= = şi cum cM P BD rezultă .P BD∈

15) Paralelele duse prin mijloacele , ,a b cM M M ale laturilor , ,BC CA respectiv AB ale

triunghiului ABC la bisectoarele interioare ale unghiurilor A, B, C sunt concurente în punctul lui Spieker al triunghiului ABC .

Demonstraţie. Deoarece ,a b a c

M M AB M M AC , iar AI este bisectoarea unghiului A

rezultă că paralela prin a

M la AI este bisectoarea interioară a c a b

M M M . Analog se arată

că paralelele considerate sunt bisectoarele interioare ale triunghiului median, deci concurente în punctul lui Spieker al triunghiului ABC . 16) Mijlocul segmentului ce uneşte ortocentrele triunghiurilor median şi ortic ale unui triunghi ABC este punctul lui Spieker al triunghiului ortic al triunghiului ABC. Demonstraţie. Vezi „ Triunghiuri ortopolare”. 17) Dreptele lui Simson ale punctelor de intersecţie cu bisectoarele exterioare ale unghiurilor unui triunghi ABC cu cercul circumscris triunghiului trec prin mijloacele laturilor triunghiului, sunt paralele cu bisectoarele interioare ale triunghiului şi sunt concurente în punctul lui Spieker al triunghiului ABC. Demonstraţie. Vezi „Dreapta lui Simson”.

Page 46: Barbu_TEOREME_FUNDAMENTALE(2008).pdf

53

18) Punctul lui Spieker Sp, ortocentru H şi punctul lui Bevan V ale unui triunghi ABC sunt coliniare şi

p pHS S V≡ .

Demonstraţie. Vezi „Punctul lui Bevan”. 19) Centrul radical al cercurilor exînscrise unui triunghi este punctul lui Spieker corespunzător acelui triunghi. Demonstraţie. Vezi „Cercurile exînscrise”.

I.9. Punctele lui Brocard13

„Nu există pe lume un stadiu care să pună mai armonios în acţiune facultăţile spiritului decât cel al matematicienilor. Matematicianul trăieşte mult timp şi totuşi rămâne tânăr, aripile sale nu se frâng de timpuriu şi porii săi nu-s obturaţi de praful ce se ridică pe marile drumuri prăfuite de vieţi obişnuite.” – James Sylvester14 1) În orice triunghi ABC există punctele Ω şi 'Ω şi unghiurile ω şi 'ω astfel încât

( ) ( ) ( )m BA m CB m AC ωΩ = Ω = Ω = şi ( ') ( ') ( ') 'm AB m CB m CA ωΩ = Ω = Ω = .

Demonstraţie. Presupunând cunoscut unghiul ω trasăm semidreaptele (AΩ şi (BΩ ,

determinând punctul lor de intersecţie Ω . Fie 1C ,

2C şi

3C cercurile circumscrise

triunghiurilor ,C A A BΩ Ω , respectiv .B CΩ

13 Henri Brocard (1845-1922) – matematician francez, contribuţii importante în geometrie 14 James Sylvester (1814-1897) – matematician englez, profesor universitar la Oxford, contribuţii importante în

algebră

A

B C

Ω

ω

Fig. 52

Page 47: Barbu_TEOREME_FUNDAMENTALE(2008).pdf

54

Dacă BA CB AC ωΩ ≡ Ω ≡ Ω ≡ , atunci cercul 1C este tangent în A la AB , cercul

2C este

tangent în B la BC şi cercul 3C este tangent în C la CA (Fig. 52 ). Ţinând cont şi de faptul

că 1C trece şi prinC ,

2C trece şi prin A ,

3C trece şi prin B, rezultă că cercurile

1C ,

2C şi

3C sunt determinate independent de .ω Fie Ω punctul comun cercurilor

1C şi

2C . Din

Ω∈1C rezultă AB CAΩ ≡ Ω ; din Ω∈

2C rezultă BA BCΩ ≡ Ω , de unde CA BCΩ ≡ Ω ,

adică cercul ce trece prin punctele B ,Ω ,C trecând prin B şi tangent în C laCA (datorită

ultimei congruenţe), coincide cu 3C , deci Ω∈

3C . Analog, considerând cercurile:

1C ′ ce

trece prin A şi este tangent în C la BC, 2C ′ ce trece prin B şi este tangent în A la AC şi

3C ′

ce trece prin C şi este tangent în B la AB, vom determina punctul 'Ω . Observaţii: 1) Punctele Ω şi 'Ω se numesc primul,respectiv al doilea punct al lui Brocard. 2) Cercurile

1C ,

2C ,

3C se numesc cercurile Brocard directe.

3) Cercurile 1C ′ ,

2C ′ ,

3C ′ se numesc cercurile Brocard retrograde.

4) Un cerc ce trece prin două vârfuri ale unui triunghi şi este tangent la una din laturile triunghiului se numeşte cerc adjunct. 5) Fiecare triunghi are şase cercuri adjuncte.

6) Notăm cercul adjunct ce trece prin C şi este tangent în A la AB cu CA . Atunci, cercurile

adjuncteCA , AB , BC trec prin primul punct al lui Brocard (Ω ), iar cercurile BA ,CB , AC trec prin al doilea punct al lui Brocard( 'Ω ).

2) Coordonatele unghiulare ale punctelor lui Brocard sunt (180 ( ),180 ( ),180 ( )m B m C m AΩ − − −o o o ) şi (180 ( ),180 ( ),180 ( )m A m B m C′Ω − − −o o o ).

Demonstraţie:

Avem: ( ) 180 [ ( ( )]m A B m AB m BAΩ = − Ω + Ωo 180 [ ( ) ( ] 180 ( )m BC m BA m B= − Ω + Ω = −o o

şi analog, ( ) 180 ( )m B C m CΩ = −o şi ( ) 180 ( )m C A m AΩ = −o (Fig. 53). Pentru punctul 'Ω

ω

Ω

A

B C

ω 'ω

'ω ω

Fig. 53

Page 48: Barbu_TEOREME_FUNDAMENTALE(2008).pdf

55

avem: ( ' ) 180 [ ( ' ) ( ' )] 180 [ ( ' ) ( ' )] 180 ( )m A B m AB m BA m AB m AC m AΩ = − Ω + Ω = − Ω + Ω = −o o o

şi analog ( ' ) 180 ( )m B C m BΩ = −o , ( ' ) 180 ( )m C A m CΩ = −o .

Observaţie: Din cele de mai sus rezultă că

( ) ( ) ( ), ( ) ( ) ( )m A B m A m C m B C m A m BΩ = + Ω = + şi ( ) ( ) ( )m C A m B m CΩ = + .

3) Dacă Ω este primul punct Brocard al triunghiului ABC, atunci distanţele

, ,A B CΩ Ω Ω sunt proporţionale cu ,b c

a brespectiv

a

c (unde a,b,c sunt lungimile laturilor

BC,CA, respectiv AB).

Demonstraţie: Din teorema sinusurilor în triunghiul A CΩ rezultă: sin sin(180 )

A AC

AωΩ

=°−

,

de unde sin

2 sinsin

bA b R

A a

ωω⋅Ω = ⋅ = . Analog, 2 sin

cB R

bω⋅Ω = şi 2 sin

aC R

cω⋅Ω = , de

unde / / /

A B C

b a c b a c

Ω Ω Ω= = .

4) Consecinţă: Dacă Ω este primul punct Brocard al triunghiului ABC,

atunci2 2 2

2 2 2 2 22 2 2

4 sinb c a

A B C Ra b c

ω

Ω + Ω + Ω = + +

.

Demonstraţia rezultă din teorema precedentă. 5) Dacă Ω este primul punct Brocard al triunghiului ABC,

atunci2 2 2

, , .A b B c C a

B ac C ba A cb

Ω Ω Ω= = =

Ω Ω Ω

Demonstraţia rezultă din proprietatea precedentă. 6) Distanţele de la primul punct Brocard al triunghiului la laturile , , AB BC CA sunt

proporţionale cu numerele ,b c

a b respectiv

a

c.

Demostraţie. Fie x, y şi z distanţele de la Ω la laturile , ,AB BC respectiv CA (Fig. 54).

B

A

C

y

z x

Ω

ω

Fig. 54 B

A

C

y'

z ' x ' 'Ω

Fig. 55

Page 49: Barbu_TEOREME_FUNDAMENTALE(2008).pdf

56

Atunci, 2sin 2 sinb

x A Ra

ω ω⋅= Ω = şi analog 22 sinc

y Rb

ω⋅= , 22 sina

z Rc

ω⋅=

7) Distanţele de la cel de-al doilea punct Brocard 'Ω al triunghiului ABC la vârfurile

, , A B C sunt proporţionale cu , ,c a b

a b c.

Demonstraţie. În triunghiul 'ABΩ (Fig. 55) teorema sinusurilor ne dă: sin '

'sin

A cA

ωΩ = ⋅

adică ' 2 sin 'c

A Ra

ω⋅Ω = . Analog, ' 2 sin 'a

B Rb

ω⋅Ω = şi ' 2 sin 'b

C Rc

ω⋅Ω =

8) Dacă 'Ω este al doilea punct Brocard al triunghiului ABC, atunci

2 2 2

' ' ', , .

' ' '

A bc B ca C ab

B C Aa b c

Ω Ω Ω= = =

Ω Ω Ω

Demonstraţia rezultă din proprietatea precedentă. 9) Distanţele de la al doilea punct Brocard al triunghiului ABC la

laturile , , AB BC CA sunt proporţionale cu ,c a

a b respectiv

b

c.

Demonstraţie. Fie ', ', 'x y z distanţele de la 'Ω la laturile , ,AB BC respectiv CA

(Fig. 55). Atunci, 2' 'sin ' 2 sin 'a

x B Rb

ω ω⋅= Ω = , 2' 2 sin 'b

y Rc

ω⋅= şi 2' 2 sin 'c

z Ra

ω⋅= .

10) Punctele lui Brocard sunt izogonal conjugate. Demonstraţie. Din proprietăţile 4) şi 6) rezultă 2 2 2' ' ' 4 sin sin 'x x y y z z R ω ω⋅⋅ = ⋅ = ⋅ = ⋅ ,

ceea ce arată că punctele Ω şi 'Ω sunt izogonale (vezi „Drepte izogonale”). Observaţii: 1) Din proprietatea precedentă rezultă că 'ω ω= 2) Unghiul ω se numeşte unghiul lui Brocard.

11) Dacă Ω şi 'Ω sunt punctele lui Brocard ale triunghiului ABC atunci ' ' 'A B C A B CΩ⋅ Ω⋅ Ω = Ω ⋅ Ω ⋅ Ω .

Demonstraţie. Din teoremele (6) şi (8) rezultă 3 3' ' ' 8 sinA B C A B C R ωΩ⋅ Ω ⋅ Ω = Ω ⋅ Ω ⋅ Ω = . 12) Triunghiurile podare ale punctelor lui Brocard Ω şi 'Ω ale triunghiului ABC sunt congruente. Demonstraţie. Fie 1 2 3Ω Ω Ω şi ' ' '

1 2 3Ω Ω Ω triunghiurile podare ale punctelor lui Brocard

(Fig. 56). Deoarece patrulaterul 1 2CΩΩ Ω este inscriptibil, din teorema sinusurilor rezultă

1 2 sin sinC C a ωΩ Ω = Ω = . Analog, 2 3 sinb ωΩ Ω = şi 3 1 sinc ωΩ Ω = . Analog, ' '1 2 sin ,b ωΩ Ω = ' '

2 3 sinc ωΩ Ω = şi ' '3 1 sin .a ωΩ Ω = Deci, triunghiurile 1 2 3Ω Ω Ω şi ' ' '

1 2 3Ω Ω Ω

sunt congruente.

Page 50: Barbu_TEOREME_FUNDAMENTALE(2008).pdf

57

13) Triunghiurile podare ale punctelor lui Brocard al triunghiului ABC sunt asemenea cu triunghiul ABC .

Demonstraţie. Din proprietatea (2) avem ( ) ( ) ( )m B C m A m BΩ = + (Fig. 56). Deoarece

patrulaterele 1 3BΩ ΩΩ şi 2 1CΩ ΩΩ sunt inscriptibile rezultă 3 3 1BΩ Ω ≡ Ω Ω Ω (1) şi

2 1 2CΩ Ω ≡ ΩΩ Ω (2). Dar

1 2B CΩ Ω ≡ Ω Ω , deci 3 1 2( )m Ω Ω Ω =

3 1 1 2 3 1( ) ( ) ( ) ( ) ( )m m m B m B m ABCΩ Ω Ω + ΩΩ Ω = Ω Ω + Ω Ω = . Analog,

1 2 3 BCAΩ Ω Ω ≡ ,

deci triunghiurile 1 2 3Ω Ω Ω şi BCA sunt asemenea. Analog se arată că ' ' '1 2 3Ω Ω Ω triunghiul

podar al lui 'Ω este asemenea cu triunghiul .CAB 14) Consecinţă: Dacă 1 2 3Ω Ω Ω şi ' ' '

1 2 3Ω Ω Ω sunt triunghiurile podare ale punctelor lui

Brocard, atunci ' ' '1 2 3 1 2 3

2[ ] [ ][ ]

sin ABCA A AωΩ Ω Ω Ω Ω Ω= = ⋅ .

Demonstraţie. Din asemănarea triunghiurilor 1 2 3Ω Ω Ω şi BCA rezultă

1 2 3

2[ ] 21 2

[ ]

sinABC

A

A BCωΩ Ω Ω Ω Ω

= =

, de unde rezultă concluzia.

15) Triunghiurile podare ale punctelor lui Brocard Ω şi 'Ω ale triunghiului ABC sunt înscrise în acelaşi cerc Tücker având centrul în mijlocul segmentului '.ΩΩ Demonstraţie. Deoarece punctele Ω şi 'Ω sunt izogonale atunci triunghiurile podare -

1 2 3Ω Ω Ω şi ' ' '1 2 3Ω Ω Ω - ale punctelor lui Brocard au vârfurile pe acelaşi cerc cu centrul în

mijlocul segmentului 'ΩΩ (vezi „Drepte izogonale”). Deoarece ' ' '2 1 3( ) ( )m m ACBΩ Ω Ω =

şi ' ' ' ' ' ' '3 1 2 3 2 2 3 2

1( ) ( ) ( )2m m mΩ ΩΩ = Ω Ω Ω = Ω Ω rezultă ' '3 2 2 ACBΩ Ω Ω ≡ , deci '

3 2 .BCΩ Ω

Atunci, 3

2

A AB

A AC

Ω=

Ω sau

'cos( )

cos( )

A A AB

A A AC

ωω

Ω −=

Ω −, deci

'A AB

A AC

Ω=

Ω (1). Deoarece

ω

Ω

A

B C

ω 'ω

'ω ω

Fig. 56

'2Ω

'3Ω

'1Ω

Page 51: Barbu_TEOREME_FUNDAMENTALE(2008).pdf

58

' ' ' ' '2 3 3 3 1 2( ) ( ) ( )m m m ACBΩ Ω Ω = Ω ΩΩ = rezultă că dreapta '

2 3Ω Ω este antiparalelă cu BC , adică '2

3

'cos '

cos

AAB A A

AC A A A

ωω

Ω Ω Ω= = =

Ω Ω Ω, de unde rezultă concluzia.

16) Raza cercului circumscris triunghiurilor podare ale punctelor lui Brocard au lungimea egală cu sinR ω (unde R este raza cercului circumscris triunghiului ABC ). Demonstraţie. Din asemănarea triunghiurilor BCA şi 1 2 3Ω Ω Ω rezultă:

1 2' sinR a

R a a

ωΩ Ω= = , de unde rezultă că ' sin .R R ω=

17) Dacă a b cΩ Ω Ω este triunghiul pedal al primului punct al lui Brocard Ω

corespunzător unui triunghi ABC , ( ), ( ), ( ),a b cBC CA ABΩ ∈ Ω ∈ Ω ∈ atunci 2

a

a

B c

C a

Ω = Ω ,

2

b

b

C a

A b

Ω = Ω şi

2

c

c

A b

B c

Ω = Ω .

Demonstraţie. Teorema sinusurilor aplicată triunghiurilor aABΩ şi aACΩ (Fig. 57) ne dă:

sinsina

a

BAB

A B ωΩ

şi sin(180 ) sin( )

a

a

CAC

A B A ω

Ω=

°− Ω −, de

unde

sin

sin( )a

a

B c

C bA

ω

ω⋅

Ω=

Ω − (1). Teorema sinusurilor

aplicată în triunghiul A BΩ dă: sin sin( )

A C

Aω ω

Ω Ω=

−, de unde

2

sin

sin( )

A bc

A C a

ωω

Ω= =

− Ω (2). Din relaţiile (1) şi (2) obţinem

2

.a

a

B c

C a

Ω = Ω Analog se arată şi relaţiile:

2

b

b

C a

A b

Ω = Ω ,

2

c

c

A b

B c

Ω = Ω .

18) Dacă a b cΩ Ω Ω este triunghiul pedal al primului punct al lui Brocard Ω

corespunzător unui triunghi ABC , atunci 2 3

2 2 2 2, .a a

ac aB C

c a c aΩ = Ω =

+ +

Demonstraţie. Din relaţia 2

a

a

B c

C a

Ω = Ω prin proporţii derivate rezultă concluzia.

19) Dacă ' ' 'a b cΩ Ω Ω este triunghiul pedal al celui de-al doilea punct Brocard, ' ( )

aBCΩ ∈ ,

' ( )b

CAΩ ∈ , ' ( )c

ABΩ ∈ , atunci 2'

'a

a

B a

cC

Ω = Ω ,

2'

'b

b

C b

aA

Ω = Ω ,

2'

'.c

c

A b

cB

Ω = Ω

Demonstraţia este analoagă cu cea din proprietatea (17).

bΩcΩ

C B

A

ω ω

ω

Ω

Fig. 57

Page 52: Barbu_TEOREME_FUNDAMENTALE(2008).pdf

59

20) Triunghiul circumpedal al unui punct al lui Brocard al triunghiului ABC este congruent cu triunghiul ABC. Demonstraţie. Vezi „Triunghiul circumpedal”. 21) Fie a b cK K K triunghiul pedal al punctului lui Lemoine al triunghiului ABC şi

punctele aΩ , ' ( )a

BCΩ ∈ ; bΩ , ' ( )b

CAΩ ∈ ; cΩ , ' ( )c

ABΩ ∈ astfel încât 'a b

K ABΩ ,

a cK ACΩ , b aK BAΩ , 'b c

K BCΩ , c bK BCΩ şi ' .c a

K ACΩ Dreptele

aAΩ , bBΩ , cCΩ sunt concurente în primul punct Brocard Ω , iar 'a

AΩ , 'b

BΩ , 'c

CΩ sunt concurente în cel de-al doilea punct Brocard 'Ω .

Demonstraţie. Din teorema lui Steiner avem: 2

a

a

BK c

K C b

=

şi

analoagele (Fig. 58). Din teorema lui Thales avem 2

a c

a c

BK B c

K C A b= =

Ω Ω

şi analoagele. Din reciproca teoremei lui

Ceva rezultă: 2 2 2

1a b c

a b c

B C A c a b

C A B a b c⋅ ⋅

Ω Ω Ω = ⋅ ⋅ = Ω Ω Ω , deci

dreptele aAΩ , bBΩ , cCΩ sunt concurente. Dacă

a A BCω = Ω∩ , atunci - conform aplicaţiei precedente - 2

a a

a a

B Bc

C a C

ωω

Ω = = Ω şi de aici rezultă

a a a a

a a

B C B CC C

ω ωω+ Ω +Ω= Ω , adică

a a

BC BCC Cω = Ω , de unde

a aC Cω = Ω , ceea ce arată că

.a a

ω = Ω Atunci, punctul de concurenţă al dreptelor a

AΩ ,b

BΩ ,c

CΩ este Ω . Analog se

arată că dreptele 'aAΩ , '

bBΩ , 'cCΩ sunt concurente în cel de-al doilea punct Brocard.

22) Dacă ω este unghiul lui Brocard al triunghiului ABC, atunci 2 2 2

[ ]4ABC

a b cctg

+ += .

Demonstraţie. Teorema sinusurilor aplicată în triunghiurile A BΩ şi A CΩ ne dă:

sin sin( )sin

cB A

BBω ωΩ Ω

= =−

şi sin sin(180 )

A b

AωΩ

=°−

de unde sin( )

sin

A B

B

ωω

Ω −=

Ωsin

sin

b B

c A= ;

din relaţia precedentă, dezvoltând sin( )B ω− , obţinem

sin

sin sin

Bctg ctgB ctgA ctgC

A Bω − = = +

⋅ şi ţinănd seama de egalitatea

2 2 2

[ ]4 ABC

a b cctgA ctgB ctgC

A

+ ++ + = rezultă concluzia.

Observaţie: Din cele de mai sus rezultă: .ctg ctgA ctgB ctgCω = + +

23) Dacă ω este unghiul lui Brocard al triunghiului ABC, atunci :

2 2 2

sin( ) sin( ) sin( ) sin.

A B C

abca b c

ω ω ω ω− − −= = =

'bΩ

C B

A

Fig. 58

Page 53: Barbu_TEOREME_FUNDAMENTALE(2008).pdf

60

Demonstraţie. Avem: 2 3sin( ) ( / )2 sin

sin ( / )2 sin

A C a c R a a

A b a R bc abc

ω ωω ω− Ω

= = = =Ω

şi relaţiile analoage.

24) Dacă ω este unghiul lui Brocard al triunghiului ABC, atunci : sin( )

,sin

A b c

c b

ωω+

= +

sin( ),

sin

B c a

a c

ωω+

= + sin( )

.sin

C a b

b a

ωω+

= +

Demonstraţie. Avem sin ( ) sin cos sin cos

sin s in

A A Aω ω ωω ω+ +

= = sin cosActg Aω + =

2 2 2 2 2 2 2 2 2 2 2 2

[ ]

.2 4 2 2 2

ABC

a a b c b c a a b c b c a b c

R A bc bc bc c b

+ + + − + + + −⋅ + = + = +

25) Unghiul lui Brocard ω are măsura mai mică sau egală cu 30° , egalitatea având loc pentru un triunghi echilateral.

Demonstraţie. Inegalitatea ( ) 30m ω ≤ ° este echivalentă cu 3ctgω ≥ sau 2 2 2

[ ]

34 ABC

a b c

A

+ +≥

⋅ de unde 2 2 2

[ ]4 3 ABCa b c A+ + ≥ şi de aici rezultă

2 2 2( ) 4 3 ( )( )( )a b c p p a p b p c+ + ≥ − − − , p fiind semiperimetrul triunghiului ABC.

Ridicând la pătrat inegalitatea precedentă obţinem: 4 4 4 2 2 2 2 2 2a b c a b b c c a+ + ≥ + + , adică

inegalitatea evidentă 2 2 2 2 2 2 2 2 2( ) ( ) ( ) 0a b b c c a− + − + − ≥ , cu egalitate pentru 2 2 2a b c= = ,adică a b c= = ,deci când triunghiul ABC este echilateral

26) Consecinţă: În orice triunghi 1

sin .2

ω ≤

27) În orice triunghi ABC, [ ]2 2 2 4' ' ' ,

3ABC

A B C AΩ + Ω + Ω ≥ ⋅ (unde [ ]ABCA reprezintă aria

triunghiului ABC). Demonstraţie. Avem: [ ] [ ' ] [ ' ] [ ' ]Ω Ω Ω= + +

ABC A B A B A BA A A A relaţie echivalentă cu :

[ ]2 ' sin ' sin ' sin= ⋅Ω ⋅ + ⋅Ω ⋅ + ⋅Ω ⋅ABC

A c A a B b Cω ω ω ,deci2[ ] 22

4( ' ' ' )

sin= Ω + Ω + ΩABCAc A a B b C

ω

(1), iar din inegalitatea lui Cauchy – Bouniakowski - Schwarz rezultă 2 2 2 2 2 2 2( ' ' ' ) ( )( ' ' ' )Ω + Ω + Ω ≤ + + Ω +Ω +Ωc A a B b C a b c A B C (2) . Atunci, din relaţiile

(1) şi (2) rezultă [ ]2

2 2 2 2 2 22

4( )( ' ' ' )

sin< + + Ω +Ω +Ω

ABCA

a b c A B Cω

(3). Aplicând teorema

cosinusului în triunghiurile ' , 'A B B CΩ Ω respectiv 'ΩC A rezultă : 2 2 2' ' 2 ' cos ;B A c c A ωΩ = Ω + − ⋅ Ω 2 2 2' ' 2 ' cos ;A C b b C ωΩ = Ω + − ⋅ Ω

2 2 2' ' 2 ' cosΩ =Ω + − ⋅ΩC B a a B ω (4). Adunând relaţiile (4) şi ridicând la pătrat relaţia

obţinută, se obţine: 2 2 2 2

22

( )( ' ' ' )

4cos

+ += Ω + Ω + Ω

a b cc A b C a B

ω şi conform relaţiei (2)

Page 54: Barbu_TEOREME_FUNDAMENTALE(2008).pdf

61

rezultă 2 2 2 2

2 2 2 2 2 22

( )( )( ' ' ' )

4cos

+ +≤ + + Ω +Ω +Ω

a b ca b c A B C

ω (5). Din inegalităţile (3) şi

(5) prin înmulţire se obţine [ ]2

2 2 2 22

4( ' ' ' )

sin (2 )≤ Ω +Ω +Ω

ABCA

A B Cω

(6). Conform observaţiei

precedente rezultă 0 23

< ≤π

ω , deci 2 3sin (2 )

4≤ω (7). Din relaţiile (6) şi (7) rezultă

[ ]2 2 2 4' ' '

3Ω + Ω + Ω ≥

ABCA B C A , cu egalitate pentru triunghiul echilateral.

28) În orice triunghi ABC este adevărată relaţia:

2 2 22 2 2 2

2 2 2' ' ' .

b a cA B C R

a c b

Ω + Ω + Ω ≤ + +

Demonstraţie. Conform proprietăţii (7) avem: ' 2 sin ,Ω =b

A Ra

ω ' 2 sinΩ =c

B Rb

ω şi

' 2 sinΩ =a

C Rc

ω . Atunci: 2 2 2

2 2 2 2 22 2 2

' ' ' 4 sin

Ω + Ω + Ω = ⋅ ⋅ + +

b a cA B C R

a c bω şi utilizând

proprietatea 1

sin2

ω ≤

rezultă 2 2 2

2 2 2 22 2 2

' ' '

Ω + Ω + Ω ≤ + +

b a cA B C R

a c b (cu egalitate

când triunghiul ABC este echilateral).

29) În orice triunghi ABC este adevărată relaţia: 2 2 2

[ ] 22 2 2

4 3.

3

≤ + +

ABCA b a cR

a c b

Demonstraţia rezultă din proprietăţile precedente. 30) Într-un triunghi isoscel, mediana şi simediana unghiurilor congruente se intersectează într-un punct al lui Brocard. Demonstraţie. Fie triunghiul ABC ( AB AC≡ ), 'BB simediana unghiului B şi 'CC mediana ce pleacă din C . Fie ' 'BB CCΩ = ∩ şi 'A A BC= Ω∩ (Fig. 59). Deoarece

'CC este mediană, 'BB simediana şi ABC ACB≡ , rezultă CA BCΩ ≡ Ω şi BA CBΩ ≡ Ω . Din teorema lui Ceva, aplicată

în triunghiul ABC rezultă ' ' '

1' ' '

A B B C C A

A C B A C B⋅ ⋅ = , de unde

' '

' '

A B B A

A C B C= şi din reciproca teoremei lui Thales rezultă

' 'A B AB . Atunci, ' ' 'B A A BAA≡ şi ' ' 'ABB BB A≡ , deci

' ' 'B A CAΩ ≡ Ω , adică patrulaterul ' 'B CAΩ este inscriptibil.

Atunci ' 'A B CAΩ ≡ Ω , de unde AC AB BCΩ ≡ Ω ≡ Ω , adică Ω este un punct Brocard al triunghiului ABC . Observaţie: Analog se arată că mediana unghiului B şi simediana unghiului C se intersectează în celălalt punct Brocard.

A '

B'C'

C B

A

Ω

Fig. 59

Page 55: Barbu_TEOREME_FUNDAMENTALE(2008).pdf

62

31) Dacă mediana şi simediana a doua unghiuri ale unui triunghi se intersectează într-unul din punctele lui Brocard ale triunghiului, atunci triunghiul este isoscel.

Demonstraţie. Fie Ω primul punct al lui Brocard, deci AB BC CAΩ ≡ Ω ≡ Ω . Fie 'BB simediană şi 'CC mediană, iar 'A A BC= Ω∩ . Din teorema lui Ceva rezultă ' 'A B AB ,

deci ' ' 'BAA AA B≡ şi ' ' 'ABB BB A≡ . Atunci, ' ' 'A B CBΩ ≡ Ω , deci patrulaterul ' 'A CBΩ

este inscriptibil de unde .' ' ' 'CA B A B BAΩ ≡ Ω ≡ . Astfel, ( ) ( ') ( ' )m B m ABB m B BC= + =

( ' ) ( ' ) ( )m C CB m C CA m C+ = , deci triunghiul ABC este isoscel.

32) Triunghiul 'OΩΩ este isoscel, unde O este centrul cercului circumscris triunghiului ABC , iar Ω şi 'Ω punctele lui Brocard ale acestuia. Demonstraţie. Prin rotaţie de centru Ω şi unghi 90 ω° − (în sens negativ) triunghiul 1 2 3Ω Ω Ω se

transformă în triunghiul ' ' 'B A C ( ' , ' , 'B B A A C C∈ Ω ∈ Ω ∈ Ω ). Evident, triunghiurile

1 2 3Ω Ω Ω şi ' ' 'B A C sunt congruente (Fig. 60). Deoarece

' ' 'sin

B C

B B Cω

Ω ΩΩ Ω= = =

Ω Ω Ω rezultă ' 'B C BC . Analog,

' 'A C AC şi ' 'A B AB . Centrul cercului circumscris

( 1O ) al triunghiului 1 2 3Ω Ω Ω se transformă prin rotaţia

dată în '1O - centrul cercului circumscris triunghiului

' ' 'A B C , deci '11( ) 90 .m O O ωΩ = °− Deoarece triunghiurile ' ' 'A B C şi ABC sunt

omotetice, rezultă că '1 ( ,O O∈ Ω deci

1( ) 90 .m O O ωΩ = °− . Printr-o rotaţie de centru 'Ω şi

unghi 90 ω° − în sens trigonometric se arată, procedând ca mai sus, că 1( ' ) 90 ,m O O ωΩ = °− deci triunghiul 'OΩΩ este isoscel.

Observaţie: Din proprietatea precedentă rezultă ( ') 180 2(90 )m O ωΩ Ω = °− °− , adică

( ') 2m O ωΩ Ω = .

33) Dacă Ω şi 'Ω sunt punctele lui Brocard ale unui triunghi ABC, atunci

2' 1 4sinO O R ωΩ = Ω = − . Demonstraţie.

Avem 1 2 3[ ] 1 2 1 3 2 1 32 sinAΩΩ Ω⋅ =ΩΩ ⋅ΩΩ ⋅ Ω ΩΩ =

2 1 3( sin ) ( sin ) sinC C B BΩ ⋅ Ω ⋅ ⋅ Ω Ω Ω (1),

2 1 3 2 1 1 3 1 2( ) ( ) ( ) ( ) ( )m m m m ABB m CΩΩΩ = ΩΩΩ + ΩΩΩ = + Ω Ω =

1 2 1( ) ( ) ( )m ACB m C m CB+ Ω Ω = Ω ,

1( ) ( )m BC m AΩ =

teorema sinusurilor în triunghiul 1CBΩ dă

2 1 3 1sin sinC B AΩ Ω Ω Ω = Ω (2) (Fig. 61). Din

relaţiile (1) şi (2) şi ţinând cont de puterea punctului Ω faţă de cercul circumscris

C B

A

ω ω

ω

Ω

Fig. 60

A'

B' C'

A

B C

Fig. 61

3Ω Ω

1B

Page 56: Barbu_TEOREME_FUNDAMENTALE(2008).pdf

63

triunghiului ABC rezultă 1 2 3[ ] 12 s in s in s inA B B A B CΩ Ω Ω⋅ = Ω ⋅ Ω ⋅ ⋅ ⋅ =

2 2( ) sin sin sinR O A B C−Ω ⋅ ⋅ ⋅ (3). Conform teoremei (14) avem

1 2 3

2[ ] [ ] s inABCA A ωΩ Ω Ω = ⋅ (4) şi 2

[ ] 2 sin sin sinABCA R A B C= ⋅ ⋅ ⋅ (5), din relaţiile (3),

(4) şi (5) rezultă 2 2 2(1 4sin )O R ωΩ = − , de unde 21 4sinO R ωΩ = − şi deoarece

'O OΩ ≡ Ω obţinem concluzia. 34) Dacă Ω şi 'Ω sunt punctele lui Brocard ale unui

triunghi ABC, atunci 2' 2 sin 1 4sinR ω ωΩΩ = − . Demonstraţie. Deoarece triunghiul 'OΩΩ este isoscel (Fig.

62) şi ( ') 2m O ωΩ Ω = avem sinOT

Oω =

Ω (unde T este

mijlocul segmentului 'ΩΩ ) şi ţinând cont că 21 4sinO R ωΩ = − rezultă ' 2 2 sinOT OωΩΩ = ⋅ = ⋅ ⋅ Ω = 22 sin 1 4sinR ω ω− .

35) Fie Ω şi 'Ω punctele lui Brocard,G centrul de greutate şi K punctul lui Lemoine al triunghiului ABC . Tripletele de drepte ( , , )A BK CGΩ , ( , , )B CK AGΩ ,

( , , )C AK BGΩ , ( ', , )A CK BGΩ , ( ', , )B AK CGΩ ,

( ', , )C BK AGΩ sunt concurente.

Demonstraţie.Fie , a bA BC K BK ACΩ = Ω∩ = ∩ şi

cM CG AB= ∩ .Avem:2 2

1 1a b c

a b c

B K C M A c a

C K A M B a c⋅ ⋅ ⋅ ⋅

Ω = = Ω

şi din reciproca lui Ceva rezultă că dreptele ,A BKΩ şi CG sunt concurente (Fig. 63). Analog, se arată concurenţa celorlalte triplete de drepte. Observaţie: Dacă P este punct de concurenţă al dreptelor ,A BKΩ ,CG atunci punctul de concurenţă al dreptelor 'AΩ ,CK , BG este izogonalul punctului P .

36) Fie Ω primul punct al lui Brocard al unui triunghi ABC şi 1 2 3, ,O O O centrele cercurilor circumscrise

triunghiurilor , ,AC AB CBΩ Ω Ω . Triunghiurile ABC şi

1 2 3OO O sunt asemenea.

Demonstraţie. Avem: 2 1 3( ) 180 ( )m O OO m A C= °− Ω =

( ),m A 1 2 3( ) 180 ( ) ( )m OO O m A B m B= °− Ω = , deci

triunghiurile 1 2 3OO O şi ABC sunt asemenea (Fig. 64).

bK

C B

A

P

Fig. 63

cM

T

O

Fig. 62

Ω 'Ω

3O

2O1O

C B

A

ω ω

ω

Ω

Fig. 64

O

Page 57: Barbu_TEOREME_FUNDAMENTALE(2008).pdf

64

37) Fie 'Ω al doilea punct al lui Brocard şi ' ' '1 2 3, ,O O O centrele cercurilor circumscrise

triunghiurilor ' , ' ,AB BCΩ Ω respectiv 'CAΩ . Triunghiurile ABC şi 1 2 3OO O sunt

asemenea. Demonstraţie analoagă cu precedenta. 38) Fie Ω primul punct al lui Brocard al triunghiului ABC şi 1 2 3, ,O O O centrele

cercurilor circumscrise triunghiurilor , ,AC AB CBΩ Ω Ω . Centrul cercului cercului circumscris triunghiului ABC este primul punct al lui Brocard al triunghiului 1 2 3OO O .

Demonstraţie. Dreptele 1 2 3, ,OO OO OO sunt mediatoarele segmentelor AC, AB, respectiv

BC. Atunci, 1 3( ) ( )m OOO m CA ω= Ω = fiind unghiuri cu laturile perpendiculare două

câte două. Analog, 1 3 2 1 3 2( ) ( ) ( )m OOO m OO O m OO O ω= = = , de unde rezultă

concluzia. 39) Fie 'Ω al doilea punct al lui Brocard al triunghiului ABC şi ' ' '

1 2 3, ,O O O centrele

cercurilor circumscrise triunghiurilor ' , ' ,AB BCΩ Ω respectiv 'CAΩ . Centrul cercului cercului circumscris triunghiului ABC este al doilea punct al lui Brocard al triunghiului ' ' '

1 2 3OO O . Demonstraţie analoagă cu precedenta. 40) Fie Ω primul punct al lui Brocard al unui triunghi ABC şi 1 2 3, ,R R R razele

cercurilor circumscrise triunghiurilor , ,CA ABΩ Ω respectiv BCΩ . Atunci, 31 2 3R R R R= ,

unde R este raza cercului cercului circumscris triunghiului ABC .

Demonstraţie. Din teorema sinusurilor rezultă 1 ,

2sin( ) 2sin2sin

AC AC ACR

A AA C π= = =

−Ω

2 2sin2sin

BA ABR

BA B= =

Ω şi 3 ,

2sin

BCR

C= de unde obţinem

31 2 3

1.

8 sin sin sin 2sin 2sin 2sin

abc a b cR R R R

A B C A B C= ⋅ = ⋅ ⋅ =

41) Fie Ω şi 'Ω punctele lui Brocard al triunghiului ABC , iar 1 ',A B C= Ω∩ Ω 1 ',B C A= Ω∩ Ω

1 'C A B= Ω∩ Ω . Punctele 1 1 1, , , , 'A B C Ω Ω sunt conciclice. Demonstraţie. Deoarece ( ) ( ' )m AB m BA ωΩ = Ω = ,

rezultă că 1( ) 180 2 .m AC B ω= °− Dar

1 1( ' ) 180 ( ) 2m A m BAC ωΩ Ω = °− = , deci patrulaterul

1 1, ', ,C AΩ Ω este inscriptibil. Analog se arată că

patrulaterul 1 1 'B CΩ Ω este inscriptibil, deci punctele

1 1 1, , , , 'A B C Ω Ω sunt conciclice (Fig. 65).

1C

1B

1A

C B

A

ω ω

ω

Ω

Fig. 65

Page 58: Barbu_TEOREME_FUNDAMENTALE(2008).pdf

65

42) Fie , ,a b c lungimile laturilor triunghiului ABC . Numerele 222 ,, cba sunt în progresie aritmetică dacă şi numai dacă ctgBctg 3=ω .

Demonstraţie. Relaţia 3ctg ctgBω = este echivalentă cu 2ctgA ctgC ctgB+ = (1). Ţinând

cont că 2 2 2cos ( )

sin

A R b c actgA

A abc

+ −= = şi de relaţiile analoage, relaţia (1) devine

2 2 22b a c= + , adică numerele 222 ,, cba sunt în progresie aritmetică.

I.10. Punctele lui Nobbs. Dreapta lui Gergonne

„Imaginaţia este mai importantă decât cunoaşterea; cunoaşterea este limitată pe când imaginaţia îmbrăţişează întreaga lume.” – A. Einstein15

Teorema lui Nobbs

Fie , ,a b cC C C punctele de contact ale cercului înscris cu laturile BC, AC, respectiv AB

ale triunghiului ABC. Dacă A b cN BC C C= I , B c aN CA C C= I , C a bN AB C C= I ,

atunci punctele , ,A B CN N N sunt coliniare.

Demonstraţie.

Teorema lui Menelaus aplicată în triunghiul ABC cu transversalele NA – Cb – Cc ,

NB – Cc – Ca şi NC – Cb – Ca ne dă: 1b aA

A b c

C C C AN B

N C C A C B⋅ ⋅ = , 1a cB

B a c

C B C AN C

N A C C C B⋅ ⋅ = ,

1.C a b

C a b

N A C B C C

N B C C C A⋅ ⋅ = Prin înmulţirea relaţiilor precedente şi ţininând cont de faptul că

1a b c

a b c

C B C C C A

C C C A C B⋅ ⋅ = , va rezulta că 1CA B

A B C

N AN B N C

N C N A N B⋅ ⋅ = , adică punctele , ,A B CN N N sunt

coliniare (Fig. 66).

15 Albert Einstein (1879-1955) – fizician german, profesor universitar la Berlin şi Princeton, laureat al Premiului

Nobel

A

C C AN

BN

CN

aC

bC cC

Fig. 66

Page 59: Barbu_TEOREME_FUNDAMENTALE(2008).pdf

66

Observaţii:

1) Punctele , ,A B CN N N se numesc punctele lui Nobbs. 2) Dreapta ce conţine punctele lui Noobs se numeşte dreapta lui Nobbs ( sau dreapta lui Gergonne). 1) Consecinţă: Dreapta Gergonne este axa de omologie dintre un triunghi ABC şi triunghiul său de contact.

2) Afixele punctelor lui Nobbs sunt egale cu: ( ) ( )

A

B C

N

p c z p b zz

a

− + −= ,

( ) ( )B

A C

N

p c z p a zz

b

− + −= ,

( ) ( ).

C

B A

N

p a z p b zz

c

− + −=

Demonstraţie. Din b a aA

A b c b

C A C B C BN B p b

N C C C C A C C p c

−= ⋅ = =

− rezultă

( ) ( )A

B CN

p c z p b zz

a

− + −= .

Analog se obţin afixele celorlalte două puncte Nobbs. 3) Consecinţă: Coordonatele baricentrice ale punctelor Nobbs sunt:

0, , ,A

p c p bN

a a

− −

,0, ,B

p c p aN

b b

− −

, ,0 .C

p b p aN

a a

− −

4) Dreapta lui Gergonne este perpendiculară pe dreapta lui Soddy. Demonstraţie. Deoarece punctul lui Gergonne şi centrul cercului înscris în triunghiul ABC aparţin dreptei Soddy, ecuaţia dreptei lui Soddy în coordonate baricentrice este:

1 1 10

x y z

p a p b p c

a b c

=− − −

sau 2 2 2( ) : ( ) ( ) ( ) ( ) ( ) ( ) 0I p a c b x p b a c y p c b a zΓ − − + − − + − − = .

Ecuaţia dreptei lui Gergonne în coordonate baricentrice este 0 0

0

x y z

p c p b

p c p a

− − =

− −

, sau

2( )( ) ( )( ) ( ) 0x p a p c y p b p c z p c− − + − − − − = . Utilizând teorema: „Dreptele

1 1 1 1 2 2 2 2( ) : 0, ( ) : 0d a x b y c z d a x b y c z+ + = + + = , având ecuaţiile scrise în coordonate

baricentrice, sunt perpendiculare dacă şi numai dacă

[ ] [ ]2 21 1 2 2 1 1 2 2 1 1 2 2 1 1 2 2( )( ) ( )( ) ( )( ) ( )( )c a a b a b c a a b c a b a b a c b− − + − − + − − + − − +

[ ] 21 1 2 2 1 1 2 2( )( ) ( )( ) 0b c c a c a b c c− − + − − = ” (vezi [16]), pentru dreptele Soddy şi Gergonne

rezultă concluzia. Observaţie: Punctul de intersecţie dintre dreptele Gergonne şi Soddy se numeşte punctul lui Fletcher.

Page 60: Barbu_TEOREME_FUNDAMENTALE(2008).pdf

67

I.11. Punctele lui Soddy16. Punctele lui Eppstein17

„A rezolva o problemă înseamnă a găsi o ieşire dintr-o dificultate, înseamnă a găsi o cale de a ocoli un obstacol, de a atinge un obiectiv care nu este direct accesibil. A găsi soluţia unei probleme este o performanţă specifică inteligenţei, iar inteligenţa este apanajul distinctiv al speciei umane; se poate spune că, dintre toate îndeletnicirile omeneşti, cea de rezolvare a problemelor este cea mai caracteristică.” – George Polya18

Fie triunghiul ABC având lungimile laturilor a, b, respectiv c şi a b cC C C triunghiul de

contact al triunghiului ABC. Considerăm cercurile C1 1( , )A r , C2 2( , )B r , C3 3( , )C r conţinând

punctele cC şi bC , aC şi cC , respectiv bC şi aC .

1) Cercurile C1 , C2 , C3 sunt tangente două câte două. Demonstraţie. Vezi „Teorema lui Descartes”. 2) Razele cercurilor C1 , C2 , C3 sunt egale cu , ,p a p b− − respectiv .p c− Demonstraţie. Vezi „Teorema lui Descartes”.

Observaţii: 1) Cercul tangent exterior cercurilor C1 , C2 , C3 aflat în interiorul triunghiului ABC se numeşte cercul interior Soddy (Fig. 67). Centrul cercului interior Soddy îl vom nota cu iS .

2) Cercul tangent exterior cercurilor C1 , C2 , C3 astfel încât triunghiului ABC este situat în interiorul său se numeşte cercul exterior Soddy (Fig. 70). Centrul cercului exterior Soddy îl vom nota cu eS .

16Frederick Soddy (1877-1956) – chimist, laureat al Premiului Nobel în anul 1921 17David Eppstein (1871-1939) – matematician american, profesor la Universitatea din California 18George Polya (1887-1985) – matematician ungur, profesor la Universitatea Stanford, contribuţii în teoria

grafurilor

A B

C

p a−

p b−

p c−

Fig. 67

aC

bC

cC

aP bP

cP

iE

Page 61: Barbu_TEOREME_FUNDAMENTALE(2008).pdf

68

3) Dreapta i eS S se numeşte dreapta lui Soddy. Punctele iS şi eS se numesc punctele lui

Soddy. 3) Razele cercurilor Soddy au lungimile egale cu:

1 2 3,

1 2 2 3 3 1 1 2 3 1 2 32 ( )i e

r r rR

r r r r r r r r r r r r=

+ + ± + + (semnul „ + ” se ia pentru raza cercului

interior şi semnul „ − ” se ia pentru raza cercului exterior). Demonstraţie. Din formula lui Descartes 2 2 2 2 2

1 2 3 4 1 2 3 42( ) ( )ε ε ε ε ε ε ε ε+ + + = + + + unde

1 2 31 2 3

1 1 1, ,

r r rε ε ε= = = , iar 4

1

iRε = (pentru cercul Soddy interior) şi 4

1

eRε = − (pentru

cercul Soddy exterior), rezolvând ecuaţia de gradul al doilea ce se obţine, va rezulta concluzia. 4) Consecinţă: Razele cercurilor Soddy au lungimile egale cu:

1 2 3,

1 2 2 3 3 1 [ ]2i e

ABC

r r rR

r r r r r r A=

+ + ± (semnul „ + ” se ia pentru raza cercului interior şi semnul

„ − ” se ia pentru raza cercului exterior).

Demonstraţie. Deoarece [ ] 1 2 3 1 2 3( )( )( ) ( )ABCA p p a p b p c r r r r r r= − − − = + + rezultă

concluzia.

5) Consecinţă: Razele cercurilor Soddy au lungimile egale cu: [ ], 4 2

ABC

i e

AR

R r p=

+ ±

(semnul „ + ” se ia pentru raza cercului interior şi semnul „ − ” se ia pentru raza cercului exterior). Demonstraţie.

[ ] [ ] [ ], 2

1 2 3[ ] [ ] [ ] [ ]

1 2 3 1 2 3

2( ) 4 2ABC ABC ABC

i e

a b cABC ABC ABC ABC

A A AR

r r r r r r R r pA A A A

r r r r r r

= = =+ + + + + + ±

+ + ±

.

6) Distanţele de la centrul Soddy interior la laturile BC, CA, AB ale triunghiului ABC

sunt egale cu: [ ] [ ]2 1 ,2 1 ,( ) ( )

ABC ABC

i i

A AR R

a p a b p b

+ +

− − respectiv [ ]2 1 .

( )ABC

i

AR

c p c

+

Demonstraţie. Paralela dusă prin punctul A la BC intersectează cercul C1 în punctele 2R şi

3R (Fig. 68). Deoarece triunghiurile 3 bAR C şi a bCC C sunt isoscele şi 3 b a bR AC C CC≡

(unghiuri alterne interne) rezultă că 3 b a bR C A C C C≡ , deci punctele 3 , ,b aR C C sunt

coliniare; analog punctele 2 , ,c aR C C sunt coliniare. Fie inversiunea T de centru aC şi

putere 3a b ak C C C R= ⋅ ( prin această inversiune cercul C1 se transformă în el însuşi). Prin

această inversiune cercurile C2 şi C3 se transformă în două drepte perpendiculare pe BC şi

Page 62: Barbu_TEOREME_FUNDAMENTALE(2008).pdf

69

tangente în punctele 2R şi 3R la

cercul C1 . Fie a cX BI C C= ∩ şi

2 3 aR C I R R= ∩ . Deoarece

3( ) ( ) 90m BXC m R RC= = ° rezultă

că patrulaterul 3XIRR este

inscriptibil. Atunci,

3 32a b a a ak C C C R C X C R= ⋅ = ⋅ =

2 2a a aC I C R rh⋅ = . Prin inversiunea

de centru aC şi putere 2 ak rh=

cercul lui Soddy interior se transformă într-un cerc C’ congruent cu C1 şi tangent acestuia. Centrul M

al acestui cerc C’ se află pe înălţimea aAH . Fie ad distanţa de la iS la dreapta BC. Atunci,

din asemănarea cercului Soddy interior cu cercul C’ rezultă 1

a i

a

d R

MH r= , de unde obţinem

că : [ ]1

1 1

(2 )2 1 2 1

2 ( )ABCi a a

a i i

AR r h hd R R

r r a p a

+= = + = +

− . Analog se arată că distanţele de la

A B

C

p a−

p b−

p c− Fig. 69

aC bC

cC

aP bP

cP

iE

aH

M

1Q

2Q

3Q

1R

2R

3R

aC

bC

cC

A

B C

I X

3R R 2R

Fig. 68

Page 63: Barbu_TEOREME_FUNDAMENTALE(2008).pdf

70

punctul iS la dreptele CA şi AB sunt egale cu [ ]2 1 ,( )

ABC

b i

Ad R

b p b

= +

− respectiv

[ ]2 1 .( )

ABC

c i

Ad R

c p c

= +

7) Coordonatele baricentrice absolute ale punctului Soddy interior sunt

[ ] [ ] [ ]

[ ] [ ] [ ]

, , .

ABC ABC ABC

i

ABC ABC ABC

i i i

A A Aa b c

p a p b p cS

A A A

R R R

+ + +

− − −

Demonstraţie. Coordonatele baricentrice relative ale punctului Soddy interior sunt

[ ] [ ] [ ]( , , ) , ,ABC ABC ABC

i a b c i

A A AS ad bd cd S a b c

p a p b p c

= + + +

− − − şi ţinând seama de proprietatea (5)

avem: [ ] [ ] [ ] [ ]4 2ABC ABC ABC ABC

i

A A A Aa b c R r p

p a p b p c R+ + + + + = + + =

− − − şi de aici rezultă concluzia.

8) Consecinţă: Pentru orice punct M din planul unui triunghi ABC este adevărată

relaţia:

[ ] [ ] [ ]

[ ] [ ] [ ]

.

ABC ABC ABC

i

ABC ABC ABC

i i i

A A Aa b c

p a p b p cMS MA MB MC

A A A

R R R

+ + +− − −

= + +uuuur uuur uuur uuuur

Demonstraţia rezultă din teorema precedentă. 9) Afixul punctului Soddy interior corespunzător unui triunghi ABC este egal cu

[ ] [ ] [ ]

[ ] [ ] [ ]

.i

ABC ABC ABC

S A B CABC ABC ABC

i i i

A A Aa b c

p a p b p cz z z z

A A A

R R R

+ + +− − −

= + +

Demonstraţia rezultă din teorema precedentă. 10) Dacă iS , I, Γ sunt punctele Soddy interior, centrul cercului înscris, respectiv punctul lui Gergonne corespunzătoare unui triunghi ABC, atunci pentru orice punct M din planul unui triunghi ABC este adevărată relaţia:

[ ]

( ) ( )a b c

i

ABC

i

a b c MI r r r MMS

A

R

+ + + + + Γ=

uuur uuuuruuuur

.

Page 64: Barbu_TEOREME_FUNDAMENTALE(2008).pdf

71

Demonstraţie. Avem:

[ ] [ ] [ ]

[ ] [ ] [ ]

ABC ABC ABC

i

ABC ABC ABC

i i i

A A Aa b c

p a p b p cM S MA MB MC

A A A

R R R

+ + +− − −

= + + =uuuur uuur uuur uuuur

[ ] [ ]

[ ] [ ]

1 1 1 1 1 1( ) ( )

ABC ABC

ABC ABC

i i

aMA bMB cMC A MA MB MC a b c MI A Mp a p b p b p a p b p c

A A

R R

+ + + + + + + + + + Γ − − − − − − =

uuur uuur uuuur uuur uuur uuuur uuur uuuur

sau [ ]

( ) ( )a b c

i

ABC

i

a b c MI r r r MMS

A

R

+ + + + + Γ=

uuur uuuuruuuur

(unde am utilizat relaţiile:

( )aMA bMB cMC a b c MI+ + = + +uuur uuur uuuur uuur

şi 1 1 1 1

M MA MB MCs p a p b p c

Γ= + + − − −

uuuur uuur uuur uuuur cu

1 1 1s

p a p b p c= + +

− − −).

11) Punctele Soddy interior, centrul cercului înscris, respectiv punctul lui Gergonne

corespunzătoare unui triunghi ABC, sunt coliniare şi 4

.2

i

i

S I R r

S p

+=

Γ

Demonstraţie. Deoarece [ ]

( ) ( )a b c

i

ABC

i

a b c MI r r r MMS

A

R

+ + + + + Γ=

uuur uuuuruuuur

, rezultă că punctele I,iS ,Γ

sunt coliniare, în această ordine, iar punctul iS împarte segmentul IΓ în raportul

4

2a b cr r r R r

a b c p

+ + +=

+ +.

12) Distanţele de la centrul Soddy exterior la laturile BC, CA, AB ale triunghiului ABC

sunt egale cu: [ ] [ ]2 1 , 2 1 ,( ) ( )

ABC ABC

e e

A AR R

a p a b p b

− − − −

− − respectiv [ ]2 1 .

( )ABC

e

AR

c p c

− −

Demonstraţie analoagă cu cea din teorema (6).

13) Coordonatele baricentrice absolute ale punctului Soddy exterior sunt

[ ] [ ] [ ]

[ ] [ ] [ ]

, , .

ABC ABC ABC

e

ABC ABC ABC

e e e

A A Aa b c

p a p b p cS

A A A

R R R

− − −

− − −

Demonstraţie analoagă cu cea din teorema (7).

Page 65: Barbu_TEOREME_FUNDAMENTALE(2008).pdf

72

14) Consecinţă: Pentru orice punct M din planul unui triunghi ABC este adevărată

relaţia:

[ ] [ ] [ ]

[ ] [ ] [ ]

.

ABC ABC ABC

i

ABC ABC ABC

i i i

A A Aa b c

p a p b p cMS MA MB MC

A A A

R R R

+ + +− − −

= + +uuuur uuur uuur uuuur

15) Dacă eS , I, Γ sunt punctele Soddy exterior, centrul cercului înscris, respectiv

punctul lui Gergonne corespunzătoare unui triunghi ABC, atunci pentru orice punct M din planul unui triunghi ABC este adevărată relaţia:

[ ]

( ) ( )a b c

i

ABC

i

a b c MI r r r MMS

A

R

+ + − + + Γ=

uuur uuuuruuuur

.

Demonstraţie analoagă cu cea din teorema (10). 16) Punctele Soddy exterior, centrul cercului înscris, respectiv punctul lui Gergonne

corespunzătoare unui triunghi ABC, sunt coliniare şi 4

.2

e

e

S I R r

S p

+=

Γ

Demonstraţie analoagă cu cea din teorema (11). 17) Consecinţă: Centrul cercului înscris, punctul lui Gergonne şi punctele lui Soddy corespunzătoare unui triunghi ABC sunt coliniare.

Demonstraţie. Din teoremele (11) şi (16) rezultă .i e

i e

S I S I

S S=

Γ Γ

Observaţie: Teorema precedentă poate fi reformulată astfel: centrul cercului înscris şi punctul lui Gergonne aparţin dreptei lui Soddy.

18) Punctele lui Soddy sunt conjugate armonic cu centrul cercului înscris şi cu punctul lui Gergonne. Demonstraţia rezultă din teoremele (11) şi (16). 19) Dacă

eS este punctul Soddy exterior şi 4a b c R r+ + > + , atunci triunghiurile

,e eS BC S CA şi

eS AB au acelaşi perimetru, agal cu 2

eR .

Demonstraţie. Avem 2 3 2 3( ) ( ) ( ) 2e e e e eS B S C BC R r R r r r R+ + = − + − + + = . Analog,

2e e eS C S A CA R+ + = şi 2

e e eS A S B AB R+ + = , deci triunghiurile ,

e eS BC S CA şi

eS AB

au acelaşi perimetru. Punctul

eS se numeşte punctul izoperimetric al triunghiului ABC.

20) Dreaptele lui Soddy şi Gergonne sunt perpendiculare. Demonstraţie. Vezi „Dreapta lui Gergonne”. 21) Fie , ,

a b cP P P punctele de tangenţă dintre cercul Soddy interior şi cercurile C1 , C2 ,

respectiv C3 . Dreptele , ,a a b b c c

C P C P C P sunt concurente.

Page 66: Barbu_TEOREME_FUNDAMENTALE(2008).pdf

73

Demonstraţie. Deoarece 3 2

2 3

a

a

CC r

BC r

εε

= = rezultă că afixul punctului a

C este:

2 3

2 3

.a

B C

C

z zz

ε εε ε+

=+

Analog, 3 1

3 1b

C A

C

z zz

ε εε ε+

=+

, 1 2

1 2c

A B

C

z zz

ε εε ε+

=+

, 1 4

1 4

i

a

A S

P

z zz

ε ε

ε ε

+=

+,

2 4

2 4

i

b

B S

P

z zz

ε ε

ε ε

+=

+, 3 4

3 4

i

c

C S

P

z zz

ε ε

ε ε

+=

+. Vom arăta că punctul

iE de afix

1 2 3 4

1 2 3 4

i

i

A B C S

E

z z z zz

ε ε ε ε

ε ε ε ε

+ + +=

+ + + aparţine dreptelor , ,

a a b b c cC P C P C P . Deoarece

1 2 3 4 1 2 3 4

1 2 3 4 1 2 3 4

( ) ( )i c cA B C S C P

z z z z z zε ε ε ε ε ε ε ε

ε ε ε ε ε ε ε ε

+ + + + + +=

+ + + + + + rezultă că punctul

iE aparţine

dreptei c c

C P . Analog, grupând convenabil termenii sumei de la numărătorul afixului

punctuluiiE , se arată că punctul

iE aparţine şi dreptelor , ,

a a b bC P C P deci

dreptele , ,a a b b c c

C P C P C P sunt concurente.

Punctul

iE de concurenţă al dreptelor , ,

a a b b c cC P C P C P se numeşte punctul Eppstein –

Oldknow interior. Triunghiul a b cP P P se numeşte triunghiul Soddy interior.

22) Fie ' ' ', ,

a b cP P P punctele de tangenţă dintre cercul Soddy exterior şi cercurile C1 , C2 ,

respectiv C3 . Dreptele ' ' ', ,a a b b c c

C P C P C P sunt concurente.

Demonstraţia este analoagă cu cea din teorema precedentă.

A B

C

p a−

p b−

p c−

Fig. 70

aC

bC

cC 'aP

'bP

'cP

eE

Page 67: Barbu_TEOREME_FUNDAMENTALE(2008).pdf

74

Punctul eE de concurenţă al dreptelor ' ' ', ,a a b b c cC P C P C P se numeşte punctul Eppstein -

Oldknow exterior. Triunghiul ' ' 'a b cP P P se numeşte triunghiul Soddy exterior.

23) Punctele Soddy interior, Eppstein – Oldknow interior şi Gergonne, corespunzătoare unui triunghi ABC sunt coliniare.

Demonstraţie. Deoarece afixul punctului lui Gergonne este 1 2 3

1 2 3

A B Cz z zz

ε ε εε ε εΓ

+ +=

+ + (vezi

„Punctul lui Gergonne”) şi 1 2 3 4 1 2 3 4

1 2 3 4 1 2 3 4

( )i i

i

A B C S A B C S

E

z z z z z z z zz

ε ε ε ε ε ε ε ε

ε ε ε ε ε ε ε ε

+ + + + + += = =

+ + + + + +

1 2 3 4

1 2 3 4

( )iS

z zε ε ε ε

ε ε ε εΓ+ + +

+ + + rezultă că punctele ,

i iE S şi Γ sunt coliniare.

24) Punctele Soddy exterior, Eppstein – Oldknow exterior şi Gergonne, corespunzătoare unui triunghi ABC sunt coliniare. Demonstraţia este analoagă cu cea din teorema precedentă. 25) Punctele lui Eppstein – Oldknow aparţin dreptei lui Soddy. Demonstraţia rezultă din teoremele (18), (23) şi (24). 26) Coordonatele baricentrice relative ale punctului Eppstein – Oldknow interior sunt:

[ ] [ ] [ ]

1 2 3

2 2 2, , .ABC ABC ABC

i

A A AE a b c

r r r

+ + +

Demonstraţie. Deoarece , ,a b cP P P sunt punctele de tangenţă dintre cercul Soddy interior şi

cercurile C1 , C2 , respectiv C3 rezultă 1

i a i

a

S P R

AP r= , deci 1

1

i i

a

i

r MS R MAMP

r R

+=

+

uuuur uuuruuuur

sau

[ ] [ ] [ ]1

[ ] 1 1 2 3

2

( )ABC ABC ABCi

a

ABC i

A A Ar RMP a MA b MB c MC

A r R r r r

= + + + + +

+

uuuur uuur uuur uuuur(unde am

utilizat teorema 14). Atunci, coordonatele baricentrice relative ale punctului aP sunt

[ ] [ ] [ ]

1 2 3

2, , .ABC ABC ABCA A A

a b cr r r

+ + +

Coordonatele baricentrice relative ale punctului aC

sunt 2 3

1 10, ,r r

. Deoarece

[ ] [ ] [ ]

1 2 3

[ ] [ ] [ ]

1 2 3

2 3

2 2 2

2 2 20

0 1/ 1/

ABC ABC ABC

ABC ABC ABC

A A Aa b c

r r r

A A Aa b c

r r r

r r

+ + +

+ + + = rezultă că punctele

aC ,

aP şi

iE sunt coliniare. Analog se arată că punctul

iE aparţine dreptelor ,

b b c cC P C P ,

deci [ ] [ ] [ ]

1 2 3

2 2 2, ,ABC ABC ABC

i

A A AE a b c

r r r

+ + +

este punctul Eppstein interior.

Page 68: Barbu_TEOREME_FUNDAMENTALE(2008).pdf

75

27) Coordonatele baricentrice relative ale punctului Eppstein – Oldknow exterior sunt:

[ ] [ ] [ ]

1 2 3

2 2 2, , .ABC ABC ABC

i

A A AE a b c

r r r

− − −

Demonstraţie analoagă cu precedenta. 28) Dacă

iE , I, Γ sunt punctele Eppstein interior, centrul cercului înscris, respectiv

punctul lui Gergonne corespunzătoare unui triunghi ABC, atunci pentru orice punct M din planul unui triunghi ABC este adevărată relaţia:

( ) 2( ).

2( )a b c

i

a b c

a b c MI r r r MME

a b c r r r

+ + + + + Γ=

+ + + + +

uuur uuuuruuuur

Demonstraţie. Ţinând cont de teorema (25) şi de faptul că

[ ] [ ] [ ][ ]

1 2 3

2 2 2 1 1 12 2 2( 4 )ABC ABC ABC

ABC

A A Aa b c p A p R r

r r r p a p b p cα

+ + + + + = + + + = + + = − − −

rezultă că pentru orice punct M din planul triunghiului ABC este adevărată egalitatea

[ ] [ ] [ ]

1 2 3

2 2 21 ABC ABC ABC

i

A A AME a MA b MB c MC

r r rα

= + + + + +

uuuur uuur uuur uuuur, egalitate echivalentă cu

[ ]

1 1 1 1( ) 2

i ABCME aMA bMB cMC A MA MB MC

p a p b p bα

= + + + + + − − −

uuuur uuur uuur uuuur uuur uuur uuuur sau

[ ]

[ ]

1 1 1( ) 2

( ) 2( )

2( )1 1 12

ABC

a b c

i

a b c

ABC

a b c MI A Mp a p b p c a b c MI r r r M

MEa b c r r r

a b c Ap a p b p c

+ + + + + Γ − − − + + + + + Γ = =

+ + + + + + + + + + − − −

uuur uuuuruuur uuuur

uuuur

29) Consecinţă: Punctul Eppstein interior împarte segmentul IΓ în raportul 2( )

a b cr r r

a b c

+ +

+ +.

Demonstraţie. Din teorema precedentă rezultă 2( )

.i a b c

i

E I r r r

E a b c

+ +=

Γ + +

30) Dacă

eE , I, Γ sunt punctele Eppstein exterior, centrul cercului înscris, respectiv

punctul lui Gergonne corespunzătoare unui triunghi ABC, atunci pentru orice punct M din planul unui triunghi ABC este adevărată relaţia:

( ) 2( ).

2( )a b c

e

a b c

a b c MI r r r MME

a b c r r r

+ + − + + Γ=

+ + − + +

uuur uuuuruuuur

Demonstraţie analoagă cu cea din teorema (28).

31) Consecinţă: 2( )

.e a b c

e

E I r r r

E a b c

+ +=

Γ + +

Page 69: Barbu_TEOREME_FUNDAMENTALE(2008).pdf

76

32) Punctele lui Eppstein sunt conjugate armonic cu centrul cercului înscris şi cu punctul lui Gergonne.

Demonstraţie. Din teoremele (28) şi (30) rezultă .i e

i e

E I E I

E E=

Γ Γ

33) Ecuaţia dreaptei lui Soddy în coordonate baricentrice este:

2 2 2( ) ( ) ( ) ( ) ( ) ( ) 0p a c b x p b a c y p c b a z− − + − − + − − =

Demonstraţie. Deoarece punctul lui Gergonne şi centrul cercului înscris în triunghiul ABC aparţin dreptei Soddy, ecuaţia dreptei lui Soddy în coordonate baricentrice este:

1 1 10

x y z

p a p b p c

a b c

=− − −

sau 2 2 2( ) : ( ) ( ) ( ) ( ) ( ) ( ) 0I p a c b x p b a c y p c b a zΓ − − + − − + − − = .

I.12. Punctul lui Steiner19

„Adevărul nu stăluceşte decât în ochii celui care l-a căutat îndelung, destul de îndelung ca să merite să –l vadă.” - Henri Lebesgue20

1) Paralelele duse prin vârfurile unui triunghi la laturile respective ale primului triunghi Brocard sunt concurente într-un punct situat pe cercul circumscris triunghiului ABC. Demonstraţie. Fie 1 1 1A B C primul triunghi Brocard al

triunghiului ABC şi S punctul de intersecţie dintre paralelele duse din B şi C la laturile 1 1AC ,

respectiv 1 1A B (Fig. 71). Atunci,

1 1 1( ) 180 ( ) 180 ( )= ° − = ° −m BSC m B A C m BAC

(vezi „Triunghiurile Brocard”), relaţie ce arată că patrulaterul ABSC este inscriptibil, deci S este pe cercul circumscris triunghiului ABC (1). Analog, fie

'S punctul de intersecţie dintre paralelele duse prin B şi A la 1 1AC , respectiv 1 1B C . Atunci,

1 1 1( ' ) 180 ( ) 180 ( )mBS A m BCA mBCA= °− = °− ,adică punctul

'S aparţine cercului circumscris triunghiului ABC (2). Din relaţiile (1) şi (2) rezultă 'S S≡ . Observaţie: Punctul S de concurenţă al celor trei paralele se numeşte punctul lui Steiner al triunghiului ABC.

19 Jakob Steiner (1796-1863) – matematician elveţian, profesor la Universitatea din Berlin, contribuţii importante în geometrie 20 Henri Lebesgue (1875 -1941) – matematician francez, contribuţii importante în analiza matematică

A

O

Fig. 71

1A

1B

1C

T

S

K

1S 3S

2S

σ

Page 70: Barbu_TEOREME_FUNDAMENTALE(2008).pdf

77

2) Punctele lui Steiner (S) şi Lemoine (K) al triunghiului ABC sunt puncte omoloage în triunghiul ABC, respectiv 1 1 1A B C - primul triunghi Brocard al triunghiului ABC.

Demonstraţie. Punctul S aparţine cercului circumscris triunghiului ABC, K aparţine cercului circumscris primului triunghi Brocard 1 1 1A B C (vezi „Triunghiurile lui Brocard”), iar

1 1 1 1( ) ( ) ( )m SBC m KAC m KBC= = , deoarece 1 1BS AC şi 1 .KA BC

3) Punctul lui Steiner al triunghiului ABC este punctul diametral opus în cercul circumscris triunghiului ABC punctului lui Tarry. Demonstraţie. Punctul lui Tarry (T) se află la intersecţia perpendicularelor coborâte din vârfurile triunghiului ABC pe laturile respective ale primului triunghi Brocard. Deoarece

( ) ( ) ( ) 90= = = °m TBS m TAS m TCS rezultă că punctul lui Tarry este punctul diametral opus

lui S în cercul circumscris triunghiului ABC. 4) Dreapta lui Simson a punctului lui Steiner în raport cu un triunghi ABC este paralelă cu dreapta OK. Demonstraţie. Fie 1 2 3, ,S S S proiecţiile punctului lui Steiner S pe laturile BC, CA, respectiv

AB . Avem 2 1 2 1 1 1( ) ( ) ( ) ( )m S S C m S SC m A BO m A KO θ= = = = deoarece 2 1SS OB şi

1 1SC A B , iar patrulaterul 1 1OB KA este inscriptibil. Atunci,

1 1 2( ) 90 ( )m KOA m SS Sθ= °− = şi deoarece 1 1SS AO , rezultă că 1 2S S OK .

5) Fie K punctul lui Lemoine şi O centrul cercului circumscris unui triunghi ABC. Paralela dusă prin A la dreapta OK intersectează cercul circumscris triunghiului ABC în punctul σ . Perpendiculara dusă din punctul σ pe dreapta BC trece prin punctul lui Steiner al triunghiului ABC. Demonstraţie. Fie 'σ punctul de intersecţie dintre perpendiculara dusă din punctul lui Steiner (S) pe dreapta BC şi cercul circumscris triunghiului ABC (Fig. 71). Dreapta 'Aσ este paralelă cu dreapta lui Simson a punctului lui Steiner (cf. th. 1– „Dreapta lui Simson”), de unde 'A OKσ (cf. th. 4), deci ' .σ σ≡ Observaţie: Teorema de mai sus ne dă un mod practic de construcţie al punctului lui Steiner al unui triunghi: ducem prin vârful A paralela la dreapta OK şi notăm cu σ punctul de intersecţie al acesteia cu cercul circumscris triunghiului ABC; din punctul σ ducem perpendiculara pe latura BC, al doilea punct de intersecţie al acesteia cu cercul circumscris este punctul lui Steiner. 6) Consecinţă: Dreapta care trece prin punctul lui Tarry şi punctul ,σ determinat de intersecţia paralelei duse prin A la dreapta OK, este paralelă cu dreapta BC. Demonstraţia este evidentă deoarece ST este diametru în cercul circumscris, deci

( ) 90m T Sσ = ° , şi cum S BCσ ⊥ , rezultă .T BCσ

7) Dreapta OΩ este tangentă cercului circumscris triunghiului "SΩΩ , unde S este punctul lui Steiner corespunzător triunghiului ABC, iar Ω şi "Ω sunt primul, respectiv cel de-al treilea punct al lui Brocard.. Demonstraţie. Vezi „Triunghiurile lui Brocard”.

Page 71: Barbu_TEOREME_FUNDAMENTALE(2008).pdf

78

8) Dreapta 'OΩ este tangentă cercului circumscris triunghiului ' "SΩ Ω , unde S este punctul lui Steiner corespunzător triunghiului ABC, iar Ω şi "Ω sunt primul, respectiv cel de-al treilea punct al lui Brocard. Demonstraţie. Vezi „Triunghiurile lui Brocard”.

I.13. Punctul lui Tarry21

„Matematica este partea exactă a cunoaşterii umane.” - G. Moisil22

Punctul lui Tarry al triunghiului ABC este punctul diametral opus punctului lui Steiner în cercul circumscris triunghiului ABC. 1) Perpendicularele duse din vârfurile triunghiului ABC pe laturile opuse ale primului triunghi Brocard corespunzător triunghiului ABC sunt concurente în punctul lui Tarry al triunghiului ABC. Demonstraţie. Fie 1 1 1A B C primul triunghi Brocard

al triunghiului ABC, iar S punctul lui Steiner, T punctul lui Tarry (Fig. 72). Avem,

( ) 90 ,= °m TBS deci ⊥BT BS şi cum 1 1BS AC

rezultă 1 1.⊥BT AC Analog se arată că 1 1⊥AT BC

şi 1 1.⊥CT A B

2) Dreapta lui Simson a punctului lui Tarry în raport cu triunghiul ABC este perpendiculară pe dreapta OK. Demonstraţie. Deoarece dreapta lui Simson a punctului lui Steiner este paralelă cu dreapta OK, iar punctele lui Tarry şi Steiner sunt diametral opuse, atunci conform teoremei 5 de la capitolul „Dreapta lui Simson”, rezultă că dreapta lui Simson a punctului lui Tarry în raport cu triunghiul ABC este perpendiculară pe dreapta OK.

3) Dreapta care trece prin punctul lui Tarry şi punctul ,σ determinat de intersecţia paralelei duse prin A la dreapta OK, este paralelă cu dreapta BC. Demonstraţie. Vezi „Punctul lui Steiner”.

4) Poligoanele TACSB şi 1 1 1OAC KB sunt invers asemenea.

Demonstraţie: Deoarece triunghiurile ABC şi 1 1 1A B C sunt asemenea (vezi „Triunghiurile

lui Brocard”), iar punctele T şi O, respectiv S şi K sunt puncte omoloage în cele două triunghiuri rezultă concluzia. 5) Punctul lui Tarry, centrul de greutate şi centrul cercului lui Brocard corespunzător unui triunghi ABC sunt coliniare. Demonstraţie. Vezi „Triunghiurile lui Brocard”. 21 Gaston Tarry (1843-1913) – matematician francez, contribuţii în geometrie şi teoria numerelor 22 Grigore Moisil (1906-1973) – matematician român, profesor la Universitatea din Iaşi, membru al Academiei Române

A

B C

O

Fig. 72

1A

1B

1C

T

S

K

Page 72: Barbu_TEOREME_FUNDAMENTALE(2008).pdf

79

6) Fie T punctul lui Tarry, L centrul cercului lui Brocard, G centrul de greutate şi O centrul cercului circumscris unui triunghi ABC. Atunci, 2 .OG GL GT= ⋅ Demonstraţie. Vezi „Triunghiurile lui Brocard”. 7) Fie T punctul lui Tarry, L centrul cercului lui Brocard, G centrul de greutate şi O

centrul cercului circumscris unui triunghi ABC. Atunci, 2

cos.

1 4sin

RGT GO

ω

ω= ⋅

Demonstraţie. Vezi „Triunghiurile lui Brocard”. 8) Al treilea punct Brocard "Ω aparţine dreptei ce uneşte punctele lui Tarry şi Steiner corespunzătoare unui triunghi ABC. Demonstraţie. Vezi „Triunghiurile lui Brocard”.

I.14. Puncte izodinamice „Geometria practică dedusă din experienţă nu se confundă cu geometria axiomatică. Întrebarea dacă geometria practică a universului este euclidiană sau nu are o importanţă evidentă şi răspunsul poate să fie dat numai de experienţă.” – Albert Einstein23

Punctul S din planul triunghiului ABC pentru care a SA b SB c SC⋅ = ⋅ = ⋅ (unde a,b,c sunt lungimile laturilor BC, CA, respectiv AB) se numeşte punct izodinamic al triunghiului ABC . 1) Distanţele de la un centru izodinamic la vârfurile triunghiului sunt invers proporţionale cu lungimile laturilor opuse. Demonstraţia rezultă din definiţia punctului izodinamic. 2) Centrele izodinamice S şi 'S ale unui triunghi neechilateral sunt puncte inverse faţă de cercul circumscris triunghiului. Demonstraţie. Deoarece cercul circumscris triunghiului ABC este ortogonal cercurilor lui Apollonius rezultă că puterea luiO (centrul cercului circumscris triunghiului ABC) faţă de

aceste cercuri este egală cu raza cercului circumscris, deci 2'OS OS R⋅ = , ceea ce arată că punctele S şi 'S sunt inverse faţă de cercul circumscris triunghiului ABC.

23 Albert Einstein (1879-1955) – fizician german, profesor universitar la Berlin şi Princeton, laureat al Premiului Nobel

A

B C O

S

aL

Fig. 73

S'

K

T

Page 73: Barbu_TEOREME_FUNDAMENTALE(2008).pdf

80

O mulţime formată din patru puncte, având proprietatea că fiecare dintre ele este centru izodinamic al triunghiului determinat de celelalte trei puncte se numeşte patrupunct izodinamic. 3) Dacă S este un punct izodinamic al triunghiului ABC , atunci mulţimea , , , A B C S este un patrupunct izodinamic. Demonstraţia rezultă din simetria relaţiei BC SA CA SB AB SC⋅ = ⋅ = ⋅ . 4) Triunghiurile podare ale punctelor izodinamice sunt triunghiuri echilaterale.

Demonstraţie. Fie a b cS S S triunghiul podar

al punctului izodinamic S în raport cu triunghiul ABC (Fig. 74). Din teorema sinusurilor în triunghiul

b cAS S avem:

sinc bS S

ASA

= , de unde

2 2c b

a a ASS S AS

R R

⋅= ⋅ = . Analog,

2a b

c CSS S

R

⋅= şi

2a c

b BSS S

R

⋅= relaţii care

împreună cu a SA b SB c SC⋅ = ⋅ = ⋅ dau:

a b b c c aS S S S S S≡ ≡ , adică

a b cS S S este

triunghi echilateral. 5) Punctele izodinamice ale triunghiului ABC neechilateral sunt punctele de intersecţie dintre dreapta OK şi cercurile lui Apollonius. Demonstraţie. Cum OK este axa radicală a cercurilor lui Apollonius, fie că punctul K aparţine interiorului cercului lui Apollonius corespunzător vârfului A al triunghiului ABC , atunci dreapta OK intersectează acest cerc în punctele distincte S şi 'S , puncte care se află pe cercurile lui Apollonius corespunzătoare vârfurilor B, respectiv C. Deoarece S şi

'S aparţin tuturor cercurilor lui Apollonius corespunzătoare vârfurilor triunghiului ABC rezultă că a SA b SB c SC⋅ = ⋅ = ⋅ , respectiv ' ' 'a S A b S B c S C⋅ = ⋅ = ⋅ , adică punctele S şi

'S sunt punctele izodinamice ale triunghiului ABC . Punctul S - interior triunghiului ABC - se numeşte punctul izodinamic interior al triunghiului ABC, iar 'S punctul izodinamic exterior al triunghiului ABC . Observaţii: i) Dacă triunghiul ABC este echilateral atunci a b c= = şi atunci punctul 'S este “aruncat la infinit”. ii) Punctele izodinamice S şi 'S ale unui triunghi neechilateral pot fi determinate de

dreapta OK prin relaţiile: 2( ) 'O OS OS Rρ = ⋅ = şi ( ) 'K KS KS KA KTρ = ⋅ = ⋅ , unde

T AK= I C(O,R).

iii) Punctele izodinamice aparţin axei Brocard OK .

A

B C

S

aS

bS cS

Fig. 74

Page 74: Barbu_TEOREME_FUNDAMENTALE(2008).pdf

81

6) Simetricele punctului izodinamic S faţă de laturile triunghiului ABC determină un triunghi echilateral omologic cu triunghiul ABC. Demonstraţie. Fie

a b cS S S triunghiul

podar al punctului izodinamic S în

raport cu triunghiul ABC şi ' ' ', ,a b cS S S

simetricele punctului S faţă de laturile BC, CA, respectiv AB (Fig. 75). Deoarece triunghiul

a b cS S S este

echilateral (cf. th. (4)), iar triunghiurile

a b cS S S şi ' ' '

a b cS S S sunt omotetice – prin

omotetia de centru S şi raport 2 – rezultă că triunghiul ' ' '

a b cS S S este

echilateral. 7) Punctele izodinamice S şi 'S ale unui triunghi neechilateral ABC simetrice faţă de dreapta lui Lemoine a triunghiului ABC . Demonstraţie. Deoarece OK este axă radicală a cercurilor lui Apollonius rezultă că dreapta OK este perpendiculară pe dreapta centrelor – adică pe dreapta lui Lemoine a triunghiului ABC - deci 'SS este perpendiculară pe dreapta lui Lemoine, ceea ce arată că punctele izodinamice S şi 'S sunt simetrice faţă de dreapta lui Lemoine a triunghiului ABC . 8) Punctele izodinamice şi izologice ale unui triunghi neechilateral ascuţitunghic sunt conciclice. Demonstraţie. Vezi „Puncte izologice”. 9) Coordonatele unghiulare ale punctului izodinamic S sunt: ( ) 60 ,m A + °

( ) 60 , ( ) 60 .m B m C+ ° + ° Demonstraţie. Coordonatele unghiulare ale lui S sunt date de măsurile unghiurilor

,ASB BSC respectiv CSA (Fig.75). Vom demonstra că ( ) ( ) 60 .m ASB m ACB= + °

După cum am arătat triunghiul podar al lui S este un triunghi echilateral (fie a b cS S S acest

triunghi). Avem ( ) 180 ( ) ( ).m ASB m SAB m SBA= °− − Dar ( ) 180 ( ) ( )m ACB m A m B= °− − =

180 [ ( ) ( )] [ ( ) ( )] ( ) ( ) ( )m SAB m SAC m SBA m SBC m ASB m SAC m SBC° − + − − = − −

(1). Deoarece patrulaterele a c

SS BS şi b c

SS AS sunt inscriptibile rezultă b c b

SAS SS S≡

şi ,a c a

SBS SS S≡ relaţia (1) devenind: ( ) ( ) ( ) ( )c b c a

m ASB m C m SS S m SS S= + + =

( ) 60 .m C + ° Analog, se arată că ( ) ( ) 60m BSC m A= + ° şi ( ) ( ) 60 .m CSA m B= + °

Observaţie: Analog se determină coordonatele unghiulare ale celui de-al doilea punct izodinamic 'S : ( ) 60 , ( ) 60 , ( ) 60 ,m A m B m C− ° − ° − ° făcând consideraţia că dacă

aceste unghiuri sunt negative sau mai mari decât 180° se vor considera complementele lor.

A

B C

S

aS

bS cS

Fig. 75

'aS

'bS

'cS

Page 75: Barbu_TEOREME_FUNDAMENTALE(2008).pdf

82

10) Punctele izodinamice S şi 'S sunt punctele izogonale punctelor 1F şi 2F ale lui Fermat. Demonstraţie. Dacă punctele izogonale M şi 'M au coordonatele unghiulare ( , , ),λ µ ν

respectiv ( ', ', ')λ µ ν ,atunci ' 180 ( ), ' 180 ( ), ' 180 ( )m C m A m Bλ λ µ µ ν ν+ = °+ + = °+ + = °+

(vezi „Drepte izogonale”). Fie *S izogonalul conjugat al primului punct izodinamic S.

Atunci, ( ) ( ) ( ) 180m BSC m BS C m A∗+ = + ° , de unde rezultă că

( ) 180 ( ) ( )m BS C m A m BSC∗ = ° + − sau ( ) 180 ( ) ( )m BS C m A m BSC∗ = ° + − =

180 ( ) [ ( ) 60 ] 120m A m A° + − + ° = ° . Analog se arată că ( ) 120m AS C∗ = ° şi

( ) 120m AS B∗ = ° , deci punctul S ∗ coincide cu primul punct al lui Fermat. Analog se

arată că izogonalul celui de-al doilea punct izodinamic este al doilea punct al lui Fermat. 11) Triunghiul podar

a b cS S S al primului punct izodinamic S al triunghiului ABC este

omotetic cu triunghiul exterior al lui Napoleon a b c

N N N .

Demonstraţie. Punctele S şi 1F sunt izogonale (cf. proprietăţii precedente). Triunghiul

antipodar " " "A B C al punctului 1F este omotetic cu triunghiul a b c

N N N . ; cum triunghiul

podar al unui punct este omotetic cu triunghiul antipodar al izogonalului său rezultă că triunghiurile

a b cS S S şi " " "A B C sunt omotetice. Deoarece relaţia de omotetie este

tranzitivă rezultă că triunghiurile a b c

N N N şi a b cS S S sunt omotetice.

12) Triunghiul podar al celui de al doilea punct izodinamic S al triunghiului ABC este omotetic cu triunghiul interior al lui Napoleon. Demonstraţie analoagă cu precedenta.

13) Punctele 'S , K, S, O formează o diviziune armonică. Demonstraţie. Lema 1. Fie T punctul de intersecţie dintre tangenta în

A la cercul circumscris unui triunghi neisoscel ABC şi

dreapta BC. Atunci, 2

.TB AB

TC AC

=

Demonstraţie: Avem 1( ) ( ) ( )

2m TAB m ACB m AB= =

şi ( ) ( ) ( )m TAC m BAC m BCA= + = 180 ( )m ABC°− (Fig. 76). Din teorema sinusurilor

rezultă: sin sin

sin sin

TB AB TAB AB C

TC AC TAC AC B= ⋅ = ⋅

de unde

2TB AB

TC AC

=

.

Lema 2. Fie un cerc ( , )O Rς , punctele , ,A B O ABς∈ ∉ , iar T punctul de intersecţie al

tangentelor în A şi B la ς . O dreapta d ce trece prin T intersectează cercul ς în punctele

M şi N, .S d AB= ∩ Atunci, .TM SM

TN SN=

A

B C T

Fig. 76

Page 76: Barbu_TEOREME_FUNDAMENTALE(2008).pdf

83

Demonstraţie: Avem: 2 2

(1)TM AM BM

TN AN BN

= =

(Fig.

77) (conform lemei precedente), de unde .AM BN AN BM⋅ = ⋅ Din teorema sinusurilor rezultă

sin sin

MB BN

MAB BAN=

şi

sin,

sin

MS AM MAB

SN AN BAN= ⋅

deci

2

(2).SM AM BM MA

SN AN BN NA

= ⋅ =

Din relaţiile (1) şi (2) rezultă

2 2

.TM SM MA MB

TN SN NA NB

= = =

Observaţie: Punctele coliniare T, M, S, N ce verifică relaţia TM SM

TN SN= spunem că

formează o diviziune armonică. Demonstraţia teoremei: Deoarece cercul circumscris triunghiului ABC este ortogonal cercurilor lui Apollonius corespunzătoare triunghiului ABC, rezultă conform lemei 2 că

'

'

S K SK

S O SO= , adică punctele 'S , K, S şi O formează o diviziune armonică.

I.15. Punctul izogon „Geometria reprezintă eterna sclipire în mintea lui Dumnezeu. Împărtăşirea acesteia şi omului reprezintă motivul pentru care omul este imaginea lui Dumnezeu.” – Johannes Kepler24

Conform teoremei lui Toricelli – Fermat, pentru un triunghi ABC ale cărui unghiuri sunt mai mici decât 120° , există un punct T pentru care suma + +TA TB TC este minimă. Punctul T pentru care se realizează minimul sumei + +TA TB TC se numeşte punct izogon sau punctul lui Toricelli-Fermat al triunghiului ABC. 1) Într-un triunghi există cel mult un punct izogon. Demonstraţie. Presupunem prin absurd că triunghiul ABC are două puncte izogone '≠T T (Fig.78). Dacă M este mijlocul

segmentului 'TT , atunci '

,2

TA T AMA

+<

',

2

+<TB T B

MB

',

2

+<TC T C

MC de unde rezultă:

' ' '

2

+ + + + ++ + < = + +

TA TB TC T A T B T CMA MB MC TA TB TC

ceea ce contrazice faptul că T este punct izogon.

24 Johannes Kepler (1571-1630) – matematician şi astronom german, considerat precursor al calculului integral

A

B M

N

T S

d

Fig. 77

A

M T T '

Fig. 78

Page 77: Barbu_TEOREME_FUNDAMENTALE(2008).pdf

84

2) Dacă triunghiul ABC are un punct izogon, atunci acest punct se află în interiorul triunghiului ABC. Demonstraţie. Presupunem prin reducere la absurd că punctul izogon T ar fi în exteriorul triunghiului ABC, fie că T şi A se află în semiplane diferite determinate de dreapta BC şi

= ∩P AT BC (Fig. 79). Fie 'T simetricul lui T faţă de

dreapta BC. Atunci, ' ' .AT AP PT AP PT AT< + = + = Totodată ', 'BT BT CT CT= = (BC fiind mediatoarea segmentului 'TT ). Astfel, ' ' 'AT BT CT AT BT CT+ + < + + relaţie ce este în contradicţie cu faptul că T este punctul

izogon al triunghiului ABC . 3) Coordonatele unghiulare ale centrului izogon sunt egale cu 120 .° Demonstraţie. ( ) ( ) ( ) 120= = = ° m ATB m BTC m CTA

(vezi „Teorema lui Fermat”).

4) Centrul izogon al unui triunghi echilateral este centrul cercului circumscris triunghiului. Demonstraţia este evidentă, deoarece ( ) ( ) ( ) 120= = = ° m AOB m BOC m COA , unde O

este centrul cercului circumscris triunghiului ABC. 5) Triunghiul antipodar al centrului izogon T al unui triunghi este un triunghi echilateral. Demonstraţie. Fie MNP triunghiul antipodar al punctului T faţă de triunghiul ABC (Fig. 80). Din patrulaterul inscriptibil TAMB rezultă ( ) 180 ( )m BMA m BTA= °− =

180 120 60 .° − ° = ° Analog, ( ) ( ) 60 ,= = ° m MNP m MPN

ceea ce arată că triunghiul MNP este echilateral. 6) Triunghiul podar al izogonalului centrului izogon T al triunghiului ABC relativ la triunghiul ABC, este un triunghi echilateral. Demonstraţie. Deoarece triunghiul podar al unui punct P relativ la triunghiul ABC este asemenea cu triunghiul antipodar al izogonalului său relativ la triunghiul ABC, atunci utilizând proprietatea precedentă rezultă concluzia. 7) Centrul izogon T al triunghiului ABC este izogonalul unuia dintre centrele izodinamice ale triunghiului ABC. Demonstraţia este evidentă deoarece singurele puncte din planul unui triunghi ale căror triunghiuri podare sunt echilaterale sunt centrele izodinamice ale triunghiului ABC (vezi„Puncte izodinamice”).

A

B C

P

T '

T

Fig. 79

A B

C

T

M

N P

Fig. 80

S

A" B"

C"

Page 78: Barbu_TEOREME_FUNDAMENTALE(2008).pdf

85

8) Fie ABC un triunghi ale cărui unghiuri sunt fiecare mai mici decât 120° , T punctul izogon al triunghiului ABC şi ' ' 'A B C triunghiul podar al lui T. Dacă cercul circumscris triunghiului ' ' 'A B C mai intersectează laturile BC, CA şi AB în ", "A B respectiv

",C atunci triunghiul " " "A B C este echilateral. Demonstraţie. Cercul circumscris triunghiului

' ' 'A B C este cercul podar al punctului izogon T în raport cu triunghiul ABC şi conform teoremei celor 6 puncte (vezi „Drepte izogonale”) rezultă că " " "A B C este triunghiul podar al punctului S – izogonalul punctului T (Fig. 81). Deci,

( " " ") ( " ") ( " ")= + = m B A C m SA B m SA C

( ") ( ") ( " ) ( ")+ = + m SCB m SBC m A CT m TBA

( " " ") 180 ( ) 180 120 60 .= °− = °− ° = ° m B A C m BTC

Analog, se arată că ( " " ") 60 ,= °m A B C adică

triunghiul " " "A B C este echilateral. 9) Într-un triunghi ABC, izogonalul punctului izogon T este un punct S pentru care BC SA CA SB AB SC⋅ = ⋅ = ⋅ . Demonstraţie. Fie " " "A B C triunghiul podar al punctului S (Fig. 81). Conform proprietăţii precedente triunghiul " " "A B C este echilateral. Din teorema sinusurilor în triunghiul

" " "A B C rezultă: " " sin .2

= ⋅ = ⋅BC

B C AS A ASR

Analog, " " , " "2 2

= ⋅ = ⋅AC AB

C A BS A B CSR R

.

Deoarece " " " " " "= =A B B C C A rezultă " " sin .2

= ⋅ = ⋅BC

B C AS A ASR

Observaţie: Punctul S cu proprietatea BC SA CA SB AB SC⋅ = ⋅ = ⋅ se numeşte punct izodinamic. 10) Triunghiul antipodar al centrului izogon T este omotetic cu triunghiul podar al punctului izodinamic S. Demonstraţie. Fie " " "A B C triunghiul podar al punctului S în raport cu triunghiul ABC şi MNP triunghiul antipodar al punctului T în raport cu triunghiul ABC. Patrulaterul " "SB AC este inscriptibil, deci " " "≡ AB C ASC şi cum " "≡ SAC TAB rezultă

( " ") ( ") ( ") ( ") 90 ,+ = + = ° m AB C m TAB m ASC m SAC adică dreptele AT şi " "B C

sunt perpendiculare, de unde rezultă " " .C B MP Analog se arată că " "A C MN şi " " ,A B NP deci triunghiurile " " "A B C şi MNP sunt omotetice.

A

B C

B"

B'

A"

C"

A '

C'

Fig. 81

T

S

Page 79: Barbu_TEOREME_FUNDAMENTALE(2008).pdf

86

I.16. Puncte izotomice

„Nu s-ar putea oare reprezenta muzica drept matematică a simţurilor şi matematica drept muzică a raţiunii? Muzicianul simte matematica, iar matematicianul concepe muzica. Muzica este vis, matematica este viaţă practică.” – James Sylvester25

Teorema lui Neuberg26

1) Fie P un punct interior în triunghiul ABC şi 1P = AP BC∩ , 2P = BP AC∩ ,

3P = PC AB∩ , iar 1Q , 2Q şi 3Q sunt simetricele punctelor 1P , 2P şi 3P faţă de

mijloacele laturilor ,BC AC , respectiv AB . Dreptele 1AQ , 2BQ şi 3CQ sunt

concurente. Demonstraţie.

Din teorema lui Ceva rezultă: 31 2

1 2 3

1P ABP P C

PC P A P B⋅ ⋅ = (1). Cum

1 1 1 1 2 2, , ,BP CQ PC BQ P C AQ= = = 2 3 32,P A CQ P B AQ= = relaţia (1) devine

31 2

1 2 3

1BQCQ AQ

BQ CQ AQ⋅ ⋅ = şi din reciproca teoremei lui Ceva rezultă că dreptele 1AQ , 2BQ şi

3CQ sunt concurente într-un punct Q.

Observaţii:

1) Punctele P şi Q se numesc izotomic conjugate . 2) Dreptele 1AP şi 1AQ ( 1 1, ( )P Q BC∈ ) se numesc drepte izotomice dacă punctele 1P şi

1Q sunt simetrice faţă de mijlocul laturii BC .

2) Retrocentrul unui triunghi este punctul izotomic al ortocentrului triunghiului. Demonstraţie. Vezi „ Retrocentrul unui triunghi”.

3) Punctele Gergonne () şi Nagel (N) ale unui triunghi sunt izotomic conjugate. Demonstraţie. Vezi „Punctul lui Gergonne”.

25 James Sylvester (1814-1897) – matematician englez, profesor universitar la Oxford, contribuţii importante în

algebră 26 Joseph Neuberg (1840-1926) – matematician luxemburghez, membru al Academiei Regale Belgiene, contribuţii

importante în geometrie

A

B C M 1P

2P3P

1Q

2Q3Q

P

Q

Fig. 82

Page 80: Barbu_TEOREME_FUNDAMENTALE(2008).pdf

87

4) Punctul lui Lemoine (K) şi centrul cercului circumscris (O) al unui triunghi ABC sunt puncte izotomice în raport cu triunghiul median al triunghiului ABC .

Demonstraţie. Vezi „Punctul lui Lemoine”.

5) În orice triunghi izogonalele izotomicelor a trei puncte coliniare sunt coliniare. Demonstraţie. Vezi „Puncte izogonale”.

6) Într-un triunghi izotomicele izogonalelor a trei puncte coliniare sunt coliniare. Demonstraţie. Vezi „Puncte izogonale”. 7) Într-un triunghi ascuţitunghic izotomicele ortocentrului (H), punctului lui Lemoine (K) şi centrului cercului circumscris (O) sunt coliniare. Demonstraţie. Deoarece O,G,H sunt punctele izogonale ale lui H, K respectiv O – conform proprietăţii precedente - rezultă că izotomicele punctelor H, K, O sunt coliniare.

8) Fie ( , , )P α β γ şi ( ', ', ')Q α β γ două puncte izotomice în raport cu un triunghi ABC,

exprimate în coordonate baricentrice. Atunci, ' ' '.= =αα ββ γγ

Demonstraţie.Fie 1 = ∩P AP BC, 2 = ∩P BP AC , 3 = ∩P PC AB , 1 = ∩Q AQ BC ,

2 = ∩Q BQ AC şi 3 = ∩Q QC AB (Fig. 82). Avem: 1 1 ,PB PCγβ

= −uuur uuur

2 2 3 3 1 1

', , ,

'PC P A P A PB QB QC

α β γγ α β

= − = − = −uuuur uuur uuur uuur uuuur uuuur

2 2 3 3

' ', .

' '= − = −

uuuur uuuur uuuur uuuurQ C Q A Q A Q B

α βγ α

Deoarece

1 1≡BP CQ şi 1 1≡CP BQ rezultă 1 1≡uuur uuuurPB CQ şi 1 1≡

uuur uuuurPC BQ , deci

1 1 1 1

'

'

⋅ = − ⋅ − ⋅ ⋅

uuur uuuur uuur uuuurPB QB PC QC

γ γβ β

, de unde ' '=ββ γγ . Analog, ' ' '.= =αα ββ γγ

9) Consecinţă: Dacă punctele P şi Q sunt izotomice şi coordonatele baricentrice ale lui P

sunt ( , , )α β γ , atunci coordonatele baricentrice ale lui Q sunt 1 1 1

, ,α β γ

.

Demonstraţia rezultă din teorema precedentă. 10) Fie P un punct din planul triunghiului ABC şi Q simetricul lui P faţă de centrul de greutate (G) al triunghiului ABC. Dacă 'P şi 'Q sunt izotomicele punctelor P şi Q,

atunci ' 'PQ P Q .

Demonstraţie. Fie ( , , )α β γ coordonatele baricentrice ale punctului P; cum G(1,1,1) rezultă

coordonatele baricentrice ale punctului Q sunt (2 ,2 , 2 )α β γ− − − . Atunci,

1 1 1' , ,Pα β γ

şi 1 1 1

' , ,2 2 2

Qα β γ

− − −

. Ecuaţiile dreptelor PQ şi ' 'P Q sunt:

Page 81: Barbu_TEOREME_FUNDAMENTALE(2008).pdf

88

0

2 2 2

x y z

α β γα β γ

=

− − −

, respectiv 1 1 1

0

1 1 1

2 2 2

x y z

α β γ

α β γ

=

− − −

, relaţii echivalente cu

( ) : ( ) ( ) ( ) 0P Q x y zβ γ γ α α β− + − + − = şi

( ) ( ) ( )( ' ') : 0

( 2 )( 2 ) ( 2 )( 2 ) ( 2 )( 2 )

x y zP Q

β γ γ α α ββ γ β γ γα α γ α β α β

− − −+ + =

− − − − − −.

Deoarece

1 1 1

0

(2 )(2 ) (2 )(2 ) (2 )(2 )

β γ γ α α ββ γ γ α α β

βγ γ β γα γ α αβ α β

− − − =

− − −

− − − − − −

, rezultă că

' 'PQ P Q .

11) Fie un punct T şi , ,M N P proiecţiile lui pe laturile triunghiului ABC . Simetricul punctului T faţă de centrul cercului circumscris triunghiului ABC se proiectează pe laturile triunghiului în punctele ', ', 'M N P . Punctele

', ', 'M N P sunt izotomicele punctelor M, N, respectiv P. Demonstraţie. În trapezul dreptunghic ' 'MTT M perpendiculara din O pe latura 'MM trece prin mijlocul segmentului 'MM ,deci mediatoarea laturii BC este şi mediatoarea segmentului 'MM , adică punctele M şi

'M sunt izotomice. Analog se arată şi pentru celelalte puncte (Fig. 83). 12) Punctele Gergonne şi Nagel sunt izotomic conjugate. Demonstraţie. Fie

a b cC C C triunghiul de contact al triunghiului ABC atunci

Γ = ∩ ∩a b c

AC BC CC şi 1 2 3, ,N N N punctele de contact ale cercurilor A -exînscris,

B -exînscris, C -exînscris cu laturile BC , CA respectiv AB, deci

1 2 3 = ∩ ∩N AN BN CN . Deoarece 1= = −a

BC CN p b , 2= = −b

CC AN p c şi

3= = −b

AC BN p a rezultă că punctele Gergonne (Γ ) şi Nagel ( N ) sunt izotomic

conjugate. 13) Fie , ,M N P puncte pe laturile ,BC AC , respectiv AB ale triunghiului ABC , astfel încât perpendicularele în , ,M N P pe laturile triunghiului sunt concurente. Dacă

', ', 'M N P sunt izotomicele punctelor , ,M N P , atunci şi perpendicularele în punctele ', ', 'M N P pe laturile triunghiului sunt concurente.

Demonstraţie.Fie T punctul de concurenţă al perpendicularelor duse în , ,M N P pe laturile

triunghiului şi notăm 1BM a= , 2MC a= , 1CN b= , 2NA b= , 1AP c= , 2PB c= . Din

teorema lui Carnot rezultă 2 2 2 2 2 21 1 1 2 2 2a b c a b c+ + = + + (1). Cum 2'BM a= , 1'CM a= ,

A

B C M M'

O T

Fig. 83

T '

Page 82: Barbu_TEOREME_FUNDAMENTALE(2008).pdf

89

2'CN b= , 1'AN b= , 2'AP c= , 1'BP c= atunci relaţia (1) este adevărată şi pentru punctele

', ', 'M N P , adică perpendicularele duse din aceste puncte pe laturile triunghiului sunt concurente într-un punct 'T . 13) Consecinţă: Punctele T şi 'T sunt simetrice faţă de centrul cercului circumscris triunghiului ABC . Demonstraţie. Mediatoarea segmentului 'MM este şi mediatoarea laturii BC (deoarece M şi 'M sunt puncte izotomice). Din trapezele dreptunghice ' 'MTT M şi ' 'NTT N rezultă că mediatoarele laturilor BC şi AC se intersectează în mijlocul segmentului 'TT . 15) Ceviana izotomică unei ceviene de rangul k este ceviana de rang (-k). Demonstraţie. Fie AD o ceviană de ordinul k şi AE izotomica sa, ( , ).D E BC∈ Atunci,

, .k k k

BD c BE DC b c

DC b EC BD c b

− = = = =

16) Fie AD o ceviană de ordinul k şi AE izotomica acesteia. Izogonala dreptei AE este o ceviană de rang (k+2). Demonstraţie. Ceviana izotomică a unei ceviene de rang k este ceviana de rang (-k), conform proprietăţii precedente. Izogonala cevienei de rang (-k) este ceviana de rang 2 ( ) 2k k− − = + (vezi „Drepte izogonale”).

17) Punctele lui Lemoine (K) şi al treilea punct Brocard ( "Ω ) sunt izotomic conjugate. Demonstraţie. Vezi „Puncte izotomice”.

A

B CM M'

N

N 'P

P '

T

T '

Fig. 84

Page 83: Barbu_TEOREME_FUNDAMENTALE(2008).pdf

90

I.17. Puncte izologice

„Matematica va fi limba latină a viitorului, obligatorie pentru toţi oamenii de ştiinţă, tocmai pentru că matematica permite accelerarea maximă a circulaţiei ideilor ştiinţifice.” - Grigore Moisil 27

Punctele izologice ale unui triunghi sunt punctele de intersecţie dintre simetricele cercurilor lui Apollonius faţă de mediatoarele laturilor triunghiului. Un punct U din planul unui triunghi ABC având laturile de lungimi a, b, c se numeşte

izologic dacă: .UA UB UC

a b c= =

Observaţie: Distanţele de la un punct izologic la vârfurile triunghiului sunt direct proporţionale cu lungimile laturilor opuse ale triunghiului. Simetricul cercului lui Apollonius corespunzător vârfului A al triunghiului ABC faţă de mediatoarea segmentului BC este un cerc Apollonius pentru segmentul BC şi conţine

punctele P pentru care .PB PC

b c=

1) Fie '( )A

C cercul lui Apollonius al punctelor P pentru care PB PC

b c= şi '

aL centrul

acestui cerc. Centrul cercului '( )AC verifică relaţia: 2'

'.a

a

L C c

bL B

=

Demonstraţie. Fie '1D şi '

2D punctele de intersecţie dintre dreapta BC şi cercul '( )AC ,

iar "A simetricul punctului A faţă de mediatoarea segmentului BC (evident "A ∈ 'AC ).

Atunci, dreapta '2 " "D A OA⊥ şi 2 1' " ' .D A D CAB≡ Dacă

aL este cercul lui Apollonius

27Grigore Moisil (1906-1973) – matematician român, profesor la Universitatea din Iaşi, membru al Academiei

Române, contribuţii importante în informatică

A

B C

T

S G

K O aL

bL

'aL

'A

"A

'S

U

'U

Fig. 85

1D

'1D

2D '2D

СA

С 'A

Page 84: Barbu_TEOREME_FUNDAMENTALE(2008).pdf

91

corespunzător vârfului A atunci 2

a

a

L B AB

L C AC

=

(vezi „Cercurile lui Apolonius”). Din

motive de simetrie avem 'a aL B L C= şi '

a aL C L B= , de unde rezultă

2 2'

'.a

a

L C AB c

AC bL B

= =

Observaţie: Fie '( )BC cercul lui Apollonius al punctelor P pentru care PA PC

a c= , respectiv

'( )CC cercul lui Apollonius al punctelor P pentru care PB PA

b a= şi fie

bL respectiv

cL

centrele acestor două cercuri. Prin permutări circulare a relaţiei din proprietatea precedentă

se obţin relaţiile 2'

'b

b

L A a

cL C

=

şi 2'

'.c

c

L B b

aL A

=

2) Centrele cercurilor ' ' ', ,A B CC C C sunt coliniare.

Demonstraţie. Avem ' ' ' 2 2 2

' ' ' 2 2 21a b c

a b c

L C L A L B c a b

L B L C L A a c a⋅ ⋅ = ⋅ ⋅ = şi din reciproca teoremei lui

Menelaus rezultă că punctele ' ' ', ,a b cL L L sunt coliniare.

Observaţii:

1) Dreapta pe care se găsesc punctele ' ' ', ,a b cL L L se numeşte dreapta lui Longchamps a triunghiului ABC. 2) Deoarece punctul O (centrul cercului circumscris triunghiului ABC) are aceeaşi putere – egală cu 2R - faţă de fiecare dintre cercurile ' ' ', ,A B CC C C rezultă că O aparţine axei lor

radicale. 3) Centrul de greutate G al triunghiului ABC este şi centrul de greutate al triunghiului

'a a

AL L .

Demonstraţia este evidentă deoarece mijlocul segmentului BC este şi mijlocul segmentului '

a aL L .

4) Centrul de greutate G al triunghiului neechilateral ABC aparţine axei radicale dintre cercurile '( )AC şi '( )BC .

Demonstraţie. Puterea centrului de greutate G al triunghiului ABC faţă de cercul '( )AC este 2 ' 2 ' 2 ' 2 ' '2

1( )a a a a

G GP GL PL GL L Dρ = = − = − , unde P este punctul de contact dintre tangenta

dusă din G la cercul '( )AC , deci ' 2 21( ) (1).

a aG GL L Dρ = − Fie 'A mijlocul laturii BC care

coincide cu mijlocul segmentului ' .a aL L Atunci, '

12 2 'A a a

R D A L L+ = şi 1 1 '2

aBD D A= + ,

de unde ( )

'2( )

a b cDA

b c

−=

+ ( unde 1

acBD

b c=

+). Atunci

2 2'

2 2

( ) (2)

a a

a b cL L

b c

+=

− (unde am

considerat fără a restrânge generalitatea că b c> ). Relaţia lui Stewart aplicată în triunghiul

Page 85: Barbu_TEOREME_FUNDAMENTALE(2008).pdf

92

'a

AL A ne dă 2 22 1 1 2'

3 3 3 3a a a a a a a a aA L m L A m L G m m m m⋅ + ⋅ − ⋅ = ⋅ ⋅ , de unde

2 22 3( ' ) 2

(3).9

a a a

a

L A L A mL G

+ −= Teorema medianei în triunghiul '

a aGL L dă:

2 2 ' 2 ' 22( )1 (4).

3 4a a a a

a

GL L G L Lm

+ − =

Din relaţiile (1), (2), (3) şi (4) şi ţinând cont că

1 2 2a A

abcL D R

b c= =

− rezultă că 2 2 2( ) 2( ) (5).G a b cρ = + + Datorită simetriei relaţiei (5)

rezultă că punctul G aparţine axei radicale cercurilor ' ' ', ,A B CC C C . Cum O este punct pe

această axă radicală rezultă că axa radicală a cercurilor ' ' ', ,A B CC C C este dreapta lui Euler

OG a triunghiului ABC.

Consecinţe: 5) Dacă un triunghi neechilateral ABC admite puncte izologice, atunci acestea aparţin dreptei lui Euler a triunghiului ABC. 6) Punctele izologice U şi 'U ale unui triunghi ABC neechilateral sunt punctele de intersecţie dintre dreapta lui Euler a triunghiului ABC şi – de exemplu – cercul '( )AC .

7) Un triunghi obtuzunghic ABC nu are puncte izologice. Demonstraţie. Un punct oarecare U de pe dreapta lui Euler a triunghiului ABC este bine

determinat de numărul real λ pentru care .GU GOλ= ⋅uuuur uuur

Aplicând relaţia lui Stewart în

triunghiul AGU rezultă: 2 2 2AG OU AU GO AO GU GO OU GU⋅ + ⋅ − ⋅ = ⋅ ⋅ , adică

2 2 2 2(1 ) (1 ) .AU AO AG GOλ λ λ λ= + − − − Din relaţia lui Leibnitz 2 2 2

2 2 2( )

9

a b cOG R

+ += −

rezultă: 2 2 2 2 29 (9 2 ) (3 2 ) (4 3 ),AU R k a k k aλ λ= − + − + − unde 2 2 2

.2

a b ck

+ += Punctul U va fi

izologic dacă există un număr pozitiv t pentru care: ,UA UB UC

ta b c= = = de

unde rezultă: 2 2 2 2 29 (3 3) (9 2 ) 2 4a t a R k k kλ λ λ= − + − − + şi de aici: 2

2 2

3 1

(9 2 ) 2 4 0 ( )

t

R k k k

λ

λ λ

= −

− − + = ∗. Ecuaţia ( )∗ are rădăcini reale dacă şi numai dacă

236 ( 4 ) 0 ( )k k R∆ = − ≥ ∗∗ , adică 2 2 24 4R k p r Rr≤ = − − sau 2 2 24 4R r Rr p+ + ≤ , deci 2 2(2 )R r p+ ≤ , de unde 2R r p+ ≤ , condiţie care nu este adevărată pentru triunghiurile

obtuzunghice. 8) Un triunghi dreptunghic are un singur punct izologic. Demonstraţie. Fie că ( ) 90m BAC = ° . Atunci 2 2 2a b c= + şi 2 .R a= Astfel,

2 2 2 222

2 2

a b c ak a

+ += = = şi din relaţia (**) din problema precedentă rezultă 0,∆ =

adică triunghiul dreptunghic ABC are un singur punct izologic.

Page 86: Barbu_TEOREME_FUNDAMENTALE(2008).pdf

93

Consecinţă: Din ecuaţia 2 2(9 2 ) 2 4 0R k k kλ λ− − + = rezultă 4,λ = deci punctul izologic

U este bine determinat de relaţia 4 .GU GO= ⋅uuuur uuur

9) Un triunghi neechilateral ascuţitunghic admite două puncte izologice U şi 'U , iar 1 1 1

.' 2GU GU GO

+ =

Demonstraţie. Deoarece într-un triunghi ascuţitunghic 2R r p+ < rezultă că ecuaţia 2 2(9 2 ) 2 4 0R k k kλ λ− − + = admite două rădăcini 1λ şi 2λ , iar din relaţia 23 1t λ= −

rezultă că 21 3 ,tλ = + adică rădăcinile ecuaţiei sunt supraunitare. Atunci, 1GU GOλ= şi

2'GU GOλ= , relaţia de demonstrat devenind: 1 2

1 1 1

2λ λ+ = echivalentă cu

( )1 2 1 22 λ λ λ λ+ = ⋅ , adică 2 2

2 42

9 2 9 2

k k

R k R k⋅ =

− − relaţie evidentă.

10) Punctele izologice şi izodinamice ale unui triunghi neechilateral ascuţitunghic sunt conciclice. Demonstraţie. Fie S şi 'S punctele izodinamice, iar U şi 'U punctele izologice ale

triunghiului ABC de centru O. Atunci, 2' 'OS OS OU OU R⋅ = ⋅ = , deci'

'

OS OU

OU OS= şi cum

' 'SOU S OU≡ rezultă că triunghiurile SOU şi ' 'U OS sunt asemenea. Atunci, ( ) ( ' ')m OSU m OU S= , deci patrulaterul ' 'SUU S este inscriptibil.

11) Consecinţă: Dreptele SU şi ' 'S U sunt antiparalele.

Observaţie: Dacă triunghiul ABC este echilateral atunci S U O≡ ≡ şi 'S şi 'U sunt „aruncate la infinit”.

I.18. Retrocentrul unui triunghi „Matematica este o formă de poezie care transcende poezia prin aceea că proclamă adevărul.” - Solomon Bochner28

Retrocentrul (R) al unui triunghi ABC este punctul izotomic al ortocentrului H al triunghiului ABC. 1) Retrocentrul unui triunghi ABC este punctul lui Lemoine al triunghiului anticomplementar al triunghiului ABC. Demonstraţie. Fie ' ' 'A B C triunghiul anticomplementar al triunghiului ABC,

a b cH H H

triunghiul ortic al triunghiului ABC şi " " "A B C triunghiul ortic al triunghiului anticomplementar

' ' 'A B C . Fie ' " .= ∩D A A BC Din congruenţa

28 Solomon Bochner (1899-1982) – matematician polonez, profesor universitar la Princeton

A

B C

A'

B' C'

E

D

R

aH

A"

Fig. 86

Page 87: Barbu_TEOREME_FUNDAMENTALE(2008).pdf

94

triunghiurilor dreptunghice 'BDA şi a

AH C ( ' , ' )≡ ≡ a

BA AC DA B H AC rezultă

,a

BD H C≡ adică punctele a

H şi D sunt izotomice pe BC, deci retrocentrul R al

triunghiului ABC aparţine dreptei AD. Cum BC este linie mijlocie în triunghiul anticomplementar rezultă că D este mijlocul înălţimii ' ",A A deci în triunghiul ' ' ',A B C AD este o dreaptă Schwatt. Analog se arată că retrocentrul (R) al triunghiului aparţine şi celorlalte drepte Schwatt şi deoarece dreptele Schwatt ale unui triunghi sunt concurente în punctul lui Lemoine al triunghiului, rezultă că retrocentrul triunghiului ABC coincide cu punctul lui Lemoine al triunghiului anticomplementar. 2) Triunghiul pedal al retrocentului R al unui triunghi ABC este triunghiul podar al punctului lui Longchamps corespunzător triunghiului ABC. Demonstraţie. Punctul lui Longchamps al triunghiului ABC este ortocentrul triunghiului anticomplementar ' ' 'A B C al triunghiului ABC. Fie , ,

a b cR R R punctele de intersecţie

dintre înălţimile triunghiului anticomplementar cu laturile triunghiului ABC. Deoarece

' 'BC B C rezultă a

LR BC⊥ şi analog,

bLR AC⊥ şi ,

cLR AB⊥ deci

a b cR R R este

triunghiul podar al punctului lui Longchamps. Din proprietatea precedentă rezultă că dreptele

,a b

AR BR şi c

CR se intersectează în

retrocentrul triunghiului ABC, deci a b cR R R este triunghiul pedal al lui R.

3) Coordonatele baricentrice ale retrocentului R al unui triunghi ABC sunt: ( , , ).ctgA ctgB ctgC

Demonstraţie. Coordonatele baricentrice relative ale ortocentrului H al triunghiului ABC sunt ( , , )tgA tgB tgC şi cum retrocentrul R este punctul izotomic al ortocentrului triunghiului

ABC rezultă că R are coordonatele baricentrice relative 1 1 1

, ,tgA tgB tgC

adică

( , , ).ctgA ctgB ctgC

4) Punctul lui Gergonne, punctul lui Nagel şi retrocentrul triunghiului ABC sunt coliniare. Demonstraţie. Fie a,b,c lungimile laturilor triunghiului ABC şi p semiperimetrul său. Coordonatele baricentrice relative ale punctului lui Nagel sunt ( , , )− − −N p a p b p c , iar

ale izotomicului său – punctul lui Gergonne – sunt 1 1 1

, , .

Γ − − − p a p b p cCondiţia de

A

B C

A'

B' C'

R L

aR

bR cR

Fig. 87

Page 88: Barbu_TEOREME_FUNDAMENTALE(2008).pdf

95

coliniaritate a punctelor ,Γ N şi R este echivalentă cu

- - -

1 1 1 0

- - -

p a p b p c

p a p b p c

ctgA ctgB ctgC

= (egalitate

adevărată utilizând relaţiile [ ] [ ] [ ]

2 2 2 2 2 2 2 2 2

, ,4 4 4

ABC ABC ABC

b c a c a b a b cctgA ctgB ctgC

A A A

+ − + − + −= = = ).

I.19. Punctul anti-Steiner29

”Nous voyons expérience qu’entre esprits égaux et toutes choses pareilles, celui qui a de la Géométrie l’emporte et acqulert une vigueur toute nouveile” - Blaise Pascal30

Teorema lui Collings Fie o dreptă d ce conţine ortocentrul H al triunghiului ABC. Simetricele dreptei d faţă de laturile triunghiului ABC sunt concurente într-un punct de pe cercul circumscris triunghiului. Demonstraţie. Fie , ,

a b cd d d

simetricele dreptei d faţă de laturile BC, CA, respectiv AB, 1 2 3, ,D D D

punctele de intersecţie dintre dreapta d şi laturile BC, CA,

respectiv AB, , ,h h hA B C simetricele

ortocentrului H faţă de laturile triunghiului (Fig. 88). Notăm cu

a bd dΣ = ∩ ,

bE d BC= ∩ ,

1( )m DCα = Σ ,

2( )h

m B D Aβ = şi

( ).h h

m A Bγ = Σ Punctele , ,h h hA B C

aparţin cercului circumscris triunghiului ABC (vezi „Ortocentrul unui triunghi”). Evident, punctele 1 2 3, ,D D D aparţin

dreptelor BC, CA, respectiv AB.

Avem, ( ) 2 ( ) 2[90 ( )]h h h h

m A AB m CAB m HB A= = °− = 2[90 ( )] 180 2 ( )m ACB m ACB° − = ° −

(1). Deoarece unghiul 1 2 hD D B este exterior triunghiului 1 2D D Σ , rezultă 2 2γ α β+ =

(triunghiurile 1hHA D şi 2h

HB D fiind isoscele), deci

2 12( ) 2[180 ( ) ( )] [180 ( )]m C m D EC m DEγ β α γ= − = °− − − °− − Σ de unde 2 2 ( )m Cγ γ= − , deci

2 ( )m Cγ = (2). Din realţiile (1) şi (2) rezultă ( ) ( ) 180h h h h

m A AB m A B+ Σ = ° , deci punctul Σ aparţine cercului circumscris triunghiului ABC. Analog se arată că punctul Σ aparţine şi

29Jakob Steiner (1796-1863) – matematician elveţian, profesor la Universitatea din Berlin, contribuţii importante în geometrie 30 Blaise Pascal (1623-1662) – matematician francez, contribuţii importante în toate ramurile matematicii

A

B C hA

hB

Fig. 88

d

H

Σ

hC

ad

bd

cd

1D

2D

3D

Page 89: Barbu_TEOREME_FUNDAMENTALE(2008).pdf

96

dreptei cd , deci dreptele , ,

a b cd d d se intersectează într-un punct de pe cercul circumscris

triunghiului ABC.

Observaţii:

1) Punctul de concurenţă al dreptelor , ,a b cd d d se numeşte punctul anti-Steiner31

corespunzător dreptei d. 2) Orice dreaptă ce trece prin ortocentrul triunghiului ABC admite un punct anti-Steiner. 1) Consecinţă: Simetricele dreptei lui Euler a unui triunghi ABC în raport cu laturile triunghiului ABC sunt concurente într-un punct ce aparţine cercului circumscris triunghiului ABC. Demonstraţia rezultă din teorema de mai sus, o a doua demonstraţie este prezentată în subcapitolul „Dreapta lui Euler”. 2) Dreapta lui Steiner a punctului Σ este dreapta d. Demonstraţia este evidentă ţinând cont de definiţia dreptei lui Steiner. 3) Punctul anti-Steiner al înălţimii din A a triunghiului ABC este vârful A. Demonstraţia este evidentă deoarece simetricele înălţimii din A faţă de laturile triunghiului trec prin punctul A.

4) Fie P un punct pe o dreaptă d ce trece prin ortocentrul H al unui triunghi ABC, iar

1 2 3, ,P P P simetricele punctului P faţă de laturile BC, CA, respectiv AB. Punctul anti-

Steiner al dreptei d aparţine cercurilor circumscrise patrulaterelor 2 3 3 1 1 2, ,AP P BP P CPP .

Demonstraţie. Din teorema lui Collings rezultă că ( ) 2 ( ).h h

m A B m CΣ = Deoarece

2 1 2 1( ) ( ) ( )m P CP m P CP m PCP= + = 2[ ( ) ( )] 2 ( )m PCA m PCB m C+ = , de unde

31 Denumirea a fost dată de matematicianul german Darij Grinberg

A

B C hA

hB

Fig. 89

d

H

Σ

hC ad

bd

cd

1P

2P

3P P

Page 90: Barbu_TEOREME_FUNDAMENTALE(2008).pdf

97

2 1( ) ( )

h hm A B m PCPΣ = , adică patrulaterul 1 2CP PΣ este inscriptibil, deci punctul Σ aparţine

cercului circumscris triunghiului 1 2CPP . Analog, se arată că punctul Σ aparţine cercurilor

circumscrise triunghiurilor 2 3AP P şi 3 1BP P .

I.20. Punctul lui Bevan „Dacă mă simt nefericit, rezolv o problemă de matematică pentru a deveni fericit ... dacă sunt fericit, atunci compun o problemă de matematică pentru a mă menţine fericit.” - Turan

Fie , ,a b cI I I centrele cercurilor A,B,C - exînscrise corespunzătoare triunghiului ABC şi

a b cI I I triunghiul antisuplementar corespunzător triunghiului ABC . Cercul circumscris

triunghiului a b cI I I se numeşte cercul lui Bevan. Centrul cercului circumscris triunghiului

a b cI I I se numeşte punctul lui Bevan (V).

1) Perpendicularele duse din punctele , ,

a b cI I I pe laturile , ,BC CA respectiv AB ale

triunghiului ABC sunt concurente în punctul lui Bevan. Demonstraţie.

Ib V

Ia

Ic

A

B

C

Fig. 90

bτ cτ

Page 91: Barbu_TEOREME_FUNDAMENTALE(2008).pdf

98

Triunghiul ABC este triunghiul ortic al triunghiului exînscris (vezi “Cercurile exînscrise”). Deoarece perpendicularele duse din vârfurile unui triunghi XYZ pe laturile triunghiului ortic corespunzător sunt concurente în centrul cercului exînscris triunghiului XYZ, atunci

perpendicularele duse din centrele cercurilor exînscrise , ,a b cI I I pe laturile ,BC CA

respectiv AB sunt concurente în centrul cercului circumscris triunghiului a b cI I I , adică în

punctul lui Bevan. 2) Consecinţă: Triunghiul podar al punctului lui Bevan al triunghiului ABC este triunghiul cotangentic a b cτ τ τ al triunghiului ABC.

3) Punctul lui Bevan este centrul de omologie între triunghiul cotangentic a b cτ τ τ al

triunghiului ABC şi triunghiul antisuplementar a b cI I I al triunghiului ABC.

Demonstraţie. Avem a a b b c cI I I Vτ τ τ =I I şi conform teoremei lui Desargues rezultă că

triunghiurile a b cτ τ τ şi a b cI I I sunt omologice.

4) Fie I centrul cercului înscris în triunghiul ABC şi O centrul cercului circumscris triunghiului ABC . Punctele I, O şi V sunt coliniare.

Demonstraţie. Punctul I este ortocentrul triunghiului antisuplementar a b cI I I , V centrul

cercului circumscris triunghiului, iar O este centrul cercului Euler al triunghiului a b cI I I , deci punctele I,

O şi V sunt coliniare, ele aparţinând dreptei lui Euler a triunghiului

a b cI I I (Fig. 91).

5) Consecinţă: Segmentele IO şi OV sunt congruente, deoarece centrul cercului lui Euler este mijlocul segmentului determinat de ortocentru, respectiv centrul cercului circumscris unui triunghi.

6) Punctul lui Bevan este centrul cercului circumscris triunghiului antisuplementar

a b cI I I corespunzător triunghiului ABC. Demonstraţie. Centrul cercului înscris (I) în triunghiul ABC este ortocentrul triunghiului

a b cI I I . Punctul lui Bevan (V) al triunghiului ABC este simetricul lui I faţă de centrul

cercului circumscris triunghiului ABC (centrul cercului lui Euler al triunghiului a b cI I I ),

deci V este centrul cercului circumscris triunghiului antisuplementar a b cI I I .

A

B

C

aI

bI cI

I

Fig. 91

V

O

Page 92: Barbu_TEOREME_FUNDAMENTALE(2008).pdf

99

7) Lungimea segmentului OV este egală cu 2 abcR

a b c−

+ +, unde R este raza cercului

circumscris triunghiului ABC , iar a, b, c sunt lungimile laturilor triunghiului ABC. Demonstraţie. Din relaţia lui Euler pentru triunghiul ABC avem: 2 2 2 ,OI R Rr= −

2 2 24 ABC

abcOI R r

A

= − ⋅ =⋅

2 abcR

a b c−

+ +, de unde 2 abc

OI OV Ra b c

= = −+ +

(unde r este

raza cercului înscris în triunghiul ABC şi 2

a b cp

+ += ).

Observaţie: Anticomplementul punctului lui Bevan se numeşte punctul lui Longuet –

Higgins (Lo), deci 2oL G GV=uuuur uuur

.

8) Complementul V ∗ al punctului lui Bevan al triunghiului ABC este mijlocul segmentului IH, unde I este centrul cercului înscris în triunghiul ABC şi H ortocentrul triunghiului ABC. Demonstraţie. Fie .V VG IH

∗ = I Deoarece 2HG GO= şi 2p

IG GS= ( unde pS este

punctul lui Spieker al triunghiului ABC) rezultă că G este centrul de greutate al triunghiului

IVH, deci VV ∗ este mediană, de unde 2VG GV ∗= , relaţie ce arată că V ∗ - mijlocul segmentului IH – este complementul punctului lui Bevan 9) Raza cercului lui Bevan este egală cu 2R, unde R este raza cercului circumscris triunghiului ABC .

Demonstraţie. Fie VR raza cercului Bevan şi ', ', 'a b c lungimile laturilor triunghiului

a b cI I I . Deoarece 1 1( ) 90 ( ), ( ) 90 ( )

2 2a bm BI C m A m CI A m B= °− = °− şi 1

( ) 90 ( )2c

m AI B m C= ° −

(vezi„Triunghiul antisuplementar”), iar triunghiul ABC este triunghiul ortic al triunghiului

a b cI I I , rezultă 1'cos 'cos(90 ( )) 'sin

2 2a

Aa a BI C a m A a= = ° − = , de unde '

sin2

aa

A= .

Analog, ' ,sin

2

bb

B= şi '

sin2

cc

C= . Atunci,

[ ]

' ' '

4V

I I Ia b c

a b cR

A= =

[ ]

224sin sin sin ( )

2 2 2ABC

abc abcR

A B C AR a b c

= =⋅⋅ + +

(unde am utilizat formulele:

( )[ ] [ ], 4 sin sin sin2 2 2I I I ABCa b c

A B CA R a b c A Rp= + + = şi

[ ]4ABC

abcR

A= ).

10) Punctul lui Bevan (V) al triunghiului ABC şi I centrul cercului înscris triunghiului ABC se află la aceeaşi distanţă faţă de dreapta lui Euler a triunghiului ABC .

Demonstraţie. Dreapta lui Euler a triunghiului ABC trece prin centrul circumscris O al

triunghiului ABC , iar cum V şi I sunt egal depărtate de O, rezultă că V şi I se află la aceeaşi distanţă faţă de dreapta lui Euler a triunghiului ABC .

Page 93: Barbu_TEOREME_FUNDAMENTALE(2008).pdf

100

11) Punctul lui Nagel (N), Longchamps (L) şi Bevan (V) ale triunghiului ABC sunt

coliniare şi NV VL≡ . Demonstraţie. Fie H, G, I, O ortocentrul, centrul de greutate, centrul cercului înscris, respectiv centrul cercului circumscris triunghiului ABC (Fig. 92). Avem HN OI şi

2HN OI= , V este simetricul lui I faţă de O, iar L este simetricul ortocentrului H al triunghiului ABC

faţă de O. Avem 3 1 4

, ,2 2 3

N I VO LG

NG V I LO= = = ,

de unde 1NI VO LG

NG VI LO⋅ ⋅ = şi din reciproca teoremei

lui Menelaus aplicată în triunghiul IGO rezultă că punctele L, V şi N sunt coliniare. Mai mult, deoarece OI HN rezultă OV HN şi cum O este mijlocul segmentului HL rezultă că V este mijlocul segmentului LN, deci LV VN≡ . Consecinţă: 2HN OV IV= = . 12) Ortocentru H, punctul lui Spieker Sp, punctul lui Bevan V ale unui triunghi ABC sunt coliniare şi p pHS S V≡ .

Demonstraţie. Punctul lui Spieker este coliniar cu I şi N şi p p aIS S N≡ . Avem 2 1

3, ,3 2

p

p

S I HG VO

S G HO VI= = = ,de unde 1

p

p

S I HG VO

S G HO VI⋅ ⋅ = şi din reciproca teoremei lui

Menelaus aplicată în triunghiul IGO rezultă că punctele H, Sp şi V sunt coliniare. Deoarece IV HN şi p pIS S N≡ rezultă p pHS S V≡ . Consecinţă: Triunghiurile p aHS N şi pVS I sunt congruente.

13) Paralelele duse prin punctul lui Bevan al triunghiului ABC la bisectoarele interioare ale unghiurilor triunghiului ABC intersectează laturile opuse în punctele

', ', 'A B C . Dreptele ', ', 'AA BB CC sunt concurente. Demonstraţie. Deoarece V este centrul cercului circumscris triunghiului antisuplementar

a b cI I I , iar dreptele care unesc vârfurile triunghiului ortic respectiv cu punctele de

intersecţie dintre mediatoarele laturilor triunghiului de referinţă sunt concurente ( vezi “Triunghiul ortic”) rezultă concluzia.

L

V

N

O

G

Sp

I

H

V∗

Fig. 92

Page 94: Barbu_TEOREME_FUNDAMENTALE(2008).pdf

101

I.21. Punctul lui Exeter

„Unu şi cu unu nu fac doi, Unu şi cu unu fac trei sau patru, sau cinci...

Matematica s-o fi scriind cu cifre dar pozia nu se scrie cu cuvinte.” Nichita Stănescu

Fie A B CT T T triunghiul tangenţial corespunzător triunghiului ABC şi ', ', 'A B C punctele în care medianele duse din vârfurile A,B,C intersectează cercul circumscris triunghiului ABC. Dreptele ', ', 'A B CT A T B T C sunt concurente.

Demonstraţie. Notăm cu 1 2( '), ( '), ( '), ( ')

A Am BT A m CT A x m BAA y m CAAα α= = = = şi fie

aM mijlocul laturii BC.Avem: ' ' '

ABAA BCA A BT≡ ≡ şi ' ' '

ACAA CBA A CT≡ ≡ . Din

teorema sinusurilor în triunghiurile 'A

BA T şi

'A

CA T rezultă: 1sin sin

' 'A

x

BA A T

α= şi

2sin sin

' 'A

y

CA A T

α= , de unde 1

2

sin sin '

sin sin '

x BA

y CA

αα

= ⋅

(1) (Fig. 93). Din teorema sinusurilor în triunghiurile '

aBA M şi '

aCA M rezultă:

sin ' sin

' 'a

a

BM A y

BA A M= şi

sin ' sin

' 'a

a

CM A x

CA A M=

de unde ' sin

' sin

BA x

CA y= (2) (unde

s – a folosit faptul că sin ' sin(180 ') sin( ')

a a aCM A BM A BM A= ° − = ). Din relaţiile (1) şi (2) rezultă

2

1

2

sin sin.

sin sin

x

y

αα

=

Notăm 1 2( '), ( '),

B Bm CT B m AT Bβ β= =

1 ( '),C

m AT Cγ =

2 ( '),

Cm BT Cγ = ( ' ), ( ' ),z m B BC t m B BA= = ( ' ),u m C CA= ( ' )v m C CB= . Analog se arată

că 2

1

2

sin sin

sin sin

z

t

ββ

=

şi 2

1

2

sin sin

sin sin

u

v

γγ

=

. Din forma trigonometrică a teoremei lui Ceva

avem că sin sin sin

1sin sin sin

x z u

y t v⋅ ⋅ = (datorită concurenţei medianelor); atunci,

1 1 1

2 2 2

sin sin sin

sin sin sin

α β γα β γ

⋅ ⋅ = 2

sin sin sin1

sin sin sin

x z u

y t v

⋅ ⋅ =

şi din reciproca teoremei lui Ceva rezultă

că dreptele ', ', 'A B CT A T B T C sunt concurente într-un punct X.

Observaţie: Punctul X de concurenţă al dreptelor ', ', 'A B CT A T B T C se numeşte punctul lui

Exeter.

A

C

C'

X

AT

BT CT

Fig. 93

B

A'

B'

aM

Page 95: Barbu_TEOREME_FUNDAMENTALE(2008).pdf

102

1) Triunghiul tangenţial al unui triunghi ABC şi triunghiul circumpedal al centrului de greutate al triunghiului ABC sunt omologice. Demonstraţia rezultă din teorema de mai sus. 2) Consecinţă: Punctul lui Exeter al triunghiului ABC este centrul de omologie dintre triunghiul tangenţial şi triunghiul circumpedal al centrului de greutate al triunghiului ABC.

3) Coordonatele baricentrice ale punctului lui Exeter sunt

2 4 4 4 2 4 4 4 2 4 4 4( ( ), ( ), ( ))X a b c a b c a b c a b c+ − + − + − (vezi [26]).

4) Punctul lui Exeter al triunghiului ABC aparţine dreptei lui Euler a triunghiului ABC. Demonstraţie. Vom demonstra teorema utilizând coordonatele baricentrice; astfel,

(1,1,1)G , (sin 2 ,sin 2 ,sin 2 )O A B C şi 2 4 4 4 2 4 4 4 2 4 4 4( ( ), ( ), ( ))X a b c a b c a b c a b c+ − + − + − .

Deoarece

2 4 4 4 2 4 4 4 2 4 4 4( ) ( ) ( )

1 1 1 0

sin 2 sin 2 sin 2

a b c a b c a b c a b c

A B C

+ − + − + −

= , rezultă că punctele G,O

şi X sunt coliniare (am ţinut cont că 2 2 2

sin 2 2sin cos 22 2

a b c aA A A

R bc

+ −= = ⋅ ⋅ ).

I.22. Punctul lui Gob „Nu te poţi rupe în două ci numai în trei,

nu ocolirea ci ruptura închide. Triunghiul, vă zic dragii mei,

e izbăvirea unei oglinde.” Nichita Stănescu32

1) Triunghiurile ortic şi tangenţial corespunzătoare unui triunghi ABC sunt omotetice. Demonstraţie: vezi „Triunghiul tangenţial”. Centrul de omotetie dintre triunghiurile ortic şi tangenţial ale unui triunghi ABC se numeşte punctul lui Gob ( )Φ corespunzător triunghiului ABC.

2) Punctul lui Gob al triunghiului ABC aparţine dreptei lui Euler a triunghiului ABC. Demonstraţie. Prin omotetia triunghiurilor ortic şi tangenţial, rezultă că centrele cercurilor circumscrise acestor două triunghiuri se corespund; deci, centrul cercului lui Euler 9( )O al triunghiului ABC, centrul

cercului circumscris triunghiului tangenţial ( )T

O şi

punctul lui Gob ( )Φ sunt coliniare. Deoarece

32Nichita Stănescu (1933 – 1983) – eseist, poet român, ales postum membru al Academiei Române

A

C aH

bH

Φ

AT

BT CT

Fig. 94

B

cH

Page 96: Barbu_TEOREME_FUNDAMENTALE(2008).pdf

103

punctele 9O şi T

O aparţin dreptei lui Euler a triunghiului ABC (vezi „Triunghiul

tangenţial”), rezultă că şi punctul lui Gob aparţine dreptei lui Euler. 3) Coordonatele baricentrice ale punctului lui Gob al unui triunghi ABC sunt:

2 2 2

2 2 2 2 2 2 2 2 2, ,

a b c

b c a c a b a b c

Φ

+ − + − + − .

Demonstraţie. vezi [ 26].

I.23. Punctul lui Gray. Triunghiul lui Gray33

„Tot ce e gândire corectă este sau matematică sau susceptibilă de matematizare.” Grigore Moisil34

1) Fie X, Y, Z simetricele centrului cercului înscris în triunghiul ABC faţă de laturile BC, CA , respectiv AB. Dreptele AX, BY şi CZ sunt concurente. Demonstraţie. Fie DEF triunghiul ortic al triunghiului 1 2 3.I I I Deoarece dreptele

1 2 3, ,AI BI CI , respectiv 1 2 3, ,I D I E I F sunt

concurente, atunci din teorema lui Dottl rezultă că dreptele AD, BE şi CF sunt concurente într-un punct U (Fig. 95). Deoarece AD, BE şi CF sunt izogonalele dreptelor AX, BY respectiv CZ – vezi „Triunghiul I cevian” – rezultă că dreptele AX, BY şi CZ sunt concurente în punctul izogonal conjugat al lui U.

Observaţie: Punctul de concurenţă al dreptelor AX, BY şi CZ se numeşte punctul lui Gray (J) al triunghiului ABC, iar XYZ se numeşte triunghiul lui Gray corespunzător triunghiului ABC. 2) Punctul lui Gray al triunghiului ABC este coliniar cu centrul cercului înscris (I) în triunghiul ABC şi cu ortocentrul triunghiului I – cevian. Demonstraţie. Fie J punctul lui Gray al triunghiului ABC, 1 2 3I I I triunghiul I cevian, DEF

triunghiul ortic al triunghiului 1 2 3I I I , 'H ortocentrul triunghiului 1 2 3I I I , X, Y, Z

simetricele lui I faţă de laturile BC, CA respectiv AB şi 1 1 1A B C triunghiul J – cevian în

raport cu triunghiul ABC. Triunghiurile DEF şi 1 1 1A B C sunt omologice, I fiind centrul de

omologie (vezi „Triunghiul I - cevian”). Deoarece triunghiurile ABC şi 1 2 3I I I sunt

omologice , I fiind centrul omologiei, atunci conform teoremei (vezi „Triunghiuri omologice”) rezultă că dreptele 1 2 3, ,I D I E I F şi IJ sunt concurente, deci ' .H IJ∈

33 Andrew Gray (1847-1925) – matematician scoţian, profesor la Universitatea din Glasgow 34 Grigore Moisil (1906-1973) – matematician român, profesor la Universitatea din Iaşi, membru al Academiei

Române, contribuţii importante în informatică

A

B C

I

1I

2I 3I

Fig. 95

X

E

Y

F Z

D

J

Page 97: Barbu_TEOREME_FUNDAMENTALE(2008).pdf

104

3) Fie XYZ triunghiul Gray corespunzător triunghiului ABC, 1 2 3I I I triunghiul I – cevian

corespunzător triunghiului ABC, a b c

H H H triunghiul său ortic,

1 3 1 , , , .b a b

M XY BC M YZ AB A H I BC C AB H H= ∩ = ∩ = ∩ = ∩ Bisectoarea

interioară a unghiului 3 1I A B intersectează latura AB în "N , iar bisectoarea interioară a

unghiului 1 aBC H intersectează latura BC în ".M Dreptele MN şi " "M N sunt paralele.

Demonstraţie. Deoarece a

BH este bisectoarea

exterioară a unghiului 1 3aC H I rezultă că "M

este centrul cercului înscris în triughiul

1 a cC H H rezultă "

cH M este bisectoarea

unghiului 1 .c a

C H H Rezultă că 3 "I M BC⊥

(vezi „Triunghiul ortic”) şi 1 " ,I N AB⊥ adică

patrulaterul 1 3" "N M I I este inscriptibil (Fig.

96). Patrulaterul 3 1MNI I este inscriptibil

(conform consecintei precedente). Cercurile circumscrise patrulaterelor 1 3" "N M I I

şi 3 1MNI I se intersectează în punctele 1I şi 3I ,

deci " "MN M N (conform teoremei lui Reim – demonstrată mai jos).

Teorema lui Reim Fie C, D punctele de intersecţie dintre cercurile

1C şi

2C . Fie A,B∈

1C şi E,F∈

2C astfel încât

punctele A, C, F şi B, D, E sunt coliniare. Dreptele AB şi EF sunt paralele. Demonstraţie. Fie T AF∈ astfel încât

[ ]F CT∈ (Fig. 97). Avem

( ) ( )m BDC m EFC= de unde rezultă că:

( ) ( ) 180 ( ),m BAC m EFT m BDC= = ° − deci

.EF AB 4) Fie 1 1" " .A N C M β∩ = Punctele ,B β şi Y sunt coliniare.

Demonstraţie. Dreptele 1 "A N şi AI sunt perpendiculare fiind bisectoarele unghiurilor

1bH AC şi respectiv BAC (patrulaterul

c bBH H C fiind inscriptibil). Cum AI ZY⊥

rezultă 1 "A N ZY şi analog 1 " .C M XY Conform proprietăţii precedente " "MN M N şi

cum " " MM NN B∩ = rezultă că triunghiurile " "N M β şi NMY sunt omotetice (centrul

de omotetie fiind punctul B), deci punctele ,B β şi Y sunt coliniare.

5) Fie XZY triunghiul lui Gray al triunghiului ABC, .M XY BC= ∩ Punctele M, C, Y şi centrul cercului (I) înscris în triunghiul ABC sunt conciclice.

A

B C

I

1I

3I

Fig. 96

X

M

Y

β Z

bH N

cH

N" M" 1A

1C

aH

A

B

C

D E

F T

Fig. 97

Page 98: Barbu_TEOREME_FUNDAMENTALE(2008).pdf

105

Demonstraţie. Deoarece 2IX IY r= = rezultă că IC XY⊥ ( XC YC CI≡ ≡ ), de unde rezultă (1)IYX ICY≡ (unghiuri cu laturile perpendiculare două câte două). Cum

ICY ICM≡ rezultă ,IYM ICM≡ deci patrulaterul IMCY este inscriptibil. Consecinţe: 1) Dreapta IX este tangentă cercului circumscris patrulaterului IMCY. Demonstraţie. Deoarece MIX IXM IYM≡ ≡ rezultă concluzia. 2) Dacă ,

aC IX BC= ∩ atunci din puterea unui punct faţă de un cerc rezultă

2a a a

C I C M C C= ⋅ sau de unde 2

a

rC M

p c=

− (relaţie care determină poziţia punctului M

pe latura BC). 6) Fie XYZ triunghiul lui Gray corespunzător unui triunghi ABC, I centrul cercului înscris în triunghiul ABC, 1 2 3I I I triunghiul I – cevian în raport cu triunghiul ABC, 'Y simetricul lui Y faţă de dreapta BI, ' ' , ' ' .M XY BC N ZY AB= ∩ = ∩ Dreptele

' 'M N şi 1 3I I sunt paralele. Demonstraţie.

Fie , M XY BC N ZY AB= ∩ = ∩

(Fig. 98). Din teorema bisectoarei 1

1

I B c

I C b= şi

3

3

,I B a

I A b= de unde 1 3, .

ab acBI BI

b c a b= =

+ + Avem: 2 ,

a ar C M C C= ⋅ sau

2 ' 'c a a c

r C N C C C M C A= ⋅ = ⋅ , deci 2

' =−a

rC M

p a. Dar ' '

a aBM BC C M= + =

2

( )r

p bp a

− + =−

( )( )− +p b b a

p. Analog,

( )( )'

− +=

p b b cBN

p şi de aici rezultă

1

3

'

'

BI BM

BI BN= şi din reciproca teoremei Thales rezultă 1 3 ' '.I I M N

Consecinţe: i) Patrulaterul ' 'MM NN este trapez isoscel. Demonstraţie. Deoarece punctele M şi 'N respectiv 'M şi N sunt simetrice faţă de BI, rezultă concluzia. ii) Punctele 3 1, , ,M N I I sunt conciclice.

Demonstraţie. Deoarece ' 'MM NN este trapez isoscel rezultă ' ' ' (1)≡ MM N MNN ;

din 1 3 ' 'I I M N rezultă 1 3 ' ' (2).BI I BM N≡ Din (1) şi (2) rezultă 1 3 3BI I MNI≡

adică patrulaterul 3 1MNI I este inscriptibil.

7) Fie XYZ triunghiul lui Gray corespunzător unui triunghi ABC. Triunghiurile XYZ şi ABC sunt bilogice, centrul de ortologie fiind centrul cercului înscris în triunghiul ABC. Demonstraţia este evidentă.

A

B C

I

1I M'

3I

Fig. 98

X

N'

Y

aC

Z

Y' N

M

Page 99: Barbu_TEOREME_FUNDAMENTALE(2008).pdf

106

Teorema lui Ayme 8) Fie

a b cH H H triunghiul ortic al triunghiului ABC şi 1 1 1A B C axa ortică a sa, ', ", '''b b b

bisectoarele interioare ale unghiurilor 1 1, ,

a cH B A H A B respectiv

1aH C A , iar

' ''', '' ''', ' ''.b b b b b bα β γ= ∩ = ∩ = ∩ Triunghiurile ABC şi αβγ sunt omologice, punctul lui Gray al triunghiului ABC fiind centrul omologiei. Demonstraţie. Deoarece , ,AX BY CZα β γ∈ ∈ ∈ - conform teoremei lui Casey (vezi

„Triunghiuri omologice”) – şi cum AX BY CZ J∩ ∩ = rezultă că A B C Jα β γ∩ ∩ = ,

unde J este punctul lui Gray al triunghiului ABC, deci triunghiurile ABC şi αβγ sunt

omologice, punctul lui Gray al triunghiului ABC fiind centrul omologiei. Observaţie: Din teorema precedentă rezultă că triunghiurile ABC, αβγ şi XYZ sunt

omologice, punctul lui Gray fiind centrul omologiei. 9) Fie XYZ triunghiul lui Gray corespunzător unui triunghi ABC,

a b cH H H triunghiul

ortic al triunghiului ABC. Axa de omologie dintre triunghiurile ABC şi XYZ este paralelă cu axa ortică a triunghiului ABC. Demonstraţie. Fie 1 1 ,

b c a cA H H BC B H H AC= ∩ = ∩ şi 1 ;

a bC H H AB= ∩ dreapta

1 1 1A B C este axa ortică a triunghiului ABC (vezi „Axa ortică”). Deoarece

AX BY CZ J∩ ∩ = (J – punctul lui Gray) atunci triunghiurile ABC şi XYZ sunt

omologice, fie d axa lor de omologie. Fie 1 2 1 2 1 2, , ,A A YZ B B ZX C C XY

3 1 2 1 2 3 1 2 1 2 3 1 2 1 2 , , .A B B CC B A A CC C AA B B= ∩ = ∩ = ∩ Din teorema lui Ayme rezultă că

triunghiurile ABC, XYZ şi 3 3 3A B C sunt omologice, centrul de omologie fiind punctul lui

Gray (J) al triunghiului ABC. Axa de omologie dintre triunghiurile ABC şi 3 3 3A B C este axa

ortică a triunghiului ABC şi conform teoremei lui Casey rezultă că dreapta 1 1.d A B

Teorema lui Gray 10) Fie J punctul lui Gray corespunzător unui triunghi ABC şi I centrul cercului înscris în triunghiul ABC. Dreapta lui Euler a triunghiului ABC este paralelă cu dreapta IJ. Demonstraţie. Punctul I este centrul de ortologie al triunghiurilor ABC şi XYZ şi J este centrul de omologie dintre aceste triunghiuri. Din teorema lui Sondat rezultă IJ d⊥ (unde d este axa de omologie dintre triunghiurile ABC şi XYZ), cum 1 1d A B (unde 1 1A B este axa

ortică a triunghiului ABC) rezultă 1 1.IJ A B⊥ Deoarece dreapta lui Euler a triunghiului ABC

este perpendiculară pe axa ortică a triunghiului ABC (vezi „Axa ortică”) rezultă că dreapta lui Euler este paralelă cu dreapta IJ.

Observaţie: Dreapta IJ se numeşte dreapta lui Gray.

Page 100: Barbu_TEOREME_FUNDAMENTALE(2008).pdf

107

I.24. Punctul lui Hexyl

„Matematica este calea de înţelegere a Universului.”- Pitagora35

1) Fie , ,a b c

O O O simetricele centrului cercului circumscris (O) al unui triunghi ABC faţă

de laturile BC, CA respectiv AB şi a b c

H H H triunghiul ortic al triunghiului ABC. Dreptele

, ,a a b b c c

O H O H O H sunt concurente.

Demonstraţie. Dacă a

M este mijlocul lui BC, atunci

2a

OM AH= (vezi „Triunghiul ortic”), deci ;=a

AH OO

cum AH BC⊥ şi ⊥a

OO BC rezultă ≡a

AH OO adică

patrulaterul a

AHO O este paralelogram. Mijlocul

diagonalei a

AO este 9O - mijlocul segmentului [ ].OH

Fie ', ', 'A B C simetricele punctelor ,a b

H H respectiv c

H

faţă de centrul cercului lui Euler 9( )O al triunghiului

ABC. Patrulaterul 'a

AHO A este paralelogram (deoarece

9O este mijlocul diagonalelor a

AO şi 'a

H A ), deci

' .a a

AA H O Analog, se arată că ' b b

BB H O şi

' .c c

CC H O Deoarece dreptele ', ', 'AA BB CC sunt

concurente în punctul lui Prasolov (vezi „Punctul lui Prasolov”) rezultă că şi simetricele acestora faţă de centrul lui Euler sunt concurente.

Observaţie: Punctul de concurenţă al dreptelor , ,a a b b c c

O H O H O H se numeşte punctul lui

Hexyl ( )x

H al triunghiului ABC.

Consecinţă: 2) Punctul lui Hexyl este simetricul punctului lui Prasolov ( )

vP faţă de centrul cercului

triunghiului ABC.

Observaţie: Punctele 9,x

H O şi vP sunt coliniare şi 9 9 .

x xH O O P≡

3) Punctul lui Hexyl este ortocentrul triunghiului tangenţial al triunghiului ABC. Demonstraţie. Vezi „Triunghiul tangenţial”. 4) Punctul lui Hexyl al unui triunghi ABC este coliniar cu punctul lui Lemoine şi centrul cercului lui Euler al triunghiului ABC. Demonstraţie. Triunghiul tangenţial al triunghiului ABC este ortologic şi omologic cu triunghiul median al triunghiului ortic al triunghiului ABC, centrele de ortologie fiind punctul lui Hexyl ( )

xH şi centrul cercului lui Euler 9( )O al triunghiului ABC, centrul de

35 Pitagora (580-500 î .Hr.) – filosof şi matematician grec

A

B

A '

C

aO

bO cO

O H 9O

aM

cH bH

aH xH

Fig. 99

Page 101: Barbu_TEOREME_FUNDAMENTALE(2008).pdf

108

omologie fiind punctul lui Lemoine (K) al triunghiului ABC (vezi „Triunghiul tangenţial”). Conform teoremei lui Sondat punctele ,

xH K şi 9O sunt coliniare.

Observaţie: Din proprietatea precedentă rezultă că punctele 9, ,x

H K O şi vP sunt coliniare.

I.25. Punctul lui Prasolov

„Un matematician încearcă în munca sa aceeaşi plăcere ca şi un artist; plăcerea este tot atât de mare şi de aceeaşi natură.” – Henri Poincaré36

Fie ', ', 'A B C simetricele vârfurilor triunghiului ortic al unui triunghi ascuţitunghic ABC

faţă de centrul cercului lui Euler ( 9O ) al triunghiului ABC.

Teorema lui Prasolov

Triunghiurile ABC şi ' ' 'A B C sunt omologice. Demonstraţie. Fie

a b cH H H triunghiul ABC, 9O centrul cercului lui Euler al triunghiului

ABC şi " 'A AA BC= ∩ , " ' ,B BB AC= ∩ " 'C CC AB= ∩ (Fig. 100). Deoarece

patrulaterul 'a

H OA H este paralelogram şi a

HH BC⊥ , rezultă că 'A O BC⊥ - unde O este

centrul cercului circumscris triunghiului ABC – deci punctul 'A este situat pe mediatoarea segmentului BC. Fie ' , .

a aA D AH D AH⊥ ∈ Din asemănarea triunghiurilor 'ADA şi

"a

AH A rezultă'

"a a

A D AD

H A AH= , adică

cos cos2" sin

a

ac B

R A

H A c B

−= (deoarece '

a aA D H M≡ ), deci

(2 sin 2 2 sin cos ) sin sin" .

cosa

R A R C B C BH A

A

− ⋅= Atunci, " "

a aC A H A H C= − =

2cos(1 2sin )

cos

b BC

A⋅ −

sin 2 cos 2

cos

R B C

A= şi

[sin 2 cos 2 sin 2 ]" "

cos

R B C AA B CA BC

A

+= + = ,

de unde " sin2 cos2 sin2 sin(2 2 ) sin2 sin2 ( cos2 ) 2

" sin2 cos2 2sin2 cos2 sin2 cos2 2

A B B C A B C A C B tg C

A C B C B C B C tg B

+ − + ⋅ −= = = = − .

36 Henri Poincaré ( 1854 -1912) – matamatician şi fizician francez, contribuţii importante în toate ramurile matematicii

A

B C aH

A'

A"

O

H 9O

Fig. 100

D

aM

Page 102: Barbu_TEOREME_FUNDAMENTALE(2008).pdf

109

Analog, " 2

" 2

B C tg A

B A tg C= − şi

" 2

" 2

C A tg B

C B tg A= . Atunci,

" " "1

" " "

A B B C C A

A C B A C B⋅ ⋅ = şi din reciproca

teoremei lui Menelaus rezultă că punctele ", ", "A B C sunt coliniare, iar din teorema lui Desargues rezultă că triunghiurile ABC şi ' ' 'A B C sunt omologice. Observaţie: Teorema s-a demonstrat pentru cazul corespunzător figurii date mai sus, teorema rămâne adevărată pentru orice configuraţie a punctelor A, B, C (triunghiul ABC rămânând ascuţitunghic), calculele suferind unele modificări. Centrul de omologie al triunghiurilor ABC şi ' ' 'A B C se numeşte punctul lui Prasolov

vP .

2) Punctul lui Prasolov este simetricul punctului lui Hexyl faţă de centrul cercului lui Euler al triunghiului ABC. Demonstraţie: vezi „Punctul lui Hexyl”. 3) Punctul lui Prasolov, punctul lui Lemoine şi centrul cercului lui Euler al triunghiului ABC sunt coliniare. Demonstraţie: vezi „Punctul lui Hexyl”.

I.26. Punctul lui Kariya

„Noi venerăm Grecia antică drept leagăn al culturii, acolo lumea a asistat pentru prima oară la miracolul unui sistem logic în care paşii se succed cu o asemenea precizie încât propoziţiile lui apăreau ca absolul indubitabile – am în vedere geometria lui Euclid.” – Albert Einstein37

1) Fie ,,a b c

C C C punctele de contact cu laturile , ,BC CA AB ale cercului înscris

triunghiului ABC şi I centrul acestui cerc. Pe dreptele , ,a b c

IC IC IC se consideră în

acelaşi sens segmentele congruente ', ', 'IA IB IC . Să se arate că dreptele ', 'AA BB şi 'CC sunt concurente.

Demonstraţie.

37 Albert Einstein (1879-1955) – fizician german, profesor universitar la Berlin şi Princeton, laureat al Premiului Nobel

aC

1A

A'

B'C '

2A

A

M

I

Q

P

bCcC

B C

D

Fig. 101

Page 103: Barbu_TEOREME_FUNDAMENTALE(2008).pdf

110

Se proiectează punctul 'A în 1A pe AC şi în 2A pe AB ; punctul 'B în 1B pe BC şi în

2B pe AB; punctul 'C în 1C pe şi în 2C pe BC . Fie D punctul de întâlnire al paralelei

prin I la AC cu 1'A A (Fig. 101). Patrulaterul 1'a

A ACC este inscriptibil, din

1'

aC A A ACB≡ . Atunci 1 1' ' cos

a bA A C D DA IA C OC= + = + . Analog, se obţin relaţiile

2' 'cosc

A A IA B IC= + , 1' 'cosa

B B IB C IC= + , 2' 'cosc

B B IB A IC= + ,

1' 'cosa

C C IC B IC= + , 2' 'cosb

C C IC A IC= + . Cum a b c

IC IC IC r= = = şi

' ' 'IA IB IC= = rezultă 1 1' 'A A B B= , 2 1' 'A A C C= , 2 2' 'B B C C= (1). Dacă M este

punctul comun dreptelor 'BB şi 'CC şi fie P, Q, R proiecţiile lui M pe

laturile , ,BC CA AB avem: 1

2

'

'

C CMP

MQ C C= , 2

1

'

'

B BMR

MP B B= (2). Înmulţind egalităţile (1) şi (2)

rezultă: 1 2 1 2

2 1 1 1

' ' ' '

' ' ' '

C C B B C C A AMR

MQ C C B B B B A A

⋅= = =

⋅, ceea ce arată că 'M AA∈ .

Observaţie: Punctul de concurenţă al dreptelor ', 'AA BB şi 'CC este un punct al lui Kariya.

2) Fie ', ', 'A B C punctele unde bisectoarele interioare ale unghiurilor triunghiului ABC intersectează cercul circumscris şi ", ", "A B C simetricele centrului cercului circumscris O al triunghiului ABC faţă de laturile ' ', ' 'B C C A , respectiv ' '.A B Triunghiurile

" " "A B C şi ABC sunt omologice, centrul de omologie fiind un punct al lui Kariya al triunghiului ABC. Demonstraţie.

A

B C

I

O

A'

B'

C'

A" A '''≡

B"

C"

Fig. 102

P

Page 104: Barbu_TEOREME_FUNDAMENTALE(2008).pdf

111

Fie I centrul cercului înscris în triunghiul ABC (Fig. 102). Fie '''A al doilea punct de intersecţie dintre perpendiculara din I pe BC cu cercul circumscris triunghiului AIO. Deoarece patrulaterul '''AA IO este inscriptibil rezultă '''IAO IA O≡ şi

'OAI OA I≡ ,deci ''' ' (3).≡ OA I OA I Întrucât '''IA BC⊥ şi 'OA BC⊥ rezultă

''' ' (4).IA OA Din relaţiile (3) şi (4) rezultă că patrulaterul ' '''A OA I este paralelogram, de

unde ''' ',A O IA adică ''' ,A O AI deci patrulaterul '''AIOA este trapez isoscel. Cum I este ortocentrul triunghiului ' ' 'A B C (vezi „Cercul înscris într-un triunghi”) şi O este centrul cercului circumscris triunghiului ' ' 'A B C rezultă că '''A O este mediatoarea segmentului

' '.B C Din "AI A O rezultă ";IAO AOA≡ cum "A este simetricul lui O faţă de ' 'B C

rezultă " " ,AOA IA O≡ de unde " (5).≡ IAO IA O Din relaţiile (3) şi (5) rezultă

" ''' ,≡ IA O IA O iar cum ", '''A A aparţin mediatoarei segmentului ' 'B C avem că punctele "A şi '''A coincid. Deoarece ", ", "A B C sunt centrele cercurilor lui Carnot corespunzătoare triunghiului ' ' 'A B C rezultă că ortocentrul triunghiului ' ' 'A B C - adică punctul I – este centrul cercului circumscris triunghiului " " "A B C (vezi „Cercurile Carnot”) deci " " ",IA IB IC= = iar întrucât " , "IA BC IB CA⊥ ⊥ , "IC AB⊥ rezultă că dreptele ", ", "AA BB CC sunt concurente într-un punct al lui Kariya al triunghiului ABC.

3) Consecinţă: Centrul de omologie P dintre triunghiurile " " "A B C şi ABC aparţine cercului circumscris triunghiului ABC. Demonstraţie. Avem: ( ") ( ") ( ) ( ) 180 ( ) (6)+ = + = °− m CAA m ACC m PAC m ACP m APC

( ") ( ") [ ( ") ( )] [ ( ") ( )]+ = − + − = m CAA m ACC m IAA m IAC m ICC m ICA

1 1[ ( ") ( ")] [ ( ) ( )] ( ) [ ( ) ( )]

2 2= + − + = − + = m IAA m ICC m A m C m AIC m A m C

[ ]1 1180 [ ( ) ( )] ( ) ( )

2 2° − + − + m A m C m A m C , deci ( ") ( ")+ = m CAA m ACC

180 [ ( ) ( )] ( ) (7)° − + = m A m C m B . Din relaţiile (6) şi (7) rezultă că

( ) ( ) 180 ,+ = ° m APC m B adică patrulaterul ABCP este inscriptibil, deci P aparţine

cercului circumscris triunghiului ABC.

Observaţii:

1) Demonstraţia suferă modificări dacă triunghiul ABC este obtuzunghic, proprietatea rămânând, însă, adevărată. 2) Punctele ", ", "A B C sunt centrele cercurilor Carnot ale triunghiului ' ' 'A B C , iar vârfurile triunghiului ABC sunt punctele unde înălţimile triunghiului ' ' 'A B C intersectează cercul circumscris triunghiului ABC.

Page 105: Barbu_TEOREME_FUNDAMENTALE(2008).pdf

112

I.27. Punctul lui Schiffler

„Infinitul e mult mai mare Decât ne închipuim

N – o să putem niciodată Să-l umplem cu sufletul nostru.”

Marin Sorescu38 Dacă I este centrul cercului înscris în triunghiul ABC, atunci dreptele lui Euler ale triunghiurilor BCI, CAI, ABI şi ABC sunt concurente. Demonstraţie. Vom demonstra proprietatea utilizând coordonatele baricentrice. Astfel,

(sin 2 , sin 2 , sin 2 ), ( , , ), (1,1,1), ( , , )O A B C H tgA tgB tgC G I a b c (unde a, b, c reprezintă

lungimile laturilor BC, CA respectiv AB). Ecuaţia dreptei lui Euler OH a triunghiului

ABC în coordonatele baricentrice este :

1 1 1 0

sin 2 sin 2 sin 2

x y z

A B C

= , adică

(sin2 sin2 ) (sin2 sin2 ) (sin2 sin2 ) 0x C B y A C z B A− + − + − = . Fie 1 2 3, ,O O O şi 1 2 3, ,G G G centrele

cercurilor circumscrise, respectiv centrele de greutate ale triunghiurilor BCI, CAI respectiv

ABI. Avem: ( )1 1(sin ,sin ,sin ) ( sin ,sin ,sin )O A B C O A B Cπ + ≡ − şi 1 ( ,1 ,1 ).G a b c+ +

Ecuaţia dreptei lui Euler a triunghiului BCI este:

1 1 0

sin sin sin

x y z

a b c

A B C

+ + =

, sau

1 1( ) : [(1 )sin (1 )sin ] [(1 )sin sin ]OG x b C c B y c A a C+ − + − + + + [(1 )sin sin ] 0.z b A a B+ + =

Analog, 2 2(sin , sin ,sin ), (1 , ,1 )O A B C G a b c− + + şi 3 3(sin ,sin , sin ), (1 ,1 , )O A B C G a b c− + + rezultă

2 2( ) : [ sin (1 )sin ] [(1 )sin (1 )sin ] [ sin (1 )sin ] 0O G x b C c B y a C c A z b A a B− − + − + − + + + + = şi

3 3( ) : [ sin (1 )sin ] [ sin (1 )sin ] [(1 )sin (1 )sin ] 0.O G x c B b C y c A a C z a B b A− − + − − + + + − + =

Deoarece

(1 )sin (1 )sin (1 )sin sin sin (1 )sin

sin (1 )sin (1 )sin (1 )sin sin (1 )sin 0,

sin (1 )sin sin (1 )sin (1 )sin (1 )sin

b C c B c A a C a B b A

b C c B a C c A b A a B

c B b C c A a C a B b A

+ − + − + − + +

− − + + − + + + =

− − + − + + − +

rezultă că dreptele 1 1 2 2 3 3, ,OG O G O G sunt concurente (1). Analog se arată că :

(1 )sin (1 )sin (1 )sin sin sin (1 )sin

sin (1 )sin (1 )sin (1 )sin sin (1 )sin 0,

sin 2 sin 2 sin 2 sin 2 sin 2 sin 2

b C c B c A a C a B b A

b C c B a C c A b A a B

C B A C B A

+ − + − + − + +

− − + + − + + + =

− − −

deci

dreptele OH, 1 1 2 2,OG O G sunt concurente (2). Din relaţiile (1) şi (2) rezultă că dreptele lui

Euler ale triunghiurilor BCI, CAI, ABI şi ABC sunt concurente. Observaţie: Punctul

hS de concurenţă al dreptelor lui Euler ale triunghiurilor BCI, CAI,

ABI şi ABC se numeşte punctul lui Schiffler. Consecinţă: Punctul lui Schiffler, ortocentrul şi centrul de greutate al unui triunghi ABC sunt coliniare.

38 Marin Sorescu (1936-1996) – scriitor român

Page 106: Barbu_TEOREME_FUNDAMENTALE(2008).pdf

113

I.28. Punctul lui Weill39

Punctul lui Weill (W) al unui triunghi este centrul de greutate al triunghiului de contact al său (Fig.103). 1) Dreapta WI este dreapta lui Euler a triunghiului de contact

a b cC C C , unde I este centrul cercului înscris în

triunghiul ABC. Demonstraţie. Deoarece W este centrul de greutate al triunghiului

a b cC C C şi I este centrul cercului circumscris

triunghiului a b c

C C C rezultă că WI este dreapta lui Euler a

triunghiului a b c

C C C .

2) Centrul cercului circumscris (O) al triunghiului ABC aparţine dreptei WI. Demonstraţie. Centrul cercului circumscris triunghiului ABC aparţine dreptei lui Euler a triunghiului

a b cC C C (vezi „Dreapta lui Euler”).

3) Dacă W este punctul lui Weill al unui triunghi ABC, atunci

1 1 1

3 3 3

p c p b p a p c p a p bMW MA MB MC

b c c a b a

− − − − − − = + + + + +

uuuur uuur uuur uuuur

Demonstraţie. Deoarece W este centrul de greutate al triunghiului ABC, rezultă 1( )

3 a b cMW MC MC MC= + +uuuur uuuuur uuuuur uuuuur

(1) pentru orice punct M din planul triunghiului ABC. Dar

a

a

BC p b

CC p c

−=

− de unde

1[( ) ( ) )]

aMC p c MB p b MC

a= − + −

uuuuur uuur uuuur (2). Analog, se obţin relaţiile:

1[( ) ( ) )]

bMC p a MC p c MA

b= − + −

uuuuur uuuur uuur (3) şi

1[( ) ( ) )]

cMC p b MA p a MB

c= − + −

uuuuur uuur uuur (4). Din

relaţiile (1), (2), (3) şi (4) rezultă concluzia. 4) Consecinţă: Coordonatele baricentrice relative ale punctului lui Weill al unui triunghi

ABC sunt : , , .p c p b p a p c p a p b

Wb c c a b a

− − − − − − + + +

Demonstraţia rezultă din teorema precedentă. 5) Consecinţă: Coordonatele baricentrice absolute ale punctului lui Weill al unui

triunghi ABC sunt : 1 1 1

, , .9 9 9

p c p b p a p c p a p bW

b c c a b a

− − − − − − + + +

Demonstraţie. Pentru a determina coordonatele baricentrice absolute ale punctului lui Weill efectuăm suma coordonatelor sale şi obţinem: 1 1 ( )(2 ) ( )(2 ) ( )(2 )

3 3

p c p b p a p c p a p b p c p c p b p b p a p a

b c c a b a ab ca cb

− − − − − − − − − − − − + + + + + = + + =

39 André Weill (1906-1998) – matematician francez, profesor la Universitatea Princeton, contribuţii importante în

algebră, analiză şi geometrie

A

B C

I

bC

aC

W

cC

Fig. 103

O

Page 107: Barbu_TEOREME_FUNDAMENTALE(2008).pdf

114

123

Rrp

abc= = , unde am folosit relaţiile 3 3 3 2 22 ( 3 6 )a b c p p r Rr+ + = − − şi

2 2 2 2 22( 4 )a b c p r Rr+ + = − − , de unde rezultă concluzia.

6) Centrul cercului înscris, centrul cercului circumscris şi punctul lui Weill sunt coliniare. Demonstraţie. Demonstrăm teorema utilizănd coordonatele baricentrice. Avem ( , , )I a b c

şi ( )sin 2 ,sin 2 ,sin 2O A B C . Deoarece 0

sin 2 sin 2 sin 2

p c p b p a p c p a p b

b c c a b a

a b c

A B C

− − − − − −+ + +

=

rezultă că punctele W,O şi I sunt coliniare.

7) Consecinţă: 3

WI r

WO R r=

+.

Demonstraţie. Se arată că (3 )

2 3

R r MI rMOMW

r R

+ +=

+

uuur uuuuruuuur

, utilizând relaţiile:

2

(sin 2 sin 2 sin 2 )2

RMO A MA B MB C MC

S= ⋅ + ⋅ + ⋅

uuuur uuur uuur uuuur şi

aMA bMB cMCMI

a b c

+ +=

+ +

uuur uuur uuuuruuur

.

I.29. Punctele lui Pelletier

„Matematicienii sunt ca francezii: orice le spui traduc în limba lor şi drept urmare rezultă ceva complet diferit.” – J. W. Goethe40

Teorema lui Pelletier

Într-un triunghi ABC , dreptele care unesc picioarele înalţimilor corespunzătoare vârfurilor B şi C , picioarele bisectoarelor corespunzătoare vârfurilor B şi C şi punctele de tangenţă ale cercului înscris cu laturile AB şi AC sunt concurente. Demonstrăm teorema utilizând coordonatele baricentrice.

Lemă: Fie punctele ( , , )i i i i

Q α β γ , 1,3i = în planul unui triunghi ABC . Prin fiecare punct

iQ , 1,3i = , ducem cevienele

iAA ,

iBB ,

iCC , 1,3i = . Dreptele 1 1BC , 2 2B C , 3 3B C sunt

concurente dacă şi numai dacă 1 1 1

2 2 2

3 3 3

1/ 1/ 1/

1/ 1/ 1/ 0

1/ 1/ 1/

α β γα β γα β γ

= .

40 Johann Goethe (1749-1832) – poet, scriitor german

Page 108: Barbu_TEOREME_FUNDAMENTALE(2008).pdf

115

Demonstraţie lemă. Avem ( ,0, )i i iB α γ , 1,3i = , ( , ,0)

i i iC α β , 1,3i = . Ecuaţiile dreptelor

i iBC sunt: 0

i i i

x y z

α β γ− + + = , 1,3i = . Condiţia de concurenţă a trei drepte conduce la

concluzia problemei. Demonstraţia teoremei. Coordonatele baricentrice ale ortocentrului, centrului cercului înscris şi ale punctului lui Gergonne corespunzătoare triunghiului ABC sunt:

( , , )H ctgB ctgC ctgC ctgA ctgA ctgB⋅ ⋅ ⋅ , ( , , )2 2 2

a b cI

p p p,

( )( ) ( )( ) ( )( ), ,

(4 ) (4 ) (4 )

p b p c p c p a p a p b

r R r r R r r R r

− − − − − −Γ + + +

.

Conform lemei, dreptele sunt concurente dacă:

1 1 1

1 1 10

tgA tgB tgC

a b c

p a p b p c

∆ = =

− − −

. Utilizăm

formulele 2

22

12

Atg

tgAA

tg

=+

, ( )2

Ar p a tg= − ,

2

22 sin 21

2

Atg

a R A RA

tg

= =+

. Notăm 2

Atg m= ,

2

Btg n= ,

2

Ctg p= .

1 1 1

1 1 10

1 1 1

m n pm n p

m n p

m n pa b c

p a p b p cm n p

− − −− − −

∆ = = =

− − −

.

Observaţii:

1) Fie 'A punctul de intersecţie al celor trei drepte. Analog se definesc punctele 'B şi 'C .

Punctele 'A , 'B şi 'C se numesc punctele lui Pelletier, iar ' ' 'A BC se numeşte triunghiul lui Pelletier. 2) Analog se arată că dreptele care unesc picioarele înalţimilor corespunzătoare vârfurilor B şi C , picioarele bisectoarelor corespunzătoare B şi C şi punctele de contact ale cercurilor exînscrise triunghiului ABC cu laturile AB şi AC sunt concurente (se

consideră punctele I , H şi punctul lui Nagel ( , , )p a p b p c

Np p p

− − −) .

1) Triunghiul ABC şi triunghiul lui Pelletier corespunzător sunt omologice. Demonstraţie : Fie 1I , 2I , 3I picioarele

bisectoarelor interioare ale unghiurilor A , B , respectiv C şi

a b cC C C triunghiul de

contact al triunghiului ABC

(Fig. 104), atunci '2 3

b cA C C I I= ∩ ,

'1 3

a cB C C I I= ∩ şi '

1 2 a b

C C C I I= ∩ iar

A

B C

I

A'

2I

3I

Fig. 104

A"

bC

cC

Page 109: Barbu_TEOREME_FUNDAMENTALE(2008).pdf

116

'' ' A AA BC= ∩ , '' ' B BB AC= ∩ , '' ' C CC AB= ∩ şi fără a restrânge generalitatea

presupunem că a b c> > . Considerând triunghiul ABC , transversala c b

C C şi ceviana ''AA

avem: ' ''

' ''1c c

b b

AC ACAC A B

AB C C AC A C⋅ ⋅ ⋅ = (1). Din teorema lui Menelaus în triunghiul

b cAC C şi

transversala '3 2A I I− − avem:

'3 2

'3 2

1c b

cb

AC I A I C

I C I AAC⋅ ⋅ = (2). Din teorema bisectoarei rezultă:

2

bcI A

a c=

+, 3

bcI A

a b=

+, de unde rezultă 2 2

( )( )b b

a c p bI C I A C A

a c

− −= − =

+ şi

3

( )( )c

a b p cI C

a b

− −=

+ (3). Din relaţiile (2) şi (3) rezultă

'

'c

b

AC a b p c

a c p bAC

− −= ⋅

− − (4). Din

relaţiile (1) şi (4) rezultă ''

''

( )( )

( )( )

A B c p a p b a c

b p c p c a bA C

− − −= ⋅ ⋅ =

− − −

2

( )( )

( )

c a c p a p b

b a b p c

− − −⋅ ⋅− −

.

Analog, se demonstrează că ''

'' 2

( ) ( )( )

( ) ( )

B C a a b p b p c

c b cB A p a

− − −= ⋅ ⋅

− − şi

''

'' 2

( ) ( )( )

( ) ( )

C A b c b p c p a

a c aC B p b

− − −= ⋅ ⋅

− −, de unde

'' '' ''

'' '' ''1

A B B C C A

A C B A C B⋅ ⋅ = , iar din reciproca teoremei

lui Ceva rezultă că dreptele 'AA , 'BB , 'CC sunt concurente. Atunci, din teorema lui

Desargues rezultă că triunghiurile ABC şi ' ' 'A BC sunt omologice.

I.30. Punctul lui Kenmotu

„Legile naturii sunt doar gândurile matematice ale lui Dumnezeu” - Euclid41 Fie pătratele congruente 1 1 1 2 2 2 3 3 3, ,

e e eM K N P M N P K PM N K aflate în interiorul

triunghiului ABC , astfel încât 1 2 2 3 1 3, ( ), , ( ), ( ).∈ ∈ ∈M M BC P P AC N N AB Punctul e

K

comun pătratelor date se numeşte punctul lui Kenmotu (Fig.105).

41 Euclid din Alexandria (330 – 275 î.e.n.) – matematician grec, contribuţii în geometrie

A

B

C

eK

1M 2M

2N

2P 3P 3M

3N

1N

Fig. 105

1P

Page 110: Barbu_TEOREME_FUNDAMENTALE(2008).pdf

117

1) Punctele 1 2 2 3 1 3, , , , ,M M P P N N sunt conciclice.

Demonstraţie. Deoarece 1 2 2 3 3 1 ( )≡ ≡ ≡ ≡ ≡ =e e e e e e

K M K M K P K P K N K N l , unde cu l am

notat lungimea laturii pătratelor congruente, punctele 1 2 2 3 1 3, , , , ,M M P P N N se află pe un

cerc cu centrul în punctul e

K şi rază l.

Observaţie: Cercul pe care se află punctele 1 2 2 3 1 3, , , , ,M M P P N N se numeşte cercul lui

Kenmotu şi are raza egală cu 1

2⋅ l .

2) Diagonalele pătratelor Kenmotu determinate de vârfurile acestora ce aparţin laturilor triunghiului ABC sunt antiparalele cu laturile triunghiului ABC.

Demonstraţie. Notăm 1 1 2 2( ) ( )x m BM P m CM N= = ,

3 3 2 2( ) ( )y m APM m CP N= = şi

3 3 1 1( ) ( )z m AN M m BN P= = . Din triunghiul 1 1BN M rezultă ( ) (45 ) (45 ) 180m B x z+ °+ + °+ = ° ,

de unde 90 ( )x z m B+ = °− . Analog, 90 ( )z y m A+ = °− şi 90 ( )y x m C+ = °− . Sumând

relaţiile precedente rezultă 45x y z+ + = ° şi de aici se obţin egalităţile: ( ) 45m A x= °+ ,

( ) 45m B y= °+ , ( ) 45 .m C z= °+ Atunci, 3 3( ) ( )m AP N m B= şi

3 3( ) ( )m AN P m C= , deci

dreptele 3 3N P şi BC sunt antiparalele. Anolog se arată că 1 1N M şi 2 2M P sunt antiparalele

cu laturile CA, respectiv AB. 3) Cercul Kenmotu este un cerc Tucker. Demonstraţie. Din teorema precedentă şi din faptul că 1 1 2 2 3 3N M M P N P≡ ≡ rezultă

concluzia. 4) 1 2 3 2 3 1, , .N P BC N M CA PM AB

Demonstraţie. Deoarece 1 1 2 2N M M P≡ rezultă că patrulaterul 1 1 2 2N M M P este trapez

isoscel, deci 1 2N P BC . Analog se arată 3 2 3 1, .N M CA PM AB

5) Patrulaterele 1 2 2 3M M P P , 3 1 2 3N N P P şi 1 2 3 1M M N N sunt inscriptibile.

Demonstraţie. Deoarece 1 2N P BC rezultă 1 2 3( ) ( )m N P P m C= şi cum

3 3( ) ( )m AN P m C= rezultă

1 2 3 3 3( ) ( )m N P P m AN P= , relaţie ce arată că patrulaterul

3 1 2 3N N P P este inscriptibil.

6) Punctul lui Kenmotu aparţine dreptei lui Brocard a triunghiului ABC. Demonstraţie. Deoarece cercul Kenmotu este un cerc Tucker, cum centrul unui cerc Tucker aparţine dreptei lui Brocard (vezi „Cercul lui Tucker”), rezultă concluzia.

7) Dacă γβα ,, sunt centrele pătratelor Kenmotu, atunci triunghiurile αβγ şi ABC

sunt omotetice, centrul de omotetie fiind centrul lui Lemoine al triunghiului ABC. Demonstraţie. Deoarece γβα ,, sunt mijloacele antiparalelor 1 1 2 2 3 3, ,N M M P N P ,

triunghiurile αβγ şi ABC sunt omotetice, centrul de omotetie fiind centrul lui Lemoine al

triunghiului ABC (vezi „Cercul lui Taylor”).

Page 111: Barbu_TEOREME_FUNDAMENTALE(2008).pdf

118

I.31. Dreapta lui Euler.Cercul lui Euler42

„Citiţi pe Euler! Citiţi pe Euler, el este Maestrul nostru, al tuturor.” - P. S. Laplace43

1) În triunghiul ABC fie , ,a b c

H H H picioarele înălţimilor, , ,a b c

M M M mijloacele

laturilor BC, CA respectiv AB şi ', ', 'A B C mijloacele segmentelor AH, BH respectiv CH. Punctele , , , , , , ', ', '

a b c a b cH H H M M M A B C sunt conciclice.

Demonstraţie. În triunghiul dreptunghic

aAH B ,

aH C este mediană, deci

2=a c

ABH M (1),

a bM M este linie mijlocie în

triunghiul ABC, deci 2

=a b

ABM M (2)

(Fig.106). Din (1) şi (2) rezultă că

a b a cM M H M= şi cum

c bM M BC

(deoarecec b

M M este linie mijlocie în

triunghiul ABC) rezultă că patrulaterul

c a a bM H M M este trapez isoscel, deci punctele

, ,a b c

M M M şi a

H aparţin unui cerc C.

Analog, se arată că punctele b

H şi c

H aparţin

cercului C. În triunghiul BHC, 'a

M C este linie

mijlocie, deci ' ,a

M C BH de unde 'a

HBC C M C≡ (3). Patrulaterul a c

BH HH fiind

inscriptibil ( ( ) ( ) 180 )+ = ° a c

m BH H m BH H rezultă că a c a

HBH HH H≡ (4). Din

relaţiile (3) şi (4) rezultă că ' ,a a c

C M C H H H≡ adică patrulaterul 'a a c

C M H H este

inscriptibil, deci C' aparţine cercului C. Analog, se demonstrează că punctele 'A şi 'B

sunt pe cercul C.

Observaţii:

i) Cercul pe care se găsesc cele nouă puncte , , , , , , ', ', 'a b c a b c

M M M H H H A B C se numeşte

cercul lui Euler sau cercul celor nouă puncte sau cerc medial. ii) Centrul cercului lui Euler îl vom nota în continuare cu 9O .

iii) Punctele ', ', 'A B C - mijloacele segmentelor AH, BH, CH – se numesc punctele euleriene ale triunghiului ABC.

2) Într-un triunghi, dreptele care unesc mijloacele laturilor, respectiv cu punctele euleriene ale înălţimilor ce pleacă din vărfurile opuse sunt diametre în cercul lui Euler al triunghiului.

42 Leonhard Euler (1707-1783) – matematician elveţian, profesor la Universitatea din Petesburg, contribuţii

fundamentale în toate ramurile matematicii 44 P. S. Laplace (1749-1827) – matematician şi astronom francez , contribuţii în algebră şi analiză

A

B C aM

bM cM

aH

bH

cH

Fig. 106

G O

H

A'

B' C' 9O

A"

Page 112: Barbu_TEOREME_FUNDAMENTALE(2008).pdf

119

3) Centrul cercului lui Euler al triunghiului ABC este mijlocul segmentului OH, unde O este centrul cercului circumscris triunghiului ABC, iar H ortocentrul acestuia. Demonstraţie. Deoarece

aOM BC⊥ rezultă ,

a aOM HH adică patrulaterul

a aHOM H este

trapez, perpendicularele ridicate din mijloacele coardelor ,a a b b

H M H M şi c c

H M ale

cercului lui Euler trec prin mijlocul segmentului OH, deci prin 9O .

Observaţii: i) Dreapta OH se numeşte dreapta lui Euler a triunghiului ABC. ii) Centrul cercului lui Euler al triunghiului ABC aparţine dreptei lui Euler a triunghiului ABC. iii) Perpendicularele ridicate pe laturile unui triunghi în mijloacele segmentelor cuprinse între picioarele înălţimilor şi mijloacele laturilor, sunt concurente în centrul cercului lui Euler al triunghiului. 4) Centrul de greutate G al triunghiului ABC se află pe dreapta lui Euler a triunghiului ABC şi GH=2OG. Demonstraţie. Fie .= ∩

aG AM HO Din asemănarea triunghiurilor 1AHG şi 1a

M OG

avem: 1 1

1 1

(1).= =a a

AG HGAH

G M OM GOFie " = ∩A AO C(ABC). Avem ( " ) 90 ,= °m A CA deci

" ,A C CA⊥ dar BH AC⊥ de unde " .BH A C Analog, " ,CH A B deci patrulaterul

"BHA C este paralelogram, deci punctele ,a

H M şi "A sunt coliniare. Din asemănarea

triunghiurilor "a

OM A şi "AHA rezultă " 2

2"

= = =a

AH AA R

OM OA R (2). Din relaţiile (1) şi (2)

rezultă 1 1

1

2= =a a

AG HG

G M GO, sau 1 12=

aAG G M , adică 1G este centrul de greutate G al

triunghiului ABC şi 2HG GO= (unde R este lungimea razei cercului circumscris triunghiului ABC). Observaţie: Din aplicaţia precedentă rezultă 9 912 6 4 3 .GO GO OO HO= = =

5) Fie H ortocentrul triunghiului ABC. Triunghiurile ABC, BHC, CHA şi AHB au acelaşi cerc al lui Euler. Demonstraţie. Deoarece triunghiurile ABC, BHC, CHA şi AHB au acelaşi triunghi ortic (vezi „Triunghiul ortic”) rezultă că ele au acelaşi cerc al lui Euler. 6) Dacă H este ortocentrul triunghiului ABC, atunci dreptele lui Euler ale triunghiurilor ABC, AHC, AHB şi BHC sunt concurente. Demonstraţie. Deoarece triunghiurile ABC, AHC, AHB şi BHC au acelaşi cerc medial, atunci dreptele lui Euler ale acestor triunghiuri trec prin punctul 9O (centrul cercului lui

Euler). 7) Raza cercului lui Euler a triunghiului ABC are lungimea egală cu jumătate din lungimea razei cercului circumscris a triunghiului ABC.

Demonstraţie. Din aplicaţia precedentă avem 1

'2aOM AH A H= = ( 'A - mijlocul

segmentului AH) şi cum 'a

OM A H rezultă că patrulaterul 'a

A HM O este paralelogram,

Page 113: Barbu_TEOREME_FUNDAMENTALE(2008).pdf

120

deci diagonalele sale se înjumătăţesc, adică 'a

A M trece prin 9O mijlocul segmentului OH,

adică prin centrul cercului lui Euler. Cum în triunghiul ", 'a

AHA A M este linie mijlocie

rezultă 1

' " ,2aA M AA R= = de unde 9

' 1' .

2 2= =aA M

A O R

8) Punctele 9, ,H O G şi O determină o diviziune armonică.

Demonstraţie. Deoarece 9 91

2

HO GO

HO GO= = rezultă că punctele 9, ,H O G şi O formează o

diviziune armonică. 9) Diametrele cercului lui Euler al triunghiului ABC care trec prin punctele lui Euler sunt mediatoarele laturilor triunghiului ortic corespunzător triunghiului ABC. Demonstraţie. Deoarece patrulaterul '

aOM HA este paralelogram rezultă că '

aA M este

diametru în cercul lui Euler corespunzător triunghiului ABC şi ' .a

A M AO Cum

b cAO H H⊥ (vezi „Triunghiul ortic”) rezultă ' .

a b cA M H H⊥

10) Diametrele cercului lui Euler al triunghiului ABC care trec prin mijloacele laturilor triunghiului sunt paralele cu razele cercului circumscris ce trec prin vârfurile opuse laturilor considerate. Demonstraţie. Dacă ', ', 'A B C sunt punctele euleriene, atunci '

aOM AA şi '

aOM AA≡ ,

deci patrulaterul 'a

AA M O este paralelogram, de unde rezultă că ' .a

A M AO

11) Tangentele în punctele euleriene la cercul lui Euler a triunghiului ABC sunt antiparalele cu laturile triunghiului ABC. Demonstraţie. Deoarece

b cH H este antiparalelă lui BC şi tangenta în 'A la cercul lui Euler

este paralelă cu b c

H H rezultă că tangenta în 'A este paralelă cu .b c

H H

12) Perpendicularele duse din mijloacele laturilor unui triunghi, respectiv pe laturile triunghiului ortic sunt concurente în centrul cercului celor două puncte ale triunghiului dat. Demonstraţie. Vezi „Triunghiul ortic”. 13) Într-un cerc dat se pot înscrie o infinitate de triunghiuri care să aibă acelaşi cerc al lui Euler. Demonstraţie. Deoarece într-un cerc dat C se pot înscrie o infinitate de triunghiuri care să

aibă acelaşi ortocentru H (vezi „Ortocentrul unui triunghi”), iar centrul cercului C este

punctul fix O, atunci mijlocul segmentului OH – punctul 9O - este centrul cercului lui

Euler corespunzător triunghiurilor înscrise în cercul C, având ortocentrul H, raza acestui

cerc având lungimea egală cu jumătate din lungimea razei cercului C.

14) Cercul lui Euler al unui triunghi ABC este locul geometric al mijloacelor segmentelor HM, când M parcurge cercul circumscris triunghiului ABC (unde H este ortocentrul triunghiului ABC).

Page 114: Barbu_TEOREME_FUNDAMENTALE(2008).pdf

121

Demonstraţie. Fie O centrul cercului circumscris triunghiului ABC şi 9O centrul

cercului lui Euler al triunghiului .ABC Fie M

un punct pe cercul C(O,R), iar P mijlocul

segmentului HM (Fig. 107). Cum 9O este

mijlocul segmentului HO, atunci 9O P este

linie mijlocie în triunghiul HOM, deci

9

1,

2 2= =OM

O P R adică P aparţine cercului

lui Euler al triunghiului .ABC Reciproc, dacă P este un punct pe cercul lui Euler al triunghiului ABC şi M este punctul de intersecţie dintre HP cu cercul circumscris

triunghiului ,ABC atunci 9 ,2 2

= =R OM

O P

deci P este mijlocul segmentului HM. 15) Cercul lui Euler al unui triunghi dreptunghic trece prin vârful care are măsura de 90° şi este tangent în acest punct cercului circumscris. Demonstraţie. Deoarece ortocentrul triunghiului dreptunghic ABC este chiar vârful A ( ( ) 90 )= °m A rezultă că centrul cercului lui Euler al triunghiului ABC este mijlocul

medianei AO (O fiind centrul cercului circumscris triunghiului ABC ), deci A este punct pe

cercul lui Euler al triunghiului ABC. Avem 9 9 ,2 2

= = =AO R

AO O O deci cercul lui Euler al

triunghiului ABC este tangent interior cercului circumscris triunghiului ABC.

16) Triunghiurile dreptunghice înscrise într-un cerc C ( , )O R având vârful unghiului

drept fix şi ipotenuza variabilă, au acelaşi cerc medial. Demonstraţie. Cercurile lui Euler corespunzătoare triunghiurilor dreptunghice au acelaşi

centru 9O – mijlocul medianei AO şi aceeaşi rază egală cu .2

R

17) Fie

aM mijlocul laturii BC a triunghiului ABC. Prin inversiunea de centru

aM şi

raport *k ∈ cercul lui Euler (fără punctul a

M ) se transformă într-o dreaptă

antiparalelă cu BC. Demonstraţie. Fie 9O centrul cercului lui Euler al

triunghiului ABC şi A∗ punctul diametral opus lui A în cercul circumscris (Fig.108). Prin inversiunea ( , )

aI M k

cercul lui Euler se transformă într-o dreaptă d

perpendiculară pe dreapta 9 .aM O Deoarece 9a

M O AA∗ şi

9ad M O⊥ rezultă că d AA∗⊥ . Fie D punctul de intersecţie

dintre AB şi tangenta în A∗ la cercul circumscris triunghiului ABC. Atunci, .d DA∗

Avem:

( ) 90 ( ) ( ) ( ),m ADA m A AD m AA B m ACB∗ ∗ ∗= °− = = adică

A

B C aH

bH

Fig. 107

O

H P

M 9O

A

B C

9O O

aM

Fig. 108 D

A∗

H

Page 115: Barbu_TEOREME_FUNDAMENTALE(2008).pdf

122

dreptele BC şi A O∗ sunt antiparalele, de unde rezultă că dreptele d şi BC sunt antiparalele. 18) Fie 1 1 1, ,A B C mijloacele segmentelor AO, BO respectiv CO – unde O este centrul

cercului circumscris unui triunghi ABC. Centrul cercului lui Euler 9( )O al triunghiului

ABC este ortocentrul triunghiului 1 1 1A BC .

Demonstraţie. Fie a

AH înălţimea triunghiului ABC şi

1 " ( " )a

A A AH A BC∈ (Fig. 109). Deoarece 1 1 BC BC

rezultă că 1 1 1"⊥A A BC şi "A este mijlocul segmentului

,a aH M ceea ce arată că 9O - centrul cercului lui Euler al

triunghiului ABC aparţine dreptei 1 ".A A Deci, 9O aparţine

înălţimii din 1A a triunghiului 1 1 1.A BC Analog se arată că 9O

aparţine şi înălţimii din 1B a triunghiului 1 1 1A BC , deci 9O este

ortocentrul triunghiului 1 1 1A BC .

19) Fie O centrul cercului circumscris triunghiului ABC. Dreptele care unesc mijloacele segmentelor OA,OB,OC cu mijloacele laturilor BC,CA, respectiv AB ( , ,

a b cM M M ) sunt dreptele

lui Euler ale triunghiurilor ,b c

M OM

,c a a b

M OM M OM .

Demonstraţie. Fie ', ', 'A B C mijloacele segmentelor AO,BO, respectiv CO. Punctul 'A este centrul cercului circumscris triunghiului

b cM OM , iar punctul

aM este ortocentrul

triunghiului b c

M OM , deci 'a

A M este dreapta lui

Euler a triunghiului b c

M OM . Analog, 'b

B M şi

'c

C M sunt dreptele lui Euler ale triunghiurilor

,c a

M OM respectiv .a b

M OM

Observaţie: Dreptele '

aA M , '

bB M şi '

cC M sunt concurente în centrul cercului lui Euler

al triunghiului median, deoarece dreptele lui Euler ale unui patrupunct ortocentric (patru puncte în care fiecare punct este ortocentrul triunghiului determinat de celelalte trei puncte) sunt concurente – conform proprietăţii 6, punctul de concurenţă fiind centrul cercului lui Euler al triunghiului

a b cM M M .

20) Triunghiul ABC şi triunghiul lui Carnot au acelaşi cerc al lui Euler şi aceeaşi dreaptă a lui Euler. Demonstraţie. Vezi „Triunghiul lui Carnot”. 21) Cercul circumscris unui triunghi este cercul celor nouă puncte al triunghiului anticomplementar. Demonstraţie. Triunghiul ABC este triunghiul anticomplementar al triunghiului median

.a b c

M M M

A

B C

9O O

aH A"

Fig. 109

1B 1C

1A

A

B C aM

bM cM

O

Fig. 110

A'

B' C'

Page 116: Barbu_TEOREME_FUNDAMENTALE(2008).pdf

123

22) Cercul circumscris unui triunghi ABC şi cercul lui Euler al acestui triunghi sunt omotetice. Demonstraţie. Dacă ', ', 'A B C sunt punctele lui Euler al triunghiului ABC, atunci

' ', ' ', ' 'A B B C C A sunt linii mijlocii în triunghiurile ABH, BHC, respectiv CHA;deci

' ' , ' ' , ' 'A B AB B C BC C A CA şi ' ' ' ,∩ ∩ =AA BB CC H adică triunghiurile ABC şi

' ' 'A B C sunt omotetice, de unde rezultă că cercurile circumscrise triunghiurilor ABC şi ' ' 'A B C sunt omotetice, centrul de omotetie fiind ortocentrul triunghiului ABC.

Observaţie: Deoarece triunghiul ABC are cu triunghiul ' ' 'A B C înălţimile comune, iar cu triunghiul median

a b cM M M medianele comune rezultă că G este centrul de omotetie

invers dintre triunghiul ABC şi a b c

M M M , deci G este centrul de omotetie invers dintre

cercul circumscris triunghiului ABC şi cercul circumscris triunghiului a b c

M M M - adică

cercul lui Euler al triunghiului ABC. 23) Triunghiurile echilaterale înscrise într-un cerc dat au acelaşi cerc al lui Euler. Demonstraţie. Într-un triunghi echilateral ABC picioarele înălţimilor coincid cu mijloacele laturilor, deci cercul lui Euler este cercul înscris în triunghiul ABC. Deoarece triunghiurile echilaterale înscrise într-un cerc au acelaşi cerc înscris, concluzia este evidentă. 24) Fie triunghiul dreptunghic ABC ( ( ) 90m BAC = ° ) şi M un punct pe cercul circumscris triunghiului ABC astfel încât A şi M se află în semicercuri diferite determinate de diametrul BC. Triunghiurile AMB şi AMC au cercurile lui Euler tangente. Demonstraţie. Fie 1O şi 2O centrele cercurilor

lui Euler ale triunghiurilor AMB şi AMC, iar P mijlocul segmentului AM (Fig. 111). Tangenta (T) în P la cercul lui Euler al triunghiului AMB este paralelă cu tangenta ( 1T ) în B la cercul

circumscris triunghiului ABM, iar tangenta în P la cercul lui Euler al triunghiului AMC este paralelă cu tangenta ( 2T ) în C la cercul

circumscris triunghiului AMC. Cum BC este diametru în cercul circumscris triunghiului ABC rezultă că 1 2BT CT de unde 1 2 ,PT BT CT

adică cercurile lui Euler sunt tangente exterior în punctul P.

Observaţie: Patrulaterele 2 1BOO O şi 1 2COOO sunt paralelograme, deoarece

1 2 2 2

R RO P PO R BO+ = + = = şi 1.OB PO

25) Fie , ,α β γ punctele diametral opuse vârfurilor A, B, C ale triunghiului ABC în

cercul circumscris acestui triunghi. Cercurile lui Euler ale triunghiurilor ,BC ACα β şi

ABγ sunt tangente cercului lui Euler al triunghiului ABC. Demonstraţia este evidentă utilizând proprietatea precedentă.

A

B C

M

P

T

O

1O 2O 1T

2T

Fig. 111

Page 117: Barbu_TEOREME_FUNDAMENTALE(2008).pdf

124

26) Raza cercului lui Euler a unui triunghi ABC este medie geometrică între raza cercului circumscris şi raza cercului lui Euler al triunghiului său ortic. Demonstraţie. Fie R raza cercului circumscris triunghiului ,ABC 1R şi 2R razele cercurilor

lui Euler ale triunghiurilor ABC şi a b c

H H H (triunghiul ortic al triunghiului ABC). Avem

1

1

2

R

R= şi 1

2

1,

2

R

R= de unde 2

1 2 .R R R= ⋅

27) Raza cercului lui Euler a triunghiului ortic al unui triunghi ABC este medie geometrică între razele cercurilor lui Euler ale triunghiului ABC şi cea a triunghiului median. Demonstraţie. Fie 1 2 3, ,R R R razele cercurilor lui Euler ale triunghiurilor ABC,

a b cH H H

(triunghiul ortic al triunghiului ABC) respectiv 1 2 3H H H (triunghiul ortic al triunghiului

median). Avem 1

2

1

2

R

R= şi 2

3

1,

2

R

R= de unde 2

1 1 3R R R= .

28) Fie

a b cH H H triunghiul ortic şi

a b cM M M triunghiul median al triunghiului ABC, O

centrul cercului circumscris triunghiului ABC, ,b c

AO M Mα = ∩ ,a c

BO M Mβ = ∩

a b

CO M Mγ = ∩ . Dreptele , ,a b c

H H Hα β γ sunt concurente în centrul cercului lui Euler al triunghiului ABC. Demonstraţie. Fie 1 1 1, ,A B C punctele de intersecţie dintre AO,

BO, CO cu BC, CA respectiv AB şi 'A mijlocul segmentului AH (Fig. 112). În triunghiul dreptunghic 1,a a

AH A H α este

mediană, iar în triunghiul 9' ,a a a

A H M H O este mediană ( 9O

fiind centrul cercului lui Euler al triunghiului ABC). Deoarece '

aAO A M (patrulaterul '

aAOM A fiind paralelogram)

rezultă că punctele 9,a

H O şi α sunt coliniare. Analog se

arată că dreptele b

H β şi c

H γ trec prin 9 .O

29) Fie O centrul cercului circumscris triunghiului ABC şi 1 1 1, ,A B C punctele de intersecţie dintre AO, BO, CO cu BC, CA respectiv AB. Cercurile având diametrele

1 1 1, ,AA BB CC sunt tangente cercului circumscris triunghiului ABC şi cercului lui Euler al triunghiului ABC. Demonstraţie. Deoarece punctele A, O şi 1A sunt coliniare rezultă că cercul circumscris

triunghiului ABC şi cercul de diametru 1AA sunt tangente interior. Deoarece punctele

9,a

H O şi α sunt coliniare (α fiind centrul cercului de diametru 1AA şi 9O centrul

cercului lui Euler al triunghiului ABC) rezultă că cercul lui Euler şi cercul de diametru 1AA

sunt tangente interior în punctul .a

H

30) Axa radicală a cercurilor circumscris şi a celor două puncte ale unui triunghi ABC este axa ortică a triunghiului ABC. Demonstraţie. Vezi „Axa ortică”.

A

B C

9O O aH aM

Fig. 112

bM

1A

H α cM A'

Page 118: Barbu_TEOREME_FUNDAMENTALE(2008).pdf

125

31) Cercul circumscris al unui triunghi ABC este cercul lui Euler al triunghiului antisuplementar

a b cI I I al triunghiului ABC.

Demonstraţie. Vezi „Triunghiul antisuplementar”.

32) Dreapta lui Euler a triunghiului antisuplementar a b cI I I trece prin centrul cercului

circumscris triunghiului ABC. Demonstraţie. Deoarece I – centrul cercului înscris în triunghiul ABC - este ortocentrul triunghiului

a b cI I I şi O este centrul cercului medial al triunghiului

a b cI I I rezultă că

dreapta lui Euler a triunghiului a b cI I I este dreapta OI.

33) În triunghiul ABC, fie

a b cH H H triunghiul ortic şi

a b cC C C triunghiul de contact.

Triunghiul a b c

C C C şi triunghiul având vârfurile în centrele cercurilor înscrise în

triunghiurile , ,b c c a a b

AH H BH H CH H au acelaşi cerc al lui Euler.

Demonstraţie. Fie ', ", "'I I I centrele cercurilor înscrise în triunghiurile ,b c c a

AH H BH H

respectiv a b

CH H . Punctele ', ", "'I I I sunt simetricele centrului cercului înscris în

triunghiul ABC faţă de laturile triunghiului de contact (vezi „Triunghiul lui Carnot”) şi cum I este centrul cercului circumscris triunghiului

a b cC C C rezultă că ', ", "'I I I sunt centrele

cercurilor lui Carnot ale triunghiului a b c

C C C , deci triunghiurile ', ", "'I I I şi a b c

C C C au

acelaşi cerc al lui Euler şi aceeaşi dreaptă a lui Euler (vezi „Triunghiul lui Carnot”). 34) Fie ', ', 'A B C punctele de intersecţie ale bisectoarelor interioare ale unghiurilor

, ,A B C ale unui triunghi ABC cu cercul circumscris acestuia şi ", ", "A B C punctele unde perpendicularele din I – centrul cercului înscris în triunghiul ABC – pe laturile BC, CA, AB intersectează a doua oară cercurile circumscrise triunghiurilor AIO, BIO, CIO. Triunghiurile ' ' 'A B C şi " " "A B C au acelaşi cerc al lui Euler. Demonstraţie. Punctele ", ", "A B C sunt centrele cercurilor lui Carnot ale triunghiului

' ' 'A B C (vezi „Punctul lui Karyia”) şi cum triunghiul lui Carnot al unui triunghi dat şi triunghiul dat au acelaşi cerc al lui Euler (vezi „Triunghiul lui Carnot”) rezultă concluzia. 35) Într-un triunghi ABC se proiecteză două vârfuri pe bisectoarea interioară a celui de-al doilea vârf şi pe bisectoarea unghiului format de înălţimile ce pleacă din primele două vârfuri; cele patru puncte obţinute aparţin unui cerc cu centrul pe cercul lui Euler al triunghiului ABC. Demonstraţie. Fie D şi E respectiv 'D şi 'E proiecţiile punctelor B şi C pe bisectoarele

unghiurilor BAC , respectiv BHC (Fig. 113). Avem ( ) 180 ( )m BHC m A= °− (

c bAH HH

fiind patrulater inscriptibil), deci 1

( ') 90 ( ).2

m BHD m A= °− Fie .T BH AD= ∩

1( ) ( ) 90 ( ),

2bm ATH m BTD m A= = °− de unde

' ,BHD BTD≡ deci ' .HD AD Atunci, patrulaterul ' 'DD E E este dreptunghi şi fie F

A

B C aH

bH

Fig. 113

E

H

F

D

A ' T

D'

E '

aM

Page 119: Barbu_TEOREME_FUNDAMENTALE(2008).pdf

126

centrul acestuia. Arătăm că F aparţine cercului lui Euler al triunghiului ABC. Fie 'A mijlocul lui AH şi

aM mijlocul lui BC. În trapezul ' , 'AHD E A F este linie mijlocie, deci

' .A F AE Din trapezul 'BDCE rezultă .a

FM CE Cum CE AE⊥ rezultă ,a

AF FM⊥

deci ( ) 90 ,a

m AFM = ° adică F aparţine cercului Euler al triunghiului ABC.

Observaţie: Proprietatea de mai sus rămâne adevărată şi pentru bisectoarele exterioare. 36) Într-un triunghi oarecare, cercul lui Euler este tangent cercului înscris şi cercurilor exînscrise corespunzătoare. Demonstraţie. Vezi „Teorema lui Feuerbach”. 37) Fie H ortocentrul unui triunghi ABC. Triunghiurile ABC, HAB, HBC, HCA au acelaşi cerc al lui Euler tangent celor 16 cercuri înscrise sau exînscrise acestor patru triunghiuri. Demonstraţie. Deoarece triunghiurile ABC, HAB, HBC, HCA au acelaşi cerc al lui Euler rezultă că cercul lui Euler - conform teoremei lui Feuerbach – este tangent celor 4 cercuri înscrise în aceste triunghiuri şi celor 12 cercuri exînscrise corespunzătoare celor patru triunghiuri. 38) Dreptele lui Euler ale celor patru triunghiuri ale unui patrupunct ortocentric sunt concurente. Demonstraţia rezultă din faptul că cele patru triunghiuri considerate au acelaşi cerc al lui Euler, deci dreptele lui Euler ale lor sunt concurente în centrul cercului lui Euler al triunghiului dat. 39) Fie H ortocentrul unui triunghi ABC şi

a b cH H H triunghiul ortic al acestuia. Cercul

lui Euler al triunghiului ABC se obţine prin inversiunea de centru H şi raport a

HH HA⋅ a cercului circumscris triunghiului ABC. Demonstraţie. Din egalitatea

a b cHH HA HH HB HH HC k⋅ = ⋅ = ⋅ = (vezi „Triunghiul

ortic”) rezultă că punctele A, B, C se obţin din , ,a b c

H H H prin inversiunea de centru H şi

raport k− , deci prin inversiunea J (H,-k) cercurile circumscrise acestor triunghiuri se

corespund, adică cercul lui Euler se obţine prin inversiunea J (H,-k) a cercului circumscris

triunghiului ABC. 40) Fie 9O centrul cercului lui Euler al triunghiului ABC. Atunci:

9 1 8cos sin sin2

RAO A B C= + , unde R este lungimea razei cercului circumscris

triunghiului ABC. Demonstraţie. În triunghiul AHO, 9AO este mediană. Din teorema medianei avem:

2 2 229

2( ).

4

AO AH OHAO

+ −= Dar, , 2 cos ,AO R AH R A= = 2 2(1 8cos cos cos )OH R A B C= −

(vezi „Centrul cercului circumscris unui triunghi”), de unde rezultă:

Page 120: Barbu_TEOREME_FUNDAMENTALE(2008).pdf

127

( )2

29 [1 8cos ( cos cos cos )]

4

RAO A B C B C= + − + + sau ( )

229 1 8cos sin sin ,

4

RAO A B C= + deci

9 1 8cos sin sin .2

RAO A B C= +

Observaţii:

i) Ţinând cont că raza cercului a

H - exînscris corespunzător triunghiului ortic al

triunghiului ascuţitunghic ABC este 2 cos sin sina

R A B Cρ = (vezi „Triunghiul ortic”)

putem scrie 9 4 .2 a

RAO R ρ= ⋅ +

ii) Din cele de mai sus rezultă 9

11 8cos cos cos .

2 2

ROO OH A B C= = − ⋅ ⋅

iii) Aplicând teorema cosinusului în triunghiul 9OAO obţinem: 2 2 29 9

99

cos2

AO AO OOOAO

AO AO

− −= =

( )1 2cos sin sin cos cos 1 2cos cos( )

1 8cos sin sin 1 8cos sin sin

A B C B C A B C

A B C A B C

+ + + ⋅ −=

+ + ⋅ ⋅ sau

9

cos [2cos 1]2cos .

1 8cos sin sin

B CA

OAOA B C

−+

=+ ⋅

iv) Din formula fundamentală a trigonometriei rezultă:

2 2

2 29 9

4cos [1 cos ( )]sin 1 cos ,

1 8cos sin sin

A B COAO OAO

A B C

⋅ − −= − =

+ ⋅ ⋅deci

( )9

2cos sinsin .

1 8cos sin sin

A B COAO

A B C

−=

+ ⋅ ⋅

41) Fie 9O şi I centrul cercului lui Euler, respectiv centrul cercului înscris în triunghiul

ABC. Atunci, 9

2,

2

R rO I

−= unde R şi r sunt razele cercurilor circumscris, respectiv

înscris în triunghiul ABC. Demonstraţie. Din teorema cosinusului aplicată în triunghiul 9IAO rezultă:

2 2 29 9 9 92 cos (1).O I AO AI AO AI O AI= + − ⋅ ⋅ Ţinând cont că

9 1 8cos sin sin ,2

RAO A B C= + 4 sin sin

2 2

B CAI R= (vezi „Cercul înscris”) şi

[ ]9

cos 2cos 12cos

1 8cos sin sin

B CA

O AIA B C

−+

=+ ⋅ ⋅

relaţia (1) devine:

2

2 2 29 1 8 c o s s i n s i n 6 4 s i n s i n

4 2 2

R B CO I A B C= + + ⋅ −

2 22 16cos sin sin cos cos sin sin 16sin sin 16sin sin cos cos2 2 2 2 2 2 2 2 2 2 2 2

B C B C B C B C B C B CA

⋅ ⋅ + − +

22 2 2 2 2 2

9 1 48sin cossin 32 1 2sin sin sin 16sin sin cos cos4 2 2 2 2 2 2 2 2 2

R B C A B C B C B CO I

= + − − −

Page 121: Barbu_TEOREME_FUNDAMENTALE(2008).pdf

128

sau 2

2 2 2 29 1 64sin sin sin 16sin sin sin sin cos cos

4 2 2 2 2 2 2 2 2 2

R A B C B C B C B CO I

= + + − , de

unde 2

2 2 2 29 1 16sin sin sin 64sin sin sin

4 2 2 2 2 2 2

R A B C A B CO I

= − +

şi de aici rezultă că:

2 22 22

9

21 8sin sin sin 1

4 2 2 2 4

R A B C R rO I

R

= − = −

de unde 29 .

2

RO I r= −

Observaţii:

i) Ţinând cont că raza cercului lui Euler este egală cu 2

R şi

2 2

R Rr r

− − =

rezultă că

cercurile lui Euler şi cel înscris în triunghiul ABC sunt tangente interior. ii) Deoarece 9 9 a

O AI O AI≡ (punctele A,I şi aI - centrul cercului A – exînscris fiind

coliniare) rezultă 9 9cos cosa

O AI O AI= şi ţinând cont că 4 cos cos2 2a

B CAI R= (vezi

„Cercurile exînscrise”) printr-o demonstraţie analoagă celei mai de sus se obţine

9 .2a a

RO I r= + Întrucât

2 2a a

R Rr r

+ − =

rezultă că cercurile lui Euler şi cel A – exînscris

corespunzătoare triunghiului ABC sunt tangente exterior. iii) Din cele de mai sus rezultă o demonstraţie trigonometrică a teoremei lui Feuerbach. 42) Cercul lui Euler al triunghiului ABC şi cercul circumscris triunghiului tangenţial al triunghiului ABC se corespund prin inversiunea de centru O şi centru 2R .

Demonstraţie. Vezi „Triunghiul tangenţial”. 43) Fie triunghiul ABC, triunghiul de contact

a b cC C C al său şi

triunghiurile extangentice ,a b c

D D D

a b cE E E şi .

a b cF F F Dreptele lui Euler

ale celor cinci triunghiuri sunt concurente în centrul cercului circumscris triunghiului ABC. Demonstraţie. Fie O centrul cercului circumscris triunghiului ABC şi R raza acestuia,

aI centrul cercului exînscris

corespunzător punctului A, iar ar raza

acestui cerc, 1H ortocentrul

triunghiului a b c

D D D şi 19O centrul

cercului lui Euler al triunghiului

a b cD D D (Fig. 114). Fie 1 2 3A A A

triunghiul ortic al triunghiului

a b cD D D în care cercul său

circumscris este cercul lui Euler al

A

B C

M

P

cD

bD

aD

1A

2A 3A

1S

2S 3S

aI

Fig. 114

Page 122: Barbu_TEOREME_FUNDAMENTALE(2008).pdf

129

triunghiului a b c

D D D , de rază 2ar

şi de centru 1N . Fie 1 2 3S S S triunghiul simetric

triunghiului 1 2 3A A A faţă de punctul 1N . Evident 1 2 3 1 2 3A A A S S S≡ şi cercul circumscris

triunghiului 1 2 3S S S este tot cercul lui Euler al triunghiului a b c

D D D . Fie M mijlocul laturii

b cD D , P mijlocul segmentului 1 .

aH D Punctele 1 1, ,P S A şi M aparţin cercului lui Euler al

triunghiului a b c

D D D , iar patrulaterul 1 1PS MA este dreptunghi având centrul în punctul 1.N

Deoarece 1 c bPA D D⊥ şi 1 1PA S M rezultă 1S M este mediatoarea segmentului ,

b cD D deci

punctul 1S aparţine bisectoarei interioare a unghiului A. Analog, punctele 2S şi 3S aparţin

bisectoarelor exterioare ale unghiurilor B respectiv C ( 2 aS BI∈ şi 3 a

S CI∈ ). Deoarece

b aD I CA⊥ şi 1 3 b a

A A D I⊥ (deoarece 1 3A A este antiparalela laturii a c

D D ). Atunci

1 3 .A A AC Dar 1 3 1 3S S A A datorită faptului că dreptele sunt simetrice faţă de 1,N deci

1 3 .S S AC Analog 1 2S S AB şi 2 3 ,S S BC deci triunghiurile 1 2 3S S S şi ABC sunt

omotetice, prin omotetia de centru aI şi rază 2 /

aR r (deoarece omotetia transformă

triunghiul 1 2 3S S S cu raza cercului circumscris / 2ar în ABC cu raza cercului circumscris

R). Prin această omotetie centrul cercului lui Euler al triunghiului a b c

D D D - adică punctul

1N - se transformă în O, deci punctele 1,aI N şi O sunt coliniare, deci dreapta lui Euler a

triunghiului a b c

D D D trece prin centrul cercului circumscris triunghiului ABC. Analog, se

arată că şi dreptele lui Euler ale celorlalte patru triunghiuri trec prin O. 44) Într-un triunghi ABC cevienele concurente în centrul cercului lui Euler al triunghiului sunt dreptele lui Euler ale triunghiurilor extangentice ale triunghiului ortic al triunghiului ABC. Demonstraţie. Deoarece vârfurile triunghiului ABC sunt centrele cercurilor exînscrise corespunzătoare triunghiului ortic al triunghiului ABC, iar centrul cercului lui Euler al triunghiului ABC este centrul cercului circumscris al triunghiului ortic, teorema este o consecinţă a teoremei precedente. 45) Fie un triunghi ABC înscris în cercul de centru O. Dacă , ,

a b cO O O sunt simetricele

lui O faţă de BC, CA şi respectiv AB, să se arate că dreptele , ,a b c

AO BO CO sunt

concurente în 9 ,O centrul cercului lui Euler al triunghiului ABC. Demonstraţie. Soluţia 1. Fie D punctul diametral opus lui A pe cercul circumscris triunghiului ABC, H ortocentrul triunghiului ( , ,

a b cH H H sunt picioarele

perpendicularelor coborâte din vârfurile A respectiv B, C), iar ,

a bM M şi

cM

mijloacele laturilor BC, AC respectiv AB (Fig. 115). DA fiind diametru, rezultă că BD AB⊥ şi DC AC⊥ . Deoarece

⊥b

BH AC şi ⊥c

CH AB rezultă că

b

BH DC şi c

CH BD , deci BDCH

este paralelogram; atunci diagonala DH trece prin mijlocul

aM al lui BC (deci

A

B

C

D

H

aM

bM

cM

aH

bH

cH

CO

BO

AO

9O O

Fig. 115

Page 123: Barbu_TEOREME_FUNDAMENTALE(2008).pdf

130

∩ =a

BC HD M ). Cum O este punctul de intersecţie al mediatoarelor triunghiului ABC,

rezultă că ,⊥ ⊥a

OM BC AH BC ,deci A

AH OO . Deoarece ≡AO OD , rezultă că a

OM

este linie mijlocie în triunghiul AHD; de aici avem 2

=a

AHOM , adică

2= =A a

OO OM AH . Patrulaterul A

AHO O fiind paralelogram diagonalele HO şi A

AO se

înjumătăţesc, deci 9 ∩ =A

AO OH O . Analog se demonstrează că 9 ∩ =B

BO OH O şi

9 ∩ =C

CO OH O ;deci 9 ∩ ∩ =A B C

AO BO CO O .

Soluţia 2. Notăm mijloacele segmentelor , , ,A B C

AO BO CO BC ,OH respectiv cu * * *, , ,A B C

aM , 9 .O Din relaţia lui Sylvester rezultă: = + +

uuuur uuur uuur uuurOH OA OB OC , rezultă

9 2

+ +=

uuur uuur uuuruuuur OA OB OCOO . Întrucât ,= =

a a a A aBM M C M O O M , rezultă că

ABOCO este

paralelogram, deci A

OO OB OC= =uuuur uuur uuur

; *A fiind mijlocul lui A

AO , avem

*2 2

AOA OO AO OB OCOA

+ + += =

uuur uuuur uuur uuur uuuruuuur

. Analog, obţinem *2

OA OB OCOB

+ +=

uuur uuur uuuruuuur

şi

*2

OA OB OCOC

+ +=

uuur uuur uuuruuuuur

. Deci, 9 * * *2

+ += = = =

uuur uuur uuuruuuur uuuuur uuuuur uuuuur OA OB OCOO OA OB OC . De aici deducem

9 * * *= = =O A B C şi prin urmare 9 .∩ ∩ =A B C

O A O B O C O

46) Dacă I este centrul cercului înscris în triunghiul ABC, atunci dreptele lui Euler ale triunghiurilor BCI, CAI, ABI şi ABC sunt concurente. Demonstraţie. Vezi „Punctul lui Schiffler”. 47) Centrul cercului lui Euler al triunghiului ABC este mijlocul segmentului ce uneşte punctele lui Hexyl şi Prasolov. Demonstraţie. Vezi „Punctul lui Hexyl”. 48) Fie

a b cH H H triunghiul ortic al triunghiului ABC, H ortocentrul triunghiului ABC.

Paralelele duse prin H la dreptele ,b c a c

H H H H şi a b

H H intersectează dreptele BC, CA respectiv AB în D, E, F iar paralelele duse prin H la dreptele BC, CA, AB intersectează dreptele , ,

b c a c a bH H H H H H în ', 'D E respectiv '.F Punctele , ,D E F şi ', ', 'D E F

aparţin unor drepte perpendiculare pe dreapta lui Euler a triunghiului ABC. Demonstraţie. Vezi „Triunghiul ortic”. 49) Fie M un punct pe un cerc C ( , ),O R H simetricul lui O faţă de M şi P un punct

arbitrar ales pe cercul C ' ,2

RM dar aflat în interiorul discului ( , ).D O R Dacă

= ∩A HP C iar perpendiculara pe dreapta HP în P intersectează cercul C în punctele B şi C, atunci punctul M este centrul cercului celor nouă puncte al triunghiului ABC.

Page 124: Barbu_TEOREME_FUNDAMENTALE(2008).pdf

131

Demonstraţie. Evident O este centrul cercului circumscris triunghiului ABC (Fig. 116). Fie

aM

mijlocul segmentului BC. Atunci , .

a aOM BC OM HP⊥ În trapezul

aOM HP , M

este mijlocul diagonalei OH, deci perpendiculara dusă din M pe

aPM cade în mijlocul

segmentului ,a

PM de unde rezultă că

,2a

RPM MM

≡ =

adică punctul ∈a

M C’

Atunci cercul C’ ce trece prin piciorul înălţimii

din A, prin mijlocul laturii BC şi are raza jumătate din raza cercului circumscris triunghiului ABC este cercul lui Euler al triunghiului ABC, deci M este centrul cercului lui Euler al triunghiului ABC. 50) Fie

a b cM M M triunghiul median al triunghiului ABC, ( ), ( ), ( )X BC Y CA Z AB∈ ∈ ∈

astfel încât 2 ( ), 2 ( ),a a b b

M X d tg B C M Y d tg C A= ⋅ − = ⋅ − 2 ( ),c c

M Z d tg A B= ⋅ −

, ,a b cd d d fiind distanţele de la centrul cercului lui Euler al triunghiului ABC la laturile BC, CA, respectiv AB. Dreptele AX, BZ şi CY sunt concurente. Demonstraţie. Fie

a b cH H H triunghiul ortic al triunghiului ABC şi 9O centrul cercului

circumscris triunghiului ABC şi P proiecţia lui 9O pe BC (Fig.117) . Avem:

cos .2a a a a

aM H BH BM c B= − = − Din triunghiul 9 a

O PM avem:

( )22 2 2 2 2 22 cos /2 [2 sin cos sin( )] [sin cos sin cos ]

2 4 4 4 4 4a

c B aR R R C B R B C R R C B B Cd

− − + − = − = − = − =

2 2 2sin ( )

4 4

R R B C−− =

22cos ( )

4

RB C− de unde

rezultă cos( ).2a

Rd B C= − Atunci,

sin( ) sin sin( )2a a

aXB BM M X R B C R A R B C= − = − − = − − =

2 cos sin .R B C Analog se arată că

2 cos sin ,XC R C B= deci .BX tgB

CX tgC= Analog se

arată că YC tgC

YA tgA= şi ,

ZA tgA

ZB tgB= de unde

1BX YC ZA

CX YA ZB⋅ ⋅ = iar din reciproca teoremei lui

Ceva rezultă că dreptele AX, BY şi CZ sunt concurente.

A

B C

M

aM

M' P

H

Fig. 116

A

B C aH aM

Fig. 117

P

ad

9O

X

Page 125: Barbu_TEOREME_FUNDAMENTALE(2008).pdf

132

51) Fie a b c

M M M şi a b c

H H H triunghiul median, respectiv triunghiul ortic al unui

triunghi neisoscel şi nedreptunghic ABC, iar ,b c c b

X H M H M= ∩

,a c c a

Y H M H M= ∩ a b b a

Z H M H M= ∩ . Punctele X, Y, Z aparţin dreptei lui Euler a triunghiului ABC. Demonstraţie. Din teorema lui Pappus aplicată punctelor coliniare , ,

c cB H M , respectiv

, ,b b

C M H , rezultă că punctele , b c b c

H BH CH G BH CM= ∩ = ∩ şi

c b b c

X H M H M= ∩ sunt coliniare. Analog se arată că punctele Y şi Z aparţin dreptei HG.

52) Fie triunghiul ABC şi punctul M situat pe cercul circumscris triunghiului ABC. Să se arate că triunghiul ale cărui vârfuri sunt centrele cercurilor lui Euler corespunzătoare triunghiurilor MAB, MBC şi respectiv MCA este asemenea cu triunghiul ABC. Demonstraţie. Notăm cu litere mici afixele punctelor corespunzătoare şi fie 1 2 3

9 9 9, ,O O O

centrele cercurilor lui Euler ale triunghiurilor MAB, MBC şi respectiv MCA . Afixele

punctelor 1 2 39 9 9, ,O O O sunt: 1 2 3, ,

2 2 2

m a b m b c m a cω ω ω

+ + + + + += = = . Atunci,

1 2| |2

a cω ω

−− = sau 1 2

9 9 2

ACO O = , 2 3

| || |

2

b aω ω

−− = sau 2 3

9 9 2

ABO O = şi

3 1| |

| |2

c bω ω

−− = sau 1 3

9 9 2

BCO O = . Avem:

1 2 2 3 3 19 9 9 9 9 9 1

2

O O O O O O

CA AB BC= = = , deci triunghiurile

1 2 39 9 9O O O şi ABC sunt asemenea.

53) Simetricele dreptei lui Euler a unui triunghi ABC în raport cu laturile triunghiului ABC sunt concurente într-un punct ce aparţine cercului circumscris triunghiului ABC. Demonstraţie. Fie 1 2 3, ,E E E

punctele de intersecţie dintre dreapta lui Euler a triunghiului ABC şi dreptele BC,CA, respectiv AB (Fig. 118). Simetricele ortocentrului H al triunghiului ABC faţă de laturile triunghiului sunt punctele , ,

h h hA B C de intersecţie a

înălţimilor AH,BH,CH cu cercul circumscris (vezi „Ortocentrul unui triunghi”). Demonstrăm că dreptele

1 2 3, ,h h hA E B E C E sunt concurente.

Centrele cercurilor Carnot ( , , )

a b cO O O aparţin dreptelor

1 2 3, ,h h hA E B E C E . Deoarece

triunghiurile h h hA B C şi

a b cO O O

sunt omologice, centrul de omologie aparţinând cercului circumscris triunghiului ABC (vezi „Triunghiul lui Carnot”), rezultă concluzia.

A

B C hA

hB

Fig. 118

O

H

P

hC

aO

bO cO

1E

2E

3E

Page 126: Barbu_TEOREME_FUNDAMENTALE(2008).pdf

133

54) Simetricele dreptei lui Euler a unui triunghi ABC în raport cu laturile triunghiului având vârfurile în punctele euleriene ale triunghiului ABC sunt concurente într-un punct ce aparţine cercului lui Euler al triunghiului ABC. Demonstraţie. Fie ', ', 'A B C mijloacele segmentelor AH,BH, respectiv CH. Punctul H este şi ortocentrul triunghiului ' ' 'A B C , iar centrul cercului circumscris triunghiului ' ' 'A B C este 9O (centrul cercului lui Euler), deci dreapta lui Euler a triunghiului ' ' 'A B C este 9HO ,

adică tocmai dreapta lui Euler a triunghiului ABC. Conform proprietăţii precedente rezultă că simetricele dreptei lui Euler triunghiului ' ' 'A B C în raport cu laturile acestuia sunt concurente într-un punct ce aparţine cercului lui Euler al triunghiului ' ' 'A B C , adică cercul lui Euler al triunghiului ABC. 55) Dreapta lui Euler a unui triunghi ABC este perpendiculară pe axa de omologie dintre triunghiul tangenţial şi triunghiul median al triunghiului ABC. Demonstraţie. Vezi „Triunghiul tangenţial”. 56) Fie

a b cM M M triunghiul median corespunzător unui triunghi ABC şi O centrul

cercului circumscris triunghiului ABC . Pe dreptele a

OM ,b

OM ,c

OM se consideră

punctele 1A , 1B , 1C astfel încât 1 1 1

a b c

OA OB OC

OM OM OM= = . Dreptele 1AA , 1BB , 1CC sunt

concurente într-un punct ce aparţine dreptei lui Euler a triunghiului ABC .

Demonstraţie. Vezi „Teorema lui Boutin”. 57) Dacă centrul cercului înscris (I) al unui triunghi ABC aparţine dreptei lui Euler a triunghiului, atunci triunghiul ABC este isoscel. Demonstraţie. Presupunem că triunghiul ABC nu este isoscel şi fie ', 'A B punctele de intersecţie al bisectoarelor AI, BI cu cercul circumscris triunghiului ABC. Atunci,

' ',

OI OA OI OB

IH AH IH BH= = şi cum ' '( )OA OB R≡ = rezultă AH BH≡ , adică AC BC≡ ,

contradicţie. 58) Fie

a b cC C C triunghiul de contact al unui triunghi .ABC Dreapta lui Euler a

triunghiului a b c

C C C trece prin centrul cercului lui Euler al triunghiului .ABC

Demonstraţie. Fie 1H ortocentrul triunghiului de contact

şi 1 1 1, ,a b c

C A C B C C înălţimile triunghiului de contact

(Fig.119). Atunci, dreapta lui Euler a triunghiului

a b cC C C este dreapta 1IH , iar 1H este centrul cercului

înscris în triunghiul 1 1 1A BC (vezi „Triunghiul ortic”).

Deoarece triunghiurile ABC şi 1 1 1A BC sunt omotetice

(vezi „Triunghiul de contact”) rezultă că prin această omotetie punctele 1H şi I se corespund şi totodată

centrele cercurilor lui Euler ale celor două triunghiuri, deci centrul cercului lui Euler al triunghiului ABC aparţine dreptei 1IH .

A

B C

1H

bC

aC

I cC

Fig. 119

1B

1A

1C

Page 127: Barbu_TEOREME_FUNDAMENTALE(2008).pdf

134

I.32. Dreapta lui Gauss44

„Cel mai frumos lucru pe care-l putem experimenta este misterul... este sursa tuturor adevărurilor şi ştiinţei” – Albert Einstein45

Teorema lui Gauss Fie triunghiul ABC. O dreaptă d intersectează dreptele BC, CA, AB în punctele

' [ , ' [ ], ' [ ].A BC B CA A AB∈ ∈ ∈ Mijloacele segmentelor ', 'AA BB şi 'CC sunt coliniare. Demonstraţie.

Soluţia 1. Fie ", ", "A B C mijloacele segmentelor ', 'AA BB respectiv 'CC şi , ,a b c

M M M

mijloacele segmentelor BC, CA, respectiv AB (Fig. 120). Din teorema lui Menelaus pentru

triunghiul ABC şi transversala ' ' 'A B C− − rezultă: ' ' '

1' ' '

A B B C C A

A C B A C B⋅ ⋅ = relaţie care este

echivalentă cu ' / 2 ' / 2 ' / 2

1,' / 2 ' / 2 ' / 2

A B B C C A

A C B A C B⋅ ⋅ = adică

" " "1

" " "c a b

b c a

A M B M C M

A M B M C M⋅ ⋅ = , (deoarece

punctele ", , ; ", ,b c c a

A M M B M M , respectiv ", ,a b

C M M sunt coliniare) şi din

reciproca teoremei lui Menelaus pentru triunghiul a b c

M M M şi punctele

" [ \ [ ], " [ ], " [ ]c b c b a b b a

A M M M M B M M C M M∈ ∈ ∈ rezultă că punctele ", ", "A B C sunt

coliniare.

Soluţia 2. Notând 'BA

pBC

= şi , , ''

BAq BC u BC v

BC= = =

uuur r uuuur r, atunci ' , ,BA pu BA qv= =

uuuur r uuur r

1" ( ),

2BC u v= +uuuuur r r

1

" ( )2

BA pu qv= +uuuur r r

. Notând ' '

,' '

B Ar

B C=

'

'

B Ct

B A= , avem

'1 1

pu rv tqv uBB

r t

+ += =

+ +

r r r ruuuur

, de unde rezultă că 1

,1 1 1 1

p r tq

r t r t= =

+ + + +. Obţinem

rt

pq= şi

( 1)

1

q pr

q

−=

− înlocuind în ' 2 "BB BB=

uuuur uuuur obţinem

1" ( 1) ( 1)

2( 1)BB p u q p v

pq = − + − −

uuuur r r. Din

44 Carl Friedrich Gauss (1777-1855) – matematician german, profesor la Universitatea Göttingen, contribuţii

majore în toate ramurile matematicii 45 Albert Einstein (1879-1955) – fizician german, profesor universitar la Berlin şi Princeton, laureat al Premiului Nobel

A

B C

bM

cM A"

Fig. 120

A'

B'

C'

aM

B"

C"

Page 128: Barbu_TEOREME_FUNDAMENTALE(2008).pdf

135

1" " " " ( 1) ( 1)

2C A C B BA p u q v = + = − + − uuuuuur uuuuur uuuur r r

rezultă " " " "B A B B BA= + =uuuuuur uuuur uuuur

( 1) ( 1) " "2( 1) 2( 1)

pq pqp u q v C A

pq pq − + − = − −

r r uuuuuur. Prin urmare punctele ", ", "A B C sunt

coliniare.

Observaţii: 1) Dreapta pe care se află punctele ", ", "A B C se numeşte dreapta lui Newton – Gauss. 2) ' ' 'BCB C A A se numeşte patrulater complet. Fie DEF triunghiul cevian al unui punct T în raport cu triunghiul ABC ( , ,D BC E AC F AB∈ ∈ ∈ ). Dreptele care unesc mijloacele laturilor triunghiului DEF cu mijloacele laturilor triunghiului ABC sunt concurente. Demonstraţie.

Fie M, N, P mijloacele segmentelor AT, BT, respectiv CT; ', ', 'D E F mijloacele

segmentelor EF, DF, respectiv DE; , ,a b c

M M M mijloacele laturilor BC, CA respectiv AB.

Teorema lui Gauss aplicată patrulaterului complet AETFBC ne dă faptul că mijloacele diagonalelor AT, EF şi BC (adică punctele , 'M D şi

aM ) sunt coliniare. Analog, punctele

, ',b

N E M sunt coliniare şi , ',c

P F M sunt coliniare. Deoarece 2

= =b a

CTMM NM şi

b a

MM NM ( ,b a

MM NM fiind linii mijlocii în triunghiurile ATC şi BTC) rezultă că

patrulaterul b a

MM M T este paralelogram, deci diagonalele sale a

MM şi b

NM se

înjumătăţesc şi fac .= ∩a b

Q MM NM Analog, patrulaterul a c

PM M M este paralelogram

şi deci dreapta c

PM trece tot prin mijlocul lui a

MM , adică prin Q. Dreptele ', 'a b

M D M E ,

respectiv 'c

M F sunt concurente în punctul Q.

A

B

C D

E

F

T

M

N

P

D'

F' E '

aM

bM cM

Fig. 121

Page 129: Barbu_TEOREME_FUNDAMENTALE(2008).pdf

136

I.33. Dreapta lui Brocard46

„Cel ce caută metode de rezolvare fără a avea o problemă bine definită în minte, caută în cea mai mare parte în zadar.”- David Hilbert47

Dreapta care uneşte centrul cercului circumscris (O) cu punctul lui Lemoine (K) se numeşte dreapta lui Brocard. 1) Dacă K este punctul lui Lemoine al triunghiului ABC, atunci

2 2 22 2

2 2 2 2

3

( )

a b cOK R

a b c= −

+ +.

Demonstraţie. Egalitatea 2 2 2 2 2 2 2 2 2

22 2 2 2 2 2 2

3

( )

a MA b MB c MC a b cMK

a b c a b c

+ += −

+ + + + (vezi

„Teorema lui Van-Aubel”) pentru M O≡ , devine 2 2 2

2 22 2 2 2

3

( )

a b cOK R

a b c= −

+ +.

2) 21 4sin

cos

−=R

KOω

ω, ω este unghiul lui Brocard.

Demonstraţie. Vezi „Punctele lui Brocard”. 3) Centrul cercului circumscris triunghiului Grebe aparţine axei Brocard a triunghiului ABC. Demonstraţie. Vezi „Triunghiul lui Grebe”. 4) Punctele izodinamice ale triunghiului ABC neechilateral sunt punctele de intersecţie dintre dreapta lui Brocard şi cercurile lui Apollonius. Demonstraţie. Vezi „Puncte izodinamice”. 5) Consecinţă: Punctele izodinamice aparţin axei Brocard OK . Demonstraţie. Vezi „Puncte izodinamice”. 6) Dreapta lui Brocard OK este perpendiculară pe dreapta Lemoine a triunghiului ABC. Demonstraţie. Vezi „Dreapta lui Lemoine”. Observaţie: Punctul de intersecţie dintre dreapta lui Brocard şi dreapta lui Lemoine se numeşte punctul lui Schoute. 7) Dreapta lui Simson a punctului lui Steiner în raport cu un triunghi ABC este paralelă cu dreapta lui Brocard.. Demonstraţie. Vezi „Punctul lui Steiner”. 8) Paralela dusă prin A la dreapta lui Brocard intersectează cercul circumscris triunghiului ABC în punctul σ . Perpendiculara dusă din punctul σ pe dreapta BC trece prin punctul lui Steiner al triunghiului ABC. Demonstraţie. Vezi „Punctul lui Steiner”.

46 Henri Brocard (1845-1922) – matematician francez, contribuţii importante în geometrie 47 David Hilbert (1962-1943) – matematician german, profesor la Universitatea din Göttingen, contribuţii

remarcabile în geometrie şi analiza matematică

Page 130: Barbu_TEOREME_FUNDAMENTALE(2008).pdf

137

9) Punctul lui Kenmotu aparţine dreptei lui Brocard a triunghiului ABC. Demonstraţie. Vezi „Punctul lui Kenmotu”.

I.34. Dreapta ortică „Matematica este cea mai educativă dintre toate materiile şcolare, deoarece atinge în gradul cel mai înalt şi elementele cele mai fine ale inteligenţei şi părţile cele mai cristaline ale sufletului omenesc.” Gh. Ţiţeica48

1) Fie ABC un triunghi neisoscel şi nedreptunghic, iar

a b cH H H triunghiul său ortic.

Dacǎ ' , ' , 'b c c a a b

A BC H H B AC H H C AB H H= ∩ = ∩ = ∩ , atunci punctele

', ', ' A B C sunt coliniare. Demonstraţie. Din teorema lui Menelaus aplicată în triunghiul ABC cu transversalele

' ,c b

A H H− − ' ,c a

B H H− − ' a b

C H H− − rezultă: '

1'

b c

b c

H C H AA B

A C H A H B⋅ ⋅ = ,

'1

'c a

c aB

H A H BB C

B A H H C⋅ ⋅ = ,

'1

'a b

a b

H B H CC A

C B H C H A⋅ ⋅ = . Înmulţind membru cu membru relaţiile precedente rezultă:

' ' '1

' ' '

A B B C C A

A C B A C B⋅ ⋅ = (deoarece dreptele

aAH ,

bBH şi

cCH sunt concurente în ortocentrul

48 Gheorghe Ţiţeica (1873-1939) – matematician român, profesor la Universitatea din Bucureşti, membru al

Academiei Române, contribuţii importante în geometrie

Ha

Hb

A

Fig. 122

B

Hc

C

H

A'

B'

C'

Page 131: Barbu_TEOREME_FUNDAMENTALE(2008).pdf

138

triunghiului ABC, deci 1a b c

a b c

H B H C H A

H C H A H B⋅ ⋅ = ). Din reciproca teoremei lui Menelaus rezultă

ca punctele ', ', ' A B C sunt coliniare. Observaţie: Dreapta pe care se găsesc punctele ', ', ' A B C se numeşte dreapta (axa) ortică a triunghiului ABC. 2) Triunghiul ABC este omologic cu triunghiul său ortic, dreapta de omologie fiind axa ortică a triunghiului ABC. Demonstraţia rezultă din proprietatea de mai sus. 3) Axa ortică a unui triunghi este axa radicală a fasciculului lui Griffiths al triunghiului (fasciculul de cercuri determinat de cercul circumscris şi de cercul lui Euler al unui triunghi). Demonstraţie. Fie ' ,

b cA BC H H= ∩

' ,c a

B AC H H= ∩

'a b

C AB H H= ∩ , unde

a b cH H H este triunghiul

ortic al triunghiului ABC (Fig. 123). Pentru a arăta că axa ortică ' 'A B este axa radicală a fasciculului lui Griffiths al triunghiului ABC este suficient sa arătăm că punctele 'A şi 'B au puteri egale faţă de cercul circumscris şi cercul lui Euler al triunghiului ABC. Patrulaterul

b cBCH H fiind

inscriptibil rezultă ' ' ' '

c bA H A H A B A C⋅ = ⋅ ,

adică puterile punctului 'A faţă de cele două cercuri sunt egale. Analog, se arată că puterile punctului 'B faţă de cercul circumscris şi cercul lui Euler al triunghiului ABC sunt egale. 4) Axa ortică a unui triunghi este perpendiculară pe dreapta lui Euler a triunghiului. Demonstraţie. Proprietatea este evidentă deoarece axa radicală a doua cercuri este perpendiculară pe dreapta care trece prin centrele cercurilor. 5) Axa ortică a triunghiului antisuplementar

a b cI I I este dreapta determinată de

picioarele bisectoarelor exterioare ale triunghiului ABC. Demonstraţia este evidentă deoarece triunghiul ortic al triunghiului

a b cI I I este triunghiul

ABC şi I -centrul cercului înscris în triunghiul ABC - este ortocentrul triunghiului a b cI I I .

A

B C aH

bH

cH

Fig. 123

O

H

A'

B'

C'

9O

Page 132: Barbu_TEOREME_FUNDAMENTALE(2008).pdf

139

Observaţie: Deoarece axa ortică a unui triunghi este perpendiculară pe dreapta lui Euler a triunghiului rezultă ca dreapta ce uneşte picioarele bisectoarelor exterioare ale unui triunghi (dreapta antiortică) este perpendiculară pe dreapta OI. 6) Diametrele ce pleacă din vârfurile triunghiului ABC, în cercul circumscris triunghiului ABC, intersectează laturile opuse în punctele 1 1,A B , respectiv 1C ; L, M şi N

sunt mijloacele segmentelor 1 1,AA BB , respectiv 1CC . Triunghiurile LMN şi ABC sunt omologice, axa de omologie fiind axa ortică a triunghiului ABC. Demonstraţie. Triunghiurile ABC şi LMN sunt evident omologice, centrul de omologie fiind centrul cercului circumscris triunghiului ABC. Fie P LM AB= I şi , ,

a b cM M M mijloacele

laturilor BC, CA, respectiv AB. Fără a restrânge generalitatea presupunem că BC CA< şi

a bQ M M LM= I (Fig. 124). Din asemănarea triunghiurilor

cPMM cu

aQMM ,

respectiv c

PLM cu b

QLM avem: 1 1

1 1

a b a b a b

c c c c c

M M QM QM MM LM BC AC

PM PM PM MM LM BA AB= − = − = − =

sin 2 sin 2 2sin( ) cos( )

sin 2 sin 2 2sin cos

A B A B A B

C C C C

− +− = =

sin( )

sin( )

A B tgA tgB

A B tgA tgB

− − − +=

+ +. Atunci,

c c c a b

c c c a b

PM M A PM M MPA tgB

PB PM M A PM M M tgA

+ += = =

− −, deci punctul P împarte latura AB, exterior, în

raportul tgB

tgA. Deoarece înălţimea din C împarte AB, înterior, în aceeaşi raţie rezultă că P

aparţine axei ortice a triunghiului ABC.

A

B C

O

Fig. 124

1A

1B 1C

aM

bM cM

L

M N

P

Q

Page 133: Barbu_TEOREME_FUNDAMENTALE(2008).pdf

140

I.35. Dreapta antiortică

„Matematica este arta de a da aceleaşi nume la diferite lucruri.” - Henri Poincaré49

1) Fie ABC un triunghi neisoscel. Bisectoarea exterioară corespunzătoare vârfului A intersectează dreapta BC în 'A şi analog se obţin punctele 'B şi ' .C Punctele

', ', 'A B C sunt coliniare. Demonstraţie. Notăm cu , ,a b c lungimile laturilor

triunghiului ABC .Din teorema bisectoarei avem:

' ',

' '

A B c B C a

A C b B A c= = şi

'

'

C A b

C B a= . Atunci,

' ' '1

' ' '

A B B C C A

A C B A C B⋅ ⋅ = şi din

reciproca teoremei lui Menelaus rezultă că puntele ', ', 'A B C sunt coliniare.

Observaţii: i) Dreapta pe care se găsesc punctele ', ', 'A B C se numeşte dreapta (axa) antiortică a triunghiului ABC . ii) Punctele de intersecţie dintre bisectoarele exterioare ale unghiurilor unui triunghi neisoscel cu dreptele suport ale laturilor opuse aparţin axei antiortice. 2) Dreapta antiortică a triunghiului ABC este axa ortică a triunghiului antisuplementar corespunzător triunghiului ABC. Demonstraţie. Vezi „Triunghiul antisuplementar”. 3) Dreapta antiortică a triunghiului ABC este perpendiculară pe dreapta OI , unde O şi I sunt centrele cercurilor circumscris, respectiv înscris în triunghiul ABC . Demonstraţie. Deoarece axa ortică a unui triunghi este perpendiculară pe dreapta lui Euler a triunghiului ( vezi „Dreapta ortică” ) rezultă că dreapta antiortică a triunghiului ABC -

care este axa ortică a triunghiului a b cI I I al triunghiului ABC - este perpendiculară pe

dreapta OI , deoarece O este centrul cercului celor nouă puncte al triunghiului a b cI I I şi

I este ortocentrul aceluiaşi triunghi ( vezi „Triunghiuri exînscrise” ). 4) Dreapta antiortică a unui triunghi ABC este axa de omologie dintre triunghiul ABC şi triunghiul său extangenţial. Demonstraţie. vezi „Triunghiul extangenţial”. 5) Axa antiortică a unui triunghi este polara triliniară a centrului său înscris.

49 Henri Poincaré ( 1854 -1912) – matamatician şi fizician francez, contribuţii importante în toate ramurile matematicii

A

A'

c b

C a B

Fig. 125

Page 134: Barbu_TEOREME_FUNDAMENTALE(2008).pdf

141

Demonstraţie. Fie 'A punctul de intersecţie dintre bisectoarea exterioară a unghiului A şi dreapta BC, 1 1 1, ,A B C picioarele bisectoarelor interioare şi a, b, c lungimile laturilor BC,

CA, respectiv AB (Fig.125). Din teorema bisectoarei rezultă 1 1

1 1

', ,

'= = =

C A BCA B c b a

A C b C B a B A c,

de unde 1 1

1 1

'1

'⋅ ⋅ =C A BCA B

A C C B B Aşi din reciproca teoremei lui Menelaus rezultă că punctele

1 1', ,A B C sunt coliniare, deci dreapta 1 1BC intersectează dreapta BC în punctul 'A ce

aparţine axei antiortice a triunghiului ABC. Analog se arată că punctul de intersecţie dintre dreapta 1 1AC şi dreapta AC coincide cu punctul de intersecţie dintre bisectoarea exterioară

a unghiului B şi dreapta AC, deci axa antiortică coincide cu polara triliniară a centrului cercului înscris în triunghiul ABC.

I.36. Dreapta lui Simson50

„De la toţi am îınvăţat. Mă surprind uneori vorbind olimpian ca Pompeiu, apăsat ca Ţiţeica, senin şi simplu ca David Emanuel. Căci noi nu suntem numai fiii părinţilor noştri, ci şi fiii profesorilor noştri.”- Miron Nicolescu51

Teorema lui Simson

Proiecţiile unui punct de pe cercul circumscris unui triunghi, pe laturile acestuia, sunt coliniare. Demonstraţie. Soluţia 1. Fie M un punct pe cercul circumscris unui triunghi ABC şi

', ', 'A B C proiecţiile punctului M pe dreptele BC, CA respectiv AB (Fig. 126). Patrulaterele

' ', ' 'MC AB MB A C sunt inscriptibile. Avem: ( ' ') ( ') 90 ( ' )m AB C m AMC m C AM= = °− =

90 ( ) ( ' ) ( ' ' ),m BCM m A MC m A B C° − = = adică unghiurile ' 'AB C şi ' 'A B C sunt

opuse la vârf, deci punctele ', 'A B şi 'C sunt coliniare. Soluţia 2. Notăm cu litere mici afixele punctelor corespunzătoare. Pentru ca punctele

', ', 'A B C să fie coliniare este suficient să arătăm că:

' '.

' '

b a

b c

−∈

− Patrulaterul ' 'MCA B fiind inscriptibil,

punctele , , ', 'M C A B sunt conciclice,

adică:' '

'

b a c m

b m c a

− −⋅ ∈

− − (1). Patrulaterul ' 'MCA B este

inscriptibil, deci ' '

' '

b m a c

b c a m

− −⋅ ∈

− − (2). Înmulţind relaţiile

(1) şi (2) rezultă: ' ' '

' ' '

b a c m a c

b c c a a m

− − −⋅ ⋅ ∈

− − − (3). Punctele A,

B, C şi M fiind conciclice rezultă: a m c b

a b c m

− −⋅ ∈

− − (4).

Înmulţind relaţiile (3) şi (4) rezultă:

50 Robert Simson (1687-1768) – matematician scoţian, profesor la Universitatea din Edinburgh, contribuţii în geometrie 51 Miron Nicolescu (1903-1975) – matematician ramân, membru al Academiei Române, contribuţii în analiza matematică

A

B C

M

A'

B'

C'

Fig. 126

Page 135: Barbu_TEOREME_FUNDAMENTALE(2008).pdf

142

' ' '

' ' '

b a a c c b

b c a b c a

− − −⋅ ⋅ ∈

− − − (5). Punctele , ',A C B fiind coliniare rezultă

'a c

a b

−∈

− (6) şi

analog punctele , ',B A C fiind coliniare '

c b

c a

−∈

− (7). Din relaţiile (5), (6) şi (7) rezultă

' ',

' '

b a

b c

−∈

− adică punctele ', 'A B şi 'C sunt coliniare.

Reciproca teoremei lui Simson

Dacă M este un punct situat în exteriorul triunghiului ABC şi proiecţiile ', ', 'A B C ale punctului M pe dreptele BC, AC respectiv AB sunt coliniare, atunci punctul M se află pe cercul circumscris triunghiului. Demonstraţie. Punctele ', ', 'A B C fiind coliniare rezultă ( ' ') ( ' ' ).m AB C m A B C=

Atunci, ( ' ') ( ') 90 ( ')m AB C m AMC m MAC= = °− şi

( ' ' ) ( ' ) 90 ( ' )m A B C m A MC m A CM= = °− , de unde rezultă

( ') ( ),m MAC m MCB= adică patrulaterul MABC este inscriptibil, deci punctul M se află

pe cercul circumscris triunghiului ABC.

Observaţie: Dreapta care conţine punctele ', ', 'A B C se numeşte dreapta lui Simson a punctului M în raport cu triunghiul ABC, iar punctul M se numeşte polul dreptei lui Simson. 1) Perpendiculara coborâtă dintr-un punct M situat pe cercul circumscris triunghiului ABC intersectează din nou cercul în '.M Dreapta lui Simson a punctului M în raport cu triunghiul ABC este paralelă cu '.AM

Demonstraţie. Avem 1( ' ) ( ' ) ( ' ),

2m M AC m M MC m M C= = iar din patrulaterul

inscriptibil ' 'A B MC rezultă ' ' ' 'A B C A MC M MC≡ ≡ de unde ' ' ' ,M AC A B C≡ adică ' ' '.AM A C

Teorema lui Steiner 2) Dreapta lui Simson a unui punct M în raport cu triunghiul ABC trece prin mijlocul segmentului determinat de punctul M şi ortocentrul H al triunghiului ABC. Demonstraţie. Fie ' 'A B dreapta lui Simson a punctului M,

aH piciorul înălţimii din A pe BC,

1 a

A AH= I C, ' 'M MA= I C (unde C este cercul

circumscris triunghiului ABC), 1 P MA BC= I şi

'S AM BC= ∩ (Fig. 127). Este cunoscut faptul că

1A este simetricul ortocentrului H faţă de latura BC.

Din 1'MM AA rezultă 1 1( ) ( ')m AAM m AMM= =

1( ),

2m AM iar 1 1( ) ( ')m AAM m AMM= (alterne

interne), deci 1 1AAM A AM≡ (1). Atunci,

( ) 90 ( )a

m ASP m H AS= °− (2) şi

1 1( ) 90 ( ) ( )a a

m H PA m H AP m MPC= °− = (3).

Din relaţiile (1), (2) şi (3) rezultă MPS ASP≡

A

B C

M'

A'

M C'

H

P S

1A

aH

O

E L

B'

Fig. 127

Page 136: Barbu_TEOREME_FUNDAMENTALE(2008).pdf

143

(4). Deoarece ' 'A C AS rezultă că triunghiul 'LPA este isoscel (unde ' 'L MP A C= I ),

deci 'LP LA LM≡ ≡ (deoarece triunghiul 'PA M este dreptunghic în 'A ). Cum L este mijlocul lui PM şi ' ' 'HP AM A C rezultă că dreapta lui Simson trece prin mijlocul segmentului HM. 3) Mijlocul segmentului determinat de punctul M şi ortocentrul triunghiului ABC aparţine cercului lui Euler al triunghiului ABC. Demonstraţie. Fie O centrul cercului circumscris triunghiului ABC, 9O mijlocul

segmentului OH este centrul cercului lui Euler al triunghiului ABC. Dacă E este mijlocul segmentului HM, atunci 9EO este linie mijlocie în triunghiul HOM, deci

9 ,2 2

OM RO E = = adică E este punct pe cercul lui Euler al triunghiului ABC.

4) Dreptele lui Simson în raport cu triunghiul ABC a două puncte M şi N fac între ele un unghi congruent cu acela a cărui măsură pe cercul circumscris triunghiului ABC este jumătatea

cercului .MN Demonstraţie. Fie 'M şi 'N intersecţiile perpendicularelor duse din M şi N pe latura BC cu cercul circumscris triunghiului ABC (Fig.128). Conform proprietăţii (1) unghiul dintre cele două drepte ale lui Simson este egal cu unghiul

' 'M AN şi deoarece ' 'MM NN rezultă ( ) ( ' ')m MN m M N= de unde se obţine:

1( ' ') ( ' ')

2m M AN m M N= = 1

( ).2m MN

5) Dreptele lui Simson a două puncte diametral opuse de pe cercul circumscris unui triunghi sunt perpendiculare şi se intersectează într-un punct ce aparţine cercului lui Euler al triunghiului. Demonstraţie. Fie P şi Q mijloacele segmentelor MH respectiv HN (unde H este ortocentrul triunghiului ,ABC iar M şi N punctele diametral opuse) (Fig. 129). Conform proprietăţii precedente unghiul dintre dreptele lui Simson ale

punctelor M şi N are măsura 1( ) 90 .

2= °m MN

Deci cele două drepte Simson sunt perpendiculare. Conform proprietăţii 2) dreapta lui Simson corespunzătoare punctului M trece prin P (mijlocul lui MH), iar dreapta lui Simson corespunzătoare punctului N trece prin Q (mijlocul lui NH), deci PQ este linie mijlocie în

triunghiul MNH, adică 1

.2

PQ MN R= =

Conform proprietăţii (3) punctele P şi Q aparţin cercului lui Euler al triunghiului ABC, deci

B C

N'

N M

M'

S

Fig. 128

A

A

B C

N

H

M

P

S O

Q

Fig. 129

Page 137: Barbu_TEOREME_FUNDAMENTALE(2008).pdf

144

PQ este diametru în cercul lui Euler al triunghiului. Dacă S este punctul de întâlnire a celor două drepte Simson, cum ( ) 90 ,m PSQ = ° rezultă că S aparţine cercului lui Euler al

triunghiului ABC. 6) Dreptele lui Simson ale vârfurilor unui triunghi sunt înălţimile triunghiului. Demonstraţie. Picioarele perpendicularelor duse din A pe AB şi AC coincid cu A, iar perpendiculara pe BC este înălţimea din A, deci dreapta lui Simson corespunzătoare vârfului A al triunghiului ABC este înălţimea din A a triunghiului. 7) Dreptele lui Simson ale punctelor diametral opuse vârfurilor unui triunghi sunt laturile triunghiului. Demonstraţie. Fie 'A punctul diametral opus vârfului A al triunghiului ABC (Fig. 130). Atunci

( ' ) ( ' ) 90 ,= = ° m A BA m A CA deci dreapta lui Simson

corespunzătoare punctului 'A este dreapta BC. 8) Dreptele lui Simson ale punctelor de intersecţie ale înălţimilor unui triunghi ABC cu cercul circumscris acestuia sunt paralele cu tangentele duse în vârfurile triunghiului ABC la cercul circumscris şi trec prin vârfurile triunghiului ortic. Demonstraţie. Fie ' = I

aA AH C , C este cercul circumscris triunghiului ABC (Fig. 131).

Evident dreapta lui Simson corespunzătoare punctului 'A trece prin

aH (deoarece '

aA H BC⊥ ). Fie TA

tangenta în A la cercul circumscris triunghiului ABC.

Atunci, 1

( ') ( ') ( ')2

= = m TAA m ACA m ABA (1) Fie

1A şi 2A proiecţiile lui 'A pe AB respectiv AC. Avem:

1 2( ') ( ' )= a

m AH A m A CA (2) (deoarece patrulaterul

2'a

A H AC este inscriptibil având

2( ' ) ( ' ) 90= = ° a

m A H C m A A C ). Din relaţiile (1) şi

(2) rezultă 1' ',TAA AHA≡ adică 1 2 .TA A A

Observaţie: Proprietatea precedentă poate fi reformulată astfel: „Într-un triunghi ABC, dreptele lui Simson ale punctelor unde înălţimile intersectează cercul circumscris triunghiului ABC sunt antiparalele laturilor opuse vârfurilor din care plecă înălţimile şi trec prin picioarele acestora.” 9) Dreptele lui Simson ale simetricelor ortocentrului H al triunghiului ABC faţă de laturile triunghiului sunt paralele cu laturile triunghiului ortic al triunghiului .ABC Demonstraţia rezultă din proprietatea precedentă.

A

B C

O

Fig. 130

A'

A

B C

O

Fig. 131

A' 1A

2A aH

T

Page 138: Barbu_TEOREME_FUNDAMENTALE(2008).pdf

145

10) Dreptele lui Simson ale punctului de intersecţie dintre bisectoarele interioare ale unui triunghi ABC cu cercul circumscris acestuia trec prin mijloacele laturilor triunghiului ABC şi sunt perpendiculare pe bisectoarele interioare ale triunghiului. Demonstraţie. Fie 'A punctul de intersecţie al bisectoarei din 'A cu cercul circumscris triunghiului ABC şi 1 2 3, ,A A A proiecţiile

punctului 'A pe laturile AB, BC respectiv CA..

Deoarece 'A este mijlocul arcului BC rezultă că 2A este mijlocul laturii BC (Fig. 132).

Patrulaterele 1 2'A A BA şi 2 3'A A CA fiind

inscriptibile rezultă

1 2 2 2 2 3' 'BA A BA A A A C A A A≡ ≡ ≡ , deci

triunghiul 1 3A AA este isoscel, de unde

1 3' .AA A A⊥

11) Dreptele lui Simson ale punctelor de intersecţie dintre bisectoarele exterioare ale unghiurilor unui triunghi ABC cu cercul circumscris triunghiului trec prin mijloacele laturilor triunghiului, sunt paralele cu bisectoarele interioare ale triunghiului şi sunt concurente în punctul lui Spieker al triunghiului. Demonstraţie. Fie "A punctul de intersecţie al bisectoarei exterioare a unghiului A cu cercul circumscris triunghiului ABC (Fig. 132). Deoarece "A este punctul diametral opus lui 'A rezultă (conform proprietăţii 5) că dreapta lui Simson a lui "A este perpendiculară pe dreapta lui Simson a punctului 'A ; conform proprietăţii precedente rezultă că dreapta lui Simson a lui "A este paralelă cu bisectoarea interioară a unghiului A. Deoarece "A este

mijlocul arcului BAC rezultă că proiecţia lui "A pe BC este mijlocul laturii BC. Fie

2 2 2A B C triunghiul median al triunghiului ABC şi 3"A proiecţia lui "A pe latura AC. Avem

"2 3 2 2', A A AA A B AB şi 2 2 ,A C AC deci " "

3 2 2 2 2 3

1( ) ( ) ( )

2= = m A A B m C A A m BAC ,

adică dreapta lui Simson corespunzătoare lui "A este bisectoarea unghiului

2 2 2 .B A C Analog, se arată că bisectoarele unghiurilor 2B şi 2C ale triunghiului median

sunt dreptele lui Simson ale punctelor "B şi "C (punctele de intersecţie ale bisectoarelor exterioare ale unghiurilor B şi C cu cercul circumscris) şi cum bisectoarele triunghiului median sunt concurente în punctul lui Spieker al triunghiului ABC rezultă concluzia.

12) Fie punctele conciclice A, B, C, D. Dacă drepta lui Simson a punctului A în raport cu triunghiul BCD este perpendiculară pe dreapta lui Euler a triunghiului BCD, atunci dreapta lui Simson a punctului B este perpendiculară pe dreapta lui Euler a triunghiului CDA .

Demonstraţie. Alegem un reper complex cu originea în centrul cercului circumscris patrulaterului ABCD. Notăm cu litere mici afixele punctelor corespunzătoare (Fig.133). Atunci, ortocentrul triunghiului BCD are afixul b c d+ + . Picioarele perpendicularelor duse din A pe

A'

A

B C

A"

2A 3A

1A

"3A

Fig. 132

A(a)

B(b)

C(c)

D(d)

X(x) Y(y) Z(z)

Fig. 133

Page 139: Barbu_TEOREME_FUNDAMENTALE(2008).pdf

146

BC , CD şi DB au afixele: 1( )

2x a b c abc= + + − ,

1( )

2y a c d acd= + + − ,

1( )

2z a d b adb= + + − . Punctele ( )X x , ( )Y y , ( )Z z aparţin dreptei lui Simson a

punctului A în raport cu triunghiul BCD. Dreapta lui Euler a triunghiului BCD este perpendiculară pe dreapta lui Simson a punctului A în raport cu triunghiul BCD dacă

numărul *2( ) ( )( )x y a a c b d

b c d b c dα

− − −= = ∈

+ + + + , adică α α= sau 0ab ac ad bc bd cd+ + + + + =

şi cum această relaţie este simetrică în a , b , c şi d rezultă concluzia.

13) Dreapta lui Simson a punctului lui Feuerbach ( )ϕ al triunghiului ABC , în raport

cu triunghiul de contact cba CCC al triunghiului ABC este paralelă cu dreapta OI (O

şi I fiind centrele cercurilor circumscris, respectiv înscris în triunghiul ABC ). Demonstraţie. Fie cba MMM triunghiul median al triunghiului ABC (Fig. 134), 9O

centrul cercului lui Euler al triunghiului ABC , ',',' CBA punctele de intersecţie dintre

dreptele ,a b

C Cϕ ϕ , respectiv c

Cϕ cu cercul lui Euler al triunghiului ABC . Deoarece cercul

înscris şi cercul lui Euler sunt tangente în punctul lui Feuerbach, laturile triunghiurilor

cba CCC şi ''' CBA sunt paralele două câte două, deci triunghiurile cba CCC şi ''' CBA

sunt omotetice, centrul de omotetie fiind punctul ϕ . Deoarece dreptele lui Simson ale

punctului ϕ în raport cu triunghiurile cba CCC şi ''' CBA sunt paralele rezultă concluzia.

14) Într-un triunghi ABC, dreapta lui Simson a punctului lui Feuerbach ( )ϕ în raport cu triunghiul median al triunghiului ABC, este paralelă cu dreapta OI (O şi I fiind centrele cercurilor circumscris şi înscris ale triunghiului ABC).

aM

bM cM

ϕ

aC

bC

cC

Fig. 134

A

B C

A'

B'

C'

I

Page 140: Barbu_TEOREME_FUNDAMENTALE(2008).pdf

147

Demonstraţie. Ortopolul diametrului cercului circumscris triunghiului ABC ce trece prin I este punctul lui Feuerbach (vezi „Ortopolul unui triunghi”) – punct ce aparţine cercului lui Euler al triunghiului ABC. Fie

a b cM M M triunghiul median al triunghiului ABC şi , ,α β γ

simetricele punctului lui Feuerbach ( )ϕ faţă de laturile , ,b c a c a b

M M M M M M ale

triunghiului median. Punctele , ,α β γ aparţin dreptei OI deoarece ϕ este ortopolul acestei

drepte. Dacă ', ', 'α β γ sunt proiecţiile lui ϕ pe laturile triunghiului median atunci ' 'α β

este dreapta lui Simson a punctului ϕ şi este paralelă cu OI, deoarece ' 'α β este linie

mijlocie în triunghiul .ϕαβ

15) Dreapta lui Simson a punctului lui Feuerbach al unui triunghi ABC, în raport cu triunghiul ortic este paralelă cu dreapta OI (O şi I fiind centrele cercurilor circumscris şi înscris al triunghiului ABC). Demonstraţie. Deoarece triunghiul ortic

a b cH H H şi triunghiul median

a b cM M M al

triunghiului ABC sunt două triunghiuri S, dreptele lui Simson ale punctului lui Feuerbach ( )ϕ al triunghiului ABC în raport cu triunghiurile

a b cH H H şi

a b cM M M sunt paralele,

deci conform proprietăţii precedente, paralele cu OI. 16) Fie M un punct pe cercul circumscris unui triunghi ABC , iar ',',' CBA punctele în care dreapta lui Simson ( )

Md a punctului M intersectează laturile CABC, respectiv AB.

Simetricele vârfurilor CBA ,, în raport cu mijloacele segmentelor '','' ACCB respectiv ''BA aparţin unei drepte perpendiculare pe dreapta Md .

Demonstraţie. Notăm ","," CBA simetricele vârfurilor CBA ,, în raport cu mijloacele

segmentelor '','','' BAACCB (Fig. 135). Deoarece patrulaterul '"" CAAC este

paralelogram rezultă '||'" ACBA şi ABCA '||'" ; cum ABMC ⊥' şi ACMB ⊥' rezultă

''" MCBA ⊥ şi '" MBCA ⊥ , relaţii ce arată că "A este ortocentrul triunghiului ''CMB , deci ''" CBMA ⊥ , adică MdMA ⊥" . Analog, MdMB ⊥" şi MdMC ⊥" , de unde rezultă că

punctele ","," CBA sunt coliniare.

17) Fie ABC şi DEF două triunghiuri înscrise în acelaşi cerc. Dreptele lui Simson ale punctelor FED ,, în raport cu triunghiul ABC determină un triunghi ''' FED asemenea cu triunghiul DEF.

A

B C

M

A'

B'

C'

A"

B"

C"

Fig. 135

B C

D

E

F

E '

D'

F'

Ed Fd

Dd

Fig. 136

Page 141: Barbu_TEOREME_FUNDAMENTALE(2008).pdf

148

Demonstraţie. Fie FED ddd ,, dreptele lui Simson ale punctelor ED, respectiv F în

raport cu triunghiul ABC (Fig.136). Avem: 1( , ) ( ) ( ),

2= =E Dm d d m ED m DFE

1( , ) ( ) ( )

2= =E Fm d d m EF m FDE , de unde rezultă că triunghiurile DEF şi ''' FED

(determinat de dreptele FED ddd ,, ) sunt asemenea.

Observaţie: Analog, triunghiul determinat de dreptele lui Simson ale punctelor CBA ,, în raport cu triunghiul DEF determină un triunghi ''' CBA asemenea cu ABC . 18) Triunghiul determinat de intersecţiile dreptelor lui Simson ale punctelor unde înălţimile unui triunghi ABC taie cercul circumscris triunghiului ABC este anticomplementarul triunghiului ortic. Demonstraţie. Fie cba HHH triunghiul ortic al

triunghiului ABC şi hhh CBA ,, punctele de intersecţie

dintre înălţimile triunghiului ABC cu cercul circumscris acestui triunghi. Notăm cu cba ddd ,,

dreptele lui Simson ale punctelor hhh CBA ,, în raport

cu triunghiul ABC (Fig.137). Atunci dreapta ad este

antiparalela laturii BC ce trece prin punctul aH .

Analog, bd si cd sunt antiparalele laturilor CA

respectiv AB şi trec prin bH respectiv cH , deci

|| , || ,a b c b a cd H H d H H ||

c a bd H H , de unde rezultă că

triunghiul determinat de dreptele cba ddd ,, este

anticomplementarul triunghiului ortic. Teorema lui Droz-Farny

19) Fie două triunghiuri ABC şi DEF înscrise în cercul C ( , )O R şi M un punct oarecare

pe cercul C. Să se arate că dreptele lui Simson ale punctului M în raport cu triunghiurile ABC şi respectiv DEF se intersectează sub un unghi constant. Demonstraţie. Fie N,P proiecţiile punctului M pe BC şi EF iar K şi L punctele în care MN, respectiv MP intersectează cercul C ( , )O R .

Dreptele lui Simson ale lui M în raport cu triunghiurile ABC şi DEF sunt paralele cu AK şi DL. Fie S punctul de intersecţie al dreptelor lui Simson (NS pe PS) (Fig. 138). Unghiul dintre ele este egal cu unghiul dintre dreptele AK şi DL. Fie

.= ∩T AK DL Considerăm arcele

de pe cercul C în sens trigonometric.

A

B C

H

aH

bH cH

hA

hB

hC ad

bd

cd

Fig. 137

A

B C

D

E

F

M

N

K

P

L

S

T

Fig. 138

Page 142: Barbu_TEOREME_FUNDAMENTALE(2008).pdf

149

Atunci, ( ) ( )

( )2

+=m AD m KL

m ATD şi cum ( ) ( ) 90= = °m BNM m CNK

rezultă ( ) ( ) 180+ = °m KC m MB şi ( ) ( ) 180m ME m FL− = °

( ) ( ) ( ) ( )m KL m KC m CF m FL= + + = 180 ( ) ( ) ( ) 180m MB m CF m ME°− + + − ° , deci

( ) ( ) ( )= +m KL m CF m BE . Astfel, ( ) ( ) ( )

( )2

+ +=m AD m BE m CF

m ATD care arată

că unghiul ATD este constant. Consecinţă: Unghiul pe care îl fac două drepte ale lui Simson ale unui punct oarecare în raport cu două triunghiuri înscrise în acelaşi cerc este egal cu unghiul care subîntinde un arc egal cu suma algebrică a celor trei arce cuprinse între vârfurile acestor triunghiuri,luate câte două. 20) Fie ABC şi DEF două triunghiuri înscrise în acelaşi cerc. Dreptele lui Simson ale vârfurilor triunghiului ABC în raport cu triunghiul DEF şi dreptele lui Simson ale vârfurilor triunghiului DEF în raport cu triunghiul ABC , determină cu înălţimile care pleacă din vârfurile considerate, unghiuri egale între ele. Demonstraţie. Din teorema lui Droz - Farny rezultă că dreptele lui Simson ale punctelor A şi D în raport cu triunghiurile ABC şi DEF determină un unghi egal (Fig.139). Dar, dreptele lui Simson Ad

şi Dd ale punctelor A şi D în raport cu

triunghiul ABC , respectiv triunghiul DEF sunt înălţimile 'AA , respectiv 'DD . Deci, unghiul dintre dreapta lui Simson a punctului A în raport cu DEF şi 'AA este egal cu unghiul dintre dreapta lui Simson a punctului D în raport cu ABC şi

'DD .

21) În cercul circumscris triunghiului ABC , ducem coarda 'MM paralelă cu BC. Dreapta lui Simson a punctului M în raport cu triunghiul ABC este perpendiculară pe dreapta AM ' .

Demonstraţie. Fie 'B punctul diametral opus lui B în cercul circumscris triunghiului ABC , Md dreapta lui

Simson a punctului M şi T proiecţia lui M pe AC,

'= IM

P d AM . Avem: 1( , ) ( '),

2Md CA MB=

( ') ( )m CM m BM= de unde rezultă că

( ) 1 8 0 [ ( ) ( ) ]= ° − + =m A P T m T A P m A T P

1 1 1180 [ ( ') ( ')] 180 [ ( ) ( ')]

2 2 2mCM m MB m BM m MB°− + = °− + =

1180 180 90

2° − ⋅ ° = ° .

A

B C D

E F

A'

D' Ad

Fig. 139

A

B C

M' M

P T

Md

Fig. 140

Page 143: Barbu_TEOREME_FUNDAMENTALE(2008).pdf

150

22) Într-un triunghi ABC , dreapta lui Simson a unui punct M este perpendiculară pe izogonala dreptei AM .

Demonstraţie. Fie d dreapta lui Simson a punctului M şi 'AM izogonala dreptei AM

(Fig.141). Avem: ( , ) ( , ')m d BC m BC AM+ =

180 ( ', ).m AM d° − Dacă 'A este punctul

diametral opus lui A, atunci

1( , ) ( ' )

2=m BC d m A M , ( , ')m BC AM =

1 1[ ( ) ( ')] ( ).

2 2m AC m BM m AM− = Atunci,

1( , ) ( , ') [ ( ' ) ( )]

2m d BC m BC AM m A M m MA+ = + =

1180 90

2⋅ ° = ° , de unde rezultă că ( ', ) 90= °m AM d .

23) Fie ABC şi ' ' 'A B C două triunghiuri ortoomologice şi τ centrul lor de omologie. Dreptele lui Simson ale punctului τ faţă de triunghiurile ABC şi ' ' 'A B C sunt paralele cu axa de omologie. Demonstraţie. Vezi „Triunghiuri ortoomologice”. 24) Fie

a b cH H H şi

a b cM M M triunghiul ortic, respectiv triunghiul median al unui

triunghi ABC. Dreptele lui Simson ale punctelor , ,a b c

M M M în raport cu triunghiul

a b cH H H sunt concurente în centrul cercului lui Taylor al triunghiului ABC.

Demonstraţie. Fie ', ', 'A B C mijloacele segmentelor AH, BH respectiv CH. Punctele 'A şi

aM sunt diametral opuse în cercul lui Euler

al triunghiului ABC, deci proiecţia lui a

M

pe b c

H H este punctul "A mijlocul

segmentului b c

H H , adică " ( ' ).a

A A M∈

Atunci dreapta Simson a punctului a

M va fi

paralelă cu 'a

H A (conform proprietăţii (1))

(Fig. 142). Fie "B şi "C mijloacele segmentelor

a cH H , respectiv

a bH H .

Triunghiul " " "A B C este triunghiul median al triunghiul ortic

a b cH H H . Atunci, dreptele

lui Simson considerate sunt bisectoarele unghiurilor triunghiului median " " "A B C şi deci concurente în punctul lui Taylor – T al triunghiului ABC.

A

B C

M M'

A'

O

d

Fig. 141

A

B C aM

bM cM

aH

bH

cH

Fig. 142

3S A"

H

A'

B"

C"

1S

2S

T

Page 144: Barbu_TEOREME_FUNDAMENTALE(2008).pdf

151

25) Fie a b c

H H H triunghiul ortic al triunghiului ABC, a b c

M M M triunghiul median al

triunghiului ABC şi ', ', 'A B C mijloacele segmentelor AH, BH, CH, unde H este ortocentrul triunghiului ABC. Dreptele lui Simson ale punctelor ', ', 'A B C raport cu triunghiul

a b cH H H determină un triunghi 1 2 3S S S ortologic şi omotetic cu triunghiul

ABC şi congruent cu a b c

M M M .

Demonstraţie. Laturile triunghiului 1 2 3S S S trec prin mijloacele ", ", "A B C ale laturilor

triunghiului a b c

H H H (Fig. 142). Dreptele lui Simson ale punctelor diametral opuse 'A şi

aM în cercul lui Euler sunt perpendiculare (conform proprietăţii 5), deci 2 3 "S S A T⊥ , iar

cum " 'TA AA şi a

AH BC⊥ rezultă că 2 3 .S S BC Analog 1 2S S AB şi 1 3 .S S AC Deci,

triunghiurile ABC şi 1 2 3S S S sunt omotetice. Perpendicularele din A, B, C pe 2 3 3 1,S S S S ,

respectiv 1 2S S sunt concurente în H ceea ce arată că triunghiurile ABC şi 1 2 3S S S sunt

ortologice. Avem 3 3 1 2" " ,b a

B A S H H C BAC S S S≡ ≡ ≡ deci " " "A B C este triunghiul

ortic al lui 1 2 3S S S şi fiind congruent cu triunghiul ortic al triunghiului a b c

M M M rezultă că

triunghiurile 1 2 3S S S şi a b c

M M M sunt congruente.

Observaţie: Dreptele 1 2 3, ,AS BS CS sunt concurente în centrul de greutate al triunghiului

a b cH H H .

Demonstraţie. Din congruenţa triunghiurilor 1 2 3S S S şi a b c

M M M rezultă 1

1"

2 aS A AH= ,

iar din asemănarea triunghiurilor 1 1"A S G şi 1"A AG (unde 1 1 "a

G AS H A= ∩ ) rezultă

1

1

"" 1

2a a

A SA G

G H AH= = , deci 1G este centrul de greutate al triunghiului

a b cH H H . Analog, se

arată că dreptele 2BS şi 3CS trec prin 1G .

26) Dreptele lui Simson ale punctelor , ,

a b cM M M , mijloacele laturilor BC, CA, respectiv

AB ale unui triunghi ABC, în raport cu triunghiul ortic a b c

H H H sunt înălţimi în

triunghiul 1 2 3S S S . Demonstraţie. Deoarece T este centrul cercului înscris în triunghiul

" " ",A B C triunghiul ortic triunghiului 1 2 3S S S (cf. th (21)), rezultă că T este ortocentrul

triunghiului 1 2 3S S S , de unde rezultă concluzia.

Consecinţă: Centrul cercului lui Taylor al triunghiului ABC este ortocentrul triunghiului

1 2 3S S S .

27) Dreptele lui Simson ale punctelor , ,

a b cM M M în raport cu triunghiul ' ' 'A B C sunt

laturile triunghiului ' ' 'A B C .

Demonstraţie. Deoarece 'a

M A este diametru în cercul lui Euler al triunghiului ABC, iar

'B şi 'C sunt puncte pe acest cerc rezultă că proiecţiile lui a

M pe ' 'A B şi ' 'A C cad în

'B , respectiv 'C adică ' 'B C este dreapta lui Simson a punctului a

M . Analog se arată că

' 'A C şi ' 'A B sunt dreptele lui Simson ale punctelor b

M , respectiv c

M .

Page 145: Barbu_TEOREME_FUNDAMENTALE(2008).pdf

152

28) Fie a b c

H H H triunghiul ortic al triunghiului ABC, a b c

M M M triunghiul median al

triunghiului ABC. Dreptele lui Simson ale punctelor , ,a b c

H H H în raport cu triunghiul

a b cM M M sunt paralele cu dreptele AO,BO, respectiv CO. Demonstraţie. Fie ', ', 'A B C mijloacele segmentelor AH, BH, CH, unde H este ortocentrul triunghiului ABC şi D,E,F proiecţiile punctului

aH pe dreptele ,

a bM M

b cM M , respectiv

a cM M

(Fig. 143). Conform proprietăţii (1) dreapta lui Simson a punctului

aH este paralelă cu dreapta

'a

M A şi întrucât 'a

M A AO (vezi „Cercul lui

Euler”) rezultă DE AO . 29) Dreptele lui Simson ale punctelor

, ,a b c

H H H în raport cu triunghiul a b c

M M M

sunt concurente în centrul cercului lui Taylor al triunghiului ABC. Demonstraţie. Vezi „Cercul lui Taylor”. 30) Fie P un punct pe cercul circumscris unui triunghi ABC, , ,

a b cP P P picioarele

înălţimilor duse din P pe dreptele BC, CA, respectiv AB. Pe înălţimile , ,a b c

AH BH CH ale

triunghiului ABC se consideră punctele ', ', 'A B C astfel încât ' , 'a b

AA PP BB PP= =uuuur uuur uuuur uuur

şi

'c

CC PP=uuuur uuur

. Punctele ', ', 'A B C aparţin

dreptei lui Simson a punctului 'P diametral opus punctului P în cercul circumscris triunghiului ABC. Demonstraţie. Fie D proiecţia lui 'P pe BC şi

"P al doilea punct de intersecţie dintre 'P D cu cercul circumscris triunghiului ABC

(Fig.144). Atunci "a

P D PP≡ (deoarece P şi

'P sunt diametral opuse şi "a

P D PP ) şi cum

'a

PP AA≡ rezultă " 'P D AA≡ şi " 'P D AA ,

deci patrulaterul " 'AP DA este paralelogram de unde " 'AP DA şi conform proprietăţii 1) rezultă că 'A D este dreapta lui Simson a punctului '.P Analog se arată că punctele 'B şi 'C aparţin acestei drepte. 31) Fie A, B, C, D patru puncte conciclice. Dreptele lui Simson ale fiecărui punct în raport cu triunghiul determinat de celelalte trei puncte, sunt concurente. Demonstraţie. Fie 1 2 3 4, , ,H H H H ortocentrele

triunghiurilor ABC, BCD, CDA, respectiv DAB şi , , ,

A B C Dd d d d dreptele lui Simson ale punctelor A, B, C, D

în raport cu triunghiurile BCD, ACD, ABD, respectiv ABC

A

B C

P

aP

bP

cP

A'

B'

C'

Fig. 144

D

P '

P"

A

B C 1H

4H 3H D

O'

2H

Fig. 145

O

A

B C aM

bM

cM

aH

bH

cH

Fig. 143

O H

D

F

A '

E

Page 146: Barbu_TEOREME_FUNDAMENTALE(2008).pdf

153

(Fig. 145). Dreptele , , ,A B C Dd d d d trec prin mijloacele segmentelor 2 3 4, ,AH BH CH ,

respectiv 1.DH Patrulaterele ABCD şi 1 2 3 4H H H H sunt congruente şi omotetice (vezi

„Ortocentrul unui triunghi”), deci segmentele 1 2 3, ,DH AH BH şi 4CH - care unesc

vârfurile omoloage – sunt concurente în centrul de omotetie '.O Deoarece patrulaterul

1 2AH H D este paralelogram rezultă că 'O este mijlocul segmentelor 1 2 3 4, , ,DH AH BH CH

- punct comun dreptelor , , ,A B C Dd d d d .

32) Dreapta lui Simson a unui punct M în raport cu un triunghi ABC intersectează laturile şi înălţimile triunghiului în D şi ',D E şi 'E , respectiv F şi '.F Segmentele ', 'DD EE şi 'FF au acelaşi mijloc ce aparţine cercului lui Euler al triunghiului ABC. Demonstraţie. Dreapta lui Simson a punctului M trece prin punctul P – mijlocul segmentului HM. Atunci, triunghiurile DPH şi MPD sunt congruente ( ' , ' , )HPD MPD D HP DMP HP PM≡ ≡ ≡ ,

deci ' .D P PD≡ Analog, 'HPE MPE≡ şi 'HPF MPF≡ de unde 'E P PE≡ şi

' ',F P PF≡ adică segmentele ', 'DD EE şi 'FF au acelaşi mijloc P. Dar prin P – mijlocul segmentului MH – trece cercul lui Euler al triunghiului ABC.

33) Două drepte Simson perpendiculare sunt transversale izotomice. Demonstraţie. Vezi „Puncte izotomice”. 34) Fie M un punct pe cercul circumscris triunghiului ABC şi X, Y, Z proiecţiile

punctului M pe dreptele BC,CA, respectiv AB. Atunci, = +BC AC AB

MX MY MZ.

Demonstraţie.

A

B C

M

F

H

D'

D

P

F'

E

E '

Fig. 146

B

A C

M

Y

X

Z

Fig. 147

Page 147: Barbu_TEOREME_FUNDAMENTALE(2008).pdf

154

Avem: ( ) 180 ( ) ,= °− = m ABM m ACM α ( ) ( )= = m BAM m BCM β şi

( ) ( ) ,= = m CAM m CBM γ deci sin 2 sin sin ,= =MX MB Rγ β γ 2 sin sin=MY R α γ şi

2 sin sin=MZ R α β ,iar 2 sin 2 sin( ),= = +BC R BAC R β γ 2 sin( ), 2 sin( )= − = +AC R AB Rα γ α β .

Egalitatea de demostrat devine: sin( ) sin( ) sin( )

sin sin sin sin sin sin

+ − += +

β γ α γ α ββ γ α γ α β

care rezultă prin

calcul direct. 35) Fie M un punct pe cercul circusmscris al unui triunghi ABC, iar 'A şi 'B proiecţiile lui M pe BC, respectiv AC. Atunci, ' 2⋅ =MA MA Rd , unde R este raza cercului circumscris triunghiului ABC şi d este distanţa de la M la dreapta ' 'A B . Demonstraţie. Fie ( ) .=m MCA ϕ Atunci, 2 sin=MA R ϕ şi

( ' ') ( ')= = m MA B m MCB ϕ , deci 'sin

=d

MAϕ

, de unde ' 2⋅ =MA MA Rd .

Observaţie: Deoarece proiecţiile lui M pe laturile triunghiului sunt puncte coliniare rezultă

' ' ' 2⋅ = ⋅ = ⋅ = ⋅MA MA MB MB MC MC R d . 36) Fie M un punct pe cercul circumscris unui triunghi ABC, iar ', ', 'A B C proiecţiile lui M pe BC, CA, respectiv AB. Lungimea segmentului ' 'A B este egală cu lungimea proiecţiei segmentului AB pe dreapta ' '.A B

Demonstraţie. Deoarece punctele 'A şi 'B aparţin unui cerc de diametru MC, rezultă

' ' sin ' ' sin (1)= =A B MC A CB MC ACB (Fig. 148). Fie

( ' ') =m BC A ϕ şi 'M proiecţia lui M pe ' 'A B .

Conform teoremei precedente ' 2 '⋅ = ⋅MC MC R MM . În

triunghiul ' 'MM C avem: '

sin(90 )'

° − =MM

MCϕ , de unde

'cos (2).

' 2= =MM MC

MC Rϕ Lungimea proiecţiei

segmentului AB pe dreapta ' 'A B este egală cu

cos 2 sin sin (3).2 2

= ⋅ = ⋅ =MC MC

AB AB R ACB MC ACBR R

ϕ

Din realaţiile (1) şi (3) rezultă concluzia.

C

A

B

M

A'

B'

C'

Fig. 148

d M'

Page 148: Barbu_TEOREME_FUNDAMENTALE(2008).pdf

155

I.37. Dreapta lui Lemoine52

„O notaţie bună are o subtilitate şi o sugestivitate care uneori o face să pară un profesor viu.” – Bertrand Russell53

Teorema lui Lemoine

1) Tangentele la cercul circumscris unui triunghi neisoscel în vârfurile acestuia intersecteazā laturile opuse în trei puncte coliniare. Demonstraţie. Fie 1 1 1, ,A B C punctele de

intersecţie dintre tangentele date cu laturile opuse ale triunghiului ABC. Triunghiurile

1A AB şi 1A AC au unghiul 1A comun şi

1 1( ) ( )=m A AB m ACA , deci sunt asemenea.

Atunci, 1

1

A B

A A=AB

AC= 1

1

A A

AC, de unde

1

1

A B

AC=

2

.

AB

ACAnalog, 1

1

BC

B A=

2BC

BA

şi

1

1

C A

C B=

2CA

CB

. Cum 1 1 1

1 1 1

1⋅ ⋅ =A B B C C A

AC B A C B,

din reciproca teoremei lui Menelaus rezultă că punctele 1 1 1, ,A B C sunt

coliniare. Dreapta 1 1A B se numeşte dreapta lui Lemoine a triunghiului ABC. 2) Dreptele 1 1 1, ,AA BB CC sunt

simedianele exterioare ale vârfurilor triunghiului ABC. Demonstraţie. Vezi „Simediane exterioare”. 3) Triunghiul ABC şi triunghiul său tangenţial sunt omologice, dreapta lui Lemoine a triunghiului ABC fiind axa de omologie. Demonstraţie. Din teorema lui Lemoine rezultă că dreapta lui Lemoine este axa de omologie dintre triunghiul ABC şi triunghiul său tangenţial, centrul de omologie fiind punctul lui Lemoine al triunghiului ABC (vezi „Punctul lui Lemoine”). Observaţie: Teorema precedentă poate fi reformulată astfel: „Dreapta lui Lemoine a unui triunghi ABC este polara triliniară a punctului simedian al triunghiului ABC.”

52 Emile Lemoine (1840-1912) – matematician francez, contribuţii importante în geometrie 53 Bertrand Russell (1872 - 1970) – filosof, logician şi matematician englez, laureat al Premiului Nobel pentru literatură

A

B C

O

1A

1B

1C

Fig. 149

Page 149: Barbu_TEOREME_FUNDAMENTALE(2008).pdf

156

4) Centrele cercurilor lui Apollonius corespunzătoare vârfurilor triunghiului ABC sunt punctele de intersecţie ale dreptei lui Lemoine a triunghiului ABC cu laturile acestui triunghi. Demonstraţie. Vezi „Cercurile lui Apollonius”. 5) Dacă O este centrul cercului circumscris triunghiului ABC şi K punctul lui Lemoine al triunghiului ABC, atunci dreapta lui Lemoine a triunghiului ABC este perpendiculară pe dreapta .OK Demonstraţie. Deoarece OK este axa radicală a cercurilor lui Apollonius, rezultă dreapta OK este perpendiculară pe dreapta centrelor – adică pe dreapta lui Lemoine a triunghiului ABC (vezi „Cercurile lui Apollonius”). Observaţii:

i) Dreapta OK se numeşte dreapta lui Brocard. Proprietatea precedentă se mai poate enunţa astfel: „Dreptele lui Lemoine şi Brocard ale triunghiului ABC sunt perpendiculare.” ii) Punctul de intersecţie dintre dreapta lui Lemoine şi dreapta lui Brocard se numeşte punctul lui Schoute. 6) Punctele izodinamice S şi 'S ale unui triunghi neechilateral ABC sunt simetrice faţă de dreapta lui Lemoine a triunghiului ABC .

Demonstraţie. Deoarece OK (unde O este centrul cercului circumscris triunghiului ABC şi K punctul lui Lemoine al triunghiului ABC) este perpendiculară pe dreapta lui Lemoine a triunghiului ABC rezultă că 'SS este perpendiculară pe dreapta lui Lemoine, ceea ce arată că punctele izodinamice S şi 'S sunt simetrice faţă de dreapta lui Lemoine a triunghiului ABC .

Page 150: Barbu_TEOREME_FUNDAMENTALE(2008).pdf

157

I.38. Transversala izotomică

„Plăcerea stă nu în descoperirea adevărului, ci în căutarea lui.” – Lev Tolstoi54

1) Fie ', ', 'A B C punctele de intersecţie ale unei drepte d cu laturile ,BC AC respectiv AB ale unui triunghi ABC . Fie "A simetricul punctului 'A faţă de mijlocul segmentului BC . Analog se construiesc punctele "B şi "C . Punctele ", ", "A B C sunt coliniare.

Demonstraţie. Punctele ', ', 'A B C fiind coliniare, din reciporca teoremei lui Menelaus

rezultă: ' ' '

1' ' '

A B B C C A

A C B A C B⋅ ⋅ = (1). Din ' " , ' " , ' " ,A B A C A C A B B C B A= = =

' " ,B A B C= ' ",C A BC= ' "C B C A= şi din relaţia (1) rezultă " " "

1" " "

A C B A C B

A B B C C A⋅ ⋅ = , care

arată că punctele ", ", "A B C sunt coliniare. Observaţie: Dreapta ce conţine punctete ", ", "A B C se numeşte transversala izotomică a dreptei d. 2) Fie M şi 'M două puncte diametral opuse în cercul circumscris unui triunghi ABC. Dreptele lui Simson ale punctelor M şi 'M sunt transversale izotomice. Demonstraţie. Vezi „Dreapta lui Simson”. 3) Transversala izotomică a unei drepte a lui Simson este o altă dreaptă a lui Simson. Demonstraţie. Fie 1M proiecţia unui punct M de pe

cercul circumscris unui triunghi ABC pe latura BC , N punctul diametral opus lui M , P MN BC= I , 1N

proiecţia lui N pe BC şi 1O proiecţia lui O pe BC.

Arătăm că 1 1 1 1N O OM= . Avem: 1

1

N P NP

PO PO= , de unde

1 1

1

N O NO

PO PO= , deci 1 1 1 1 1N O PO OM

NO PO OM= = . Deoarece

54 Lev Tolstoi (1828-1910) – scriitor rus

A'

A

B C A"

B"

C"

C'

B'

d

Fig. 150

A

B C

M

N

P

O

1O 1M

2M

1N 2N

Fig. 151

Page 151: Barbu_TEOREME_FUNDAMENTALE(2008).pdf

158

ON OM= rezultă 1 1 1 1N O OM= , adică punctele 1M şi 1N sunt izotomice, deci transversala

izotomică a punctului M este dreapta lui Simson a punctului diametral opus lui M. 4) Consecinţă: Transversala izotomică a unei drepte a lui Simson d este o dreaptă perpendiculară pe dreapta d.

Observaţie: Punctul de intersecţie dintre transversalele izotomice perpendiculare aparţine cercului medial al triunghiului ABC, deoarece punctul de intersecţie al dreptelor lui Simson a două puncte diametral opuse aparţine cecului medial al triunghiului.

I.39. Dreapta lui Steiner

„Am vrut în versificările mele să dau echivalentul unor stări absolute ale intelectului şi viziunii: starea de geometrie şi deasupra ei, extaza.”

Ion Barbu55

Fie M un punct situat pe cercul circumscris al unui triunghi ABC şi 1 2 3, ,M M M

simetricele acestuia faţă de laturile BC, CA, respectiv AB. Punctele 1 2 3, ,M M M sunt coliniare. Demonstraţie. Fie ', ', 'A B C proiecţiile punctului M pe laturile CABC, , respectiv AB (Fig. 152). Deoarece punctele ',',' CBA sunt coliniare (aparţin dreptei lui Simson a punctului M în raport cu triunghiul ABC ) rezultă că punctele 1 2 3, ,M M M sunt coliniare deoarece

2 1' ' ||B A M M şi 2 3' ' ||B C M M ( ' 'B A şi ' 'B C fiind

linii mijlocii în triunghiurile 2 1MM M şi 2 3MM M ).

Observaţie: Dreapta pe care se afla punctele 1 2,M M şi

3M se numeşte dreapta lui Steiner a punctului M în

raport cu triunghiul ABC . 1) Dreapta lui Steiner corespunzătoare punctului M este paralelă cu drepta lui Simson a punctului M în raport cu un triunghi ABC. Demonstraţia rezultă din aplicaţia precedentă. 2) Dreapta lui Steiner a punctului M trece prin ortocentrul H al triunghiului ABC . Demonstraţie. Deoarece punctul P - mijlocul segmentului MH - aparţine dreptei lui Simson a punctului M (vezi „Dreapta lui Simson”) şi 'B este mijlocul segmentului 2MM rezultă că

'PB este linie mijlocie în triunghiul 2MHM , deci 2' ||PB HM , adică paralelele prin 2M la

dreapta lui Simson a punctului M trece prin H, deci dreapta lui Steiner a punctului M conţine ortocentrul triunghiului ABC .

55 Ion Barbu (1895-1961) –matematician roman, profesor la Universitatea din Bucureşti, contribuţii în algebră şi geometrie

A

B C

M

A'

B'

C'

Fig. 152

1M

2M

3M

H

P

Page 152: Barbu_TEOREME_FUNDAMENTALE(2008).pdf

159

3) Dreptele lui Steiner ale simetricelor ortocentrului H al triunghiului ABC faţă de laturile triunghiului sunt paralele cu laturile triunghiului ortic al triunghiului .ABC Demonstraţie. Fie 1A , simetricul lui H faţă de latura BC . Punctul 1A aparţine cercului

circumscris triunghiului ABC (vezi „Ortocentrul unui triunghi”). Deoarece dreapta lui Simson a punctului 1A este paralelă cu latura cbHH a triunghiului ortic (vezi „Dreapta lui

Simson”), atunci utilizând proprietatea 1) rezultă concluzia.

I.40. Drepte izogonale. Puncte izogonale

„Matematica este regina ştiinţelor.” - Carl Gauss56 Semidreptele [AM şi [ 'AM se numesc izogonale faţă de unghiul BAC dacă sunt

simetrice faţă de bisectoarea unghiului BAC (Fig. 153).

Observaţie: Dacă AM şi 'AM sunt izogonale în raport cu unghiul BAC , atunci 'BAM M AC≡ şi ' .BAM CAM≡

1) În triunghiul ABC fie izogonalele AM şi 'AM , D şi 'D , E şi 'E proiecţiile

punctelor M şi 'M respectiv pe AB şi AC. Atunci , ' '

' '

MD M E

ME M D= .

Demonstraţie. Fie N simetricul punctului M faţă de bisectoarea unghiului BAC , iar 'N şi "N proiecţiile punctului N pe laturile AC, respectiv AB (Fig. 154). Avem:

' ' ', ' ,

" ' '

NN M ENN MD

NN M D= ≡ "NN ME≡ (deoarece 'ANN AMD∆ ≡ ∆ şi "ANN AME∆ ≡ ∆ ),

de unde: ' '

' '

MD M E

ME M D= .

56 Carl Gauss (1777-1855) – matematician, fizician şi astronom german,contribuţii în teoria numerelor, geometrie

diferenţială, analiză matematică, statistică

A

B C F F'

O'

M

E

N

N"

D

D' E '

M'

N'

Fig. 154

A

B C

M M' Fig. 153

Page 153: Barbu_TEOREME_FUNDAMENTALE(2008).pdf

160

Teorema celor şase puncte

2) Proiecţiile a două puncte izogonale pe laturile triunghiului de referinţă sunt şase puncte conciclice.

Demonstraţie. Avem: ' "

' '

AN AN

AE AD= sau

' '

AD AE

AE AD= , adică dreptele DE şi ' 'D E sunt

antiparalele, deci punctele , ', , 'D D E E aparţin unui cerc cu centrul în mijlocul segmentului 'MM (perpendicularele ridicate din mijloacele segmentelor 'DD şi 'EE intersectându-se

în mijlocul segmentului 'MM ). Analog, punctele , , ', 'D F F D (unde F şi 'F sunt proiecţiile punctelor M şi 'M pe latura BC ) sunt pe un cerc cu centrul în mijlocul segmentului 'MM ,deci punctele , , , ', ', 'D E F D E F aparţin aceluiaşi cerc. 3) Dreptele DE şi 'AM respectiv ' 'D E şi AM sunt perpendiculare. Demonstraţie. Patrulaterul DAEM fiind inscriptibil rezultă MDE MAE≡ , deoarece

( ) ( ) 90m ADE m MDE+ = ° rezultă

( ) ( ) ( ) ( ) 90 ,m ADE m MAE m ADE m DAM+ = + = ° adică '.DE AM⊥ Analog,

' ' .D E AM⊥

Teorema lui Steiner57 4) Dacă AP şi AQ sunt două izogonale în raport cu unghiul BAC al triunghiului ABC şi , ,P Q BC∈

atunci 2

2.

BP BQ AB

CP CQ AC⋅ =

Demonstraţie. Fie , , ,BR AC AB CS R AP S AQ∈ ∈

(Fig. 155). Din asemănarea triunghiurilor BPR şi CPA; CSQ şi BAQ, respectiv ABR şi ACQ obţinem:

, , .CP AC CQ CS AB BR

BP BR BQ AB AC CS= = = Înmulţind membru

cu membru egalităţile precedente obţinem

CP CQ AB AC

BP BQ AC AB⋅ ⋅ = , de unde

2

.BP BQ AB

CP CQ AC

⋅ =

5) Fie P un punct din planul triunghiului ABC şi d o transversală ce trece prin P izogonalele dreptei d faţă de unghiurile BPC, CPA şi APB intersectează dreptele BC, CA şi AB în trei puncte coliniare ', ', '.A B C Demonstraţie. Fie ", ", "A B C intersecţiile dreptei d cu laturile triunghiului ABC (Fig. 156). Din teorema lui Steiner rezultă:

2 2" ' " '

, ," ' " '

BA BA BP AC AC AP

CA CA CP BC BC BP

⋅ = ⋅ =

2" '

." '

CB CB CP

AB AB AP

⋅ =

Înmulţind relaţiile

precedente membru cu membru rezultă:

57 Jacob Steiner (1796-1863) –matematician elveţian, profesor la Universitatea din Berlin, contribuţii în geometrie

A

B C A'

S

R

P Q

Fig. 155

A

B

C A'

A"

B" d

C" P

Fig. 156

Page 154: Barbu_TEOREME_FUNDAMENTALE(2008).pdf

161

2 2 2" " " ' ' '

1." " " ' ' '

BA CB AC BA CB AC BP AP CP

CA AB BC CA AB BC CP BP AP

⋅ ⋅ ⋅ ⋅ ⋅ = ⋅ ⋅ =

Din teorema lui

Menelaus rezultă " " "

1," " "

BA CB AC

CA AB BC⋅ ⋅ = de unde rezultă

' ' '1

' ' '

BA CB AC

CA AB BC⋅ ⋅ = şi din reciproca

teoremei lui Menelaus rezultă că punctele ', ', 'A B C sunt coliniare.

Transversala izogonală a unei drepte 6) O transversală d intersectează dreptele suport ale laturii triunghiului ABC în punctele

', 'A B şi 'C . Simetricele dreptelor ', 'AA BB şi 'CC faţă de bisectoarele AI, BI, respectiv CI intersectează dreptele BC, AC şi AB în punctele ", ",A B respectiv "C ce aparţin unei drepte 'd .

Demonstraţie. Conform teoremei lui Steiner avem 2 2

' " ' ", ,

' " ' "

BA BA BA AC AC AC

CA CA CA BC BC BC

⋅ = ⋅ =

2

' ".

' "

CB CB CB

AB AB AB

⋅ =

Înmulţind membru cu

membru relaţiile precedente rezultă " " " ' ' '

1." " " ' ' '

BA CB AC BA CB AC

CA AB BC CA AB BC

⋅ ⋅ ⋅ ⋅ ⋅ =

Din

teorema lui Menelaus în triunghiul ABC rezultă ' ' '

1' ' '

BA CB AC

CA AB BC⋅ ⋅ = , de unde

" " "1

" " "

BA CB AC

CA AB BC⋅ ⋅ = şi din reciproca teoremei lui Menelaus rezultă că punctele ", "A B şi

"C sunt coliniare. Observaţie: Dreapta 'd se numeşte transversala izogonală a dreptei d. 7) Într-un triunghi izogonalele a trei ceviene concurente sunt la rândul lor concurente. Demonstraţie. Vezi „Teorema lui Mathieu”. Observaţie: Punctul de concurenţă al cevienelor concurente şi punctul de concurenţă al izogonalelor acestora se numesc puncte izogonale. Consecinţe:

8) Simedianele sunt concurente într-un punct numit punctul lui Lemoine58. 9) Centrul de greutate şi punctul lui Lemoine sunt puncte izogonale.

58 Emile Lemoine (1840-1912) – inginer francez, contribuţii în geometrie

Fig. 157

A

B C A" A'

I

B'

C'

Page 155: Barbu_TEOREME_FUNDAMENTALE(2008).pdf

162

10) Izogonalul centrului cercului înscris într-un triunghi este el însuşi. 11) Înălţimea coborâtă dintr-un vârf al triunghiului pe latura opusă şi diametrul cercului circumscris triunghiului ce trece prin acel vârf sunt drepte izogonale.

Demonstraţie. Fie ,a aAH BC H BC⊥ ∈ şi 'A

punctul diametral opus punctului A în cercul circumscris triunghiului ABC. Avem:

1( ' ) ( ) ( )

2m AA B m AC B m AB= = ,

( ') ( ) 90am ABA m AH C= = ° , deci

' aBAA H AC≡ .

12) Consecinţă: Într-un triunghi ABC, unghiul format de înălţimea şi diametrul cercului circumscris ce pleacă din acelaşi vârf are măsura egală cu diferenţa măsurilor unghiurilor triunghiului din celelalte două vârfuri.

Demonstraţie. ( ' ) ( ) 2[90 ( )] ( ) ( )am A AH m A m C m C m B= − °− = − .

13) Izogonalul ortocentrului unui triunghi este centrul cercului circumscris triunghiului. Demonstraţia rezultă din proprietatea precedentă. 14) Fie P un punct nesituat pe cercul circumscris al unui triunghi ABC şi

' ' 'A B C triunghiul podar al lui P în raport cu triunghiul ABC. Perpendicularele din A, B, C pe ' ', ' 'B C A C respectiv ' 'A B sunt concurente într-un punct ',P izogonalul punctului P în raport cu triunghiul ABC. Demonstraţie. Triunghiul ' ' 'A B C este ortologic cu triunghiul ABC, centrul acestei ortologii fiind P. Conform teoremei de ortologie şi triunghiul ABC este ortologic cu triunghiul ' ' 'A B C , centrul acestei ortologii fie că este punctul 'P (Fig. 159). Patrulaterul

' 'BA PC fiind inscriptibil rezultă: ( ') 90 ( ' ' ) 90 ( ')= °− = °− = m ABP m AC B m BPA

( ') ( )= = m PBA m PBC , de unde

'ABP P BC≡ adică dreptele BP şi 'BP sunt izogonale. Analog, se arată că dreptele AP şi 'AP sunt izogonale, deci punctele P şi

'P sunt izogonale. 15) Fie M şi 'M două puncte izogonale în raport cu triunghiul ABC, iar , ,X Y Z şi

', ', 'X Y Z proiecţiile acestor puncte pe laturile triunghiului ABC. Să se arate că: i) dacă din punctele , ,X Y Z ca centre descriem cercuri care trec prin punctul 'M , atunci punctele 1 2 1 2 1 2, ; , ; ,A A B B C C de intersecţie ale cercurilor cu laturile ,BC CA , respectiv

A

B C

O

Fig. 158

aH

A'

A

B C

PP

P '

A'

B'

C'

Fig. 159

Page 156: Barbu_TEOREME_FUNDAMENTALE(2008).pdf

163

AB se află pe un cerc cu centrul în M. ii) dacă prin punctele ', ', 'X Y Z ca centre

descriem cercuri care trec prin punctul M, atunci punctele ' ' ' ' ' '1 2 1 2 1 2A ,A ;B ,B ;C ,C de

intersecţie ale cercurilor respective cu laturile BC, CA şi AB se află pe un cerc cu centrul în 'M şi egal cu precedentul. Demonstraţie. i) 2 2 2 2 2

1 1 'MA MX XA MX XM= + = +

( 1' =XM XA ca raze în acelaşi cerc) (Fig. 160). Din

teorema medianei în triunghiul 'XMM rezultă: 2 2 2

2 2( ' ) '

4

+ −=

XM XM MMXT , T fiind mijlocul

segmentului 'MM , de unde 2

2 2 2 '' 2

2

MMMX XM XT+ = + ,

deci 2

21

'2

2= +

MMMA XT . Cum XT YT ZT= =

rezultă 1 2 1 2= = = =MA MA MB MB 1 2=MC MC .

ii) Demonstraţie analoagă cu cea de la subpunctul precedent. 16) Dacă punctele izogonale M şi 'M au coordonatele normale ( , , )α β γ , respectiv

( ', ', ')α β γ , atunci ' ' '.α α β β γ γ⋅ = ⋅ = ⋅

Demonstraţie. Din proprietatea (1) rezultă ' ' '.α α β β γ γ⋅ = ⋅ = ⋅

Observaţie: Dacă punctul M este în interiorul (exteriorul) triunghiului, atunci izogonalul său 'M va fi în interiorul (exteriorul) triunghiului, deoarece ambele izogonale sunt în interiorul (exteriorul) unui unghi al triunghiului.

17) Fie ( , , )P α β γ şi ( ', ', ')Q α β γ două puncte izogonale exprimate în coordonate

baricentrice în raport cu un triunghi ABC. Atunci, 2 2 2

' ' '

a b c

αα ββ γγ= = , unde a,b,c sunt

lungimile laturilor BC,CA, respectiv AB.

A

B C

Fig. 160

M

M'

X

Y Z

X'

Y' Z'

1A 1B 2A

2B 1C

2C T

A

B C M 1P

2P3P

1Q

2Q3Q

P

Q

Fig. 161

Page 157: Barbu_TEOREME_FUNDAMENTALE(2008).pdf

164

Demonstraţie.Fie 1 = ∩P AP BC, 2 = ∩P BP AC , 3 = ∩P PC AB , 1 = ∩Q AQ BC ,

2 = ∩Q BQ AC şi 3 = ∩Q QC AB (Fig. 161). Avem:

1 1 2 2 3 3 1 1

', , , ,

'= − = − = − = −

uuur uuur uuuur uuur uuur uuur uuuur uuuurPB PC PC P A PA PB QB QC

γ α β γβ γ α β

2 2 3 3

' ', .

' '= − = −

uuuur uuuur uuuur uuuurQ C Q A Q A Q B

α βγ α

Din teorema lui Steiner rezultă 2

1 12

1 1

PB QB c

PC QC b⋅ = sau

2

2

'

'

c

b

γγββ

= , de unde 2 2

' '.

c b

γγ ββ= . Analog, 2 2 2

' ' '

a b c

αα ββ γγ= = .

18) Dacă punctele izogonale M şi 'M au coordonatele unghiulare ( , , ),λ µ ν respectiv

( ', ', ')λ µ ν , atunci ' 180 ( ), ' 180 ( ), ' 180 ( ).m C m A m Bλ λ µ µ ν ν+ = °+ + = °+ + = °+ Demonstraţie. Fie M în interiorul triunghiului ABC. Conform observaţiei precedente, punctul 'M va fi situate tot în interiorul triunghiului ABC. Avem:

1 8 0 ( ) ( )m M B C m M C Bµ = ° − − =

180 [ ( ) ( )] [ ( ) ( )]m ABC m MBA m ACB m MCA°− − − − =

( ) ( ) ( )m BAC m MBA m MCA= + + şi analog

' 180 ( ' ) ( ' ),m M BC m M CBµ = °− − de unde rezultă

că ' 180 ( ).m Aµ µ+ = °+ Analog,

' 180 ( )m Cλ λ+ = ° + şi ' 180 ( ).m Bν ν+ = ° +

Observaţie: Dacă una dintre coordonate are valoare negativă, vom considera suplementul său. 19) În triunghiul ABC se consideră izogonalele 1 2,AA AA , 1 2( , )A A BC∈ . Atunci

2

1 1 1

2 2 2

AA BA CA

AA BA CA

⋅=

⋅ .

Demonstraţie.Fie '1 1 ,A AA BC= ∩ '

2 2 .A AA BC= ∩

Avem 1 2 .BAA CAA≡ Dar 1 2 1 2' ' ' 'A BA A CA≡ ,

1 2 1 2' 'BAA CAA BCA CBA≡ ≡ ≡ de unde

rezultă că 1 2' ' ,BCA BCA≡ deci 1 2' 'BA CA≡ (1)

(Fig. 163). Triunghiurile 1 1'BA A şi 1CA A sunt

asemenea, rezultă 1

1

' 'BA BA

AC AA= , de unde

11

1

'AC BA

BAAA

⋅= (2). Din asemănarea triunghiurilor

A

B

C

M M'

Fig. 162

A

B C

Fig. 163

1A 2A

'1A

'2A

Page 158: Barbu_TEOREME_FUNDAMENTALE(2008).pdf

165

2 2'CA A şi 2BA A rezultă 2 2

2

'CA CA

AB AA= , adică 2

22

'AB CA

CAAA

⋅= (3). Din relaţiile (1), (2) şi

(3) rezultă: 1 2

1 2

AC BA AB CA

AA AA

⋅ ⋅= sau 1 2

2 1

BA AAAB

AC CA AA

⋅=

⋅ (4). Din relaţia lui Steiner

2

2 1

2 1

BA BAAB

AC CA CA

⋅ = ⋅ şi relaţia (4) rezultă

2

1 1 1

2 2 2

AA BA CA

AA BA CA

⋅=

⋅ .

20) Consecinţă: Lungimea simedianelor triunghiului ABC. Dacă 2A este mijlocul laturii BC, atunci 1AA devine simediana corespunzătoare laturii BC.

Astfel, din relaţia (4) rezultă 1

1

2a

BA mc

b a AA

⋅=

⋅ sau 1

1

2a

b m BAAA

a c

⋅ ⋅=

⋅. Din relaţia lui Steiner

rezultă 2

1

1

,BAc

b CA

=

de unde 2

1 2 2

acBA

b c=

+ . Atunci,

2 2

2a a

bcs m

b c= ⋅

+ (unde prin

as am

notat lungimea simedianei 1AA ). Analog, se obţin lungimile celorlalte două simediane, şi

anume: 2 2

2b b

acs m

a c= ⋅

+ respectiv

2 2

2.

c c

abs m

a b= ⋅

+

Fie triunghiul ABC şi C o curbă în planul triunghiului. Curba

trasată de izonalul unui punct ce variază pe curba C se

numeşte transformata izogonală a curbei. 21) Transformata izogonală a unui cerc ce trece prin două vârfuri ale triunghiului de referinţă este tot un cerc care trece prin cele două vârfuri ale triunghiului. Demonstraţie. Fie ( ) ( ' )= = m MBC m M BA α (Fig. 164)

şi ( ) ( ' )= = m MCB m M CA β . Atunci,

( ) = − −m M π β α ,

( ') 180 ( ( ) ) ( ( ) ) ( )= °− − − − = + +m M m B m C m Aα β β α ,

de unde rezultă că ( ) ( ') 180 ( )+ = °+ m M m M m A , deci

dacă punctul M descrie un cerc, atunci şi punctul 'M descrie un cerc. 22) În triunghiul ABC se consideră izogonalele AM şi AN; ,M N BC∈ , iar cercul circumscris triunghiului AMN intersectează laturile AB şi AC în E respectiv F. Dacă X BF CE= ∩ şi P AX BC= ∩ să se arate că pentru orice poziţie a izogonalelor AM şi AN, intersecţia bisectoarei unghiului A cu perpendiculara din P pe segmentul BC este un punct fix.

A

B C

M'

M α β β α

Fig. 164

Page 159: Barbu_TEOREME_FUNDAMENTALE(2008).pdf

166

Demonstraţie. Din EAM FAN≡ , de unde ( ) ( )m EM m FN= şi de aici EF MN . Fie

Q AP EF= ∩ (Fig. 165). Deoarece

triunghiurile AEQ şi ABP, respectiv AFQ şi ACP sunt asemenea, rezultă EQ AQ

BP AP= şi

FQ AQ

CP AP= , de unde

EQ FQ

BP CP= , deci (1)

EQ BP

FQ CP= . Din

asemănarea triunghiurilor EXQ şi PXC, respectiv XQF şi XPB rezultă EQ XQ

CP XP= , respectiv

FQ XQ

BP XP= deci

EQ FQ

CP BP= sau (2)

EQ CP

FQ BP= .Din (1)

şi (2) rezultă BP CP

CP BP= , deci BP CP= şi EQ FQ= . Perpendiculara în P pe segmentul

BC intersectează cercul circumscris triunghiului ABC în mijlocul arcului ,BC punct prin care mai trece şi bisectoarea unghiului A. 23) Într-un triunghi ABC , dreapta lui Simson a unui punct M este perpendiculară pe izogonala dreptei AM .

Demonstraţie. vezi „Dreapta lui Simson”. 24) În orice triunghi izogonalele izotomicelor a trei puncte coliniare sunt coliniare. Demonstraţie. Vom demonstra proprietatea utilizând coordonatele baricentrice relative.Dacă X este un punct de coordonate 1 2 3( , , )k k k atunci izotomicul său 'X are

coordonatele 1 2 3

1 1 1, ,

k k k

, iar izogonalul lui 'X are coordonatele 2 2 21 2 3"( , , )X a k b k c k -

rezultă din teorema lui Steiner – (a,b,c fiind lungimile laturilor triunghiului dat). Fie

1 2 3 1 2 3 1 2 3( , , ), ( , , ), ( , , )M N Pα α α β β β γ γ γ punctele coliniare. Atunci izogonalele

izotomicelor acestor puncte au coordonatele 2 2 2 2 2 2 2 2 2

1 2 3 1 2 3 1 2 3"( , , ), "( , , ), "( , , )M a b c N a b c P a b cα α α β β β γ γ γ . Deoarece PNM ,, sunt

coliniare rezultă: 0

321

321

21

=

γγγβββααα

, de unde 0

32

22

12

32

22

12

32

22

12

=

γγγβββααα

cba

cba

cba

, adică punctele

","," PNM sunt coliniare. 25) Într-un triunghi ABC , izogonalele punctelor Gergonne (Γ ) şi centrului antibisector ( )Z se află pe aceeaşi dreaptă a punctului lui Lemoine (K) al triunghiului ABC .

Demonstraţie. Deoarece punctul lui Nagel (N), iar centrul cercului înscris (I) în triunghiul ABC şi centrul de greutate (G) al triunghiului ABC sunt punctele izotomice ale lui , ,ZΓ respectiv G , iar IN , şi G sunt coliniare, atunci – conform proprietăţii

A

B C M P N

F E

X

Q

Fig. 165

Page 160: Barbu_TEOREME_FUNDAMENTALE(2008).pdf

167

precedente - izogonalele punctelor ,ZΓ şi G sunt coliniare. Cum punctul lui Lemoine este izogonalul centrului de greutate (G) al triunghiului ABC problema este demonstrată. 26) Într-un triunghi ABC , izogonalele centrului cercului înscris (I), punctului lui Gergonne (Γ ) şi punctul lui Nagel (N) sunt coliniare. Demonstraţie. Deoarece centrul antibisector ( )Z , Γ şi N sunt coliniare ( vezi

„Antibisectoare”) iar NI , şi T sunt punctele izotomice ale acestora - conform proprietăţii (24) – rezultă concluzia. 27) Într-un triunghi izotomicele izogonalelor a trei puncte coliniare sunt coliniare. Demonstraţie analoagă cu cea din proprietatea (24). 28) Izogonalele a trei drepte care prin vârfurile unui triunghi sunt concurente pe cercul circumscris. Demonstraţie. Fie AM şi 'AA drepte izogonale (punctele M şi 'A sunt pe cercul circumscris

triunghiului ABC ). Avem: ( ) ( ' )m MB m A C= (1). Fie

' 'BB AA , 'B fiind pe cercul circumscris triunghiului

ABC (Fig. 166). Atunci, ( ) ( ' ')m AB m A B= , deci

( ) ( )m AM m MB+ = ( ' ) ( ')m A C m CB+ (2). Din

relaţiile (1) şi (2) rezultă ( ) ( ')m AM m CB= , relaţie ce

arată că dreptele BM şi 'CB sunt izogonale. Analog se arată că paralela dusă prin C la 'AA şi dreapta CM sunt izogonale, deci izogonalele dreptelor ', ', 'AA BB CC sunt concurente în punctul M de pe cercul circumscris triunghiului ABC.

29) Fie C cercul circumscris triunghiului ABC, C’ un

cerc tangent interior în punctul A cercului C care

intersectează latura BC în punctele D şi E. Dreptele AD şi AE sunt izogonale în raport cu unghiul .BAC

Demonstraţie. Fie 'D = C AB∩ şi 'E = C ' .AC∩ Fie

(TA tangenta în A la cele două cercuri (Fig. 167). Avem:

( ) ( ) 1( )

2m TAB m ACB m AB= = şi ( ')m TAD =

( ' )m AE D = 1( '),

2m AD de unde rezultă că

' 'ACB AE D≡ adică dreptele ' 'D E şi BC sunt

paralele, deci ' ' ,D E DE adică ( ') ( ')m DD m EE= sau

( ') ( ')m BAD m EAE= , deci dreptele AD şi AE sunt izogonale.

30) Cercul care trece prin picioarele unei perechi de drepte izogonale şi prin vârful triunghiului, opus acestuia, este tangent cercului circumscris triunghiului considerat.

A

B C D E

E ' D'

A'

O

T

Fig. 167

A

B C

M

Fig. 166

A'

B'

C'

Page 161: Barbu_TEOREME_FUNDAMENTALE(2008).pdf

168

Demonstraţie. Fie O şi 'O centrele cercurilor C şi C’. Să demonstrăm că cercul circumscris

triunghiului ADE este tangent cercului circumscris C al triunghiului ABC (Fig. 167). Fie

'A piciorul înălţimii din A pe latura BC. Dreptele AO şi 'AA fiind izogonale şi cum AD şi AE sunt izogonale din ipoteză rezultă: ' 'O AE A AD≡ şi EAB DAC≡ care prin sumare dau ' ' ,O AB A AC OAB≡ ≡ adică punctele , 'O O şi A sunt coliniare, adică

cercul C’ este tangent cercului C.

31) Fie 1 1 1, ,A B C punctele de intersecţie ale unei transversale oarecare cu laturile

,BC CA , respectiv AB ale unui triunghi ABC şi , ,a b cO O O centrele cercurilor

circumscrise triunghiurilor 1 1 1 1,AB C BC A , respectiv 1 1CA B . Dreptele ,a bAO BO şi cCO

sunt concurente într-un punct M situat pe cercul circumscris triunghiului ABC . Demonstraţie. Dreapta aAO şi înălţimea

'AA a triunghiului 1 1AB C sunt izogonale

(Fig. 168). Analog, bBO şi înălţimea 'BB a

triunghiului 1 1BAC , cCO şi înălţimea 'CC

a triunghiului 1 1CA B sunt drepte izogonale.

Fie a c

M AO CO= I . Deoarece

' ' 'AA BB CC rezultă 1 1' 'A AB BCC≡ ≡

BCM MAB≡ ; deci M aparţine cercului circumscris triunghiului ABC . Analog ,se arată că bO B trece prin M.

32) Triunghiul antipodar al unui punct P şi triunghiul podar al izogonalului său 'P sunt omotetice. Demonstraţie. Vezi „Triunghiul antipodar”.

Fie 1 1 1 1 2 2 2 2( ) : 0, ( ) : 0d a x b y p d a x b y p+ − = + − = scrise sub formă normală (deci 2 2 2 21 1 2 2 1a b a b+ = + = ). O dreaptă (d) care aparţine fasciculului determinat de dreptele 1d şi

2d are ecuaţia *1 1 1 2 2 2( ) 0, .+ + + + + = ∈ a x b y p a x b y pλ λ Panta dreptei d este

1 2

1 2

.a a

mb b

λλ

+= −

+

33) Dacă dreapta (d) are ecuaţia 1 1 1 2 2 2( ) 0+ + + + + =a x b y p a x b y pλ (cu

2 2 2 21 1 2 2 1a b a b+ = + = ) atunci izogonala sa, dreapta ( '),d are ecuaţia

1 1 1 2 2 2

1( ) 0.+ + + + + =a x b y p a x b y p

λ

Demonstraţie. O dreapta ( ')d a acestui fascicul de drepte este izogonală dreptei d are

ecuaţia 1 1 1 2 2 2'( ) 0+ + + + + =a x b y p a x b y pλ şi panta 1 2

1 2

'' .

'

a am

b b

λλ

+= −

+ Fie α şi β

unghiul dintre dreptele d şi 1d respectiv 'd şi 2d (Fig.169). Avem:

A

C'

C B

B'

1A

1C

1B Oa

A' Ob

M Oc

Fig. 168

Page 162: Barbu_TEOREME_FUNDAMENTALE(2008).pdf

169

1 2 1

1 2 1 1 2 2 1

1 1 2 1 2 1 2

1 1 2

( )

1 ( )1

a a a

b b b a b a btg

a a a a a b b

b b b

λλ λ

αλ λλ

+− +

+ −= =

+ + ++ ⋅+

(1)

2 1 2

2 1 2 1 2 2 1

2 1 2 1 2 1 2

2 1 2

'1

a a a

b b b a b a btg

a a a a a b b

b b b

λλ

βλ λλ

+− +

+ −= =

+ + ++ ⋅+

(2)

deoarece 2 2 2 21 1 2 2 1.a b a b+ = + = Drepta ( ')d este izogonala

lui (d) dacă tg tgα β= şi din relaţiile (1) şi (2) rezultă

1.'

=λλ

Deci, dacă dreapta (d) are ecuaţia 1 1 1 2 2 2( ) 0+ + + + + =a x b y p a x b y pλ atunci

izogonala sa, dreapta ( '),d are ecuaţia 1 1 1 2 2 2

1( ) 0.+ + + + + =a x b y p a x b y p

λ

34) Fie M un punct pe bisectoarea unghiului BAC a triunghiului ABC şi 'M

izogonalul conjugat al lui M în raport cu triunghiul ABC. Cercul ce trece prin , 'M M şi este tangent laturii BC este tangent şi cercului circumscris triunghiului ABC. Demonstraţie. Fie că un cerc C este

tangent interior cercului circumscris triunghiului ABC în D şi tangent laturii BC în punctul E, iar M şi 'M punctele de intersecţie dintre bisectoarea

unghiului A şi cercul C. Dacă F, al

doilea punct de intersecţie dintre dreapta DE şi cercul circumscris triunghiului ABC. Fie P punctul de intersecţie dintre

tangentele duse la cercul C în punctele D

şi E. Atunci, ,PED PDE≡ adică

1 1[ ( ) ( )] [ ( ) ( )],

2 2+ = +m BD m FC m BD m BF

de unde ( ) ( ), ( ) ( )= =m BF m FC m BF m FC ,

relaţie ce arată că punctele F, M şi 'M sunt coliniare. Din puterea punctului faţă de un cerc avem ' (1)FE FD FM FM⋅ = ⋅ (Fig.170). Din asemănarea triunghiurilor BFE şi DFB

( ( ) ( )= m BFD m BFE şi 1

( ) ( ) ( )2

= = m FBE m BDF m BAC ) rezultă

2 (2).⋅ =FE FD FB Din relaţiile (1) şi (2) rezultă 2'FM FM FB⋅ = relaţie ce arată că

triunghiurile 'BFM şi MFB sunt asemenea, deci ' (3).FBM BMF≡ Atunci,

( ) ( ') ( ) ( ),+ = + m FBC m CBM m BAF m MBA de unde ' ,CBM MBA≡ adică

punctele M şi 'M sunt izogonal conjugate.

O

α β

d d ' 1d

2d

Fig. 169

A

B

C

D

E

F

P

O

M

M'

Fig. 170

Page 163: Barbu_TEOREME_FUNDAMENTALE(2008).pdf

170

35) Ceviana izogonală unei ceviene de rangul k este ceviana de rang (2-k) şi reciproc. Demonstraţie. Fie AD o ceviană de ordinul k în triunghiul

ABC şi AE izogonala sa, atunci .k

BD c

DC b

=

Din teorema

sinusurilor rezultă: s in ( ) sin sin sin

, ,B C

BE AE EC AE

ϕ θ ϕ+= = de unde

sin( ) sin sin( ) (1).

sin sin sin

BE C c

EC B b

ϕ θ ϕ θϕ ϕ+ +

= ⋅ = ⋅ Din

sin sin B

BD AD

ϕ= şi

sin( ) sin,

C

DC AD

ϕ θ+= de unde

sin sin

sin( ) sin

BD C

DC B

ϕϕ θ

= ⋅ =+

sin

,sin( )

kc c

b b

ϕϕ θ

⋅ = + deci

1sin

(2).sin( )

kc

b

ϕϕ θ

− = +

Din relaţiile (1) şi (2) rezultă

1 2 2

.k k k

BE b c b c

EC c b c b

− − − = ⋅ = =

36) Fie AE izogonala antibisectoarei AD a triunghiului ABC, , ( ).E D BC∈ Atunci,

3

.BE c

EC b

=

Demonstraţie. Deoarece antibisectoarea este ceviană de rangul (-1) rezultă conform proprietăţii precedente că ceviana izogonală antibisectoarei este ceviana de rang

(2 ( 1)) 3,− − = deci 3

.BE c

EC b

=

37) Izogonalele punctelor 1F şi 2F ale lui Fermat sunt punctele izodinamice S şi 'S ale triunghiului ABC. Demonstraţie. Vezi „Puncte izodinamice”.

A

B C

Fig. 171

D E

ϕ ϕ θ

Page 164: Barbu_TEOREME_FUNDAMENTALE(2008).pdf

171

I.41. Izogonale exterioare „Specificul meseriei mele, matematica, e că se face oriunde, oricum şi că trebuie să o faci oriunde, oricând şi oricum.” - Grigore Moisil59

Două drepte simetrice faţă de bisectoarea unui unghi, situate în exteriorul unghiului şi care trec prin vârful acestuia se numesc izogonale exterioare. Prin fiecare vârf al triunghiului ABC se construiesc izogonalele exterioare Ax şi 'Ax , By

şi 'By , Cz şi 'Cz ; fie 1 A By Cz'= ∩ , 1 B Cz AX '= ∩ , 1C Ax By' .= ∩ Să se arate

că triunghiul 1 1 1ABC este omologic cu triunghiul ABC. Demonstraţie.

Fie 1 ' ,A AA BC= ∩ 1 ' ,B BB AC= ∩ 1 'C CC BA= ∩ , 1 1( ) ( ) ,m BAC m CAB α= =

1 1( ) ( )m ABC m CBA= = β , 1 1( ) ( ) .m BCA m ACB= = γ Avem:

1

1

[ ] 1

[ ] 1

sin( )'

' sin( )ABA

ACA

A AB BA BA B

A C A AC CA C

⋅ ⋅ += =

⋅ ⋅ +

βγ

, 1

1

sin( )'

' s in( )

BC CB CB C

B A AB AB A

⋅ ⋅ +=

⋅ ⋅ +

γα

, respectiv

1

1

sin( )'

' sin( )

AC AC BC A

C B BC BC B

⋅ ⋅ +=

⋅ ⋅ +

ββ

. Înmulţind membru cu membru relaţiile precedente rezultă

că: ' ' '

1 1 1 1 1 1

1 1 1 1 1 1' ' '

AB BC C A AB BC C AAB BC C A AB BC CA

A C B A C B AC BA CB AC B A C B AC B A C B

⋅ ⋅ = ⋅ ⋅ ⋅ ⋅ ⋅ = ⋅ ⋅

(1) .

Teorema sinusurilor aplicată în triunghiurile 1 1 1, ,BAC ABC AC B ne dă :

1

1

sin

sin

BA

CA=

γβ

, 1

1

sin

sin

CB

AB=

αγ

, 1

1

sin

sin

AC

BC=

βα

(2). Din relaţiile (1) şi (2) rezultă

' ' '

1' ' '

AB BC C A

A C B A C B⋅ ⋅ = , iar din reciproca teoremei lui Ceva rezultă că dreptele 1 1 1, ,AA BB CC

sunt concurente, deci triunghiurile 1 1 1ABC şi ABC sunt omologice.

59Grigore Moisil (1906-1973) – matematician român, profesor la Universitatea din Iaşi, membru al Academiei

Române, contribuţii importante în informatică

A

B C

1A

1B

1C

A'

C'

x

y

x '

y '

Fig. 172

B' z

z '

Page 165: Barbu_TEOREME_FUNDAMENTALE(2008).pdf

172

I.42. Dreptele lui Schwatt60

„În fiecare ştiinţă este numai atâta ştiinţă adevărată câtă matematică conţine.” Immanuel Kant61

Dreptele care conţin mijloacele înălţimilor unui triunghi şi mijloacele laturilor corespunzătoare se numesc dreptele lui Schwatt. Teorema lui Schömilch Dreptele lui Schwatt sunt concurente în punctul lui Lemoine al triunghiului. Demonstraţie. Fie ,ABC , ,

a b cH H H picioarele înălţimilor; D, E şi F mijloacele

înălţimilor

,

a bAH BH , respectiv

cCH ; , ,

a b cM M M mijloacele laturilor BC, CA, respectiv AB (Fig.

173); ', 'a b

M A M B şi 'c

M C înălţimi în triunghiul a b c

M M M (ele sunt mediatoarele

laturilor triunghiului ABC ) şi ' ' '= I Ia b c

O M A M B M C (Fig. 174). Din congruenţa

triunghiurilor c

AM D şi 'a b

M A M rezultă că ' ,≡c b

M D A M adică punctele D şi 'A sunt

simetrice faţă de mijlocul segmentului b c

M M . Analog, punctele E şi 'B sunt izotomice pe

(a c

M M ), F şi 'C sunt izotomice pe (a b

M M ). Cum dreptele ', ', 'a b c

M A M B M C sunt

concurente, rezultă că şi dreptele , ,a b c

M D M E M F sunt concurente într-un punct K care este

izotomicul centrului cercului circumscris O al triunghiului ABC , în raport cu triunghiul median (vezi „Simediane”), adică K este punctul lui Lemoine al triunghiului .ABC Consecinţă: Punctul simedian al unui triunghi dreptunghic este mijlocul înălţimii corespunzătoare ipotenuzei.

60 Isaac Schwatt (1867-1934) – profesor la Universitatea din Pennsylvania 61 Immanuel Kant (1724-1804) – filosof german, profesor la Universitatea din Königsberg

A

B C

bM

aM

cM D

E F K

bH

aH

cH

Fig. 173

bM cM

aM

B'

E

A'

C'

F O

K

Fig. 174

D

Page 166: Barbu_TEOREME_FUNDAMENTALE(2008).pdf

173

I.43. Ortopolul unei drepte

„Matematica este ca dragostea... o simplă idee, dar poate să devină complicată.” - Robert Drabek62

Teorema ortopolului Fie triunghiul ABC şi o dreaptă oarecare d ce nu trece prin vârfurile triunghiului ABC. Fie 1A , 1B , 1C proiecţiile vârfurilor A, B, C ale triunghiului ABC pe dreapta d. Să se

arate că proiecţiile duse din punctele 1A , 1B , 1C pe laturile BC, CA, respectiv BA sunt

concurente. Demonstraţie. Fie 1 2 A A BC⊥ ,

1 2B B AC⊥ şi 1 2C C AB⊥ , 2A ∈ BC ,

2B AC∈ , 2C AB∈ , 1 2 1 2 A A B Bθ = I ,

1 2 1 2 '=A A C C ABθ I I , 1D= ,AA BCI

1 2 E BB BC= I , 1 1 2T=AA B BI (Fig. 175).

Din asemănarea triunghiurile ACD şi

1 1B Aθ (( 1 1 2( ) ( ) 90 ( )m DCA m B A m B ECθ= = °−

şi 1 1 2( ) ( ) 90 ( )m DAC m B A m ATBθ= = °− ),

avem: 1 1

1

(1).A BAD

CD Aθ= Analog, din

asemănarea triunghiurilor BAD şi 1 1'C Aθ

rezultă 1

1 1

'(2).

ABD

AD AC

θ= Din faptul că

1AA || 1BB || 1CC rezultă că punctele θ şi 'θ coincid.

Observaţii: Punctul θ de concurenţă al celor trei perpendiculare se numeşte ortopolul dreptei d. 1) Ortopolii a două drepte paralele între ele în raport cu acelaşi triunghi se află pe dreapta perpendiculară pe cele două drepte, distanţa dintre cei doi ortopoli fiind egală cu distanţa dintre dreptele paralele. Demonstraţie: Fie ' ''d d , 'θ şi "θ ortopolii

dreptelor 'd şi "d în raport cu triunghiul ABC (Fig. 176). Deoarece ' '',d d ' ' '' '',B Bθ θ

' ' " "C Cθ θ rezultă că triunghiurile ' ' 'B Cθ şi '' '' ''B Cθ sunt omotetice. Cum ''BB şi ''CC sunt

perpendiculare pe dreptele paralele 'd şi "d , avem că ' '' ' ''B B C C≡ şi de aici rezultă că triunghiurile ' ' 'B Cθ şi '' '' ''B Cθ sunt congruente, deci ' '' ' '' ' ''.B B C C θ θ≡ ≡

62Robert Drabek – matematician ceh

A

B

1A 1B

D C

2B

θ

d

T 2C

1C

2A

E

Fig. 175

A

B

C

B' C'

B" C"

d '

d"

Fig. 176

Page 167: Barbu_TEOREME_FUNDAMENTALE(2008).pdf

174

Fie M şi N intersecţiile unei drepte d cu cercul circumscris triunghiului ABC, O centrul cercului circumscris triunghiului ABC, ', ', 'A B C proiecţiile vârfurilor A, B, C pe dreapta d,

, ,a b c

M M M mijloacele laturilor BC, CA respectiv AB, R mijlocul segmentului PQ,

( , ), ( , ), ( , )a a b b c cP Q P Q P Q proiecţiile punctelor P şi Q pe BC, CA respectiv AB. Fie θ

ortopolul dreptei d în raport cu triunghiul ABC.

2) Patrulaterele ' ' ,a a

B C Q P ' 'b b

C A PQ şi ' 'c c

A B PQ sunt inscriptibile, iar cercurile circumscrise lor se intersectează în ortopolul dreptei d. Demonstraţie. Dreptele '

aB P şi CQ sunt paralele, ambele fiind antiparalele cu coarda BP,

deci ' '.aP B P CQC≡ Patrulaterul '

aQC CQ fiind inscriptibil rezultă

' 'a

CQC C Q C≡ , de unde ' ' 'a aP B C C Q C≡ , adică patrulaterul ' '

a aB C Q P este

inscriptibil. Analog, ' 'b b

C A PQ şi ' 'c c

A B PQ sunt patrulatere inscriptibile (Fig. 177). Fie

, ,a b c

O O O centrele cercurilor circumscrise acestor patrulatere. Punctul a

O aparţine

perpendicularei duse în mijlocul segmentului ' ',B C perpendiculară paralelă cu 'BB şi

'.CC Atunci, a

M aparţine acestei perpendiculare, deci .a a

M O OR Deoarece a

O aparţine

şi perpendicularei duse din prin mijlocul segmentului ,a aP Q perpendiculară ce conţine

punctul R şi este paralelă cu ,a

OM rezultă că patrulaterul a a

OM O R este paralelogram, de

unde a a

M O OR≡ şi .a a

M O OR Analog, ,b b b bM O OR M O OR≡ şi

,c c c cM O OR M O OR≡ , deci patrulaterele , a b b a

O O M M b c c b

O O M M şi c a a c

O O M M

sunt paralelograme, adică laturile triunghiurilor a b c

O O O şi 1 1 1A B C sunt respectiv egale şi

paralele. Deoarece a b a b

O O M M AB rezultă cercurile circumscrise patrulaterelor

' 'a a

B C Q P şi ' 'b b

C A PQ (care trec prin 'C ) au axa radicală perpendiculară pe AB, deci axa

radicală este chiar perpendiculara dusă din 'C pe AB, adică cercurile circumscrise

A

B

C

d

R

aM

θ

Qs

Ps

O

P A' aO

aP cP

aQ

cQ

B' C'

Fig. 177

Q

Page 168: Barbu_TEOREME_FUNDAMENTALE(2008).pdf

175

patrulaterelor ' 'a a

B C Q P , ' 'b b

C A PQ şi ' 'c c

A B PQ se intersectează în ortopolul dreptei d

în raport cu triunghiul ABC.

Observaţie: Dacă dreapta d este un diametru în cercul circumscris triunghiului ABC, atunci punctele , ,

a b cO O O coincid cu punctele , ,

a bM M respectiv

cM .

3) Triunghiurile a b c

O O O şi a b c

M M M sunt omotetice şi congruente. Demonstraţia rezultă din aplicaţia precedentă.

4) Triunghiul a b c

O O O este ortologic cu triunghiul ABC .

Demonstraţia este evidentă deoarece perpendicularele duse din , ,a b c

O O O pe BC, CA,

respectiv AB sunt concurente în R. Observaţie: Dacă în loc de trei lungimi egale cu OR se consideră pe cele trei perpendiculare duse din , ,

a b cM M M pe d trei puncte 1 1 1, ,A B C astfel încât 1 1 1 ,

a b cAM B M C M≡ ≡ atunci

se poate da următoarea generalizare: 5) Fie d o dreaptă în planul triunghiului ABC, ', ', 'A B C proiecţiile vârfurilor sale pe d şi

, ,a b c

M M M mijloacele laturilor BC, CA, respectiv AB. Pe perpendicularele duse din

', ', 'A B C pe dreapta d se consideră, în acelaşi sens, punctele 1,A 1B , respectiv 1C astfel

încât 1 1 1 .a b c

AM B M C M≡ ≡ Cercurile având centrele în punctele 1,A 1B , respectiv 1C şi

trec prin punctele ( ', '), ( ', ')B C C A respectiv ( ', ')A B se intersectează în ortopolul dreptei d.

6) Ortopolul dreptei aparţine cercului circumscris triunghiului

a b cO O O .

Demonstraţia rezultă din reciproca teoremei lui Salmon. 7) Dacă prin proiecţiile ', ', 'A B C ale vârfurilor unui triunghi ABC, pe o dreaptă d, ducem paralele la laturile triunghiului ABC, se formează un triunghi omotetic cu triunghiul ABC şi pe al cărui cerc circumscris se află ortopolul dreptei d în raport cu triunghiul ABC. Demonstraţie. Proprietatea rezultă din faptul că triunghiul format şi triunghiul

a b cO O O sunt

omotetice, având ortopolul dreptei d drept centru de omotitie. 8) Dreptele lui Simson ale punctelor M şi N în raport cu triunghiul ABC, trec prin ortopolul dreptei d. Demonstraţie. Din patrulaterul inscriptibil '

aBB QQ rezultă ',

aBQP BQ B≡ iar în

cercul de centru a

O avem ' ' ,a a

BQ B PC Q≡ de unde 'a

BQP PC Q≡ , deci

'a

BQ C P . Cum dreptele 'C θ şi c

QQ sunt paralele, fiind pependiculare pe AB, rezultă că

'a a a a cPQ Q C BQ Qθ θ≡ ≡ , deci dreapta lui Simson a lui Q trece prin ortopolul dreptei

d. Analog se arată că dreapta lui Simson a punctului P trece prin ortopolul dreptei d.

Page 169: Barbu_TEOREME_FUNDAMENTALE(2008).pdf

176

9) Ortopolii a două drepte paralele între ele în raport cu un triunghi aparţin unei drepte a lui Simson. Demonstraţie. Fie 'd d , θ şi

'θ ortopolii dreptelor d şi 'd în raport cu triunghiul ABC (Fig. 178). Fie M şi 'M punctele de intersecţie dintre dreapta d cu cercul circumscris triunghiului ABC . Dreapta 'θθ este perpendiculară pe dreptele d şi

'd (conform proprietăţii 1). Dreptele lui Simson

Ms şi 'M

s se

intersectează în θ (cf. proprietăţii precedente), iar dreptele prpendiculare duse din punctele M şi 'M pe dreptele lui Simson 'M

s

şi Ms sunt concurente într-un punct N ce aparţine cercului circumscris

triunghiului ABC (vezi „Triunghiuri ortopolare”), triunghiul 'MNM fiind un triunghi S în raport cu triunghiul ABC . Deoarece într-un triunghi S dreapta lui Simson a unui vârf în raport cu celălalt triunghi este perpendiculară pe latura opusă vârfului considerat (vezi „Triunghiuri ortopolare”) rezultă că dreapta lui Simson a punctului N este perpendiculară pe 'MM , trece prin punctul comun dreptelor

Ms şi 'M

s - adică prin θ - deci şi prin 'θ . Am

arătat astfel, că dreapta 'θθ este dreapta lui Simson a punctului N. 10) Dacă dreapta d trece prin centrul cercului circumscris triunghiului ABC, atunci ortopolul dreptei d aparţine cercului lui Euler al triunghiului ABC. Demonstraţie. Fie M şi 'M punctele de intersecţie dintre dreapta d cu cercul circumscris triunghiului ABC . Deoarece dreptele lui Simson

Ms şi 'M

s ale punctelor M şi 'M se

intersectează în ortopolul θ (cf. th. (8)), iar punctul de intersecţie al dreptelor lui Simson ale punctelor M şi 'M , diametral opuse, aparţine cercului lui Euler al triunghiului ABC (vezi „Dreapta lui Simson”) rezultă că ortopolul dreptei d aparţine cercului lui Euler al triunghiului ABC.

11) Proiecţiile vârfurilor triunghiului ABC pe un diametru al cercului circumscris triunghiului ABC sunt simetricele ortopolului respectiv faţă de laturile triunghiului median al triunghiului ABC .

Demonstraţie. Fie 'MM un diametru al cercului circumscris triunghiului ABC (Fig. 179). Fie 1A

proiecţiile punctului A pe 'MM şi a b c

M M M

triunghiul median al triunghiului ABC . Punctul 1A

aparţine cercului circumscris triunghiului b c

AM M

având AO drept diametru. Cercul este simetric cercului celor nouă puncte ale triunghiului ABC (vezi „Cercul lui Euler”), deci θ simetricul lui 1A faţă de

b cM M

aparţine cercului celor nouă puncte al triunghiului ABC . Întrucât ortopolul unui diametru

A

B C

M Ms

N

θ M'

M's

Fig. 178

d

d '

A

B C aM

bM cM

M

M'

aH

1A A"

O

Fig. 179

θ

Page 170: Barbu_TEOREME_FUNDAMENTALE(2008).pdf

177

aparţine cercului lui Euler al triunghiului (cf. proprietăţii (10)), iar ortopolul unei drepte d aparţine perpendicularei ridicate din proiecţia ( 1A ) pe d a unui vârf (A) al triunghiului pe

latura opusă rezultă că punctul θ este ortopolul dreptei d. Observaţie: Fie 1B şi 1C simetricele ortopolului θ faţă de laturile

a cM M ,

respectiva b

M M ; deci 1 1,a c a b

B M M C M Mθ θ⊥ ⊥ , iar θ aparţinând cercului celor nouă

puncte al triunghiului ABC , atunci punctele '', '', ''A B C de intersecţie a dreptelor

1 1 1, ,A B Cθ θ θ cu laturile triunghiului median determină dreapta lui Simson a punctului θ

în raport cu cercul circumscris triunghiului median. 12) Fie θ ortopolul unui diametru (d) al cercului circumscris triunghiului ABC . Dreapta lui Simson a punctului θ în raport cu triunghiul median

a b cM M M al

triunghiului ABC este paralelă cu dreapta d şi echidistantă de θ şi diametrul d. Demonstraţie. Deoarece '' ''A B este dreapta lui Simson a punctului θ , rezultă - conform teoremei (1) - că este paralelă cu d şi va trece prin mijlocul distanţei dintre ortopolul θ şi dreapta d. 13) Fie

a b cH H H triunghiul ortic al unui triunghi ABC ,θ ortopolul unui diametru al

cercului circumscris triunghiului ABC şi 1 1 1, ,A B C proiecţiile punctelor , ,A B respectiv

C pe acest diametru. Patrulaterele 1aAH Aθ , 1 1,b cBH B CH Cθ θ sunt trapeze isoscele.

Demonstraţie. Deoarece 1A şi a

H sunt simetricele punctelor θ , respectiv a

H faţă de

latura b c

M M a triunghiului median a b c

M M M al triunghiului ABC , rezultă că patrulaterul

1aAH Aθ este trapez isoscel. Observaţii: 1) Distanţa dintre un vârf al unui triunghi ABC şi ortopolul unui diametru al cercului circumscris triunghiului este egală cu distanţa dintre proiecţiile aceluiaşi vârf pe latura opusă şi pe diametru. 2) Într-un triunghi ABC , distanţa dintre ortopolul unui diametru al cercului circumscris şi piciorul unei înălţimi este egală cu distanţa dintre vârful din care pleacă înălţimea şi diametrul dat.

14) Fie 0B şi 0C punctele diametral opuse vârfurilor B şi C ale unui triunghi ABC , în

cercul circumscris acestui triunghi. Ortopolul dreptei 0 0B C în raport cu

triunghiul ABC este punctul A . Demonstraţie. Deoarece patrulaterul 0 0BCB C este

dreptunghi, 0C şi 0B vor fi proiecţiile punctelor B,

respectiv C pe dreapta 0 0B C (Fig.180). Cum 0CC şi 0BB

sunt diametre în cercul circumscris triunghiului ABC

rezultă că 0 0( ) ( ) 90 ,m C AC m B AB= = ° adică

perpendicularele duse din punctele 0C şi 0B pe AC

respectiv AB se intersectează în punctul A care va fi ortopolul dreptei 0 0B C .

A

B C

O

0B 0C

Fig. 180

Page 171: Barbu_TEOREME_FUNDAMENTALE(2008).pdf

178

Consecinţe: 15) Ortopolii diametrelor care trec prin vârfurile triunghiului sunt picioarele respective ale înălţimilor. 16) Ortopolul unei laturi este ortocentrul triunghiului. 17) Ortopolul unui diametru paralel cu o latură, se află în punctul eulerian al înălţimii care cade pe latura respectivă. 18) Ortopolul unui diametru paralel cu o înălţime se află în mijlocul laturii pe care cade înălţimea respectivă.

19) Punctul lui Feuerbach al triunghiului ABC este ortopolul diametrului ce trece prin I - centrul cercului înscris în triunghiul ABC .

Demonstraţie. Deoarece ortopolul unei drepte se află la intersecţia dreptelor lui Simson ale celor două puncte unde dreapta intersectează cercul circumscris triunghiului ABC , iar dreptele lui Simson ale extremităţilor diametrului ce trece prin I se intersecteză în punctul lui Feuerbach al triunghiului ABC (vezi „Punctele lui Feuerbach”), rezultă concluzia. Observaţie: Ortopolul unui diametru al cercului circumscris unui triunghi ABC care trece prin centrul unui cerc tritangent este punctul lui Feuerbach corespunzător. 20) Ortopolii corespunzători la doi diametri perpendiculari ai cercului circumscris unui triunghi ABC sunt două puncte diametral opuse în cercul lui Euler al triunghiului ABC .

Demonstraţie. Dacă θ şi 'θ sunt ortopolii corespunzători diametrelor perpendiculare d şi 'd , atunci θ şi 'θ sunt puncte pe cercul lui Euler al triunghiului ABC . Dreapta lui Simson

dθ a punctului θ în raport cu triunghiul median este paralelă cu dreapta 'd , deci 'd dθ θ⊥ .

Dacă "θ este punctul diametral opus lui θ , atunci dreptele lui Simson ale punctelor θ şi "θ sunt perpendiculare, deci "d dθ θ⊥ , de unde rezultă că punctele "θ şi 'θ coincid.

21) Fie MN coardă a cercului circumscris unui triunghi ABC , perpendiculară pe latura BC . Distanţa dintre ortopolii

Mθ şi

Nθ ai dreptelor AM, respectiv AN în raport cu

triunghiul ABC este egală cuMN . Demonstraţie. Fie P MN BC= ∩ . Ortopolii

Mθ şi

Nθ sunt punctele de intersecţie dintre

înălţimea din A (dreapta lui Simson a punctului A) şi dreptele lui Simson ale punctelor ,M respectiv N. Dreptele lui Simson ale punctelor M şi N sunt paralele duse prin P la AN respectiv AM, deci patrulaterele

MPNAθ şi

NA CMθ sunt paralelograme.

Atunci, M

A PNθ ≡ şi ,N

A MPθ ≡ de unde

rezultă M N

A A PN PMθ θ+ = + adică

.N M

NMθ θ ≡

Consecinţă: 22) Distanţa dintre ortopolii celor două bisectoare ale unui unghi al triunghiului ABC este egală cu diametrul cercului circumscris triunghiului ABC.

A

B

C

M

P N

Fig. 181

Page 172: Barbu_TEOREME_FUNDAMENTALE(2008).pdf

179

23) Ortopolul unei drepte în raport cu un triunghi coincide cu centrul radical al cercurilor având centrele în vârfurile triunghiului anticomplementar şi tangente la dreapta dată.

Demonstraţie. Fie triunghiul ABC, ' ' 'A B C triunghiul anticomplementar al triunghiului ABC, 1 1 1, ,A B C şi

", ", "A B C proiecţiile punctelor A, B, C respectiv ', ', 'A B C pe dreapta d, iar θ ortopolul dreptei d în raport cu triunghiul ABC. Dar ", ", "A B C sunt punctele de tangenţă dintre cecurile având centrele în ', ', 'A B C şi dreapta d, iar axele radicale dintre aceste cercuri luate câte două se obţin ducând perpendiculare din mijloacele segmentelor

" ", " ", " "A B B C C A pe linia centrelor ' ', ' ',A B B C respectiv ' 'C A , intersecţia acestor axe fiind centrul radical al acestor cercuri. Deoarece mijloacele segmentelor

" ", " ", " "B C C A A B sunt punctele 1 1, ,A B respectiv 1C , iar perpendicularele din 1 1 1, ,A B C

pe ' ', ' ',B C A C respectiv ' 'A B sunt perpendiculare BC, CA respectiv AB (pentru că ' ' ,B C BC ' 'A C AC şi ' 'A B AB ) , deci sunt concurente în ortopolul θ , rezultă

concluzia.

A

B C

A'

B' C'

A" B"

C" 1A 1B 1C

Fig. 182

θ

Page 173: Barbu_TEOREME_FUNDAMENTALE(2008).pdf

180

I.44. Dreapta lui Aubert

„Poezia este o creaţie, o compoziţie, o ficţiune, iar matematica a fost numită cea mai sublimă şi mai prodigioasă dintre ficţiuni.” – Emil Picard63

Fie ',',' CBA punctele de intersecţie dintre o dreaptă d cu laturile CABC, respectiv AB ale unui triunghi .ABC Ortocentrele triunghiurilor '','', CABBCAABC şi CBA '' se află pe aceeaşi dreaptă. Demonstraţie. Fie H ortocentrul triunghiului ABC , cba HHH triunghiul ortic şi 321 ,, CCC cercurile de

diametre ', 'AA BB respectiv '.CC Atunci rezultă că :

cba HHHCHHHBHHHA ⋅=⋅=⋅ (vezi „Triunghiul ortic”), deci punctul H are

puteri egale fata de cercurile 21,CC şi 3C . Notăm cu ijd axa radicală a cercurilor iC şi

jC ( jibji ≠= ,,1, ). Deoarece ba HHHBHHHA ⋅=⋅ rezultă că 12dH ∈ şi din

cb HHHCHBHH ⋅=⋅ rezultă că .23dH ∈ Din teorema lui Gauss ştim ca mijloacele

segmentelor ',' BBAA şi 'CC sunt coliniare, ele aparţinând dreptei lui Gauss (g). Atunci

12dg ⊥ şi 13dg ⊥ se cum 2312 ddH I= rezultă că dreptele 12d şi 23d coincid. Deci

132312 ddd == şi gd ⊥' , deci '.dH ∈ Analog se arată că ortocentrele triunghiurilor

'','' CABBCA şi CBA '' se afla pe dreapta 'd . Observaţie: Dreapta 'd se numeşte dreapta lui Aubert.

63 Emil Picard (1856-1941) – matematician francez, contribuţii în geometrie şi algebră

A

B C aH

B'

cH

C'

A'

d bH H

Fig. 183

Page 174: Barbu_TEOREME_FUNDAMENTALE(2008).pdf

181

I.45. Antibisectoarea

„Ideile, ca şi plantele, au epoca lor, în care apar în diverse locuri,la fel cum primăvara ghioceii răsar pretutindeni unde luminează soarele.” – János Bolyai64

Se numeşte antibisectoare a unui triunghi izotomica unei bisectoare interioare a unui unghi al triunghiului.

1) Antibisectoarele unui triunghi sunt concurente. Demonstraţie. Fie ', ', 'AA BB CC bisectoarele triunghiului ABC şi 1,AZ 2 ,BZ 3CZ

antibisectoarele triunghiului ABC . Evident, 1 ',BZ CA≡ 1 ',CZ BA≡ 2 'AZ CB≡ ,

2 'CZ AB≡ , 3 3', 'AZ BC BZ AC≡ ≡ . Deoarece bisectoarele sunt concurente, din teorema

lui Ceva rezultă ' ' '

1' ' '

BA AC CB

A C C B B A⋅ ⋅ = sau 31 2

1 3 2

1BZCZ AZ

BZ AZ CZ⋅ ⋅ = şi din reciproca teoremei lui

Ceva rezultă că antibisectoarele sunt concurente.

Punctul de concurenţă al antibisectoarelor îl vom nota cu Z şi îl vom numi centrul antibisector al triunghiului ABC .

2) Consecinţă : Centrul cercului înscris I şi centrul antibisector Z al triunghiului ABC sunt puncte izotomice.

Observaţie: Antibisectoarea este o ceviană de rang (-1), deoarece 1

1

1

BZ AB

Z C AC

− =

.

3) Într-un triunghi ABC, izogonalele centrului antibisector Z şi punctului lui Gergonne Γ se află pe aceeaşi dreaptă cu punctul lui Lemoine K al triunghiului ABC. Demonstraţie. Vezi „Puncte izogonale”. 4) Fie M un punct din planul unui triunghi ABC şi Z centrul antibisector al triunghiului

ABC. Atunci: bcMA caMB abMC

MZab bc ca

+ +=

+ +

uuur uuur uuuuruuuur

, unde a, b, c sunt lungimile laturilor BC,

CA, respectiv AB. Demonstraţie. Vezi „Relaţia lui Van – Aubel”.

64 János Bolyai (1802-1860) – matematician român, de origine maghiară, contribuţii fundamentale în geometrie

A

B

C A'

2Z

3Z

C' B'

I Z

1Z

Fig. 184

Page 175: Barbu_TEOREME_FUNDAMENTALE(2008).pdf

182

5) Fie M un punct din planul unui triunghi ABC şi Z centrul antibisector al triunghiului

ABC. Atunci :2 2 2 3 3 3

22

( ).

( )

bcMA caMB abMC abc a b cMZ

ab bc ca ab bc ca

+ + + += −

+ + + +

Demonstraţie. Vezi „Relaţia lui Van – Aubel”. 6) Consecinţă: Fie O centrul cercului circumscris al triunghiului ABC şi Z centrul său

antibisector. Atunci: 3 3 3

2 22

( )

( )

abc a b cOZ R

ab bc ca

+ += −

+ +, unde R este raza cercului circumscris

triunghiului ABC. Demonstraţie. În relaţia demonstrată în aplicaţia precedentă considerăm M O≡ şi obţinem concluzia.

7) În triunghiul ABC fie ( ), ( ).M AB N AC∈ ∈ Dreapta MN trece prin centrul

antibisector Z al triunghiului ABC dacă şi numai dacă: 1 1 1

.MA NC

b MA c NA a⋅ + ⋅ =

Demonstraţie. Vezi „Relaţia lui Van – Aubel”. 8) Centrul antibisector Z, punctul lui Gergonne Γ şi punctul lui Nagel N sunt coliniare. Demonstraţie. Vezi „Punctul lui Nagel”. 9) Dacă Z este centrul antibisector al triunghiului ABC, iar x, y, z distanţele de la Z la

laturile BC, CA, respectiv AB atunci: 1 1

1 1 1 1 1 1,a ba h b h

x ya b c a b c

− −

− − − − − −= =

+ + + + şi

1

1 1 1cc h

za b c

− − −=

+ + (unde , ,

a b ch h h sunt lungimile înălţimilor triunghiului ABC).

Demonstraţie. Deoarece antibisectoarele sunt ceviene de rang (-1) atunci 1 1

[ ]

1 1 1 1 1 1

2 ABC aa A a h

xa b c a b c

− −

− − − − − −= =

+ + + + (vezi „Teorema lui Van-Aubel”). Analog se arată şi

celelalte două egalităţi. 10) Fie Z centrul antibisector al triunghiului ABC. Prin Z se duc paralelele la laturile triunghiului ABC: 1 4 6 3, ,M M AB M M BC 2 5M M AC ( 1 2 3 4, ; , ;M M BC M M AC∈ ∈

5 6, )M M AB∈ . Atunci, 1 2 3 4 5 6 .M M M M M M≡ ≡

Demonstraţie. Fie x distanţa de la Z la latura BC. Avem

11 2

1 1 1a

M M x a

a h a b c

− − −= =

+ + şi deci

1 2 1 1 1

1.M M

a b c− − −=

+ + Analog se

demonstrează că 3 4 5 6M M M M≡

1 1 1

1.

a b c− − −

= + +

A

B

C 1M

2Z

3Z 5M 4M

6M

Z

1Z

Fig. 185

3M

2M

x

ah

Page 176: Barbu_TEOREME_FUNDAMENTALE(2008).pdf

183

11) Fie AE izogonala antibisectoarei AD a triunghiului ABC, , ( ).E D BC∈ Atunci, 3

.BE c

EC b

=

Demonstraţie. Vezi „Drepte izogonale”. 12) Fie , ,

a b cZ Z Z proiecţiile centrului antibisector (Z) al triunghiului ABC pe laturile

BC, CA, respectiv AB. Atunci, 2 2 2

.a b cZZ ZZ ZZ

a b c− − −= =

Demonstraţie. Antibisectoarea este o ceviană de rang (-1) şi rezultă 2 2 2a b c

ZZ ZZ ZZ

a b c− − −= =

(vezi teorema (12) – Teorema lui Van – Aubel). 13) Fie 1 2,AZ BZ şi 3CZ antibisectoare în triunghiul ABC, Z centrul său antibisector,

1 2 3' ( ), ' ( ), ' ( )A AZ B BZ C CZ∈ ∈ ∈ astfel încât 1 2 3' , ' , ' .AA ZZ BB ZZ CC ZZ≡ ≡ ≡ Prin

punctele ', ', 'A B C ducem respectiv paralelele 1 2 3 4 5 6, ,M M M M M M la laturile BC, CA,

AB. Patrulaterul 1 2 3 4 5 6M M M M M M

este hexagon regulat. Demonstraţie. Fie x, y, z lungimile distanţelor de la Z la laturile BC, CA,

respectiv AB, iar , ,a b ch h h lungimile

înălţimilor triunghiului ABC (Fig. 186).

Avem: 1 1 2

1 1

'

a

ZZ M Mx AA

h AZ AZ a= = = şi

deoarece 1

1 1 1a

x a

h a b c

− − −=

+ + (conform

proprietăţii (9)) rezultă 1

1 21 1 1

,M M a

a a b c

− − −=

+ + adică

1 2 1 1 1

1.M M

a b c− − −=

+ + Analog se arată că 3 4 5 6 1 1 1

1.M M M M

a b c− − −

≡ = + + Din

asemănarea triunghiurilor ABC şi 3 4M BM rezultă: 1 1 1

21

4 '

BZa a b c

BM BB b

− − −

+ += = de

unde 1

4 1 1 1.

abBM

a b c

− − −=

+ + Analog se arată că

1

5 1 1 1,

acCM

a b c

− − −=

+ + de unde rezultă

că 1 1

4 5 1 1 1 1 1 1 1 1 1

1.

ab acM M a

a b c a b c a b c

− −

− − − − − − − − −= − − =

+ + + + + + Analog se obţine faptul că

1 6 2 3 1 1 1

1,M M M M

a b c− − −

≡ = + + deci hexagonul 1 2 3 4 5 6M M M M M M este regulat.

A

B

C

1M

2Z 3Z

5M 4M

6M Z

1Z

Fig. 186

3M

2M A'

B' C'

ah

x

Page 177: Barbu_TEOREME_FUNDAMENTALE(2008).pdf

184

I.46. Simediane. Punctul lui Lemoine65

„Matematica nu este un edificiu ancorat undeva într-o absolută soliditate, ci o construcţie aeriană care rezistă ca prin minune.., matematica reprezintă cea mai îndrăzneaţă şi mai neverosimilă aventură a spiritului.” – F. Gouseth Numim simediană a unui triunghi izogonala medianei. Dacǎ 'AA este bisectoarea

interioarǎ a unghiului BAC a triunghiului ABC, atunci simetrica medianei

aAM , ( )∈

aM BC faţǎ de bisectoarea 'AA este simediana corespunzǎtoare laturii BC .

Analog se definesc şi simedianele corespunzǎtoare laturilor CA şi AB. Teorema lui Grebe66

1) Simediana este locul geometric al punctelor a cǎror distanţe la laturile adiacente sunt proporţionale cu lungimile acestor laturi. Demonstraţie. Fie P un punct pe

aAK , 'P şi

''P proiecţiile lui P pe AB respectiv AC; a

M

mijlocul laturii BC , 'a

M , ''a

M proiecţiile lui

aM pe AB respectiv AC (Fig. 187). Evident,

' ' " ''⋅ = ⋅a a a a

M M PP M M PP (1). Cum

triunghiurile a

ABM şi a

ACM au aceeaşi arie

rezultă ' "⋅ = ⋅a a a a

M M AB M M AC (2). Din

relaţiile (1) şi (2) rezultǎ ' "

PP PP

AB AC= .

2) Dacǎ PQ ( ,P AB Q AC∈ ∈ ) este o antiparalelǎ la latura BC a triunghiului ABC ,

atunci simediana din A determinǎ pe antiparalelă douǎ segmente congruente. Demonstraţie. Fie

aM mijlocul laturii BC ,

aK

piciorul simedianei din A, = Ia

T PQ AK (Fig.

188). Din asemǎnarea triunghiurilor APT cu

aABM rezultǎ:

a a

AT PT

AM M C= , respectiv

a a

AT TQ

AM BM= , de unde ţinând cont cǎ

a aBM M C= , rezultǎ PT TQ= .

Observaţie. Rezultatul de mai sus poate fi reformulat astfel: locul geometric al mijloacelor antiparalelelor la una din laturile unui triunghi îl

reprezintǎ simediana ce pleacǎ din vârful opus acelei laturi ( Teorema lui Lhuilier67).

65 Emile Lemoine (1840-1912) – matematician francez, contribuţii importante în geometrie 66 Ernst Wilhelm Grebe (1804-1874) - matematician german, contribuţii în geometrie 67 Simon Lhuilier (1750-1840) – matematician elveţian, membru al Academiei din Berlin, contribuţii în geometrie

A

B C

aM A' aK

Fig. 187

P"

P '

'aM

"aM

P

A

B C

aM A' aK

Fig. 188

Q

P

T

Page 178: Barbu_TEOREME_FUNDAMENTALE(2008).pdf

185

3) Simediana corespunzǎtoare unei laturi împarte latura respectivǎ într-un raport egal cu pǎtratul raportului lungimilor celorlalte douǎ laturi ale triunghiului. Demonstraţie. Soluţia 1. Fie

aM mijlocul laturii BC a triunghiului ABC , 'A piciorul

bisectoarei din A pe latura BC , a

AK simediana din A , a

K BC∈ (Fig. 188). Avem :

[ ]

[ ]

[ ]

[ ]

22 2

2 2

sin sin

sin sin

⋅ ⋅ ⋅ ⋅ = = = ⋅ = ⋅ = ⋅ ⋅ ⋅ ⋅

a a

a a

ABK ACMa a a a a

a a a a aACK ABM

A AK B AB AK BAK AC AM BAKAB AB AB

K C A AC AK K AC AB AM M AC A ACAC AC

deci 2

a

a

K B AB

K C AC

=

.

Soluţia 2. Din teorema lui Steiner (vezi „Drepte izogonale”) rezultǎ 2

2⋅ =a a

a a

K B M B c

K C M C b,

cum a a

M B M C= , rezultǎ 2

a

a

K B c

K C b

=

.

Observaţie : Dacă

bK ,

cK sunt picioarele simedianelor corespunzǎtoare laturilor AC

respectiv AB atunci: 2

b

b

K C a

K A c

=

şi 2

c

c

K A b

K B a

=

.

4) Fie , ,

a b cm m m lungimile medianelor triunghiului ABC, iar , ,

a b cs s s lungimile

simedianelor corespunzătoare. Atunci: 2 2

2= ⋅

+a a

bcs m

b c,

2 2

2= ⋅

+b b

cas m

c a,

2 2

2= ⋅

+c c

abs m

a b.

Demonstraţie. Din 2

2a

a

K B c

K C b= , rezultǎ

2

2 2a

acBK

b c=

+ şi

2

2 2a

abK C

b c=

+. Din relaţia lui

Stewart aplicatǎ în triunghiul ABC rezultă:

2 2 2⋅ + ⋅ − ⋅ ⋅ = ⋅a a a a aAB K C AC BK BC BK CK AK BC ,adică 2 2

2 2 2 22 2 2

2( )( )

= + − +a

b cAK b c a

b c, de

unde 2 2

2= ⋅

+a a

bcs m

b c, unde am utilizat formula medianei. Prin permutǎri circulare se obţin

relaţiile ce dau lungimile celorlalte doua simediane : 2 2

2= ⋅

+b b

acs m

a c, respectiv

2 2

2= ⋅

+c c

abs m

a b .

5) Lungimea simedianei duse din A este mai mică sau egală decât lungimea medianei duse din acelaşi vârf.

Demonstraţie. Deoarece 2 2

2= ⋅

+a a

bcs m

b c şi

2 2

21

bc

b c≤

+, rezultă

a as m≤ , cu egalitate dacă

b c= .

Page 179: Barbu_TEOREME_FUNDAMENTALE(2008).pdf

186

6) Într-un triunghi simedianele sunt concurente. Demonstraţie. Dacǎ

aAK ,

bBK ,

cCK sunt simedianele triunghiului ABC (Fig. 189)

atunci 2 2 2

2 2 21⋅ ⋅ = ⋅ ⋅ =a b c

a b c

K B K C K A c a b

K C K A K B b c a şi din

reciproca teoremei lui Ceva rezultă cǎ simedianele sunt concurente. Punctul de concurenţǎ al simedianelor K, se numeşte punct simedian sau punctul lui Lemoine. 7) Centrul de greutate al unui triunghi şi punctul lui Lemoine al triunghiului sunt puncte izogonale. Demonstraţia este evidentă datoritǎ proprietǎţii precedente. 8) Dacă K este punctul lui Lemoine al triunghiului ABC şi

a b cK K K triunghiul său

cevian, atunci 2 2 2 2 2 2

2 2 2, , .

a b c

AK b c BK c a AK a b

KK KK KKa b c

+ + += = =

Demonstraţie. Din teorema lui Van-Aubel rezultă 2 2 2 2

2 2 2.b c

a b c

AK AKAK c b c b

KK K C K B a a a

+= + = + = Analog se arată şi celelalte două egalităţi.

9) Consecinţă: Dacă K este punctul lui Lemoine al triunghiului ABC, atunci

2 2 2 2

2 2 2 2 2 2

( ) ( ),a bb c s c a s

AK BKa b c a b c

+ += =

+ + + + şi

2 2

2 2 2

( ) ca b sCK

a b c

+=

+ +.

Demonstraţie. Din relaţia 2 2

2a

AK b c

KK a

+= rezultă

2 2

2 2 2a

AK b c

AK a b c

+=

+ +, de unde rezultă

concluzia. 10) Dacă K este punctul lui Lemoine al triunghiului ABC, atunci pentru orice punct M din planul triunghiului ABC este adevărată relaţia:

2 2 2

2 2 2 2 2 2 2 2 2.

a b cMK MA MB MC

a b c a b c a b c= + +

+ + + + + +

uuuur uuur uuur uuuur

Demonstraţie. Din proprietăţile precedente avem: 2 2

2a

AK b c

KK a

+= şi

2

2a

a

K B c

K C b= de unde

rezultă

2 2

2 2 22

2 2 2 2 2

2

( )

1

aa

b cMA MK

a MA b c MKaMKb c a b c

a

++ + +

= =+ + +

+

uuur uuuuuruuur uuuuur

uuuur, respectiv

2

2 22

2 2 2

21

a

cMB MC

b MB c MCbMKc b c

b

+ += =

++

uuur uuuuruuur uuuur

uuuuur de unde rezultă concluzia.

A

B C

aM A' aK

Fig. 189

bK

cK cM

bM

K G

Page 180: Barbu_TEOREME_FUNDAMENTALE(2008).pdf

187

11) Coordonatele baricentrice ale punctului lui Lemoine al unui triunghi ABC sunt: 2 2 2

2 2 2 2 2 2 2 2 2, ,

a b cK

a b c a b c a b c

+ + + + + + .

Demonstraţia rezultă din proprietatea precedentă. 12) Fie , ,A B Cz z z afixele vârfurilor triunghiului ABC. Afixul punctului lui Lemoine al

triunghiului ABC este egal cu 2 2 2

2 2 2 2 2 2 2 2 2K A B C

a b cz z z z

a b c a b c a b c= + +

+ + + + + +.

Demonstraţia rezultă din proprietatea (9). 13) Distanţele de la punctul lui Lemoine K al triunghiului ABC la laturile acestuia sunt proporţionale cu lungimile laturilor triunghiului. Demonstraţie. Dacǎ x, y şi z sunt lungimile distanţelor de la punctul K la laturile BC, CA,

respectiv AB atunci, ,x y y z

a b b c= = şi

z x

c a= de unde rezultă:

x y z

a b c= = .

14) Suma pǎtratelor distanţelor de la punctul lui Lemoine al triunghiului ABC la laturile acestuia este minimǎ. Demonstraţie. Fie , ,x y z distanţele de la un punct K la laturile BC , CA respectiv AB. Din

identitatea lui Lagrange rezultǎ: 2 2 2 2 2 2 2( )( ) ( )+ + + + − + + =x y z a b c ax by cz 2 2 2( ) ( ) ( )− + − + −bz cy cx az ay bx . Cum [ ]2

ABCax by cz A+ + = ⋅ rezultǎ cǎ suma

2 2 2x y z+ + este minimǎ cand membrul al doilea se anuleazǎ, adicǎ x y z

a b c= = ,deci când

K este punctul lui Lemoine al triunghiului ABC. 15) Triunghiul tangenţial al triunghiului ABC este triunghiul anticevian al punctului lui Lemoine al triunghiului ABC . Demonstraţie. Fie A B CT T T triunghiul tangenţial

al triunghiului ABC . Dreptele AAT , BBT , CCT

sunt concurente (vezi „Triunghiul tangenţial”) şi fie K punctul lor de concurenţǎ (Fig. 190). Arătăm cǎ K este punctul lui Lemoine al triunghiului ABC . Prin AT ducem antiparalela

PQ la BC , P AB∈ şi Q AC∈ . Din

( ) ( ) ( ) ( )A C Am PBT m ABT m ACB m BPT= = =

rezultǎ cǎ triunghiul ABT P este isoscel şi analog

triunghiul ACT Q este isoscel, deci A ABT T P= şi

A ACT T Q= . Cum A AT B T C= rezultǎ A AT P T Q=

şi conform proprietǎţii 2) rezultǎ cǎ punctul AT

se aflǎ pe simediana din A a triunghiului ABC . Analog se aratǎ cǎ BBT este simedianǎ, deci K este punctul lui Lemoine al triunghiului

ABC .

A

C

P Q

K

AT

BT CT

Fig. 190

B

Page 181: Barbu_TEOREME_FUNDAMENTALE(2008).pdf

188

Observaţie: Triunghiul ABC şi triunghiul sǎu tangenţial A B CT T T sunt omologice, centrul

de omologie fiind punctul lui Lemoine al triunghiului ABC iar axa de omologie este dreapta lui Lemoine (vezi „Dreapta lui Lemoine). Observaţie: Din aplicaţia precedentǎ rezultǎ un procedeu simplu de contrucţie a simedianelor: simediana din A uneşte acest punct cu punctul de întâlnire al tangentelor în B şi C la cercul circumscris triunghiului ABC.

16) Fie 1 2 3K K K triunghiul podar al punctului lui Lemoine

K. Medianele , ,a b cAM BM CM sunt perpendiculare pe

dreptele 2 3 3 1, ,K K K K respectiv 1 2 .K K

Demonstraţie. Fie aM mijlocul segmentului BC şi 'A

simetricul lui A faţǎ de aM . Dar aAM şi 1AK sunt

simediane , deci 1 ≡ aBAK M AC (Fig. 191). Din

patrulaterul inscriptibil 3 2KK AK rezultǎ 3 2 3K K A AKK≡ ,

de unde rezultǎ că 2 2 3( ) ( )am M AK m AK K+ =

1 3 3( ) ( ) 90m K AK m AKK+ = ° , deci 2 3' ⊥AA K K . Analog

se arată că 2 1bBM K K⊥ şi 1 3cCM K K⊥ .

Teorema lui Lemoine

17) Fie 1 2 3K K K triunghiul podar al punctului lui Lemoine K. Punctul K este centrul de

greutate al triunghiului 1 2 3K K K .

Demonstraţie. Fie aM mijlocul segmentului BC şi 'A simetricul lui A faţǎ de aM

(Fig. 191). Din ' CA AB rezultǎ 3' '≡ AA C K AA . Atunci

3 2 3 2( ' ) 90 ( ) ( )= °− = m AA C m AK K m KK K (1). Dar 3 2 3 '≡ ≡ K K K KAK A AC

(2), atunci din relaţiile (1) şi (2) rezultă cǎ triunghiurile 3 2KK K şi 'CA A sunt asemenea

(3). Fie 1 2 3 = IM K K K K . Patrulaterele 2 1KK CK şi 1 3KK BK fiind inscriptibile rezultǎ

MKC ACB≡ şi 3 '≡ ≡ MKK ABC BCA (4). Din relaţiile (3) şi (4) rezultă cǎ prin

asemǎnarea datǎ punctului 2 3M K K∈ îi corespunde punctul '∈aM AA . Cum aM este

mijlocul segmentului 'AA rezultǎ cǎ M este mijlocul segmentului 2 3K K , adicǎ aKK este

medianǎ în triunghiul 1 2 3K K K . Analog se aratǎ cǎ 2KK este medianǎ şi deci K este centrul

de greutate al triunghiului 1 2 3K K K .

18) Dintre toate triunghiurile înscrise într-un triunghi ABC , triunghiul podar al punctului lui Lemoine K are suma pătratelor laturilor minimǎ. Demonstraţie. Fie 1 2 3K K K triunghiul podar al punctului K în raport cu triunghiul ABC şi

x , y , z distanţele de la K la laturile BC , CA respectiv AC . Din proprietatea 9 rezultă cǎ

suma 2 2 2x y z+ + este minimǎ. Utilizând problema precedentǎ şi teorema medianei : 2 '2 '2 '23 2( )

2 4

+ − =

x b c a,

2 '2 '2 '23 2( )

2 4

+ − =

y a c b,

2 '2 '2 '23 2( )

2 4

+ − =

z b c c (unde am

notat cu 'a , 'b , 'c lungimile segmentelor 2 3K K , 1 3K K ,respectiv 1 2K K ). Sumând relaţiile

A

B C

1K

aM

A'

K

Fig. 191

2K 3K M

Page 182: Barbu_TEOREME_FUNDAMENTALE(2008).pdf

189

precedente rezultǎ 2 2 2

2 2 2' ' '3

x y za b c

+ ++ + = suma fiind minimǎ deoarece

2 2 2x y z+ + este minimǎ.

19) Triunghiul podar al punctului lui Lemoine şi triunghiul podar al centrului de greutate sunt înscrise în acelaşi cerc. Demonstraţie. Vezi „Triunghiul podar”. 20) Antiparalele duse prin punctul lui Lemoine la laturile triunghiului ABC sunt egale între ele. Demonstraţie.

Fie 1 2PP şi 1 2QQ antiparalelele duse prin punctul lui

Lemoine K la laturile BC respectiv AB. Atunci,

2 2 2 2KPQ KQ P≡ , deci 2 2≡KP KQ . Cum K se

aflǎ la mijlocul fiecǎrei antiparalele rezultǎ

1 2 1 2PP QQ= .

Consecinţe:

1) Patrulaterul 1 1 2 2PQ PQ este un dreptunghi cu

centrul în punctul lui Lemoine K. 2) 1 1PQ AC .

21) Construcţia punctului lui Lemoine. Fie 'A şi 'B punctele de intersecţie dintre medianele duse din A şi B ale triunghiului ABC şi cercul circumscris triunghiului ABC . Fie "A şi "B punctele de intersecţie dintre paralelele duse din 'A şi

'B la laturile BC , respectiv AC cu cercul circumscris triunghiului ABC . Vom arăta cǎ punctul lui Lemoine se aflǎ la intersecţia dreptelor "AA şi

"BB . Deoarece ' "A A BC rezultǎ ' " "≡ A A C A CB (1) (alterne interne). Dar

1( " ) ( " ) ( " )

2= = m A CB m A AB m A B (2) şi

1( " ') ( ' ) ( ' )

2= = m CA A m A AC m A C (3). Din

relaţiile (1), (2) şi (3) rezultǎ ( " ) ( ' )= m A AB m A AC , adicǎ dreptele 'AA şi

"AA sunt izogonale. Analog, se aratǎ cǎ 'BB şi "BB sunt izogonale, deci punctul lui Lemoine este dat de intersecţia dreptelor "AA şi "BB (Fig. 193). Teorema lui Schömilch

22) Dreptele care unesc mijloacele laturilor cu mijloacele înǎlţimilor respective se întâlnesc în punctul lui Lemoine al triunghiului. Demonstrăm mai întîi lema: Locul geometric al centrelor dreptunghiurilor avand o laturǎ paralelǎ cu una din laturile unui triunghi dat şi înscris în acel triunghi este o dreaptǎ ce uneşte mijlocul laturii şi mijlocul înǎlţimii perpendiculare pe laturǎ.

A

B C

K

2P

1Q

1P

Fig. 192

2Q

A

B C

K G

A' A"

B' B"

aM

bM

Fig. 193

Page 183: Barbu_TEOREME_FUNDAMENTALE(2008).pdf

190

Demonstraţie. Fie aAH dreptunghiul înscris în

triunghiul ABC şi = ∩L PR SQ , SR BC . Cand

SR se deplaseazǎ astfel încât tinde spre BC , centrul dreptunghiului tinde cǎtre mijlocul aM al

lui BC . Dar RQ se deplaseazǎ astfel încat tinde

cǎtre înǎlţimea aAH , atunci L tinde cǎtre

M ,mijlocul înǎlţimii. Cum variaţia uneia dintre cele douǎ laturi SR sau RQ implicǎ variaţia

celeilalte rezultǎ cǎ locul geometric este segmentul ( )aMM .

Demonstraţia teoremei. Dacǎ 1 2PP şi 1 2QQ sunt

douǎ antiparalele ce trec prin K (Fig. 194), atunci conform proprietǎţii precedente rezultǎ cǎ patrulaterul 1 1 2 2PQ PQ este un dreptunghi înscris în triunghiul ABC şi 2 1PQ AB , deci

conform lemei, centrul său (adicǎ punctul lui Lemoine K) aparţine segmentului ce uneşte mijlocul laturii AB şi mijlocul înǎlţimii din C . Analog se aratǎ cǎ K apartine şi celorlalte douǎ drepte ce unesc mijloacele laturilor cu mijloacele înǎlţimilor triunghiului. Observaţie: Dreptele ce unesc mijloacele laturilor ce mijloacele înǎlţimilor corespunzǎtoare se numesc dreptele lui Schwatt. 23) Punctul lui Lemoine şi centrul cercului circumscris unui triunghi ABC sunt douǎ puncte izotomice în raport cu triunghiul median al triunghiului ABC .

Demonstraţia rezultă din aplicaţia precedentǎ. 24) Punctul lui Lemoine al unui triunghi dreptunghic este mijlocul înǎlţimii corespunzǎtoare ipotenuzei. Demonstraţie. Proprietatea este o consecinţǎ a teoremei lui Schömilch. 25) În triunghiul ABC fie AD, ( )D BC∈ , bisectoarea

interioarǎ a unghiului BAC . Simediana din B a triunghiului ABD intersecteazǎ simediana din C a triunghiului ADC într-un punct P situat pe ( )AD .

Demonstraţie. Fie P şi 'P picioarele simedianelor duse din B respectiv C în triunghiurile ABD respectiv ACD .

Avem 2

AP AB

PD BD

=

şi 2

'

' =

AP AC

P D DC. Din teorema

bisectoarei rezultǎ BD AB

DC AC= , de unde

2 2BD AB

DC AC

=

,

relaţie echivalentǎ cu 2 2

AB AC

BD DC

=

, adicǎ '

'=

AP AP

PD P D, deci '≡P P .

A

K

Fig. 194

C B

1P 2P

1Q

2Q

A

B C

P

P '

D

Fig. 195

Page 184: Barbu_TEOREME_FUNDAMENTALE(2008).pdf

191

26) În triunghiul ABC fie AD ( ( ))∈D BC bisectoarea interioarǎ a unghiului BAC .

Simediana din C a triunghiului ADC intersectează AB în E şi simediana din B a triunghiului ABD intersecteazǎ AC în F. Dreapta EF trece prin punctul lui Lemoine al triunghiului ABC .

Demonstraţie. Conform aplicaţiei precente dreptele AD , BF şi CE sunt concurente într-

un punct P. Avem : BD c

DC b= , de unde

BD c

a b c=

+,

acBD

b c=

+ iar ,

2 2AP AB b c

PD BD a

+ = =

. Din teorema lui

Menelaus în triunghiul ABD şi transversala EC rezultǎ 2

( )= ⋅ =

+EB PD BC a

EA AP DC b b c. Analog se obţine relaţia

2

( )=

+FC a

FA c b c. Atunci

2 22 2 2EB FC a b a cb c aEA FA b c

++ = =

+

relaţie care aratǎ cǎ punctul lui Lemoine al triunghiului ABC aparţine dreptei EF (vezi „Relaţia lui Van - Aubel”).

27) Într-un triunghi ABC , punctul lui Lemoine (K) este coliniar cu izogonalele punctului lui Gergonne ( Γ ) şi centrului antibisector (Z). Demonstraţie. Vezi „Puncte izogonale”. 28) În triunghiul ascuţitunghic neisoscel ABC , fie înălţimea aAH , mediana aAM şi

simediana aAK , ( , , )a a aH M K BC∈ . Atunci, 2

2 2 2a a

a a

K M a

H K b c a=

+ −.

Demonstraţie. Fără a restrânge generalitatea, putem presupune că b c> (Fig. 197). Din

teorema lui Steiner avem 2

2a

a

K C b

K B c= , de unde

2

2 2a

a bK C

b c

⋅=

+. Atunci, a a a aK M K C M C= − =

2 2 2

2 2 2 2

( )

2 2( )

ab a a b c

b c b c

−− =

+ + (1). Deoarece cosaH C b C= ⋅ şi

2 2 2

cos2

a b cC

ab

+ −= , deci

2 2 2

2a

a b cH C

a

+ −= . Cum a a a aH K H C K C= − rezultă:

2 2 2 2 2

2 2

( )( )

2 ( )a a

b c b c aH K

a b c

− + −=

+

(2). Din relaţiile (1) şi (2) se obţine: 2

2 2 2a a

a a

K M a

H K b c a=

+ −.

29) Dacă 'A este simetricul lui A faţă de latura BC a triunghiului ascuţitunghic neisoscel ABC şi L este proiecţia ortocentrului H al triunghiului ABC pe mediana

aAM , iar aAK este simediană, atunci punctele 'A , aK , L sunt coliniare.

A

B C

F

P E

D

Fig. 196

C aM

A

c

aH

b

a

aK B

Fig. 197

Page 185: Barbu_TEOREME_FUNDAMENTALE(2008).pdf

192

Demonstraţie. Dacă L este proiecţia ortocentrului triunghiului ABC pe mediana aAM şi 1L

este simetricul lui L faţă de aM , atunci 1L aparţine cercului circumscris triunghiului ABC

(vezi „Ortocentrul unui triunghi”), deci 1a aM L M L≡ . Din puterea punctului M faţă de C -

cercul circumscris triunghiului ABC, avem 2

1 4a a

aAM M L⋅ = sau

2

4a a

aAM M L⋅ = (Fig.

198). Deoarece 2 2 2

2a

b c aAM AL

+ −⋅ = ( )a aAM M L AL− = , de unde

2 2 2

2

2( )

a

AL b c a

M L a

+ −= . Deoarece

' 1

' 2aA H

A A= , rezultă

'

'a a

a a

NM A HAL

LM H N A A⋅ ⋅ =

2 2 2 2

2 2 2 2

2( ) 11

2

b c a a

a b c a

+ −⋅ ⋅ =

+ − şi conform reciprocei teoremei lui Menelaus, rezultă că

punctele ', ,aA K L sunt coliniare.

30) În triunghiul ascuţitunghic neisoscel ABC , fie L proiecţia ortocentrului H al triunghiului mediana aAM , aH piciorul înălţimii duse din A pe BC , K punctul lui

Lemoine al triunghiului ABC, 'B AC BL= I , 'C AB CL= I şi ' 'aQ AM B C= I .

Atunci, punctele aH , K şi Q sunt coliniare.

Demonstraţie. Avem: '

'a

AQ B A

QM B C= (1). Teorema lui

Menelaus aplicată în triunghiul aAM C ne dă:

'1

'a

a

LMB A BC

B C LA BM⋅ ⋅ = , de unde

' 1

' 2a

B A LA

B C LM= ⋅ ,

egalitate care cu relaţia (1) ne dă : 2 2 2

2

2 2( )

a a

AL AQ b c a

LM QM a

⋅ + −= = (am utilizat

proprietatea precedentă), de unde 2 2 2

2a

AQ b c a

QM a

+ −= . Deoarece K este punctul de

intersecţie al simedianelor rezultă: 2 2 2

2 2a a

a a

H K b c a

H M b c

+ −=

+. Astfel avem:

2 2 2 2 2 2

2 2 2 2 2 21a a a

a a

QM H KAK a b c b c a

QA KN H M b c a a b c

+ + −⋅ ⋅ = ⋅ ⋅ =

+ − +, adică punctele aH , K şi Q

sunt coliniare.

Observaţie: Punctul de intersecţie al simedianelor K se găseşte la intersecţia lui aAK cu

aH Q .

Ha

Hb

A

Fig. 198

B

Hc

C

H

A' 1L

aM

L

aK

K

Q B'

C'

Page 186: Barbu_TEOREME_FUNDAMENTALE(2008).pdf

193

Consecinţe: a) aQN AH , b) QN BC⊥ .

Proprietăţile sunt evidente deoarece 2 2 2

2a

a a

H NAQ b c a

QM NM a

+ −= = .

31) Fie ABCD un patrulater inscriptibil şi P punctul de intersecţie a diagonalelor. Dacă AP este simediană în triunghiul ABD, atunci BD este simediană în triunghiul ABC. Demonstraţie. Din asemănarea triunghiurilor BPC şi APD

rezultă ,BP PC BC

AP PD AD= = deci

2

(1).BP PC BC

AP PD AD

⋅ = ⋅

Deoarece AP este simediană în triunghiul ABD rezultă 2

(2).BP AB

PD AD

=

Din relaţia (1) şi (2) rezultă 2

,PC BC

PA AB

=

deci BP este simediană în triunghiul ABC. Observaţii: 1) Cercurile lui Apollonius intersectează cercul circumscris după simedianele triunghiului. 2) Triunghiul tangenţial este omologic cu triunghiul dat centru de omologie fiind punctul lui Lemoine, iar axă de omologie este dreapta lui Lemoine (dreapta lui Lemoine este deci polara triliniară a punctului lui Lemoine). 32) Fie cba ,, lungimile laturilor unui triunghi neisoscel ABC . Dacă cercul ce trece prin picioarele simedianelor triunghiului ABC este tangent unei laturi a acestuia atunci

cantităţile 222222 ,, baaccb +++ - considerate într-o anumită ordine- sunt termenii consecutivi ai unei progresii geometrice. Demonstraţie. Fie că cercul circumscris triunghiului simedian a b cK K K este

tangent laturii BC în aK şi

intersectează a doua oară laturile CA şi AB în M respectiv N . Avem:

2 2 2

2 2 2, ,a b c

a b c

BK CK AKc a b

K C K A K Bb c a= = = , de

unde 2

2 2a

abK C

b c=

+,

2

2 2a

acK B

b c=

+ şi

relaţii omoloage pentru , ,b b cCK K A AK

şi cK B . Din puterea unui punct faţă de

un cerc rezultă 2 2,

c a b aBN BK BK CM CK K C⋅ = ⋅ = iar

din NBABAN −= şi MCACAM −= avem: 4 2 2 4 2 2

2 2 2

( )

( )

c c b c c c aAN

b c

+ + −=

+ şi

4 2 2 4 2 2

2 2

( )b b b c c a bAM

b c

+ + −=

+ (Fig. 200).

Din b cAK AM AK AN⋅ = ⋅ rezultă: 2 2 2 2 2 2 2 2 2( ) ( )( )( )b c b c b a a c+ = − + + şi cum

triunghiul ABC nu este isoscel, avem: 2 2 2 2 2 2 2( ) ( )( )b c b a a c+ = + + .

A

B C

P D

Fig. 199

A

B

C

aK

bK cK

K

N M

Fig. 200

Page 187: Barbu_TEOREME_FUNDAMENTALE(2008).pdf

194

33) Punctul lui Lemoine al triunghiului anticomplementar al unui triunghi ABC coincide cu retrocentrul triunghiului ABC . Demonstraţie. Vezi „ Triunghiul anticomplementar”. 34) Fie a b cH H H triunghiul ortic al triunghiului ABC . Simetricele 1 2 3, ,K K K în raport

cu punctul lui Lemoine al triunghiului ABC ale proiecţiilor lui K pe laturile triunghiului ABC sunt punctele lui Lemoine ale triunghiurilor ,b cAH H c aBH H ,

respectiv a bCH H . Demonstraţie. Fie 'A mijlocul segmentului aAH ,

aM mijlocul laturii BC, aK

piciorul simedianei din A, '1K piciorul perpendicularei

din K pe BC (Fig. 201). Din teorema lui Schömilch rezultă că ' aK A M∈ , iar

cum '1 1 a

K K AH , rezultă că

punctul 1K aparţine

medianei aAM . Simediana

aAK trece prin punctul ''A

- mijlocul antiparalelei

b cH H , iar aAM este simediană în triunghiul b cAH H şi fie ' a a b c

K AM H H= ∩ .

Deoarece '' ,c a c bA AH M AC AH H ACB≡ ≡ rezultă că '( '' ) ( '' ) ( '')a c

m AA K m A AB m AH A= + = '( ) ( ) ( )a a a a am M AC m ACM m K M K+ = ,

deci ' '''a a a a

AA K K M K≡ , relaţie ce arată că patrulaterul '''a a a

A K M K este inscriptibil.

Întrucât ''a b cM A H H⊥ rezultă 'a a

K K BC⊥ , deci ' '1 1a a

K K K K ; astfel punctele K şi 1K

împart în acelaşi raport simedianele aAK şi 'a

AK ale triunghiurilor asemenea ABC ,

b cAH H , deci 1K este punctul lui Lemoine ale triunghiului b cAH H . Analog, se arată că

punctele 2K şi 3K sunt punctele lui Lemoine ale triunghiurilor c aBH H , respectiv a bCH H .

35) Fie 1 1 1, ,A B C simetricele vârfurilor , ,A B C ale unui triunghi ABC în raport cu

punctele , ,C A respectiv B. Tangentele duse în , ,A B C la cercurile circumscrise triunghiurilor 1 1, ,ABC BC A respectiv 1CA B sunt concurente în punctul lui Lemoine al

triunghiului ABC . Demonstraţie. Fie M punctul diametral opus lui A în cercul circumscris triunghiului 1ABC ,

D un punct oarecare pe tangenta în A la cercul circumscris triunghiului 1ABC , E şi F

proiecţiile lui D pe dreptele AC respectiv AB (Fig. 202). Deoarece 1 ,B AM ADF≡

1AB M AFD≡ rezultă că triunghiurile 1AB M şi DFA sunt asemenea, deci 1ABAM

AD DF=

(1). Deoarece ,ACM DEA≡ AMC DAE≡ rezultă că triunghiurile MAC şi AED sunt

A

aK B C

aM

H

aH

bH

cH

Fig. 201

1K

K

'aK

'1K

A' A"

Page 188: Barbu_TEOREME_FUNDAMENTALE(2008).pdf

195

asemenea, de unde AM AC

AD DE= (2).

Din relaţiile (1) şi (2) rezultă

1AB DF

AC DE= , adică

AB DF

AC DE= ,

relaţie ce arată că punctul D aparţine simedianei vârfului A al triunghiului ABC . Analog, se arată că tangentele în B şi C la cercurile circumscrise triunghiurilor 1BC A ,

respectiv 1CA B sunt simedianele

vârfurilor B respectiv C, adică tangentele sunt concurente în punctul lui Lemoine al triunghiului ABC .

36) Dreptele care unesc vârfurile unui triunghi ABC cu mijloacele corespunzătoare ale laturilor triunghiului său ortic, sunt concurente în punctul lui Lemoine al triunghiului ABC .

Demonstraţie. Fie 1 2 3, ,M M M mijloacele segmentelor

, ,b c c aH H H H respectiv a bH H (Fig. 203). Deoarece

b cH H este o antiparalelă la latura BC rezultă că 1M

aparţine simedianei din A a triunghiului ABC (vezi proprietatea (2)). Analog, punctele 2M şi 3M aparţin

simedianelor vârfurilor B respectiv C, deci dreptele

1 2 3, ,AM BM CM sunt concurente în punctul lui Lemoine

al triunghiului ABC .

Observaţie: Punctul lui Lemoine al triunghiului ABC este centrul de omologie dintre triunghiul ABC şi triunghiul median al triunghiului ortic corespunzător triunghiului ABC .

37) Paralelele la laturile unui triunghi ABC duse prin punctul lui Lemoine al triunghiului ABC intersectează laturile triunghiului său ortic în puncte izotomice. Demonstraţie. Deoarece b cH H este antiparalelă laturii

BC rezultă că mijlocul 1M al segmentului b cH H

aparţine simedianei din A, deci 1AM este mediană în

triunghiul b cAH H , atunci paralelele duse din K, punct al

medianei 1AM la laturile ,b cAH AH intersectează latura

b cH H în punctele 1A şi 2A , egal depărtate de punctul

1M , adică punctele 1A şi 2A sunt izotomice.

A

B C

D

E

F

M

1A

1B

1C

Fig. 202

Ha

Hb

A

Fig. 204

B

Hc

C

K

1M 1A

2A

Ha

Hb

A

Fig. 203

B

Hc

C

H K

1M

3M 2M

Page 189: Barbu_TEOREME_FUNDAMENTALE(2008).pdf

196

38) Fie K punctul lui Lemoine al triunghiului ABC , O centrul cercului circumscris triunghiului ABC, R raza cercului circumscris triunghiului ABC . Atunci

2 2 22 2

2 2 2 2

3

( )

a b cOK R

a b c= −

+ + ( , ,a b c sunt lungimile laturilor , ,AB AC BC ).

Demonstraţie. În relaţia: 2 2 2 2 2 2 2

22 2 2 2 2 2 2

3

( )

a MA bMB cMC a b cMK

a b c a b c

+ += −

+ + + + valabilă pentru

orice punct din planul triunghiului ABC ( vezi „Relaţia lui Van – Aubel”), pentru M O≡ rezultă concluzia. 39) Fie ,M N puncte pe laturile ,AB AC ale triunghiului ABC . Dreapta MN trece prin

punctul lui Lemoine al triunghiului ABC dacă şi numai dacă 2 2 2MB NCb c a

MA NA⋅ + ⋅ = ,

unde a, b, c sunt lungimile laturilor BC, CA, respectiv AB. Demonstraţie. Vezi „Relaţia lui Van-Aubel”. 40) Triunghiul circumpedal al punctului lui Lemoine al unui triunghi ABC are acelaşi punct simedian ca şi triunghiul ABC . Demonstaţie. Vezi „Triunghiuri cosimediane”. 41) Dreapta care uneşte izotomicul punctului lui Lemoine al unui triunghi ABC cu ortocentrul (H) triunghiului ABC este paralelă cu dreapta care uneşte punctul lui Lemoine (K) cu centrul cercului circumscris (O) al triunghiului ABC. Demonstraţie. Fie ' ' 'A B C triunghiul anticomplementar al triunghiului ABC şi CBA TTT

triunghiul tangenţial corespunzător triunghiului ' ' '.A B C Atunci, ' ' ' ',∩ ∩ =A B CT A T B T C K 'K fiind punctul lui Lemoine al triunghiului ' ' '.A B C

Deoarece triunghiurile ABC şi ' ' 'A B C sunt omotetice, centrul de omotetie fiind centrul de greutate (G) al triunghiului ABC şi raportul de omotetie fiind egal cu 2, rezultă

' (1)K H KO şi H este centrul cercului circumscris triunghiului ' ' '.A B C Fie

'= ∩BX B T CA şi ' , ' , '= ∩ = ∩ = ∩CY C T AB X BK CA Y CK AB şi

1 .K BX CY= ∩

Observaţie: Deoarece punctul lui Lemoine al unui triunghi şi centrul cercului circumscris al triunghiului sunt puncte izotomice în raport cu triunghiul median al triunghiului dat, rezultă că punctele 'K şi H sunt puncte izotomice în raport cu triunghiul ABC. Fig. 205

B'

A' C'

A

B

AT

BT

CT

C K '

X X'

O

G

H

K

K '

Page 190: Barbu_TEOREME_FUNDAMENTALE(2008).pdf

197

Deoarece ' 'B K este simediană rezultă 2 2

' ',

' '

CX B C AB AX

XA B A BC X C

= = =

deci punctele X

şi 'X sunt izotomice faţă de mijlocul segmentului AC. Analog, punctele Y şi 'Y sunt izotomice faţă de mijlocul segmentului AB, de unde rezultă că 1K este punctul izotomic al

punctului lui Lemoine (K). Fie Z XY BC= ∩ şi ' .= ∩B CZ T T BC Din teorema lui

Menelaus aplicată în triunghiurile ABC şi BCH tăiate de transversalele XYZ respectiv

'B CT T Z rezultă 2

(2)ZC CX AY AB

ZB AX BY BC

= ⋅ =

şi '

(3).'

= ⋅C B

C B

T C HTZ C

Z B T H T BDin asemanarea

triunghiurilor dreptunghice ' CCA T şi ' CA HT rezultă ' (4)

' '=C

C

T C CA

A T A H iar din asemănarea

triunghiurilor dreptunghice 'CA H şi ' CA HT rezultă '

(5).' '

=C

C

T H A H

A T CA Din relaţiile (4) şi

(5) rezultă 2

' (6).

' =

C

C

T C CA

T H A H Analog, din asemănarea triunghiurilor dreptunghice

' , 'B BBA T A HT şi ' , ' BBA H A HT rezultă 2

' (7).

' =

B

B

T H A H

T B BA Din relaţiile (3), (6) şi (7)

rezultă 2 2

' ' (8).

' '

Z C CA AB

Z B BA AC

= =

Din relaţiile (2) şi (8) rezultă '

,'

ZC Z C

ZB Z B= adică

punctele Z şi 'Z coincid. Atunci, din teorema lui Desargues rezultă că dreptele BX, CY şi 'HK sunt concurente şi punctele , 'H K şi 1K sunt coliniare. Conform relaţiei (1) rezultă

atunci că 1 .HK KO

42) Pe laturile unui triunghi ABC, se construiesc în exterior pătrate. Laturile acestor pătrate opuse laturilor triunghiului formează un triunghi ' ' 'A B C omotetic cu ABC, centrul de omotetie fiind punctul lui Lemoine al triunghiului ABC. Demonstraţie. Vezi „Triunghiurile lui Vecten”. 43) Axa radicală a unui cerc Apollonius corespunzător unui vârf al triunghiului ABC şi a cercului circumscris triunghiului ABC este simediana corespunzătoare vârfului comun celor două cercuri. Demonstraţie. Vezi „Cercurile lui Apollonius”. 44) Punctul lui Lemoine are puteri egale faţă de cercurile lui Apollonius. Demonstraţie. Vezi „Cercurile lui Apollonius”.

Page 191: Barbu_TEOREME_FUNDAMENTALE(2008).pdf

198

I.47. Dreapta lui Housel „Ştiu că nu ştiu nimic” - Socrate68

Dreapta care uneşte centrul cercului înscris (I) într-un triunghi ABC cu centrul cercului înscris ( mI ) în triunghiul său median a b cM M M se numeşte dreapta lui Housel (h). În triunghiul ABC sunt adevărate relaţiile:

, ,a m b m c mAI M I BI M I CI M I .

Demonstraţie: Deoarece patrulaterul c a bAM M M

este paralelogram rezultă b c b a cM AM M M M≡ , deci

1 1( ) ( )

2 2b c b a cm M AM m M M M= sau

( ) ( )c m a bm IAM m I M M= şi cum c a bM A M M

rezultă .a mAI M I Analog se arată că b mBI M I şi

c mCI M I .

I.48. Simediana exterioară

„Dacă geometria doreşte să devină o adevărată ştiinţă deductivă, este esenţial ca modul prin care sunt obţinute deducţiile să fie complet independent de semnificaţia conceptelor geometrice ca şi de figuri; tot ceea ce este necesar sunt relaţiile între conceptele geometrice afirmate prin propoziţii şi definiţii” - Moritz Pasch69

Se numeşte simediană exterioară a unui vârf al triunghiului ABC ,locul geometric al punctelor exterioare triunghiului ale căror distanţă la laturile adiacente ale triunghiului sunt proporţionale cu lungimile acestor laturi. 1) Simediana exterioară a vârfului A este tangentă în A la cercul circumscris triunghiului ABC .

Demonstraţie. Fie M un punct pe tangenta în A la cercul circumscris triunghiului ABC , iar

'M şi "M proiecţiile lui M pe laturile AC respectiv AB (Fig. 207). Avem

'M AM ABC≡ şi "MAM ACB≡ . Din triunghiurile dreptunghice 'MAM şi

"MAM rezultă ' sin=MM AM B şi " sinMM AM C= , de unde ' sin

" sin

MM B b

MM C c= = .

68 Socrate (470 -399 î.H.) – filosof al Greciei antice 69 Moritz Pasch (1843-1930) – matematician german, profesor universitar la Gissen, contribuţii importante în geometrie

A

B C aM

bM cM

I

Fig. 206

mI

h

M

M"

M' A

B

O

a

c b

C

Fig. 207

Page 192: Barbu_TEOREME_FUNDAMENTALE(2008).pdf

199

2) Două simediane exterioare şi o simediană interioară ale unui triunghi sunt concurente. Demonstraţie. Fie AT punctul de intersecţie al simedianelor exterioare duse în B, respectiv

C. Prin AT ducem antiparalela PQ la BC , ,∈ ∈P AB Q AC (Fig. 208). Din

1( ) ( )

2Am PBT m AB= = ( ) ( )A

m ACB m BPT= rezultă că triunghiul A

BT P este isoscel,

deci =A A

PT BT ; analog, triunghiul ACT Q este isoscel cu =A AT C T Q , de unde rezultă că

=A A

PT T Q , adică AT aparţine simedianei din A.

Observaţii: 1) O simediană interioară trece prin punctul de întâlnire al tangentelor la cercul circumscris, duse prin celelalte două vârfuri ale triunghiului. 2) Punctul

AT aparţine cercului circumscris triunghiului BOC , deoarece patrulaterul

AOBT C este inscriptibil.

3) Dacă 'A este piciorul simedianei exterioare pe dreapta BC , atunci 2

''

=

A B c

A C b.

Demonstraţie. Triunghiurile 'A AB şi 'A AC (Fig. 209) sunt asemenea deoarece au

unghiul 'A comun şi ' 'A AB ACA≡ , deci ' '

' '

A B AB A A

A C AC A C= = , de unde rezultă

2 2''

A B AB c

A C AC b

= =

.

A

B C

AT

O

P

Q

Fig. 208

A

B C

A'

O

Fig. 209

Page 193: Barbu_TEOREME_FUNDAMENTALE(2008).pdf

200

Observaţii:

1) Dacă ', 'B C sunt punctele de intersecţie dintre simedianele exterioare ale vârfurilor

,B C ale triunghiului ABC , atunci 2

''

=

B C a

B A cşi

2''

=

C A b

C B a.

2) Punctele ', ', 'A B C aparţin dreptei lui Lemoine a triunghiului ABC .

I.49. Cercuri exînscrise

„Natura vorbeşte în limbajul matematicii: literele acestei limbi sunt cercuri, triunghiuri şi alte figuri geometrice.” –Galileo Galilei70

1) Două bisectoare exterioare şi una interioară, trecând fiecare prin câte un vârf al triunghiului sunt concurente. Demonstraţie. Fie

aI este punctul de concurenţă dintre bisectoarele exterioare ale

unghiurilor B şi C, atunci punctul aI este egal depărtat de laturile BC şi AB, respectiv de

laturile BC şi AC, deci el este egal depărtat şi de laturile AB şi AC, adică aI aparţine

bisectoarei interioare a unghiului A.

70Galileo Galilei (1564-1642) – matematician, fizician şi astronom italian, profesor la Universitatea din Padova,

contribuţii remarcabile în fizică şi astronomie

A

B

C

Ia

Ib

Ic

Fig. 210

aD

bE cF

I

aC

cC

bC

Page 194: Barbu_TEOREME_FUNDAMENTALE(2008).pdf

201

Observaţie: Punctul aI fiind situat la aceeaşi distanţă – notată cu

ar - faţă de laturile

triunghiului ABC este centrul unui cerc tangent exterior laturii BC şi prelungirilor laturilor AB şi AC. Un cerc care este tangent la o latură a unui triunghi şi la prelungirile celorlalte două laturi ale triunghiului se numeşte cerc exînscris al triunghiului. Evident, un triunghi are trei cercuri exînscrise

Observaţie: Cercurile exînscrise şi cercul înscris corespunzătoare unui triunghi se mai numesc cercuri tritangente. Notăm cu A-exînscris, cercul exînscris tangent laturii BC a triunghiului ABC; analog se notează cu B-exînscris şi C-exînscris celelalte două cercuri exînscrise; , ,

a b cI I I sunt centrele cercurilor exînscrise corespunzătoare şi notăm cu

, ,a br r respectiv

cr razele corespunzătoare cercurilor exînscrise (Fig. 210). Triunghiul

a b cI I I se numeşte triunghiul antisuplementar corespunzător triunghiului ABC.

2) Consecinţă: Vârfurile triunghiului ABC aparţin laturilor triunghiului antisuplementar. 3) Distanţele la laturile triunghiului ABC ale punctului

aI sunt egale cu

ar .

4) Distanţele de la punctul

aI la vârfurile triunghiului ABC sunt egale cu

,sin

2

a

a

rAI

A= , .

cos cos2 2

a a

a a

r rBI CI

B C= =

Demonstraţie. Fie , ,a b c

D D D proiecţiile lui aI pe laturile BC, CA, respectiv AB. Din

triunghiul dreptunghicc a

D I A rezultă sin2

=a a

Ar AI , iar din triunghiurile dreptunghice:

a cBI D şi

a bCI D avem sin( / 2 / 2) cos

2= − =a a a

Br BI B BIπ şi cos

2=a a

Cr CI .

5) Distanţele de la centrele cercurilor exînscrise respectiv la vârfurile triunghiului ABC

sunt egale cu: cos , cos , cos .2 2 2a b c

A B CAI p BI p CI p= ⋅ = ⋅ = ⋅

Demonstraţie. Din triunghiul dreptunghic a b

AI D rezultă cos2

b

a a

ADA p

AI AI= = , de unde

rezultă concluzia.

6) Triunghiul ABC este triunghiul ortic al triunghiului a b cI I I .

Demonstraţie. Evident ( ) ( ) ( ) 90 ,c a c a

m I AI m I AB m BAI= + = ° deci bisectoarea a

AI

este înălţime în triunghiul a b cI I I . Analog,

b a cBI I I⊥ şi ,

c a bCI I I⊥ deci ABC este

triunghiul ortic al triunghiului a b cI I I .

Consecinţe: 7) Centrul cercului înscris în triunghiul ABC este ortocentrul triunghiului

a b cI I I .

8) Înălţimile unui triunghi sunt bisectoarele triunghiului ortic al triunghiului dat.

Page 195: Barbu_TEOREME_FUNDAMENTALE(2008).pdf

202

9) Fie , ,a b c

D D D ; , ,a b cE E E ; , ,

a b cF F F punctele de contact dintre cercurile A – exînscris,

B – exînscris, respectiv C – exînscris cu dreptele BC, CA şi AB. Atunci, ,c b

AD AD p= =

a cBD BD p c= = − şi

a bCD CD p b= = − , unde p este semiperimetrul triunghiului

ABC.

Demonstraţie.

Avem: ,

c c cAD AB BD c BD= + = +

b b bAD AC CD b CD= + = + (1) (Fig. 211). Sumând

relaţiile precedente şi ţinând cont că , ,b c a c a b

AD AD BD BD CD CD= = = rezultă

2 2 ,c

AD b c a p= + + = adică .c

AD p= Analog se arată că a c

BE BE p= = şi

.a b

CF CF p= = Din relaţiile (1) rezultă c

BD p c= − şi .b

CD p b= −

Observaţie: Analog

a bCE CE p a= = − şi .

c bAE AE p c= = −

10) Distanţele dintre punctele de contact aflate în prelungirea aceeaşi laturi determinate de câte două cercuri exînscrise sunt egale cu , ,a b b c+ + respectiv .c a+ Demonstraţie.

a a a aE F E C CB BF p a a p a= + + = − + + − = 2 .p a b c− = + Analog,

b bD F a c= + şi .

c cE D a b= +

11) Distanţa între punctele interioare

aC şi

aD este egală cu diferenţa celorlalte laturi.

Demonstraţie. Avem: ( ) ( ) .a a a a

D C CD CC p b p c c b= − = − − − = − Analog se arată că

b bE C a c= − şi

c cF C a b= −

12) Distanţa de la punctul de contact

aC al cercului înscris la punctele exterioare

aE şi

aF sunt respectiv egale cu b şi c.

Demonstraţie. Avem, ( ) ( )a a a a

C E C C CE p c p a b= + = − − − = şi analog .a a

C F c=

A

B C

aI

bI cI

aF

bF

cF

aD

bD

cD

aE

bE

cE

ar

Fig. 211

τ

Page 196: Barbu_TEOREME_FUNDAMENTALE(2008).pdf

203

Teorema lui Nagel

13) Perpendicularele pe laturile unui triunghi duse din centrele cercurilor exînscrise sunt concurente. Demonstraţie. Soluţia 1. Deoarece perpendicularele duse din vârfurile unui triunghi ABC pe laturile triunghiului ortic corespunzător sunt concurente în centrul cercului circumscris triunghiului ABC, atunci, utilizând proprietatea 3) rezultă că perpendicularele duse din centrele cercurilor exînscrise , ,

a b cI I I pe laturile BC, CA respectiv AB sunt concurente în

centrul cercului circumscris triunghiului a b cI I I .

Soluţia 2. Triunghiul ABC este triunghiul ortic al triunghiuluia b cI I I (Fig. 211).

Fie , ,a b c

D E F proiecţiile punctelor ,a bI I , respectiv

cI pe laturile BC, AC, respectiv AB.

Atunci, ,a

BD p c= − ,a

CD p b= − ,c

AF p b= − ,c

BF p a= − ,b

AE p c= −b

CE p a= − ,

de unde rezultă că 2 2 2a b c

BD CE AF+ + = 2 2 2 2 2 2( ) ( ) ( )b c a

p c p a p b AE BF CD− + − + − = + +

şi din teorema lui Carnot rezultă că dreptele ,a a b bI D I E şi

c cI F sunt concurente.

14) Dacă O este centrul cercului circumscris triunghiului ABC, R şi

ar razele cercului

circumscris, respectiv A-exînscris în triunghiul ABC, atunci: 2 2 2 .a a

OI R Rr= +

Demonstraţie. Fie "A cel de-al doilea punct în care dreapta a

AI intersectează cercul

circumscris triunghiului ABC. Utilizând puterea punctului aI faţă de cercul circumscris

triunghiului ABC obţinem: 2 2 "a a a

OI R AI A I− = ⋅ (1). În triunghiul ,a c

AI A sin2

a

a

rA

AI=

sau sin

2

a

a

rAI

A= (2) iar în triunghiul "ABA din teorema sinusurilor rezultă

"2

sin2

BAR

A= ,

adică " 2 sin "2 a

ABA R A I= = (3). Din relaţiile (1), (2) şi (3) rezultă 2 2 2 ,

a aOI R Rr− = de

unde 2 2 2 .a a

OI R Rr= +

Observaţie: 2 2 2a a

OI R Rr= + este relaţia lui Euler. 15) Patrulaterele ,

a bBICI AICI şi

cAIBI sunt inscriptibile.

Demonstraţie. Deoarece ( ) ( ) 90a a

m I BI m I CI= = ° rezultă

( ) ( ) 180 ,a a

m I BI m I CI+ = ° adică patrulaterul a

BICI este inscriptibil. Analog, se arată

patrulaterele b

AICI şi c

AIBI sunt inscriptibile.

16) Centrul cercului circumscris patrulaterului

aBICI este mijlocul segmentului .

aII

Demonstraţie. Evident, deoarece ( ) 90a

m IBI = ° rezultă că a

II este diametru în cercul

circumscris patrulaterului .a

BICI

Teorema lui Beltrami

17) Mijlocul segmentului a

II aparţine cercului circumscris triunghiului ABC.

Page 197: Barbu_TEOREME_FUNDAMENTALE(2008).pdf

204

Demonstraţie. Fie "A cel de-al doilea punct de intersecţie dintre a

II şi cercul circumscris

triunghiului ABC (Fig. 212). Cum 1 1

( " ) ( ) ( ) ( ) ( )2 2

m A IB m IAB m IBA m A m B= + = +

şi 1 1

( ") ( ) ( ") ( ) ( )2 2

m IBA m IBC m CBA m B m A= + = + rezultă că triunghiul "IA B este

isoscel, deci " " .A I A B= Analog, " "IA A C= , adică "A este centrul cercului circumscris patrulaterului .

aBICI

18) Unghiul

aBI C are măsura egală cu

190 ( ).

2m BAC° −

Demonstraţie. Deoarece 1

( ) 90 ( )2

m BIC m A= °+

şi patrulaterul a

BICI este inscriptibil rezultă

1( ) 180 ( ) 90 ( ).

2am BI C m BIC m A= °− = °−

Observaţie: Analog, 1

( ) 90 ( )2bm CI A m ABC= °− şi

1( ) 90 ( ).

2cm AI B m ACB= °−

19) Axa radicală a cercurilor înscris în triunghiul ABC şi A – exînscris corespunzător triunghiului ABC este bisectoarea exterioară a vârfului

aM a triunghiului median al

triunghiului ABC. Demonstraţie. Deoarece

a aF D este tangenta comună (diferită de laturile triunghiului)

cercurilor înscris şi A – exînscris, rezultă că axa radicală trece prin mijlocul segmentului

a aF D , adică prin

aM mijlocul laturii BC. Cum axa radicală este perpendiculară pe linia

centrelor a

II rezultă că este perpendiculară şi pe bisectoarea interioară a mijlocului

c a bM M M , adică axa radicală este bisectoarea exterioară a vârfului

aM a triunghiului

median c a b

M M M .

Consecinţă: Centrele radicale ale cercurilor înscris, A – exînscris, B – exînscris şi C – exînscris sunt centrele cercurilor înscrise şi exînscrise în triunghiul median al triunghiului ABC. 20) Axa radicală dintre cercurile B – exînscris şi C – exînscris ale triunghiului ABC este bisectoarea interioară a vârfului

aM a triunghiului medial al triunghiului ABC.

Demonstraţie. Deoarece a aE F este tangenta exterioară comună cercurilor B – exînscris şi C

– exînscris, rezultă că axa radicală a acestor două cercuri trece prin mijlocul segmentului

a aE F , adică în punctul

aM şi este perpendiculară pe linia centrelor ,

b cI I deci axa radicală

este paralelă cu bisectoarea unghiului A, adică este bisectoarea unghiului .c a b

M M M

A

B C

aI

Fig. 212

I

A"

Page 198: Barbu_TEOREME_FUNDAMENTALE(2008).pdf

205

Observaţie: Analog, axele radicale dintre perechile de cercuri (C – exînscris, A - exînscris) şi (A – exînscris, B - exînscris) sunt bisectoarele interioare ale vârfurilor

bM , respectiv

cM ale triunghiului median.

21) Cercurile circumscrise triunghiurilor ,

a b b cII I II I şi

c aII I sunt congruente.

Demonstraţie. Deorece I este ortocentrul triunghiului a b cI I I (vezi „Triunghiul

antisuplementar”) rezultă concluzia (vezi „Ortocentrul unui triunghi”- th. (25) ). 22) Centrul radical al cercurilor exînscrise este centrul cercului înscris în triunghiul median. Demonstraţia rezultă din proprietatea precedentă. 23) Distanţele dintre centrele cercurilor exînscrise şi centrul cercului înscris într-un triunghi ABC sunt egale cu:

4 sin , 4 sin , 4 sin2 2 2cos cos cos

2 2 2

a b c

a A b B c CII R II R II R

A B C= = = = = = .

Demonstraţie. Din teorema (16) rezultă că 2 " 2 " 2 2 sin 4 sin .2 2a

A AII IA BA R R= = = ⋅ =

Dar, sin

4 sin 22 cos cos

2 2

A A aR R

A A= = , de unde rezultă concluzia.

24) Distanţele dintre centrele cercurilor exînscrise într-un triunghi ABC sunt egale cu:

4 cos , 4 cos , 4 cos2 2 2b c c a a b

A B CI I R I I R I I R= = = .

Demonstraţie. Deoarece măsurile unghiurilor triunghiului antisuplementar a b cI I I sunt

egale cu: 1

90 ( ),2m A° −

190 ( ),

2m B° −

190 ( )

2m C° − şi raza cercului circumscris

acestui triunghi este 2R (vezi „Triunghiul antisuplementar”) din teorema sinusurilor rezultă

) ) )2 2

sin(90 /2 sin(90 /2 sin(90 /2b c c a a bI I I I I I

RA B C

= = = ⋅°− °− °−

sau

4 cos , 4 cos , 4 cos2 2 2b c c a a b

A B CI I R I I R I I R= = = .

25) Punctele , , ',

aI I A A formează o diviziune armonică, unde ' .

aA II BC= ∩

Demonstraţie. Deoarece '

'a a

A I r

A I r= şi

a

AI p a

AI p

−= , iar [ ] ( )

ABC aA pr p a r= = − , rezultă

'

'a a

A I AI

A I AI= , deci punctele , , ',

aI I A A formează o diviziune armonică.

Page 199: Barbu_TEOREME_FUNDAMENTALE(2008).pdf

206

26) Punctele de contact a

C şi a

D sunt conjugate armonic în raport cu picioarele

bisectoarei interioare ( ')A şi a înălţimii din A ( )a

H .

Demonstraţie. Deoarece punctele , , ',a

I I A A formează o diviziune armonică, rezultă că şi

proiecţiile lor a

C ,a

D , 'A , respectiv a

H formează o diviziune armonică.

27) Punctele , , ,

b cI I A P formează o diviziune armonică, unde .

b cP I I BC= ∩

Demonstraţie. Deoarece b

c

AI p c

AI p b

−=

− şi b b

c c

PI r

PI r= , iar [ ] ( ) ( )ABC c bA p c r p b r= − = − ,

rezultă b b

c c

AI PI

AI PI= .

28) Punctele , , ',B C A P formează o diviziune armonică, unde .

b cP I I BC= ∩

Demonstraţie. Avem: '

'

A B PB AB

A C PC AC= = ,deci punctele , , ',B C A P formează o diviziune

armonică 29) Aria unui triunghi ABC este egală cu

[ ] ( ) ( ) ( )ABC a b c a b c

A pr p a r p b r p c r rr r r= = − = − = − = , unde r este raza cercului înscris,

iar , ,a b cr r r sunt razele cercurilor exînscrise.

Demonstraţie. Vezi „Aria unui triunghi”. 30) Lungimile razelor cercurilor tritangente corespunzătoare triunghiului ABC sunt

egale cu: 4 sin cos cos2 2 2a

A B Cr R= , 4 cos sin cos

2 2 2b

A B Cr R= , 4 cos cos sin

2 2 2c

A B Cr R= .

Demonstraţie. Avem 2

[ ] 2 sin sin sin

4 cos sin sin2 2 2

ABC

a

A R A B Cr

A B Cp aR

= =−

, de unde rezultă

(2sin cos ) (2sin cos ) (2sin cos )2 2 2 2 2 2 4 sin cos cos

2 2 22 cos sin sin2 2 2

a

A A B B C CR

A B Cr R

A B CR

⋅ ⋅ ⋅= = . Analog se

arată şi celelalte două egalităţi. 31) În orice triunghi ABC sunt adevărate egalităţile: i) 4a b cr r r r R+ + = + ; ii)

1 1 1 1

a b cr r r r+ + = ;

Demonstraţie. i) Avem: [ ]4 ABC

abcR

A=

⋅, [ ]ABCAr

p= , [ ]ABC

a

Ar

p a=

−, [ ]ABC

b

Ar

p b=

− si [ ]ABC

c

Ar

p c=

−,

unde 2

a b cp

+ += , deci

3 2 2 2

[ ]

2a b c

ABC

p ap bp cp abcr r r r

A

− − − ++ + − = =

2

[ ]

[2 ( )]

ABC

p p a b c abc

A

− + + +=

[ ]

4ABC

abcR

A= . Analog se arată că ii).

Page 200: Barbu_TEOREME_FUNDAMENTALE(2008).pdf

207

Observaţie: Ecuaţia Rrrrr cba 4+=++ se numeşte relaţia lui Steiner.

32) În orice triunghi ABC este adevărată relaţia: 2 2 2 2 212

a b cOI OI OI OI R+ + + = .

Demonstraţie. Din relaţiile lui Euler avem 2 2 2OI R Rr= − , 2 2 2a a

OI R Rr= + , şi

analoagele. Avem: 2 2 2 2 2 2 24 2 ( ) 4 2 4 12

a b c a b cOI OI OI OI R R r r r r R R R R+ + + = + + + − = + ⋅ = , unde am utilizat

relaţia 4a b cr r r R r+ + = + .

33) În orice triunghi ABC sunt adevărate egalităţile: 1) 2

a b b c c ar r r r r r p+ + = ;

2) 2( )( )( ) 2a b b c c ar r r r r r Rp+ + + = ;

3) ( )( )( ) ( )( )( )a b b c c ar r r r r r r p a b b c c a− − − = − − − ;

4) 2( )( )( ) 4a b cr r r r r r Rr− − − = ;

5) [ ]

( )ABC

a

a Ar r

p p a

⋅− =

−;

6) ( )( )( )/ ( )( )( )/a b cr r r r r r r a b b c c a p+ + + = + + +

7) ( )a br r p p c= − ;

8) 2 2 2

2 2 2 2 2[ ]

1 1 1 1

a b c ABC

a b c

r r r r A

+ ++ + + = ;

9) 2 2 2

2( ) ( ) ( )a b c b a c c a b

a b c

r r r r r r r r r+ + =

+ + +;

10) 2( ) ( ) ( ) 2 .a b c b c a c a br r r r r r r r r p+ + + + + =

Demonstraţie. Se utilizează egalităţile

[ ] ( ) ( ) ( )ABC a b c a b cA pr p a r p b r p c r rr r r= = − = − = − = .

34) În orice triunghi ABC sunt adevărate egalităţile:

.2 2 2a b c

A B Cp r ctg r ctg r ctg= ⋅ = ⋅ = ⋅

Demonstraţie. În triunghiul dreptunghic a cAI D , avem imediat .2a

Ap r ctg= Analog se

obţin celelalte egalităţi. 35) În orice triunghi ABC sunt adevărate egalităţile:

,2 2 2 2 2b c a

C B A B Cp a r tg r tg r ctg tg tg− = ⋅ = ⋅ = ⋅ ⋅ ⋅

,2 2 2 2 2a c b

C A B C Ap b r tg r tg r ctg tg tg− = ⋅ = ⋅ = ⋅ ⋅ ⋅

.2 2 2 2 2a b c

B A C A Cp c r tg r tg r ctg tg tg− = ⋅ = ⋅ = ⋅

Page 201: Barbu_TEOREME_FUNDAMENTALE(2008).pdf

208

Demonstraţie. În triunghiul dreptunghic ,a cBI D de exemplu, avem: aBI p c= − =

(90 / 2) .2a a

Br ctg B r tg⋅ ° − = ⋅ Pentru următoarele relaţii: p p b p c+ − + − =

.2 2 2 2 2 2a a

A B C A B Cr ctg tg B tg r ctg tg tg + + = ⋅ ⋅ ⋅

Analog se arată celelalte egalităţi.

36) În orice triunghi ABC este adevărată relaţia 1/ 1/ 2 /a ar r h− = , unde ah reprezintă lungimea înălţimii duse din vârful A. Demonstraţie. Avem 1/ 1/ / ( ) / / 2 /a ar r p S p a S a S h− = − − = = , unde S reprezintă aria

triunghiului ABC.

37) Consecinţe:

1) 1 / 1 / 1 / 1a b cr r r r r r

a b c p

− − −= = = ; 2) ( ) / ( ) / ( ) / 6.a b c b c a c a bh h r h h r h h r+ + + + + =

Demonstraţie. Se utilizează relaţia 1/ 1/ 2 /a ar r h− = şi analoagele.

38) În orice triunghi ABC sunt adevărate egalităţile: 2 2 ,b c b crr rr r r p a+ + = −

2 2 ,c a c arr rr r r p b+ + = − 2 2 .a b a brr rr r r p c+ + = −

Demonstraţie: 2 2( )( ) ( )( ) ( ) ( )( ) .b c b crr rr r r p a p c p a p b p p a p a p a p a+ + = − − + − − + − = − + = −

39) În orice triunghi ABC sunt adevărate egalităţile: : ( )( )aar p a r r= − − şi

( ).a aar p r r= − Demonstraţie. Avem [ ] .( )

ABC a

a

a A ararr r

p p a p a p

⋅− = = =

− −

40) Consecinţă: 2 ( ).a b car br cr p R r+ + = −

Demonstraţie. Se utilizează teorema precedentă.

Page 202: Barbu_TEOREME_FUNDAMENTALE(2008).pdf

209

I.50. Cercurile lui Lemoine71 „Matematica este nici mai mult, nici mai puţin decât partea exactă a gândirii noastre.” – L. Brower72

1) Punctele de intersecţie dintre laturile triunghiului ABC şi paralelele duse la laturile triunghiului prin punctul său simedian aparţin unui cerc. Demonstraţie. Fie M, N, P, Q, R, S punctele de intersecţie (Fig. 213) şi K punctul lui Lemoine al triunghiului ABC. Deoarece ARKQ este paralelogram rezultǎ cǎ mijlocul diagonalei RQ aparţine simedianei AK, deci RQ este antiparalelǎ cu BC . Analog, dreptele SM şi NP sunt antiparalele cu laturile AC , respectiv

AB. Atunci, ARQ ACB BSM≡ ≡ şi cum MQ AB

rezultǎ cǎ patrulaterul SMQR este trapez isoscel, deci

punctele S, M, Q,R sunt pe un cerc L. Din RN AC

rezultǎ BRN BAC≡ (1); din MQ AB rezultǎ

QMC ABC≡ (2) şi BSM SMQ ACB≡ ≡

(3). Din relaţiile (1), (2) şi (3) rezultǎ ( ) ( ) ( ) ( ) ( ) 180m SRN m SMN m BAC m ABC m BCA+ = + + = ° , adicǎ patrulaterul

SMNR este inscriptibil, deci punctul N ∈L. Analog, patrulaterul SMNP este trapez isoscel,

deci şi P∈L. Astfel punctele M,N,P,Q,R,S sunt conciclice.

Observaţii:

i) Cercul L se numeşte primul cerc al lui Lemoine.

ii) Paralelele duse prin punctul lui Lemoine la laturile triunghiului ABC se numesc paralelele lui Lemoine. 2) Centrul primului cerc al lui Lemoine este mijlocul segmentului OK (O este centrul cercului circumscris triunghiului ABC ). Demonstraţie. Fie V şi L mijloacele segmentelor RQ, respectiv KO. Atunci VL este linia mijlocie în triunghiul AKO , deci VL AO şi / 2 / 2.VO AO R= = Deoarece AO RQ⊥

rezultǎ VL RQ⊥ , deci triunghiul RLQ este isoscel, de unde .RL LQ≡ Analog, se aratǎ cǎ

LS LM LN LP LR≡ ≡ ≡ ≡ , adicǎ mijlocul segmentului KO este centrul primului cerc al lui Lemoine al triunghiului ABC . 3) Consecinţă: Primul cerc al lui Lemoine şi cercul lui Brocard sunt concentrice. Demonstraţia este evidentă ambele cercuri având centrul în mijlocul segmentului OK. 4) Primul cerc al lui Lemoine determină pe laturile triunghiului ABC segmente proporţionale cu pătratul lungimilor laturilor triunghiului ABC .

71 Emile Lemoine (1840-1912) – matematician francez, contribuţii importante în geometrie 72 L. Brower (1881-1966) – matematician danez, contribuţii în logica matematică

A

B C M N

P

Q

R

S K L

V

O

Fig. 213

Page 203: Barbu_TEOREME_FUNDAMENTALE(2008).pdf

210

Demonstraţie. Fie a, b, c lungimile laturilor BC , CA respectiv AB. Avem: 2

2

BM RK c

MN KN a= = , deoarece (MK BR ) sau 2 2

BM MN

c a= (1) şi

2

2

MN KM a

NC KQ b= = , adică

2 2

MN NC

a b= (2). Din relaţiile (1 ) şi (2) rezultă 2 2 2 .

MN NC BM

a b c= =

5) CoardeleMN , PQ şi RS sunt proporţionale cu 3 3,a b , respectiv 3.c

Demonstraţie. Din proprietatea precedentă avem

2 2 2 2 22

MN NC BM MN NC BM

a b a b cc

+ += = = =

+ + 2 2 2

a

a b c+ +, de unde

3

2 2 2

aMN

a b c=

+ +.

Analog, 3

2 2 2

bPQ

a b c=

+ + şi

3

2 2 2

cRS

a b c=

+ + , deci 3 3 3

MN PQ RS

a b c= = .

6) Triunghiurile RMP şi SNQ sunt congruente şi asemenea cu triunghiul ABC .

Demonstraţie.Avem: 1( ) ( ) ( ) ( )

2m MRP m MNP m MSP m BAC= = = , 1

( ) ( )2

m MPR m RSM= =

( ) ( )m RQM m ACB= , deci triunghiurile RMP şi ABC sunt asemenea. Analog se arată că

triunghiurile QSN şi ABC sunt asemenea. Deoarece SN MP≡ (ca diagonale într-un trapez isoscel) şi SQ RM≡ rezultă că triunghiurile RMP şi SNQ sunt congruente.

7) Antiparalelele RQ , SM şi NP sunt congruente.

Demonstraţie. Deoarece patrulaterele SMNP şi NPQR sunt trapeze isoscele, rezultă că .RQ SM NP≡ ≡

8) Primul cerc al lui Lemoine este un cerc Tucker. Demonstraţie. Vezi „Cercurile Tucker”. 9) Punctele de intersecţie dintre antiparalelele la laturile unui triunghi duse prin punctul lui Lemoine al triunghiului sunt şase puncte conciclice. Demonstraţie. Fie ' ', ' 'S P R N şi ' 'M Q antiparalele duse la laturile BC , CA respectiv AB prin punctul K al lui Lemoine al triunghiului ABC (Fig. 214). Atunci, K este mijlocul segmentelor ' ', ' ', ' 'S P R N M Q . Dar

' ' ' ' 'ABC S P Q M Q C≡ ≡ , deci triunghiul ' 'KP Q

este isoscel, de unde ' 'KP KQ≡ adică ' ' ' '.S P M Q≡

Analog, ' ' ' ',M Q R N≡ deci antiparalelele ' ', ' 'S P R N şi

' 'M Q sunt congruente şi au acelaşi mijloc K, deci

punctele ', ', ', ', ', ' M N P Q R S aparţin unui cerc L’.

Observaţii:

i) Cercul L’ este al doilea cerc al lui Lemoine.

ii) Centrul cercului L’ este punctul lui Lemoine al triunghiului ABC .

A

B C M' N'

P '

Q' R '

S' K

Fig. 214

Page 204: Barbu_TEOREME_FUNDAMENTALE(2008).pdf

211

10) Coardele ' ', ' 'M N P Q şi ' 'R S sunt proporţionale cu cos ,cos ,A B respectiv cos .C

Demonstraţie. Din triunghiul ' 'KM N rezultă ' '

cos ' ' cos2 '

M NKM N A

KN= =

⋅ sau

' '2 ' 2 '

cos

M NKN

Aρ= = ( unde 'ρ este raza celui de al doilea cerc Lemoine). Analog,

' ' ' '2 '

cos cos

P Q R S

B Cρ= = .

11) Triunghiurile ' ' ' R M P şi ' ' 'Q S N au laturile paralele două câte două şi

perpendiculare pe laturile triunghiului ABC .

Demonstraţie. Deoarece ' ' R N şi ' 'M Q sunt diametre în cercul L’ rezultă că

( ' ' ') ( ' ' ') 90m R M N m Q N M= = ° , deci ' ' ' 'R M Q N şi ' 'R M BC⊥ , ' ' .N Q BC⊥

Analog, pentru celelalte perechi de laturi. Observaţie: Deoarece patrulaterele ' ' ' ', ' ' ' ' B N Q R N P R S şi ' ' ' ' S M P Q sunt

dreptunghiuri rezultă că ' ' , ' 'R Q BC N P AB şi ' ' .S M AC

12) Triunghiurile ' ' 'R M P şi ' ' 'Q S N sunt congruente şi asemenea cu triunghiul

ABC . Demonstraţie.Avem ( ) ( ) ( )' ' ' ' ' 'm S N Q m S P Q m B= = , ( ' ' ') ( ) ( )' ' 'm M R P m M Q P m B= = ,

deci ' ' ' ' ' 'S N Q M R P≡ , iar deoarece ' ' ' ' M R N Q şi ' ' , ' ' M R BC N Q BC⊥ ⊥ rezultă

că patrulaterul ' ' ' 'M N Q R este dreptunghi, ' ' ' '.R M N Q≡ Analog, ' ' ' 'R P S N≡ , de

unde rezultă că triunghiurile ' ' 'R M P şi ' ' 'N Q S sunt congruente şi asemenea cu

triunghiul BCA.

13) Într-un triunghi ABC, dreapta KH care uneşte punctul lui Lemoine cu ortocentrul triunghiului ABC este paralelă cu dreapta 9LO care uneşte centrul primului cerc al lui

Lemoine cu centrul cercului celor nouă puncte al triunghiului ABC şi 92 .KH LO=

Demonstraţie. Deoarece L şi 9O sunt mijloacele segmentelor OK, respectiv OH, rezultă că

segmentul 9LO este linie mijlocie în triunghiul OHK, deci 9KH LO şi 92 .KH LO=

Page 205: Barbu_TEOREME_FUNDAMENTALE(2008).pdf

212

I.51. Cercul lui Taylor73

„În geometrie nu există drumuri speciale pentru regi.” - Euclid74

1) Proiecţiile pe laturile unui triunghi ABC ale picioarelor înălţimilor triunghiului ABC sunt situate pe acelaşi cerc. Demonstraţie. Fie aH , bH , cH picioarele înălţimilor triunghiului ABC şi

' " ', , ,a a b

H H H " ,b

H ' ",c c

H H proiecţiile punctelor, aH , bH , cH pe laturile triunghiului

ABC (Fig. 215) ' " ' "( , ; ,a a b b

H H BC H H∈ ' "; , )c c

AC H H AB∈ ∈ . Patrulaterul " 'a a a

AH H H

fiind inscriptibil rezultă '' ' 'a a a a

AH H AH H≡ , dar 'a a

H AH ACB≡ (fiind

complemente ale unghiului aH AC ), deci '' 'a a

AH H ACB≡ , adică

patrulaterul ' ''a a

CH H B este inscriptibil, de unde rezultă că "a

H aparţine cercului circumscris

triunghiului ' " 'c b a

H H H . Analog, se arată că punctele 'b

H şi "c

H aparţin aceluiaşi cerc.

Observaţie: Cercul pe care se găsesc punctele ' " ' " ' ", , , , ,a a b b c c

H H H H H H se numeşte cercul

lui Taylor al triunghiului ABC . Notăm centrul cercului lui Taylor cu Y. 2) Dreptele ' "

a bH H , ' "

b cH H , ' "

c aH H sunt paralele cu laturile AB, ,BC respectiv CA ale

triunghiului ABC .

Demonstraţie. Deoarece a bH H este antiparalelă cu BC , iar ' "a b

H H este antiparalelă cu

a bH H rezultă că " "a b

H H AB . Analog se arată că " "b c

H H BC şi " "a c

H H CA .

73 Brook Taylor (1685-1731) – matematician englez, profesor la Universitatea din Cambridge, contribuţii

importante în analiza matematică şi geometrie 74 Euclid din Alexandria (330 – 275 î.e.n.) – matematician grec, contribuţii în geometrie

A

B C aH

'aH

"aH

bH

'bH

"bH

cH

'cH

"cH

H

A'

B'

C'

Fig. 215

Y

Page 206: Barbu_TEOREME_FUNDAMENTALE(2008).pdf

213

3) Triunghiurile " " "a b c

H H H şi ' ' 'a b c

H H H sunt asemenea cu triunghiul ABC .

Demonstraţie. Deoarece ' "a a

H H este antiparalelă cu BC rezultă " ' "a a c

H H H ABC≡ . În

cercul lui Taylor avem " " " " ' 'a b c a a c

H H H H H H≡ , de unde rezultă " " "a b c

H H H ABC≡ .

Analog " " "a c b

H H H ABC≡ , deci triunghiurile ABC şi " " "a b c

H H H sunt asemenea.

Analog se arată că ' ' ' " " 'b c a b a a

H H H H H H ABC≡ ≡ şi ' ' 'a b c

H H H ABC≡ , adică

triunghiurile ' ' 'a b c

H H H şi ABC sunt asemenea .

4) Triunghiurile " " "

a b cH H H şi ' ' 'a b cH H H sunt congruente.

Demonstraţie. Deoarece punctele ' '' ' '' ' '', , , , ,a a b b c c

H H H H H H aparţin cercului lui Taylor şi

triunghiurile ' ' 'a b cH H H şi " " "

a b cH H H sunt asemenea, atunci din ' ' ' '' '' ''c a b c a bH H H H H H≡ rezultă congruenţa coardelor ' "

b cH H şi " 'b cH H , adică triunghiurile

" " "a b cH H H şi ' ' '

a b cH H H sunt congruente.

5) Centrul cercului lui Taylor (Y) al triunghiului ascuţitunghic ABC este centrul cercului înscris în triunghiul median al triunghiului ortic al triunghiului ABC .

Demonstraţie. Fie ', ', 'A B C mijloacele laturilor , ,b c a c a bH H H H H H ale triunghiului ortic.

Deoarece dreptele ' "b b

H H şi ' "c cH H trec prin ', 'A C , respectiv ', 'A B (vezi Triunghiul

ortic”) iar coardele ' "b cH H şi ' "

c bH H sunt paralele, rezultă că triunghiurile ' "' b cA H H şi ' "'c b

A H H sunt isoscele, deci bisectoarea unghiului ' "'b c

H A H - adică bisectoarea unghiului

' ' 'B A C - este perpendiculară pe mijlocul coardei ' "b cH H , ceea ce arată că bisectoarea

unghiului ' ' 'B A C trece prin centrul cercului lui Taylor. Analog se arată că bisectoarea unghiului ' ' 'A B C trece prin centrul cercului lui Taylor, deci centrul cercului înscris în triunghiul median al triunghiului ortic al triunghiului ABC este centrul cercului lui Taylor (Y) al triunghiului ABC . Observaţie: Centrul cercului lui Taylor (Y) al triunghiului ascuţitunghic ABC este punctul lui Spieker al triunghiului ortic al triunghiului ABC . 6) Consecinţă: ' , 'YA BC YB CA⊥ ⊥ şi ' .YC AB⊥

Demonstraţie. Deoarece 'YA este bisectoarea interioară a unghiului ' ' 'B A C , patrulaterul

' ' 'aA B H C este paralalogram şi aAH bisectoarea unghiului ' 'aB H C , rezultă că

' aYA AH . Dar aAH BC⊥ , de unde rezultă că ' .YA BC⊥

7) Fie 1 2 3, ,H H H ortocentrele triunghiurilor , ,b c c aAH H BH H respectiv a bCH H .

Triunghiurile ' ', ' ', ' 'YB C YC A YA B sunt respectiv omotetice cu triunghiurile

1 2 3, , .b c c a a bH H H H H H H H H

Demonstraţie. Deoarece 1' cYB H H , 1' bYC H H şi ' 'b cH H B C , rezultă că triunghiurile

1 b cH H H şi ' 'YB C sunt omotetice, raportul de omotetie fiind ½ deoarece ' ' 1

2b c

B C

H H= .

Page 207: Barbu_TEOREME_FUNDAMENTALE(2008).pdf

214

8) Centrul cercului lui Taylor al triunghiului ABC este mijlocul segmentelor

1 2, ,a bH H H H respectiv 3cH H .

Demonstraţie. Deoarece 1

' 1

2c

YB

H H= şi

' 1

2a

a c

H B

H H= , rezultă că triunghiurile 'aH YB şi

1a cH H H sunt omotetice, raportul de omotetie fiind ½ , deci prin această omotetie punctele

1, ,aH Y H sunt coliniare şi 1aH Y YH≡ . Analog se arată că Y este mijlocul segmentelor

2bH H şi 3cH H .

9) Triunghiurile 1 2 3H H H şi a b cH H H sunt congruente.

Demonstraţie. Deoarece Y este mijlocul segmentelor 1 2,a bH H H H rezultă că patrulaterul

1 2a bH H H H este paralelogram, deci 1 2a bH H H H≡ . Analog se arată că 2 3b cH H H H≡ şi

3 1c aH H H H≡ , de unde rezultă că 1 2 3 .a b cH H H H H H≡

10) Cercul lui Taylor al triunghiului ABC este un cerc Tücker. Demonstraţie. Deoarece antiparalelele ' "

a aH H , ' "b bH H şi ' "

c cH H (vezi „Triunghiul ortic”)

sunt congruente, rezultă că cercul lui Taylor este un cerc Tücker. 11) Raza cercului lui Taylor are lungimea egală cu

2 2 2 2 2 2sin sin sin cos cos cos .TR R A B C A B B= +

Demonstraţie. Fie P proiecţia lui Y pe dreapta ' "a aH H , atunci YP este raza cercului înscris în

triunghiul median al triunghiului ortic a b cH H H , deci are lungimea jumătate din lungimea

razei cercului înscris în triunghiul ortic, astfel cos cos cosYP R A B C= (vezi „Triunghiul

ortic”). Deoarece ' " 2 sin sin sina aH H R A B C= (vezi „Triunghiul ortic”), din triunghiul

dreptunghic 'aYH P rezultă concluzia.

12) Dreptele lui Simson ale punctelor ', ', 'A B C , mijloacele segmentelor AH, BH, respectiv CH unde H este ortocentrul triunghiului ABC, în raport cu triunghiul

a b cH H H determină un triunghi 1 2 3S S S .Centrul cercului lui Taylor al triunghiului ABC

este ortocentrul triunghiului 1 2 3S S S . Demonstraţie. Vezi „Dreapta lui Simson”. 13) Cercul lui Taylor al triunghiului ABC este ortogonal cercurilor exînscrise triunghiului ortic al triunghiului ABC. Demonstraţie. Vârfurile triunghiului ABC sunt centrele cercurilor exînscise corespunzătoare triunghiului ortic a b cH H H . Din triunghiurile '

a aAH H şi aAH C rezultă

' 2sin sina a

AH AH C b C= = , iar din triunghiurile "c cAH H şi cACH rezultă

" 2cos cosc c

AH AH A b A= = . Atunci, ' " 2 2 2 2 2 2 2sin cos 4 sin sin cosa cAH AH b C A R B C A⋅ = = şi ţinând cont că

cos 2 sin cosbAH c A R C A= = rezultă ' " 2 2 2sina c bAH AH AH B AZ⋅ = = , unde Z este

proiecţia lui A pe b cH H (adică pătratul razei cercului aH - exînscris), deci cercul lui Taylor

este ortogonal cercului aH - exînscris; analog se arată şi ortogonalitatea celorlalte cercuri

exînscrise cu cercul lui Taylor.

Page 208: Barbu_TEOREME_FUNDAMENTALE(2008).pdf

215

14) Fie a b cH H H şi a b cM M M triunghiul ortic, respectiv triunghiul median al unui

triunghi ABC. Dreptele lui Simson ale punctelor , ,a b cM M M în raport cu triunghiul

a b cH H H sunt concurente în centrul cercului lui Taylor al triunghiului ABC. Demonstraţie. Vezi „Dreapta lui Simson”. 15) Fie a b cH H H şi a b cM M M triunghiul ortic, respectiv triunghiul median al unui

triunghi ABC. Dreptele lui Simson ale punctelor , ,a b cH H H în raport cu triunghiul

a b cM M M sunt concurente în centrul cercului lui Taylor al triunghiului ABC. Demonstraţie.

Fie D, E, F proiecţiile punctului aH pe dreptele ,a bM M b cM M , respectiv a cM M .

Centrul cercului circumscris triunghiului ABC este ortocentrul triunghiului median a b cM M M , deci dreapta DE trece prin punctul Q - mijlocul segmentului aOH (cf.

th. (1)- „Dreapta lui Simson”) şi deoarece aAE EH≡ rezultă că EQ este linie mijlocie în

triunghiul aAH O . Cum ortocentrul triunghiului b cAH H - punctul 1H - aparţine dreptei AO

(deoarece b cAO H H⊥ - vezi „Triunghiul ortic”) rezultă că mijlocul segmentului 1aH H -

adică centrul cercului lui Taylor corespunzător triunghiului ABC – aparţine liniei mijlocii EQ, adică dreptei lui Simson a punctului aH în raport cu triunghiul median. Analog se

arată că şi dreptele lui Simson ale punctelor ,b cH H în raport cu triunghiul a b cM M M sunt

concurente în centrul cercului lui Taylor al triunghiului ABC.

A

B C aM

bM cM

aH

bH

cH

Fig. 216

O H

D

F

1H E

Page 209: Barbu_TEOREME_FUNDAMENTALE(2008).pdf

216

I.52. Cercul lui Tücker75

„Un matematician încearcă în opera sa aceeaşi plăcere ca şi un artist; plăcerea este tot atât de mare şi de aceeaşi natură. – Henri Poincaré76

Numim antiparalelă a laturii BC a triunghiului ABC , o dreaptă paralelă cu

tangenta în vârful A la cercul circumscris triunghiului ABC .

1) Dacă 212121 ,, CCBBAA sunt trei

antiparalele la laturile CABC, , respectiv AB, ( CAACBCCBABBA ∈∈∈ 212121 ,;,;, ), ale

triunghiului ABC , astfel încât

212121 CCBBAA ≡≡ , atunci punctele

212121 ,,,,, CCBBAA sunt conciclice.

Demonstraţie. Avem: 1 2 1 2≡ ≡ AA A ACB B B B

şi 2121 BBAA ≡ , deci patrulaterul 2121 BBAA

este trapez isoscel (Fig. 217), deci punctele

2121 ,,, BBAA sunt conciclice. Analog,

1212 AACC este trapez isoscel, de unde rezultă

1221 || CACA şi de aici 1 2 2 1AC B ACB BB B≡ ≡ ,

adică patrulaterul 2121 CBBA este inscriptibil,

deci punctul 2C aparţine cercului circumscris patrulaterului 2121 BBAA . Analog se arată că

1C este punct pe acest cerc, deci punctele 212121 ,,,,, CCBBAA sunt conciclice.

Cercul determinat de punctele 212121 ,,,,, CCBBAA se numeşte cercul lui Tücker.

2) Dacă γβα ,, sunt mijloacele antiparalelor 21AA , 21BB respectiv 21CC , atunci

triunghiurile αβγ şi ABC sunt omotetice.

Demonstraţie. Dreptele αA , βB si γC sunt simedianele triunghiului ABC , deci sunt

concurente în punctul lui Lemoine (K) al triunghiului ABC (vezi „Simediane”). Atunci, αβ , βγ şi γα sunt linii mijlocii în trapezele 2121 BBAA , 2121 CCBB , respectiv

2121 CAAC , deci || , || , ||AB BC CAαβ βγ γα , deci triunghiurile αβγ şi ABC sunt

omotetice, centrul de omotetie fiind punctul lui Lemoine K al triunghiului ABC . 3) Centrul cercului lui Tücker aparţine dreptei lui Brocard. Demonstraţie. Fie O centrul cercului circumscris triunghiului ABC . Deoarece 21AA ,

21BB şi 21CC sunt antiparalele la laturile triunghiului ABC , rezultă că perpendicularele

duse din vârfurile BA, respectiv C pe acestea sunt concurente în O (vezi „Drepte

75 Robert Tücker (1832-1905) – matematician englez, contribuţii în geometrie 76 Henri Poincaré ( 1854 -1912) – matamatician şi fizician francez, contribuţii importante în toate ramurile matematicii

A

B

C

1A 2A

1B

2B 1C

2C

A'

B' C' K

α

β γ

Fig. 217

Page 210: Barbu_TEOREME_FUNDAMENTALE(2008).pdf

217

izogonale”). Deoarece triunghiurile αβγ şi ABC sunt omotetice rezultă că

perpendicularele duse din γβα ,, pe dreptele 21AA , 21BB , respectiv 21CC sunt

concurente într-un punct Ü care aparţine dreptei OK . Cum Ü aparţine mediatoarelor

segmentelor 21AA , 21BB şi 21CC rezultă că Ü este centrul cercului lui Tücker.

4) Centrul cercului lui Tücker este centrul cercului circumscris triunghiului αβγ .

Demonstraţie. Evident, deoarece Ü se află la egale distanţe de coardele 21AA , 21BB şi

21CC în cercul lui Tücker.

5) Triunghiurile 111 CBA şi 222 CBA sunt congruente.

Demonstraţie. Avem 2211 BABA ≡ , 2211 CBCB ≡ , 2211 ACAC ≡ ca diagonale în trapeze

isoscele, deci triunghiurile 1 1 1A BC şi 2 2 2A B C sunt congruente.

6) Triunghiurile 111 CBA şi 222 CBA sunt asemenea cu triunghiul ABC .

Demonstraţie. Deoarece 1 1 1 1 2 1 1 1 1 1 2 1,≡ ≡ ≡ ≡ A B C A B C ABC BC A BC A ACB ,

rezultă că triunghiurile ABC şi 1 1 1A BC sunt asemenea. Analog se arată că triunghiurile

ABC şi 2 2 2A B C sunt asemenea.

7) Un cerc Tücker determină cu laturile triunghiului ABC trei coarde paralele şi trei coarde antiparalele laturilor triunghiului ABC . Demonstraţie. Evident 21AA , 21BB , 21CC sunt antiparalele laturilor CABC, , respectiv

AB, iar BCCB ||12 , CAAC ||12 , 2 1 || .A B AB

8) Fie 1 2 2 1 2 1 2 1 2 1 1 2 ' , ' , '= = =I I IA AC A B B B C A B C B C AC . Triunghiurile ''' CBA

şi ABC sunt omotetice. Demonstraţie. Patrulaterul 21 ' AAAA este paralelogram, deci 'AA trece prin mijlocul

segmentului 21AA , deci este simediană. Atunci, dreptele ',',' CCBBAA sunt concurente în

punctul lui Lemoine K al triunghiului ABC . Evident ' ' || , ' ' || , ' ' ||A B AB B C BC C A CA ,

deci triunghiurile ''' CBA şi ABC sunt omotetice.

9) Primul cerc al lui Lemoine este un cerc Tücker. Demonstraţie. Antiparalelele 212121 ,, CCBBAA sunt congruente în cercul lui Lemoine

(vezi „Cercul lui Lemoine”) deci primul cerc al lui Lemoine (L)este un cerc Tücker.

Observaţie: În acest caz triunghiul αβγ se reduce la punctul K , iar Ü este mijlocul

segmentului OK . 10) Al doilea cerc al lui Lemoine este un cerc Tücker. Demonstraţie. Antiparalelele 212121 ,, CCBBAA fiind congruente în cercul (L’) rezultă că

(L’) este un cerc Tücker (vezi „Cercul lui Lemoine”).

Observaţie: Al doilea cerc al lui Lemoine este cercul Tücker în care antiparalelele sunt congruente şi concurente.

Page 211: Barbu_TEOREME_FUNDAMENTALE(2008).pdf

218

11) Cercul lui Taylor al triunghiului ABC este un cerc Tücker. Demonstraţie. Vezi „Cercul lui Taylor”.

I.53. Cercurile lui Lucas

„Universul este un cerc al cărui centru este pretutindeni, iar circumferinţa nicăieri.” – Blaise Pascal77

1) Să se arate că există trei cercuri 1 2 3, ,C C C tangente interior la cercul circumscris triunghiului ABC în vârfurile A, B, C şi tangente între ele două câte două. Demonstraţie. Presupunem cercurile construite, fie 1 1 1 2 2 2 3 3 3( , ), ( , ), ( , ).C L l C L l C L l

Deoarece cercurile sunt tangente interior

cercului circumscris C al triunghiului ABC

rezultă că 1 2 3; ;L OA L OB L OC∈ ∈ ∈ (unde O

este centrul cercului circumscris triunghiului ABC). Teorema cosinusului aplicată în triunghiurile 2 3L L O şi OBC ne dă:

2 3cos cosL OL BOC= sau 2 2 2 2 2 2

2 3 2 32

2 3

( ) ( ) ( )

2( )( ) 2

R l R l l l R R a

R l R l R

− + − − + + −=

− −

de unde 2 22 3 2 34 ( )( ) (1).R l l a R l R l= − −

Analog, se demonstrează că 2 2

1 3 1 34 ( )( )R l l b R l R l= − − şi 2 2

1 2 1 24 ( )( ) (2).R l l c R l R l= − − Înmulţind

membru cu membru relaţiile (1) şi (2) rezultă 3

1 2 3 1 2 38 ( )( )( )R l l l abc R l R l R l⋅ ⋅ = − − −

(3), sau 21 2 3 1 2 34 ( )( )( )

aR l l l ah R l R l R l⋅ ⋅ = − − − (4). Relaţile (4) şi (1) prin împărţire dau

11

( )ah R ll

a

−= şi analog 2

2

( )bh R ll

b

−= , 3

3

( ),ch R l

lc

−= de unde se obţine

1 2 3, ,a b c

a b c

Rh Rh Rhl l l

a h b h c h= = =

+ + +, (unde , ,a b ch h h sunt lungimile înălţimilor triunghiului

ABC). Deci, cercurile având centrele pe razele OA, OB respectiv OC şi razele

1 1 2 2,AL l BL l= = şi 3CL sunt cercurile căutate.

Observaţii:

1) Cercurile 1 1 1 2 2 2 3 3 3( , ), ( , ), ( , )C L l C L l C L l se numesc cercurile Lucas interioare ale

triunghiului ABC (Fig 218).

2) Razele cercurilor Lucas au lungimile 1 2 3, ,a b c

a b c

Rh Rh Rhl l l

a h b h c h= = =

+ + + (unde , ,a b ch h h

sunt lungimile înălţimilor triunghiului ABC, iar R raza cercului circumscris triunghiului ABC).

77 Blaise Pascal (1623 – 1662) – matematician, fizician şi filosof francez, contribuţii în geometria proiectivă,

algebră şi teoria probabilităţilor

Fig. 218

A

B

C

1L

2L

3L

O

Page 212: Barbu_TEOREME_FUNDAMENTALE(2008).pdf

219

2) Să se arate că există trei cercuri ' ' '

1 2 3, ,C C C tangente exterior la cercul circumscris triunghiului ABC şi tangente între ele, două câte două. Demonstraţie. Presupunem cercurile construite. Fie ' ' '

1 2 3, ,L L L centrele lor şi ' '1 2,l l , respectiv

'3l razele acestor cercuri (Fig. 219). Printr-un raţionament analog celui precedent se

determină razele acestor cercuri ca fiind ' ' '1 2 3, ,a b c

a b c

Rh Rh Rhl l l

a h b h c h= = =

− − − , (unde

, ,a b ch h h sunt lungimile înălţimilor triunghiului ABC, deci cele trei cercuri pot fi construite,

având centrele pe semidreptele ( , ( , (OA OB OC şi razele ' ' ' '1 1 2 2,AL l BL l= = , respectiv

' '3 3CL l= ).

Observaţii:

1) Cercurile ' ' ' ' ' ' ' ' '1 1 1 2 2 2 3 3 3( , ), ( , ), ( , )C L l C L l C L l se numesc cercurile Lucas exterioare.

2) Razele cercurilor Lucas au lungimile: ' ' '1 2 3, , .a b c

a b c

Rh Rh Rhl l l

a h b h c h= = =

− − −

3) Cercurile Lucas 1 2 3, ,C C C , ' ' '

1 2 3, ,C C C sunt tangente la cercurile Apollonius ale

vârfurilor triunghiului ABC. Demonstraţia este evidentă ţinând cont că cercurile Apollonius corespunzătoare vârfurilor triunghiului ABC sunt ortogonale cercului circumscris triunghiului ABC.

A

B C

Fig. 219 '1L

'2L '

3L

Page 213: Barbu_TEOREME_FUNDAMENTALE(2008).pdf

220

I.54. Cercurile lui Apollonius78

„Demonstraţia este idolul în faţa căruia matematicianul pur se torturează.” Arthur Eddington79

1) Locul geometric al punctelor pentru care raportul distanţelor la două puncte fixe este o constantă 1k ≠ este un cerc. Demonstraţie. Fie A şi B punctele fixe şi P un punct ce aparţine locului geometric, adică

1.PA

kPB

= ≠ Fie C şi D sunt picioarele

bisectoarelor interioare şi exterioare a unghiului .APB Din teorema bisectoarei rezultă

PA CA DAk

PB CB DB= = = , deci punctele C

şi D aparţin locului geometric.Deoarece ( ) 90m CPD = ° , rezultă că punctul P aparţine

cercului de diametru CD unde punctele C şi D sunt punctele fixe determinate mai sus (Fig. 220).

Fie ,C D AB∈ astfel încât CA DA

kCD DB

= = (1) şi

P un punct pe cercul de diametru CD . Vom arăta că PA

kPB

= , deci locul geometric va fi

cercul de diametru CD . Fie BE PC şi BE PC , , .E F AP∈ Atunci, din teorema lui

Thales rezultă: PA DA

PF DB= ,

PA CA

PE CB= care împreună cu relaţia (1) dau

PA PA

PF PE= şi de

aici .PF PE= Deoarece ( ) 90m CPD = ° şi BF PD , BE PC rezultă

( ) 90m FBE = ° , adică PB este mediană în triunghiul FBE, deci ( ) ( )m PBF m PFB=

(2). Dar ( ) ( )m PBF m BPD= (3) (unghiuri alterne interne), iar ( ) ( )m EPD m EPB=

(4) (unghiuri corespondente). Din relaţiile (2) , (3) şi (4) rezultă că

( ) ( )m BPD m EPD= , adică PD este bisectoarea exterioară a unghiului APB şi din

teorema bisectoarei rezultă (1)PA DAk

PB DB= = , ceea ce arată că locul geometric căutat este

cercul de diametru CD. Fiind dat triunghiul ABC , se numeşte cerc al lui Apollonius corespunzător vârfului A , cercul loc geometric al punctelor P din planul triunghiului ABC pentru care bPB cPC= (unde b , c sunt lungimile laturilor AC , respectiv AB).

78 Apollonius (262-190 î.H.) – matematician grec, contribuţii fundamentale în geometrie 79 Arthur Eddington (1882-1944) – astrofizician englez, contribuţii importante în teoria relativităţii

A B C

P E

F

Fig. 220

D

Page 214: Barbu_TEOREME_FUNDAMENTALE(2008).pdf

221

Observaţii: i) Analog, se defineşte cercul lui Apollonius corespunzător vârfului B (respectiv C ) ca fiind locul geometric al punctelor P din planul triunghiului ABC pentru care

PC a

PA c= (respectiv

PA b

PB a= ).

ii) Cercul lui Apollonius corespunzător vârfului A , conţine punctul A şi are ca diametru segmentul determinat de picioarele bisectoarelor interioare şi exterioare ale unghiului A . 2) Centrele cercurilor lui Apollonius corespunzătoare vârfurilor triunghiului ABC sunt punctele de intersecţie ale dreptei lui Lemoine a triunghiului ABC cu laturile acestui triunghi. Demonstraţie. Fie 1D şi 2D picioarele

bisectoarelor interioare, respectiv exterioare ale unghiului A şi M mijlocul segmentului 1 2D D , adică centrul cercului lui

Apollonius corespunzător vârfului A (Fig. 221). Deoarece 1 2D A D A⊥ avem

1 2a a aL A L D L D= = şi de aici rezultă că

2 2( ) ( )a am D AL m L AD= şi

1 1( ) ( )a am L AD m L D A= . Atunci,

( )m ACB = 1 1( ) ( )am L D A m D AC− = 1 1( ) ( )am L AD m BAD− = ( )am L AB , ceea ce

arată că dreapta aL A este tangentă în A la cercul circumscris triunghiului ABC , adică

punctul aL aparţine dreptei lui Lemoine a triunghiului ABC . Analog, se arată că şi centrele

celorlalte două cercuri ale lui Apollonius aparţin dreptei lui Lemoine. Consecinţă: Centrele cercurilor lui Apollonius aparţin dreptei lui Lemoine al triunghiului.

3) Cercurile lui Apollonius corespunzătoare vârfurilor triunghiului sunt ortogonale cercului circumscris. Demonstraţia este evidentă (de exemplu pentru cercul A-Apollonius avem aL A OA⊥ ).

4) Dacă S este un punct comun cercurilor lui Apollonius corespunzător vârfurilor A şi B ale unui triunghi ABC, atunci punctul S aparţine şi cercului lui Apollonius corespunzător vârfului C. Demonstraţie. Deoarece S este un punct comun cercurilor lui Apollonius corespunzătoare vârfurilor A şi B, avem: bSB cSC= şi respectiv aSA cSC= ,de unde rezultă aSA bSB= , adică punctul S aparţine şi cercului lui Apollonius corespunzător vârfuluiC . Punctul S din planul triunghiului ABC pentru care aSA bSB cSC= = (unde a,b,c sunt lungimile laturilor BC, CA, respectiv AB) se numeşte punct izodinamic al triunghiului ABC .

5) Raza cercului lui Apollonius corespunzător vârfului A al unui triunghi ABC este

egală cu 2 2A

abcR

b c=

−.

A

B C

O

2D aL

Fig. 221

1D

Page 215: Barbu_TEOREME_FUNDAMENTALE(2008).pdf

222

Demonstraţie. Fără a restrânge generalitatea presupunem că AC AB> ( )b c> (Fig. 221).

Folosind teorema bisectoarei şi proporţiile derivate din aceasta rezultă 1

acBD

b c=

+ şi

2

acD B

b c=

−. Atunci, 1 2 1 2D D D B BD= + dau 1 2 2 2

22 A

abcD D R

b c= =

−, de unde

2 2A

abcR

b c=

−.

Observaţie: Analog se arată că razele cercurilor lui Apollonius corespunzătoare vârfurilor

B şi C sunt 2 2B

abcR

a c=

− , respectiv

2 2C

abcR

b a=

−.

6) Axa radicală a unui cerc Apollonius corespunzător unui vârf al triunghiului ABC şi a cercului circumscris triunghiului ABC este simediana corespunzătoare vârfului comun celor două cercuri. Demonstraţie. Fie T al doilea punct de intersecţie dintre cercul lui Apollonius corespunzător vârfului A şi cercul circumscris triunghiului ABC . Atunci b TB c TC⋅ = ⋅ , iar din teorema sinusurilor

2 sin( )TB R BAT= , 2 sin( )TC R TAC= ,

deci sin( ) sin( )b BAT c TAC= . Dacă 1T

şi 2T sunt proiecţiile lui T dreptele AB,

respectiv AC, atunci 1sin( )TT

TABAT

= şi

2sin( )TT

TACAT

= , iar relaţia precedentă

devine 2 1

b c

TT TT= , adică distanţele de la punctul T la laturile AB, respectiv AC sunt

respectiv proporţionale cu lungimile acestora , adică AT este simediana din A, adică axa radicală a cercului lui Apollonius corespunzător vârfului A şi a cercului circumscris triunghiului ABC este simediana din A. Observaţie: Cercurile lui Apollonius intersectează cercul circumscris după simedianele triunghiului. 7) Punctul lui Lemoine are puteri egale faţă de cercurile lui Apollonius. Demonstraţia este evidentă, punctul lui Lemoine fiind punctul de intersecţie al simedianelor – privite ca axe radicale pentru cercurile lui Apollonius şi cercul circumscris triunghiului. 8) Dreapta OK este axa radicală a cercurilor lui Apollonius corespunzătoare vârfurilor triunghiului ABC. Demonstraţie. Deoarece cercurile lui Apollonius corespunzătoare vârfurilor triunghiului ABC sunt ortogonale cercului circumscris triunghiului ABC rezultă că puterea lui O faţă

A

B C

O

2D aL

Fig. 222

1D 1T

2T

T

Page 216: Barbu_TEOREME_FUNDAMENTALE(2008).pdf

223

de cercurile lui Apollonius este egală cu 2R , deci O aparţine axei radicale a cercurilor lui Apollonius. Cum şi punctul lui Lemoine K aparţine acestei axe rezultă că OK este axa radicală a cercurilor lui Apollonius corespunzătoare vârfurilor triunghiului ABC . 9) Fie P şi 'P două puncte, simetrice faţă de BC, ale cercului lui Apollonius corespunzător vârfului A al triunghiului ABC. Dreptele PA şi 'P A sunt izogonale. Demonstraţie. Fie P şi 'P puncte ce aparţin cercului lui Apollonius corespunzător vârfului A şi aparţin discului având centrul în centrul circumscris triunghiului ABC şi raza egală cu raza cercului circumscris triunghiului ABC, iar 1D şi 2D

picioarele bisectoarelor interioare, respectiv exterioare ce pleacă din A.

(Fig. 223). Deoarece 1 1( ) ( ' )m PD m P D=

rezultă '1 1( ) ( )am D AP m P AD= adică

dreptele AP şi 'AP sunt izogonale. Dacă punctele P şi 'P sunt în exteriorul discului considerat anterior, atunci AP şi 'AP sunt izogonale faţă de bisectoarea exterioară a unghiului A . 10) Triunghiul podar al unui punct P de pe un cerc al lui Apollonius, în raport cu triunghiul ABC este isoscel. Demonstraţie. Fie a b cP P P triunghiul podar

al punctului P situat pe cercul lui Apollonius al punctului A. Patrulaterele

b aPPCP şi a cPP BP fiind inscriptibile

rezultă: ( ) 180 ( )a bm P PP m ACB= ° −

şi ( ) ( )a cm P PP m ABC= , de unde

obţinem sin( )a bPP CP C= şi

sin( )a cP P BP B= . Cum sin

sin

BP c C

CP b B= = ,

rezultă a b a cP P P P= , adică triunghiul

a b cP P P este isoscel.

11) Dacă aL este centrul cercului lui Apollonius corespunzător vârfului A al

triunghiului ABC , atunci 2

a

a

L B AB

L C AC

=

.

Demonstraţie. Deoarece aL A este tangentă la cercul circumscris triunghiului ABC şi

( ) ( )a am L AB m ACL= rezultă că triunghiurile aL AB şi aL AC sunt asemenea deci,

a a

a a

L B L AAB

L A AC L C= = , de unde rezultă a a

a a

L B L A AB AB

L A L C AC AC⋅ = ⋅ , deci

2

.a

a

L B AB

L C AC

=

A

B C

O 2D aL

Fig. 223

1D P '

P

A

B C

aP

bP

Fig. 224

cP

P

Page 217: Barbu_TEOREME_FUNDAMENTALE(2008).pdf

224

12) Punctele izodinamice ale triunghiului ABC neechilateral sunt punctele de intersecţie dintre dreapta OK şi cercurile lui Apollonius. Demonstraţie. Vezi „Puncte izodinamice”. 13) Cercurile Lucas L1,L2,L3,L

’1,L2

’,L3’ sunt tangente la cercurile Apollonius ale

vârfurilor triunghiului ABC. Demonstraţie. Vezi „Cercurile lui Lucas”.

I.55. Cercurile adjuncte

„Poezia este o ştiinţă la fel de exactă ca şi geometria.”- Gustave Flaubert80

Un cerc ce trece prin două vârfuri ale unui triunghi şi este tangent la una din laturile triunghiului se numeste cerc adjunct. Unui triunghi îi corespund şase cercuri adjuncte. Notăm, de exemplu, cercul adjunct ce trece prin C şi este tangent în A la AB cu

CA . Atunci, cercurile adjuncte , ,CA AB BC trec prin primul punct al lui Brocard ( )Ω - fie

OA, OB, respectiv OC centrele lor- , iar cercurile adjuncte , ,AC CB BA trec prin al doilea punct al lui Brocard (vezi „Punctele lui Brocard”) - fie ' ', ,

C BO O respectiv '

AO centrele lor.

1) Fie R raza cercului circumscris triunghiului ABC. Razele cercurilor adjuncte

corespunzătoare triunghiului ABC sunt egale cu , , , , ,c b a c b a

R R R R R Rb c c a a b

(unde a,b,c

sunt lungimile laturilor BC, CA respectiv AB ).

Demonstraţie. Fie BR şi '

CR razele cercurilor adjuncte AB , respectiv AC , iar O centrul

cercului circumscris triunghiului ABC. În triunghiul '

COO C avem '

COO C ACB≡ , având

1Gustave Flaubert (1821-1880) – scriitor francez

A

B C

O

A'

BO '

CO

Fig. 225

Page 218: Barbu_TEOREME_FUNDAMENTALE(2008).pdf

225

laturile respectiv perpendiculare, iar ' 1( )

2CO OC ABC m AOC

≡ =

, deci triunghiurile ABC

şi '

COO C sunt asemenea, de unde

'

CR R

b c= , adică '

C

bR R

c= ⋅ şi analog

B

cR R

b= ⋅ .

Procedând analog se determină razele celorlalte patru cercuri adjuncte. Observaţie: Aplicaţia de mai sus nu permite să determinăm razele cercurilor adjuncte atunci când cunoaştem laturile triunghiului şi raza cercului circumscris acestuia. 2) Raza cercului circumscris unui triunghi este medie geometrică a razelor a două cercuri tangente la aceeaşi latură a triunghiului.

Demonstraţie. Din B

cR R

b= ⋅ şi '

C

bR R

c= ⋅ rezultă ' 2

BC

R R R⋅ = , adică 'B

CR R R= ⋅ .

3) Cubul razei cercului circumscris triunghiului ABC este egal cu produsul razelor cercurilor adjuncte care trec prin acelaşi punct Brocard.

Demonstraţie. Avem A

bR R

a= ⋅ , , , ' , ' , 'B C A B C

c a c a bR R R R R R R R R

b c a b c= ⋅ = = ⋅ = ⋅ = ⋅ de

unde 3 ' ' ' .A B C A B C

R R R R R R R= ⋅ ⋅ = ⋅ ⋅

4) Linia centrelor a două cercuri adjuncte, tangente la aceeaşi latură a triunghiului, intersectează latura în acelaşi punct cu simediana exterioară a vârfului opus acestei laturi. Demonstraţie. Fie ' ' .

B CA O O BC= I Din asemănarea triunghiurilor 'BO BA şi ' '

CO CA

rezultă ' 2

'

'C

B

RA C b

A B R c

= =

, deci 'A coincide cu punctul în care simediana exterioară a

vârfului A intersectează dreapta BC (vezi „Simediane exterioare”). 5) Punctele de intersecţie dintre dreptele ce unesc centrele perechilor de cercuri adjuncte, tangente la aceeaşi latură a triunghiului, şi laturile respective coliniare. Demonstraţie. Fie A’, B’, C’ punctele căutate. Conform proprietăţii precedente avem:

2''

A B c

A C b

=

, 2 2

' ',

' 'B C a C A b

B A c C B a

= =

, de unde ' ' '

1' ' '

A B B C C A

A C B A C B⋅ ⋅ = şi din reciproca

teoremei lui Menelaus rezultă că punctele A’, B’, C’ sunt coliniare. 6) Triunghiurile OAOBOC şi

' ' '

A B CO O O sunt asemenea cu triunghiul ABC.

Demonstraţie. Dacă Ω este primul punct Brocard, atunci ( ) 180 ( ),m A B m ABCΩ = °−

( ) 180 ( ), ( ) 180 ( )m B C m ACB m C A m BACΩ = °− Ω = °− (vezi „Punctele lui

Brocard”). Fie A B

M O O A= ΩI şi A C

N O O C= ΩI . Cum A B

O O A⊥ Ω şi

A CO O C⊥ Ω rezultă că patrulaterul

AO M NΩ este inscriptibil, deci

( ) 180 ( ) ( )A

m NO M m M N m BAC= °− Ω = , de unde B A CO O O BAC≡ . Analog se

arată că A B CO O O ABC≡ , deci triunghiurile A B CO O O şi ABC sunt asemenea. Analog

se arată că triunghiurile ' ' 'A B CO O O şi ABC sunt asemenea.

Page 219: Barbu_TEOREME_FUNDAMENTALE(2008).pdf

226

7) Triunghiurile OAOBOC şi ' ' '

A B CO O O au acelaşi unghi Brocard.

Demonstraţia este o consecinţă a proprietăţii precedente. 8) Centrul cercului circumscris triunghiului ABC este un punct Brocard pentru triunghiurile A B CO O O şi ' ' '

A B CO O O .

Demonstaţie. Avem ,A A C A

OO AC OO O O⊥ ⊥ , iar ( )m CA ωΩ = , de unde rezultă că

( )A Cm OO O ω= .Analog ( ) ( )B A C B

m OO O m OO O ω= = , deci O este punct Brocard în

triunghiul A B CO O O . Analog se arată că O este punct Brocard şi în triunghiul ' ' '

A B CO O O .

9) Primul punct Brocard (Ω) al triunghiului ABC este primul punct Brocard şi în triunghiul A B CO O O .

Demonstaţie: Fie .B CP O O AB= I Deoarece PM este linie mijlocie în triunghiul ABΩ

rezultă PM AB , de unde ( ) ( )m PM m BA ωΩ = Ω = . Din patrulaterul

inscriptibil BO P MΩ rezultă BPO PMΩ ≡ Ω , deci ( ) .Bm PO ωΩ = Analog,

( ) ( )C A A B

m O O m O O ωΩ = Ω = , deci Ω este primul punct Brocard al triunghiului

A B CO O O .

10) Al doilea punct Brocard (Ω’) al triunghiului ABC este al doilea punct Brocard şi în triunghiului ' ' '

A B CO O O .

Demonstraţie analoagă cu cea precedentă. Observaţii: i) Din cele prezentate mai sus rezultă că în triunghiul A B CO O O primul punct Brocard este

Ω şi al doilea punct Brocard este O - centrul cercului circumscris triunghiului ABC. ii) În triunghiul ' ' '

A B CO O O primul punct Brocard este O, centrul cercului circumscris

triunghiului ABC, şi al doilea punct Brocard este Ω’ (al doilea punct Brocard al triunghiului ABC). 11) Fie ρ raza cercului circumscris triunghiului A B CO O O . Este adevarată relaţia:

3 38 sinA B COO OO OO ρ ω⋅ ⋅ = .

Demonstraţie. Din formula cunoscută 3 38 sinA B C R ωΩ ⋅ Ω ⋅ Ω = (vezi “Punctele lui Brocard”) şi asemănarea triunghiurilor A B CO O O şi ABC rezultă:

3 38 sinA B COO OO OO ρ ω⋅ ⋅ = .

12) Dacă ρ este raza cercului circumscris triunghiului A B CO O O , atunci 2 sinR ρ ω= .

Demonstraţie.Din paralelogramul 'B BOO BO rezultă ' '

B BB

aOO BO R R

b= = = . Analog

A

cOO R

a= şi C

bOO R

c= . Utilizând relaţia 3 38 sinA B COO OO OO ρ ω⋅ ⋅ = rezultă

3 3 38 sinR ρ ω= , de unde 2 sinR ρ ω= .

Page 220: Barbu_TEOREME_FUNDAMENTALE(2008).pdf

227

Observaţii: i) Relaţia 2 sinR ρ ω= este echivalentă cu 2sinR

ωρ= şi cum triunghiurile

ABC şi A B CO O O sunt asemenea rezultă că raportul de asemănare dintre acestea este egal

cu 2 sinω .

ii) Deoarece unghiul lui Brocard este acelaşi şi pentru triunghiul ' ' '

A B CO O O rezultă că

raportul de asemănare dintre triunghiurile ABC şi ' ' '

A B CO O O este egal tot cu 2sinω , de unde

rezultă că triunghiurile asemenea A B CO O O şi ' ' '

A B CO O O sunt congruente.

13) Dacă A2 este punctul de intersecţie dintre cercurile adjuncte CA şi BA , atunci punctul A2 aparţine cercului circumscris triunghiului BOC. Demonstaţie. Vezi „Triunghiurile lui Brocard”. 14) Vârfurile celui de-al doilea triunghi Brocard al triunghiului ABC sunt intersecţiile dintre cercurile adjuncte corespunzătoare vârfurilor A, B respectiv C. Demonstraţie. Vezi „Triunghiurile lui Brocard”. 15) Axa radicală dintre două cercuri adjuncte tangente, în acelaşi vârf, a două laturi ale triunghiului ABC este simediana acestui vârf. Demonstraţie. Vezi „Triunghiurile lui Brocard”. 16) Axa radicală dintre două cercuri adjuncte ce trec prin acelaşi vârf şi sunt tangente la aceeaşi latură unui triunghi este mediana ce pleacă din vârful considerat. Demonstraţie. Fie aL punctul de intersecţie dintre cercurile adjuncte ce trec prin A şi sunt

tangete în B, respectiv în C la BC, iar .a aM AL BC= I Din puterea punctului aM faţă de

cele două cercuri rezultă ' 2 2a a a a aM B M L M A M C= ⋅ = , de unde a aM B M C≡ , deci aAM

este mediană în triunghiul ABC.

Observaţie: Punctul aL aparţine cercului ortocentroidal al triunghiului ABC (vezi “Cercul

ortocentroidal”).

I.56. Cercul ortocentroidal

„Fiecare posedă un anumit orizont. Când se îngustează şi devine infinit de mic el se transformă în punct şi atunci zice: Acesta este punctul meu de vedere.” - David Hilbert81 Prin vârful A al triunghiului ABC se duc două cercuri tangente la latura BC în vârfurile B respectiv C şi fie aL al doilea punct de întâlnire al lor. Analog, se obţin punctele bL şi cL .

Cercul circumscris triunghiului a b cL L L se numeşte cercul ortocentroidal al

triunghiului .ABC

81 David Hilbert (1962-1943) – matematician german, profesor la Universitatea din Göttingen, contribuţii

remarcabile în geometrie şi analiza matematică

Page 221: Barbu_TEOREME_FUNDAMENTALE(2008).pdf

228

1) Punctele , ,a b cL L L aparţin medianelor triunghiului ABC. Demonstraţie.

Fie a aM AL BC= ∩ (Fig. 226). Din puterea punctului aM faţă de cele două cercuri

rezultă: 2 2a a a a aM B M L M A M C= ⋅ = , de unde rezultă că a aM B M C= , adică aAM este

mediană în triunghiul .ABC Analog se demonstrează că punctele bL şi cL aparţin

medianelor bBM şi cCM ale triunghiului .ABC

2) Punctele , ,a b cL L L aparţin cercurilor circumscrise triunghiurilor BHC, AHC respectiv

AHB, unde H este ortocentrul triunghiului .ABC

Demonstraţie. Din ( ) ( )a am L BC m BAL= şi ( ) ( )a am L CB m L AC= rezultă:

( ) ( ) ( ).a am L BC m L CB m A+ = Atunci ( ) 180 ( ) ( )a a am BL C m L BC m L CB= ° − − =

180 ( ),m A° − şi cum ( ) 180 ( )m BHC m A= °− rezultă ,aL CB BHC≡ adică punctul

aL aparţine cercului circumscris triunghiului .BHC Analog, bL şi cL aparţin cercurilor

circumscrise triunghiurilor AHC, respectiv AHB. 3) Fie 1 aB BL AC= I şi 1 aC CL AB= I .

Patrulaterul 1 1aAB L C este inscriptibil.

Demonstraţie. 1 1( ) ( ) 180 ( )a am BL C m BL C m A= = °−

rezultă că patrulaterul 1 1aAB L C este inscriptibil

(Fig. 227). 4) Punctele , ,a b cL L L aparţin cercului de diametru HG, unde H este ortocentrul şi G centrul de greutate al triunghiului ABC. Demonstraţie. Fie 1 1, .BB AC B AC⊥ ∈ Patrulaterul

aBHL C fiind inscriptibil rezultă 1 .a aBCL B HL≡ Dar ,a aBCL L AC≡ de unde

1 1a aL HB L AB≡ adică patrulaterul 1aHL B A este inscriptibil, deci

A

B C A'

1C 1B

aL

Fig. 227

A

B C aM

Fig. 226

G

H

aH

aL

Page 222: Barbu_TEOREME_FUNDAMENTALE(2008).pdf

229

1( ) ( ) 90am HB A m HL A= = ° rezultă că ( ) 90am HL G = ° (deoarece aL aparţine

medianei 'AA ). Am demonstrat că punctul aL aparţine cercului de diametru HG. Analog,

se demonstrează că şi punctele bL şi cL aparţin acestui cerc.

Observaţie: Cercul ortocentroidal al triunghiului ABC mai poate fi definit ca fiind cercul având diametrul segmentul HG, unde H este orocentrul, iar G centrul de greutate al triunghiului .ABC

Consecinţe: 5) Punctele , ,a b cL L L aparţin cercurilor circumscrise triunghiurilor ce au ca vârfuri

mijloacele segmentelor AH, BH respectiv CH, mijloacele segmentelor ,b c a cH H H H

respectiv a bH H şi punctele A, B, respectiv C ( , ,a b cH H H sunt picioarele înălţimilor

triunghiului ABC ).

6) Punctele ,a b cL L L aparţin cercurilor circumscrise triunghiurilor ce conţin ortocentrul triunghiului şi mijloacele înălţimilor duse din vârfurile B şi C, A şi C respectiv B şi A. 7) Punctul aL aparţine arcurilor 1 1 1 1, ' ', ' ,ABC A BC A AH ' ' "A C B şi ' ' "A B C ( "B şi "C fiind intersecţiile cercului BHC cu laturile AB şi AC).

8) Dacă aK este punctul în care axa ortică a triunghiului ABC intersectează latura BC

atunci cercurile circumscrise triunghiurilor 1aK C C şi 1aK B B se întâlnesc în punctul aL .

Demonstraţie. Punctele B şi 1B sunt inversele punctelor C şi 1C în inversiunea de centru

aK şi raport aK B , respectiv .aK C Astfel, dreptele 1BB şi 1CC sunt inversele cercurilor

1aK C C , respectiv 1 .aK B B Dar dreptele 1BB şi 1CC sunt concurente în H care este inversul

lui aL , de unde rezultă concluzia.

9) Fie H ortocentrul şi O centrul cercului circumscris triunghiului ABC , 1B piciorul

înălţimii din B şi 2 1 .B CO BB= ∩ Cercurile circumscrise triunghiurilor 2AB B şi BHC

se intersectează în punctul aL al cercului ortocentroidal.

Demonstraţie. Avem: ( ) ( ) ( ) 90 ( )m AOB m HAC m HBC m ACB= = = °− , ceea ce

arată că cercul circumscris triunghiului 2AB B este tangent laturii BC în B. Punctul de

intersecţie aL dintre cercurile circumscrise triunghiurilor 2ABB şi BHC se află pe cercul

ortocentroidal după cum s-a definit acest cerc.

Page 223: Barbu_TEOREME_FUNDAMENTALE(2008).pdf

230

I.57. Cercurile lui Neuberg82

„Pythagoras a sacrificat pe altarul lui Zeus o sută de boi şi acesta numai pentru un singur adevăr geometric. Dar dacă în zilele noastre am proceda în acelaşi fel, este puţin probabil că am putea găsi atâtea vite cornute pe întreg globul pământesc.” - M. V. Lomonosov83

1) Fie un triunghi ABC cu baza fixă BC. Să se determine locul geometric al vârfului A, dacă unghiul lui Brocard al triunghiului ABC este constant. Demonstraţie. Fie M punctul de intersecţie dintre paralela prin A la BC şi mediatoarea segmentului BC (Fig. 228). Din teorema medianei rezultă

2 2 2 212

2ab c m a+ = + . Utilizând egalitatea

[ ]

2 22 2 2 3 4

4 4a

aABC

a ma b cctg

A a MMω

++ += =

⋅ ⋅ ( aM fiind mijlocul

laturii BC) obţinem 2 230

4a am a MM ctg aω− ⋅ ⋅ + = .

Considerăm punctul aN pe aMM astfel încât

1

2a aN M a ctgω= ⋅ şi astfel a a a aBN M CN M ω= = .

Din teorema lui Pitagora generalizată rezultă: 2 2 2 2a a a a a a aN A AM N M N M M M= + − ⋅ =

2 2 22 3 41 1

22 2 4

a

a

a mm actg actg

actgω ω

ω+ + − ⋅

, relaţie

echivalentă cu: 2 2 21( 3)

4aN A a ctg kω= − = . Egalitatea precedentă arată că locul

geometric căutat este un cerc (Na) cu centrul aflat în punctul aN şi raza egală cu

213

2an a ctg ω= − .

Observaţii: 1) Analog, pe laturile AC şi AB se construiesc triunghiurile cu acelaşi unghi Brocard ca şi cel al triughiului ABC, vârful liber va descrie câte un cerc (Nb) şi (Nc), de raze

213

2bn b ctg ω= − , respectiv 213

2cn c ctg ω= − .

2) Cercurile (Na), (Nb) şi (Nc) se numesc cercurile lui Neuberg. 3) Triunghiul a b cN N N ale carui vârfuri sunt centrele cercurilor lui Neuberg se numeşte

triunghiul lui Neuberg. 2) Razele cercurilor lui Neuberg, ale unui triunghi ABC sunt proporţionale cu lungimile laturilor triunghiului ABC.

82Joseph Neuberg (1840-1926) – matematician luxemburghez, preşedinte al Academiei Regale Belgiene,

contribuţii importante în geometrie 83 Mihail Lomonosov (1711-1765) – savant, poet şi filolog rus

A

B C

D

aM

2A

A'

aN 1T M

1A

Fig. 228

Page 224: Barbu_TEOREME_FUNDAMENTALE(2008).pdf

231

Demonstraţia este evidentă deoarece 213.

2a b cn n n

ctga b c

ω= = = −

3) Ecuaţia carteziană a cercului lui Neuberg (Na) în raport cu latura BC şi mediatoarea

sa este:2

2 2 30.

4

ax y ayctgω+ − + =

Demonstraţie. Considerăm un reper cartezian cu centrul în mijlocul segmentului BC, axa

absciselor fiind dreapta BC. Atunci, 1

0,2aN ctgω

, deci ecuaţia cercului cu centrul în

punctul aN şi rază 213

2a ctg ω − este

22 2 21 1

( 0) ( 3)2 4

x y actg a ctgω ω − + − = −

sau

22 2 3

0.4

ax y ayctgω+ − + =

4) Distanţele de la centrele cercurilor lui Neuberg la centrul circumscris triunghiului ABC sunt proporţionale cu cuburile lungimilor laturilor triunghiului ABC.

Demonstraţie. Din teorema sinusurilor în triunghiul aON B rezultă ,sin( ) sin

aON R

A ω ω=

−de

unde [ ]

23sin( )

sina ABC

A aON R R a A

bc

ωω−

= = ⋅ = ⋅ . Analog, [ ] [ ]3 3,b cABC ABC

ON b A ON c A= ⋅ = .

5) Consecinţă: 3.

a b cON ON ON R⋅ ⋅ =

Demonstraţia rezultă din teorema precedentă. 6) Cercul lui Neuberg (Na) este ortogonal cercurilor cu raza a şi centrele în punctele B şi C. Demonstraţie: Din teorema lui Pitagora în triunghiul a aN M B

avem:2

2 2 2 2 2 2( 1) ,4a a a a a

aN B N M BM ctg a nω= + = + = + deci cercul (Na) şi cercurile având

centrele în B şi C sunt ortogonale. Observaţie: Proprietăţi analoge se obţin pentru cercurile (Nb) şi (Nc). 7) Fie D şi 'D puncte pe mediatoarea laturii BC a triunghiului ABC, astfel încât triunghiurile BCD şi 'BCD sunt echilaterale. Pentru diferite valori ale unghiului lui Brocard al triunghiului ABC cercurile lui Neuberg au pe BC drept axă radicală. Demonstraţie. Fie 1 2,A A punctele de intersecţie dintre aM D şi cercul (Na), iar

1 1 T CA= ∩ (Na),. Avem: 2 2 2 21 2

3

4a a a a a aM A M A M N n a M D⋅ = − = = şi

2 2 2 2 21 1 a a

CA CT CN n a CB CD⋅ = − = = = , adică triunghiurile 1CBA şi 1CBT sunt asemenea,

deci 1

1 1

BT BC

BA AC= . Cum 1 1A B AC≡ rezultă 1BT BC BD≡ ≡ , deci 1BT este tangentă

Page 225: Barbu_TEOREME_FUNDAMENTALE(2008).pdf

232

cercului (Na) în punctul 1T . Analog, dacă 2 2 T BA= ∩ (Na) , atunci 2CT este tangent

cercului (Na) în punctul 2T .

8) Dacă 1 2,A A sunt punctele de intersecţie dintre mediatoarea laturii BC a triunghiului

ABC cu cercul lui Neuberg (Na) , atunci 1sin( )2

sin

ϕ ωω+

= şi 2sin( )2

sin

ϕ ωω+

= , unde

1 1( )m BACϕ = şi 2 2( )m BA Cϕ = .

Demonstraţie. Din asemanarea triunghiurilor 1BAC şi 1T BC rezultă 1 1 .T BC BAC≡ Din

patrulaterul inscriptibil 1 a aBT N M rezultă 1 ( ).a a aBTM BN M ω≡ = Unghiul 1 aTM C

fiind exterior triunghiului 1 aTM B rezultă: 1 1 1( ) ( ) ( )a a am T M C m BT M m T BM= + =

1( ),m BACω + de unde 11 1

1

sin( )sin( )2.

sin sin( )a

a a a

TM C BT BC

BTM BM BM

ϕ ωω+

= = = =

Analog se arată

că 2sin( )2

sin

ϕ ωω+

= .

9) Dacă 1 2,A A sunt punctele de intersecţie dintre mediatoarea laturii BC a triunghiului

ABC cu cercul lui Neuberg (Na) , atunci 21 3,2

ctg ctg ctgϕ

ω ω= − −

22 3,2

ctg ctg ctgϕ

ω ω= + − unde 1 1( )m BACϕ = şi 2 2( )m BA Cϕ = .

Demonstraţie. Avem: 1 1a a a a

a a a

AM M N N A

M B M B M B= − sau 21 3

2ctg ctg ctg

ϕω ω= − − şi analog

2 2a a a a

a a a

A M M N N A

M B M B M B= + sau 22 3.

2ctg ctg ctg

ϕω ω= + −

Observaţie: Unghiurile 1

2

ϕ şi 2

2

ϕ se numesc unghiurile lui Steiner.

10) Dreptele , ,a b cAN BN CN sunt concurente. Lemă: Pe laturile triunghiului ABC se

construiesc în exterior (sau în interior)

triunghiurile ' , ' , 'BC A AB C BA C isoscele şi

asemenea. Dreptele ', ', 'AA BB CC sunt

concurente. Demonstraţie. Fie " ' ,A A A B C= ∩

" ' ,B BB AC= ∩ " 'C CC AB= ∩

şi ( ' ) ( ' ) ( ' )m A BC m A CB m B CA= = =

( ' ) ( ' ) ( ' )m B AC m C AB m C BA α= = =

Avem: [ ]

[ ]

'

'

" ' sin( )

" ' sin( )

ABA

ACA

AA B AB A B B

B A A AC A C C

αα

⋅ += = =

⋅ +

C B

A C'

A'

B'

A"

B" C"

Fig. 229

Page 226: Barbu_TEOREME_FUNDAMENTALE(2008).pdf

233

sin( ).

sin( )

AB B

AC C

αα

⋅ +

⋅ + Analog se arată că

" sin( )

" sin( )

B C BC C

B A BA A

αα+

=+

şi " sin( )

" sin( )

C A CA A

C B CB B

αα

+=

+, de

unde rezultă că " " "

1" " "

A B B C C A

A C B A C B⋅ ⋅ = şi din reciproca teoremei lui Ceva rezultă că

dreptele ', ', 'AA BB CC sunt concurente.

Demonstraţia teoremei. Deoarece triunghiurile , ,a a aBCN CAN ABN sunt isoscele, având

unghiurile de la bază egale cu 90 ω° − , atunci rezultă că dreptele , ,a b cAN BN CN sunt

concurente conform lemei. 11) Triunghiul lui Neuberg a b cN N N şi triunghiul ABC au acelaşi centru de greutate. Lemă: Pe laturile triunghiului ABC se construiesc în exterior (sau în interior) triunghiurile

' , ' , 'BC A AB C BA C isoscele şi asemenea. Triunghiurile ABC şi ' ' 'A B C au acelaşi centru

de greutate. Demonstraţie. Notăm cu litere mici afixele punctelor corespunzătoare şi ( ' )m A BCα = .

Punctul B se obţine prin rotaţia de centru 'A şi unghi (180 2 )α° − a punctului C, deci

' ( ')b a k c a= + − , unde cos( 2 ) sin( 2 )k iπ α π α= − + − şi de aici rezultă ' .1

b kca

k

−=

Analog obţinem relaţiile : '1

c kab

k

−=

− şi '

1

a kbc

k

−=

−. Afixul centrului de greutate al

triunghiului ' ' 'A B C este egal cu:' ' '

'3 3

a b c a b cg g

+ + + += = = , deci ' .G G≡

Demonstraţia teoremei rezultă din lema de mai sus.

I.58. Cercul lui Van Lamoen „Cu intuiţia descoperi, cu logica stabileşti.” J. Hadamard84

Fie a b cM M M triunghiul median al triunghiului ABC, G centrul de greutate al triunghiului

ABC, , , , , ,A A B B C C+ − + − + − centrele cercurilor circumscrise triunghiurilor

, , , ,b c c aGCM GM B GAM GM C ,aGBM respectiv .bGM A şi P mijlocul segmentului C C+ −

(Fig. 230). 1) Fie D B C C A+ − + −= ∩ şi .E A C C B+ − + −= ∩ Patrulaterul DC EC+ − este

paralelogram. Demonstraţie. Deoarece DC− este mediatoarea segmentului AG rezultă aDC AM− ⊥ şi

C E+ este mediatoarea segmentului aGM rezultă ,aC E AM+ ⊥ deci .DC C E− + Analog

se arată că ,DC C E+ − deci patrulaterul DC EC+ − este paralelogram.

84 J. Hadamard (1865-1963) – academician francez , profesor la Collége de France, contribuţii importante în analiză şi geometrie

Page 227: Barbu_TEOREME_FUNDAMENTALE(2008).pdf

234

2) Punctele D, P, G şi E sunt coliniare. Demonstraţie. Fie 1 2 1 2, , ,A A B B mijloacele segmentelor , , ,aAG GM BG respectiv .bGM

Atunci 1 1 2 2, ,a b a bA B AB M M AB A B M M ( 1 1, a bA B M M şi 2 2A A fiind linii mijlocii), de

unde rezultă 1 1 2 2 ,A B A B deci triunghiurile 1 1DAB şi 2 2EA B sunt omotetice,

1 2 1 2 G B B A A= ∩ fiind centrul omotetiei. Atunci, punctele D, G şi E sunt coliniare.

Observaţie: Dreapta DG trece prin mijlocul segmentului .C C+ −

3) Punctul D este centrul cercului circumscris triunghiului ABG. Demonstraţie. Deoarece 1DA şi 1DB sunt mediatoarele segmentelor AG, respectiv BG

rezultă concluzia.

Observaţie: Punctul D aparţine mediatoarei segmentului AB. 4) Dreapta cM D este simediana unghiului .C DC+ −

Demonstraţie. Deoarece AB C C+ − şi cDM AB⊥ rezultă .cDM C C+ −⊥ Deoarece

1 1,GA DC GB DC− +⊥ ⊥ şi cDM C C+ −⊥ rezultă că dreptele DG şi cDM sunt izogonale

(vezi „Drepte izogonale”). Cum DP este mediană în triunghiul DC C+ − rezultă că cDM

este simediana corespunzătoare .C DC+ −

5) Dreapta cDM trece prin mijlocul segmentului .A B− +

Demonstraţie. Fie 1C şi

2C cercurile circumscrise triunghiurilor ,cGM B respectiv

.cGAM Fie .cQ DM A B− += ∩ Fie 'A A AB− ⊥ şi 'B B AB− ⊥ , deci ' ' .A A B B− +

Deoarece triunghiurile cBM A− şi cAM B+ sunt isoscele rezultă ' ' cBA A M≡ şi

' ' ;cM B B A≡ cum c cBM M A≡ rezultă ' '.c cA M M B≡ Deoarece cM Q AB⊥ rezultă

' ,cM Q A A− deci cM Q este linie mijlocie în trapezul ' 'A A B B− + adică Q este mijlocul

segmentului .A B− +

A

B C aM

bM

cM G

Fig. 230 A+

A−

B+ B−

C+

C−

P

1B

2B 1A

2A

D E

Page 228: Barbu_TEOREME_FUNDAMENTALE(2008).pdf

235

Observaţie: Deoarece cDM este simediană în triunghiul DC C+ − şi Q este mijlocul

segmentului A B− + rezultă că A B− + este antiparalelă dreptei C C+ − (vezi „Simediane”).

6) Punctele , ,A B C− + + şi C− sunt conciclice.

Demostraţie. Cum A B− + este antiparalela dreptei C C+ − rezultă că patrulaterul A B C C− + − +

este inscriptibil, deci punctele , , ,A B C C− + − + aparţin unui cerc ( )α

7) Punctele , , ,B C A A− + + − respectiv , , ,C A B B− + + − sunt conciclice. Demonstraţie analoagă cu precedenta. Fie ( )β şi ( )γ cercurile circumscrise patrulaterelor − + + −B A C A , respectiv .C B A A− − + −

Teorema lui Van Lamoen

8) Punctele , , , , ,A A B B C C+ − + − + − sunt conciclice.

Demonstraţie. Deoarece axele radicale ,A C A B− + + − şi B C+ − ale perechilor de cercuri

considerate mai sus nu sunt concurente, rezultă că cercurile ( ), ( )α β şi ( )γ coincid, deci

punctele , , , , ,A A B B C C+ − + − + − sunt conciclice.

Observaţie: Cercul ce trece prin punctele , , , , ,A A B B C C+ − + − + − se numeşte cercul lui Van Lamoen.

I.59. Cercul lui Conway

„Se desenează pe nisip un cerc după care se taie în două,

cu acelaşi băţ de alun se taie în două. După aceea se cade în genunchi,

după aceea se cade în brânci. După aceea se izbeşte cu fruntea nisipul

şi i se cere iertare cercului. Atât.” – Nichita Stănescu 85

1) În prelungirea laturilor triunghiului ABC se construiesc segmentele

1 2 1 2 1 2, , .AA AA BC BB BB AC CC CC AB≡ ≡ ≡ ≡ ≡ ≡ Punctele 1 2 1 2 1 2, , , , ,A A B B C C sunt

conciclice. Demonstraţie. Fie ( ) , ( ) , ( )= = = m A m B m Cα β γ şi a, b, c lungimile laturilor BC, CA

respectiv AB. Deoarece 2 1BC BA a c= = + rezultă că triunghiul 1 2BAC este isoscel, deci

2 1

180( ) .

2

° −=m BC A

β Deoarece 2 1CA CB b c= = + rezultă că triunghiul 2CBA este

isoscel, deci 2 1

180( ) .

2

° −=m CA B

γ Triunghiul 2 1AA A fiind isoscel rezultă

2 1

180( ) .

2

° −=m AA A

α Atunci, 1 2 1 1 2 1( ) ( )m B C A m B A A+ =

85 Nichita Stănescu (1933 – 1983) – eseist, poet român, ales postum membru al Academiei Române

Page 229: Barbu_TEOREME_FUNDAMENTALE(2008).pdf

236

180 180 180180

2 2 2

β α γ°− °− °−+ + = ° , deoarece 180+ + = °α β λ , deci patrulaterul

1 2 1 2BC A A este inscriptibil. Analog, se arată că patrulaterul 2 1 2 1A B B C este inscriptibil, deci

punctele 1 2 1 2 1 2, , , , ,A A B B C C sunt conciclice (Fig. 231).

Observaţie: Cercul pe care se află punctele 1 2 1 2 1 2, , , , ,A A B B C C se numeşte cercul lui Conway corespunzător triunghiului ABC. 2) Centrul cercului lui Conway este punctul I – centrul cercului înscris în triunghiul ABC.

Demonstraţie. Deoarece triunghiurile 2 1AB C şi 1 2A BC sunt isoscele rezultă că bisectoarele

AI, respectiv BI sunt şi mediatoarele segmentelor 2 1,B C respectiv 1 2AC deci I – centrul

cercului înscris în triunghiul ABC – este centrul cercului Conway corespunzător triunghiului ABC. 3) Sunt adevărate relaţiile: 1 2 2 1 1 2 2 1 1 2 2 1, , .A A B C B B C A C C A B

Demonstraţie. Deoarece 1 2AI A A⊥ şi 2 1AI B C⊥ rezultă 1 2 2 1.A A B C Analog

1 2 2 1,B B C A 1 2 2 1.C C A B

4) Dacă r este raza cercului înscris în triunghiul ABC şi p semiperimetrul triunghiului

ABC, atunci raza cercului Conway este egală cu 2 2 .r p+

B'

A

B C aC

bC cC

1A 2A

A'

1B

2B

C'

2C

1C

I

Fig. 231

a a

a b

b

b c

c c

r

D

Page 230: Barbu_TEOREME_FUNDAMENTALE(2008).pdf

237

Demonstraţie. În triunghiul isoscel 2 1B IA ( 1 2 CIA IB R= = - raza Conway), fie P proiecţia

lui I pe 1 2 ,A B deci .IP r= Avem: 2 2 22 2IB IP PB= + adică 2 2 2 ,

CR r p= + de unde

2 2CR r p= + (deoarece 2 1 2

1 1( )

2 2PB A B a b c p= = + + = ).

5) Intersecţiile dreptelor 1 2 1 2 1 2, ,A A B B C C determină un triunghi omotetic cu triunghiul

de contact a b cC C C al triunghiului ABC, centrul de omotetie fiind punctul lui Gergonne

al triunghiului .ABC Demonstraţie. Fie 1 2 1 2 1 2 1 2 1 2 1 2 ' , ' , ' .= ∩ = ∩ = ∩A B B C C B A A C C C A A B B Deoarece

triunghiul a bAC C este isoscel rezultă că 180 ( )

( ) ,2

° −=

c b

m Am AC C deci

2 1 ,c bA A A AC C≡ de unde 1 2 .c bA A C C Analog se arată că 1 2 a cB B C C şi 1 2 ,a bC C C C

deci triunghiurile a b cC C C şi ' ' 'A B C sunt omotetice. Fie 2 1 .= ∩aD AC B C În triunghiul

2 1,AB C considerând ceviana AD şi secanta BC avem: 1 2

2 1

1a

a

C CAC DBAB

AB AC C B DC⋅ ⋅ ⋅ = (vezi

„Relaţia lui Van Aubel”) de unde 2

1

( ) (1)

( )

DB p b b

DC p c c

− ⋅=

− ⋅ (am ţinut cont că

2 1 , aAB AC b c C C p c= = + = − şi aC B p b= − ). Fie 1 2 1 ' .= ∩aD A C B C Avem:

12 2 1 1

1 1 1 1 1

' '1 (2).

' 'a

a

C BA B A C DC

A B A C D B C C⋅ ⋅ ⋅ = Din teorema sinusurilor aplicată triunghiurilor a b cC C C

şi 1 2'A BC rezultă 1 2' '

sin(90 ) sin(90 )2 2

=°− °−

A B A C

C B şi

sin sina c a b

a b c a c b

C C C C

C C C C C C=

de unde:

1

2

cos' 2' cos

2

CA B

BA C= şi

cos2 (3).

cos2

a c

a b

BC C

CC C= Deoarece 1 1 ' 'a bB C C C B C rezultă

2

1

' ' ' (4).

' ' 'a c

a b

C CA B A C

A C A B C C= = Din relaţiile (2), (3) şi (4) rezultă: 12 2 1 2

1 1 2 1 1

' '

' '⋅ ⋅ =a

a

C BA B A C D B

A C A B C C DC

adică 1 2

1 1

cos cos2 2

cos cos2 2

⋅ ⋅ =

B BD Bp

C C p DC şi de aici

2

1 2

1 2

( )cos ( )2 (5).

( ) ( )cos2

p p bBD B p b bac

C p p cDC p c c

ab

− −

= = = − −

Din relaţiile (1) şi (5) rezultă 2 1 2

1 1 2

,DB D B

DC DC= adică 1.D D≡ Deci, punctele , ,aA C D şi 'A

sunt coliniare. Analog, se arată că , bB C şi 'B respectiv , cC C şi 'C sunt coliniare, deci

centrul de omotetie este punctul Γ de intersecţie al dreptelor ,a bAC BC şi cCC - adică

punctul lui Gergonne. 6) Intersecţiile dreptelor 1 2 1 2 1 2, ,A A B B C C determină un triunghi omologic cu triunghiul ABC, centrul de omologie fiind punctul lui Gergonne al triunghiului ABC. Demonstraţie. Proprietatea este o consecinţă a aplicaţiei precedente.

Page 231: Barbu_TEOREME_FUNDAMENTALE(2008).pdf

238

I.60. Cercul lui Adams86

„Gândirea este o pasăre a înălţimilor care, în colivia cuvintelor, izbuteşte doar să-şi desfăşoare aripile, dar nu poate zbura.”

Kahlil Gibran87

Teorema lui Adams

Fie , ,a b cC C C punctele de tangenţă ale cercului înscris în triunghiul ABC cu laturile

BC,CA, respectiv AB. Dreptele , ,a b cAC BC CC sunt concurente în punctul Γ ( punctul lui

Gergonne). Prin punctul Γ se duc paralele la laturile triunghiului a b cC C C care

intersecteaza laturile triunghiului ABC în punctele , , , , ,P Q R S T U . Punctele

, , , , ,P Q R S T U aparţin unui cerc concentric cu cercul înscris în triunghiul ABC.

Demonstraţie. Fie I centrul cercului înscris în triunghiul ABC. Vom arăta că punctele , , , , ,P Q R S T U se află la aceeaşi distanţa faţă de I.

Cum , ,a b cC C C sunt punctele podare ale lui I rămâne

să demonstrăm că segmentele a a, ,C P C Q

b b c c, , ,C R C S C T C U au aceeaşi lungime (Fig.232).

Cum bAC şi cAC sunt tangente la cercul înscris

rezultă b cAC AC≡ , adică triunghiul b cAC C este

isoscel. Cum b cC C UR rezultă UA RA≡ , adică

c bC U C R≡ ; analog se arată că a bC Q C T≡ şi

a bC P C S≡ . Ducem prin A o paralelă d la BC şi fie

, , , .a b c aX C C d Y C C d Z PS d W TQ d= ∩ = ∩ = ∩ = ∩ Atunci, triunghiurile

bAC X şi a bC C C sunt asemenea, de unde rezultă că bAC AX≡ şi analog se arată că

cAY AC≡ . Cum aC Y WΓ şi aC X ZΓ rezultă AW AZ≡ , de unde WY ZX≡ , sau

a aC Q C P≡ ceea ce completează demonstraţia.

Cercul circumscris celor şase puncte se numeşte cercul lui Adams corespunzător triunghiului ABC. 1) Centrul cercului Adams este centrul cercului înscris în triunghiul ABC. Demonstraţie: Din congruenţa triunghiurilor , , , , ,a a b b c cIC P IC Q IC R IC S IC U IC T

rezultă IP IQ IR IS IT IU≡ ≡ ≡ ≡ ≡ , deci centrul cercului înscris în triunghiul ABC este

centrul cercului lui Adams. 2) Sunt adevărate relaţiile: , ,UR AI TQ BI PS CI⊥ ⊥ ⊥ .

Demonstraţie: Deoarece triunghiul AUR este isoscel, iar dreptele AR şi AU sunt tangente crecului înscris în triunghiul ABC, rezultă că UR AI⊥ ; analog se arată că

, .TQ BI PS CI⊥ ⊥

86 Edwin Adams (1878-1956) - matematician ceh, profesor la Princeton, contribuţii în geometrieai 87 Kahlil Gibran (1883-1931) – poet libanez

aC

bC

cC

A

B C

X

P Q

R

S T

U Γ

Fig. 232

Z Y W d

Page 232: Barbu_TEOREME_FUNDAMENTALE(2008).pdf

239

CAPITOLUL II

TEOREME FUNDAMENTALE DIN GEOMETRIA TRIUNGHIULUI

II.1. Teorema bisectoarei interioare

„Teorema este mai presus de constatare şi mai presus de greşeală.”-Gh. Ţiţeica89

Teorema bisectoarei Fie triunghiul ABC şi AD , ( )D BC∈ bisectoarea unghiului BAC . Atunci, .=

BD AB

DC AC

Demonstraţie.

Fie CE AD , ∈E AB (Fig. 233 ). Atunci ≡ACE DAC ( unghiuri alterne interne) şi ≡BAD CEA . Cum ≡BAD DAC , rezultă ≡ACE AEC , adică triunghiul ACE este isoscel,

deci ≡AC AE . Din teorema lui Thales rezultă: .= =BD AB AB

DC AE AC

Reciproca teoremei bisectoarei interioare

În triunghiul ABC, fie ( )∈D BC astfel încât ,DB AB

DC AC= atunci (AD este bisectoarea

interioară a unghiului .BAC

Demonstraţie. Fie , .CE AD E AB∈ Din teorema lui Thales în triunghiul BCE rezultă

BD AB

DC AE= , iar cu relaţia din ipoteză

DB AB

DC AC= obţinem ,AE AC= adică triunghiul AEC

este isoscel, deci ≡ AEC ACE (1). Cum AD CE rezultă DAC ACE≡ (2) (unghiuri alterne interne) şi BAD AEC≡ (3) (unghiuri corespondente). Din relaţiile (1), (2) şi (3) rezultă ,BAD DAC≡ adică AD este bisectoarea unghiului .BAC

89 Gheorghe Ţiţeica (1873-1939) – matematician român, profesor la Universitatea din Bucureşti, membru al

Academiei Române, contribuţii importante în geometrie

Fig. 233

A

B C D

E

Page 233: Barbu_TEOREME_FUNDAMENTALE(2008).pdf

240

1) Segmentele determinate pe latura BC de bisectoarea AD au lungimea egală cu

+

ac

b c, respectiv

+

ab

b c.

Demonstraţie. Din teorema bisectoarei avem =BD c

DC b, sau = =

+

BD CD a

c b b c, de unde

=+

acBD

b c, şi =

+

abCD

b c.

2) În triunghiul ABC, fie D piciorul bisectoarei interioare a unghiului A, ( )D BC∈ .

Atunci 2

cos2a

bc Al

b c=

+, unde cu

al am notat lungimea segmentului AD.

Demonstraţie. Soluţia1. Din [ ] [ ] [ ]ABD ADC ABCA A A+ =

rezultă 1 1

sin sin2 2 2 2a a

A Ac l b l⋅ ⋅ + ⋅ ⋅ =

1sin

2bc A , adică

2cos

2a

bc Al

b c=

+.

Soluţia 2. Din teorema bisectoarei avem :

= ⇒ = ⇔ =+ + +

BD c BD c acBD

DC b BD DC b c b c şi

.=+

abDC

b c Teorema sinusurilor aplicată în ABD ne

dă: sin sin

2

=al BD

AB de unde

sin.

sin2

+=a

acB

b clA

Dar ,sin sin

a b

A B= de unde:

2cos .

2a

bc Al

b c= ⋅

+

Analog, se obţin lungimile celorlalte bisectoare interioare: 2

cos2b

ac Bl

a c= ⋅

+ şi

2cos .

2c

ab Cl

a b= ⋅

+

II.2. Teorema bisectoarei exterioare

„Toate invenţiile unui om sunt adevărate, poţi fi sigur de asta. Poezia este atât ştiinţă cât şi geometrie.” – Gustave Flaubert (1821-1880)

Teorema bisectoarei exterioare

Fie triunghiul ABC şi .AB AC≠ Dacă (AE este bisectoarea exterioară a unghiului A,

,E BC∈ atunci .EB AB

EC AC=

Demonstraţie. Fie ,b c> deci

( ).∈B EC Paralela prin B la AE

intersectează latura AB în 1B (Fig. 235). Din

teorema lui Thales rezultă 1ABEB

EC AC= (1).

A

B D C

Fig. 234

A

B C E

T

1B

Fig. 235

Page 234: Barbu_TEOREME_FUNDAMENTALE(2008).pdf

241

Dar 1TAE AB B≡ (unghiuri corespondente) şi 1EAB ABB≡ (unghiuri alterne

interne), deci 1 1,AB B ABB≡ adică triunghiul 1ABB este isoscel, de unde 1AB AB= (2).

Din relaţiile (1) şi (2) rezultă .EB AB

EC AC=

Observaţii:

1) Condiţia AB AC≠ din teoremă este esenţială pentru că dacă ,AB AC= atunci bisectoarea exterioară a unghiului A este paralelă cu BC, deci nu ar mai exista punctul E.

2) Din teorema bisectoarei EB c

EC b= (presupunând b c> ) rezultă

c EB EB

b c EC EB a= =

− −,

adică ac

EBb c

=−

şi analog .ab

ECb c

=−

Reciproca teoremei bisectoarei exterioare

Fie triunghiul ABC şi \ [ ]∈E BC BC astfel încât ,EB AB

EC AC= atunci (AE este

bisectoarea exterioară a unghiului A. Demonstraţie. Evident ,AB AC≠ deoarece astfel ar rezulta =EB EC ceea ce este imposibil

datorită faptului că \ [ ]∈E BC BC . Fie 1 1, .BB AE B AC∈ Din teorema lui Thales în

triunghiul EAC rezultă 1ABEB

EC AC= , care cu relaţia din ipoteză dă 1,AB AB= adică

triunghiul 1ABB este isoscel, de unde obţinem că 1 1.AB B ABB≡ Din 1AE BB rezultă

1TAE AB B≡ (unghiuri corespondente) şi 1≡ EAB ABB (unghiuri alterne interne) şi

de aici obţinem că ,TAE EAB≡ adică (AE este bisectoarea exterioară a unghiului A. 1) Segmentele determinate pe dreapta BC de bisectoarea exterioară a unghiului A au

lungimile egale cu −

ac

b c, respectiv

ab

b c.

Demonstraţie. Din teprema bisectoarei exterioare avem: ,=EB c

EC b sau = =

EB EC a

c b b c

(unde am considerat >b c ).

2) Fie ',',' CBA picioarele bisectoarelor exterioare ale unghiurilor triunghiului isoscel .ABC Punctele

',',' CBA sunt coliniare. Demonstraţie: Din teorema bisectoarei obţinem:

',

'=

A B AB

A C AC

'

'=

B C BC

B A BA şi .

'

'

CB

CA

BC

AC= Avem:

1'

'

'

'

'

'=⋅⋅=⋅⋅

CB

CA

BA

BC

AC

AB

BC

AC

AB

CB

CA

BA şi din reciproca

teoremei lui Menelaus rezultă că punctele ',',' CBA sunt coliniare.

C'

B'

A'

A

C B

Fig. 236

Page 235: Barbu_TEOREME_FUNDAMENTALE(2008).pdf

242

Observaţie: Teorema de mai sus aparţine geometrului grec Pappus 90. 3) În triunghiul ABC, fie 'D piciorul bisectoarei exterioare a unghiului A, ' (D CB∈ .

Atunci ' 2sin

2a

bc Al

b c=

−, unde cu '

al am notat lungimea segmentului 'AD .

Demonstraţie. Deoarece:

[ ' ] [ ' ] [ ]AD C AD B A B CA A A− = rezultă

'sin 90 'cos sin2 2

A Ab l c l bc A

⋅ ° + − ⋅ =

adică 2

' sin2a

bc Al

b c=

−.

II.3. Teorema lui Pitagora91 „După ce a descoperit celebra sa teoremă, Pitagora a sacrificat o sută de boi. De atunci, de fiecare dată, când se descoperă vreun adevăr nou, vitele cornute mari au palpitaţii.” - Ludwig Björne

Într-un triunghi dreptunghic pătratul lungimii ipotenuzei este egal cu suma pătratelor lungimilor catetelor.

Demonstraţia 1. În triunghiul dreptunghic ABC ( ( ) 90m BAC = ° ) fie

înălţimea AD , ( )D BC∈ (Fig. 238). Din asemănarea triunghiurilor

ABD şi CBA rezultă =AB BDBC AB

şi de aici 2 = ⋅AB BC BD (1), iar din

asemănarea triunghiurilor ADC şi BAC rezultă =AC DC

BD AC, de unde

2 = ⋅AC BC DC (2). Din relaţiile (1) şi (2) rezultă :

2 2 2( )+ = = ⋅ =+AB AC BC BC BC BCBD DC .

Demonstraţia 2. Pe ipotenuza BC se construieşte pătratul CBNQ (Fig. 239). În prelungirea catetelor AB şi AC se construieşte pătratul AMPR având latura de lungime b+c. Atunci,

[ ] [ ] [ ]4AMPR BCQN ABCA A A= + sau 2 2( ) 4

2+ = + ⋅

bcb c a

de unde rezultă 2 2 2a b c= + .

90 Pappus (290 – 350) – matematician şi filosof grec; a pus bazele geometriei proiective 91 Pitagora (Pythagoras) (c. 560 – c. 500 î. Hr.) – matematician, om politic şi filosof grec

A

B C

D D'

Fig. 237

A B

D

C

Fig. 238

A

C

B M

N

Q P R

b

c

c

c c

b

b

b

a

a

a

a

Fig. 239

Page 236: Barbu_TEOREME_FUNDAMENTALE(2008).pdf

243

II.4. Teorema lui Pitagora generalizată

„Când apa frânge o vargă, raţiunea o îndreaptă Raţiunea mi-e stăpână înţeleaptă Şi astfel, ochi-mi, ajutaţi de gând,

Nici nu mă înşeală, deşi mă mint oricând.” La Fontaine92

Fie triunghiul ABC şi D proiecţia punctului A pe dreapta BC. Dacă

( ) 90 ,m ACB < ° atunci 2 2 2 2 .AB CA CB CB CD= + − ⋅ Dacă ( ) 90 ,m ACB > ° atunci 2 2 2 2 .AB CA CB CB CD= + + ⋅

Demonstraţie.

Din triunghiurile dreptunghice ABD şi ACD (Fig. 240) rezultă

2 2 2 2 2 2, .AB BD AD AC AD DC= + = + Dacă ( ) 90 ,m ABC < ° atunci

( )D BC∈ şi .BD BC CD= − Dacă ( ) 90 ,m ABC > ° atunci ( )B CD∈ şi BD DC BC= − ,

deci 2 2 2 2 2 2( ) ( ) 2AB AD BC CD AD CD BC BC CD= + − = + + − ⋅ , de unde rezultă 2 2 2 2 .AB CA CB CB CD= + − ⋅ Dacă ( ) 90 ,m ACB > ° atunci BD BC CD= + (Fig. 241).

Avem: 2 2 2 2 2 2 2 2( ) ( ) 2 2 .AB AD BC CD AD DC BC BC CD CA CB CB CD= + + = + + + ⋅ = + + ⋅

1) Consecinţă: Teorema cosinusului În orice triunghi ABC, având laturile de lungimi a, b, c, au loc relaţiile: i)

2 2 2 2 cos ,a b c bc A= + −

ii) 2 2 2 2 cos ,b a c ac B= + −

iii) 2 2 2 2 cos .c a b ab C= + −

Demonstraţie.

i) Fie D proiecţia lui A pe BC. Dacă ( ) 90 ,m ACB < ° atunci

din teorema lui Pitagora generalizată avem: 2 2 2 2c a b a CD= + − ⋅ . Cum cosCD b C= rezultă 2 2 2 2 cos .c a b ab C= + − Dacă ( ) 90m ACB = ° atunci 2 2 2c a b= + , adică teorema lui Pitagora. Dacă ( ) 90 ,m ACB > ° atunci 2 2 2 2 ,c a b a CD= + − ⋅

92 La Fontaine (1621-1695) – poet, dramaturg francez

A

B C D

Fig. 240

A

B C D

Fig. 241

A

B C D

Fig. 242

Page 237: Barbu_TEOREME_FUNDAMENTALE(2008).pdf

244

cos(180 ) cosCD CA C b C= °− = − adică 2 2 2 2 cos .c a b ab C= + −

ii) şi iii) se demonstrează analog cu i). Observaţii:

1) În orice triunghi ABC, 2 2 2

cos2

b c aA

bc

+ −= ,

2 2 2

cos2

a c bB

ac

+ −= ,

2 2 2

cos .2

b a cC

ba

+ −=

2) i) Dacă ( ) 90m BAC < ° , atunci 2 2 2cos 0A a b c> ⇔ < + .

ii) Dacă ( ) 90m BAC = ° , atunci 2 2 2cos 0A a b c= ⇔ = + .

iii) Dacă ( ) 90m BAC > ° , atunci 2 2 2cos 0 .A a b c< ⇔ > +

2) Teorema lui Pappus. Formula medianei Dacă M este mijlocul laturii BC a triunghiului ABC atunci:

2 2 2 22( ).AB AC AM BM+ = +

Demonstraţie. Teorema cosinusului aplicată în triunghiurile ABM şi AMC dă:

2 2 2 2 cos (1)AB BM AM BM AM AMB= + − ⋅ ⋅ 2 2 2 2 cos( )= + − ⋅ ⋅ − =AC MC AM AM MC AMBπ 2 2+AM +2AM MC cos AMB (2)⋅ ⋅ MC . Din relaţiile

(1) şi (2) prin sumare obţinem: 2 2 2 22( )AB AC AM BM+ = + , unde am ţinut cont că

.BM MC= Observaţii:

1) Expresia 2 2 2 22( )+ = +AB AC AM BM se numeşte relaţia lui Pappus.

2) Dacă a,b,c sunt lungimile laturilor triunghiului ABC iaram lungimea medianei AM,

relaţia lui Pappus devine: 2 2 2

2 2( )

4

+ −=a

b c am ( Formula medianei ).

3) Prin permutări circulare ale relaţiei precedente se obţin următoarele egalităţi: 2 2 2

2 2( )

4

+ −=b

c a bm ,

2 2 22 2( )

4

+ −=c

b a cm .

4) Teorema lui Pappus ne oferă un mod de a determina lungimile medianelor în funcţie de lungimile laturilor triunghiului. 3) Consecinţă: Dacă a, b, c sunt lungimile laturilor unui triunghi ABC şi am , bm , cm sunt lungimile medianelor triunghiului ABC, atunci: i)

2 2 2 2 2 24( ) 3( )+ + = + +a b cm m m a b c , ii)

4 4 4 4 4 416( ) 9( )+ + = + +a b cm m m a b c ( Relaţia lui Cesaro)

Demonstraţia se realizează înlocuind formula medianei în relaţiile date.

A

B C

M

Fig. 243

Page 238: Barbu_TEOREME_FUNDAMENTALE(2008).pdf

245

II.5. Teorema lui Stewart93

„Geometria este cea mai bună şi mai simplă dintre toate logicile, cea mai potrivită să dea inflexibilitate judecăţii şi raţiunii.” – Denis Diderot94

Fie triunghiul ABC şi M un punct pe latura BC. Atunci:

2 2 2 .AB MC AC BM AM BC BC BM MC⋅ + ⋅ − ⋅ = ⋅ ⋅

Demonstraţie.

Aplicând teorema cosinusului în triunghiurile ABM şi AMC obţinem:

2 2 2

2 2 2

2 cos

2 cos .

AB AM BM AM BM AMB

AC AM MC AM MC AMC

= + − ⋅ ⋅

= + − ⋅ ⋅

Cum cos( ) cos(180 ) cos ,AMC AMB AMB= °− = −

rezultă: 2 2 2

2 2 2

2 cos

2 cos .

AB MC AM MC BM MC AM BM CM AMB

AC MB AM MB CM MB AM BM CM AMB

⋅ = ⋅ + ⋅ − ⋅ ⋅ ⋅

⋅ = ⋅ + ⋅ + ⋅ ⋅ ⋅

Sumând egalităţile precedente obţinem: 2 2 2( ) ( )AB MC AC BM AM MC MB BM MC MB MC⋅ + ⋅ = + + ⋅ +

adică 2 2 2 .AB MC AC BM AM BC BM MC BC⋅ + ⋅ = ⋅ + ⋅ ⋅ Consecinţe: 1) Teorema medianei

Fie M mijlocul laturii BC a triunghiului ABC. Atunci, 2 2 2

2 2( )

4a

b c am

+ −= (unde am

reprezintă lungimea medianei AM).

Demonstraţie. Avem .2

aBM MC= = Din relaţia lui Stewart aplicată în triunghiul ABC şi

punctului M rezultă 2 2 2

2 2( ).

4a

b c am

+ −=

2) Lungimea bisectoarei interioare Fie triunghiul ABC, ( AD bisectoarea interioară a unghiului ,BAC unde D BC∈ ).

Atunci 22

4( )

( )

bcAD p p a

b c= −

+, unde p este semiperimetrul triunghiului ABC.

Demonstraţie. Din teorema bisectoarei rezultă ,c BD

b DC= de

unde c b BD DC

b DC

+ +=

abDC

b c=

+ (1) şi

acBD

b c=

+ (2)

(Fig. 245). Teorema lui Stewart în ABC pentru M D≡ dă: 2 2 2 2AD a c DC b BD a DB DC⋅ = ⋅ + ⋅ − ⋅ ⋅ (3). Din relaţiile (1),

93 Matthew Stewart (1714-1785) – geometru scoţian, profesor la Universitatea din Edinburgh 94 Denis Diderot (1713-1784) – scriitor şi filosof francez

A

B C M

Fig. 244

A

B C D

Fig. 245

Page 239: Barbu_TEOREME_FUNDAMENTALE(2008).pdf

246

(2) şi (3) rezultă 22

4( )

( )a

bcl p p a

b c= −

+ (unde prin al am notat lungimea bisectoarei AD).

3) Lungimea bisetoarei exterioare Fie triunghiul ABC, (AE bisectoarea exterioară a unghiului A, ).E BC∈ Atunci,

22

4 ( )( ).

( )

bc p b p cAE

b c

− −=

Demonstraţie.Fie ,b c> deci ( )B EC∈ (Fig. 246). Din teorema lui Stewart aplicată în

triunghiul AEC rezultă: 2 2 2 ( ).AE BC AC EB AB EC AB EB BC⋅ + ⋅ − ⋅ = ⋅ ⋅ ∗ Din teorema

bisectoarei exterioare avem: c EB

b EC= de unde rezultă:

c EB EB

b c EC EB a= =

− − şi

acEB

b c=

−; analog .

abEC

b c=

Relaţia ( )∗ devine

2 2 2ac ab acAE a b c c a

b c b c b c⋅ + ⋅ − ⋅ = ⋅ ⋅

− − −

de unde rezultă: 22

4 ( )( ),

( )

bc p b p cAE

b c

− −=

− unde

2

a b cp

+ += .

II.6. Teorema sinusurilor

„Fiecare problemă pe care am rezolvat-o a devenit o regulă care pe urmă mi-a servit la rezolvarea altor probleme.” – René Descartes95

În orice triunghi ABC, raza R a cercului circumscris verifică egalitatea:

2 .sin sin sin

a b cR

A B C= = =

Demonstraţie. Vom demonstra teorema pentru cele trei cazuri date de natura triunghiului .ABC i) Triunghiul ABC este ascuţitunghic (Fig. 247). Fie diametrul BD. Atunci BCD este dreptunghic .

Avem: 1( ) ( ) ( )

2m BAC m BDC m BXC= = de unde

sin( ) sin( )BAC BDC= = .2

BC a

BD R= Analog, avem :

sin( )2

bABC

R= şi sin( ) .

2

cACB

R=

ii) Triunghiul ABC este dreptunghic. Fie

95René Descartes (1596-1650) – matematician şi filosof francez, contribuţii în geometrie

A

B C E

Fig. 246

A

B

C

D

• X

Fig. 247

Page 240: Barbu_TEOREME_FUNDAMENTALE(2008).pdf

247

( ) 90m BAC = ° . Avem sin( ) 1, sin( )b

BAC ABCa

= = şi sin( ) .c

BCAa

= Cum 2a R=

concluzia este evidentă. iii) Triunghiul ABC este obtuzunghic (Fig. 248).

Fie sin( ) 90 .BAC > ° În triunghiul BCD

( ( ) 90 )m BCD = ° , avem: sin( ) .2

aBDC

R=

Deoarece patrulaterul ABCD este inscriptibil

rezultă ( ) ( ) 180 ,m BAC m BDC+ = ° deci

sin( / 2 ) sin( ) .2

aBAC BAC

Rπ − = = Pentru unghiurile

ascuţite ABC şi BCA se repetă demonstraţia de la subpunctul i).

II.7. Teorema lui Ceva96

„Geometria este ştiinţa care restaurează situaţia dinainte de creaţia lumii şi încearcă să umple "golul", renunţând la oficiile materiei.” - L. Blaga97

Teorema lui Ceva

Fie triunghiul ABC şi punctele D BC∈ , E CA∈ , F AB∈ . Dacă dreptele AD, BE şi CF

sunt concurente, atunci 1AF BD CE

FB DC EA⋅ ⋅ = .

Demonstraţie: Fie K AD BE CF= ∩ ∩ .

Prin A ducem o paralelă la BC, iar G şi F sunt punctele de intersecţie dintre dreptele BE respectiv CF cu această paralelă. Din

AHF BCF∼ rezultă AF AH

FB BC= (1),

BCE AEG∼ rezultă CE BC

EA AG= (2),

AGK BDK∼ rezultă AG AK

BD DK= (3),

CDK AHK∼ rezultă AH AK

DC DK= (4). Din

relaţiile (3) şi (4) obţinem AG AH

BD DC= de unde

AG BD

AH DC= (5). Din relaţiile (1) , (2) şi (5)

rezultă 1AF BD CE AH AG BC

FB DC EA BC AH AG=⋅ ⋅ = ⋅ ⋅

96 Giovanni Ceva (1647-1734) – matematician italian, profesor la Universitatea din Mantua, contribuţii în geometrie 97Lucian Blaga (1895-1961) - filozof, umanist, jurnalist, poet, dramaturg, traducător, profesor universitar şi

diplomat român, membru titular al Academiei Române

A

B

C

Fig. 248

O D

A

B C D

E F

G H

K

Fig. 249

Page 241: Barbu_TEOREME_FUNDAMENTALE(2008).pdf

248

Reciproca teoremei lui Ceva

Fie triunghiul ABC şi punctele D BC∈ , E CA∈ , F AB∈ . Dacă 1AF BD CE

FB DC EA⋅ ⋅ = ,

atunci dreptele AD, BE şi CF sunt concurente. Demonstraţie. Fie K BE CF= ∩ şi 'D AK BC= ∩ . Conform primei părţi rezultă

'1

'

AF BD CE

FB D C EA⋅ ⋅ = care împreună cu relaţia din ipoteză dă :

'

'

BD BD

D C DC= de unde

' '

'

BD D C BD DC

D C DC

+ += deci

'

BC BC

D C DC= şi de aici rezultă că 'D C DC= , adică '.D D≡

Observaţii:

1) Reciproca teoremei lui Ceva este adevărată şi în cazul în care unul din punctele D,E, sau F aparţine unei laturi – de exemplu D BC∈ - şi celelalte două puncte E CA∈ , F AB∈ verifică condiţia: “dreapta BE nu este paralelă cu dreapta CF. 2) Dacă BE CF reciproca teoremei lui Ceva nu mai este adevărată, aşa cum o arată următorul exemplu: „Fie D mijlocul segmentului BC, F simetricul lui B faţă de A şi E simetricul lui C faţă de A. Atunci,

11 2 1

2

AF BD CE

FB DC EA⋅ ⋅ = ⋅ ⋅ = , dar dreptele AD, BE, CF nu

sunt concurente (deoarece AD BE CF , AD fiind linie mijlocie în triunghiurile BEC şi BFC.”

II.8. Teorema lui Menelaus98

„Un punct pierdut e lumea în haosul imens. Toată ştiinţa noastră: cuvinte fără sens. Om, pasăre şi floare sunt umbre în abis.

Zadarnic este gândul, iar existenţa - vis.” Omar Khayyam99

Teorema lui Menelaus Fie triunghiul ABC şi punctele 'A BC∈ , 'B CA∈ , 'C AB∈ . Punctele ', ', 'A B C sunt coliniare dacă şi numai dacă

' ' '1

' ' '

A B B C C A

A C B A C B⋅ ⋅ = .

Demonstraţie. Presupunem că punctele ', ', 'A B C sunt coliniare. Conform axiomei lui Pasch, cel puţin unul din punctele ', ', 'A B C se află pe prelungirea laturilor triunghiului ABC . Fără a restrânge generalitatea putem presupune că

98 Menelaus (70-130) – mathematician grec, contribuţii importante în geometrie 99 Omar Khayyam (1048-1122) – matematician, poet, filosof, astronom persan, contribuţii în algebră şi geometrie

A

B C

Fig. 250

D

E F

A

B C

A1

B1

C1

Fig. 251

A'

B'

C' P

Page 242: Barbu_TEOREME_FUNDAMENTALE(2008).pdf

249

' ( )B AC∈ , ' ( )C AB∈ şi ' [ \ [ ]A CB CB∈ (Fig. 251).

Soluţia 1. Fie 1 ,A 1B , 1C proiecţiile punctelor A, B, C pe dreapta ' '.A B .Din asemănările

triunghiurilor: 1'A BB şi 1'A CC ; 1'B CC şi 1'B AA ; 1'C AA şi 1'C BB rezultă egalităţile

1 1 1

1 1 1

' ' ', ,

' ' '

BB CC AAA B B C C A

A C CC B A AA C B BB= = = care prin înmulţire dau:

' ' '1

' ' '

A B B C C A

A C B A C B⋅ ⋅ = .

Soluţia 2. Egalitatea evidentă [ ' '] [ ' '] [ ' ']

[ ' '] [ ' '] [ ' ']

1AC B BC A CA B

BC A CA B AC B

A A A

A A A⋅ ⋅ = este echivalentă cu:

' ' ' sin ' ' ' ' ' sin ' ' ' ' ' sin ' ' 1,' ' ' sin ' ' ' ' ' sin ' ' ' ' ' sin( ' ' )C AC B AC B A C A B C A B B A B C A B C

C B C A A C B A B A C CA B B A B C A B Cπ⋅ ⋅

⋅ ⋅ ⋅ ⋅ ⋅ ⋅ =⋅ ⋅ ⋅ ⋅ ⋅ ⋅ −

adică

' ' '1.

' ' '

A B B C C A

A C B A C B⋅ ⋅ =

Soluţia 3. Fie ' ' ( )BP A B P AC∈ . Din asemănarea triunghiurilor BPC cu ' 'A B C ,

respectiv a triunghiurilor ' 'AC B cu ABP rezultă: ' '

' '

B P A B

B C A C= şi

' '

' '

B A C A

B P C B= care prin

înmulţire dau concluzia.

Reciproc, presupunem că ' ' '

1' ' '

A B B C C A

A C B A C B⋅ ⋅ = (1) şi demonstrăm că punctele ', ', 'A B C

sunt coliniare. Fie ' [ \ [ ]A CB CB∈ , ' ( )C AB∈ şi " ' 'B A C AC= ∩ . Atunci, conform

primei părţi rezultă: ' " '

1' " '

A B B C C A

A C B A C B⋅ ⋅ = care cu relaţia (1) dă

' "

' "

B C B C

B A B A= şi de aici

' "B C B C

AC AC= ,adică ' "B C B C= şi cum există doar un punct interior laturii AC pentru care

' "B C B C= , rezultă ' "B B≡ , deci punctele ', ', 'A B C sunt coliniare. Soluţia 4. Considerăm cazul când două puncte sunt pe laturi şi unul pe prelungirea unei

laturi Notăm: ' ' '

, , .' ' '

= = =A B B C C A

A C B A C Bα β γ Din

' 1

'=

A C

A B α rezultă

' 1

1=

+A C

BC α, deci

1'

1= ⋅

+

uuuur uuurCA CB

α. Din

'

'=

B C

B Aβ rezultă '

1=

+

uuuur uuurCB CA

ββ

, iar

'' '

1= + = + ⋅ = + ⋅

uuuur uuur uuuur uuur uuur uuur uuurACCC CA AC CA AB CA AB

AB

γγ

. Putem exprima acum vectorii ' 'uuuuurB A

şi ' 'uuuuurB C :

1' ' ' '

1 1= − = −

+ +

uuuuur uuuur uuuur uuur uuurB A CA CB CB CA

βα β

, ' ' ' '1 1

B C CC CB CA AB CAγ β

γ β= − = + − =

− +

uuuuur uuuur uuuur uuur uuur uuur

1 1( )

1 1 (1 )( 1) 1CA AC CB CA CB

γ βγ γβ γ β γ γ

− −+ + = +

+ − + − −

uuur uuur uuur uuur uuur. Din condiţia ca vectorii ' '

uuuuurB A şi ' '

uuuuurB C

să fie coliniari, rezultă 1 (1 )( 1)

(1 ) (1 )(1 )

γ β β γα γ β βγ− + −

=+ + +

, de unde obţinem 1=αβγ sau

' ' '1

' ' '

A B B C C A

A C B A C B⋅ ⋅ = .Analog se tratează cazul când punctele ', ', 'A B C sunt pe prelungirile

laturilor. Pentru demonstraţia afirmaţiei reciproce, fie 1=αβγ şi notând

Page 243: Barbu_TEOREME_FUNDAMENTALE(2008).pdf

250

' ' ' 1, ,

' ' '= = = =A B B C C A

A C C B C Bα β γ

αβ, avem

1' ' ' '

1 1= − = −

+ +

uuuuur uuuur uuuur uuur uuurB A CA CB CB CA

βα β

(1),

1' ' ' '

(1 )( 1) 1

+= − = − +

+ − −

uuuuur uuuur uuuur uuur uuurB C CC CB CA CB

βγ γβ γ γ

şi înlocuind 1

γαβ

= obţinem

1 1' '

1 1 1

+= − − + +

uuuuur uuur uuurB C CB CA

α βαβ β β

(2). Din (1) şi (2) avem ' ' ' '1

=−

uuuuur uuuuurB C B A

ααβ

, deci

', ', 'A B C sunt puncte coliniare. Teorema lui Menelaus pentru patrulatere

Dacă X, Y, Z, W sunt puncte coliniare pe laturile AB, BC, CD, respectiv DA ale

patrulaterului ABCD, atunci 1.AX BY CZ DW

XB YC ZD WA⋅ ⋅ ⋅ =

Demonstraţie.

Fie .T BD XY= ∩ Din teorema lui Menelaus aplicată în triunghiurile ABD şi BCD

rezultă: 1XA WD TB

XB WA TD⋅ ⋅ = şi 1,

TD ZC YB

TB ZD YC⋅ ⋅ = relaţii care prin înmulţire dau concluzia.

II.9. Teorema transversalei

„Matematica nu se face în stare de urgenţă.” – Ion Cucurezeanu100

Teorema transversalei Dacă într-un triunghi ABC se duce o ceviană AD, iar o secantă oarecare intersectează

dreptele AB, AC şi AD în punctele M, N, respectiv P, atunci 1.AM AC PN DB

AB AN PM DC⋅ ⋅ ⋅ =

Demonstraţie. Fie ', ', 'B C M şi 'N proiecţiile punctelor B, C, M, respectiv N pe AD. Din asemănarea triunghiurilor 'AMM şi 'ABB , 'ACC şi 'ANN , 'PNN şi 'PMM , 'BB D şi

100 Ion Cucurezeanu – matematician român, profesor la Universitatea din Constanţa,contribuţii în studiul ecuaţiilor diofantice

A

B C

Y

Fig. 252

W

Z

T

X

D

Page 244: Barbu_TEOREME_FUNDAMENTALE(2008).pdf

251

'CC D rezultă: ',

'=

AM MM

AB BB

',

'=

AC CC

AN NN

'

'=

PN NN

PM MM şi

'.

'

DB BB

DC CC= Înmulţind

membru cu membru relaţiile precedente rezultă: 1.AM AC PN DB

AB AN PM DC⋅ ⋅ ⋅ =

1) Dacă într-un triunghi ABC se duce o ceviană , ( )∈AD D BC şi o secantă intersectează pe AB, AC şi AD în punctele M, N, respectiv P, atunci:

.MB NC PD

DC BD BCMA NA PA

⋅ + ⋅ = ⋅

Demonstraţie. Din aplicaţia precedentă în triunghiurile ABD şi ADC cu cevianele AC respectiv AB şi secanta

MN rezultă: NP AP AB DC

NM AM AM BC= ⋅ ⋅ şi

.MP AP AC BD

MN AD AN BC= ⋅ ⋅ Sumând relaţiile

precedente obţinem 1 ,AP AB DC AC BD

AD AM BC AN BC

= ⋅ + ⋅

de unde .AD AB DC AC BD

AP AM BC AN BC= ⋅ + ⋅

Cum ,= +AB AM MB ,= +AC AN NC = +BC BD DC şi AD AP PD= + rezultă

.MB NC PD

DC BD BCMA NA PA

⋅ + ⋅ = ⋅

2) Fie triunghiul ABC şi punctele ( ), ( ), ( ), ( ).D BC E AB F AC M AD∈ ∈ ∈ ∈ Dacă

,EB FC MD

DC BD BCEA FA MA⋅ + ⋅ = ⋅ atunci .M EF∈

Demonstraţie. Fie ' .M AD EF= ∩ Din teorema transversalei rezultă

'

'

EB FC M DDC BD BC

EA FA M A⋅ + ⋅ = ⋅ care împreună cu relaţia din ipoteză dă:

'

'

MD M D

MA M A= sau

' '

'

MD MA M D M A

MA M A

+ += , adică

'

AD AD

MA M A= . Din relaţia precedentă avem : 'MA M A= ,

deci 'M M≡ . Observaţie: Teorema lui Menelaus este o consecinţă a teoremei transversalei.

A

B C

M N P

D

M '

N' B'

C'

Fig. 253

Page 245: Barbu_TEOREME_FUNDAMENTALE(2008).pdf

252

II.10. Teorema lui Leibniz101

„Sub aspect elementar, numeroase teoreme interesante sunt create mereu fie de către amatori devotaţi, fie de către marii matematicieni, care ori de câte ori au înţelegerea să revină la problemele elementare, le-au privit sub aspecte noi, dând demonstraţii mai simple sau încadrări mai naturale.” – N. Mihăileanu102 Fie G centrul de greutate al triunghiului ABC. Pentru orice punct M din planul triunghiului ABC este adevărată relaţia:

2 2 22 2 2 23 ( ).

3

AB BC CAMA MB MC MG

+ ++ + = + ∗

Demonstraţie.

Fie 'A mijlocul laturii BC. Relaţia lui Stewart aplicată în triunghiul 'AMA dă:

2 2 2' ' ' ' 'MA A G MA AG AA AG GA MG AA⋅ + ⋅ − ⋅ ⋅ = ⋅

Egalităţile ' 2

' , ',3 3

AAA G AG A A= =

2 2 22 2 ( )'

4

M B M C B CM A

+ −= ,

2 2 22 2( )'

4

AB AC BCAA

+ −= înlocuite în relaţia

precedentă dau concluzia.

Consecinţe:

1) Dacă ,M G≡ atunci 2 2 2

2 2 2

3

AB BC CAGA GB GC

+ ++ + = şi relaţia din teorema lui

Leibniz devine 2 2 2 2 2 2 23 .MA MB MC GA GB GC MG+ + = + + +

2) Din relaţia lui Leibniz rezultă că 2 2 2

2 2 2

3

AB AC BCMA MB MC

+ ++ + ≥ cu egalitate

dacă punctul M coicide cu G. 3) Dacă M coincide cu O – centrul cercului circumscris triunghiului ABC - atunci relaţia

( )∗ devine: 2 2 2

2 23 33

a b cOA OG

+ += + , adică

2 2 22 2

9

a b cOG R

+ += − .

4) În orice triunghi ABC este adevărată relaţia: 2 2 2 29 .R a b c≥ + +

Demonstraţie: Cum 2 0OG ≥ avem: 2 2 2

2

9

a b cR

+ +≥ , adică 2 2 2 29R a b c≥ + + .

5) Fie H şi O ortocentrul respectiv centrul cercului circumscris triunghiului ABC. Atunci:

i) 2 2 2 2 29 ( ),OH R a b c= − + + ii) 2 2 2

2 2 4( )4

9

a b cGH R

+ += − unde R este lungimea razei

cercului circumscris triunghiului ABC şi a, b, c lungimile laturilor triunghiului ABC.

101 Gottfried von Leibniz (1646-1716) – matematician şi filosof german, contribuţii importante în analiza matematică 102 Nicolae Mihăileanu (1912-1998) – matematician român

A

B C

M

A'

G

Fig. 254

Page 246: Barbu_TEOREME_FUNDAMENTALE(2008).pdf

253

Demonstraţie: i) Din relaţia cunoscută 3OH OG= rezultă 2 29OH OG= care împreună cu

relaţia de la observaţia 3) ne dă 2 2 2 2 29 ( )OH R a b c= − + + . ii) Cum 1

2OG HG= rezultă

2 21

4OG HG= , de unde

2 2 22 2 4( )

49

a b cHG R

+ += − .

II.11. Teorema lui Toricelli - Fermat

„În matematică nu există ignorabimus, nu vom şti...., trebuie să ştim şi vom şti!” - David Hilbert103

Să se găsească punctul P din planul unui triunghi ABC pentru care suma PA PB PC+ + este minimă. Demonstraţie. Soluţia 1. Prin rotaţia de centru B şi unghi de 60° a triunghiului ABP se obţine triunghiul ' 'C BP . Atunci,

'PB P P= şi ' 'PA C P= , de unde ' 'PA PB PC P P PC C C+ + = + ≥ . Suma

este minimă atunci când punctul 'P C C∈ , adică ( ') 60m BPC = ° . Analog, prin

rotaţia de centru A şi unghi de 60° a triunghiului ABP se obţine:

( ') 60m APC = ° , deci ( ) 120m APB = ° .

Analog, se arată că punctul P aparţine dreptelor ', 'BB AA ( 'B şi 'A se obţine ca mai sus), deci punctul P căutat se află la intersecţia dreptelor ', ', 'AA BB CC .

Soluţia 2: Fie P punctul pentru care suma PA PB PC+ + este minimă şi ad dreapta ce

conţine punctele P şi A. Arătăm că dacă, de exemplu, punctul P se plimbă pe dreapta ad

punctul căutat P rămâne acelaşi. Fie că 1A AP∈ şi presupunem că 1P este punctul pentru

care suma 1 1 1 1PA PB PC+ + este minimă. Astfel, pentru triunghiul ABC avem:

1 1 1PA PB PC PA PB PC+ + < + + şi pentru : 1 1 1 1 1PA PB PC PA PB PC+ + < + + relaţii care

sumate dau 1 1 1 1PA PA PA PA+ < + , sau 1 1 1 1 1 1PA A A PA PA PA+ + < + , de unde

rezultă 1 1 1 1A A PA PA+ < , absurd. Deci, dacă aA d∈ , atunci poziţia punctului P pentru care

se realizează minimul nu se schimbă. Analog, se demonstrează proprietatea de mai sus şi pentru punctele B şi C . Astfel, putem alege punctele bB d∈ şi cC d∈ astfel încât

triunghiul ABC să fie echilateral, acest lucru poate fi realizat. De exemplu, alegem aA d∈

astfel încât AB BC= . Evident, dacă triunghiul ABC este isoscel, punctul P aparţine axei de simetrie a triunghiului ABC. Plimbăm acum punctul bB d∈ ( iar bP d∈ ) astfel încât

triunghiul ABC devine echilateral şi atunci ( ) ( )m APB m APC= = ( ) 120m BPC = ° .

103 David Hilbert (1962-1943) – matematician german, profesor la Universitatea din Göttingen, contribuţii

remarcabile în geometrie şi analiza matematică

A

B C

P

C'

60°

P '

Fig. 255

Page 247: Barbu_TEOREME_FUNDAMENTALE(2008).pdf

254

Soluţia 3: Fie P un punct situat în interiorul triunghiului ABC astfel încât ( ) ( )m APB m APC=

Presupunem că lungimea segmentului [ ]PA este

constantă. Fie cercul cu centrul în A şi rază PA şi tangenta d în P la cerc. Fie 1 1, ,P d P P∈ ≠

1AP ∩ C(A,PA) .R= Cum APB APC≡ rezultă:

1 1PB PC PB PC RB RC+ < + < + şi de aici

PA PB PC+ + < PA RB RC RA RB RC+ + = + + . Repetând raţionamentul pentru PB sau PC constante rezultă că minimul se obţine pentru

( 120 )APB APC BPC≡ ≡ = ° .

Observaţii:

1) Punctul P se numeşte punctul lui Fermat104 sau punctul izogon al triunghiului ABC. 2) Demonstraţia de mai sus nu mai este valabilă dacă un unghi al triunghiului ABC are măsura mai mare de 120° (vezi „Triunghiurile lui Napoleon. Punctele lui Fermat”). Generalizarea teoremei lui Toricelli - Fermat Fie ABC şi DEF două triunghiuri de laturi a, b, c respectiv d, e, f. În exteriorul triunghiului ABC se construiesc triunghiurile ' , ' , 'A B C A B C A B C asemenea

cu DEF, ( ) ( ) 180 , ( ) ( ) 180 , ( ) ( ) 180+ < ° + < ° + < °m A m B m B m E m C m F . Atunci:

a) ' ' 'd AA e BB f CC⋅ = ⋅ = ⋅ ;

b) cercurile circumscrise triunghiurilor ' , 'A BC AB C şi 'ABC au un punct comun T; c) dreptele ', 'AA BB şi 'CC sunt concurente în punctul T; d) ' ' ' 2( )⋅ + ⋅ + ⋅ = ⋅ + ⋅ + ⋅d TA e TB f TC d TA e TB f TC ;

e) suma d MA e MB f MC⋅ + ⋅ + ⋅ este minimă când M coincide cu T;

f) 2 2 2 2 2 2 2 2 2 2 2 2 22( ) ( ) ( ) ( ) 16 ',⋅ + ⋅ + ⋅ = − + + + − + + + − + ⋅ ⋅d TA e TB f TC a d e f b d e f c d e f S S

unde S şi 'S sunt ariile triunghiurilor ABC respectiv DEF ; g) Dacă , ,A B CO O O sunt centrele cercurilor circumscrise triunghiurilor ' , 'A BC AB C

respectiv ',ABC triunghiurile A B CO O O şi DEF sunt asemenea.

Demonstraţie. a) Din asemănarea triunghiurilor 'A BC şi 'AB C rezultă '

'

CA BC

AC B C= şi

cum ' 'A CA BCB≡ rezultă că triunghiurile 'A CA şi 'BCB de unde

',

' '

AA AC DF e

BB B C EF d= = = adică ' '.d AA e BB⋅ = ⋅ Analog se arată că ' 'e BB f CC⋅ = ⋅ de

unde rezultă ' ' 'd AA e BB f CC⋅ = ⋅ = ⋅ .

b) Fie T al doilea punct de intersecţie dintre cercurile circumscrise triunghiurilor 'BCA şi ' .AB C Atunci, ( ) 180 ( ' ) 180 ( )= °− = °− m BTC m BA C m D şi

( ) 180 ( ' ) 180 ( ).= °− = °− m ATC m CB A m E Pentru că ( ) ( ) 180+ < ° m E m B rezultă că T

aparţine arcelor cercurilor considerate aflate în interiorul triunghiului ABC. Atunci: ( ) 360 ( ) ( )= °− − = m ATB m BTC m ATC 360 (180 ( )) (180 ( ))m D m E° − ° − − ° − =

104 Pierre de Fermat (1601-1665) – matematician francez, contribuţii în teoria probabilităţilor şi teoria numerelor

A

B C

P d

1P

R

Fig. 256

Page 248: Barbu_TEOREME_FUNDAMENTALE(2008).pdf

255

180 ( )m F° − = 180 ( ' )° − m AC B , adică patrulaterul 'TAC B este inscriptibil, deci T

aparţine şi cercului circumscris triunghiului '.ABC c) Deoarece patrulaterul 'BTCA este inscriptibil rezultă

' ' 'BTA BCA DFE AC B≡ ≡ ≡ şi cum ( ' ) ( ) 180+ = ° m AC B m ATB rezultă

( ') ( ) 180 ,+ = ° m BTA m BTA adică punctele ,A T şi 'A sunt coliniare. Analog se arată că

punctele , , 'B T B şi respectiv , , 'C T C sunt

coliniare, deci ' ' '.T AA BB CC= ∩ ∩

d) Din teorema lui Ptolemeu pentru patrulaterul inscriptibil 'TBA C rezultă

' ' ' (1).TA BC TB A C TC A B⋅ = ⋅ + ⋅ Din

asemănarea triunghiurilor 'A BC şi DEF avem:

' ' (2).

A B A C BCk

DE DF EF= = = Din relaţiile (1) şi

(2) rezultă ' ,TA k d TB e f TC f k⋅ ⋅ = ⋅ ⋅ + ⋅ ⋅ adică

' .TA d TB e TC f⋅ = ⋅ + ⋅ Analog se arată că:

'TB e TA d TC f⋅ = ⋅ + ⋅ şi ' .TC f TA d TB e⋅ = ⋅ + ⋅

Sumând ultimele trei egalităţi membru cu membru rezultă:

' ' ' 2( ).⋅ + ⋅ + ⋅ = ⋅ + ⋅ + ⋅TA d TB e TC f TA d TB e TC f

e) Fie M un punct arbitrar situat în planul triunghiului ABC. Atunci,

' ( ')⋅ ≤ + = ⋅ + ⋅ + ⋅d AA d AM MA d AM e BM f CM

cu egalitate atunci când ',∈ IM BTC AA adică când M coincide cu T.

f) Din subpunctul precedent ' .d AA d AM e BM f CM⋅ = ⋅ + ⋅ + ⋅ Determinăm pe 'AA din

triunghiul 'BAA aplicând teorema cosinusului: 2 2 2' ' 2 'cos( )= + − ⋅ + AA BA BA BA BA B E ,

adică 2 2 2( ') ( ) ( ') 2( ) ( ') [cos cos sin sin ]⋅ = + ⋅ − ⋅ ⋅ ⋅ ⋅ −d AA dc d BA d BA d BA B E B E

sau 2 2 2 2 2 2

2 2 2 2 2( ') 2 sin sin2 2

+ − + −⋅ = + − ⋅ ⋅ ⋅ ⋅ − ⋅

a c b d f ed AA d c a f d c a f B E

ac df,

de unde 2 2 2 2 2 2

2 2 2 2 2 ( )( )( ') 2( sin ) ( sin )

2

+ − + −⋅ = + − − ⋅

a c b d f ed AA d c a f ac B df E şi

deci: 2 2 2 2 2 2 2 2 2 2 2 2 22( ) ( ) ( ) ( ) 16 '.⋅ + ⋅ + ⋅ = − + + + − + + + − + ⋅ ⋅d TA e TB f TC a d e f b d e f c d e f S S

g) Fie , , .= ∩ = ∩ = ∩B C A C A BP AT O O Q BT O O R CT O O Deoarece A BO O CT⊥ şi

A CO O BT⊥ rezultă că patrulaterul AO RTQ este inscriptibil, deci

( )( ) 180 180 ( ) 180 [180 ( ' )] ( ' ) ( ).= °− = °− = °− °− = = Am QO R m QTR m BTC m BA C m BA C m D

Analog se arată că ( ) ( )= A B Cm O O O m E şi ( ) ( ),= A C Bm O O O m F adică triunghiurile

A B CO O O şi DEF sunt asemenea.

A

B C

P

Q R

T

AO

BO CO

A'

B' C'

Fig. 257

Page 249: Barbu_TEOREME_FUNDAMENTALE(2008).pdf

256

Observaţii:

1) Dacă ρ este raza cercului circumscris triunghiului DEF atunci BO C b ρ= ⋅ şi

' ' ' '.

4 ' 4 'A B

AA d AA d AA f d AAO O f

e d e S S

ρ ρ ρρ

⋅ ⋅ ⋅ ⋅ ⋅ ⋅ ⋅= = = = ⋅

⋅ Analog

'

4 'B C

e BBO O d

S

⋅= ⋅ şi

',

4 'C A

f CCO O e

S

⋅= ⋅ deci

'

4 'B C C AA B O O O OO O d AA

f d e S

⋅= = = ( ' ' 'd AA e BB f CC⋅ = ⋅ = ⋅ ).

2) Dacă triunghiul DEF este echilateral se obţine teorema lui Toricelli.

II.12. Teorema lui Feuerbach105 „Ca să te îndoieşti de linia dreaptă trebuie să ştii mai întâi din câte puncte e făcută.” – Nichita Stănescu106 Teorema lui Feuerbach

Într-un triunghi, cercul lui Euler este tangent cercului înscris şi cercurilor exînscrise corespunzătoare. Demonstraţie. Soluţia 1. Fie 'A intersecţia

bisectoarei interioare a unghiului BAC cu latura BC, a b cH H H triunghiul ortic al triunghiului ABC,

, ,a b cM M M mijloacele laturilor BC , AB respectiv

AC ale triunghiului ABC , a b cC C C triunghiul de

contact, iar , ,a b cD D D proiecţiile punctului aI –

centrul cercului A - exînscris - pe dreptele BC, CA respectiv AB (Fig. 258). Vom demonstra mai întâi că: 2 '

a a a a aM C M H M A= ⋅ . Din teorema bisectoarei

rezultă '

'

BA c

A C b= , de unde '

a cBA

b c

−=

+. Avem:

( )'

2 2( )a

a ac a b cM A

b c b c

−= − =

+ + (1). Din triunghiul

dreptunghic aAH B şi aAH C rezultă 2 2 2 2 2

a aA H A B B H A C C H= − = −

2 2 ( )( )a a a ab c H C H B H C H B− = + − , 2 2 ( 2 )a

b c a a H B− = − , de unde 2 2 2

2a

a c bH B

a

+ −= şi de aici

2 2

2a a a a

b cM H M B BH

a

−= − = (2). Deoarece

a aBC CD p b= = − unde

2

a b cp

+ += , (vezi „Cercul înscris într-un triunghi” ) rezultă

( )2 2a a a a

a b cM C M B BC p b

−= − = − − = (3). Din relaţiile (1), (2) şi (3)

105 Karl Feuerbach (1800-1834) – matematician german, contribuţii importante în geometrie 106 Nichita Stănescu (1933 – 1983) – eseist, poet român, ales postum membru al Academiei Române

A

B C

I cC

bD aH

bC

cD aI

aC A ' aD

'aD

Fig. 258

aM

ϕ

Page 250: Barbu_TEOREME_FUNDAMENTALE(2008).pdf

257

rezultă 2 'a a a a a

M C M H M A= ⋅ (4). Egalitatea (4) arată că punctul aH (care aparţine

cercului lui Euler al triunghiului ABC ) se transformă prin inversiunea de centru a

M şi

raport 2a a

M C în punctul 'A . Prin aceasta inversiune, cercul lui Euler (fără punctul a

M ) se

transformă într-o dreaptă d antiparalelă cu BC în raport cu A ce trece prin 'A (vezi „Cercul lui Euler”). Dreapta d este a doua tangentă comună interioară a cercului înscris şi A – exînscris. Prin această inversiune cercul înscris se transformă în el însuşi deoarece modulul inversiunii este egal cu puterea polului inversiunii faţă de cercul considerat. Deoarece

a a a aM C M D= rezultă că şi cercul A – exînscris se transformă în el

însuşi. Dreapta d fiind tangentă cercului înscris şi A –exînscris (invariante în inversiunea considerată) rezultă că şi cercul lui Euler ar fi tangent acestor cercuri în punctele ϕ şi

(punctele de intersecţie dintre dreapta d şi cercurile inverse şi A – exînscris). Analog se arată că cercul lui Euler este tangent cercurilor exînscrise corespunzând vârfurilor B şi C.

Soluţia 2. Fie C ( , )I r cercul înscris în triunghiul ABC (Fig. 259). Utilizăm teorema lui

Casey, considerând cercurile (a

M ,b

M ,c

M ,C) obţinem: 2

=a bM M

ct ,

2=

cAM

bt ,

2=

b cM M

at ,

( )2 2

−= − − =

aM

a b ct p b , ( )

2 2

−= − − =

bM

b a ct p c , ( )

2 2

−= − − =

cM

c b at p a (unde

prin distanţa tangenţială ijt dintre cercurile iC şi jC înţelegem lungimea tangentei comune

exterioare duse la cele două cercuri, cele două cercuri aflându-se de aceeaşi parte a tangentei). Pentru ca cercul înscris C şi cercul medial să fie tangente trebuie să demonstrăm că pentru o combinaţie a

semnelor + şi – rezultă ( ) ( ) ( ) 0c b a a b c b a c± − ± − ± − = , ceea ce este evident. Din

teorema lui Casey rezultă că există un cerc care trece prin a

M ,b

M ,c

M şi C. Cum cercul

circumscris triunghiului median este cercul lui Euler urmează ca cercul celor nouă puncte şi C sunt tangente.

aM

bM cM

ϕ

aH

bH

cH

Fig. 259

A

B C

Page 251: Barbu_TEOREME_FUNDAMENTALE(2008).pdf

258

Soluţia 3. Fie c

M mijlocul laturii AB, cH piciorul înălţimii din H, 9O centrul cercului lui

Euler al triunghiului ABC, DE diametrul perpendicular pe AB, F şi K mijloacele

segmentelor HD respectiv HE (Fig. 260). Deoarece 2

DEKF R= = şi cum 9O KF∈

rezultă că KF este diametru în cercul lui Euler al triunghiului ABC, deci ( ) 90 .= °

cm KM F

Fie ,XY AB⊥ XY diametru în cercul înscris în triunghiul ABC ( )∈Y AB şi

( ).XL MK L KF⊥ ∈ Atunci, 2LM XY r= = unde .= ∩M KF AB Drepta c

FM este

dreapta lui Simson a punctului D şi este perpendiculară pe dreapta CD în punctul S (vezi „Dreapta lui Simson”). Avem ( ).

c c cH CB SM M DIY M DS α≡ ≡ ≡ = Fie

.= ∩T FY KX Arătăm că ( ) 90 ,= °m KTY deci cercurile de diametre KF şi XY – adică

cercul lui Euler şi cercul înscris în triunghiul ABC – sunt tangente în ϕ . Din

sin , sinc c

M Y DI YH ICα α= = rezultă 2 2sin 2 sin ;c c

M Y YH DI IC Rrα α⋅ = ⋅ ⋅ = dar 2 2sin sin sin ,

cR FK M F MFα α α= = = de unde 2 .

c cM Y YH r MF LM MF⋅ = ⋅ = ⋅ Din

puterea unui punct M faţă de cercul lui Euler rezultă 2 ( ) (1).⋅ = = ⋅ = + = ⋅ + ⋅ = ⋅ + ⋅c c c c cMM MH MM MF MK MF KL LM MF KL MF LM MF KL M Y YH

Dar 2 2 ( ) ( ) ,− = − + = ⋅c c c c cM H M Y M H M Y M H M Y YH YM deci 2 2 2 (2).c c c cMH MM MY YH YM= = + ⋅ Din relaţiile (1) şi (2) rezultă

2 2 ,MY MF KL LX= ⋅ = adică LX MF

KL MY= relaţie care arată că ,⊥K FYϕ deci cercul lui

Euler şi cercul înscris în triunghiul ABC sunt tangente în punctul ϕ .

Soluţia 4. Teorema medianei aplicată în triunghiul OIH ne dă: 2 2 2

29 2 4

OI IH OHIO

+= −

sau 2 2 2

29

2 2 2 4

2 4h h

R Rr r r R R r RIO

− + − −= − , unde

hr este raza cercului înscris în

triunghiul ortic al triunghiului ABC (vezi „Cercul înscris” şi „Cercul circumscris”), şi de

A

B C

D

E

O

F

H

S cH

cM

9O I

F

K ϕ X

Y

X K

Y M

9O I

L ϕ

Fig. 260

Page 252: Barbu_TEOREME_FUNDAMENTALE(2008).pdf

259

aici rezultă că 2 2 2

29

4 4 ( 2 )

4 4

R Rr r R rIO

− + −= = , deci 9 2

RO I r= − . Cum

2

R este raza

cercului Euler rezultă cercul lui Euler şi cercul înscris sunt tangente interior.

Observaţii: 1) Punctele ϕ ,

aϕ ,

bϕ ,

cϕ de tangenţă dintre cercul lui Euler şi cu cercurile tritangente se

numesc punctele lui Feuerbach ale triunghiului ABC. 2) Într-un triunghi ABC se duce cea de-a doua tangentă interioară a cercului înscris cu fiecare cerc exînscris (primele tangente fiind laturile triunghiului). Dreptele ce unesc punctele de contact ale acestor trei tangente cu mijloacele laturilor corespunzătoare trec prin punctele lui Feuerbach. 1) Dreptele care unesc punctelele lui Feuerbach ale cercurilor exînscrise cu punctul lui Feuerbach al cercului înscris trec prin piciorul bisectoarei situate pe laturile respective.

Demonstraţie. Piciorul bisectoarei interioare a unghiului BAC - punctul 'A - este centrul de omotetie inversă dintre cercurile înscris şi A - exînscris; punctul lui Feuerbach ϕ este

centrul de omotetie directă dintre cercurile lui Euler şi cercul înscris, iar a

ϕ cetrul de

omotetie inversă între cercul lui Euler şi cercul A - exînscris, deci punctele 'A ,ϕ şi a

ϕ

sunt coliniare. Triunghiul lui Feuerbach

a b cϕ ϕ ϕ este triunghiul a cărui vârfuri sunt punctele de tangenţă

dintre cercul celor nouă puncte cu cercurile exînscrise unui triunghi ABC.

2) Cercul ce trece prin picioarele bisectoarelor interioare ale unui triunghi conţine punctul lui Feuerbach al triunghiului. Demonstraţie. Vom arăta că triunghiul determinat de picioarele bisectoarelor este asemenea şi omologic cu triunghiul lui Feuerbach.Vom utiliza în demonstraţia teoremei două leme:

Lema 1. Cercul C ( , )O R este tangent exterior cercurilor C1 1 1( , )O r şi C2 2 2( , )O r în punctele

A , respectiv B. Dacă 1A şi 1B sunt punctele de tangenţă ale tangentei exterioare comune

cercurilor C1 şi respectiv C2 , atunci 1 1

1 2( )( )

RAB A B

R r R r= ⋅

+ +.

Demonstraţie. Teorema cosinusului aplicată în triunghiurile AOB şi 1OOB (Fig. 261)

ne dă: 2 2 2

2 2

2cos 1

2 2

−= = −R AB AB

AOBR R

,

2 2 21 2 1 2 1 2 1 2( ) ( ) 2( )( ) cos( )OO R r R r R r R r OOO= + + + − + + ⋅

Din relaţiile precedente rezultă: 2

2 21 2 1 2 1 2( ) ( )( ) . = − + + + ⋅

ABOO r r R r R r

RDin

trapezul 1 1 2 1A BO O avem: 2 2 22 1 2 1 1( )OO r r A B= − + , de unde rezultă

concluzia.

O

1O

2O

A

B

1r

2r

R R

Fig. 261 1A

1B

Page 253: Barbu_TEOREME_FUNDAMENTALE(2008).pdf

260

Lema 2. Fie a, b, c lungimile laturilor triunghiului

ABC şi C ( , )O R cercul circumscris triunghiului

ABC. Dacă ( , )a aI r este A -cercul exînscris, iar 1B

şi 1C picioarele bisectoarelor interioare ale

unghiurilor B şi C, atunci 1 1

( 2 )

( )( )

+=

+ + ⋅aabc R R r

BCa b a c R

.

Demonstraţie. Fie 2 ,⊥aI B AC 2 ∈B AC şi

2 ⊥aI C AB , 2 ∈C AC , 2 2,⊥ ∈

a aOQ I B Q I B ,

2 2,⊥ ∈a a

OP I C P I C , 2 2 2

a b cAB AC p

+ += = =

(Fig. 262). Atunci, 2 2

c a bOP p

+= − = şi

2 2

b a cOQ p

+= − = . Din teorema bisectoarei

rezultă: 1

bcAB

a c=

+, 1

cbAC

a b=

+, de unde: 1

1

AB a b OP

AC a c OQ

+= =

+. Cum

2 2POQ C AB=

rezultă că triunghiurile 1 1ABC şi OPQ sunt asemenea şi 1 1 1 2( )

( )( )= = ∗

+ +

BC AB bc

PQ OP a c a b.

Ţinând cont că punctele , , ,a

O P Q I sunt pe cercul de diametru a

OI , din teorema sinusurilor

rezultă sin sin2

= ⋅ = ⋅ = ⋅a a a

aPQ OI POQ OI A OI

R care împreună cu ( )∗ dă:

1 1 ( )( )= ⋅

+ +a

abcB C OI

R a c a b. Utilizând relaţia lui Euler 2 ( 2 )= +

a aOI R R r rezultă.

1 1

( 2 )

( )( )

+=

+ +aabc R R r

BCR a c a b

.

Demonstraţia teoremei. Fie ϕ punctul lui

Feuerbach al triunghiului ABC şi 9O centrul

cercului lui Euler. Fie , ,a b c

ϕ ϕ ϕ punctele de

tangenţă al cercului lui Euler al triunghiului ABC cu cercurile sale exînscrise şi ,X Y punctele de tangenţă ale cercurilor A - exînscris şi B – exînscris cu latura AB. Avem:

2 2

a b c a b cZY AY BX AB c a b

+ + + += + − = + − = +

Din lema 1, rezultă :

( ) ( )2( 2 )( 2 )

2 2

+ ⋅ += =

+ + + +

a b

a b

a b

Ra b

a b R

R r R rR Rr r

ϕ ϕ

A

B

C O

P Q

I

aI

1C

2C

1B

2B

Fig. 262

1A

1B 1C

bϕ cϕ

ϕ

Fig. 264

Page 254: Barbu_TEOREME_FUNDAMENTALE(2008).pdf

261

Din lema 2 rezultă 1 1

( 2

( )( )

+=

+ +cabc R R r

A Bc a c b R

(Fig. 264).

Atunci, 1 12

( 2 )( 2 )( 2 )

( )( )( )

+ + +=

+ + +a b c

a b

abc R R r R r R rA B

a b c a b c Rϕ ϕ. Din simetria relaţiei precedente rezultă

că: 1 1 1 1 1 1= =a b b c c a

A B BC C A

ϕ ϕ ϕ ϕ ϕ ϕ,adică triunghiurile 1 1 1A B C şi

a b cϕ ϕ ϕ sunt asemenea (1). Arătăm

că punctele ϕ , 1B şi b

ϕ sunt coliniare. Din faptul că 9

2=

O R

I r

ϕϕ

, 1

1

,=b b

IB r

B I r

9

2=b b b

b

I r

O R

ϕϕ

rezultă : 9 1

1 9

1⋅ ⋅ =b b

b b

O IIB

I B I O

ϕ ϕϕ ϕ

şi din reciproca teoremei lui Menelaus rezultă că punctele

ϕ , 1B şi b

ϕ sunt coliniare. Analog se arată că punctele ϕ , 1C şi c

ϕ şi ϕ , 1A şi a

ϕ sunt

coliniare, ceea ce arată că triunghiurile 1 1 1A B C şi a b c

ϕ ϕ ϕ sunt omologice (2). Din relaţiile

(1) şi (2) rezultă 1 1 1 1 1( ) ( ) ( ) ( ) 180+ = + = °

c b c a bm C B m C A B m mϕ ϕ ϕϕ ϕ ϕ ϕ adică ϕ aparţine

cercului circumscris triunghiului 1 1 1A B C .

A

B C

X

Y

ϕ

I aϕ

bϕ cϕ

aI

bI

cI

Fig. 263

Page 255: Barbu_TEOREME_FUNDAMENTALE(2008).pdf

262

3) În triunghiul ABC fie , ,a b cC C C punctele de contact ale cercului înscris cu laturile

BC , AC, respectiv AB, X şi Y punctele de intersecţie dintre paralela dusă prin A la BC cu dreptele

a bC C , respectiv

a cC C . Dreapta lui Euler a triunghiului

aC XY trece prin

punctul lui Feuerbach al triunghiului ABC. Demonstraţie. Vom arata mai întâi că punctele , , , ', ',

b cA C C X Y I (Fig. 265) sunt conciclice

(unde 'X şi 'Y sunt mijloacele segmentelor aC X , respectiv

aC Y ), ele aparţinând cercului

celor nouă puncte al triunghiului aC XY . Avem, ≡ ≡ ≡

b a b b a bAXC CCC CCC AC X de unde

≡b

AX AC şi analog ≡c

AY AC .

Cum ≡b c

AC AC rezultă

≡AY AX , deci A este mijlocul segmentului XY . Cercul celor nouă puncte al triunghiului

aC XY

conţine punctele , ', 'A X Y (fiind mijloacele laturilor triunghiului

aC XY ). Din

= =b

AY AX AC rezultă că bC

este piciorul înălţimii din X pe

aYC ; analog

cC este piciorul

înălţimii din X pea

YC , deci

punctele bC şi

cC aparţin cercului

celor nouă puncte al triunghiului cC XY . Fie 'H ortocentrul triunghiului

aC XY . Atunci,

' '≡b a c a

H C C H C C , deci punctul 'H aparţine cercului înscris în triunghiul ABC şi mai

mult este diametru în acest cerc, adică I este mijlocul segmentului '

aH C , ceea ce arată

că I aparţine cercului lui Euler al triunghiului

aC XY .

Demonstraţia teoremei. Fie ,O H şi 9O centrul cercului

circumscris, ortocentrul şi centrul cercului lui Euler al triunghiului ABC ( 9O este

mijlocul segmentului HO ) şi d dreapta lui Euler a triunghiului

.aC XY Fie '

9 9 aO O IH , '

9 ∈O d şi

'd AH H=I , iar 'O punctul

de intersecţie dintre paralela prin O la 'IH cu dreapta d. Cum

'AD IH şi M este mijlocul lui AI rezultă că 'AD IH r= = (raza cercului înscris în ABC ). Dacă

aM este mijlocul laturii

BC , atunci 2a

AH OM= . Fie

A

B C

H'

X' I

Y'

X Y

aC

bC

cC

Fig. 265

A

B C

H' '9O

I H

X

Y

aC

bC

cC

Fig. 266

D

J

aD

O '

O I '

aI

9O

M

Page 256: Barbu_TEOREME_FUNDAMENTALE(2008).pdf

263

aI centrul cercului exînscris corespunzător laturii BC şi ra – raza sa. Cum punctele A , I

şi aI sunt coliniare (vezi „Cercuri exînscrise”), atunci J, punctul de intersecţie dintre AIa cu

cercul circumscris triunghiului ABC , este mijlocul arcului BC (Fig. 266). Fie 'I

simetricul lui I faţǎ de O . Deoarece 'a

I I trece prin punctul aD de tangenţǎ a cercului

exînscris corespunzǎtor laturii BC . Din asemǎnarea triunghiurilor MAD şi 'MJO ,

respectiv 'MIH cu 'MJO avem ' 'JO MO

AD MD= , de unde rezultă că

' ' 2' '

' 2a a

a

AI rMO MO MJ MJJO IH r r r r r r

MD MH MI MI AI r= ⋅ = ⋅ = ⋅ = ⋅ = ⋅ = ⋅ = În trapezul 'HOO D avem:

'9 92 ' ( ' ) ( ) 2a aO O O O H D JO R H A D A r R O M r= + = − + − = − + − =

( ) 'a a ar R r C I D I− + + + , de unde '

9 92 =O O ( ) (2 )− + + + − =a ar R r r R r R , rezultă că

'9 9 / 2=O O R , adică '

9O aparţine cercului lui Euler al triunghiului ABC . Cum '9 9 aO O IH ,

dreptele '9 aO H şi 9O I se intersecteazǎ în centrul de asemǎnare al cercurilor înscris,

respectiv al lui Euler al triunghiului ABC . Dar cele două cercuri sunt tangente interior în punctul lui Feuerbach care este centrul de asemǎnare pentru cele două cercuri. Astfel, dreapta lui Euler a triunghiului

aC XY conţine punctul lui Feuerbach al triunghiului ABC.

Observaţii : 1) Vom nota triunghiul

aC XY cu

aT . Analog cu

aT se construiesc triunghiurile

bT şi

cT .

Punctul lui Feuerbach este punctul de intersecţie dintre dreptele lui Euler corespunzătoare triunghiurilor

aT ,

bT şi

cT .

2) Punctul 'H este punctul antipodal al punctului

aC al triunghiului ABC.

3) Centrul cercului lui Euler al triunghiuluiaC XY

este punctul M, mijlocul segmentului IA . 4) Dreapta 'MH este dreapta lui Euler a triunghiului

aC XY .

4) Punctul lui Feuerbach

aϕ de pe cercul A -

exînscris se află pe bisectoarea unghiului A a

triunghiului ABC dacă ( ) 60= °m A sau

( ) ( )=m B m C .

Demonstraţie. Fie aH piciorul înalţimii din A,

aI

centrul cercului A - exînscris şi , ,a b cD D D punctele

de tangenţă ale acestuia cu dreptele BC, CA respectiv AB , 'A punctul diametral opus lui

aD în

cercul A - exînscris 1 '= ∩A AA BC , a

M mijlocul

laturii BC, "A mijlocul segmentului

aAI , ' "= ∩

aQ A A AH , T intersecţia dintre BC şi

tangenta în a

ϕ la cercul A - exînscris,

= ∩a a

P I T AH , unde ' "= ∩a A Aϕ C ( , )a aI r

A

B C

aD bD

cD

A'

A" aM

1A aH T aϕ

Q

P

Fig. 267

aI

Page 257: Barbu_TEOREME_FUNDAMENTALE(2008).pdf

264

(Fig. 267). Deoarece =a a

T TDϕ rezultă ⊥a a a

TI D ϕ şi cum '⊥a a aD Aϕ ϕ

rezultă ' a

A Q I P deci patrulaterul ' aA QPI este paralelogram, deci

1'≡ ≡a a

QP A I I A .Cum a aI D QP rezultă că

a aI D PQ este paralelogram a aD P QI (1).

Cum ' a

A I AQ şi "A este mijlocul segmentului 'a

A I rezultă că patrulaterul 'a

QAA I

este paralelogram,deci 'aI Q AA (2). Din relaţiile (1) şi (2) rezultă '

aD P AA . Deoarece

punctele aD şi 1A sunt izotomice rezultă că

aI Q trece prin punctul

aM . Din asemănarea

triunghiurilor a a

TM I şi a

TPD , respectiv a a

TD I şi a

TH P rezultă = =a a

a a

TI TDTM

TD TP TH şi de

aici 2 2= ⋅ =a a a a

TD TH TM Tϕ , adică T este pe axa radicală a cercului lui Euler a triunghiului

ABC şi a cercului A -exînscris, această axă este tangenta în a

ϕ la cercul A-exînscris , ceea

ce arată că punctul a

ϕ de intersecţie al cercului A-exînscris cu dreapta ' "A A este punctul

lui Feuerbach de pe cercul A - exînscris. Punctul lui Feuerbach se află pe bisectoarea 'a

A I

dacă şi numai dacă a

ϕ coincide cu "A ceea ce este echivalent cu "2

= =a

a

AIAA r . Din

triunghiul c a

AD I rezultă sin

2

= a

a

rAI

A, de unde

2sin2

= a

a

rr

A, adică

1sin

2 2=

A, deci

( ) 60= °m A . Dacă dreptele ' "A A şi a

AH coincid – adică triunghiul ABC este isoscel,

atunci punctele ,a aD M şi "A coincid cu

aϕ şi reciproc.

5) Punctul ϕ al lui Feuerbach este ortopolul dreptei OI în raport cu triunghiul ABC. Demonstraţie. Cercul înscris în triunghiul ABC conţine ortopolul dreptei OI (vezi „Ortopolul unei drepte). Deoarece ortopolul unui diametru al cercului circumscris unui triunghi ABC aparţine cercului lui Euler al triunghiului ABC rezultă că ortopolul dreptei OI în raport cu triunghiul ABC aparţine atât cercului înscris cât şi cercului lui Euler al triunghiului ABC, deci ortopolul dreptei OI este punctul lui Feuerbach (ϕ ) triunghiului

ABC. 6) Distanţele de la punctul lui Feuerbach corespunzător unui triunghi ABC la picioarele înălţimilor triunghiului sunt egale, respectiv, cu perpendicularele coborâte din vârfurile triunghiului pe dreapta OI. Demonstraţia rezultă din faptul că punctul ϕ al lui Feuerbach este ortopolul dreptei OI în

raport cu triunghiul ABC, iar distanţa dintre ortopolul unui diametru al cercului circumscris şi piciorul unei înălţimi este egală cu distanţa între vârful din care pleacă această înălţime şi vârful considerat (vezi „Ortopolul unei drepte”). 7) Distanţele de la punctul lui Feuerbach corespunzător unui triunghi ABC la vârfurile triunghiului sunt egale, respectiv, cu distanţele de la picioarele înălţimilor la proiecţiile vârfurilor pe dreapta OI. Demonstraţia rezultă din faptul că punctul ϕ al lui Feuerbach este ortopolul dreptei OI în

raport cu triunghiul ABC, iar distanţa între un vârf al triunghiului ABC şi ortopolul unui diametru al cercului circumscris este egală cu distanţa între proiecţiile aceluiaşi vârf pe latura opusă şi pe diametru (vezi „Ortopolul unei drepte”).

Page 258: Barbu_TEOREME_FUNDAMENTALE(2008).pdf

265

8) Punctul lui Feuerbach ϕ al triunghiului ABC este punctul anti - Steiner al dreptei IO

în raport cu triunghiul median al triunghiului ABC. Demonstraţie. Vezi „Punctul anti-Steiner”. 9) Dreapta lui Simson a punctului lui Feuerbach ϕ al triunghiului ABC în raport cu

triunghiul median al acestuia este paralelă cu dreapta OI. Demonstraţie. Deoarece punctul lui Feuerbach al triunghiului ABC este ortopolul dreptei OI, dreapta lui Simson a punctului ϕ în raport cu triunghiul median al triunghiului ABC se

află la egală distanţă de punctul ϕ şi dreapta OI, deci dreapta lui Simson a punctului ϕ este

paralelă cu OI. 10) Fie ϕ punctul lui Feuerbach al triunghiului ABC şi , ,

a b cM M M mijloacele laturilor

BC, AC respectiv AB. Una din distanţele , ,a b c

M M Mϕ ϕ ϕ este egală cu suma celorlalte

două. Demonstraţie. Fără a restrânge generalitatea presupunem că .> >b c a Fie P punctul de intersecţie dintre

aMϕ şi

cercul înscris în triunghiul ABC şi aC punctul de contact

al cercului înscris cu latura BC. Atunci, 2

2 ( ),

4

−⋅ = =a a a a

b cM P M M Cϕ iar ,

2=a

M R

P r

ϕϕ

deoarece

ϕ este centrul de asemănare dintre cercul medial şi

cercul înscris în triunghiul ABC. Astfel,

,2

=−

a

a

M R

M P R r

ϕde unde

( ),

2 2

−=

−a

b c RM

R rϕ distanţa

aM ϕ este proporţională cu diferenţa −b c . Analog, se arată că

( )

2 2

−=

−b

c a RM

R rϕ şi

( ).

2 2

−=

−c

b a RM F

R r Evident, .+ =

a b cM M Mϕ ϕ ϕ

11) Dreapta lui Simson a punctului lui Feuerbach al triunghiului ABC în raport cu triunghiul ortic

a b cH H H al triunghiului ABC este paralelă cu OI.

Demonstraţie. Deoarece triunghiurile a b cH H H şi

a b cM M M sunt triunghiuri S în cercul

median (vezi „Triunghiuri ortopolare”) cum dreapta lui Simson a unui punct în raport cu triunghiurile S din aceeaşi familie păstrează aceeaşi direcţie rezultă concluzia.

A

B C

I

aM aC

ϕ

Fig. 268

P

Page 259: Barbu_TEOREME_FUNDAMENTALE(2008).pdf

266

12) Dreapta lui Simson a punctului lui Feuerbach în raport cu triunghiul de contact

a b cC C C al triunghiului ABC este paralelă cu dreapta OI.

Demonstraţie. Fie a b cP P P triunghiul determinat

de mijloacele arcelor ,a aH M ,

b bH M

c cH M ale

cercului medial. Triunghiurile a b cP P P şi

a b cM M M sunt triunghiuri S deoarece

1( ) ( ) ( ) ,

2= − a am P M m B m C

1( ) ( ) ( ) ,

2= − b bm P M m C m A

1( ) ( ) ( ) ,

2= − c cm PM m A m B deci suma algebrică a

măsurilor lor este egală cu zero. Deoarece triunghiul de contact

a b cC C C este omotetic cu

triunghiul a b cP P P (centrul de omotetie fiind

punctul lui Feuerbach ) rezultă că dreapta lui Simson a punctului ϕ în raport cu triunghiul

a b cC C C este paralelă cu OI.

13) Dreapta lui Simson a punctului lui Feuerbach al triunghiului ABC în raport cu triunghiul de contact

a b cC C C coincide cu dreapta lui Simson punctul lui Feuerbach al

triunghiului ABC în raport cu triunghiul median a b c

M M M .

Demonstraţie. Deoarece dreapta lui Euler a triunghiului a b cC C C este OI, ortocentrul

triunghiului a b cC C C aparţine dreptei OI şi cum O este ortocentrul triunghiului median

a b cM M M , rezultă că dreapta lui Simson comună este paralelă cu OI şi trece la o distanţă

egală de punctul lui Feuerbach şi de dreapta OI. 14) Fie P mijlocul segmetului HI şi O centrul cercului lui Euler al triunghiului ABC. Punctul lui Feuerbach (ϕ ) şi centrul cercului înscris ( I ) al triunghiului ABC sunt

puncte inverse în cercul de centru 9O şi raza 9PO .

Demonstraţie. Deoarece punctele 9O , I şi ϕ sunt coliniare

rezultă: 92OI PO= , 2 294=OI PO (1), iar din teorema lui

Euler avem : 2 2 2= −OI R Rr (2). Deoarece 9 2= −R

O I r şi

9 2=R

O ϕ (3) rezultă: 29 9 94 4= ⋅O P O O Iϕ , adică

29 9 9= ⋅O P O O Iϕ , de unde rezultă concluzia.

15) Fie 1A proiecţia vârfului A al triunghiului ABC pe dreapta OI şi ϕ punctul lui

Feuerbach corespunzător triunghiului ABC. Punctele ϕ şi 1A sunt simetrice faţă de

latura b c

M M a triunghiului median. Demonstraţia rezultă din teorema 11- „Ortopolul unei drepte”.

A

B C aM

bM cM

aH

bH

cH

Fig. 269

aP

bP

cP

H

P

O ϕ

9O

I Fig. 270

Page 260: Barbu_TEOREME_FUNDAMENTALE(2008).pdf

267

16) Fie 1A proiecţia vârfului A al triunghiului ABC pe dreapta OI, ϕ punctul lui

Feuerbach corespunzător triunghiului ABC şi aH piciorul perpendicularei duse din A

pe BC. Dreapta φaH şi perpendiculara 1AA coborâtă din A pe OI sunt simetrice în

raport cu înălţimea a

AH şi se intersectează pe latura b c

M M a triunghiului median.

Demonstraţia rezultă din simetria punctelor 1A şi φ în raport cu laturab c

M M .

17) Fie a b c

M M M triunghiul median, a b cH H H triunghiul ortic,

a b cC C C triunghiul de

contact al unui triunghi ABC şi ϕ punctul lui Feuerbach corespunzător. Dreapta aCϕ

este bisectoarea unghiului a aH Mϕ .

Demonstraţie. Fie D punctul în care tangenta în ϕ la cercul lui Euler intersectează latura

BC şi E punctul de intersecţie dintre dreapta aCϕ cu cercul lui Euler. Atunci,

aD DCϕ ≡ ,

deci a a

D C DCϕ ϕ≡ . Dar 1( ) [ ( ) ( )]

2a a am DC m H m EMϕ ϕ= + şi

1 1( ) ( ) [ ( ) ( )]

2 2a a a a amD C m C m H mH Cϕ ϕ ϕ= = + , de unde rezultă că ( ) ( )]a a a

m EH m H C= , deci

a a a aH C C Mϕ ϕ≡ , adică

aCϕ este bisectoarea unghiului

a aH Mϕ .

aM

I

aC

ϕ

aH E

D

Fig. 271

A

B C

Page 261: Barbu_TEOREME_FUNDAMENTALE(2008).pdf

268

18) Fie a b cC C C triunghiul de contact al unui triunghi ABC , I centrul cercului înscris în

triunghiul ABC şi ' ' ', ,a b cC C C punctele diametral opuse punctelor , ,

a b cC C C în cercul

înscris, iar ', ', 'A B C mijloacele segmentelor , ,AI BI respectiv CI . Dreptele ' ' '' , ' , 'a b c

A C B C C C sunt concurente în punctul lui Feuerbach ( )ϕ

Demonstraţie. Fie "A punctul de intersecţie dintre paralela dusă din aC la AI şi înălţimea

aAH . Atunci, 1

" " [ ( ) ( )]2a a

A AI H A C m B m C ≡ = −

(vezi „Drepte izogonale”). Cum

[ ]1( ) ( ) ( )

2a am H C m B m Cϕ = − rezultă "a a a a aH AI H C H A Cϕ≡ ≡ , deci

patrulaterul "a aH C Aϕ este inscriptibil ,de unde " " ( 90 )

a a aA C A H Cϕ ≡ = ° şi cum

'a aC C este diametru în cercul lui Euler rezultă că punctele ', ",

aA Cϕ sunt coliniare. Cum

", 'A A şi 'aC sunt colinire rezultă că punctele ', ',

aA Cϕ sunt coliniare.

19) Consecinţă: Fie ' ' ', ,a b cC C C punctele diametral opuse vârfurilor , ,

a bC C respectiv

cC

ale triunghiului de contact al triunghiului ABC, în cercul înscris triunghiului ABC. Cercurile de diametru ' ' ', ,a a b b c cC C C C C C se intersecteaza în puntul lui Feuerbach al

triunghiului ABC. Demonstraţia rezultă din teorema precedentă.

A

B C

A'

A"

I

aH aM

'aC ϕ

aC

bC

cC

Fig. 272

Page 262: Barbu_TEOREME_FUNDAMENTALE(2008).pdf

269

II.13. Teorema lui Desargues107

„Matematica a apărut şi se dezvoltă, printr-un continuu proces de modelare la nivelul Gândirii a fenomenelor lumii fizice,Matematica servind, pe această cale, înţelegerii acestor fenomene.” - Aristotel108

Teorema lui Desargues

Punctele de intersecţie ale dreptelor omologe, a două triunghiuri omologe coplanare, sunt coliniare. Demonstraţie. Fie ABC şi ' ' 'A B C două triunghiuri coplanare astfel încât ' ' ' AA BB CC O∩ ∩ = .

Teorema lui Menelaus aplicată triunghiurilor , ,OBC OCA OAB şi transversalelor ' 'B C ,

' 'C A respectiv ' 'A B dă:' '

1' '

⋅ ⋅ =LC B B C O

LB B O C C,

' '1

' '⋅ ⋅ =

MA C C A O

MC C O A A,

'

' '1

'⋅ ⋅ =

NB A A B O

NA A O B B, relaţii care prin înmulţire membru cu membru dau:

1LC NB MA

LB NA MC⋅ ⋅ = şi conform teoremei lui Menelaus aplicată triunghiului ABC şi

punctelor , ,L M N rezultă că punctele , ,L M N sunt coliniare. Observaţie: Dreapta ce conţine punctele , ,L M N se numeşte axa de omologie. Triunghiurile ABC şi ' ' 'A B C se numesc omologice.

107 Gérard Desargues (1591-1661) – matematician francez , fondatorul geometriei proiective 108 Aristotel (384-322 î.e.n.) – filosof grec

M

A

B

C

L

C'

B'

A' N

Fig. 273

O

Page 263: Barbu_TEOREME_FUNDAMENTALE(2008).pdf

270

Reciproca teoremei lui Desargues Fie triunghiurile ABC şi ' ' 'A B C cu proprietatea că există punctele , ,L M N astfel încât ' '= ∩L BC B C , ' '= ∩M AC A C şi ' '= ∩N AB A B , iar dreptele 'AA şi 'BB

nu sunt paralele. Doar punctele ,L M şi N sunt coliniare, atunci dreptele 'AA , 'BB şi 'CC sunt concurente.

Demonstraţie. Fie ' '= ∩O AA BB . Dar ' '= ∩ ∩N AB A B MN şi conform teoremei lui

Desargues, dreptele suport ale laturilor triunghiului 'LB B şi 'MAA se intersectează doar câte două în trei puncte coliniare O , C şi 'C : ' '= ∩O AA BB , = ∩C LB MA .,

' ' '= ∩C MA LB , deci '∈O CC , adică dreptele ', ', 'AA BB CC sunt concurente în

punctul O .

Fie triunghiurile omologice ABC şi

' ' ',A B C P centrul lor de omologie, M un punct în planul triunghiului ABC,

1 2 3M M M triunghiul cevian al lui M în raport cu triunghiul ABC şi

1 1 2 2 ' ', ' ',M PM B C M PM C A= ∩ = ∩

3 3 ' '.M PM A B= ∩ Dreptele PM, '1' ,A M

'2'B M şi 3' 'C M sunt concurente.

Demonstraţie. Fie ' ',X BC B C= ∩

' ',Y CA C A= ∩ ' '.Z AB A B= ∩ Din

teorema lui Desargues rezultă că punctele X, Y, Z sunt coliniare. Din reciproca teoremei lui Desargues rezultă că triunghiurile 'AA Y şi 1 1 'M M X sunt omologice, 'PCC fiind

axa lor de omologie, deci dreptele

1 1, ' 'AM A M şi YX sunt concurente într-un

punct Q. Analog, triunghiurile 'CC X şi

3 3 'M M Z sunt omologice, 'PB B fiind axa

lor de omologie, deci dreptele 3 3, ' 'CM C M

şi XZ sunt concurente într-un punct R . Fie ' ' ' .M A Q C R= ∩ Triunghiurile 'CC R şi

'AA Q sunt omologice, centrul de omologie

fiind punctul Y iar axa de omologie este '.PMM Atunci dreptele 3 1' ', ' 'C M A M şi PM sunt

concurente şi analog 1 2' ', ' 'A M B M şi PM sunt concurente de unde rezultă concluzia.

Y

A

B

C

X

C'

B'

A' Z

Fig. 274

P M 1M 2M

3M

Q

R

'1M

'2M

'3M

M '

Page 264: Barbu_TEOREME_FUNDAMENTALE(2008).pdf

271

II.14. Teorema lui Döttl

„Lucrul cel mai uimitor este că raţionamentele matematice cele mai abstracte sfârşesc prin a lărgi cunoaşterea noastră despre lume.” - Albert Einstein109

Fie triunghiul ABC şi ' ( ), ' ( ), ' ( )A BC B CA C AB∈ ∈ ∈ astfel încât dreptele ', ', 'AA BB CC

sunt concurente. Dacă " ( ' '), " ( ' '), " ( ' ')A B C B C A C A B∈ ∈ ∈ astfel încât dreptele

' ", ' ", ' "A A B B C C sunt concurente, atunci dreptele ", ", "AA BB CC sunt concurente. Demonstraţie. Fie " , " , "X AA BC Y BB AC Z CC AB= ∩ = ∩ = ∩ (Fig. 275). Din

teorema lui Menelaus (pentru patrulatere) aplicată patrulaterului ' 'BCB C şi punctelor coliniare

, , " ,X A A A avem: ' " '

1,' " '

BX CA B A C A

XC AB A C AB⋅ ⋅ ⋅ =

de unde ' " '

' " '

BX AB AB A C

XC AC AC A B= ⋅ ⋅ (1). Analog,

' " '

' " '

CY BC BC B A

YA BA BA B C= ⋅ ⋅ (2) şi

' " '

' " '

AZ CA CA C B

ZB CB CB C A= ⋅ ⋅

(3). Deoarece dreptele ', ', 'AA BB CC respectiv ' ", ' ", ' "A A B B C C sunt concurente, din teorema lui

Ceva rezultă: ' ' '

1' ' '

A B B C C A

A C B A C B⋅ ⋅ = şi

" ' " ' " '1

" ' " ' " '

A B B C C A

A C B A C B⋅ ⋅ = (4). Înmulţind relaţiile (1), (2) şi (3) membru cu membru şi ţinând

cont de relaţiile (4) rezultă 1,BX YC ZA

XC YA ZB⋅ ⋅ = iar din reciproca teoremei lui Ceva rezultă că

dreptele ", ", "AA BB CC sunt concurente.

II.15. Teorema lui Van - Aubel

„Învăţând matematica, înveţi sa gandeşti.” – Grigore Moisil110

Teorema lui Van-Aubel Dacă AD, BE şi CF sunt trei ceviene concurente într-un punct P interior triunghiului

ABC, atunci .AP AF AE

PD FB EC= +

Demonstraţie. Avem: [ ]

[ ]

[ ]

[ ]

[ ] [ ]

[ ] [ ]

APB APC APB APC

BPD PCD BPD PCD

A A A AAP

PD A A A A

+= = =

+de unde rezultă că:

109Albert Einstein (1879-1955) – fizician german, profesor universitar la Berlin şi Princeton, laureat al Premiului Nobel 110 Grigore Moisil (1906-1973) – matematician român, profesor la Universitatea din Iaşi, membru al Academiei Române

A

B C A'

B' C'

X

Y

Z

Fig. 275

A"

B"

C"

Page 265: Barbu_TEOREME_FUNDAMENTALE(2008).pdf

272

[ ] [ ]

[ ]

[ ]

[ ]

[ ]

[ ]

(1).APB APC APB APC

BPC BPC BPC

A A A AAP

PD A A A

+= = + Dar [ ]

[ ]

[ ]

[ ]

[ ] [ ]

[ ] [ ]

[ ]

[ ]

(2)ACF APF ACF APF APC

FCB FPB FCB FPB BPC

A A A A AAF

FB A A A A A

−= = = =

şi analog [ ]

[ ]

(3).APB

BPC

AAE

EC A= Din relaţiile (1), (2) şi (3) rezultă concluzia.

1) Dacă D, E, F sunt mijloacele laturilor triunghiului ABC, atunci P este centrul de greutate al triughiului ABC şi relaţia lui Van - Aubel devine 2 .AG GD= 2) Dacă P este I, centrul cercului înscris triunghiului

ABC, atunci , ,AF b AE c

FB a EC a= = relaţia lui Van Aubel

devenind 1,AI b c

ID a

+= > relaţie ce arată că I este mai

„aproape” de piciorul bisectoarei D, decât de vârful A. 3) Dacă P este H, ortocentrul triunghiului ABC, atunci

, ,AF tgB AE tgC

FB tgA EC tgA= = relaţia lui Van Aubel devenind

cos.

cos cos

AH A

HD B C=

4) Dacă P este punctul lui Lemoine K al triunghiului ABC, atunci

2 2

2 2, ,

AF b AE c

FB ECa a= = relaţia lui Van - Aubel devenind

2 2

2

AK b c

KD a

+= .

5) Dacă P este punctul lui Gergonne ,Γ atunci , ,AF p a AE p a

FB p b EC p c

− −= =

− −de unde

( )

( )( )

A a p a

D p b p c

Γ −=

Γ − −.

6) Dacă P este primul punct BrocardΩ , atunci 2 2

2 2, ,

AF b AE b

FB ECc a= = de unde

2 2 2 2

2 2

A a b b c

D a c

Ω +=

Ω.

7) Dacă P este al doilea punct al lui Brocard 'Ω , atunci 2 2

2 2,

AF c AE c

FB ECa b= = şi relaţia lui

Van Aubel devine 2 2 2 2

2 2

'.

'

A b c a c

D a b

Ω +=

Ω

A

B C

E P

D

F

Fig. 276

Page 266: Barbu_TEOREME_FUNDAMENTALE(2008).pdf

273

8) În triunghiul ABC fie cevienele AD, BE şi CF concurente într-un punct P astfel încât

,k k

BD AB AF AC

DC AC FB BC

= =

şi , .k

AE ABk

EC BC

= ∈

Atunci: .k k

k

AP AC AB

PD BC

+=

Demonstraţie: Notăm cu a, b, c lungimile laturilor BC, CA respectiv AB. Din teorema lui

Van – Aubel rezultă că .k k

k

AP AF AE AC AB

PD FB EC BC

+= + =

Observaţii:

1) Dacă pe latura BC a triunghiului ABC se consideră un punct D astfel încât

,k

BD ABk

DC AC

= ∈

atunci dreapta AD se numeşte ceviană de rang k.

2) i) Mediana AD este o ceviană de rang 0, ( 0),k = deoarece 0

1BD AB

DC AC

= =

.

ii) Bisectoarea AD este o ceviană de rang 1, ( 1),k = deoarece 1

BD AB

DC AC

=

.

iii) Simediana AD este o ceviană de rang 2, ( 2)k = , deoarece 2

BD AB

DC AC

=

.

iv) Antibisectoarea AD este o ceviană de rang ( 1), ( 1),k− = − deoarece 1

.BD AB

DC AC

− =

9) Dacă P este punctul de concurenţă a trei ceviene de rang k şi M este un punct din

planul triunghiului ABC, atunci: ,k k k

k k k

a MA b MB c MCMP

a b c

⋅ + ⋅ + ⋅=

+ +

uuur uuur uuuuruuur

(unde a, b, c sunt

lungimile laturilor BC, CA respectiv AB). Demonstraţie. Fie cevienele AD, BE şi CF ceviene de rang k . Din triunghiul MAD:

,1

MA MBMP

λλ

+=

+

uuur uuuruuur

unde .AP

PDλ = Conform teoremei

(1), ,k k

k

AP b c

PD a

+= deci

( ).

k k k

k k k

a MA b c MDMP

a b c

⋅ + +=

+ +

uuur uuuuruuur

În triunghiul MBC, ,k k

BD AB c

DC AC b

= =

deci

1

k

k k

k k k

cMB MC

b MD c MCbMD

b cc

b

+ + = =+ +

uuur uuuuruuuur uuuur

uuuur de unde

.k k k

k k k

a MA b MB c MCMP

a b c

⋅ + ⋅ + ⋅=

+ +

uuur uuur uuuuruuur

A

B C

E P

D

F

Fig. 277

M

Page 267: Barbu_TEOREME_FUNDAMENTALE(2008).pdf

274

Cazuri particulare:

1) Pentru 0,k P G= ≡ relaţia din teoremă devine .3

MA MB MCMG

+ +=

uuur uuur uuuuruuuur

2) Pentru 1, ,k P I= ≡ relaţia din teoremă devine: .aMA bMB cMC

MIa b c

+ +=

+ +

uuur uuur uuuuruuur

3) Pentru 2,k P K= ≡ (punctul lui Lemoine), relaţia din teoremă devine: 2 2 2

2 2 2.

a MA b MB c MCMK

a b c

+ +=

+ +

uuur uuur uuuuruuuur

4) Pentru 1,k P Z= − ≡ (punctul de recurenţă al antibisectoarelor), relaţia din teoremă

devine: .bcMA acMB abMC

MZab bc ac

+ +=

+ +

uuur uuur uuuuruuur

10) Fie P punctul de concurenţă a trei ceviene de rang k, M un punct din planul unui

triunghi ABC. Atunci: 2 2 2 2 2 2

22

( ).

( )

k k k k k k k k k

k k k k k k

a MA b MB c MC a b c a b cMP

a b c a b c

− − −+ + + += −

+ + + +

Demonstraţie. Utilizând teorema precedentă avem: 2⋅ = =uuur uuurMP MP MP

2 2 2 2 2 22

1( 2 2 2 )

( )+ + + ⋅ + ⋅ + ⋅ ⋅

+ +

uuur uuur uuur uuuur uuur uuuurk k k k k k k k k

k k ka MA b MB c MC a b MA MB b c MB MC a c MA MC

a b c

Dar 2 2 2

2 2 21cos ( )

2 2

MA MB ABMA MB MA MB AMB MA MB MA MB AB

MA MB

+ −⋅ = ⋅ ⋅ = ⋅ ⋅ = + −

uuur uuur şi

analoagele. Înlocuind în relaţia precedentă va da relaţia cerută. Cazuri particulare: 1) Pentru 0,k P G= ≡ , relaţia din teoremă

devine:2 2 2 2 2 2

2

3 9

MA MB MC a b cMG

+ + + += − (relaţia lui Leibniz).

2) Pentru 1, ,k P I= ≡ relaţia devine 2 2 2

2 aMA bMB cMC abcMI

a b c

+ + −=

+ +.

3) Pentru 2, ,k P K= ≡ relaţia devine 2 2 2 2 2 2 2 2 2

22 2 2 2 2 2 2

3

( )

a MA b MB c MC a b cMK

a b c a b c

+ += −

+ + + +.

4) Pentru 1, ,k P Z= − ≡ relaţia devine 2 2 2 3 3 3

22

( ).

( )

bcMA acMB abMC abc a b cMZ

ab bc ac ab bc ac

+ + + += −

+ + + +

11) În triunghiul ABC fie cevienele de ordin k AD, BF şi CE ( )∈ k concurente într-un

punct P. Dacă ( )∈M AB şi ( )∈N AC dreapta MN trece prin P dacă şi numai dacă:

,k k kMB NCb c aMA NA⋅ + ⋅ = (unde a, b, c sunt lungimile laturilor BC, CA, respectiv AB).

Demonstraţie. Utilizând teorema transversalei în triunghiul ABC cu ceviana AD şi secanta

MN rezultă (1).MB DC NC BD PD

MA BC NA BC PA⋅ + ⋅ = Din relaţia lui Van-Aubel avem:

Page 268: Barbu_TEOREME_FUNDAMENTALE(2008).pdf

275

+ =AE AF AP

EB FC PD adică

+=

k k

k

b c AP

PDa, deci =

+

k

k k

PD a

AP b c. Din

k

k

DC b

BD c= rezultă

k

k k

DC b

BC b c=

+ şi (2).

k

k k

BD c

BC b c=

+ Din relaţiile (1) şi (2) rezultă .k k kMB NC

b c aMA NA⋅ + ⋅ =

Reciproc, fie .k k kMB NCb c aMA NA⋅ + ⋅ = Fie .= ∩R MN AD Atunci,

.MB DC NC BD RD

MA BC NA BC RA⋅ + ⋅ = Din ipoteză avem 1,

k kb MB c NC

a MA a NA

⋅ + ⋅ =

adică

1.AE MB AF NC

EB MA FC NA⋅ + ⋅ = Din teorema lui Menelaus aplicată în triunghiul ABD şi

transversala EC rezultă AE AP DC

EB PD BC= ⋅ şi analog .

AF AP BD

FC PD BC= ⋅ Atunci,

,MB DC NC BD PD

MA BC NA BC PA⋅ + ⋅ = deci ,

PD RD

PA RA= de unde rezultă că .P R≡

Cazuri particulare:

1) Dacă ,P G≡ atunci 0.k = Dreapta MN trece prin G dacă şi numai dacă 1.MB NC

MA NA+ =

2) Dacă ,P I≡ atunci 1=k . Dreapta MN trece prin I dacă şi numai dacă

.MB NC

b c aMA NA⋅ + ⋅ =

3) Dacă ,≡P K atunci 2=k . Dreapta MN trece prin punctul lui Lemoine al triunghiului

ABC dacă şi numai dacă 2 2 2 .MB NC

b c aMA NA⋅ + ⋅ =

4) Dacă P Z≡ (punctul de concurenţă al antibisectoarelor), atunci 1= −k . Dreapta MN

trece prin Z dacă şi numai dacă 1 1 1

.MB NC

b MA c NA a⋅ + ⋅ =

12) Orice ceviană de ordinul k este locul geometric al punctelor pentru care distanţele la două laturi ale triunghiului sunt proporţionale cu acele laturi la puterea (k-1). Demonstraţie. Fie AD o ceviană de ordinul k, ( ), ( )D BC M AD∈ ∈ , iar 1 2 3, ,M M M

proiecţiile lui M pe AC, AB, respectiv BC. Notăm cu x, y, z lungimile segmentelor

1 2,MM MM şi 3MM , iar cu 1ϕ şi 2ϕ măsurile unghiurilor

BAD , respectiv .CAD Avem: ,k

BD c

DC b

=

[ ] 1 1

[ ] 2 2

sin sin,

sin sinBAD

DAC

A AD cBD c

A DC AD b b

ϕ ϕϕ ϕ

⋅ ⋅= = = ⋅

⋅ ⋅ de unde

1

1

2

sin

sin

kc

b

ϕϕ

− =

(1) (Fig. 278). Din triunghiurile

dreptunghice 2AM M şi 3AM M rezultă 1sinz

AMϕ = şi

A

B C

2M

M

D

3M

Fig. 278

1M x

y z

1ϕ 2ϕ

Page 269: Barbu_TEOREME_FUNDAMENTALE(2008).pdf

276

2sin ,y

AMϕ = de unde 1

2

sin (2).

sin

z

y

ϕϕ

= Din relaţiile (1) şi (2) rezultă 1 1

.k k

z y

c b− −= Analog,

se arată că 1 1

,k k

z y

a b− −= de unde

1 1 1.

k k k

z y z

a b c− − −= =

13) Fie AD, BE, CF ceviene de ordinul k în triunghiul ABC, M AD BE CF= ∩ ∩ şi x, y, z proiecţiile lui M pe laturile BC, CA, respectiv AB. Atunci,

[ ]

1 1 1

2.ABC

k k k k k k

Az y z

a b c a b c− − −= = =

+ +

Demonstraţie: Din 1 1 1k k k

z y z

a b c− − −= = rezultă [ ]2

.ABC

k k k k k k k k k

Aax by cz ax by cz

a b c a b c a b c

+ += = = =

+ + + +

II.16. Teorema lui Descartes111

„Esenţa Matematicii constă în libertatea sa”- Georg Cantor112

1) Fie triunghiul ABC şi I centrul cercului înscris în acest triunghi. Există doar trei cercuri care să aibă centrele în vârfurile triunghiului, sunt tangente exterioare două câte două şi I are puteri egale faţă de cele trei cercuri. Demonstraţie. Fie ', ', 'A B C punctele de tangenţă dintre cele trei cercuri (Fig. 279). Evident, punctele de tangenţă sunt situate pe laturile triunghiului. Notând

1' 'AB AC r= = , 2' 'BA BC r= = şi 3' 'CA CB r= = obţinem

2 3 3 1 1 2, ,r r a r r b r r c+ = + = + = , de unde 1 2 3, ,r p a r p b r p c= − = − = − ( 1 2 3p r r r= + +

fiind semiperimetrul triunghiului ABC) relaţii care arată că cercurile sunt unic determinate. Punctele ', ', 'A B C coincid cu punctele de tangenţă ale cercului înscris cu laturile triunghiului (vezi „Cercul înscris”), deci I are puteri egale faţă de cele trei cercuri. 2) Consecinţă: Dacă r este raza cercului înscris în triunghiul ABC, atunci

2 1 2 3

1 2 3

.r r r

rr r r

=+ +

Demonstraţie. Din relaţiile 2[ ] 1 2 3 1 2 3( )( )( ) ( )ABCA p p a p b p c r r r r r r= − − − = + + şi [ ]ABC

A rp=

rezultă concluzia.

Fie patru cercuri C1 , C2 , C3 , C4 tangente două câte două în şase puncte distincte. Fiecărui

cerc îi corespunde numărul1

, 1,4i

i

iR

ε = = , undeiR reprezintă raza cercului Ci , i 1, 4.=

Dacă toate cercurile sunt tangente exterior, atunci numerelei

ε se consideră cu semnul „+”,

(Fig. 279), iar dacă trei dintre aceste cercuri sunt tangente interior celui de-al patrulea cerc,

111 René Descartes (1596-1650) – matematician şi filosof francez, contribuţii în geometrie 112 Georg Cantor (1845-1918) – matematician german, creator al teoriei mulţimilor

Page 270: Barbu_TEOREME_FUNDAMENTALE(2008).pdf

277

atunci număruli

ε corespunzător cercului ce are raza de lungime maximă este egală

cu1

= −i

iR

ε (Fig. 280) .

Teorema lui Descartes 2 2 2 2 21 2 3 4 1 2 3 42( ) ( )ε ε ε ε ε ε ε ε+ + + = + + + .

Demonstraţie. Cercurilor C1 , C2 , C3 , C4 le corespund cercurile ' ' ' '1 2 3 3, , ,C C C C , tangente două

câte două în şase puncte (Fig. 281) şi notăm cu i

η numerele '

1, 1,4=

i

iR

, unde 'iR sunt

razele cercurilor ' , 1, 4=iC i . Cercul '

1C - de exemplu – are trei puncte de tangenţă în comun

cu cercurile C2 , C3 , C4 ; analog se definesc cercurile ' ' '2 3 3, ,C C C . Dacă , ,A B C sunt centrele

cercurilor C1 , C2 , respectiv C3 , atunci '4C este cercul înscris sau un cerc exînscris al

triunghiului ABC . Corespunzător primului caz (Fig.279) avem:

41 2 3

1 1 1 1, , ,= = = = −

− − −p a p b p c rε ε ε η , iar corespunzător celui de-al doilea caz

avem: 41 2 3

1 1 1 1, , ,= − = = = −

− −a

p p c p b rε ε ε η .Atunci: 2 3 1 2 1 3 1 2 3

1 2 3

1 1 1 + + = + + ⋅ =

ε ε ε ε ε ε ε ε ε

ε ε ε

242

( ) ( ) ( ) 1

( )( )( ) ( )( )( )

− + − + −= = =

− − − − − −p a p b p c p

p a p b p c p a p b p c rη (în primul caz); sau

A

B

C p p b−

p c−

Fig. 280

A B

C

p a−

p b−

p c−

Fig. 279

A'

B'

C'

Page 271: Barbu_TEOREME_FUNDAMENTALE(2008).pdf

278

22 3 1 2 1 3 4( )( ) ( ( )

−− −+ + = = =

− − − − −p ap b c

p p c p b p p b p cε ε ε ε ε ε η , deci 2

2 3 1 2 1 3 4ε ε ε ε ε ε η+ + = .

Analog se arată că 2 3 1 1 2

23 4η η η η ηη ε+ + = , iar permutând indicii se obţine altă relaţie

derivată. Atunci, 4 4

2 2 2 2 2 2 21 2 3 4 1 2 3 4

1 4 1 1

( ) 2 i j i i

i j i i

ε ε ε ε ε ε ε ε ε ε ε η≤ < ≤ = =

+ + + = + + + + = + =∑ ∑ ∑

21 2 3 4( )η η η η+ + + (datorită simetriei relaţiilor precedente), de unde

1 2 3 4 1 2 3 4 0ε ε ε ε η η η η+ + + = + + + > , 2 21 2 3 4 1 2 3 4 1 2 3 4( )( ) ( )ε ε ε ε ε ε ε ε ε ε ε ε+ + − + + + = + + − =

2 2 2 2 21 2 3 4 4 2 3 3 4 4 2 1 3 3 4 4 1 1 2 2 4 1 42 ( ) ( ) ( )ε ε ε ε η η η η η η η η η η η η η η η η η ηη+ + − − = + + + + + + + + −

2 21 2 2 3 3 1 4 1 4 2 4 3 4 4 4 1 2 3 4 4 1 2 3 4( ) 2 2( ) 2 2 ( ) 2 ( )ηη η η ηη η ηη η η ηη η η η η η η η ε ε ε ε+ + + = + + + = + + + = + + +

de unde 1 2 3 4 42ε ε ε ε η+ + − = . Analog se arată că

1 2 3 4 12ε ε ε ε η− + + + = , 1 2 3 4 22ε ε ε ε η− + + = − , 1 2 3 4 32ε ε ε ε η+ − + = − . Ridicând relaţiile

precedente la pătrat şi sumându-le rezultă: 2 2 2 2 2 2 2 21 2 3 4 1 2 3 4ε ε ε ε η η η η+ + + = + + + , de unde

4 42 2 2 2 2 2 21 2 3 4 1 2 3 4

1 1

2( ) ( )i i

i i

ε ε ε ε ε η ε ε ε ε= =

+ + + = + = + + +∑ ∑ .

Soluţia 2. Fie A,B,C,D centrele cercurilor C1 , C2 , C3 , respectiv C4 şi 1 2 3 4, , ,R R R R razele

acestor cercuri. Semiperimetrul triunghiului BCD este egal cu 2 3 4+ +R R R . Din teorema

cosinusului şi formula unghiului pe jumătate rezultă:

2 4 2 3 4

2 4 3 4

( )1 coscos

2 2 ( )( )

+ ++= =

+ +

R R R RBDC BDC

R R R R şi

2 2 3

2 4 3 4

1 cossin

2 ( )( )

−= =

+ +

R RBDC

R R R R. Utilizând egalitatea

2 2 2sin sin sin 2sin sin sin 0− − + =x y z x y z pentru

C1

C3

C4

4

'2C

'4C '

3C

C2

'1C

Fig. 281

Page 272: Barbu_TEOREME_FUNDAMENTALE(2008).pdf

279

1 1 1( ), ( ), ( )

2 2 2= = = x m BDC y m ADC z m ADB relaţia precedentă devine:

1 2 3 4 2 3 42 3 1 3 1 2

2 4 3 4 1 4 3 4 1 4 2 4 1 4 2 4 3 4

2 ( )0

( )( ) ( )( ) ( )( ) ( )( )( )

+ +− − + =

+ + + + + + + + +

R R R R R R RR R R R R R

R R R R R R R R R R R R R R R R R R

sau 3 4 4 2 3 41 4 2 4

1 2 3 2 3

( )2 0

+ + ++ +− − + =

R R R R R RR R R R

R R R R R, egalitate echivalentă cu

1 2 3 4 2 3 3 4 4 22 0− − − + + + =ε ε ε ε ε ε ε ε ε ε . Atunci 2 21 2 3 4 1 2 3 4( ) ( )+ + + = − − − +ε ε ε ε ε ε ε ε

2 2 2 2 21 2 1 3 1 4 2 3 3 4 4 2 1 2 1 3 1 4 1 2 3 4 1 2 3 44( ) 4( ) 4( ) 2( ) 2( )+ + + + + + + + = + + + − + + +ε ε ε ε ε ε ε ε ε ε ε ε ε ε ε ε ε ε ε ε ε ε ε ε ε ε

de unde rezultă 2 2 2 2 21 2 3 4 1 2 3 42( ) ( ) .+ + + = + + +ε ε ε ε ε ε ε ε

Observaţie: Cantităţile iε şi

iη , 1, 4i = se numesc numerele lui Beecroft..

Consecinţe: i) 1 1 2 2 3 3 4 4ε η ε η ε η ε η+ = + = + = + .

ii) 1 1 2 2 3 3 4 4 0ε η ε η ε η ε η⋅ + ⋅ + ⋅ + ⋅ = .

Demonstraţie.

i) Avem 1 2 3 4 12ε ε ε ε η− + + + = , deci 1 2 3 4 1 1 2 2

1( )

2ε ε ε ε η ε ε η+ + + = + = + =

3 3 4 4 .ε η ε η+ = +

ii) Avem: 24 4 4 4 4

2 2

1 1 1 1 1

1( ) 0

2i i i i i i i i

i i i i i

ε η ε ε η ε ε ε= = = = =

= + − = + =

∑ ∑ ∑ ∑ ∑ .

II.17. Teorema lui Pompeiu113 „Dimitrie Pompeiu ştia să privească lucrurile vechi cu ochi noi.” – Paul Montel114

Fie triunghiul echilateral ABC şi M un punct în planul sau ce nu aprţine cercului circumscris triunghiului. Distanţele MA, MB, MC reprezintă lungimile laturilor unui triunghi. Demonstraţie. Soluţia 1. Fie 'M punctul obţinut din M prin rotaţia de centru A şi unghi de 60º. Atunci 'MM MA≡ (deoarece triunghiul AMM’ este echilateral). Din congruenţa triunghiurilor BAM şi 'CAM ( 'AM AM≡ , BA CA≡ ,

) )( ( 'm BAM m CAM= = )60 (m CAM° + ), rezultă

',MB CM≡ deci lungimile laturilor triunghiului

113 Dimitrie Pompeiu (1873-1954) –matematician român, profesor la Universitatea din Iaşi, membru al Academiei

Române, contribuţii importante în analiza matematică 114 Paul Montel (1876-1975) – matematician francez, membru al Academiei Franceze, contribuţii în analiza

matematică

A

B C

M

M '

Fig. 282

60°

Page 273: Barbu_TEOREME_FUNDAMENTALE(2008).pdf

280

'MM C sunt egale cu cele ale segmentelor MA, MB, MC (Fig.282). Observaţie: Dacă punctul M se află pe centrul cercului circumscris triunghiului echilateral ABC, atunci conform teoremei lui Schooten segmentul cu cea mai mare lungime dintre segmentele MA, MB, şi MC au lungimea egală cu suma lungimilor celorlalte două. Soluţia 2. Notăm cu litere mici afixele punctelor corespunzătoare. Plecând de la relaţia evidentă: ( ) ( ) ( ) ( ) ( ) ( ) 0 (1)m a b c m b c a m c a b− − + − − + − − = rezultă:

( )( ) ( )( ) ( )( )m a b c m b c a m c a b− − = − − − − − − . Trecând la modul în egalitatea precedentă

obţinem: ( )( ) ( )( )m a b c m a c a m c a b− − = − − + − − ≤ m b c a m c a b− − + − − , de

unde: m a m b m c− ≤ − + − adică .MA MB MC≤ + Cum M nu aparţine cecului

circumscris triunghiului ABC rezultă .MA MB MC< + Din simetria relaţiei (1) rezultă inegalităţile MB MC MA< + şi MC MA MB< + , adică segmentele MA, MB, MC determină un triunghi.

II.18. Teorema lui Erdös – Mordell „Nu poate exista un limbaj mai universal şi mai simplu, mai lipsit de greşeli şi de confuzii, adică mai demn de a exprima raporturile invariabile dintre realităţile naturale. Matematica este tot atât de cuprinzătoare ca însăşi natura. Ea defineşte toate raporturile sensibile, măsoară timpul, spaţiile, forţele şi temperaturile. Ştiinţa aceasta dificilă se formează cu încetul, dar păstrează toate principiile odată ce şi le-a însuşit. Ea creşte şi se consolidează fără încetare, în mijlocul atâtor erori ale spiritului uman.” - Baptiste Joseph Fourier 115

Dacă P este un punct în interiorul unui triunghi ABC , atunci

1 1 12( )PA PB PC PA PB PC+ + ≥ + + , unde 1 1 1, ,A B C sunt proiecţiile punctului P pe

laturile BC, AC, BA ale triunghiului ABC . Demonstraţie. Soluţia 1. Notăm cu

', ', 'a b c lungimile segmentelor PA, PB, PC şi cu x, y, z lungimile segmentelor

1 1 1, , .PA PB PC Din teorema cosinusului

în triunghiul 1 1PBC rezultă:

2 21 1 2 cos= + +BC y z yz A (1). Cum

patrulaterul 1 1AC PB este inscriptibil

rezultă 1 1 1 (2).ABC APC= Deoarece

11sinAC

APCAP

= , atunci 1 1'sin

= =BC

PA aA

de unde 2 2 2 cos

'sin

+ +=y z yz A

aA

.

Analog se arată că 2 2 2 cos

' ,sin

+ +=x z xz B

bB

2 2 2 cos

'sin

+ +=x y xy C

cC

, de unde rezultă:

115 Baptiste Joseph Fourier (1768-1830) – matematician şi fizician francez, membru al Academiei Franceze,

contribuţii în toate domeniile matematicii

A

B C 1A

1B 1C

P

a '

b ' c '

x

y z

Fig. 283

2C

2B

Page 274: Barbu_TEOREME_FUNDAMENTALE(2008).pdf

281

2 2( s in sin ) ( cos cos )' ' '

s in

+ + −+ + = + + = +

y C z B y C z BPA PB PC a b c

A

2 2 2 2( sin sin ) ( cos cos ) ( sin sin ) ( cos cos )

sin sin

z A x C z A x C x B y A x B y A

B C

+ + − + + −+ + şi de

aici avem: sin sin sin sin sin sin

' ' 'sin sin sin

+ + ++ + ≥ + +

y C z B z A x C x B y Aa b c

A B C,

adică sin sin sin sin sin sin

' ' 'sin sin sin sin sin sin

+ + ≥ + + + + +

C A B A B Ca b c y z x

A C A B C B, deci

' ' ' 2( )+ + ≥ + +a b c x y z (unde am utilizat inegalitatea 2, , 0x y

x yy x+ ≥ ∀ > ), cu egalitate

dacă triunghiul ABC este echilateral. Soluţia2. Fie 2B şi 2C proiecţiile punctelor B şi C pe dreapta 1 1.BC Avem

2 2 2 1 1 1 1 2 (1')≥ = + +BC B C B C C B B C . Cum 2 1 1 1,B C B AC P APB= = rezultă că

triunghiurile dreptunghice 2 1BB C şi 1AB P sunt asemenea, de unde rezultă că

12 1 1 (2')= ⋅

BCB C PB

APşi analog se arată că 1

1 2 1 (3')= ⋅B C

BC PCAP

. Cum patrulaterul

1 1AC PB este inscriptibil, din inegalitatea lui Ptolemeu rezultă:

1 1 1 1 1 1,AC PB AB PC AP BC⋅ + ⋅ = ⋅ de unde 1 11 1 1 1 (4')= ⋅ + ⋅

PB PCBC AC AB

AP AP. Din relaţiile

(1'), (2 '), (3') şi (4 ') rezultă: 1 1 1 11 1 1 1≥ ⋅ + ⋅ + ⋅ + ⋅PB PB PC PC

BC BC AC AB BCAP AP AP AP

, de unde :

1 1 1 1 1 1( ) ( )⋅ ≥ + + +BC AP PB BC C A PC AB BC inegalitate echivalentă cu

1 1⋅ ≥ ⋅ + ⋅BC AP PB AB PC AC , deci 1 1 (5')≥ ⋅ + ⋅AB AC

AP PB PCBC BC

. Urmărind acelaşi

raţionament se obţin inegalităţile: 1 1 (6')≥ ⋅ + ⋅BA BC

BP PA PCAC AC

şi

1 1 (7')≥ ⋅ + ⋅AC BC

PC PA PBAB AB

. Sumând inegalităţile ( 5 ') , ( 6 ') , ( 7 ')

şi ţinem cont de inegalitatea 2 , , 0+ ≥ ∀ >x y

x yy x

, rezultă:

1 2 2 1 1 12( )AB AC AB BC AC BC

PA PB PC PA PB PC PA PB PCAC AB BC AB BC CA

+ + ≥ + ⋅ + + ⋅ + + ⋅ ≥ + +

Consecinţe: 1) Dacă triunghiul ABC este ascuţitunghic şi H este ortocentrul triunghiului ABC , atunci 6+ + ≥HA HB HC r , unde r este raza cercului înscris în triunghiul ABC . Demonstraţie. Dacă triunghiul ABC este ascuţitunghic şi P coincide cu ortocentrul H al triunghiului ABC din inegalitatea lui Erdös obţinem:

1 1 12( ) (4)HA HB HC HA HB HC+ + ≥ + + . Dacă , ,a b ch h h sunt lungimile înălţimilor

triunghiului ABC , atunci (4) devine: 2( )+ + ≥ − + − + −a b c

HA HB HC h HA h HB h HC ,

Page 275: Barbu_TEOREME_FUNDAMENTALE(2008).pdf

282

adică 2

( ) (5)3

+ + ≥ + +a b c

HA HB HC h h h , de unde se obţine inegalitatea

2 2 2 2 4 1 1 1.

3 3 + + ≥ + + = + +

S S S SHA HB HC

a b c a b c Aplicând inegalitatea mediilor

rezultă 4 9 12

63 2

+ + ≥ ⋅ = =+ +

S SHA HB HC r

a b c p, deci: 6 (5)+ + ≥HA HB HC r .

2) Într-un triunghi ascuţitunghic ABC, 3

cos cos cos .2

+ + ≤A B C

Demonstraţie. Dacă P coincide cu centrul cercului circumscris (O) al triunghiului

ascuţitunghic ABC , avem: 2 2 2

2 2 21 1 13 2( ) 2

4 4 4

≥ + + = − + − + −

a b cR OA OB OC R R R ,

adică: 2 2 2 2 2 23 4 4 4≥ − + − + −R R a R b R c inegalitate echivalentă cu

3 2 (cos cos cos )≥ + +R R A B C , deci 3

cos cos cos (6)2

A B C+ + ≤ (unde am ţinut cont de

2 sin=a R A şi de relaţiile analoage). 3) Dacă triunghiul ABC este ascuţitunghic şi H este ortocentrul triunghiului atunci

3+ + ≤HA HB HC R .

Demonstraţie. Avem: 2 (cos cos cos ) 2 3 / 2 3 (7)+ + = + + ≤ ⋅ =HA HB HC R A B C R R , unde

am utilizat relaţia (6). 4) În orice triunghi ABC este adevărătă relaţia: 2r R≤ (Relaţia lui Euler). Demonstraţie. Din relaţiile (5) şi (7) rezultă 6 3 ,r HA HB HC R≤ + + ≤ de unde rezultă

relaţia lui Euler: 2 (8)r R≤ .

5) Dacă I este centrul cercului înscris în triunghiul ABC, atunci 6 4 2r AI BI CI R r≤ + + ≤ − . Demonstraţie. Dacă P coincide cu centrul cercului înscris I al triunghiului ABC , atunci

relaţia lui Erdös devine: 6 (9)AI BI CI r+ + ≥ . Cu sin

2

rAI

A= şi analoagele rezultă

sin sin sin sin sin sin1 1 1 2 2 2 2 2 2

sin sin sin sin sin sin2 2 2 2 2 2

⋅ + ⋅ + ⋅ + + = + + =

⋅ ⋅

A B B C C A

AI BI CI r rA B C A B C

.

Ţinând cont că sin sin sin2 2 2 4

A B C r

R⋅ ⋅ = şi de inegalitatea

2 2 2 , , ,x y y z z x x y z x y z+ + ≤ + + ∀ ∈ rezultă:

2 2 24 1 cos 1 cos 1 cossin sin sin 4

2 2 2 2 2 2

− − − + + ≤ ⋅ + + = + +

R A B C A B CAI BI CI r R

r de

unde: ( )2 3 cos cos cos 2 3 1 4sin sin sin2 2 2

+ + ≤ − + + = − −

A B CAI BI CI R A B C R ,

Page 276: Barbu_TEOREME_FUNDAMENTALE(2008).pdf

283

deci 2 2 4 2 (10) + + ≤ − = −

rAI BI CI R R r

R. Din relaţiile (9) şi (10) rezultă:

6 4 2r AI BI CI R r≤ + + ≤ − . 6) Dacă G este centrul de greutate al triunghiului ABC, atunci 6 .+ + ≥GA GB GC r

Demonstraţie. Dacă punctul P coincide cu centrul de greutate al triunghiului ABC , relaţia lui Erdös devine: 1 1 12( )+ + ≥ + +GA GB GC GA GB GC . Ţinând cont de faptul că

1

1

3 aGA h= şi de relaţiile analoage avem:

( )2 2 2 2 2

3 3a b c

S S SGA GB GC h h h

a b c

+ + ≥ + + = + +

, adică:

4 1 1 1 4 9 126

3 3 2

S S SGA GB GC r

a b c a b c p

+ + ≥ + + ≥ ⋅ = = + + .

7) Dacă P este un punct în interiorul unui triunghi ABC , atunci

*1 1 12( ),+ + ≥ + + ∀ ∈

m m m m m mPA PB PC PA PB PC m , unde 1 1 1, ,A B C sunt proiecţiile

punctului P pe laturile BC, AC, BA ale triunghiului ABC .

Demonstraţie. Din teorema lui Erdös rezultă: sin sin

sin sin

C BPA y z

A A≥ + şi analoagele.

Obţinem: sin sin sin sin sin sin

sin sin sin sin sin sin

m m m m m mm m m m m m

m m m m m m

C A C B B APA PB PC x y z

A C B C A B

+ + ≥ + + + + + ≥

( ) *2 ,m m mx y z m≥ + + ∀ ∈ unde am aplicat inegalitatea dintre media aritmetică şi

geometrică.

8) Generalizarea teoremei lui Erdös - Mordell Fie P un punct arbitrar în planul triunghiului ABC şi ', ', 'a b c distanţele de la P la vârfurile A,B respectiv C şi , ,x y z distanţele de la P la laturile BC, CA, respectiv AB.

Atunci, ' ' ' + + ≥ + + + + +

b c c a a ba b c x y z

c b a c b a, cu egalitate dacă P este centrul

cercului circumscris triunghiului ABC . Demonstraţie. Fie 1h lungimea înălţimii duse din A pe latura BC. Avem:

[ ] 12 .= ⋅ = + +ABCA a h ax by cz Evident, 1'+ ≥a x h cu egalitate dacă şi numai dacă punctul P

aparţine înălţimii din A. Avem 1'+ ≥ = + +aa ax ah ax by cz , de unde ' (11)≥ +aa by cz .

A

B C

B'

C'

P x O x

Fig. 284

Page 277: Barbu_TEOREME_FUNDAMENTALE(2008).pdf

284

Fie ' 'AB C simetricul triunghiului ABC faţă de bisectoarea unghiului A (Fig.284 ). Atunci, aplicând inegalitatea (11) pentru triunghiul ' 'AB C obţinem: ' ≥ +aa cy bz adică

' z (12)≥ +c b

a ya a

cu egalitate dacă punctul P aparţine înălţimii din A a

triunghiului ' 'AB C , dreaptă ce trece prin centrul cercului circumscris triunghiului ABC .

Analog, se obţin relaţiile: ' x (13)≥ +a c

b zb b

şi ' y (14)≥ +b a

c xc c

. Sumând inegalităţile

(12), (13) şi (14) rezultă concluzia. Observaţie: Dacă P este un punct interior triunghiului ABC , , , 0>x y z , avem:

' ' ' 2( )+ + ≥ + +a b c x y z (unde am utilizat faptul că 2, , 0a b

a bb a+ ≥ ∀ > ) care este

inegalitatea lui Erdös-Mordell. Egalitatea are loc dacă = =a b c , adică dacă triunghiul ABC este echilateral şi P este centrul cercului circumscris triunghiului ABC .

Teorema lui Barrow

9) Dacă P este un punct interior triunghiului ABC , atunci 2( ' ' ')PA PB PC PA PB PC+ + ≥ + + , unde ', ', 'P A P B P C sunt bisectoarele

unghiurilor , ,B P C C P A A P B ( ' ( ), ' ( ), ' ( ))A BC B AC C AB∈ ∈ ∈ .

Demonstraţie. Notăm cu a,b,c lungimile segmentelor PA,PB respectiv PC şi cu

, ,α β γ măsurile unghiurilor , , BPC CPA

respectiv APB . Prin [ ]XYZA notăm aria

triunghiului XYZ. Din

[ ] [ '] [ ']= +P B C P B A P C AA A A rezultă:

sin 'sin 'sin2 2

⋅ = ⋅ + ⋅PB PC PB PA PC PAα α

α ,

deci 2

' cos2

=+

bcPA

b c

α. Utilizând

inegalitatea 2

, , 01 1

xy x y

x y

≤ >+

, rezultă

' cos2

PA bcα

≤ şi analoagele.Atunci, 2( ' ' ') 2 cos 2 cos 2 cos2 2 2

PA PB PC bc ca abα β γ

+ + ≤ + +

(*). Rămâne să demonstrăm că 2 cos 2 cos 2 cos2 2 2

a b c bc ac bcα β γ

+ + ≥ + + .

Deoarece 2+ + =α β γ π rezultă cos cos cos cos sin sin ,2 2 2 2 2 2 2

= − + = − +

γ α β α β α β

inegalitatea de mai sus fiind echivalentă cu: 2

sin sin cos cos 02 2 2 2

a b a b cβ α β α − + − − ≥

, ceea ce este adevărat.

A

B C

P

Fig. 285

B' C'

A'

Page 278: Barbu_TEOREME_FUNDAMENTALE(2008).pdf

285

Egalitatea din ( )∗ se obţine pentru = =a b c , adică dacă P este centrul cercului

circumscris triunghiului. Generalizare a teoremei lui Erdös116

Fie 1 2 3, ,λ λ λ +∈ şi [ ]0,1 ,t∈ iar P un punct în interiorul triunghiului .ABC Se notează

distanţele PA, PB, PC cu 1 2,x x respectiv 3x şi cu 1 2 3, ,d d d distanţele de la laturile AB,

BC, respectiv CA. Atunci: 31 21 1 2 2 3 3 1 2 3

1 2 3

2tt t

t t t t dd dx x xλ λ λ λ λ λ

λ λ λ

+ + ≥ + +

cu egalitate

dacă şi numai dacă 31 22 2 2t t ta b c

λλ λ= = şi P este centrul cercului circumscris triunghiului

.ABC

Demonstraţie. Se cunoaşte că 1 1 3 2 3 1 3 1 2, ,c b a c b b

x d d x d d x d da a b b c a

≥ + ≥ + ≥ + .

Pentru 0 1t< < rezultă 1 31 3

1 2 22 2

t ttt t

t t t

c bc b d dd da aa ax

⋅ + ⋅+ ≥ ≥ ⋅

şi analoagele.

Utilizând inegalitatea 1

2xx

+ ≥ pentru 0x > rezultă: 1 1 2 2 3 3t t tx x xλ λ λ+ + ≥

2 3 3 1 1 2

1 2 322 2 2

t t t t t t

t t t t

c b a c b a

b c c a a bd d d

λ λ λ λ λ λ ⋅ + ⋅ ⋅ + ⋅ ⋅ + ⋅ ⋅ + ⋅ + ⋅ ≥

2 3 1 3 2 1 2 32 ( 1 )t t t td d dλ λ λ λ λ λ⋅ + ⋅ + ⋅ .

Observaţie: Dacă 1t > atunci, 31 21 1 2 2 3 3 1 2 3

1 2 3

2tt t

t t t dd dx x xλ λ λ λ λ λ

λ λ λ

+ + ≥ + +

Consecinţe:

1) 31 21 2 3

1 2 3 1 1 2 2 3 3

1 1 12t

t t t t t td d d x x x

λλ λλ λ λ

λ λ λ

+ + ≥ + +

2) ( ) ( ) ( ) ( )

1 2 33 31 2 1 2

1 2 31 1 2 2 3 3 1 2 3

2t tt t

t t t t

xx x

x d x d x d x x x

λ λ λλλ λ

λ λ λ

+ + ≥ + +

3) ( ) ( ) ( ) ( )1 1 1 2 2 2 3 3 3 1 2 3 1 2 3

1 1 2 2 3 3

1 1 12

t t t tt

t t tx d x d x d d d d

d d dλ λ λ λ λ λ

λ λ λ

+ + ≥ + +

116 Paul Erdös (1913-1996) – matematician ungur, profesor la Universitatea Notre Dame, contribuţii importante în

teoria numerelor şi matematici discrete

Page 279: Barbu_TEOREME_FUNDAMENTALE(2008).pdf

286

4) ( )1 1 2 2 3 3 1 2 3 1 2 3

1 1 2 2 3 3

1 1 12

tt t t t

t t td d d d d d

d d dλ λ λ λ λ λ

λ λ λ

+ + ≥ + +

i) Dacă P este centrul cercului înscris în triunghiul ABC , atunci 1 2 3d d d r= = = şi

1 2 3cos , cos , cos .2 2 2

A B Cx r x r x r= = = Din consecinţa (2) şi utilizând egalitatea

sin sin sin2 2 2 4

A B C r

R= obţinem: 1 2 3sin sin sin

2 2 2t t tA B C

λ λ λ+ + ≥

1 2 31 2 3

2 1 1 1cos cos cos

2 2 2 2

t

t t tA B C

Rλ λ λ

λ λ λ + +

( (0,1]).t∈

ii) Dacă P este centrul de greutate al triunghiului ABC , atunci 3i

i

hd = (

ih , 1,3i = reprezintă

înălţimile ABC ,) şi 1 2 3

2 2 2, , .

3 3 3a b cx AG m x m x m= = = = Din consecinţa (1) pentru

(0,1]t∈ rezultă: 31 2

1 2 3 1 2 3

1 1 1t t t t t t

a b ch h h m m m

λλ λ

λ λ λ

+ + ≥ + +

. Dacă înlocuim t

i iλ λ=

obţinem: 2 3 3 1 1 2 3

t t t

a b c

h h h h h h

m m m

+ + ≤

II.19. Teoremele lui Fagnano117 „În univers urla un punct de durerea unui cerc

care-l înconjoară.” N. Stănescu118

1) Fiind dat triunghiul ascuţitunghic ABC să se determine triunghiul înscris XYZ în triunghiul ABC a cărui perimetru este minim. Demonstraţie. Soluţia 1. Fie 'X şi ''X simetricele punctului ( )X BC∈ faţă de laturile

AC, respectiv AB (Fig. 286). Atunci, ''XZ ZX= şi 'XY YX= , deci perimetrul triunghiului XYZ este egal cu ' ''XY YZ ZX X Y YZ ZX+ + = + + . Dacă X este fixat pe BC, atunci triunghiul cu perimetrul minim înscris în triunghiul ABC se obţine atunci când punctele Y şi Z aparţin dreptei ' "X X . Deoarece AB şi AC sunt mediatoarele segmentelor "XX , respectiv 'XX rezultă 'YAX YAX≡ şi "XAZ ZAX≡ , de unde ( ' ") 2[ ( ) ( )] 2 ( ).m X AX m YAX m XAZ m BAC= + =

Cum ' ''AX AX AX= = rezultă că triunghiul

117 Giovanni Fagnano (1715-1797) – matematician italian, contribuţii în geometrie 118 Nichita Stănescu (1933 – 1983) – eseist, poet român, ales postum membru al Academiei Române

A

B C

Y

X

X'

X"

Fig. 286

Z

Page 280: Barbu_TEOREME_FUNDAMENTALE(2008).pdf

287

' "AX X este isoscel. Cum unghiul ' "X AX este constant pentru orice alegere a lui X rezultă că toate triunghiurile ' "AX X sunt asemenea. Latura ' "X X are lungimea minimă atunci când latura 'AX are lungimea minimă. Dar ' "AX AX AX= = care are lungimea minimă atunci când X este proiecţia lui A pe BC. Analog, Y este proiecţia lui B pe AC şi Z este proiecţia lui C pe AB. Perimetrul minim al unui triunghi înscris este acela al triunghiului ortic.

Observaţie: Dacă ( ) 90m A = ° , atunci punctele Z şi Y coincid cu A, iar dacă

( ) 90m BAC > ° , atunci triunghiul XYZ este triunghiul degenerat XAA.

Soluţia 2.

Fie a

AH ,b

BH ,c

CH înălţimile triunghiului ABC,H ortocentrul său şi

punctele ( )X BC∈ , ( )Y AC∈ , ( ),Z AB∈

(Fig. 287). Avem:

b c c a a b

b c c a a b

Y Z H H X Z H H X Y H HY Z Z X X Y

H H H H H H≥

⋅ ⋅ ⋅+ + = + +

b c c a a b

b c c a a b

YZ H H XZ H H X Y H H

H H H H H H

⋅ ⋅ ⋅+ +

uur uuuuuur uuur uuuuuuur uuur uuuuuuur

sau YZ ZX XY+ + ≥

( )b b c c b c

b c

YH H H H Z H H

H H

+ + ⋅+

uuuur uuuuuur uuuur uuuuuur( ) ( )

c c a a c a a a b b a b

c a a b

ZH H H H X H H XH H H H Y H H

H H H H

+ + ⋅ + + ⋅+ =

uuuur uuuuuur uuuuur uuuuuur uuuuur uuuuuur uuuur uuuuuur

a b a c b c b a c a c bb c c a a b a b c

a b a c b c b a c a c b

H H H H H H H H H H H HH H H H H H XH YH ZH

H H H H H H H H H H H H

+ + + + + + + +

uuuuuur uuuuuur uuuuuur uuuuuur uuuuuur uuuuuuruuuuur uuuur uuuur

(1). Deoarece triunghiul ABC este ascuţitunghic, înălţimile sale sunt bisectoarele

triunghiului ortic, deci vectorii a b a c

a b a c

H H H H

H H H H+

uuuuuuur uuuuuuur

, b c b a

b c b a

H H H H

H H H H+

uuuuuur uuuuuuur

, c a c b

c a c b

H H H H

H H H H+

uuuuuuur uuuuuur

sunt

perpendiculari pe vectorii aXHuuuuur

, bYHuuuur

, respectiv cZHuuuur

, deci din relaţia (1) rezultă

b c c a a bXY ZX XY H H H H H H+ + ≥ + + (2). Dacă vectorii YXuur

, ZXuur

, XYuur

au aceeaşi

direcţie cu vectorii b cH Huuuuuur

, c aH Huuuuuuur

, respectiv a bH Huuuuuuur

, atunci există numerele pozitive

, ,α β γ astfel încât , , .b c c a a bYZ H H Z X H H X Y H Hα β γ= = =uur uuuuuur uuur uuuuuuur uuur uuuuuuur

Avem

0b c c a a bH H H H H Hα β γ+ + =uuuuuur uuuuuuur uuuuuuur r

şi 0b c c a a bH H H H H H+ + =uuuuuur uuuuuuur uuuuuuur r

, de unde rezultă că

Ha

Hb

A

Fig. 287

B

Hc

C

H

Z

Y

X

Page 281: Barbu_TEOREME_FUNDAMENTALE(2008).pdf

288

A

B C

M

D

G

Fig. 288

α β γ= = , deci , ,b c c a a bYZ H H ZX H H XY H Hα α α= = =uur uuuuuur uuur uuuuuuur uuur uuuuuuur

, ceea ce implică

, , .b c c a a bYZ H H ZX H H XY H Hα α α= = = Astfel, ( )b c c a a bYZ ZX XY H H H H H Hα+ + = + + (3)

care cu relaţia (2) ne dă 1α = . Atunci , , ,b c c a a bYZ H H ZX H H XY H H= = =uur uuuuuur uuur uuuuuuur uuur uuuuuuur

,ceea ce

înseamnă că punctele X, Y, Z coincid cu punctele , ,a bH H respectiv cH . În concluzie,

cel mai mic perimetru al unui triunghi înscris este cel al triunghiului ortic.

2) Fie M un punct variabil în planul triunghiului ABC. Suma 2 2 2MA MB MC+ + este minimă dacă şi numai dacă M coincide cu centrul de greutate al triunghiului ABC. Demonstraţie. Fie G centrul de greutate al triunghiului ABC şi a, b, c lungimile laturilor triunghiului. Din teorema lui Leibniz rezultă:

2 2 2 2 2 2 213 ( ) 0,

3MA MB MC MG a b c+ + = + + + ≥ minimul sumei se realizează atunci

când punctul M coincide cu G. 3) Fie x, y şi z distanţele de la un punct M situate în interiorul triunghiului ABC la laturile BC, CA respectiv AB. Produsul x y z⋅ ⋅ este minim dacă M coincide cu centrul de greutate G al triunghiului ABC. Demonstraţie. Produsul x y z⋅ ⋅ este minim când produsul

( ) ( ) ( ) ( ) ( )abc xyz ax by cz⋅ = ⋅ ⋅ este minim. Cum [ ]2ABC

ax by cz A+ + = ⋅ rezultă că minimul

se realizează atunci când ,ax by cz= = adică când M coincide cu G.

Page 282: Barbu_TEOREME_FUNDAMENTALE(2008).pdf

289

II.20. Dreapta lui Droz – Farny119

„Dacă cineva va găsi demonstraţia axiomei paralelelor, ar merita un diamant cât Pământul de mare. Celui care îi va reuşi aceasta, acestuia, muritori, să-i ridicaţi un monument nepieritor.” – Farkaş Bolyai120

Teorema lui Droz - Farny

Fie 'd şi "d două drepte perpendiculare ce trec prin ortocentrul H al unui triunghi .ABC Fie 'A şi "A , 'B şi ",B 'C şi "C punctele de intersecţie ale dreptelor 'd şi

"d cu laturile BC, AC, respectiv AB. Să se arate că mijloacele segmentelor ' "A A , ' "B B şi ' "C C sunt trei puncte coliniare. Demonstraţie. Soluţia 1. Dacă triunghiul ABC este dreptunghic, teorema este evidentă. Presupunem că triunghiul ABC este oarecare. Fie C cercul circumscris triunghiului ABC ,

, ,α β γ mijloacele segmentelor ' "A A , ' "B B , respectiv ' "C C . Fie Ca, Cb, Cc cercurile

circumscrise triunghiurilor ' "HA A , ' "HB B , respectiv ' "HC C şi (respectiv , )a b cH H H simetricele punctului H faţă de dreapta BC (respectiv CA, AB).

Cercurile Ca, Cb, Cc au centrele în punctele , ,α β respectiv γ . Cum ∈aH C, ∈H Ca şi ' "A A

este diametrul în Ca, , rezultă că punctul aH ∈ Ca , deci punctul aH aparţine cercurilor C şi

Ca şi perpendicularei duse din H pe BC. Analog, bH aparţine cercurilor C şi Cb precum şi

dreptei BH. Fie punctul cH simetricul lui H faţă de AB. Punctul cH ∈C. Din lemă rezultă

că dreptele ', 'a bH A H B şi 'cH C se intersectează în punctul N ∈C. Din teorema lui Miquel

aplicată triunghiului ' 'A NB cu ' , ',a bH A N H NB∈ ∈ ' 'H A B∈ , rezultă că cercurile trec

printr-un punct comun M. Analog se poate demonstra că C , Cc şi Cb conţin punctele H şi M, de unde rezultă că cercurile sunt coaxiale, deci au centrele coliniare.

119 Arnold Droz -Farny (1856-1912) – matematician elveţian, contribuţii în geometrie 120 Farkaş Bolyai (1775-1856) – matematician român de origine maghiară, contribuţii fundamentale în geometrie

A

B C

A'

B'

B"

A"

H

bH

cH

aH

N

C

Cb

Ca

Fig. 289

Page 283: Barbu_TEOREME_FUNDAMENTALE(2008).pdf

290

Soluţia 2. Deoarece transversalele sunt ortogonale rezultă: ' ' ' ' ' '

(1)" " " " " "

A B B C C A

A B B C C A= =

(vezi „Ortocentrul unui triunghi”). Fie " "E A Bβ , " 'E A B∈ , ' " .D B C Eβ= I Avem

'' ",

2

A BEβ =

" "

2

B CDβ = ,

' ',

2

A BEα =

' '(2).

2

B CDγ = Din relaţiile (1) şi (2) rezultă că

E E

D D

β αβ α

= . Relaţia precedentă arată că triunghiurile dreptunghice Dβγ şi Eβα sunt

asemenea, deci punctele , ,α β γ sunt coliniare.

Observaţie: Dreapta αβγ se numeşte dreapta Droz-Farny.

Generalizarea teoremei lui Droz - Farny Printr-un punct oarecare P din planul unui triunghi ABC , se duc două drepte perpendiculare ', ''d d care intersectează laturile , ,BC CA AB , respectiv în punctele

( ', ', '), ( '', '', '')A B C A B C . Dacă , ,α β γ sunt proiecţiile punctelor ', ', 'A B C pe dreptele

, ,PA PB PC , iar 1 1 1, ,A B C sunt punctele de intersecţie dintre dreapta ''d cu dreptele

', ',A Bα β respectiv 'Cγ , atunci mijloacele segmentelor 1 1 1' , ' , 'A A B B C C sunt coliniare. Demonstraţie. Triunghiurile

1 ',PA A 1 1', 'PB B PC C sunt

dreptunghice. Fie , ,a b cω ω ω

mijloacele segmentelor

1 1 1' , ' , 'A A B B C C şi , ,a b cµ µ µ

mijloacele segmentelor ', ', 'PA PB PC . Dreapta 1 aA µ

intersectează perpendiculara în 'A pe 'd în punctul 2A

şi fie 2 2,B C punctele analog

construite. Patrulaterul 1 2'PA A A

este paralelogram deoarece laturile opuse sunt paralele şi congruente. Deci 2 1 'PA A A şi

cum 1 'A A PA⊥ rezultă

2PA PA⊥ . Fie 3 3 3, ,A B C

simetricele punctelor 2 2, ,A B

respectiv 2C faţă de dreapta 'd .

Atunci, patrulaterul 1 3 'PA A A

este dreptunghi, deoarece

1 3' ,PA A A 1 2 3' ' ,PA A A A A= = 1 'PA PA⊥ , deci cercul circumscris al său este punctul

aω - mijlocul segmentului 1' .A A Deoarece punctele 2 2,A B şi 2C sunt coliniare, rezultă şi

simetricele lor în raport cu dreapta 'd - punctele 3 3 3, ,A B C - sunt coliniare, deci şi punctele

, ,a b cω ω ω sunt coliniare.

Observaţie: Dacă punctul P este ortocentrul triunghiului ABC, atunci se obţine teorema lui Droz – Farny.

A

B C

A1

P

A'

B1

C1

Fig. 290

d '

d"

B' C'

2A

3A

α

β γ

Page 284: Barbu_TEOREME_FUNDAMENTALE(2008).pdf

291

II.21. Teorema lui Steiner - Lehmus121

„Steiner este cel mai mare geometru de la Apollonius încoace.” – W. Ball

Un triunghi care are două bisectoare interioare egale (măsurate de la vârf la latura opusă) este isoscel. Demonstraţie.

Soluţia 1. Fie BE şi CF bisectoarele unghiurilor B, respectiv C ale ABC (Fig. 291). Presupunem că AB AC≠ şi anume fie AB AC< , atunci

( ) ( )<m ACB m ABC de unde rezultă

că ( ) ( )

2 2<

m ACB m ABC. În

triunghiurile BEC şi BFC, rezultă CE BF> (1) construim paralelogramul BEGF. Astfel

EG BF≡ , ( )

( )2

=m ABC

m FGE ,

FG BE FC= = , de unde

( ) ( )=m FGC m FCG . Din

( ) ( )

( ) ( )2 2

= > =m ABC m ACB

m FGE m FCE rezultă că

( ) ( )<m EGC m ECG , de unde : CE EG BF< = contradicţie cu (1). Deci, presupunerea

făcută că AB AC< este falsă. Analog se tratează cazul în care AB AC> şi atunci rezultă AB AC= , adică triunghiul ABC este isoscel.

Soluţia 2. Utilizăm faptul că 2

cos2a

bc Al

b c=

+(unde al este lungimea bisectoarei interioare a

unghiului BAC ). Fie că b cl l= adică cos ( )2

( )cos2

B

b a c

C c a b

+=

+. Presupunem prin absurd că B C> ,

adică b c> şi atunci ( )

1( )

c a b

b a c

+>

+, de unde cos cos

2 2

B C> ,deci B C< absurd. Analog,

dacă B C< se ajunge la o contradicţie. Urmează că ( ) ( )m ABC m ACB= , adică triunghiul

ABC este isoscel. Observaţie: Dacă bisectoarea exterioară a unghiului B întâlneşte prelungirea laturii AC în punctul F atunci segmentul BF se numeşte bisectoare externă a lui B. Fie CG bisectoarea externă a lui C. Este uşor de demonstrat că dacă AB AC= atunci .FB CG=

120Jakob Steiner (1796 – 1863) – matematician german, profesor la Universitatea din Berlin, contribuţii în

geometria proiectivă Daniel Lehmus (1780 – 1863) – matematician german, profesor la Universitatea din Berlin

A G

C

F E

B

Fig. 291

Page 285: Barbu_TEOREME_FUNDAMENTALE(2008).pdf

292

Reciproca (dacă doua bisectoare externe ale unui triunghi sunt egale, atunci triunghiul este isoscel) nu este neaparat adevarată. Un exemplu elocvent în acest sens este triunghiul lui Emmerich.Triunghiul lui Emmerich are unghiurile de masuri egale cu 132° , 36° şi respectiv 12° şi are două bisectoare externe egale. Fie triunghiul ABC în care

( ) 132= °m ABC , ( ) 36 ,= °m CAB ( ) 12= °m BCA . Fie BF şi CG bisectoarele externe ale

unghiurilor B, respectiv C. Avem: 180 132( ) 24

2

° − °= = °

°m FBA

( ) 2 4 1 3 2 1 5 6= ° + ° = °m FBC , ( ) 12= °m BCF , ( ) 180 156 12 12= °− °− °= °m BFC adică

triunghiul FBC este isoscel, cu FB BC= (1) . În triunghiul BCG avem:

180 12( ) 84

2

°− °= = °m BCG , ( ) 48= °m GBC , ( ) 180 86 48 48= ° − ° − ° = °m BGC , de unde

rezultă că triunghiul BCG este isoscel cu CG BC= (2). Din relaţiile (1) şi (2) rezultă că FB CG= .

A

C

G

F

B

Fig. 292

Page 286: Barbu_TEOREME_FUNDAMENTALE(2008).pdf

293

II.22. Teorema lui Barbilian122

Fie ABC şi ' ' 'A B C două triunghiuri echilaterale de acelaşi centru (O), cu vârfurile notate în acelaşi sens de rotaţie. Să se arate că triunghiurile sunt de trei ori omologice în ordinele: ( , ' ' '), ( , ' ' '), ( , ' ' ').ABC C B A ABC B A C ABC A C B Demonstraţie.

Fie 1 2 3 1 ' ', ' ' , ' ' , ' ' ,A BC B C A A C BC A A B BC B B C AC= ∩ = ∩ = ∩ = ∩

2 ' ' ,B A C AC= ∩ 3 ' 'B A B AC= ∩ , 1 2 3 ' ', ' ' , ' ' .C AB B C C A C AB C A B AB= ∩ = ∩ = ∩

Pentru ca triunghiurile ( , ' ' ')ABC C B A să fie omologice vom arăta că dreptele ', 'AC BB şi

'CA sunt concurente. Din congruenţa triunghiurilor ', 'OAA OBB şi 'OCC rezultă

' ' '.AA BB CC≡ ≡ Deoarece 2 3 1' ' 'C AA A BB B CC≡ ≡ rezultă 2 3 1AC BA CB≡ ≡ şi de

aici 1 2 3 (1).B A C B A C≡ ≡ Deoarece 2 2 3 3 1 1AB C BA C CA B≡ ≡ rezultă 2 3 1,AB BC CA≡ ≡

de unde 2 3 1 (2)B C C A A B≡ ≡ (unde am utilizat faptul că

2 3( ') ( ') ( ') ( ') ( ')m AOA m BOB m COC m AB A m BC B= = = = = 2( ')m CB C ). Din

teorema lui Menelaus aplicată în triunghiul ABC şi transversalei ' 'B C rezultă:

1 1 1

1 1 1

1,A B BC C A

AC B A C B⋅ ⋅ = de unde se obţine egalitatea 1 1 1

1 1 1

.C A AC B A

C B A B BC= ⋅ Atunci,

31 2 1 1 2

1 3 2 1 1 2

1A BC A B C AC B A B C

C B A C B A A B B C B A⋅ ⋅ = ⋅ ⋅ = (unde am utilizat relaţiile (1) şi (2)), de unde rezultă

că punctele 1 3,C A şi 2B sunt coliniare, adică triunghiurile ABC şi ' ' 'C B A sunt omologice.

Analog, se arată omologia celorlalte perechi de triunghiuri.

122Dan Barbilian (1895-1961) – matematician român, profesor la Universitatea din Bucureşti, contribuţii în algebră

şi geometrie

A

B C

A'

A1

B1

C1 Fig. 293

B2

C2

A2

A3

B3

C3

B'

C'

Page 287: Barbu_TEOREME_FUNDAMENTALE(2008).pdf

294

II.23. Teorema lui Bottema

„În larg azur ca Sfinxul stau mândră şi ciudată. Mi-i inima de gheaţă şi trupul cum sunt crinii.

Urăsc tot ce e zbucium, tulburător de linii Şi nu plâng niciodată, şi nu râd niciodată.”

Charles Baudelaire123

Pe laturile ABC se construiesc în exterior pătratele ABDE şi ACFG. Fie M mijlocul segmentului DF. Să se arate că triunghiurile BMC şi EMG sunt dreptunghice şi isoscele. Demonstraţie.

Notăm cu litere mici afixele punctelor corespunzătoare. Din ( )2BD R Aπ

=

rezultă ( )d b i a b= + − (unde prin ( )2BR Aπ

am notat rotaţia de centru B şi unghi 2

π a

punctului A), iar ( )2CF R Aπ

−= , deci ( ),f c i a c= − − de unde

( ).

2 2

d f b c i c bm

+ + + −= =

Atunci 2

2BM m b c b= − = − şi

2,

2CM m c c b= − = − de unde rezultă că triunghiul

BMC este isoscel. Din * ,m b

i im c

−= − ∈ ⋅

− rezultă că ,BM MC⊥ adică triunghiul BMC

este dreptunghic isoscel. Analog, se demonstrează că şi triunghiul EMG este dreptunghic isoscel.

123Charles Baudelaire (1821 – 1867) – poet francez

A

B C

M

D

E

G

F

T

Fig. 294

Page 288: Barbu_TEOREME_FUNDAMENTALE(2008).pdf

295

II.24. Teorema lui Goormaghtigh „Fără a o baza pe intuiţie, geometria are totuşi un caracter intuitiv. Se pare, că idea de figură, figurile elementare: cercul, triunghiul, sfera etc., formate în practica milenară a omului, fac parte din zestrea informaţională a celulei nervoase, sunt transmise ereditar şi dau o mare mobilitate intuiţiei.” – Radu Miron124

1) Fie A B CT T T triunghiul tangenţial al triunghiului ABC şi punctele , ,X Y Z

aparţinând dreptelor AOT , BOT respectiv COT astfel încât A B C

OX OY OZt

OT OT OT= = = .

Dreptele , ,AX BY CZ sunt concurente în izogonalul conjugat al punctului P ce

aparţine dreptei lui Euler astfel încât 12

OP

PH t= .

Demonstraţie. Izogonala dreptei AX intersectează AOT în

punctul 'X . Triunghiurile OAX şi 'OX A sunt asemenea,

deci '

OX OA

OA OX= de unde rezultă 2'OX OX OA⋅ = , adică

punctele X şi 'X sunt inverse în cercul circumscris. De asemenea punctele aM (mijlocul laturii BC) şi AT sunt

inverse. Fie 'P AX OH= I . Avem,

' ' 1 1

2 2 2AOTOP OX OX

PH AH OM OX t= = = ⋅ = . Urmând acelaşi

raţionament se arată că dreptele ,B CBT CT conţin punctul P,

adică izogonalele dreptelor , ,AX BY CZ sunt concurente, de unde rezultă că şi dreptele , ,AX BY CZ sunt concurente.

Observaţie: Pentru 12t = , X, Y, Z sunt centrele cercurilor circumscrise triunghiurilor

OBC, OCA, respectiv OAB. Dreptele AX, BY, CZ sunt concurente în punctul izogonal conjugat al mijlocului segmentului OH (centrul cercului lui Euler al triunghiului ABC) – punctul de concurenţă se numeşte punctul lui Coşniţă .

Teorema lui Goormaghtigh

Fie O centrul cercului circumscris unui triunghi ABC, punctele 1 1 1, ,A B C pe dreptele OA,

OB, respectiv OC astfel încât 1 1 1 .OA OB OC

tOA OB OC

= = = i) Intersecţiile perpendicularelor

din 1A pe OA, 1B pe OB şi 1C pe OC cu laturile BC, CA respectiv AB se află pe o dreaptă d. ii) Dacă M este proiecţia lui O pe dreapta d, 'M un punct pe OM astfel încât 'OM t OM= ⋅ ,atunci punctul invers al punctului 'M în raport cu punctul O este izogonalul conjugat al punctului P de pe dreapta lui Euler ce are proprietatea

1.

2

OP

PH t=

Demonstraţie. i) Triunghiul XYZ determinat de intersecţiile perpendicularelor în 1 1 1, ,A B C , se obţine prin omotetia de centru O şi raport t a triunghiului tangenţial (Fig. 296). Vârfurile X, Y, Z aparţin dreptelor ', ',OA OB respectiv '.OC

124 Radu Miron (1927- ) – matematician român, profesor la Universitatea din Iaşi, membru al Academiei Române

A

C O

H

AT

X'

Fig. 295

B aM

P

X

Page 289: Barbu_TEOREME_FUNDAMENTALE(2008).pdf

296

Din teorema 1) dreptele AX, BY, CZ sunt concurente în punctul izogonal conjugat al

punctului P de pe dreapta OH pentru care 1

.2

OP

PH t= Fie

'X BC YZ= I , ' ,Y CA ZX= I ' .Z AB XY= I Vom arăta că punctele ', ', 'X Y Z

aparţin unei drepte d. Fie R raza cercului circumscris triunghiului ABC. Considerăm

inversiunea J de centru O şi raport 2t R⋅ , iar 'M = J(M). Inversiunea J transformă

punctele A, B, C în 1A , 1B respectiv 1C . Fie , ,a b cM M M mijloacele laturilor BC, CA,

respectiv AB. Deoarece patrulaterul 1 aBB M X este inscriptibil rezultă 2

1aOM OX OB OB t R⋅ = ⋅ = ⋅ . Analog, 2b cOM OY OM OZ t R⋅ = ⋅ = ⋅ ceea ce arată că

prin inversiunea J punctele X, Y, Z

se transformă respectiv în , ,a b cM M M . Imaginea punctului

'X prin inversiunea J este

punctul 2A , al doilea punct de

intersecţie dintre cercurile circumscrise triunghiurilor

b cOM M şi 1 1OBC . Analog,

imaginea punctelor 'Y şi 'Z prin

inversiunea J sunt respectiv

punctele 2B (al doilea punct de

intersecţie dintre cercurile circumscrise triunghiurilor

a cOM M şi 1 1OAC şi 2C (al doilea

punct de intersecţie dintre cercurile circumscrise triunghiurilor b aOM M şi 1 1OB A .

Deci, punctele ', ', 'X Y Z aparţin

unei drepte d iar punctele 2 2 2, , ,O A B C aparţin unui cerc C. ii) Prin inversiunea, J

dreapta AX se transformă în cercul 1 aOAM de diametru 'OX şi conţine punctul M,

proiecţia lui O pe dreapta d. Analog, imaginile dreptelor BY şi CZ sunt cercurile de diametru 'OY şi 'OZ ce-l conţin şi pe M. Deci punctul comun dreptelor AX, BY şi

CZ este imaginea lui M prin inversiunea J, adică intersecţia dintre OM şi cercul C.

Acesta este punctul diametral opus punctului O în cercul C .

Teorema lui Musselman Fie O centrul cercului circumscris triunghiului ABC, H ortocentrul triunghiului ABC şi A∗ , B∗ , C∗ simetricele vârfurilor , ,A B respectiv C faţă de laturile BC, CA şi AB.

Cercurile circumscrise triunghiurilor AOA∗ , BOB∗ şi COC∗ se întâlnesc într-un punct care este inversul punctului izogonal conjugat al centrului cercului lui Euler.

A

B C

X

Y

Z O 1A

1B 1C

aM

cM

2A

Fig. 296

X'

Page 290: Barbu_TEOREME_FUNDAMENTALE(2008).pdf

297

Demonstraţia rezultă din teorema lui Goormaghtigh pentru 1/ 2t = . Centrele cercurilor

circumscrise triunghiurilor BOB∗ şi COC∗ sunt coliniare, cercurile având un al doilea punct comun, simetricul lui O faţă de linia care uneşte centrele cercurilor. Acest punct este inversul punctului izogonal al centrului cercului lui Euler .

II.25. Teorema lui Dergiades

„Nu există ramură a matematicii – oricât de abstractă ar fi ea – care să nu se poată aplica într-o zi fenomenelor lumii reale.” – Nikolai Lobacevski125

Fie 1 1 1 2 2 2 3 3 3( , ), ( , ), ( , )C O R C O R C O R trei cercuri care trec prin vârfurile B şi C, C şi A, respectiv A şi B ale unui triunghi ABC şi D, E, F al doilea punct de intersecţie dintre cercurile 2( )C şi 3( )C , 3( )C şi 1( )C , respectiv 1( )C şi 2( )C . Perpendicularele duse în D, E, F pe AD, BE, respectiv CF intersectează laturile BC, CA, AB în punctele X, Y, respectiv Z. Punctele X, Y şi Z sunt coliniare. Demonstraţie.

Fie a b cM M M triunghiul median

al triunghiului ABC (Fig. 297). Dreptele 1 2 3, ,a b cO M O M O M

fiind mediatoarele laturilor triunghiului ABC sunt concurente în centrul cercului circumscris (O) al triunghiului ABC, deci triunghiurile a b cM M M şi 1 2 3OO O

sunt omologice. Fie ', ', 'A B C mijloacele segmentelor AX, BY respectiv CZ. Deoarece

2 3O O AD⊥ rezultă 2 3O O DX şi

cum 2 3O O este mediatoarea

segmentului AD rezultă că

2 3' .A O O∈ Deoarece b cM M BC

rezultă că ' ,b cA M M∈ deci

2 3 ' .b cA O O M M= ∩ Analog se

arată că 1 3 ' a cB OO M M= ∩ , 1 2 ' .a bC OO M M= ∩ Conform teoremei lui Desargues

punctele ', ', 'A B C sunt coliniare. Din teorema lui Newton – Gauss aplicată patrulaterului BCYZ rezultă că punctele X, Y şi Z sunt coliniare.

125Nikolai Lobacevski (1792-1856) – matematician rus, profesor la Universitatea din Kazan, contribuţii

fundamentale în geometrie

A

B

C aM

bM cM F

D

E

1O

2O 3O A'

X

Y

Z

Fig. 297

Page 291: Barbu_TEOREME_FUNDAMENTALE(2008).pdf

298

II.26. Teoremele lui Pappus126

„Cea mai neglijată teoremă de existenţă în matematică este existenţa oamenilor. Matematica a fost creată de oameni şi ea poartă amprenta lor.” – Hammer Presten

Fie triunghiul ABC şi punctele ( ), ( ), ( )M BC N AC P BA∈ ∈ ∈ care împart aceste segmente în acelaşi raport. Să se arate că triunghiurile ABC şi MNP au acelaşi centru de greutate.

Demonstraţie. Soluţia 1. Fie aM mijlocul laturii

BC şi 'M simetricul lui M faţă de aM (Fig.

298). Fie G centrul de greutate al triunghiului ABC , 1G centrul de greutate al

triunghiului MNP şi 2G centrul de greutate al

triunghiului ' .M NP Cum punctele M,N,P împart laturile triunghiului ABC în acelaşi raport rezultă că patrulaterul 'APM N este paralelogram de centru Q, de unde rezultă că 2 'aGG M M şi

23 2 ' 'aGG M M MM= = . În 'QMM avem:

1 2 'GG MM şi 1 2

1',

3GG MM= de unde rezultă

că punctele G şi 1G coincid. Soluţia 2 .Fie k raportul în care sunt împărţite laturile. Atunci,

, , .AP k PB BM k MC CN k NA= ⋅ = ⋅ = ⋅uuur uuur uuuur uuuur uuur uuur

Fie G centrul de greutate al triunghiului ABC .

Atunci, ,1

+ ⋅=

+

uuur uuuruuur GA k GBGP

k ,

1

+ ⋅=

+

uuur uuuruuuur GB k GCGM

k

1

+ ⋅=

+

uuur uuuruuur GC k GAGN

k. Sumând relaţiile

precedente rezultă: 0GM GN GP GA GB GC+ + = + + =uuuur uuur uuur uuur uuur uuur r

, deci G este şi centrul de greutate al triunghiului .MNP Reciproca teoremei lui Pappus Dacă un triunghi MNP înscris într-un triunghi ABC are acelaşi centru de greutate ca şi triunghiul ABC, atunci vârfurile triunghiului MNP împart laturile triunghiului ABC în acelaşi raport. Demonstraţie. Fie , ,a b cM M M mijloacele

laturilor triunghiului ABC, G centrul de greutate comun triunghiurilor ABC şi MNP, iar 'P piciorul medianei PG a triunghiului MNP (Fig. 299). Vom demonstra că

.AP BM CN

PB MC NA= = Evident, 2

' b

PG BG

P G M G= = ,

deci 'bM P BP . Cum b aM M BP rezultă că

punctele , ',b aM P M sunt coliniare. Fie

" 'P CP AB= I . Deoarece ' 'MP NP≡ şi

' ' "CP P P≡ rezultă că patrulaterul "MCEP este

126 Pappus din Alexandria (290-350) – matematician grec, contribuţii în geometrie

A

B C M

N

P

M' aM

Q

2G G

Fig. 298

A

B C M

N

P

M' aM

P"

P '

G

Fig. 299

Page 292: Barbu_TEOREME_FUNDAMENTALE(2008).pdf

299

paralelogram, deci "MP CE şi "EP BC , de unde "

"

BM BP CN

MC AP NA= = . Analog se arată

că BM AP

MC PB= , de unde rezultă concluzia.

Soluţia 2. Notăm afixele punctelor cu litere mici corespunzătoare, fie

, , .AP BM CN

x y zPB MC NA

= = = Atunci, , ,1 1 1

a bx b cy c azp m n

x y z

+ + += = =

+ + +. Trebuie să

demonstrăm că 3 3

a b c m n p+ + + += dacă şi numai dacă x y z= = . Dacă x y z= = ,

atunci evident că a b c m n p+ + = + + . Reciproc, fie a b c m n p+ + = + + . Atunci,

1 1 1 1 1 10

1 1 1 1 1 1a b c

x z y x z y

− + − + − = + + + + + +

. Cum

1 1 1 1 1 10

1 1 1 1 1 1x z y x z y

− + − + − = + + + + + +

şi punctele A,B,C nu sunt coliniare,

rezultă că : 1 1 1 1 1 1

,1 1 1 1 1 1x z y x z y

− = − = −+ + + + + +

de unde : x y z= = .

2) În triunghiul ABC, fie ( ), ( ),D AC E AB∈ ∈ astfel încât ,M BD CE= ∩

,N DE BC= ∩ .= ∩P AM BC Atunci .NB PB

NC PC=

Demonstraţie. Din teorema lui Menelaus aplicată în triunghiul ABC şi transversalei

N – E – D rezultă: 1EA DC NB

EB DA NC⋅ ⋅ = (1) (Fig. 300). Teorema lui Ceva aplicată în

triunghiul ABC ne dă: (2)EA DC PB

EB DA PC⋅ ⋅ = . Din relaţiile (1) şi (2) rezultă .

NB PB

NC PC=

Observaţie: Relaţia NB PB

NC PC= ne arată că punctele N, B, P, C sunt conjugate armonic.

A

B C

P

Fig. 300

N

D

E M

Page 293: Barbu_TEOREME_FUNDAMENTALE(2008).pdf

300

II.27. Teorema lui Salmon127 „Matematica este formată din insule de cunoaştere într-un ocean de ignoranţă.”

Fie M un punct pe cercul circumscris unui triunghi .ABC Cercurile de diametre ( ), ( ), ( )AM BM CM se intersectează două câte

două în trei puncte coliniare. Demonstraţie. Fie ',',' CBA proiecţiile punctului M pe laturile BC , CA respectiv .AB Punctele ',',' CBA sunt punctele de intersecţie

dintre cercurile de diametre ( ), ( )AM BM şi

( )CM , iar conform teoremei lui Simson rezultă

că punctele ',',' CBA sunt coliniare.

II.28. Teorema lui Pedoe „Nici un om nu se întăreşte citind un tratat de gimnastică, ci făcând exerciţii; nici un om nu se învaţă a judeca citind judecăţile scrise de alţii, ci judecând singur şi dându – şi singur seama de natura lucrurilor.” – Mihai Eminescu128

Fie triunghiul ABC şi ' ' 'A B C situate în acelaşi plan. Dacă laturile lor au lungimile a, b, c respectiv ', ', 'a b c , atunci: 2 2 2 2 2 2 2 2 2 2 2 2( ' ' ' ) ( ' ' ' ) ( ' ' ' ) 16 '− + + + − + + + − ≥ ⋅a a b c b a b c c a b c S S ,

cu egalitate dacă triunghiurile sunt asemenea ( unde cu S şi 'S am notat ariile triunghiurilor ABC , respectiv ' ' 'A B C ).

Demonstraţie.

127 George Salmon (1819-1904) – matematician irlandez, contribuţii în algebră şi geometrie 128 Mihai Eminescu (1850-1889) – poet, jurnalist român, considerat cel mai important scriitor romantic din literatura română

A

B C

A"

a

c b

Fig. 302

A'

B' C' a '

b ' c '

Fig. 303

C A' B

M C'

A

B'

Fig. 301

Page 294: Barbu_TEOREME_FUNDAMENTALE(2008).pdf

301

Pe latura BC se construieşte triunghiul "A BC asemenea cu ABC, de unde:

" "

' ' ' ' ' '= =

BC CA A B

B C C A A B, adică

" "

' ' '= =

a CA A B

a b c, deci:

'"

'=ab

A Ca

(1) (Fig.302). Din

teorema cosinusului în triunghiul "A CA rezultă: 2 2 2" " 2 " cos "AA b A C b A C ACA= + − ⋅ ⋅ (2). Din relaţiile (1) şi (2) rezultă

2

2 2 ' '" 2 cos( ' ' ' )

' ' = + − ⋅ −

ab abAA b b A C B ACB

a a egalitate echivalentă cu

2 2 2 2 2 2' " ' ' 2 ' ' cos cos ' s in sin ' 0 ⋅ = + − ⋅ + ⋅ ≥ a AA a b a b aa bb C C C C (3).

Dar sin ' 'sin '

'2 2

⋅ = ⋅ab C a b C

S S 4 ' ' 'sin sin '= ⋅SS aa bb C C (4), iar 2 2 2

cos2

+ −=a b c

Cab

şi

2 2 2' ' '

cos '2 ' '

+ −=a b c

Ca b

(5). Din relaţiile (3), (4) şi (5) rezultă concluzia. Egalitatea are loc

atunci când 2" 0,AA = adică ",A A≡ deci când triunghiurile ABC şi ' ' 'A B C sunt asemenea. Consecinţe:

1) Dacă triunghiul ' ' 'A B C este echilateral, atunci inegalitatea devine: 2 2 2 4 3a b c S+ + ≥ ⋅ .

2) Dacă triunghiul ' ' 'A B C este CBA, deci ' , ' , ' ,= = =a b b c c a atunci 'S S= şi inegalitatea

devine 4 4 4 216 .a b c S+ + ≥

3) Dacă triunghiul ' ' 'A B C este dreptunghic în 'A , atunci: 2 2 2 2' ' 8 '.b c c b SS+ ≥

II.29. Teorema lui Simson generalizată

„Din ceas, dedus adâncul acestei calme creste, Intrată prin oglindă în mântuit azur,

Tăind pe înecarea cirezilor agreste, În grupurile apei, un joc secund mai pur.”

Ion Barbu129

Teorema lui Simson generalizată

Fie M un punct pe cercul circumscris triunghiului ABC , ' , ' , 'A BC B CA C AB∈ ∈ ∈ .

Dacă ( ' ) ( ' ) ( ' ) , (0, )m MC A m MB C m MA C ϕ ϕ π≡ ≡ = ∈ , atunci punctele ', ', 'A B C sunt coliniare. Demonstraţie. Patrulaterele ,ABMC ' ',AB MC ' 'A B MC sunt inscriptibile (Fig. 304). Atunci, ' ' 'C B M C AM MCB≡ ≡ de unde rezultă:

( ' ' ) ( ' ')m A B M m MB C+ = ( ' ' ) ( ') 180m A B M m MCA+ = ° adică punctele

', ', 'A B C sunt coliniare.

129 Ion Barbu (1895-1961) – matematician român, profesor la Universitatea din Bucureşti, contribuţii în algebră şi geometrie

Page 295: Barbu_TEOREME_FUNDAMENTALE(2008).pdf

302

Observaţii:

1) Teorema de mai sus aparţine lui Lazare Carnot. 2) Dreapta ce conţine punctele ', ', 'A B C se numeşte dreapta lui Simson generalizată de unghi ϕ a punctului M faţă de triunghiul ABC (notaţie cu Md ).

3) Pentru a determina de exemplu poziţia punctului M pe cercul circumscris triunghiului ABC procedăm astfel: alegem punctele N şi P arbitrar pe cerc, construim dreapta

1AA astfel încât măsura unghiului dintre

1AA şi NP să fie egală cu π ϕ−

( 1A aparţine cercului circumscris

triunghiului ABC), iar din 1A construim

dreapta 1AM astfel încât

1( , )m AM BC ϕ= (M aparţinând

cercului circumscris triunghiului ABC ). Fie , ,M N P puncte pe cercul circumscris al unui triunghi ABC astfel încât unghiul dintre dreapta lui Simson generalizată de unghi ϕ a punctului M

şi dreapta NP să aibă măsura π ϕ− .

Triunghiul MNP se numeşte triunghi − Sϕϕϕϕ faţă de triunghiul ABC .

Dreapta lui Simson generalizată de unghi de măsură ϕ a punctului M faţă de

triunghiul ABC este paralelă cu dreapta 1AA .

Demonstraţie. Fie ', ', 'A B C proiecţiile de unghi ϕ ale punctului M pe dreptele ,BC CA

respectiv AB şi 1A punctul de intersecţie dintre 'MA cu cercul circumscris triunghiului

ABC . Deoarece patrulaterul ' 'AB MC este inscriptibil rezultă: ( ' ') ( ') ( ' )m AC B m AMB m B AMϕ= = − = 1 1( ) ( ) ( )m MBC m BMA m BAAϕ − = = ,

deci 1 MAA d .

A

B

C

M

B'

C'

A'

Fig. 304

Md

P

N

1A

Page 296: Barbu_TEOREME_FUNDAMENTALE(2008).pdf

303

II.30. Teorema lui Sondat „Pe linie de cercetare, geometria cuprinde domenii abstracte foarte generale, dar geometria elementară rămâne foarte importantă în învăţământ fie prin aplicaţiile ei derecte diverse, fie ca o verigă în înţelegerea problemelor moderne de teoria spaţiilor generalizate.” - Nicolae Mihăileanu130

Fie triunghiurile 1 1 1,ABC A BC ortologice şi omologice, 1,Q Q centrele de ortologie şi

P centrul de omologie, iar d axa de omologie a acestor două triunghiuri. Punctele ,P Q

şi 1Q aparţin unei drepte perpendiculare pe dreapta d .

Demonstraţie. Punctul Q aparţine perpendicularelor duse din 1 1 1, ,A B C respectiv pe laturile

, ,BC CA AB , iar punctul 1Q aparţine perpendicularelor duse din , ,A B C pe laturile

1 1 1 1 1 1, ,BC AC A B .Fie 1 1 1 1 ' , ' , ', ' .B CA C A C AB AB B C d= ∩ = ∩ ∈ Avem 1 1 1 1, ,AP AA BP BBα β= =uuuur uuuur uuuur uuuur

2 2 2 2 2 2

1 1 , , ,a b cC A C C PA QA l PB QB l PC QC lγ= − = − = − =uuuur uuuur

.Arătăm că

2 2 2 2' ' ' 'B P B Q C P C Q− = − (1), ceea ce implică PQ d⊥ . Avem 1 1

1 1

'

'

A P C CB C

B A A A C P

αγ

= ⋅ = .

Din teorema lui Leibniz rezultă : 2 2 2 2 2' ( ) ' 'PC PA PB B C B Aγ α γ α γ α⋅ − ⋅ = ⋅ − + ⋅ − ⋅ şi

2 2 2 2 2' ( ) ' 'QC QA QB B C B Aγ α γ α γ α⋅ − ⋅ = ⋅ − + ⋅ − ⋅ , iar de aici 2 2' ' c al lPB QB

γ αγ α−

− =−

.

Relaţia (1) este echivalentă cu ( ) ( ) ( ) 0b c c a a bl l l l l lβγ γα αβ− + − + − = (2). Deoarece

1BC AQ⊥ rezultă 2 2 2 2 b cl lBA CA CQ BQ

α−

− + − = (3). Din relaţia lui Leibniz rezultă

2 2 22 2 2 1 1( 1) ,BA BP BA A A A Pα α α− = − + − 2 2 2 2 2

1 1 1( 1)CA CP CA A A A Pα α α− = − + − (4).

130N. Mihăileanu (1912-1998) – matematician român

A

B

C

1A

1B

1C C' B'

d

Fig. 305

A'

Q

P

Page 297: Barbu_TEOREME_FUNDAMENTALE(2008).pdf

304

Din relaţiile de mai sus rezultă 2 2 2 2 b cl l

BA CA CQ BQα−

− + − = . Analog rezultă că

2 2 2 2 2 2 2 2,c a a bl l l l

CB AB AQ CQ AC BC BQ AQβ γ− −

− + − = − + − = . Sumând relaţiile

precedente rezultă 0b c c a a bl l l l l l

α β γ− − −

+ + = , adică tocmai relaţia PQ d⊥ . Analog se

arată că 1PQ d⊥ , deci punctele 1, ,P Q Q sunt coliniare.

II.31. Teoremele lui Maxwell131

„Vis al Dreptei Simple! Poate, geometria Săbiilor trase la Alexandria, Libere, sub ochiul de senin oţel, În neclătinatul idol El Gahel.”

- Ion Barbu132

1) Fie P un punct în planul triunghiului ABC şi ' ' 'A B C un triunghi care are laturile paralele cu cevienele punctului P în raport cu triunghiul ABC . Cevienele triunghiului

' ' 'A B C paralele cu laturile triunghiului ABC sunt concurente. Demonstraţie. Sunt o infinitate de triunghiuri ' ' 'A B C asemenea, deci este suficient să

demonstrăm problema pentru unul din aceste triunghiuri. Fie 1A , 1B , 1C picioarele

cevienelor corespunzătoare punctului P. Fie 2'A A AB , 2'B B BC , 2' ACC C ,

( 2 ' 'A B C∈ , 2 ' 'B A C∈ , 2 ' 'C A B∈ ). Din asemănarea triunghiurilor 1PC B şi 2' 'B A A ,

respectiv 1PAC cu 2' 'C A A rezultă 1 1

2 2' '

C B PC

A A B A= şi 1 1

2 2' '

AC PC

A A A C= , de unde :

1 2

1 2

'

'

C B A C

C A A B= . Analog, se obţin relaţiile 1 2

1 2

'

'

B A C B

B C C A= şi 1 2

1 2

'

'

AC B A

A B B C= . Deoarece dreptele

131 James Clerk Maxwell (1831-1879) – matematician şi fizician scoţian, profesor la Cambridge 132 Ion Barbu (1895-1961) – matematician român, profesor la Universitatea din Bucureşti, contribuţii în algebră şi geometrie

A

C'

B'

B A1

B1 C1

A'

C

P

C2

A2

B2

Q

Fig. 306

Page 298: Barbu_TEOREME_FUNDAMENTALE(2008).pdf

305

1AA , 1BB şi 1CC sunt concurente rezultă 1 1 1

1 1 1

1AC B A C B

A B B C C A⋅ ⋅ = adică 2 2 2

2 2 2

' ' '1

' ' '

B A C B A C

B C C A A B⋅ ⋅ = ,

deci dreptele 2AA , 2BB şi 2CC sunt concurente.

2) Fie P un punct în planul triunghiului ABC şi ' ' 'A B C un triunghi ce are laturile perpendiculare pe cevienele punctului P în raport cu triunghiul ABC . Cevienele triunghiului ' ' 'A B C perpendiculare pe laturile triunghiului ABC sunt concurente. Demonstraţia este evidentă deoarece triunghiurile ABC şi ' ' 'A B C sunt ortologice (vezi „Triunghiuri ortologice”).

II.32. Teorema trisecţiei În triunghiul ABC fie medianele BB’ şi CC’. Printr-un punct ( )T BC∈ se duc paralelele TD şi TE la medianele BB’, respectiv CC’,( D (AB), E (AC)∈ ∈ ). Atunci, medianele BB’ şi CC’ împart segmentul DE în trei segmente congruente. Demonstraţie.

Fie M=DE∩BB’ , N=DE∩CC’ , ' 'G BB CC= ∩ , K=DT∩BB’, L=TE∩CC’.

Din 'DT CC rezultă că triunghiurile BKT şi BGC sunt asemenea şi cum 1

' '3

C G CC=

rezultă 1

3DK DT= . Din asemănarea triunghiurilor DKM şi DTE rezultă

1

3DM DE= .

Analog, 1

,3

NE DE= de unde rezultă1

.3

DM MN NE DE= = =

L

N

A

B C

D

B' C'

M

E

K

T

Fig. 307

Page 299: Barbu_TEOREME_FUNDAMENTALE(2008).pdf

306

II.33. Teoremele lui Harcourt

„Dacă numai aş avea teoremele! Atunci aş putea destul de uşor să găsesc demonstraţiile.” - Bernhard Riemann133 1) Dacă distanţele de la vârfurile A, B, C ale unui triunghi ABC la o tangentă dusă la cercul înscris în triunghiul ABC au lungimile 1 1,a b respectiv 1,c atunci

1 1 1 [ ]2 .+ + =ABC

aa bb cc A (a, b, c sunt lungimile laturilor BC, AC respectiv AB).

Demonstraţie. Fie ( , , )x y z coordonatele baricentrice ale unui punct P şi l o dreaptă ce trece

prin P, iar 1d , 2d şi 3d sunt distanţele de la vârfurile , ,A B C la dreapta l (Fig.309). Vom

arăta că 1 2 3 0.⋅ + ⋅ + ⋅ =d x d y d z Fie ' = ∩A AP BC şi considerăm 1d negativ iar 2d şi

3d pozitive. Atunci, '=

+AP x

PA y z şi

'

'=

BA z

A C y,deci distanţa de la 'A la l este egala cu

' 2 31

+=

+

yd zdd

y z, de unde 1

'1 '

− += =

d AP y z

PA xd, relaţie echivalentă cu

1 2 3 0d x d y d z⋅ + ⋅ + ⋅ = . Revenind, presupunem că dreapta l trece prin ( , , )I a b c şi este

paralelă cu tangenta la cercul înscris. Atunci,distanţele de la A la l sunt: 1 1d a r= − ,

2 1= −d b r , 3 1= −d c r . Avem: 1 1 1+ + =aa bb cc

1 2 3 1 2 3 [ ] [ ]( ) ( ) ( ) ( ) ( ) 0 2 2+ + + + + = + + + + + = + =ABC ABC

a d r b d r c d r ad bd cd a b c r A A .

2) Dacă distanţele de la vârfurile A, B, C la o tangentă dusă la cercul exînscris corespunzător laturii BC a triunghiului ABC , au lungimile 1 1,a b respectiv 1,c atunci

1 1 1 2 ABCaa bb cc A− + + = (relaţii analoge se obţin prin celelalte două cercuri exînscrise).

Demonstraţie. Prin centrul ( , , )aI a b c− ducem o paralelă la tangenta considerată şi fie

1d , 2d , 3d distanţele de la , ,A B C la această paralelă. Atunci, 1 2 3 0ad bd cd− + + = iar

1 3 ac d r= + , 1 2 ab d r= + , 1 3 ac d r= + , unde ar este raza cercului A -exînscris. Avem:

1 1 1− + + =aa bb cc 1 2 3 [ ]( ) ( ) 0 2 ( ) 2 .− + + + − + + = + ⋅ − =a a ABC

ad bd cd r a b c r p a A

133 Bernhard Riemann (1826-1866) – matematician german, profesor la Universitatea Göttingen, contribuţii

fundamentale în analiza matematică şi geometria diferenţială

A

B C

C'

B' A'

P

I

Fig. 308

A

B C

P

A'

1d

2d 3d

'1d

Fig. 309

Page 300: Barbu_TEOREME_FUNDAMENTALE(2008).pdf

307

I.34. Teorema lui Zaslavsky „Geometria proiectivă ne - a deschis cu cea mai mare usurinţă teritorii noi în ştiinţa noastră, a fost şi numită pe bună dreptate un drum regal, conducând în domeniul său particular de cunostiinţe.” - Poncelet134

Fie P un punct în planul triunghiului ABC şi ' ' 'A B C simetricul triunghiului ABC faţǎ de punctul P. Prin punctele ', ', 'A B C ducem trei drepte paralele care intersecteazǎ dreptele BC, AC şi AB în punctele X ,Y respectiv Z. Sǎ se arate cǎ punctele X,Y şi Z sunt coliniare.

Demonstraţie. Vom arata cǎ 1⋅ ⋅ =BX CY AZ

XC AY BZ. Prin punctul C ducem o paralelǎ (d) la

dreptele paralele date. Fie 'Z simetricul lui Z fatǎ de P. Atunci, ' 'BZB Z este paralelogram, deci ' '≡BZ B Z şi ' ' ' ' BZ B Z B A de unde rezultǎ cǎ punctele ', ', 'B A Z sunt coliniare

si ' '≡AZ A Z . Astfel, AZ

BZ=

' '

' '

A Z

B Z (1). Fie '= ∩N BA d şi '= ∩K AB d . Atunci, din

teorema lui Thales rezultǎ: '

'=

BX BA

XC A N şi

'

'=

CY B K

AY B A. Avem, ⋅ ⋅

BX CY AZ

XC AY BZ=

' '

' '⋅ ⋅ =

BA B K AZ

BZA N B A

' ' '

' ' '⋅ = ⋅ =

B K AZ B Z AZ

BZ BZA N AZ 1⋅ =BZ AZ

AZ BZ, unde am utilizat faptul cǎ

triunghiurile ' 'A NZ şi ' 'B KZ sunt coliniare. Din reciproca teoremei lui Menelaus rezultǎ cǎ punctele X,Y şi Z sunt coliniare.

134Jean Poncelet (1788 - 1867) – matematician francez, contribuţii importante în geometria proiectivă

B

A

C

Z

X

K Z'

C'

B' P

A'

N

1d

2d

3d

Y

Fig. 310

d

Page 301: Barbu_TEOREME_FUNDAMENTALE(2008).pdf

308

II.35. Teorema lui Zajic „Ne vom aminti de Arhimede când îl vom fi uitat pe Eschil fiindcă limbile mor, iar ideile matematice sunt fară moarte. Nemurirea poate părea un cuvânt inept, dar matematicianul are, probabil, cea dintâi şansă de a se bucura de binefacerile ei, oricare ar fi acelea.” - G.H. Hardy135

În triunghiul ABC, fie ( )∈X BC şi aC punctul de tangenţă al cercului înscris în triunghiul ABC cu latura BC. Dacă cercurile înscrise în triunghiurile ABX şi ACX sunt tangente laturii AX în punctele 1T , respectiv 2T , atunci 1 2=aC X TT . Demonstraţie. Fie a, b, c lungimile laturilor triunghiurilor ABC şi , ,b cp p p semiperimetrele

triunghiurilor ABC, ABX respectiv ACX. În triunghiul ABC avem:

= + = − +a a aBX BC C X p b C X şi

= − =a aCX CC C X − − ap c C X . În triunghiul

ABX, 1 = −bXT p c şi în triunghiul ACX,

2 .= −cXT p b Atunci, 1 2 1 2TT XT XT= − =

b cp p b c− + − . Dar, 2( )b cp p− =

2[( ) ( )]c A X BX b AX C X+ + − + + =

2( )− + ac b C X , de unde rezultă că

− = − +b c ap p c b C X , deci

1 2 .= − + + − =a aTT c b C X b c C X

Consecinţă: Dacă aC este punctul de tangenţă al cercului înscris în triunghiul ABC cu

latura BC, atunci cercurile înscrise în triunghiurile aABC şi AC aC sunt tangente laturii

aAC în acelaşi punct.

Demonstraţia rezultă din teorema lui Zajic pentru = aX C .

II.36. Teorema lui Viviani136 Suma distanţelor de la un punct, situat în interiorul unui triunghi echilateral la laturile triunghiului este egală cu înalţimea triunghiului. Demonstraţie. Fie P un punct în interiorul ABC şi

1 2 3, ,P P P proiecţiile lui P pe laturile BC, AC respectiv AB

(Fig. 312). Fie a lungimea laturii AB şi h lungimea triunghiului echilateral ABC. Avem:

[ ] [ ] [ ] [ ]= + +ABC PBC PAC PABA A A A , adică

31 2

2 2 2 2

⋅⋅ ⋅ ⋅= + +

PP aa h PP a PP a, de unde rezultă

1 2 3= + +h PP PP PP .

135 G.H. Hardy (1877-1947) – matematician englez, profesor la Universitatea Cambridge, contribuţii importante

în teoria numerelor şi analiza matematică 136 Vincenzo Viviani (1622-1703) – inginer italian, contribuţii în fizică şi geometrie

A

B C X

1I 2I 1T

2T

aC

Fig. 311

A

B C

P3

P2

P1

P

Fig. 312

Page 302: Barbu_TEOREME_FUNDAMENTALE(2008).pdf

309

II.37. Teorema lui Véronèse137

„Iubirea nu se dăruie decât pe sine şi nu ia decât de la sine. Iubirea nu stăpâneşte şi nu vrea să fie stăpânită, fiindcă iubirii îi este de ajuns iubirea.” - Gibran Kahlil138

Fie triunghiurile omologice ABC şi ' ' ',A B C " ' ',= ∩A BC CB " ' ',= ∩B CA AC

" ' '.= ∩C AB BA Triunghiul " " "A B C este omologic cu fiecare din triunghiurile

, ' ' 'ABC A B C iar cele trei centre de omologie sunt coliniare. Demonstraţie. Fie 1 1 1 ' ', ' ', ' '= ∩ = ∩ = ∩A BC B C B AC A C C AB A B (Fig.313).

Triunghiurile ABC şi ' ' 'A B C fiind omologice rezultă că punctele 1 1 1, ,A B C sunt coliniare.

Omologia triunghiurilor ABC şi ' ' 'A B C implică omologia triunghiurilor 'ABC şi ' 'A B C de unde rezultă că punctele 1, ", "C A B sunt coliniare. Analog, punctele

1( , ", ")A B C şi 1( , ", ")B C A sunt coliniare, deci triunghiul " " "A B C este omologic cu

fiecare din triunghiurile ABC şi ' ' ',A B C având drept axă de omologie dreapta 1 1,AC iar

cum dreapta "AA nu trece prin centrul O de omologie dintre triunghiurile ABC şi ' ' 'A B C rezultă că centrele de omologie 1O şi 2O - dintre triunghiurile " " "A B C şi ABC respectiv

" " "A B C şi ' ' 'A B C - şi punctul O sunt coliniare.

137 Giuseppe Véronèse (1854-1917) – matematician italian, profesor la Universitatea din Roma, contribuţii

importante în geometria proiectivă 138 Gibran Kahlil (1883-1931) – poet, filosof, sculptor libanez

A

B C

A' B'

C'

1A

1B

1C

A"

B"

Fig. 313

C"

Page 303: Barbu_TEOREME_FUNDAMENTALE(2008).pdf

310

II.38. Teorema lui Coşniţă139

„Matematica este o ştiinţă în care nu se ştie niciodată despre ce se vorbeşte şi nici dacă este adevărat ce se vorbeşte.” – Bertrand Russel140

Teorema lui Coşniţă Fie O centrul cercului circumscris unui triunghi ABC şi X, Y, Z centrele cercurilor circumsrise triunghiurilor BOC, COA, respectiv AOB. Dreptele AX, BY şi CZ sunt concurente. Demonstraţia1. Fie 1 ,A AX BC= ∩

1 ,B BY CA= ∩ 1 = ∩C CZ AB (Fig. 314). Deoarece

( ) ( )= = m BOX m OCX1

( ) ( ),2

= m BOC m A

rezultă ( ) 90 ( )= ° − = m OBC m BOX 90 ( )° − m A şi

( ) ( ) ( ) ( )= = − = m CBX m BCX m A m OBC

2 ( ) 90 .− ° =m A α Analog, ( ) 2 ( ) 90= − °= m ACY m B β

şi ( ) 2( ) 90 .= − ° = m AZB C γ Avem: [ ]1

1

= =ABX

ACX

ABA

AC A

1s in ( ) s in ( )2

1 s in ( )s in ( )2

⋅ ⋅ ⋅ + +=

+⋅ ⋅ ⋅ +

A B B X BA B B

A C CA C C X C

α ααα

sau 1

1

cos( ).

cos( )

BA AB C A

AC AC B A

⋅ −=

⋅ − Analog, 1

1

cos( )

cos( )

CB CB A B

B A BA C B

⋅ −=

⋅ − şi 1

1

cos( ).

cos( )

C A CA B C

C B CB A C

⋅ −=

⋅ −

Atunci, 1 1 1

1 1 1

1BA CB C A

AC B A C B⋅ ⋅ = (unde am ţinut cont că cos( ) cos ,x x x− = ∀ ∈ ) şi din

reciproca teoremei lui Ceva rezultă că dreptele AX, BY şi CZ sunt concurente. Demonstraţia 2: Fie * * , A ZY BC B XZ AC= ∩ = ∩ şi * .C XY AB= ∩ Deoarece ZY,

XZ, XY sunt mediatoarele segmentelor AO, BO respectiv CO din teorema lui Ayme rezultă că punctele * * *, ,A B C sunt coliniare şi din reciproca teoremei lui Desergues aplicată triunghiurilor ABC şi XYZ rezultă că dreptele AX, BY şi CZ sunt concurente.

Observaţii: i) Punctul de concurenţă al dreptelor AX, BY şi CZ se numeşte punctul lui Coşniţă al triunghiului ABC. ii) Triunghiul XYZ se numeşte triunghiul lui Coşniţă al triunghiului ABC. iii) AX, BY, CZ se numesc dreptele lui Coşniţă. 1) Într-un triunghi ABC, punctul lui Coşniţă ( *O ) şi centrul cercului lui Euler ( 9O ) sunt

izogonal conjugate. Demonstraţie. Fie 'A şi "A punctele de intersecţie dintre bisectoarele interioară şi exterioară a unghiului A cu cercul circumscris triunghiului ABC şi aO centrul cercului

139 Cesar Coaşniţă (1910-1962) – matematician roman, profesor la Universitatea din Bucureşti 140 Bertrand Russell (1872 - 1970) – filosof, logician şi matematician englez, laureat al Premiului Nobel pentru literatură

A

B

C

X

Y Z

O 1C 1B

O∗

1A

Fig. 314

Page 304: Barbu_TEOREME_FUNDAMENTALE(2008).pdf

311

circumscris triunghiului BHC. Punctele 9, , aA O O sunt coliniare (vezi „Cercurile lui

Carnot”) (Fig. 315). Avem: , ' , 2 cos2cos a

ROX XA XO R OO AH R A

A= = − = = (vezi

„Cercurile lui Carnot”). Atunci, " '

2 cos ," '

a aA O A OR A

A X A X= =

deci fasciculul 1( , )aA NO MO este armonic şi cum

( ' ") 90= °m A AA rezultă că dreptele 'AA şi "AA sunt

bisectoarele unghiurilor ,aXAO respectiv ,aO AP unde

( \ [ ])P BA AB∈ (vezi „Centrul cercului înscris în triunghi”),

deci ' ' aXAA A AO≡ sau 9' 'XAA A AO≡ (1), adică

dreptele AX şi 9AO sunt izogonale. Analog se arată că

dreptele BY şi 9BO sunt izogonale, deci punctele lui Coşniţă

( *O ) şi centrul cercului lui Euler ( 9O ) sunt izogonal

conjugate.

2) Raza cercului circumscris triunghiului BOC are raza egală cu .2cos

R

A

Demonstraţie. Avem: 2

2[ ]

.4 2sin 2 2cossin 2

42

BOC

R R a R a a RXO

A A AR A

⋅ ⋅ ⋅= = = =

⋅ ⋅⋅

3) Fie , ,∗ ∗ ∗A B C simetricele vârfurilor triunghiului ABC faţă de laturile opuse şi O

centrul cercului circumscris triunghiului ABC. Cercurile circumscrise triunghiurilor ,∗ ∗

AOA BOB şi ∗COC se întâlnesc într-un al doilea punct care este inversul în cercul circumscris al punctului lui Coşniţă. Demonstraţie. Vezi „Teorema lui Goormaghtigh - Musselman”. Teorema lui Yiu

4) Fie , ,∗ ∗ ∗A B C simetricele vârfurilor triunghiului ABC faţă de laturile opuse.

Cercurile circumscrise triunghiurilor , ,∗ ∗ ∗ ∗ ∗ ∗AB C BC A CA B trec prin inversul punctului

lui Coşniţă în cercul circumscris triunghiului ABC .

Demonstraţie. Fie Q inversul punctului lui Coşniţă *( )O în cercul circumscris

triunghiului .ABC Din teorema lui Musselman Q aparţine cercurilor circumscrise

triunghiurilor ∗BOB şi ∗COC (Fig. 316). Atunci, ∗ ∗≡ B QO B BO şi

,∗ ∗≡ C QO C CO de unde rezultă că ( ) ( ) ( )m B QC m BQO m C QO∗ ∗ ∗= + =

( ) ( ) [ ( ) ( )] [ ( ) ( )]m B BO m C CO m CBB m CBO m BCC m BCO∗ ∗ ∗ ∗+ = − + − =

[ ( ) ( ) ] [ ( ) ( ) ]∗ ∗+ − + = m C B B m B C C m C B O m B C O

( ) ( ) (180 ( ))∗ ∗+ − ° − m CBB m BCC m BOC . Dar ( ) 90 ( ) 90 ( )∗ = °− = °− m CBB m C m B

( ) 2 ( ).= m BOC m BAC Astfel, ( ) (90 ( )) (90 ( )) 180 2 ( )∗ ∗ = °− + °− − °− = m B QC m C m B m A

2 ( ) ( ) ( ) 3 ( ) 180 .− − = − ° m A m B m C m A Deci, 180 ( ) 2 180 3 ( ).∗ ∗°− = ⋅ °− m B QC m A

Pe de altă parte, ∗ ≡ BAC BAC şi ,∗ ≡ CAB BAC de unde rezultă că

A

B A '

C

X

aO

O H

1A

P A ''

Fig. 315

Page 305: Barbu_TEOREME_FUNDAMENTALE(2008).pdf

312

( ) 2 180 [ ( ) ( ) ( )] 2 180 3 ( ).∗ ∗ ∗ ∗= ⋅ ° − + + = ⋅ ° − m B AC m BAC m BAC m CAB m BAC Ur

mează că, ( ) 180 ( ),∗ ∗ ∗ ∗= ° − m B AC m B QC de unde rezultă că Q aparţine cercului

circumscris triunghiului .∗ ∗AB C Analog, se arată că punctul Q aparţine şi cercurilor

circumscrise triunghiurilor ∗ ∗BC A şi .∗ ∗CA B

Observaţie: În general, dat fiind un triunghi ABC şi punctele , ,∗ ∗ ∗A B C astfel încât

cercurile circumscrise triunghiurilor ,∗ ∗A BC B CA şi ∗C AB au un punct comun, atunci

cercurile circumscrise triunghiurilor ,∗ ∗ ∗ ∗AB C BC A şi ∗ ∗CA B au de asemenea un punct

comun. 5) Centrul cercului circumscris triunghiului de simetrie ∗ ∗ ∗A B C al triunghiului ABC este simetricul centrului cercului circumscris triunghiului ABC faţă de punctul lui Coşniţă. Demonstraţie. Centrul cercului circumscris triunghiului podar al unui punct P este mijlocul segmentului *PP ( *P fiind izogonalul conjugat al lui P). Fie ∗O centrul cercului circumscris

triunghiului ∗ ∗ ∗A B C (Fig. 317). Cum 9O şi

*N sunt izogonal conjugate rezultă că ∗O este

imaginea mijlocului segmentului *9O N prin

A

B C

O

Fig. 316

D

E F X

Y

Z

A∗

B∗

C∗

Q

H 9O G O

T

N∗

O∗

Fig. 317

Page 306: Barbu_TEOREME_FUNDAMENTALE(2008).pdf

313

omotetie H ( , 4)G (unde am folosit teorema lui Boutte – „Triunghiul celor trei imagini”).

Fie T mijlocul segmentului *9O N . Din teorema lui Menelaus aplicată triunghiului 9O TG

avem: 9

9

4 3 11

3 2 2

∗ ∗

∗ ∗⋅ ⋅ = ⋅ ⋅ =OOO G N T

OGO T O N rezultă punctele *,O N şi ∗

O sunt coliniare. Cum

9

9

3∗

= =HOO T

TG O G rezultă 9 ,∗

O T O H adică *9 ,∗

O N O H deci *N este mijlocul segmentului

.∗OO

II.39. Teorema lui Kiepert

„Cum se face că matematica - produs prin excelenţă al gândirii umane, independent de experienţă – poate fi atât de admirabil adaptat obiectelor lumii reale?” – A. Einstein141

Pe laturile unui triunghi ABC, în exteriorul său, se construiesc triunghiurile isoscele asemenea ' , ' , 'BA C AB C BC A . Dreptele 'AA , 'BB şi 'CC sunt concurente. Demonstraţie. Notăm afixele punctelor cu litere mici corespunzătoare. Fie ' = A CB α şi "A mijlocul

laturii BC. Avem A 'A"

sinA 'C

α = , deci " sin

'1 sin

+=

+

a ca

αα

sau 2 sin

'2(1 sin )

+ +=

+b c c

α şi analog

se obţin egalităţile: 2 sin

'2(1 sin )

+ +=

+a c a

α,

2 sin'

2(1 sin )

+ +=

+b a b

α. Sumând ecuaţiile dreptelor

'AA , 'BB ,respectiv 'CC obţinem :

2 sin 2 sin ( 2sin ) ( 2 sin )0.

2(1 sin ) 2(1 sin ) 2(1 sin ) 2(1 sin )

b c c b c c a b c c a b c ca z a z

α α α αα α α α

+ + + + + + + +− − − + − = + + + +

Obţinem identitatea ( ) ( ) ( ) 0− + − + − =a c b c b a b a c , de unde rezultă că dreptele

'AA , 'BB şi 'CC sunt concurente. 141 Albert Einstein (1879-1955) – fizician german, profesor universitar la Berlin şi Princeton, laureat al Premiului

Nobel

A

B C

A'

B' C'

A"

Fig. 318

Page 307: Barbu_TEOREME_FUNDAMENTALE(2008).pdf

314

A

B C D

Fig. 320

II.40. Teorema lui Gergonne142

„Infinitul este numai un fel de a vorbi.” – C. Gauss143

Dacă cevienele AD,BE şi CE sunt concurente într-un punct P interior triunghiului ABC, atunci:

i) 1PD PE PE

AD BE CF+ + = ; ii) 2

AP BP CP

AD BE CF+ + = .

Demonstraţie.

i) Avem: [ ]

,[ ]

APD BPD

AD A ABC=

[ ]

[ ]

APE APC

BE A ABC= şi

[ ]

[ ]

APF APB

CF A ABC= , de unde prin sumare rezultă concluzia.

ii) Cum 1 , 1 , 1PD AP AP AP PE BP PF CP

AD AD AD BE BE CF CF

−= = − = − = − ,

prin sumare rezultă 1 3AP BP CP

AD BE CF

= − + +

, adică

2AP BP CP

AD BE CF+ + = .

II.41. Teorema lui Heron144

„Un matematician care nu are şi fire de poet nu poate fi niciodată un matematician complet.” - K. Weierstrass145

Fie a, b, c lungimile laturilor BC, CA respectiv AB ale unui triunghi ABC, iar 2p=a+b+c.

Aria triunghiului ABC este dată de formula [ ] ( )( )( ).= − − −ABCA p p a p b p c

Demonstraţie. Fie D piciorul înălţimii din A a triunghiului ABC. Notăm .

aAD h= Din teorema lui Pitagora generalizată rezultă

2 2 2 2 ,c a b a DC= + − ⋅ de unde 2 2 2

.2

a b cDC

a

+ −= Din triunghiul

dreptunghic ADC rezultă: 22 2 2

2 2

2

+ −= − =

a

a b ch b

a

2 2 2 2 2 22

1(2 ) (2 ),

4− − + ⋅ + + −ab a b c ab a b c

a de unde

2( )( )( )= − − −

ah p p a p b p c

a şi de aici obţinem: [ ] ( )( )( ).

2

⋅= − − −a

ABC

h aA p p a p b p c

142 Joseph Gegonne (1771-1859) – matematician francez, fondator al revistei Annales de Mathématiques în 1810 143 Carl Gauss (1777-1855) – matematician, fizician şi astronom german,contribuţii în teoria numerelor, geometrie

diferenţială, analiză matematică, statistică 144 Heron (10-75) – geometru egiptean 145 Karl Weierstrass (1815-1897) – matematician german, contribuţii importante în analiza matematică

A

B C D

E

F

P

Fig. 319

Page 308: Barbu_TEOREME_FUNDAMENTALE(2008).pdf

315

II.42. Teorema lui Catalan146 „Geometria este cea mai bună şi mai simplă dintre toate logicile, cea mai potrivită să dea inflexibilitate judecăţii şi raţiunii.” - Denis Diderot147

Trei antiparalele egale relative la laturile unui triunghi determină pe laturi puncte conciclice. Demonstraţie. Fie ' ", ' ", ' "A C B A C B trei

antiparalele egale ', " ( )A A BC∈ ,

', " ( )B B CA∈ , ', " ( ).C C AB∈ Din

' " " 'BA C B AC C A B≡ ≡ rezultă " ' " 'C A C BA B≡ ; cum ' " " 'A C A B≡ rezultă că ' " ' "A A B C este trapez isoscel, deci inscriptibil (1), Atunci ' "B C BC , deci dreapta ' "C B este antiparalelă cu ' "B C , de unde rezultă că patrulaterul ' " ' "B B C C este inscriptibil (2). Analog, patrulaterul

' " ' "A C C B este inscriptibil (3). Din relaţiile (2) şi (3) rezultă că punctele

', ", ', ", 'A C C B B sunt conciclice (4). Din relaţiile (4) şi (1) rezultă că punctele

', ", ', ", 'A C C B B ", 'C C sunt conciclice. Observaţie: Cercul ce conţine punctele ', ", ', ", 'A C C B B ", 'C C se numeşte cercul lui Taylor.

II.43. Teorema lui Blanchet

„Geometria se bazează pe această sinteză succesivă a imaginaţiei productive în generarea figurilor. Este o bază a axiomelor care formulează condiţiile intuiţiei sensibile a priori, potrivit căreia două drepte nu pot încadra un spaţiu.”- I. Kant148

Fie M un punct oarecare pe înalţimea AD a triunghiului ABC, ( )D BC∈ şi

E BM AC∈ ∩ , F CM AB∈ ∩ . Să se arate că AD este bisectoarea unghiului FDE . Demonstraţie. Ducem prin A o paralelă d la BC şi fie P DE d= ∩ ,

Q FD d= ∩ . Din asemănarea

triunghiurilor AFQ şi BFD, respectiv

146 Éugéne Catalan (1814-1894) – matematician belgian, contribuţii în geometrie, algebră şi analiză 147 Denis Diderot (1713-1734) –filosof şi scriitor francez, figură centrală a iluminismului 148 Immanuel Kant (1724-1804) – filosof german

A

B C A'

B'

C'

A"

B"

C"

Fig. 321

B C

Q P A

D

M

E

F

Fig. 324

Page 309: Barbu_TEOREME_FUNDAMENTALE(2008).pdf

316

AEP şi CED rezultă AQ AF

BD FB= şi

AP AE

DC EC= , de unde 1

AQ BD AF EC

AP DC FB AE= ⋅ ⋅ = (s-a utilizat

teorema lui Ceva), deci AQ AP= . Cum BC PQ şi AD BC⊥ rezultă AD PQ⊥ . Din

relaţiile (1) şi (2) rezultă că triunghiul PDQ este isoscel,deci AD este bisectoarea unghiului .PDQ

II.44. Teorema lui Alasia149 Un cerc intersectează laturile AB, BC, CA ale unui triunghi ABC în punctele

, '; , ',D D E E respectiv , '.F F Dreptele ', 'DE EF şi 'FD determină un triunghi ' ' 'A B C omologic cu triunghiul ABC. Demonstraţie.Fie ' ' ', ' ' ',A DE EF B FD EF= ∩ = ∩

' ' ',C FD DE= ∩ '' ' ' ,A B C BC= ∩

" ' ' ,B A C AC= ∩ '' ' ' .C A B AB= ∩

Teorema lui Menelaus aplicată triunghiului ABC cu transversalele

" ' , " ',A D F B D E− − − − respectiv " 'C E F− − ne dă:

" '1

'' '

A B D A F C

A C D B F A⋅ ⋅ = ,

" '1

" '

B C E B D A

B A E C D B⋅ ⋅ = ,

' ' '1

" '

C A F C E B

C B F A E C⋅ ⋅ = ,

de unde rezultă: " " "

" " "

A B B C C A

A C B A C B⋅ ⋅ =

' ' '1

' ' '

D B FA E C DB F A EC

D A FC E B DA F C ED

⋅ ⋅ ⋅ ⋅ ⋅ =

(cf. th. lui Carnot) şi din reciproca teoremei lui Menelaus rezultă că punctele ", ", "A B C sunt coliniare, iar din reciproca teoremei lui Desargues rezultă că triunghiurile ABC şi ' ' 'A B C sunt omologice. Observaţie: Din teoremă rezultă că dreptele ', 'AA BB şi 'CC sunt concurente.

149 Cristoforo Alasia (1869-1918) –matematician italian

A

B C E '

D

F'

C"

D'

Fig. 322

E

F

A'

B' C'

B"

A"

Page 310: Barbu_TEOREME_FUNDAMENTALE(2008).pdf

317

II.45. Teorema lui Ayme

„Nu există pe lume un stadiu care să pună mai armonios în acţiune facultăţile spiritului decât cel al matematicienilor. Matematicianul trăieşte mult timp şi totuşi rămâne tânăr; aripile sale nu se frâng de timpuriu şi porii săi nu-s obturaţi de praful ce se ridică pe marile drumuri prăfuite de vieţi obişnuite.” – James Sylvester150

Fie O centrul cercului circumscris unui triunghi ABC şi X, Y, Z punctele de intersecţie dintre mediatoarele segmentelor OA, OB, OC cu dreptele BC, CA respectiv AB. Demonstraţie. Fie D, E, F mijloacele segmentelor OA, OB respectiv OC. Deoarece EF este linie mijlocie în triunghiul isoscel BOC rezultă că patrulaterul BCFE este trapez isoscel,

deci punctele B, C, F şi E sunt conciclice (Fig. 323). Analog, punctele C, A, F şi D respectiv A, B, D şi E sunt conciclice. Conform teoremei lui Dergiades aplicată cercurilor precedente rezultă că punctele X, Y şi Z sunt conciclice.

Teorema lui Musselman

Fie * * *, ,A B C simetricele vârfurilor triunghiului ABC faţă de laturile opuse şi O centrul cercului circumscris triunghiului ABC. Cercurile circumscrise triunghiurilor

* * *, ,AOA BOB COC se întâlnesc într-un al doilea punct. Demonstraţie. Fie X, Y, Z centrele cercurilor considerate. Mediatoarele segmentelor OA, OB, OC trec prin punctele X, Y respectiv Z şi conform teoremei lui Ayme punctele X, Y şi Z

150James Sylvester (1814-1897) – matematician englez , professor la Universitatea Oxford, contribuţii importante în algebră

A

B C

O

Fig. 323

D

E F X

Y

Z

A∗

B∗

C∗

iG

Page 311: Barbu_TEOREME_FUNDAMENTALE(2008).pdf

318

sunt coliniare. Cum punctul O aparţine cercurilor considerate şi centrele lor sunt coliniare, rezultă că cercurile se întâlnesc într-un al doilea punct.

Observaţii:

i) Cercurile circumscrise triunghiurilor * * *, ,AOA BOB COC se numesc cercurile lui Musselman. ii) Al doilea punct de intersecţie dintre cercurile considerate se numeşte punctul lui Gibert ( )

iG al triunghiului ABC.

II.46. Teorema lui Bobillier151

„Ajung tot mai mult la concluzia că necesitatea geometriei noastre nu poate fi demonstrată...Poate că în altă viaţă vom reuşi să definim spaţiul, pentru că acum este practic imposibil.” - Carl Gauss152

Fie M un punct în planul triunghiului ABC. Perpendicularele ridicate din punctul M pe dreptele MA, MB, MC intersectează laturile BC, CA respectiv AB în trei puncte coliniare. Demonstraţie. Fie N, P şi Q punctele de intersecţie ale dreptelor AC, BC şi AB cu perpendicularele duse din M pe MB, MA respectiv MC. Avem:

[ ]

[ ]

sin

sin

MBP

MPC

APB MB MP BMP

PC A MC MP PMC

⋅ ⋅= = =

⋅ ⋅

sin

sin

MB BMP

MC PMC

sau

cos.

cos

⋅=

PB MB AMB

PC MC AMC

Analog se arată că cos

cos

NC MC BMC

NA MA AMB= ⋅

şi cos

.cos

QA MA AMC

QB MB BMC= ⋅

Deci,

1PB NC QA

PC NA QB⋅ ⋅ = şi din reciproca teoremei

lui Menelaus rezultă că punctele N, P şi Q sunt coliniare.

151 Étienne Bobiller (1898-1940) –geometru francez 152 Carl Gauss (1777-1855) – matematician, fizician şi astronom german,contribuţii în teoria numerelor, geometrie

diferenţială, analiză matematică, statistică

A

B C

M

N

P

Q

Fig. 325

Page 312: Barbu_TEOREME_FUNDAMENTALE(2008).pdf

319

II.47. Teorema lui Boutin „O demonstraţie matematică nu înseamnă o simplă alăturare de silogisme, ci silogisme aşezate într-o anumită ordine, iar ordinea în care sunt aşezate aceste elemente este mai importantă decât elementele însăşi.” - Henri Poincaré153

Fie

a b cM M M triunghiul median corespunzător unui triunghi ABC şi O centrul

cercului circumscris triunghiului ABC . Pe dreptele a

OM ,b

OM ,c

OM se consideră

punctele 1A , 1B , 1C astfel încât 1 1 1

a b c

OA OB OC

OM OM OM= = . Dreptele 1AA , 1BB , 1CC sunt

concurente într-un punct ce aparţine dreptei lui Euler a triunghiului ABC .

Demonstraţie. Din 1 1 1

a b c

OA OB OC

OM OM OM= = rezultă că triunghiurile

a b cM M M şi 1 1 1A B C sunt

asemenea, iar cum 1 1 1

a b cO AM B M C M= ∩ ∩ rezultă că triunghiul

a b cM M M şi 1 1 1A B C

sunt omotetice, centrul de omotetie fiind punctul O (Fig. 326). Dar şi triunghiurile

a b cM M M şi ABC sunt omotetice, centrul de omotetie fiind punctul G centrul de greutate

al triunghiului ABC . Atunci, rezultă că triunghiurile ABC şi 1 1 1A B C sunt omotetice,

centrul de omotetie aparţinând dreptei determinate de celelalte două centre de omotetie - dreapta OG - adică dreptele 1AA , 1BB , 1CC sunt concurente într-un punct ce aparţine

dreptei lui Euler a triunghiului ABC . Observaţie: Punctul de concurenţă al dreptelor 1AA , 1BB şi 1CC se numeşte punctul lui Franke.

153Henri Poincaré ( 1854 -1912) – matamatician şi fizician francez, contribuţii importante în toate ramurile matematicii

A

B C

1B 1C

O

G P

1A

aM

bM cM

Fig. 326

Page 313: Barbu_TEOREME_FUNDAMENTALE(2008).pdf

320

II.48. Teorema lui Cantor154 „Infinitul? Nici o întrebare nu a mişcat atât de profund spiritul omului.”- D. Hilbert155

Perpendicularele duse din mijloacele laturilor unui triunghi pe laturile opuse ale triunghiului tangenţial corespunzător sunt concurente. Demonstraţie.

Alegem ca reper complex cu originea în centrul triunghiului ABC şi fie A B CT T T triunghiul

său tangenţial. Notăm cu litere mici afixele punctelor corespunzătoare. Fie ', ', 'A B C mijloacele laturilor BC, AC, respectiv AB.

Avem: ' , ' , ' .2 2 2

b c a c a ba b c

+ + += = = Afixul centrului cercului lui Euler este egal cu

.2

+ +=a b c

n Deoarece ' 12 2

2

a b c b c

n a

o a a

+ + +−−

= = − ∈− −

rezultă că ' .A N AO

Cum ⊥B C

AO T T rezultă că ' ⊥ B CA N T T . Analog, se demonstrează că şi perpendicularele

din B’ şi C’ pe laturile A CT T respectiv

A BT T trec prin punctul N (centrul cercului lui lui

Euler al triunghiului ABC).

154 Georg Cantor (1845-1918) – matematician german, contribuţii remarcabile în teoria mulţimilor; este considerat

unul din fondatorii matematicii moderne. 155 David Hilbert (1962-1943) – matematician german, profesor la Universitatea din Göttingen, contribuţii

remarcabile în geometrie şi analiza matematică

A C"

B

C A’

O

B’

B" A "

C’

N

BT

AT

CT

Fig. 327

Page 314: Barbu_TEOREME_FUNDAMENTALE(2008).pdf

321

II.49. Teorema Carnot156

„…dacă Dumnezeu există cu adevărat şi a creat lumea, atunci, după cum ştim cu toţii, a creat-o conform Geometriei Euclidiene şi a înzestrat mintea umană cu concepţia a numai trei dimensiuni spaţiale. Cu toate acestea au existat şi mai există încă matematicieni, unii chiar geniali, care se îndoiesc că întregul univers a fost creat conform geometriei euclidiene.” – Feodor Dostoievski157

Teorema lui Carnot Fie triunghiul ABC şi punctele 'A BC∈ , 'B AC∈ respectiv 'C AB∈ . Perpendicularele duse din punctele ', ', 'A B C pe laturile , ,BC AC respectiv AB sunt concurente dacă şi

numai dacă 2 2 2 2 2 2' ' ' ' ' ' 0AC BC BA CA CB AB− + − + − = (0). Demonstraţie.

Presupunem că perpendicularele se întâlnesc într-un punct P. Din teorema lui Pitagora rezultă:

2 2 2' 'AC C P AP+ = (1)

2 2 2' 'BC C P BP+ = (2)

2 2 2' 'BA A P BP+ = (3)

2 2 2' 'CA A P CP+ = (4)

2 2 2' 'CB B P CP+ = (5)

2 2 2' 'AB B P AP+ = (6) Din ecuaţiile (1), (3) şi (5) respectiv (2), (4) şi (6) prin sumare rezultă:

2 2 2 2 2 2 2 2 2' ' ' ' ' 'AC C P BA A P CB B P AP BP CP+ + + + + = + + (7), respectiv

2 2 2 2 2 2 2 2 2' ' ' ' ' 'BC C P CA A P AB B P BP CP AP+ + + + + = + + (8). Scăzând membru cu membru relaţiile (7) şi (8) rezultă concluzia. Pentru a demonstra reciproca, fie ' 'P A P B P= ∩ . Fie D piciorul perpendicularei duse din P pe latura AB. Conform primei

părţi avem 2 2 2 2 2 2' ' ' ' 0AD BD BA CA CB AB− + − + − = , care cu ipoteza dă 2 2 2 2' 'AD BD AC BC− = − ( )∗ . Fie ,BD x= 'DC y= şi ' .C A z= Atunci, x y z c+ + = şi

din relaţia ( )∗ rezultă '.D C≡

Consecinţe: 1) Fiecare din relaţiile următoare este echivalentă cu relaţia (0): (9) ' ' ' ' ' ' ,c AC a BA b CB c C B a A C b B A⋅ + ⋅ + ⋅ = ⋅ + ⋅ + ⋅

(10) 2 2 22( ' ' ') ,c AC a BA b CB c a b⋅ + ⋅ + ⋅ = + +

(11) ' sin 'sin 'sin ' sin ' sin ' sinAC C BA A CB B C B C A C A B A B⋅ + + = ⋅ + +

Demonstraţie. Relaţia (0) este echivalentă cu ( ' ' )( ' ' ) ( ' ' )( ' ' )AC C B AC C B BA A C BA A C− + + − + + ( ' ' )( ' ' ) 0CB B A CB B A− + = sau

156 Lazare Carnot (1753-1823) – matematician şi inginer francez 157 Feodor Dostoievski (1821-1881) - scriitor rus

A

B C A'

B' C'

P

Fig. 328

Page 315: Barbu_TEOREME_FUNDAMENTALE(2008).pdf

322

' ' ' ' ' ' 0AC c C B c BA a A C a CB b B A b⋅ − ⋅ + ⋅ − ⋅ + ⋅ − ⋅ = (adică relaţia (9)). Relaţia (10) se obţine din relaţia (9) astfel: 2( ' ' ') 2( ' ' ' )c AC a BA b CB c C B a A C b B A⋅ + ⋅ + ⋅ = ⋅ + ⋅ + ⋅ =

2 2 2' ' ' ' ' ' .c C B a A C b B A c AC a BA b CB c a b= ⋅ + ⋅ + ⋅ + ⋅ + ⋅ + ⋅ = + + Relaţia (11) este echivalentă cu relaţia (9) utilizând teorema sinusurilor. 2) Fie punctele diferite A şi B. Să se determine locul geometric al punctului M din plan care diferenţa 2 2AM BM− este constantă.

Demonstraţie. Fie ,MN AB⊥ N MN AB= ∩ . Atunci, 2 2 2 2 2AM AN MN BM BN− = = −

de unde rezultă că : 2 2 2 2 .AM BM AN BN const− = − = Dacă M aparţine locului geometric, atunci şi N aparţine locului geometric şi reciproc. Locul geometric este o dreaptă.

3) Mediatoarele laturilor unui triunghi sunt concurente. Demonstraţie. Fie ', ', 'A B C mijloacele laturilor ,BC AC respectiv AB ale triunghiului ABC (Fig. 329). Din reciproca teoremei lui Carnot rezultă :

2 2 2 2 2 2' ' ' ' ' ' 0,AC BC BA CA CB AB− + − + − = deoarece ' ' ,BA A C= ' 'B A B C= şi ' 'C A C B= .

4) Înălţimile unui triunghi sunt concurente. Demonstraţie. Fie ', ', 'AA BB CC înălţimile triunghiului ABC (Fig. 330). Avem:

2 2 2 2 2' ' 'AB BA AA AC CA− = = − de unde 2 2 2 2' 'AB AC BA CA− = − . Analog se obţin

relaţiile: 2 2 2 2' 'AC BC AC BC− = − şi 2 2 2 2' ' .BC AB CB AB− = − Sumând relaţiile

precedente obţinem : 2 2 2 2 2 2' ' ' ' ' ' 0AC BC BA CA CB AB− + − + − = , relaţie care arată că înălţimile ', ', 'AA BB CC sunt concurente.

5) Perpendiculare duse în punctele de tangenţă ale cercului înscris în triunghiul ABC cu laturile acestuia pe laturile triunghiului sunt concurente . Demonstraţia este evidentă.

C

A

A'

B' C'

O

B Fig. 329

C

H

A

B

B'

A'

C'

Fig. 330

Page 316: Barbu_TEOREME_FUNDAMENTALE(2008).pdf

323

6) Teorema lui Soons (Existenţa ortopolului unei drepte) Vârfurile , ,A B C ale triunghiului ABC se proiectează pe o dreaptă d oarecare ce nu trece prin vârfurile triunghiului ABC în ,L M respectiv N. Perpendicularele din L pe BC , M pe AC şi N pe AB sunt concurente într-un punct numit ortopolul dreptei d a triunghiului ABC. Demonstraţie. Fie ,AL BM şiCN perpendicularele

duse din A, B şi C pe dreapta d , ( , ,L M N d∈ ). Fie ' ,LA BC⊥ ' ,MB AC⊥ 'NC AB⊥ , ' ,A BC∈

' ,B AC∈ 'C AB∈ (Fig. 331). Avem: 2 2 2 2 2AM ML AL AN LN− = = − , de unde rezultă 2 2 2 2AM AN LM LN− = − . Analog ,

2 2 2 2BN BL MN LM− = − şi 2 2 2 2CL CM LN MN− = − . Sumând relaţiile precedente rezultă

2 2 2 2 2 2 0AM AN BN BL CL CM− + − + − = , adică 2 2 2 2 2 2( ' ' ) ( ' ' ) ( ' ' )MB AB C N AC C B C N+ − + + + −

2 2 2 2 2 2( ' ' ) ( ' ' ) ( ' ' ) 0BA A L A C LA B M B C+ + + − + =

egalitate echivalentă cu : 2 2 2 2 2 2' ' ' ' ' ' 0AB AC C B BA A C B C− + − + − = şi din reciproca teoremei lui Carnot rezultă

concluzia.

II.50. Teoremele lui Carnot158

„există matematicieni.... care îndrăznesc să viseze că două paralele, care conform teoriei lui Euclid, nu se pot întâlni niciodată pe pământ, se întâlnesc undeva la infinit. Eu.... am ajuns la concluzia că, din moment ce nu înţeleg nici măcar atâta lucru, cum aş putea să - l înţeleg pe Dumnezeu? ” – Feodor Dostoievski159

Teorema lui Carnot Într-un triunghi ascuţitunghic ABC suma distanţelor de la centrul cercului circumscris (O) la laturile triunghiului este egală cu suma lungimilor razelor cercului înscris şi circumscris triunghiului. Demonstraţie. Fie A1, B1, C1, proiecţiile lui O pe laturile BC, CA, respectiv AB. Avem de demonstrat faptul că: 1 1 1+ + = +OA OB OC r R . Notăm cu x, y, z

lungimile distanţelor 1OA , 1OB , respectiv 1OC . Din

teorema lui Ptolomeu pentru patrulaterul inscriptibil

1 1OB AC rezultă: 1 1 1 1 1 1⋅ + ⋅ = ⋅OB AC OC AB OA B C ,

adică 2 2 2⋅ + ⋅ = ⋅c b a

y z R , de unde .+ =cy bz Ra

Analog se demonstrează că + =ax cz Rb şi + =bx ay Rc . Sumând relaţiile precedente rezultă:

2( ) ( ) 2+ + − + + =x y z ax by cz Rp ,unde

158 Lazare Carnot (1753-1823) – matematician şi inginer francez 159 Feodor Dostoievski (1821-1881) - scriitor rus

C A'

B

N

M

C' A

L

B' d

Fig. 331

A

B

C A1

B1 C1

Fig. 332

Page 317: Barbu_TEOREME_FUNDAMENTALE(2008).pdf

324

2

+ +=a b c

p . Deoarece [ ]2 2+ + = =ABC

ax by cz A rp rezultă .+ + = +x y z R r

Observaţii:

1) Egalitatea 1 1 1+ + = +OA OB OC r R se numeşte relaţia lui Carnot.

2) Dacă triunghiul ABC este obtuzunghic (de exemplu fie ( ) 90> °m A )), atunci teorema

lui Carnot devine .− + + = +x y z R r

3) Cum 1( ) ( )= m BOA m BAC rezultă cos=x R A şi analog rezultă

cos=y R B , cos=z R C , care înlocuite în relaţia lui Carnot dau:

cos cos cos 1+ + = +r

A B CR

.

4) O formă echivalentă a teoremei lui Carnot este:

[ ]

( cos cos cos )

4

+ ++ + =

⋅ABC

abc A B Cx y z

A.

Consecinţă: Dacă ABCD este un patrulater inscriptibil şi r1, r2, r3, r4, sunt razele cercurilor înscrise în triunghiurile ABC, BCD, CDA, respectiv DAB să se arate că 1 3 2 4+ = +r r r r .

Demonstraţie. Notăm cu x, y, z, t, u, şi v distanţele de la centrul cercului circumscris patrulaterului (O) la AB, BC, CD, DA, AC, respectiv DB. Fie R raza cercului circumscris patrulaterului. Teorema lui Carnot aplicată triunghiurilor ABC, BCD, CDA, şi DAB dă: + + = +x y u R r (1),

+ + = +y z v R r (2), + − = +z t u R r (3), + − = +x t v R r (4). Din relaţiile (1) şi (2)

respectiv (3) şi (4) 1 2+ − − = −x y z v r r şi 3 4− − + = −z u x v r r , de unde:

1 2 3 4 0− + + =r r r r , adică 1 3 2 4+ = +r r r r .

A

B

C

D v x

z u

t

y

O

Fig. 333

Page 318: Barbu_TEOREME_FUNDAMENTALE(2008).pdf

325

Teorema lui Carnot Dacă un cerc taie laturile unui triunghi în punctele D, M, E, N, F, P atunci:

1.DB MB EC NC FA PA

DC MC EA NA FB PB⋅ ⋅ ⋅ ⋅ ⋅ =

Demonstraţie.

Din puterea punctului faţă de un cerc avem:

( ) ,A AF AP AN AEρ = ⋅ = ⋅ ( ) ,B BD BM BP BFρ = ⋅ = ⋅

( )C CM CD CE CNρ = ⋅ = ⋅ . Cum ( )( )

( )( )

( )( )

1B C A

C A B

ρ ρ ρ

ρ ρ ρ⋅ ⋅ =

rezultă : 1DB MB EC NC FA PA

DC MC EA NA FB PB⋅ ⋅ ⋅ ⋅ ⋅ = .

II.51. Teorema lui Casey160 „Ca şi în geometrie, înţeleg prin poezie o anumită simbolică pentru reprezentarea formelor posibile de existenţă. Pentru mine poezia este o prelungire a geometriei, aşa că, rămânând poet, nu am părăsit niciodată domeniul divin al geometriei.” – Ion Barbu

Teorema lui Casey

Fie cercurile ( , )C O r , 1 1( , )C O r , 2 2( , )C O r , 3 3( , )C O r , 4 4( , )C O r . Dacă cercurile

1C , 2C , 3C şi 4C sunt tangente interior la cercul C (orientarea fiind în ordinea

numerotării), atunci avem următoarea relaţie între distanţele tangenţiale dintre cercuri : 12 34 23 41 13 24d d d d d d⋅ + ⋅ = ⋅ (prin distanţă tangenţială ijd dintre cercurile iC şi jC

înţelegem lungimea tangentei comune exterioare duse la cele două cercuri, cele două cercuri aflându-se de aceeaşi parte a tangentei). Demonstraţie.

160 John Casey (1820-1891) – matematician britanic, profesor la Universitatea Dublin, contribuţii importante în

geometrie

A

B

C

F N

P

D

E

M

Fig. 334

1T

4T 3T

2T

2O 1O

B A

3O 4O

Fig. 335

O

Page 319: Barbu_TEOREME_FUNDAMENTALE(2008).pdf

326

Fie 1 2 3 4, , ,T T T T punctele de tangenţă ale cercurilor 1C , 2C , 3C şi respectiv 4C cu cercul C .

Din teorema cosinusului în triunghiul i jTOT şi i jOOO obţinem : 2 2

2

2cos( )

2i j

i j

r TTTOT

r

−= şi 2 2 2( ) ( ) 2( )( )cos( )i j i j i j i jOO r r r r r r r r TOT= − + − − − − sau

22 2

2( ) ( )( ) i j

i j i j i j

TTOO r r r r r r

r= − + − − . Fie tangenta comună interioară cercurilor ( , )i iC O r

şi ( , )j jC O r , ijd = AB . Din trapezul dreptunghic j iABO O rezultă: 2

2 2 2 2 2 22

( ) ( ) ( )( ) ( )i j

ij i j i j i j i j i j

TTd AB OO O A O B r r r r r r r r

r= = − − = − + − − − − adică

22

2( )( ) i j

ij i j

TTd r r r r

r= − − , de unde rezultă ( )( )i j

ij i j

TTd r r r r

r= − − . Egalitatea

1 2 3 4 2 3 4 1 1 3 2 4⋅ + ⋅ = ⋅d d d d d d este echivalentă cu

3 4 2 31 2 4 11 2 3 4 2 3 4 1( )( ) ( )( ) ( )( ) ( )( )

T T T TTT T Tr r r r r r r r r r r r r r r r

r r r r− − ⋅ − − + − − ⋅ − − =

1 3 2 41 3 2 4( )( ) ( )( )

TT T Tr r r r r r r r

r r− − ⋅ − − , adică 1 2 3 4 2 3 4 1 1 3 2 4⋅ + ⋅ = ⋅TT T T T T T T TT T T care

este teorema lui Ptolemeu.

Fig. 336

A

B

C

C

C1

C2

C3

Page 320: Barbu_TEOREME_FUNDAMENTALE(2008).pdf

327

1) Fie triunghiul ABC înscris în cercul C şi cercurile C1 ,C2 ,C3 tangente interioare

cercului C şi laturilor BC, CA respectiv AB, astfel încât A şi C1 , B şi C2, C şi C3 să fie de

părţi diferite faţă de BC, CA, respectiv AB. Notăm cu 1 2 3, ,l l l lungimile tangentelor din A,

B, C la cercurile C1 ,C2 respectiv C3 şi prin ijt lungimea tangentei comune exterioare a

cercurilor Ci şi Cj , , 1,3,= ≠i j i j .Atunci: 12 23 31= =t t t dacă şi numai dacă

1 2 3, ,2 2 2

+ + += = =b c c a a b

l l l

Demonstraţie. Cercurile C1 ,C2 ,C3 sunt tangente laturilor triunghiului ABC în mijlocul

acestora. Aplicând teorema lui Casey pentru cercurile: C şi A,C2, C1, C3 ; C şi B, C3 ,C1 ,C2;

C şi C, C1 ,C3 ,C2 se obţin egalităţile:

13 12 1 23 12 23 2 13 23 13 3 12; ; , ( )2 2 2 2 2 2⋅ + ⋅ = ⋅ ⋅ + ⋅ = ⋅ ⋅ + ⋅ = ⋅ ∗b c c a a bd d l d d d l d d d l d de unde

rezultă 1 2 3, , .2 2 2

b c c a a bl l l

+ + += = = Reciproc, dacă 1 2 3, ,

2 2 2

b c c a a bl l l

+ + += = = prin

înlocuirea acestora în relaţiile ( )∗ rezultă 13 23 23 12( ) ( ),− = −b d d c d d

12 13 13 23( ) ( )− = −c d d a d d , 23 12 12 13( ) ( )− = −a d d b d d şi de aici:

12 13 23 12 13 23 0− − −= = =

+ +

d d d d d d

a b c a b c, de unde 12 23 31.= =d d d

2) În triunghiul ABC , fie 1C şi 2C două cercuri tangente exterior în punctul I, tangente

laturii BC a ABC şi tangente interior cercului circumscris triunghiului ABC . Să se arate că punctul I este centrul cercului înscris în triunghiul ABC .

B D C α

I

X

α

α Y

z

c b

A

Fig. 337

Page 321: Barbu_TEOREME_FUNDAMENTALE(2008).pdf

328

Demonstraţie. Fie 1 =XC BCI , 2 =YC BCI , D=AI BCI . Fie AI z= ,

BX x= , CY y= , DX DY DI α= = = . Din teorema lui Casey aplicată cercurilor

1( , , , )A C B C şi respectiv 2( , , , )A C C B ne dă : (2 )az bx c yα+ = + (1) şi

(2 )az cy b xα+ = + (2), de unde ( )bx cy c bα− = − adică x c

y b

αα

+=

+, relaţie echivalentă cu

BD AB

DC AC= , ceea ce implică că AI este bisectoarea unghiului A şi

acBD

b c=

+. Din (1) şi

(2) rezultă ( )az b cα= + , de unde z b c

aα+

= , adică AI AB

ID BD= , egalitate care arată că BI

este bisectoarea ABC , deci I este centrul cercului înscris în triunghiul ABC . TEOREMA LUI CASEY ( T. GENERALIZATĂ A LUI PTOLEMEU) Prin ijt vom nota lungimea tangentei comune exterioare cercurilor 1C şi 2C (cele două

cercuri se afla de aceaşi parte a tangentei). Dacă cercurile 1C , 2C , 3C şi 4C toate tangente

interior (sau exterior) unui cerc C în ordinea ciclică 1C , 2C , 3C , 4C , atunci

12 34 23 41 13 24t t t t t t⋅ + ⋅ = ⋅ . Mai mult dacă există relaţiile 12 34 14 23 13 24 0t t t t t t± ⋅ ± ⋅ ± ⋅ = pentru

o anumită alegere a semnelor + sau - , există un cerc tangent (interior sau exterior) tuturor celor 4 cercuri.

II.52. Teorema lui Clairaut161

„Matematicianul este îmblânzitorul ce a domesticit infinitul.” – Lucian Blaga162

Pe laturile AB şi AC ale unui triunghi ABC sc construiesc în exterior (sau în interior) paralelogramele 1 1AA B B şi 2 1AA C C . Fie M= 1 1 2 1A B A C∩ , 3A =MA BC∩ ; construim

punctul 4A astfel încat 3 4( )A AA∈ , 3 4A A =AM şi paralelogramul 2 2BCC B cu

2 3 4BB A A şi 2 3 4≡BB A A (Fig. 338).

Teorema lui Clairaut

Suma ariilor paralelogramelor 1 1AA B B şi 2 1AA C C este egală cu aria paralelogramului

2 2BCC B .

Demonstraţie. Fie 3 2 1 1 = ∩B BB A B şi 3C = 2 2 1CC A C∩ . Din

AM= 2BB = 3BB = 2CC = 3CC , 3 =MB AB şi 3 =MC AC rezultă că: 1 1 3[ ] [ ]=AA B B BB MAA A ,

2 1 3[ ] [ ]=AA C C CC MAA A , 3 3 2 2[ ] [ ]BB C C BB C CA A= şi

3 3[ ] [ ]MB C ABCA A= . Avem:

1 1 2 1 3 3 3 3[ ] [ ] [ ] [ ] [ ] [ ]AA B B AA C C BB MA CC MA BCC MB ABCA A A A A A+ = + = − =

3 3 3 3 3 3 2 2[ ] [ ] [ ] [ ] [ ]MB C BB C C ABC BB C C BB C CA A A A A+ − = = .

161 Claude Clairaut (1713-1765) – matematician, fizician francez, membru al Academiei Franceze, contribuţii în

analiza matematică şi geometrie 162 Lucian Blaga (1895-1961) - filozof, umanist, jurnalist, poet, dramaturg, traducător, profesor universitar şi

diplomat român, membru titular al Academiei Române

Page 322: Barbu_TEOREME_FUNDAMENTALE(2008).pdf

329

Observaţie: O consecinţă a teoremei lui Clairaut este teorema lui Pitagora.

Astfel, dacă ( ) 90= °m BAC , 1 1AA B B şi 2 1AA C C sunt pătrate şi 3 4MA A A BC= =

(Fig. 339). Din teorema lui Clairaut avem: 1 1 2 1 2 2[ ] [ ] [ ]AA B B AA C C BB C CA A A+ =

relaţie echivalentă cu: 2 2 2+ =AB AC BC .

M

B

C

A

1B

1C

2A

3C

1A 3B

3A

2B

4A

2C

Fig. 339

C B

M

A

2B 2C

2A

1A

1B

3A

4A

1C

3C

Fig. 338

3B

Page 323: Barbu_TEOREME_FUNDAMENTALE(2008).pdf

330

II.53. Teorema lui Mathieu163 „Istoria ne arată, că viaţa este doar un episod între două veşnicii ale morţii şi în acest episod gândirea conştientă durează doar o clipă. Gândirea este doar o explozie de lumină în mijlocul unei nopţi lungi, dar această explozie este totul.” - Henri Poincaré164

Într-un triunghi izogonalele a trei ceviene concurente sunt la rândul lor concurente.

Demonstraţie. Fie triunghiul ABC şi cevienele ', ', 'AA BB CC concurente în punctul 'M .

Fie ", ", "AA BB CC ( A'' BC∈ , B'' AC∈ , C'' AB∈ ) izogonalele dreptelor ', 'AA BB ,

respectiv 'CC (Fig. 340). Atunci, ( ') ( " ),=m BAA m A AC ( ' ) ( " )=m B BA m B BC şi

( ') ( " ).=m ACC m C CB Din forma trigonometrică a teoremei lui Ceva aplicată pentru

cevienele concurente în M rezultă: sin BAA' sin ' sin '

1sin A'AC sin ' sin '

⋅ ⋅ =

ABB ACC

B BC C CB sau

sin '' sin '' sin '' 1

sin " sin '' sin ''

A AC B BC C CB

BAA ABB ACC⋅ ⋅ =

şi din reciproca teoremei lui Ceva rezultă că

izogonalele AA'',BB'',CC '' sunt concurente într-un punct ".M Observaţie : Punctele 'M şi "M se numesc puncte izogonale.

163 Claude Mathieu (1783-1875) – matematician francez, profesor la Ecole Polytechnique din Paris 164 Henri Poincaré ( 1854 -1912) – matematician şi fizician francez, contribuţii importante în toate ramurile matematicii

A

B C A' A"

C"

C'

B'

B" M ''

M '

Fig. 340

Page 324: Barbu_TEOREME_FUNDAMENTALE(2008).pdf

331

II.54. Teorema lui Miquel

„Dincolo de pamânt şi infinit Cătam să aflu cerul unde vine.

Şi-un glas solemn atunci s-a auzit Şi cerul şi infernul sunt în tine.” Omar Khayyam165

Teorema lui Miquel Pe laturile triunghiului ABC se consideră punctele necoliniare D, E, F ( , , )D BC E CA F AB∈ ∈ ∈ . Să se arate că cercurile circumscrise triunghiurilor AEF, BFD, CDE au un punct comun P. Demonstraţie. Fie P punctul comun cercurilor circumscrise triunghiurilor BDF şi DCE. Deoarece patrulaterele FBDP şi CEPD sunt inscriptibile

rezultă: ( ) 360 ( ) ( )m FPE m FPD m DPE= °− − =

360 [180 ( )] [180 ( )] 180 ( ).m B m C m A° − ° − − ° − = ° − deci patrulaterul FPEA este inscriptibil, adică punctul P aparţine cercului circumscris triunghiului AFE.

Observaţii:

1) Punctul P de concurenţă a celor 3 cercuri se numeşte punctul pivot al triunghiului DEF. 2) Triunghiul DEF se numeşte triunghiul lui Miquel . 3) Cercurile circumscrise triunghiurilor AFE, BFD, CDE se numesc cercurile lui Miquel.

4) Din teorema lui Miquel rezultă ( ) 180 ( ), ( ) 180 ( ),m FPE m A m FPD m B= ° − = ° −

( ) 180 ( )m DPE m C= °− .

5) Fie P un punct şi a b cP P P triunghiul său cevian în raport cu triunghiul ABC. Cercurile

circumscrise triunghiurilor b c

AP P , a c

BP P şi a b

CP P se intersectează într - un punct MP

numit punctul pivot asociat lui P. 1) Coordonatele unghiulare ale punctului pivot P sunt:

( ) ( ), ( ) ( ),m EDF m A m DEF m B+ + respectiv ( ) ( ).m EFD m C+

Demonstraţie. Avem: ( ) ( ) ( ) ( ) ( )m BPC m BPD m DPC m BFD m DEC= + = + =

[180 ( ) ( )] [180 ( ) ( )] 360 [ ( ) ( )] [ ( ) ( )]m BDF m B m EDC m C m BDF m EDC m B m C° − − + ° − − = °− + − + =

360 [180 ( )] [180 ( )] ( ) ( ).m EDF m A m EDF m A° − ° − − ° − = + Analog se arată că

( ) ( ) ( )m CPA m FED m B= + şi ( ) ( ) ( )m APB m EFD m C= + .

2) În triunghiul ABC fie punctele , ' [ ], , ' [ ], , ' [ ].D D BC E E CA F F AB∈ ∈ ∈ Dacă P şi 'P sunt punctele pivot ale triunghiului DEF, respectiv ' ' 'D E F atunci punctele P şi 'P coincid dacă şi numai dacă triunghiurile DEF şi ' ' 'D E F sunt asemenea.

165 Omar Khayyam (1048-1122) – matematician, poet, filosof, astronom persan, contribuţii în algebră şi geometrie

A

B C

E F

P

D

Fig. 341

Page 325: Barbu_TEOREME_FUNDAMENTALE(2008).pdf

332

Demonstraţie. Avem: ( ) ( ) ( )= + =m BPC m BPD m DPC ( ) ( )+m BFD m DEC =

[180 ( ) ( )] [180 ( ) ( )] ( ) ( )m DFE m EFA m DEF m FEA m EDF m BAC° − − + ° − − = + . Analog se

arată că ( ) ( ) ( )m CPA m DEF m ABC= + , ( ) ( ) ( )m APB m EFD m ACB= + (Fig. 342). Dacă

punctele P şi 'P coincid, atunci ( ) ( ' )m BPC m BP C= , de unde ( ) ( )m EDF m PAC+ =

( ' ' ') ( )m E D F m BAC+ , adică ( ) ( ' ' ')m EDF m E D F= şi analoagele de unde rezultă că

triunghiurile DEF şi ' ' 'D E F sunt asemenea. Dacă triunghiurile DEF şi ' ' 'D E F sunt

asemenea atunci, ( ) ( ' ), ( ) ( ' ), ( ) ( ' ),m BPC m BP C m APB m AP B m APC m AP C= = =

deci P coincide cu 'P .

3) Triunghiul podar al punctului pivot P al triunghiului DEF este asemenea cu triunghiul DEF. Demonstraţie: Fie 1 2 3PP P triunghiul podar al punctului pivot P (Fig. 343). Deoarece

patrulaterele 2 3AP PP , 1 3BPPP şi 1 2CPPP sunt inscriptibile rezultă că punctul pivot al

triunghiului 1 2 3PP P este tocmai punctul P şi conform teoremei precedente rezultă că

triunghiurile 1 2 3PP P şi DEF sunt asemenea.

4) Centrul cercului circumscris (O) al triunghiului ABC este punctul pivot asociat al centrului de greutate (G) al triunghiului ABC . Demonstraţie. Dacă

a b cM M M este triunghiul median al triunghiului ABC , atunci

patrulaterele , ,c b a c a b

AM OM BM OM M CM O sunt inscriptibile, deci O este punctul pivot

asociat al lui G. 5) Ortocentrul (H) al triunghiului ABC este punctul pivot asociat tot al lui H Demonstraţie: Dacă

a b cH H H este triunghiul ortic al triunghiului ABC ,atunci

patrulaterele b c

HH AH , c aH HH B ,

b aHH CH sunt inscriptibile, deci H este punctul pivot

asociat al lui H.

A

B

C

E

E '

D D'

F

F' x P

Fig. 342

A

B C

E

F P

D

Fig. 343

1P

3P 2P

Page 326: Barbu_TEOREME_FUNDAMENTALE(2008).pdf

333

6) Centrul cercului înscris (I) în triunghiul ABC este punctul pivot asociat al punctului lui Gergonne () al triunghiului ABC . Demonstraţie. Dacă

a b cC C C este triunghiul de contact al triunghiului ABC , atunci

patrulaterele c b

AC IC ,a c

BC IC ,b a

CC IC sunt inscriptibile, deci cercurile circumscrise

triunghiurilor c b

AC C ,a c

BC C şi b a

CC C se interesectează în I şi cum

a b c

AC BC CCΓ = I I rezultă concluzia.

7) Dreptele ce unesc punctul pivot (M) asociat unui punct P cu picioarele cevienelor lui P intersecteaza laturile triunghiului ABC sub acelaşi unghi. Demonstraţie. Fie

a b cP P P triunghiul cevian al punctului P în raport cu triunghiul ABC .

Deoarece patrulaterele , ,a b b c c aMP CP MP AP MP BP sunt inscriptibile rezultă că

( ) ( ) ( )a b c

m MPC m MP A m MP B= = .

II.55. Teorema lui Sawayama - Thebault

„Matematica e arta de a gândi prin teoreme.” - Ion Barbu166

Fie D un punct pe latura BC a triunghiului ABC, 'O centrul unui cerc (C’) tangent

dreptelor AD, DC şi cercului (C) circumscris triunghiului ABC în punctele M, N respectiv

P centrul cercului înscris (I) în triunghiul ABC aparţine dreptei MN. Demonstraţie. Fie 'M şi 'N punctele de intersecţie dintre MP şi PN cu cercul circumscris triunghiului ABC iar ' .= ∩J AN MN Fie "N punctul de intersecţie dintre mediatoarea

segmentului BC şi dreapta NP, iar O centrul cercului circumscris triunghiului ABC (Fig. 344). Atunci, ' "O N ON de unde rezultă că ' "O NP ON P≡ (1). Dar

' 'O NP O PN≡ (2), triunghiul 'O NP fiind isoscel ' 'O P O N≡ . Din relaţiile (1) şi (2) rezultă " ,OPN ON P≡ deci triunghiul

"OPN este isoscel. Atunci " ( ),ON OP R≡ =

adică punctul "N aparţine cercului circumscris triunghiului ABC, deci punctele

"N şi 'N coincid. Punctul 'N este astfel mijlocul arcului BC, deci PN este bisectoarea unghiului BPC (3) şi 'AN este bisectoarea

unghiului .BAC Deoarece cercurile (C) şi

(C’) sunt tangente interior în punctul P,

' ',MN M N de unde rezultă că

1' ' ' ( ' )

2MJA M N A M PA MPA mM A

≡ ≡ ≡ =

deci patrulaterul MAPJ este inscriptibil (4).

166 Ion Barbu (1895-1961) – matematician român, profesor la Universitatea din Bucureşti, contribuţii în algebră şi geometrie

A

B

C

O

N

M

P O'

D

N'

M '

J

Fig. 344

Page 327: Barbu_TEOREME_FUNDAMENTALE(2008).pdf

334

Din teorema lui Miquel aplicată triunghiului AMJ (cu ,M AM J AJ∈ ∈ şi N MJ∈ ) rezultă că cercul circumscris triunghiului NPJ este tangent dreptei AJ în J (5). Cercul cu centrul în 'N şi raza BN trece prin centrul cercului înscris (I) în triunghiul ABC (vezi „Cercuri exînscrise”). Deoarece ( ) ( ' ) ( ' )m BPN m N AC m N BC= = rezultă că cercul

circumscris triunghiului BNP este tangent dreptei 'BN în B, de unde rezultă că cercul

C ( ', ' )N N B este ortogonal cercului circumscris triunghiului BNP, în consecinţă şi cercului

circumscris triunghiului MNP. Cum cercul C ( ', ' )N N B este ortogonal şi cercului

circumscris triunghiului JNP rezultă că ' ' ,N J N I= deci ,J I≡ adică I aparţine dreptei MN. Teorema lui Sawayama – Thebault Fie punctul D pe latura BC a triunghiului ABC, I centrul cercului înscris în acest triunghi, C 1 1 1( , )O r un cerc tangent interior cercului circumscris triunghiului ABC şi

segmentelor AD şi BD, iar C 2 2 2( , )O r un cerc tangent interior cercului circumscris

triunghiului ABC şi segmentelor AD şi CD. Punctele 1,O I şi 2O sunt coliniare.

Demonstraţie. Fie 1 2, ' ( , ' )O N BC O N BC N N BC⊥ ⊥ ∈ de unde 1 2 '.O N O N Conform

teoremei de mai sus dreptele MN şi ' 'M N trec prin I (unde M şi 'M sunt punctele de

tangenţă ale cercurilor C 1 şi C 2 cu AD) (Fig. 345). Triunghiurile DMN şi 1O NM fiind

isoscele rezultă că 1DO este mediatoarea segmentului MN şi bisectoarea unghiului

.MDN Analog 2DO este bisectoarea unghiului ' ',N DM deci 1 2 ,DO DO⊥ de unde

1 ' 'DO N M şi 2 .DO MN Conform teoremei lui Pappus aplicată hexagonului

1 2 1,O NIO DO punctele 2 ,O I şi 1O sunt coliniare.

Consecinţe:

1) Dacă ( ) 2 ,m ADC θ= atunci 21

2

.O I

tgIO

θ=

Demonstraţie. Fie aC proiecţia lui I pe

BC. Avem: 1

2 'a

a

NCO I

IO C N= (1). Deoarece

1DO este bisectoarea unghiului ADN

rezultă 1( )m O DN θ= şi cum

1'N I DO rezultă ( ' ) .m IN N θ=

Atunci, ,'a

a

ICtg

N Cθ = deci

'a

N C rctgθ= (2). Iar în triunghiul

: (90 ) ,a

a

rIC N tg ctg

C Nθ θ° − = =

deci aC N rtgθ= (3). Din relaţiile (1),

(2) şi (3) rezultă 21

2

.O I

tgIO

θ=

A

B

C

2O

N

M aC

1O

D

N'

M '

I

Fig. 345

Page 328: Barbu_TEOREME_FUNDAMENTALE(2008).pdf

335

2) 2 21 2cos sin .r r rθ θ= +

Demonstraţie. Avem: ' ' ( )a a

N N N C C N r ctg tgθ θ= + = + şi

1 2' 'N N ND DN rctg r tgθ θ= + = + , deci 2 21 21 2cos sin .

r ctg r tgr r r

ctg tg

θ θθ θ

θ θ+

= = ++

3) În triunghiul ABC , fie 1C şi 2C două cercuri tangente exterior în punctul I, tangente

laturii BC a ABC şi tangente interior cercului circumscris triunghiului ABC . Să se arate că punctul I este centrul cercului înscris în triunghiul ABC . Demonstraţia rezultă din teorema lui Sawayama – Thebault. 4) Fie patrulaterul inscriptibil ABCD şi 1 1 1 2 2 2 3 3 3 4 4 4( , ), ( , ), ( , ), ( , )C I r C I r C I r C I r cercurile

înscrise în triunghiurile BCD, CDA, DAB, respectiv ABC. Atunci, patrulaterul 1 2 3 4I I I I

este dreptunghi şi 1 3 2 4+ = +r r r r . Demonstraţie.

Fie =IAC BD E , 1 1 1 2 2 2 3 3 3 4 4 4' ( , ), ' ( , ), ' ( , ), ' ( , )C O C O C O C Oρ ρ ρ ρ cercurile tangente

cercului circumscris triunghiului ABC şi laturilor AE, EB; BE, CE; CE, DE; respectiv DE şi AE. Fie ( ) ( )= = m AEB m CED θ , de unde ( ) ( )= = − m AED m CEB π θ (Fig.

346). Conform teoremei lui Thébault rezultă 1 2 3 2 3 4 3 4 1 1 1 2, , ,∈ ∈ ∈ ∈I O O I O O I O O I OO şi

2 24 3 2 1

3 1 1 3 2 2

− = = =

O I O Itg ctg

I O I O

π θ θ, 23 21 4

4 2 2 4 2= =O IO I

tgI O I O

θ. Analog se arată că

1O

2O

3O

4O

A B

C D

E

1I 2I

3I 4I

Fig. 346

Page 329: Barbu_TEOREME_FUNDAMENTALE(2008).pdf

336

4 3 3 1 3 2 1 32 4 1 4 2 1 4 2

3 1 4 1 4 2 1 2 1 3 2 3 2 4 3 4

, , ,= = = =O I O I O I O IO I O I O I O I

I O I O I O I O I O I O I O I O, de unde rezultă

3 4 2 4 1 4 1 3, , I I O O I I OO 1 2 2 4 2 3 1 3, I I O O I I OO . Deoarece 1 3 2 4⊥OO O O ( 1 3OO şi 2 4O O

sunt bisectoarele unghiurilor DEC , respectiv CEB ) rezultă că patrulaterul 1 2 3 4I I I I

este paralelogram. Avem: 2 21 3 1 3 4 2 2 4( ) cos ( )sin .

2 2+ = + + + = +r r r r

θ θρ ρ ρ ρ

II.56. Teorema lui Schooten167

„Atâtea claile de fire stângi! Găsi-vor gest închis, să le rezume,

Să nege, dreapta, linia ce frângi: Ochi în virgin triunghi taiat spre lume?”

Ion Barbu168

Dacă M este un punct situat pe arcul BC al cercului circumscris triunghiului echilateral ABC , atunci CMBMAM += .

Demonstraţie.

Soluţia 1. Fie ( )D AM∈ astfel încât MD BD≡ .

Deoarece ( ) ( ) 60m ACB m AMB= = ° rezultă că

triunghiul MBD este echilateral, deci MD BM≡ .

Deoarece ,AB BC≡ BD BM≡ şi ABD CBM≡ rezultă că CBMABD ∆≡∆ , de unde .MCAD ≡ Atunci, .MBMCDMADAM +=+= Soluţia 2. Din prima teorema a lui Ptolemeu rezultă: ,BMACMCABBCAM ⋅+⋅=⋅ adică

MBMCAM += ( deoarece AB BC AC≡ ≡ )

Generalizarea teoremei lui Schooten Fie ABC un triunghi echilateral. Pentru orice punct M din plan are loc relatia

,MCMAMB +≤ cu egalitatea dacă şi numai dacă punctul M aparţine cercului circumscris triunghiului .ABC

Demonstraţie. Avem ,ABMCBCMAACMBMCMAMB ⋅+⋅≤⋅⇔+≤ care reprezintă inegalitatea lui Ptolemeu .

1 Frans van Schooten (1615 – 1660) – matematician olandez, promotor al geometriei carteziene 168 Ion Barbu (1895-1961) – matematician român, profesor la Universitatea din Bucureşti, contribuţii în algebră şi geometrie

M

C

A

B

D

Fig. 347

Page 330: Barbu_TEOREME_FUNDAMENTALE(2008).pdf

337

II.57. Teorema lui Smarandache169 „Noi ştim că unu ori unu fac unu,

dar un inorog ori o pară nu ştim cât face. Ştim că cinci fără patru fac unu, dar un nor fară o corabie nu ştim cat face.” – N Stănescu170

Fie , ,

a b cH H H picioarele înălţimilor unui triunghi ascuţitunghic ABC. Dacă ', ', 'a b c

sunt lungimile laturilor triunghiului podar a b cH H H , atunci

( ) 2 2 24 ' ' ' ' ' 'a b a c b c a b c+ + ≤ + + , unde a, b, c reprezintă lungimile laturilor triunghiului

ABC.

Demonstraţie.

Lemă: Daca p şi 'p sunt semiperimetrele triunghiurilor

ABC şi a b cH H H atunci '

2

pp ≤ .

Demonstraţie: Avem: c

AH = bcosA, b

AH = ccosA, de

unde 2 2 2 2 22 cos cosb c b c b cH H AH AH AH AH A a A= + − ⋅ = ,

adică cosb cH H a A= . Analog, cosa cH H b B= şi

cosa bH H c C= (Fig. 348). Astfel, ' ' '

'2

a b cp

+ += =

c o s c o s c o s

2

a A b B c C+ + sau

( )sin 2 sin 2 sin 2' 2 sin sin sin

2

R A B Cp R A B C

+ += =

(unde am utilizat teorema sinusurilor, R fiind raza cercului circumscris triunghiului ABC ),

deci: [ ]' 22 2 2

ABCAa b c

p RR R R R

= ⋅ ⋅ = , (unde [ ]ABCA reprezintă aria triunghiului ABC). Cum

[ ]ABCA r p= ⋅ , (r–raza cercului înscris în triunghiul ABC) rezultă: '2

r pp p

R= ⋅ ≤ (unde am

utilizat inegalitatea lui Euler 2r ≤ R). Demonstraţia teoremei lui Smarandache. Utilizând inegalităţile cunoscute:

2 2 2 23( ) ( ) 3( ) , , ,xy x z yz x y z x y z x y z+ + ≤ + + ≤ + + ∀ ∈ rezultă

( ) ( )2 222 2 2 22 '1 1

' ' ' ' ' ' ( ' ' ') ( ).3 3 3 3 4 4

p a b cpa b a c b c a b c a b c

+ ++ + ≤ + + = ≤ = ≤ + +

169 Florentin Smarandache (1954 - ) - matematician român, profesor la Universitatea New-Mexico contribuţii în

teoria numerelor şi statistică matematică 170 Nichita Stănescu (1933 – 1983) – eseist, poet român, ales postum membru al Academiei Române

A

B

C Ha

Hb

Hc

Fig. 348

Page 331: Barbu_TEOREME_FUNDAMENTALE(2008).pdf

338

II.58. Teorema lui Snapper

„...poezia nu este lacrimă ea este însuşi plânsul,

plânsul unui ochi neinventat, lacrima ochiului celui care trebuie să fie frumos,

lacrima celui care trebuie să fie fericit.” Nichita Stănescu171

Fie un punct oarecare Q în planul triunghiului ABC, , ,a b cM M M mijloacele laturilor

BC,AC respectiv AB. Fie (a) o dreaptă ce trece prin aM şi este paralelă cu AQ, (b) o

dreaptă ce trece prin bM paralelă cu BQ şi ( c) o dreaptă ce trece prin cM paralelă cu CQ. Atunci : i) dreptele a,b,c sunt concurente într-un punct P; ii) centrul de greutate G al triunghiului ABC se află pe dreapta PQ astfel încât 2PG=GQ.

Demonstraţie. Prin omotetia de centru G şi raport 1

2 −

triunghiul ABC se transformă în

triunghiul a b cM M M . Omotetia H 1,

2G

transformă dreptele AQ,BQ,CQ în dreptele a,b,c

respectiv paralele. Cum dreptele AQ,BQ,CQ sunt concurente în Q rezultă că dreptele a,b,c dreptele a,b,c sunt concurente şi fie P acest punct. Cum prin omotetia de centru G şi raport

1

2 −

punctul Q se transformă în punctul P rezultă : 1

2GP GQ= −uuur uuur

de unde rezultă

concluzia.

171 Nichita Stănescu (1933 – 1983) – eseist, poet român, ales postum membru al Academiei Române

Fig. 349

P

Q

A

B C Ma

Mc

Mb

a

b

c

Page 332: Barbu_TEOREME_FUNDAMENTALE(2008).pdf

339

II.59. Teorema lui Urquhart - Pedoe

„Matematica va fi limba latină a viitorului, obligatorie pentru toţi oamenii de ştiinţă,tocmai pentru că matematica permite accelerarea maximă a circulaţiei ideilor ştiinţifice.” – Grigore Moisil172 În triunghiul ABC fie transversala ' 'B D C− − astfel încât ( ' )B B A∈ , ( )D BC∈ ,

' ( )C AC∈ . Să se arate că ' 'AB BD AC C D+ = + dacă şi numai dacă

' 'AB B D AC CD+ = + . Demonstraţie. Vom demonstra mai întîi teorema:

Lema lui Breusch

Fie triunghiurile 1 1 1A B C şi 2 2 2A B C astfel încât 1 1( ) 2=m A α ,

2 2( ) 2=m A α , 1 1( ) 2=m B β ,

2 2( ) 2=m B β ,

1 1( ) 2=m C γ , 2 2( ) 2=m C γ şi 1 1 2 2BC B C= . Atunci 1 1 1 2 2 2( ) ( )p A B C p A B C≥

dacă şi numai dacă 1 1 2 2tg tg tg tgβ γ β γ⋅ ≥ ⋅ , unde am notat cu ( )p XYZ perimetrul triunghiului XYZ . Demonstraţie.

1 1 1 1 1 1 1

1 1 1 1

( )1

+= + =

p A B C A B A C

B C B C

1 1 1 1 1 1

1 1 1 1 1

sin2 sin2 2sin( ) cos( )1 1

sin 2sin( ) cos( )

+ + ⋅ −+ = + =

+ ⋅ +

γ β γ β γ βα γ β γ β

1 1

1 1 1 1 1 1

2cos cos 2

cos cos sin sin 1 tg tg

γ βγ β γ β γ β

⋅= =

⋅ − ⋅ − ⋅ (Fig. 350) şi analog,

2 2 2

2 2 2 2

( ) 2.

1

p A B C

B C tg tgγ β=

− ⋅

Condiţia 1 1 1 2 2 2( ) ( )≥p A B C p A B C

este echivalentă cu

1 1 2 2

2 2

1 1≥

− ⋅ − ⋅tg tg tg tgγ β γ β adică cu

1 1 2 2⋅ ≥ ⋅tg tg tg tgγ β γ β .

Demonstraţia teoremei. Fie ( ) 2 '=m BAD β , ( ) 2=m BDA β , ( ') 2=m DAC γ ,

( ') 2 '=m ADC γ . Din lema lui Breusch rezultă că ( ' ) ( )=p AB D p ACD dacă şi numai dacă

' (90 ') (90 )⋅ °− = ⋅ °−tg tg tg tgβ γ γ β adică ' '⋅ = ⋅tg ctg tg ctgβ γ γ β sau ' '⋅ = ⋅tg tg tg tgβ β γ γ ,

condiţie echivalentă cu ( ) ( ' )=p ABD p AC D , ceea ce trebuia demonstrat.

172 Grigore Moisil (1906-1973) – matematician român, profesor la Universitatea din Iaşi, membru al Academiei Române

A

B C

D

C'

B'

Fig. 350

2 'β 2γ

2 'γ

Page 333: Barbu_TEOREME_FUNDAMENTALE(2008).pdf

340

II.60. Relaţii metrice în triunghiul dreptunghic

„Lumea este o imensă problemă matematică. Dumnezeu este geometrul atotputernic care pune problema şi o rezolvă.” – Gottfried Leibniz173

Teorema înălţimii: Într-un triunghi dreptunghic lungimea înălţimii duse din vârful

unghiului drept este medie proporţională între lungimile catetelor pe ipotenuză.

Demonstraţie.

Fie triunghiul ABC cu ( ) 90= °m BAC şi D piciorul înălţimii

duse din A pe latura BC. Deoarece ADB ADC≡ şi ≡BAD ACD rezultă că triunghiurile ABD şi CAD să fie

asemenea, de unde =AD BD

DC AD, adică 2 .= ⋅AD BD DC

Reciproca teoremei înălţimii: Fie ( )∈D BC proiecţia vârfului A al triunghiului ABC pe

latura BC. Dacă 2 ,AD BD DC= ⋅ atunci triunghiul ABC este dreptunghic.

Demonstraţie. Deoarece ≡ADB CDA şi AD BD

DC AD= rezultă că triunghiurile ABD şi CAD

sunt asemenea, de unde avem ≡ABD DAC şi ≡BAD ACD . Dar ( ) ( ) ( ) 180+ + = °m ABC m BAC m ACB

Adică 2 ( ) ( ) 180 + = ° m ABC m ACB , de unde rezultă

( ) ( ) 90+ = °m ABC m ACB şi de aici

( ) 180 90 90 ,= °− ° = °m BAC adică triunghiul ABC este

dreptunghic. Observaţie: Dacă ( ) \∈D BC BC , atunci teorema reciprocă nu

mai este adevărată. Se observă că triunghirile DAB şi DCB sunt asemenea, de unde rezultă că 2 ,AD BD DC= ⋅ dar triunghiul

ABC nu este dreptunghic. Deci, condiţia ca ( )∈D BC este esenţială pentru ca triunghiul

ABC să fie dreptunghic. Teorema catetei: Într-un triunghi dreptunghic lungimea

unei catete este medie proporţională între lungimile ipotenuzei şi a proiecţiei acestei catete pe ipotenuză.

173 Gottfried Leibniz (1646-1716) – matematician şi filosof german, contribuţii importante în analiza matematică şi algebră

A

B C D

Fig. 352

A

B C D

Fig. 353

A

B C D

Fig. 351

Page 334: Barbu_TEOREME_FUNDAMENTALE(2008).pdf

341

Demonstraţie.Fie D proiecţia vârfului A al triughiului ABC ( ( ) 90= °m BAC ) pe ipotenuza

BC. Deoarece ≡BAD ACB şi ≡ADB BAC rezultă că triunghiurile ABD şi CBA sunt

asemenea, de unde =BD AB

AB BC, adică Analog se arată că 2 .AC BC DC= ⋅

Reciproca teoremei catetei: Fie ( )D BC∈ proiecţia vârfului A al triunghiului ABC pe

latura BC. Dacă 2AB BC BD= ⋅ (sau 2AC BC DC= ⋅ ) atunci triunghiul ABC este dreptunghic.

Demonstraţie. Deoarece ≡ABD ABC şi AB BD

BC AB= rezultă că triunghiurile ABD şi CAB

sunt asemenea, de unde ≡BDA BAC (1). Analog, din asemănarea triunghiurilor CDA şi

CAB rezultă ≡CDA CAB (2). Din relaţiile (1) şi (2) rezultă ≡BDA CDA , iar ( ) ( ) 180+ = °m BDA m CDA , de unde : ( ) 90= °m BDA şi deci ( ) 90= °m BAC .

Teorema lui Pitagora: Într-un triunghi dreptunghic pătratul lungimii ipotenuzei este egal cu suma pătratelor lungimilor catetelor.

Demonstraţie. Fie D proiecţia vârfului A pe ipotenuza BC a triunghiului ABC. Din teorema catetei obţinem 2AB BD BC= ⋅ şi 2 ,AC CD BC= ⋅ de unde

2 2 2( ) .AB AC BC BD CD BC+ = + =

Reciproca teoremei lui Pitagora: Dacă într-un triunghi pătratul lungimii unei laturi este

egal cu suma pătratelor lungimilor celorlalte două laturi, atunci triunghiul este dreptunghic.

Demonstraţie. Fie triunghiul ABC în care 2 2 2= +BC AB AC şi triunghiul dreptunghic ' ' 'A B C astfel încât ' ' =A B AB şi ' ' .A C AC= Din teorema lui Pitagora aplicată în

triunghiul ' ' 'A B C rezultă 2 2 2' ' ' ' ' '= + =B C A B A C 2 2 2 ,+ =AB AC BC de unde ' '.BC B C= Din congruenţa triunghiurilor ABC şi ' ' 'A B C (conform cazului de congruenţă

LLL) rezultă ( ) ( ') 90 .m A m A= = °

II.61. Aria unui triunghi

„Ar fi trebuit sa fii un cerc subţire, dar n-ai fost, n-ai fost asa.

Ar fi trebuit să fiu un romb subţire, dar n-am fost, n-am fost aşa..." Nichita Stănescu În cele ce urmează notăm cu [ ]ABC

A aria triunghiului ABC, cu a,b,c lungimile laturilor

BC,CA, respectiv AB, iar cu , ,a b ch h h lungimile înălţimilor triunghiului duse din A,B,

respectiv C. Din definiţia ariei unui triunghi avem: [ ] 2 2 2a b c

ABC

a h b h c hA

⋅ ⋅ ⋅= = = .

Page 335: Barbu_TEOREME_FUNDAMENTALE(2008).pdf

342

1) Aria unui triunghi este egală cu jumătatea produsului a două laturi înmulţit cu sinusul unghiului dintre ele.

Demonstraţie. Din triunghiul a

ABH avem sin ah

Bc

= (Fig. 354), sau sin( ) ah

Bc

π − =

(Fig. 355), de unde sinah c B= , deci [ ]

sin

2ABC

a c BA

⋅ ⋅= .

Observaţie: Prin permutări circulare obţinem [ ]

sin sin

2 2ABC

c b A b a CA

⋅ ⋅ ⋅ ⋅= = .

2) Aria unui triunghi este egală cu [ ] ( )( )( )ABCA p p a p b p c= − − − , unde 2

a b cp

+ +=

(formula lui Heron). Demonstraţie. Avem

[ ]

sin ( )( ) ( )2 sin cos

2 2 2 2ABC

a b C ab C C p a p b p p cA ab

ab ab

⋅ ⋅ − − −= = ⋅ ⋅ ⋅ = ⋅ sau

[ ] ( )( )( )ABCA p p a p b p c= − − − .

3) Aria unui triunghi este egală cu [ ] 4ABC

abcA

R= , unde R este raza cercului circumscris

triunghiului ABC.

Demonstraţie. [ ]

sinsin

2 2 2 2 4ABC

a c B ac ac b abcA B

R R

⋅ ⋅= = ⋅ = ⋅ = .

4) Aria unui triunghi ABC este egală cu

[ ] ( ) ( ) ( )ABC a b c a b cA pr p a r p b r p c r rr r r= = − = − = − = , unde r este raza cercului înscris,

iar , ,a b cr r r sunt razele cercurilor exînscrise.

Demonstraţie: Aceste formule rezultă imediat din descompunerea triunghiului ABC. Dacă I este centrul cercului înscris în triunghiul ABC atunci:

[ ] [ ] [ ] [ ]

1 1 1

2 2 2ABC AIB BIC CIAA A A A ar br cr pr= + + = + + = şi [ ] [ ] [ ] [ ]a a aABC ABI ACI BCIA A A A= + − =

A

B C a

H

aH

bH

cH

Fig. 354

c ah

b

A

B C a aH

Fig. 355

c b

Page 336: Barbu_TEOREME_FUNDAMENTALE(2008).pdf

343

( )( )

2 2 2 2a a a a

a

c r b r a r b c a rp a r

⋅ ⋅ ⋅ + − ⋅+ − = = − ⋅ . Ultima formulă se obţine înmulţind

primele patru expresii ale ariei triunghiului ABC şi ţinând seama de formula lui Heron.

5) Aria unui triunghi ABC este egală cu 2 2[ ] .

2 2 2 2 2 2ABC a

A B C A B CA r ctg ctg ctg r ctg tg tg= =

Demonstraţie. Din .2 2 2

= ⋅ ⋅ ⋅A B C

p r ctg ctg ctg (vezi „Cercul înscris”) şi [ ]ABCA pr= rezultă

2[ ] 2 2 2ABC

A B CA r ctg ctg ctg= . A doua egalitate se obţine utilizând teorema 27) – „Cercuri

exînscrise”, efectuând produsul ( )( )( )p p a p b p c− − − .

6) Dacă vârfurile A,B,C ale triunghiului ABC au coordonatele carteziene ( , ), ( , ),A A B Bx y x y respectiv ( , )

C Cx y , atunci aria triunghiului ABC este egală cu:

[ ]

1

2ABCA = ⋅ ∆ , unde

1

1

1

A A

B B

C C

x y

x y

x y

∆ = .

Demonstraţie. Ecuaţia dreptei BC este:

1

1 0

1B B

C C

x y

x y

x y

= , iar distanţa de la punctul A la

dreapta BC este egală cu: 2 2

( , )( ) ( )C B C B

d A BCx x y y

∆=

− + −, deci aria triunghiului ABC

este egală cu: [ ]

1

2ABCA = ⋅ ∆ .

7) Consecinţă: Dacă vârfurile A,B,C ale triunghiului ABC au afixele , ,

A Bz z respectiv

Cz

atunci aria triunghiului ABC este egală cu: [ ]ABCA = ∆ , unde

1

1 .4

1

A A

B B

C C

z ziz z

z z

∆ = ⋅ .

Demonstraţia rezultă imediat utilizând proprietatea 5) şi faptul că 2

A A

A

z zx

+= şi

2A A

A

z zy

i

−= .

8) Pe laturile triunghiului ABC se consideră punctele 1 1 1, ,A B C astfel încât

1 1 1

1 1 1

, ,= = =BA CB AC

p q rAC B A C B

. Dacă 1 1 1 1 1 1 , , = ∩ = ∩ = ∩P AA CC R BB CC Q BB AA ,

atunci 2

[ ] [ ]

(1 )

(1 )(1 )(1 )

−= ⋅

+ + + + + +PQR ABC

pqrA A

p pq q qr r pr.

Page 337: Barbu_TEOREME_FUNDAMENTALE(2008).pdf

344

Lemă: Pe laturile BC şi AC ale unui triunghi ABC se consideră punctele 1A , respectiv 1B

astfel încât 1 1

1 1

,= =BA CB

p qAC B A

. Dacă 1 1= ∩Q AA BB să se arate

că1 1

1, ( 1).

+= = +

AQ p BQp q

QA pq QB

Demonstraţie. Fie 1 2 1 2, ( )∈B B AA B BC . Avem :

1

1 1 2

= =BABQ

BQ A B 1

1 2 1

( 1)⋅ = ⋅ = +AC AC

p p p qA B AB

. Analog,

fie 1 2 1 2, ( )∈A A BB A AC ,

avem: 1 1

1 1 2 1 2

1AB CBAQ p

QA B A qB A pq

+= = = (Fig. 356).

Observaţie: Din 1

( 1)= +BQ

p qBQ

rezultă

1 1

+=

+ +BQ p pq

BB p pq şi din

1

1+=

AQ p

QA pq rezultă

1

1

1

+=

+ +AQ p

AA p pq.

Demonstraţie teoremă. Avem 1

1

[ ][ ] 1 1 1 1

[ ] 1 [ ] 1

( , ), ,

( , )

⋅= ⋅ = = ⋅

⋅RB CPQR

RB C ABC

AA BC d R BC B C RBQR PR

A RB RC A AC d B AC AC BB

de unde rezultă că

[ ] 1 1 1

[ ] 1 1 1

(1)= ⋅ ⋅ = ⋅ ⋅ ⋅PQR

ABC

A BC CC BCQR PR QR PR

A BB RC AC BB CC RC AC.

Utilizând lema de mai sus obţinem:

1 1

, (2).1 1

+= =

+ + + +CR pr CR q qr

CC r pr CC q qr Din 1

1

=CB

qB A

rezultă 1 (3)1

=+

B C q

AC q. Mai mult,

1

1 1 1 1

1 1= − = − − =B QQ R B Q B R

B B B B B B B B

(1 )(1 )

1 (4)1 1 (1 )(1 )

+ + −− − =

+ + + + + + + +p pq qr q pqr

q pq q qr q pq q qr şi 1

1 1 1

1RCPR PC

CC CC CC= − − =

(1 )(1 )(5).

(1 )(1 )

r pqr

q qr r pr

+ −+ + + +

Din relaţiile (1), (2), (3), (4) şi (5) rezultă concluzia.

A

B C 1A

1B Q

Fig. 356

2A

2B

A

B C 1A

1B

Q

Fig. 357

1C P R

Page 338: Barbu_TEOREME_FUNDAMENTALE(2008).pdf

345

CAPITOLUL III

TRIUNGHIURI REMARCABILE

III.1. Triunghiul ortic

„Matematica poate să descopere o anumită ordine chiar şi în haos.” - Ch. Stein174

În triunghiul ABC, fie , ,a b c

H H H picioarele înălţimilor duse din vârfurile A, B, respectiv C

pe laturile triunghiului ABC. Triunghiul a b c

H H H se numeşte triunghiul ortic al

triunghiului ABC. 1) Triunghiul ortic este triunghiul cevian al triunghiului ABC corespunzător ortocentrului. 2) Triunghiul ortic este triunghiul podar al ortocentrului triunghiului ABC în raport cu triunghiul ABC. 3) Triunghiurile , ,

b c a c b aAH H H BH H H C

sunt asemenea cu triunghiul ABC. Demonstraţie. Deoarece patrulaterul

b cBCH H este inscriptibil

( )b c

BH C BH C≡ rezultă că

c bAH H ACB≡ şi ,

b cAH H ABC≡

deci triunghiurile c b

AH H şi ACB sunt

asemenea. Analog se arată că triunghiurile

a cH BH şi

b aH H C sunt asemenea cu

triunghiul ABC. 4) Dreptele , ,

a b b c c aH H H H H H sunt antiparalele cu dreptele AB, BC, respectiv CA.

Demonstraţia rezultă din proprietatea precedentă. 5) Semidreptele [ ,[ ,[

a b cAH BH CH sunt bisectoarele unghiurilor triunghiului ortic.

Demonstraţie. Deoarece a c a b

BH H BH H BAC≡ ≡ rezultă că

( ) ( )c a b a

m H H A m H H A= = 90 ( ),m BAC° − iar a

AH este bisectoarea unghiului

b a cH H H . Analog,

bBH şi

cCH sunt bisectoarele unghiurilor

a b cH H H , respectiv

b c aH H H .

174Charles Stein (1920 - ) – matematician american, profesor la Universitatea Stanford, contribuţii în statistica

matematică

A

B C

D

H

aH

bH

cH

Fig. 358

Page 339: Barbu_TEOREME_FUNDAMENTALE(2008).pdf

346

6) Ortocentrul H al triunghiului ABC este centrul cercului înscris în triunghiul ortic

a b cH H H .

Demonstraţie. Deoarece , ,a b c

AH BH CH sunt bisectoarele unghiurilor triunghiului ortic

rezultă că punctul de intersecţie al lor (adică H) este centrul cercului înscris în triunghiul ABC.

7) Vârfurile triunghiului ABC sunt centrele cercurilor exînscrise triunghiului ortic

.a b c

H H H

Demonstraţie. Fie c a

D H H∈ astfel încât [ ]a c

H H D∈ (Fig. 358). Avem

,a c a b a

BH H DH C H H C≡ ≡ deci a

H C este bisectoarea exterioară a unghiului

a bDH H . Cum

cCH este bisectoarea exterioară a unghiului

a c bH H H rezultă că punctul

C este centrul cercului exînscris triunghiului a b c

H H H tangent laturii a b

H H .

8) Cercul circumscris triunghiului ortic al unui triunghi ABC este cercul lui Euler al triunghiului ABC. Demonstraţie. Vezi „Cercul lui Euler”.

9) Raza cercului circumscris triunghiului ortic are lungimea egală cu jumătate din lungimea razei cercului circumscris triunghiului ABC. Demonstraţie. Fie ', ', 'A B C punctele de intersecţie dintre înălţimile triunghiului ABC (Fig. 359) cu cercul circumscris triunghiului

.ABC Deoarece ', ', 'A B C sunt simetrice ortocentrului faţă de laturile triunghiului ABC, rezultă că triunghiul ortic

a b cH H H

este asemenea cu triunghiul ' ' '.A B C Cum , ,

b c c aH H H H

a bH H sunt linii mijlocii în

triunghiul ' ' 'A B C rezultă că raportul de

asemănare este 1,

2deci '

2

RR = (R este

raza cercului circumscris triunghiului ABC şi 'R raza cercului circumscris triunghiului ortic

a b cH H H ).

10) Triunghiul ortic a b c

H H H al triunghiului ABC este omotetic cu triunghiul

circumpedal ' ' 'A B C al ortocentrului H al triunghiului ABC, centrul de omotetie fiind ortocentrul triunghiului ABC Demonstraţia rezultă din teorema precedentă. 11) Lungimile laturilor triunghiului ortic

a b cH H H sunt egale cu cos , cosa A b B şi

cosc C , (a, b, c sunt lungimile laturilor BC, CA, respectiv AB).

A

B C

H

aH

bH

cH

A'

B'

C'

O

Fig. 359

Page 340: Barbu_TEOREME_FUNDAMENTALE(2008).pdf

347

Demonstraţie. Dacă triunghiul ABC este ascuţitunghic atunci din asemănarea triunghiurilor ABC şi

a b cH H H rezultă

a b bH H H C

c a= , de unde

cos cos= ⋅ = ⋅ = ⋅a b b

c cH H H C a C c C

a a .

Analog, cos=b c

H H a A şi

cos .=c a

H H b B Fie triunghiul ABC

obtuzunghic (Fig. 360) ( ( ) 90 ).> °m A

Din asemănarea triunghiurilor ABC şi

c bAH H rezultă b c b

H H aH

a b= , de

undeb c b

aH H AH

b= ⋅ . În triunghiul

bAH C avem cos(180 )°− = b

AHBAC

b,

adică cos= −b

AH b A , de unde: cos cos .= − =b c

H H a A a A Analog, cos=a c

H H b B şi

cos .=a b

H H c C

12) Perimetrul triunghiului ortic este egal cu cos cos cos .a A b B c C+ +

Demonstraţia rezultă din proprietatea precedentă. Observaţie: Pentru un triunghi ascuţitunghic perimetrul triunghiului ortic este egal cu

(sin 2 sin 2 sin 2 ) 4 sin sin sinop R A B C R A B C= + + = .

13) Fie p perimetrul unui triunghi ascuţitunghic ABC şi

op perimetrul triunghiului

ortic. Atunci, ,o

p R

p r= unde R şi r sunt razele cercului circumscris, respectiv înscris în

triunghiul ABC.

Demonstraţie. Avem [ ]2ABC

o

A rpp

R R= = , sau

o

p R

p r= .

14) Dintre toate triunghiurile înscrise într-un triunghi ascuţitunghic ABC, triunghiul ortic are perimetrul minim. Demonstraţie. Vezi „Teorema lui Fagnano”. 15) Dacă tringhiul ABC este ascuţitunghic atunci ( ) 180 2 ( ),= °−

b a cm H H H m BAC

( ) 180 2 ( )= °− a b c

m H H H m ABC şi ( ) 180 2 ( ).= °− a c b

m H H H m ACB

Demonstraţie. Din ( ) ( ) ( )= = a c b a

m BH H m H H C m BAC rezultă ( ) 180 2 ( ).= °− b a c

mH H H m BAC

Analog se arată şi celelalte relaţii.

H

B C

A

aH

bH

cH

Fig. 360

Page 341: Barbu_TEOREME_FUNDAMENTALE(2008).pdf

348

16) Dacă triunghiul ABC este obtuzunghic ( ( ) 90 )> °m BAC , atunci

( ) 2 ( ) 180= − ° b a c

m H H H m BAC , ( ) 2 ( )= a b c

m H H H m BCA , ( ) 2 ( )= b c a

m H H H m ABC .

Demonstraţie.

( ) 180 2 ( ) 180 2(180 ( )) 2 ( ) 180 ;= ° − = ° − ° − = − ° b a c a c

m H H H m BH H m BAC m BAC

( ) ( ) ( ) 2 ( )= + = a b c a b b c

m H H H m H H A m BH H m ACB şi ( ) 2 ( )= b c a

m H H H m ABC .

17) Aria triunghiului ortic este egală cu [ ]cos cos cos

2a b cH H H

abc A B CA

R

⋅ ⋅= (R este raza

cercului circumscris triunghiului ABC ).

Demonstraţie. Avem [ ]sin

.2a b c

a c a b b a c

H H H

H H H H H H HA

⋅ ⋅=

Dar cos ,=a cH H b B

cos .=a bH H c C Dacă triunghiul ABC este ascuţitunghic, atunci

sin sin(180 2 ) sin(2 )= ° − = = b a cH H H A A 2sin cos cos cos ,⋅ = ⋅ = ⋅a a

A A A AR R

de unde

rezultă concluzia. Dacă triunghiul ABC este obtuzunghic ( ( ) 90 )> °m BAC , atunci

sin sin(2 180 ) sin(2 ) 2sin cos ( cos ) cos ,= − ° = − = − ⋅ = ⋅ − = ⋅ b a c

a aH H H A A A A A A

R R de unde rezultă concluzia. 18) Fie a b cH H H triunghiului ortic al triunghiului ascuţitunghic ABC. Atunci:

[ ][ ] 2cos cos cosa b cH H H ABC

A A B C A= ⋅ ⋅ ⋅ .

Demonstraţia rezultă din proprietatea precedentă, ţinând cont că [ ] 4ABC

abcA

R= .

Observaţii:

1) Aria triunghiului ortic este maximă când produsul 2cos cos cosA B C este maxim, adică ( ) ( ) ( ) 60m A m B m C= = = ° , caz în care triunghiul ortic coincide cu triunghiul

median. 2) Dacă triunghiul ABC este obtuzunghic de exemplu ( ) 90m A > ° , atunci

( )[ ] [ ]2cos cos cosa b cH H H ABC

A A B C Aπ= − ⋅ ⋅ ⋅ .

19) Aria triunghiului ortic al ortocentrului triunghiului ABC este egală cu:

[ ] [ ]

2 2

24a b c ABCH H H

R OHA A

R

−= ⋅

Demonstraţie. Vezi „Triunghiul podar”. 20) Raza cercului înscris în triunghiul ortic

a b cH H H al triunghiului ascuţitunghic ABC

este egală cu 2 cos cos cos .R A B C

Page 342: Barbu_TEOREME_FUNDAMENTALE(2008).pdf

349

Demonstraţie. Deoarece în triunghiul ABC, 4 sin sin sin2 2 2

= ⋅ ⋅A B C

r R (vezi „Cercul

înscris”), raza cercului circumscris triunghiului ortic este egală cu 2

R, rezultă:

2 2 24 sin sin sin 2 sin sin sin

2 2 2 2 2 2 2

π π π− − −= ⋅ ⋅ ⋅ ⋅ = ⋅ ⋅ =a b c

h

H H HR A B Cr R

2 cos cos cos .R A B C

Observaţie: Deoarece [ ] 4ABC

abcA

R= rezultă [ ]2 cos cos cos .

= ⋅

H H Ha b cABC

A A A B C

21) Laturile triunghiului ortic al triunghiului ABC sunt paralele cu tangentele la cercul circumscris triunghiului ABC duse prin vârfurile triunghiului ABC. Demonstraţie. Fie (TA tangenta în A la cercul circumscris triunghiului

ABC. Avem: 1

( ) ( ) ( ' ).2

m TAB m ACB m AC B= = Dar ,c b

ACB AH H≡ deci

,c c b

TAH AH H≡ de unde rezultă .c b

TA H H

22) Triunghiul ortic şi triunghiul determinat de tangentele la cercul circumscris unui triunghi ABC în vârfurile triunghiului ABC sunt omotetice. Demonstraţia rezultă din proprietatea precedentă. 23) Dacă H este ortocentrul triunghiului ABC şi

a b cH H H triunghiul său ortic, atunci

.a b c

HA HH HB HH HC HH⋅ = ⋅ = ⋅

Demonstraţie. Din asemănarea triunghiurilor b

HH A şi a

HH B , respectiv a

HH C cu

cHH A rezultă

a bHA HH HB HH⋅ = ⋅ şi

a cHA HH HC HH⋅ = ⋅ , de unde rezultă concluzia.

24) Dreptele ce unesc mijloacele laturilor triunghiului ortic

a b cH H H corespunzător

unui triunghi ABC cu vârfurile corespunzătoare ale triunghiului ABC, sunt concurente în punctul lui Lemoine al triunghiului ABC. Demonstraţie. Vezi „Punctul lui Lemoine”. 25) Dreptele care unesc vârfurile triunghiului ortic al triunghiului ABC cu proiecţiile ortocentrului triunghiului ABC pe laturile triunghiului ortic sunt concurente în punctul lui Gergonne al triunghiului ortic. Demonstraţie. Vezi „Punctul lui Gergonne”. 26) Dreptele care unesc vârfurile triunghiului ortic al unui triunghi ABC cu punctele de intersecţie dintre laturile triunghiului ortic şi dreptele AO, BO respectiv CO (unde O este centrul cercului circumscris triunghiului ABC) sunt concurente în punctul lui Nagel al triunghiului ortic. Demonstraţie. Vezi „Punctul lui Nagel”.

Page 343: Barbu_TEOREME_FUNDAMENTALE(2008).pdf

350

27) Fie H ortocentrul triunghiului ABC. Triunghiurile ABC, BHC, CHA şi AHB au acelaşi triunghi ortic. Demonstraţie. Fie

a b cH H H triunghiul ortic al triunghiului ABC. Deoarece

, , , , ,a b c a b c

HH BC CH BH BH HC H BC H HB H HC⊥ ⊥ ⊥ ∈ ∈ ∈ rezultă că triunghiul

a b cH H H este triunghiul ortic al triunghiului ABC. Analog, se arată că triunghiurile CHA şi

AHB au acelaşi triunghi ortic. 28) Triunghiurile ortic şi extangenţial corespunzătoare unui triunghi ABC sunt omotetice. Demonstraţie. Vezi „Triunghiul extangenţial”.

29) Razele cercurilor exînscrise corespunzătoare triunghiului ortic a b c

H H H al

triunghiului ascuţitunghic ABC au lungimile: 2 cos sin sin ,a

R A B Cρ =

2 sin cos sinb

R A B Cρ = şi 2 sin sin cos .c

R A B Cρ =

Demonstraţie. Într-un triunghi ABC, 4 sin cos cos2 2 2a

A B Cr R= (vezi „Cercurile

exînscrise”). Deoarece triunghiul ABC este triunghiul ortic al triunghiului ,a b cI I I raza

cercului circumscris triunghiului este egală cu 2

R (vezi „Cercurile exînscrise) .Avem:

4 sin cos cos2 2 2 2

a b c

a

H H HRρ = , de unde

2 2 22 sin cos cos

2 2 2a

A B CR

π π πρ

− − −= ⋅ ⋅ =

2 cos sin sin .R A B C Analog se determină bρ şi .

Observaţie: Dacă triunghiul ABC este obtuzunghic ( ( ) 90 )m BAC > ° atunci

2 cos cos cos , 2 sin sin cos , 2 sin cos sina b c

R A B C R A B C R A B Cρ ρ ρ= ⋅ ⋅ = =

(deoarece ( ) 2 ( ) 180 , ( ) 2 ( ),b a c a b c

m H H H m BAC m H H H m CBA= − ° =

( ) 2 ( )a c b

m H H H m BCA= ).

30) Mediatoarele laturilor BC, CA, AB ale triunghiului ascuţitunghic ABC intersectează laturile ,

b c c aH H H H respectiv

a bH H ale triunghiului ortic

a b cH H H în punctele ', ',A B

respectiv '.C Dreptele ', ', 'a b c

H A H B H C sunt concurente.

Demonstraţie. Fie P şi Q proiecţiile punctelor c

H şi b

H pe

BC (Fig. 361). Avem cos ,c

c

BHBPB

BH BC= = de unde

2cosBP a B= şi 2(1 2cos ) cos22 2a a a

a aM P M B M P B B= − = − =

şi analog obţinem: cos 2 .2a

aM Q C= Din teorema lui Thales

avem: ' cos 2

.' cos 2

b a

c a

A H M Q C

A H M P B= = Analog se arată că

' cos 2

' cos 2c

a

B H A

B H C= şi

' cos 2,

' cos 2a

b

C H B

C H A= deci

A

B C P

H

aH

bH cH

Fig. 361

Q

A'

aM

Page 344: Barbu_TEOREME_FUNDAMENTALE(2008).pdf

351

' ' '1

' ' 'b c a

c a b

A H B H C H

A H B H C H⋅ ⋅ = , iar din reciproca teoremei lui Ceva rezultă că dreptele

', ', 'a b c

H A H B H C sunt concurente.

31) Fie , , ,

a b cO O O O centrele cercurilor circumscrise triunghiurilor ABC, BOC, COA

respectiv AOB. Triunghiul a b cO O O este omotetic cu triunghiul ortic

a b cH H H al

triunghiului ABC. Demonstraţie. Fie , ,

a b cR R R razele cercurilor circumscrise triunghiurilor BOC, COA

respectiv AOB. Deoarece AO BO CO R= = = şi ( ) ( )a

m BOO m A= rezultă

2cosa

RR

A= şi analog ,

2cosb

RR

B=

2cosc

RR

C= (Fig. 362). Deoarece , ,

a b b c c aO O O O O O

sunt mediatoarele segmentelor OC, OA respectiv OB rezultă că O este centrul cercului înscris în triunghiul .

a b cO O O Întrucât unghiurile triunghiurilor

a b cO O O şi

a b cH H H sunt

respectiv congruente şi , ,a a b b

OO H H OO H H c c

OO H H (H fiind centrul cercului

înscris în triunghiul ortic) rezultă că triunghiurile a b cO O O şi

a b cH H H sunt omotetice.

Cum punctele O şi H se corespund în această omotetie rezultă că dreptele , ,

a a b b c cO H O H O H sunt concurente într-un punct P ce aparţine dreptei

HO. Deci, raportul de omotetie este: 2 cos cos cos

4cos cos cos/ 2

h

o

rPH R A B Ck A B C

PO r R= = = = ⋅ ⋅

unde 2 cos cos coshr R A B C= este raza cercului înscris în triunghiul

a b cH H H şi

2o

Rr =

este raza cercului a b cO O O .

A

bO

B C

aO

H

aH

bH

cH

Fig. 362

cO

O

Page 345: Barbu_TEOREME_FUNDAMENTALE(2008).pdf

352

32) Fie H ortocentrul unui triunghi ascuţitunghic ABC , cba HHH ,, picioarele

înălţimilor duse din CBA ,, şi ',',' CBA picioarele perpendicularelor duse din H pe

accb HHHH , , respectiv baHH .

Atunci, [ ] [ ]'''16 CBAABC AA ⋅≥ .

Demonstraţie. Deoarece patrulaterul HBHA c '' este inscriptibil (Fig. 363) ,

rezultă că c cA 'B'H A 'HH≡ ≡

b c cH H A B'H B≡ , deci ABBA ||'' .

Analog, BCCB ||'' şi ACCA ||'' . Fie

21 ,, OOO centrele cercurilor

circumscrise triunghiurilor ABC ,

cba HHH şi ''' CBA iar R, 1R şi 2R

razele acestora. Atunci, 21R

R =

(deoarece C 1 1(O ,R ) este cercul lui Euler al triunghiului ABC ), iar 2R poate avea cel mult

lungimea razei cercului lui Euler al triunghiului cba HHH , adică 42R

R ≤ şi din asemărea

triunghiurilor ''' CBA şi ABC rezultă [ ] [ ]'''16 CBAABC AA ⋅≥ . Egalitatea are loc atunci când

cercul lui Euler al triunghiului cba HHH coincide cu cercul înscris în triunghiul

cba HHH , adică când triunghiul cba HHH este echilateral, adică când triunghiul ABC

este echilateral.

Teorema lui Nagel 33) Fie O centrul cercului circumscris triunghiului ABC. Dreptele AO, BO, CO sunt perpendiculare pe laturile , ,

b c c a a bH H H H H H ale triunghiului ortic.

Demonstraţie. Din ( ) ( ) 90c a

m AH C m AH C= = ° rezultă că patrulaterul a c

ACH H este

inscriptibil deci .c a

BH H ACB≡ Fie D punctul diametral opus lui B în cercul

circumscris triunghiului ABC (Fig. 362). Atunci, ( ) 90 ( )= °− = m ABD m ADB

90 ( ) 90 ( ),° − = ° − c a

m ACB m BH H de unde ( ) ( ) 90 ,c a

m ABD m BH H+ = ° adică

.c a

BO H H⊥ Analog, b c

AO H H⊥ şi .a b

CO H H⊥

Observaţie: Perpendicularele din A, B, C pe , ,

b c c aH H H H respectiv

a bH H sunt concurente

în punctul O, centrul cercului circumscris triunghiului ABC. 34) Fie

a b cH H H triunghiul ortic al triunghiului ABC, ' ( ), " ( ).

a AC a a AB aH pr H H pr H= =

Dreapta ' "a a

H H trece prin mijloacele laturilor a b

H H şi .a c

H H

Demonstraţie. Fie ', ', 'A B C mijloacele laturilor ,b c a c

H H H H respectiv b a

H H (Fig. 364).

Deoarece ' "a a

H H şi b c

H H sunt antiparalele cu BC rezultă ' "a a b c

H H H H , de unde " 'a a b a b

H H H ABC H H C≡ ≡ şi ' " 'b a a a a a

H H H H H H≡ (complemente la unghiuri

congruente). Atunci, ' "a aH H trece prin mijlocul ipotenuzei

a bH H a triunghiului

A

B'

B C

A ' H

aH

bH

cH

Fig. 363

C'

Page 346: Barbu_TEOREME_FUNDAMENTALE(2008).pdf

353

dreptunghic 'a a b

H H H . Cum ' "a a b cH H H H rezultă că ' "

a aH H trece şi prin mijlocul laturii

.a c

H H

Observaţii:

1) Dacă ' " ' "( ), ( ), ( ), ( )b AB b b BC b c BC c c AB c

H pr H H pr H H pr H H pr H= = = = , atunci dreptele ' "b bH H şi ' "

c cH H trec prin mijloacele laturilor triunghiului ortic al triunghiului .ABC

2) Punctele de intersecţie dintre dreptele ' "a aH H , ' "

b bH H şi ' "c cH H determină triunghiul

median al triunghiului ortic al triunghiului ABC. 35) Dacă '

aH şi "

aH , '

bH şi "

bH , '

cH şi "

cH sunt proiecţiile punctelor , ,

a bH H respectiv

cH

pe laturile triunghiului ABC, atunci dreptele ' "a aH H , ' "

b bH H şi ' "c cH H sunt antiparalele

dreptelor BC, CA, respectiv AB. Demonstraţie. Fie

a b cH H H triunghiul ortic

al triunghiului ABC. Din teorema lui Thales

rezultă: '"

( ),= c

c a a

ac a

AH AHHH H H

HHH H

'=b

ab a

AH AH

HHH H '( )

b a aHH H H , de unde

" ',c b

c a b a

AH AH

H H H H= adică ' " .

b c a aH H H H Cum

b cH H este antiparalelă cu BC rezultă că

' "a a

H H este antiparalelă cu BC.

36) Segmentele ' " ' " ' ", ,

a a b b c cH H H H H H sunt

congruente. Demonstraţie.

' " ' " 1 1 1' ' ' ' '

2 2 2a a a a a b b c a cH H H C C B B H H H H H H H p= + + = + + = (unde 'p este

semiperimetrul triunghiului ortic). Analog, ' " ' " ( 2 sin sin sin )b b c c

H H H H R A B C≡ =

(cf. th. (9)). 37) Fie

a b cH H H triunghiul ortic corespunzător triunghiului ABC, 'BB bisectoarea

interioară a unghiului ABC, ' ( )B AC∈ şi "B piciorul bisectoarei unghiului .b c

AH H Dreptele ' "B B şi AB sunt perpendiculare. Demonstraţie. Patrulaterul

b cBCH H fiind inscriptibil rezultă ,

c bH H A ABC≡ deci

1" ' ( ) ,

2bB H A ABB m ABC

≡ =

adică patrulaterul " 'b

BB H B este inscriptibil, de

unde ( " ') ( ') 90 .b

m BB B m BH B= = °

A

B C

H B’

C’

aH

bH

cH

"aH

D

'aH

Fig. 364

Page 347: Barbu_TEOREME_FUNDAMENTALE(2008).pdf

354

38) Fie a b c

H H H şi a b cC C C triunghiurile ortic, respectiv de contact ale triunghiului

ABC. Triunghiul a b cC C C şi triunghiul având vârfurile în centrele cercurilor înscrise în

triunghiurile ,b c

AH H ,a c

BH Ha b

CH H au aceeaşi dreaptă a lui Euler. Demonstraţie. Vezi „Cercul lui Euler”. 39) Fie ", ", "A B C punctele de intersecţie dintre înălţimile triunghiului ABC cu laturile respective ale triunghiului ortic. Triunghiurile ABC şi " " "A B C sunt ortologice. Demonstraţia este evidentă deoarece " , "

a bA H BC B H CA⊥ ⊥ şi "

cC H AB⊥ , iar

" " " a b c

A H B H C H H∩ ∩ = , al doilea centru de ortologie fiind centrul cercului lui Euler

al triunghiului ABC. 40) Perpendicularele coborâte din mijloacele laturilor triunghiului ortic ale unui triunghi ABC pe laturile respective ale triunghiului ABC sunt concurente. Demonstraţia rezultă din teorema lui Döttl.

III.2. Triunghiul median

„Geometria e liniştea întâmplării.” – Nichita Stănescu175

Fie triunghiul ABC şi , ,a b c

M M M mijloacele laturilor BC, AC, AB. Triunghiul a b c

M M M

se numeşte triunghi median (sau auxiliar) (Fig. 365). 1) Triunghiul median

a b cM M M este triunghiul

cevian al triunghiului ABC corespunzător centrului de greutate G al triunghiului ABC . Demonstraţia este evidentă deoarece medianele

,a

AM ,b

BM c

CM sunt concurente în G.

2) Triunghiul median este triunghiul podar al centrului cercului circumscris triunghiului ABC . Demonstraţia. Mediatoarele laturilor triunghiului ABC sunt concurente în O – centrul cercului circumscris triunghiului ABC. 3) Triunghiul median

a b cM M M al triunghiului

ABC este asemenea cu triunghiul ABC . Demonstraţie. Laturile triunghiului median sunt linii mijlocii în triunghiul ABC, deci

1

2a b b c c a

M M M M M M

AB BC CA= = = , de unde rezultă că triunghiurile

a b cM M M şi ABC sunt

asemenea.

175 Nichita Stănescu (1933 – 1983) – eseist, poet român, ales postum membru al Academiei Române

A

B C aM

bM cM

G

Fig. 365

Page 348: Barbu_TEOREME_FUNDAMENTALE(2008).pdf

355

Consecinţe:

4) Laturile triunghiului median au lungimile: ' , ' , '2 2 2

a b ca b c= = = , unde am notat cu a,

b, c lungimile laturilor BC, CA, respectiv AB.

5) Semiperimetrul triunghiului median este: ' ' '

' .2 4 2

a b c a b c pp

+ + + += = =

6) Măsurile unghiurilor triunghiului median sunt egale cu: , ,

a b c b c aM M M ABC M M M BCA≡ ≡ .

c a bM M M CAB≡

7) Coordonatele baricentrice absolute ale vârfurilor triunghiului median sunt: 1 1

0, , ,2 2a

M

1 1,0,

2 2bM

şi 1 1, ,0 .

2 2cM

8) Triunghiul ABC şi triunghiul median corespunzător au acelaşi centru de greutate. 9) Aria triunghiului median este egală cu o pătrime din aria triunghiului ABC.

Demonstraţie. Avem: [ ] [ ]

1

4a b cM M M ABCA A= , deoarece a b c c b c a b a

M MM AMM M BM MM C≡ ≡ ≡ .

Cercul înscris în triunghiul median se numeşte cercul Spieker, iar centrul acestui cerc se numeşte punctul lui Spieker.

10) Raza cercului Spieker este jumătate din raza cercului înscris în triunghiul ABC . Demonstraţie. Vezi „Punctul lui Spieker”. Cercul circumscris triunghiului median se numeşte cerc medial (sau cercul celor nouă puncte). 11) Orice triunghi omotetic cu triunghiul median al unui triunghi ABC în raport cu punctul O, centrul cercului circumscris triunghiului ABC, este omologic cu triunghiul ABC. Demonstraţie. Fie ' ' 'A B C triunghiul obţinut din triunghiul median

a b cM M M prin omotetia

de centru O şi raport k. Fie 'T OH AA= ∩

(Fig. 366). Din asemănarea triunghiurilor AHT

şi 'TOA rezultă: ' '= = =

TA TO OA

TA TH AH

.2 2

=a

a

kOM k

OM Analog, se arată că 'BB şi 'CC

trec tot prin punctul T care este centrul de omologie.

Observaţie: Centrul de omologie între triunghiul omotetic triunghiului median al triunghiului ABC şi triunghiul ABC este situat pe dreapta lui Euler a triunghiului ABC.

A

B C

B' C'

O H

T

A'

aM

bM cM

Fig. 366

Page 349: Barbu_TEOREME_FUNDAMENTALE(2008).pdf

356

12) Proiecţiile unui vârf al unui triunghi ABC pe bisectoarele interioare şi exterioare ale unghiurilor din celelalte două vârfuri aparţin dreptelor ce trec prin vârfurile triunghiului median

a b cM M M .

Demonstraţie. Fie M , 'M şi N , 'N proiecţiile vârfurilor B şi C pe bisectoarea interioară, respectiv exterioară a unghiului A. Deoarece patrulaterul 'BMAM este dreptunghi, rezultă că '.

cM MM∈ Fie 'P BM AC= ∩ .

Triunghiul APB este isoscel deoarece 'AM este bisectoare şi înălţime, deci 'M este mijlocul segmentului BP (Fig. 367). Atunci

'c

M M este linie mijlocie în triunghiul APB ,

deci 'c

M M AP , adică 'MM AC , de unde

rezultă că punctele M şi 'M aparţin dreptei

a cM M . Analog, se arată că punctele N şi 'N

aparţin dreptei a b

M M .

13) Triunghiul median

a b cM M M şi triunghiul

tangenţial A B CT T T al unui triunghi ABC sunt

omologice. Demonstraţie. Vezi „Triunghiul tangenţial”. 14) Triunghiurile median şi antisuplementar ale unui triunghi ABC sunt omologice, centrul de omologie fiind punctul lui Lemoine al triunghiului antisuplementar. Demonstraţie: vezi „Triunghiul antisuplementar”. 15) Triunghiul median al triunghiului cotangentic al unui triunghi ABC este omologic cu triunghiul ABC . Demonstraţie. Vezi „Triunghiul cotangentic”. 16) Fie

a b cM M M triunghiul median al triunghiului ABC şi

aI ,

bI ,

cI centrele

cercurilor exînscrise corespunzătoare triunghiului ABC . Dreptele a aI M ,

b bI M şi

c cI M

sunt concurente. Demonstraţie. Triunghiul ABC este triunghiul ortic al triunghiului

a b cI I I ; dreptele care

unesc mijloacele laturilor triunghiului ortic corespunzător unui triunghi XYZ cu vârfurile corespunzătoare triunghiului XYZ sunt concurente în punctul lui Lemoine al triunghiului XYZ (vezi „Triunghiul ortic”), deci dreptele

a aI M ,

b bI M ,

c cI M sunt concurente în

punctul lui Lemoine al triunghiului a b cI I I .

17) Consecinţă: Triunghiurile median şi antisuplementar ale unui triunghi ABC sunt omologice. 18) Bisectoarele interioare ale unui triunghi ABC intersectează laturile necorespunzătoare ale triunghiului median în şase puncte care determină trei drepte ce trec prin punctele de contact determinate de cercul înscris in triunghiul ABC şi laturile triunghiului ABC .

aM

Fig. 367

A

B C

bM cM

M

N

M'

N'

P

Page 350: Barbu_TEOREME_FUNDAMENTALE(2008).pdf

357

Demonstraţie.Fie 1 2 2 2 , = =I Ia c

A A B BC A M M AI ,

2 b c

B M M BI= I , 1 2 2 B AB AC= I , 2cM B BC ,

2 1 2 2( ) ( ) ( )c c

m M B B m A BB m M BB= = , deci

triunghiul 2cBM B este isoscel, deci

2 2 .2c c c

cB M BM A M= = = Mai mult 1 2b c

M B A M ,

de unde rezultă că triunghiul 1 2bM B B este isoscel,

deci 1 2 2 2b b

c aM B M B= = − (Fig. 368). Atunci,

1

1( )

2BC b c a p a= + − = − , deci 1B este un punct de

contact al cercului înscris în triunghiul ABC cu latura AC . 19) Fie O centrul cercului circumscris triunghiului ABC, ', ', 'A B C mijloacele segmentelor AO,BO, respectiv CO şi

a b cM M M triunghiul median al triunghiului ABC.

Dreptele 'a

A M , 'b

B M şi 'c

C M sunt concurente în centrul cercului lui Euler al

triunghiului median. Demonstraţie. Vezi „Cercul lui Euler”. 20) Fie

a b cM M M triunghiul median al triunghiului ABC. Pentru orice punct M din

planul triunghiului ABC este adevărată relaţia: a b cMM MM MM MA MB MC+ + = + +uuuuur uuuuur uuuuur uuur uuur uuuur

. Demonstraţie. Teorema medianei scrisă sub formă vectorială aplicată în triunghiul

MBC dă: 1( ).

2aMM MB MC= +uuuuur uuur uuuur

Analog se obţin egalităţile: 1( )

2bMM MC MA= +uuuuur uuuur uuur

şi

1( )

2cMM MC MA= +uuuuur uuuur uuur

. Sumând aceste egalităţi obţinem:

a b cMM MM MM MA MB MC+ + = + +uuuuur uuuuur uuuuur uuur uuur uuuur

.

21) Coordonatele baricentrice ale vârfurilor triunghiului median sunt:

1 1 1 1 1 10, , , ,0, , , ,0 .

2 2 2 2 2 2a b cM M M

Demonstraţie. Din egalitatea 1 1

2 2aMM MB MC= ⋅ + ⋅uuuuur uuur uuuur

rezultă 1 1

0, , .2 2a

M

Analog pentru

celelalte vârfuri.

A

B C aM

bM cM

I

Fig. 368

1A

2A

1B 2B

Page 351: Barbu_TEOREME_FUNDAMENTALE(2008).pdf

358

22) Triunghiul ortic al triunghiul median corespunzător unui triunghi ABC este omologic cu triunghiul ABC. Demonstraţie. Fie ', ', 'A B C picioarele înălţimilor

triunghiului median a b c

M M M (Fig. 369) şi

" ' , " ' , " ' .A AA BC B BB CA C CC AB= ∩ = ∩ = ∩

Cum b c

M M BC rezultă că

' cos" cos.

" ' cos cosc c a

b b a

M A M M CBA b C

CA M A M M B c B= = = Analog,

" cos

" cos

CB c A

AB a C= şi

" cos

" cos

AC a B

BC b A= , de unde rezultă că

" " "1

" " "

BA CB AC

CA AB BC⋅ ⋅ = şi din reciproca teoremai lui

Ceva rezultă că dreptele ', ', 'AA BB CC sunt concurente.

III.3. Triunghiul de contact

„O cameră fară cărţi este un trup fără suflet.” - Pitagora176

Fie , ,a b cC C C punctele de tangenţă ale cercului înscris în triunghiul ABC cu laturile

triunghiului. Triunghiul a b cC C C se numeşte triunghiul de contact al triunghiului ABC

(sau triunghiul Gergonne).

176 Pitagora (580-500 î.e.n.) – matematician grec

A

B C aM

bM cM

G

Fig. 369

A'

B'

A"

B"

aC

bC cC

A

B C

I

y z

z

x x

y

Γ

Fig. 370

Page 352: Barbu_TEOREME_FUNDAMENTALE(2008).pdf

359

1) Triunghiul de contact a b cC C C este triunghiul podar corespunzător centrului cercului

înscris (I) în triunghiul ABC, în raport cu triunghiul ABC.

2) Unghiurile triunghiului de contact cba CCC corespunzător triunghiului ABC au

măsurile egale cu: 190 ( )

2° − m A , 1

90 ( )2

° − m B , respectiv 190 ( ).

2° − m C

Demonstraţie. Fie I centrul cercului înscris în triunghiul ABC . Deoarece patrulaterele

ca ICBC , abICCC sunt inscriptibile rezultă 1( ) ( ) ( )

2= =a b cm IC C m IBC m B şi

1( ) ( ) ( )

2= =a b bm IC C m ICC m C . Atunci, ( ) ( ) ( )= + =

c a b c a b am C C C m C C I m C C I

1 1 1 1( ) ( ) [180 ( )] 90 ( )

2 2 2 2+ = °− = ° −m B m C m A m A . Analog se arată că

1( ) 90 ( )

2= °−a b cm C C C m B şi 1

( ) 90 ( )2

= ° −a c bm C C C m C .

3) Lungimile laturilor triunghiului de contact

a b cC C C sunt egale cu:

' ( )sin2

= − + +A

a a b c , ' ( )sin2

= − +B

b a b c , ' ( )sin .2

= + −C

c a b c

Demonstraţie. În triunghiul isoscel b c

AC C avem: '/ 2

sin2

A a

p a=

−, unde p reprezintă

semiperimetrul triunghiului ABC, deci ' ( )sin2

= − + +A

a a b c . Analog se determină

lungimile celorlalte două laturi ale triunghiului a b cC C C .

4) Lungimile laturilor triunghiului de contact a b cC C C sunt egale cu: 2 cos ,

2b c

AC C r=

2 cos , 2 cos .2 2c a a b

B CC C r C C r= =

Demonstraţie. Din teorema sinusurilor aplicată în triunghiul de contact rezultă

) ) )2

sin(90 /2 sin(90 /2 sin(90 /2b c c a a bCC CC C C

rA B C

= = =°− °− °−

, de unde rezultă concluzia.

5) Consecinţă: 2

2 2 2 2 (4 )a b b c b c

r R rC C C C C C

R

++ + = .

Demonstraţie. 2

2 2 2 2 2 2 2 2 (4 )4 cos cos cos

2 2 2a b b c b c

A B C r R rC C C C C C r

R

+ + + = + + =

.

Page 353: Barbu_TEOREME_FUNDAMENTALE(2008).pdf

360

6) Segmentele determinate de punctele de contact ale cercului înscris pe laturile

triunghiului ABC au lungimile , ,p a p b− − respectiv p c− , unde .2

a b cp

+ +=

Demonstraţie.

Fie , , .b c a c a b

AC AC x BC BC y CC CC z= = = = = = Avem: 2( ) 2+ + = + + =x y z a b c p de

unde x y z p+ + = , deci , ,x p a y p b z p c= − = − = − .

7) Triunghiul de contact al unui triunghi ABC neisoscel şi triunghiul ABC sunt omologice, centrul de omologie fiind punctul lui Gergonne al triunghiului ABC . Demonstraţie: vezi „Punctul lui Gergonne”. 8) Punctul lui Lemoine al triunghiului de contact

a b cC C C al triunghiului ABC coincide

cu punctul lui Gergonne al triunghiului ABC. Demonstraţie. Vezi „Punctul lui Gergonne”. 9) Fie

a b cC C C triunghiul de contact al triunghiului ABC . Atunci:

[ ] sin sin sin( )a b cC C C

abcA B C

R a b cA = ⋅ ⋅ ⋅

+ +.

Demonstraţie. Avem [ ]

[ ]

2c bAC C c

ABC

AC

bc

A

A= ,

[ ]

[ ]

2( )a cBC C c

ABC

c AC

ac

A

A

−= ,

[ ]

[ ]

2( )a bCC C c

ABC

c AC

ab

A

A

−= . Cu

relaţia [ ] [ ] [ ] [ ] [ ]a b c b c a c a bABC C C C AC C BC C CC CA A A A A= + + + şi ţinând cont că

2

c

rAC

Atg

= sau ( )( )

( )

c

rAC

p b p c

p p a

=− −

rezultă [ ][ ]

22( )

2 2a b c

ABC

CCC

pr

pR pR

AA = = sau

[ ]sin sin sin

( )a b cC C C

abcA B C

R a b cA = ⋅ ⋅ ⋅

+ +.

Observaţie: Aria triunghiului de contact a b cC C C este maximă când triunghiul ABC este

echilateral deoarece [ ][ ]

2 2

2 2 2 4 4a b c

ABC

C C C

pr pr pr

R r

AA = ≤ = =

⋅ (deoarece 2R r≥ ). Cum relaţia

2R r= are loc doar în triunghiul echilateral, atunci afirmaţia este demonstrată.

10) Aria triunghiului de contact a b cC C C este egală cu: [ ]

2[ ]

[ ] 2 2a b c

ABC

C C C ABC

A rA A

R p R= = ⋅

⋅.

Demonstraţia rezultă din proprietatea precedentă.

Page 354: Barbu_TEOREME_FUNDAMENTALE(2008).pdf

361

11) Perpendicularele duse din mijloacele laturilor triunghiului de contact a b cC C C pe

laturile opuse ale triunghiului ABC sunt concurente. Demonstraţie. Fie W centrul de greutate al triunghiului

a b cC C C

şi ', ', 'A B C mijloacele laturilor

triunghiului .a b cC C C Punctele

, ,a b cC C C se obţin din punctele

', ', 'A B C printr-o omotetie de centru W şi raport (-2)

( 2 'a

WC WA= −uuuur uuuur

). Printr-o

omotetie o dreaptă se transformă într-o dreaptă paralelă cu cea dată. Atunci, dreapta

aC I se

transformă într-o dreaptă ad ce

trece prin A' şi este paralelă cu

aC I ,deci

ad BC⊥ . Analog,

dreptele b

IC şi c

IC se transformă în dreptele ,b cd d paralele cu

bIC , respectiv

cIC şi

' , ' .b c

B d C d∈ ∈ Cum , ,a b c

IC IC IC sunt concurente în I, rezultă că dreptele , ,a b cd d d sunt

concurente şi fie S punctul de concurenţă al lor. Evident, punctul I se transformă prin omotetia de centru W şi raport (-2) în punctul S, deci 2 .SI WI= 12) Fie , ,

a b cM M M mijloacele laturilor BC, AC respectiv AB ale triunghiului ABC şi

a b cC C C triunghiul de contact al triunghiului .ABC Perpendicularele din punctele

, ,a b c

M M M pe laturile ,b c c aC C C C respectiv

a bC C sunt concurente. Demonstraţie. Fie P, Q, R picioarele perpendicularelor duse din ,

a bM M

respectiv c

M pe laturile triunghiului

.a b cC C C Deoarece AI este bisectoarea

BAC şi c bC A C A= rezultă

b cAI C C⊥ .

Cum şi a b c

M P C C⊥ rezultă că .a

AI M P

Patrulaterul a b c

M M AM este

paralelogram, deci c a b

BAC M M M=

şi cum AI este bisectoarea unghiului A rezultă că

aM P este bisectoarea unghiului

.c a b

M M M Analog, se demonstrează că

bM Q şi

cM R sunt bisectoarele

unghiurilor a b c

M M M , respectiv .b c a

M M M Cum bisectoarele interioare ale

unghiurilor unui triunghi sunt concurente, rezultă că dreptele ,a b

M P M Q şi c

M R sunt

concurente.

aC

bC cC

W

A’

B’

A

B C

I C’

S

Fig. 371

A

B C

P

T

I R

Q

aM

bM cM

aC

bC

cC

Fig. 372

Page 355: Barbu_TEOREME_FUNDAMENTALE(2008).pdf

362

13) Triunghiurile de contact a b cC C C şi median

a b cM M M corespunzătoare unui

triunghi ABC sunt ortologice. Demonstraţia rezultă din proprietatea de mai sus. 14) Triunghiul de contact al triunghiului ABC şi primul triunghi Sharygin al triunghiului ABC sunt omotetice. Demonstraţie. Vezi „Triunghiul Sharygin”. 15) Triunghiurile de contact

a b cC C C al unui triunghi ABC este omotetic cu triunghiul

antisuplementar a b cI I I al triunghiului ABC.

Demonstraţie. Vezi „Triunghiul antisuplementar”. 16) Triunghiul de contact

a b cC C C şi triunghiul antisuplementar

a b cI I I corespunzătoare

unui triunghi ABC au aceeaşi dreaptă a lui Euler OI. Demonstraţie. Deoarece I este ortocentrul triunghiului

a b cI I I şi O este centrul cercului

medial al triunghiului a b cI I I (vezi „Triunghiul antisuplementar”) rezultă că dreapta OI este

dreapta lui Euler a triunghiului a b cI I I . Deoarece triunghiurile

a b cI I I şi

a b cC C C sunt

omotetice (cf. th. (13)), rezultă că omologul lui I – centrul cercului circumscris triunghiului

a b cC C C - este punctul 'O – centrul cercului circumscris triunghiului

a b cI I I şi evident 'O

este punct pe dreapta lui Euler (OI) a triunghiului a b cI I I , deci centrul omotetiei (Q)

aparţine dreptei OI. Cum raza cercului circumscris triunghiului ortic unui triunghi are

lungimea jumătate din raza cercului triunghiului de referinţă rezultă că '

2

RR = adică

' 2R R= (unde 'R este raza cercului circumscris triunghiului a b cI I I , triunghiul ABC fiind

triunghiul ortic al triunghiului a b cI I I ). Astfel,

' 2QO R

QI r= (adică raportul de omotetie este

egal cu 2R

r), iar punctul I – ortocentrul triunghiului

a b cI I I - prin omotetia de centru Q şi

raport 2R

r se transformă în ortocentrul triunghiului

a b cC C C , care evident aparţine dreptei

QI adică dreapta OI, adică OI este dreapta lui Euler a triunghiului a b cC C C .

17) Triunghiul de contact

a b cC C C şi triunghiul circumpedal al centrului cercului înscris

în triunghiul ABC sunt omotetice, centrul de omotetie fiind pe drapta OI.

Demonstraţie. Fie ", ", "A B C mijloacele arcelor , ,BC CA

respectiv AB (Fig. 373). Cum " ,a

OA IC

" , "b c

OB IC OC IC şi " " " a b c

A C B C C C I∩ ∩ = rezultă că

triunghiurile " " "A B C şi a b cC C C sunt omotetice, raportul de

omotetie fiind R

r, centrul omotetiei aparţinând liniei centrelor

celor două cercuri, adică dreptei OI. Dacă punctul M este centrul

A

C

A"

B"

M O

aC

I

Fig. 373

B

C"

Page 356: Barbu_TEOREME_FUNDAMENTALE(2008).pdf

363

omotetiei considerate, atunci 2 2r R Rr

MIR r

−=

− şi

2 2R R RrMO

R r

−=

− (vezi „Triunghiul

circumpedal”). 18) Dreapta lui Simson a punctului lui Feuerbach ( )ϕ al triunghiului ABC , în raport

cu triunghiul de contact cba CCC al triunghiului ABC este paralelă cu dreapta OI Demonstraţie. Vezi „Dreapta lui Simson”. 19) Fie cba CCC triunghiul de contact al triunghiului ABC , I centrul cercului înscris

în triunghiul ABC şi cba MMM triunghiul median al triunghiului ABC . Dreptele

ICb , caCC şi bBM sunt concurente.

Demonstraţie. Fie bH piciorul înălţimii din B a

triunghiului ABC şi = Ib b

M BM C I . Din

bb BHMC || rezultă bb

bb

b MC

CH

MM

BM= . Dar

AcapCH bb cos⋅−−= , ( )2

= − −b b

bC M p a , deci

b

cba

MM

BM

b

+−= . Fie '= I

a b bM CC BM . Atunci,

ACBM

MMCM

BC

ACAM

BC

CC bb

c

cb

a

a ⋅=⋅+⋅'

(vezi „Relaţia lui Van-Aubel” ), adică

bBM

MMb

bp

apb

bp

cp b ⋅=⋅−

−+⋅

'

'

22, de unde

b

cba

MM

BM

b

+−=

'

', deci 'MM ≡ .

20) Triunghiul ortic al triunghiului de contact al unui triunghi ABC este omotetic cu triunghiul ABC. Demonstraţie. Fie 1H ortocentrul triunghiului de contact

şi 1 1 1, ,a b cC A C B C C înălţimile triunghiului de contact.

Dar 1 1

1( ) ( ) 90 ( ) ( )

2c b b am ACC m AIC m A m BC C= = °− = ,

iar 1 1 1 1( ) ( )c a

m C A B m BC C= (deoarece patrulaterul

1 1 a bA B C C este inscriptibil, deci 1 1c b c

AC C C A B≡ ,

adică 1 1AB A B . Analog se arată că

1 1 1 1,BC BC AC AC şi cum triunghiul a b cC C C este

ascuţitunghic rezultă că triunghiurile 1 1 1,ABC A B C sunt omotetice.

21) Fie

a b cC C C triunghiul de contact al unui triunghi .ABC Dreapta lui Euler a

triunghiului a b cC C C trece prin centrul cercului lui Euler al triunghiului .ABC

Demonstraţie. Vezi „Cercul lui Euler. Dreapta lui Euler”.

A

B C

M

bC

aC

I cC

Fig. 374

bH

bM

A

B C

1H

bC

aC

I cC

Fig. 375

1B

1A

1C

Page 357: Barbu_TEOREME_FUNDAMENTALE(2008).pdf

364

III.4. Triunghiul extangenţial

„Omul pentru care faptul că 2 2 4⋅ = e de la sine înţeles nu va deveni niciodată mare matematician.” - Bertolt Brecht177

Fie triunghiul ABC şi , ,a b cI I I centrele cercurilor A -exînscris, B -exînscris, respectiv

C - exînscris corespunzǎtoare triunghiului ABC . Tangentele comune cercurilor exînscrise

(diferite de dreptele ce conţin laturile triunghiului ABC ) determinǎ triunghiul a b cT T T numit

triunghiul extangenţial al triunghiului ABC (Fig. 376). Cercul circumscris triunghiului extangenţial se numeşte cercul extangenţial.

177 Bertolt Brecht (1898-1956) – poet german

A

B

C

Ia

Ib

Ic

Fig. 376

bH

aH

cH

A'

A"

TI

B'

C'

aT

bT

cT

Page 358: Barbu_TEOREME_FUNDAMENTALE(2008).pdf

365

1) Triunghiul extangenţial a b cT T T corespunzǎtor triunghiului ABC este omotetic cu

triunghiul ortic a b c

H H H al triunghiului ABC .

Demonstraţie. Fie '= ∩b c

C BA T T şi '= ∩b c

B AC T T . Din congruenţa triunghiurilor

ABC şi ' ' 'A B C rezultǎ cǎ ' '.ACB AC B≡ Dar a b

H H este antiparalelǎ laturii

,BC deci ,≡ c b

AH H ACB de unde rezultǎ ' 'c b

B C A AH H≡ şi ,' 'c b

B C H H adicǎ

.c b c bT T H H Analog se aratǎ cǎ

c a c aT T H H şi .

a b a bT T H H

Avem ( ) 180 2 ( ).= ° − a b c

m H H H m A Fie '= ∩a c

A BC T T şi . "= ∩a b

A BC T T Atunci,

' ' ( ( ))≡ = c a a a

H H A H A T m BAC şi '' ( ( ))≡ = b a a a

H H C H A T m BAC (alterne

interne), de unde rezultǎ ( ) 180 2 ( )= °− c a b

m TT T m A , deci c a b c a b

H H H T T T≡ . Analog,

c b a c b aH H H T T T≡ , unde rezultǎ cǎ triunghiurile

a b cT T T şi

a b cH H H sunt asemenea şi

deoarece au şi laturile respective paralele rezultǎ cǎ sunt omotetice. Observaţie: Centrul de omotetie dintre triunghiurile

a b cT T T şi

a b cH H H se numeşte

punctul lui Clawson. 2) Triunghiurile extangenţial

a b cT T T şi antisuplementar

a b cI I I corespunzătoare unui

triunghi ABC sunt omologice. Demonstraţia este evidentă deoarece dreptele , ,

a a b b c cI T I T I T sunt concurente în centrul

cercului înscris în triunghiul a b cT T T , fiind bisectoare în triunghiul extangenţial.

3) Centrul cercului înscris în triunghiul extangenţial coincide cu centrul cercului triunghiului antisuplementar. Demonstraţie. Fie

TI centrul cercului înscris în triunghiul

a b cT T T ; conform proprietăţii

precedente rezultă: .= ∩ ∩T a a b b c cI I T I T I T Avem:

( ) 180 [ ( ) ( )]= ° − + = a T c c a T a c T

m T I T m T T I m T T I

180 [(90 ( )) (90 ( ))] 180 ( ) 2 ( ).° − ° − + ° − = ° − = a b c

m A m C m B m I I I Analog se arată că

( ) 2 ( )= a T b a b c

m T I T m I I I şi ( ) 2 ( )= c T b b a c

m T I T m I I I , deci TI este centrul cercului

circumscris triunghiului antisuplementar a b cI I I .

Observaţie: Cum

TI este centrul cercului circumscris triunghiului

a b cI I I rezultă că

≡ ≡T a T b T cI I I I I I .

4) Raza cercului înscris în triunghiul extangenţial corespunzător unui triunghi ABC, are lungimea egală cu 2 ,+R r unde R şi r sunt razele cercurilor circumscris, respectiv înscris în triunghiul ABC.

Demonstraţie. Fie ' = ∩b c

B AB T T şi ' = ∩b c

C AC T T . Deoarece triunghiurile ABC şi

' 'AB C sunt congruente (vezi „Cercuri exînscrise”) rezultă că raza cercului înscris în triunghiul ' 'AB C este egală cu r. Fie 'I centrul cercului înscris în triunghiul ' 'AB C . Atunci, distanţele de la A şi 'I la ' 'B C sunt egale cu

ah (înălţimea din vârful A a

Page 359: Barbu_TEOREME_FUNDAMENTALE(2008).pdf

366

triunghiului ABC) şi r. Cum A este mijlocul segmentului 'II , atunci distanţa de la I la ' 'B C este egală cu 2 .−

ah r Fie

TI simetricul lui I faţă de O – centrul cercului circumscris

triunghiului ABC. Distanţele de la I şi O la b cT T sunt egale cu 2 −

ah r , respective +

ah R

(deoarece =AO R ), deci distanţa de la TI la

b cT T este egală cu

2( ) (2 ) 2+ − − = +a a

R h h r R r (unde am aplicat teorema liniei mijlocii într-un trapez).

Analog, se arată că distanţele de la punctul TI la

a cT T şi sunt egale tot cu 2 ,+R r deci

TI

este centrul cercului înscris în triunghiul extangenţial. Observaţie: Deoarece 4 2+ = + + +

a b cR r r r r r rezultă că raza cercului înscris în triunghiul

extangenţial este egală cu 1

2 ( ).2

= + = + + +T a b cr R r r r r r

5) Centrul cercului înscris în triunghiul extangenţial al unui triunghi ABC este simetricul centrului cercului înscris în triunghiul ABC, faţă de centrul cercului circumscris triunghiului ABC. Demonstraţie. Proprietatea este o consecinţă a aplicaţiei precedente. 6) Triunghiurile extangenţial şi tangenţial corespunzătoare unui triunghi ABC sunt omotetice. Demonstraţie. Deoarece triunghiurile extangengenţial

a b cT T T şi tangenţial

A B CT T T sunt

omotetice cu triunghiul ortic a b c

H H H al triunghiului ABC, iar relaţia de omotetie fiind

tranzitivă, rezultă că şi triunghiurile a b cT T T şi

A B CT T T sunt omotetice.

Observaţie: Din omotetia triunghiurilor a b cT T T şi

A B CT T T rezultă că cercurile înscrise în

aceste triunghiuri sunt omotetice, deci raportul de omotetie este egal cu 2

,R r

R

+ unde

2R r+ şi R sunt lungimile razelor cercurilor înscrise în triunghiurile a b cT T T respectiv

A B CT T T .

A

B C

bT

cT

I O

I ' r

TI

C'

B'

Fig. 377

Page 360: Barbu_TEOREME_FUNDAMENTALE(2008).pdf

367

7) Triunghiul neisoscel ABC este omologic cu triunghiul său extangenţial. Demonstraţie. Fie 1 1 1 , , .= ∩ = ∩ = ∩

b c a c a bA T T BC B T T AC C T T AB Din teorema

bisectoarei exterioare rezultă: 1 1

1 1

,A B BCc a

AC b B A c= = şi 1

1

.C A b

C B a= Atunci, 1 1 1

1 1 1

1A B BC C A

AC B A C B⋅ ⋅ =

şi din reciproca teoremei lui Menelaus rezultă că punctele 1 1 1, ,A B C sunt coliniare, iar din

reciproca teoremei lui Desargues rezultă că triunghiurile ABC şi a b cT T T sunt omologice,

dreapta de omologie fiind axa antiortică a triunghiului ABC.

III.5. Triunghiul cotangentic

„Adevărul matematic indeferent unde, la Paris sau la Toulouse, este unul şi acelaşi.” – Blaise Pascal178

Triunghiul a b cτ τ τ care are ca vârfuri punctele de tangenţă dintre laturile BC, CA şi AB ale

unui triunghi ABC cu cercurile exînscrise corespunzătoare triunghiului ABC, se numeşte triunghiul cotangentic corespunzător triunghiului ABC.

178 Blaise Pascal (1623 – 1662) – matematician, fizician şi filosof francez, contribuţii în geometria proiectivă,

algebră şi teoria probabilităţilor

A

B

C

Ia

Ib

Ic

N

Fig. 378

bτ cτ

Page 361: Barbu_TEOREME_FUNDAMENTALE(2008).pdf

368

1) Dreptele , ,a b cA B Cτ τ τ sunt concurente în punctul lui Nagel al triunghiului ABC. Demonstraţie. Vezi „Punctul lui Nagel”. 2) Triunghiul cotangentic a b cτ τ τ este triunghiul cevian al punctului lui Nagel în raport cu triunghiul ABC. 3) Triunghiul cotangentic a b cτ τ τ este triunghiul podar al punctului lui Bevan al triunghiului ABC. Demonstraţie. Vezi „Punctul lui Bevan”. 4) Laturile triunghiului cotangentic a b cτ τ τ al triunghiului ABC au lungimile egale cu:

2 2( ) ( ) 2( )( ) cosb c

p b p c p b p c Aττ = − + − − − − ,

2 2( ) ( ) 2( )( ) cosc a

p c p a p c p a Bττ = − + − − − − ,

2 2( ) ( ) 2( )( ) cosa b

p a p b p a p b Cττ = − + − − − − .

Demonstraţie. În triunghiul b c

A ττ , din teorema cosinusului rezultă

2 2 22 cosA A A A A

b c b c b cτ τ ττ τ τ= + − ⋅ ⋅ , adică 2 2( ) ( ) 2( )( )cos

b cp b p c p b p c Aττ = − + − − − − .

Analog se determină lungimile laturilor b aττ şi a cττ .

5) Aria triunghiului cotangentic este egală cu 2[ ][ ]

1

2ABCa b c

A ApR

τ τ τ = ⋅ , unde R este

raza cercului circumscris triunghiului ABC. Demonstraţie.

Avem [ ]

[ ]

s i n ( ) ( )

s i nb cA

A B C

c bA A A A p b p c

A A B A C A b c

τ τ τ τ⋅ ⋅ − −= =

⋅ ⋅ şi analoagele. Dar

[ ] [ ] [ ] [ ] [ ]A B C A B Cb c a c a b a b c

A A A A Aτ τ τ τ τ τ τ τ τ= + + + adică,

[ ] ( )( ) ( )( ) ( )( )1

[ ]

− − − − − −= − − −a b c

Ap b p c p a p c p a p b

A bc ac abABC

τ τ τ şi de aici rezultă

[ ] [ ]( )( )( )

4ABC

a b c

a b c a b c a b cA A

abcτ τ τ

+ − − + − + += ⋅ =

2[ ]

[ ]

22

2

ABC

ABC

Ar pA

abc pR⋅ = .

Observaţie: Aria triunghiului cotangentic este egală cu aria triunghiului de contact.

Page 362: Barbu_TEOREME_FUNDAMENTALE(2008).pdf

369

6) Aria triunghiului cotangentic este maximă când triunghiul ABC este echilateral.

Demonstraţie. Avem [ ]

[ ]

2 2

2 2 2 4 4

ABC

a b c

Apr pr prA

R rτ τ τ = ≤ = =

⋅ (deoarece 2R r≥ ). Cum

relaţia 2R r= au loc doar în triunghiul echilateral rezultă că aria triunghiului cotangentic este maximă atunci când triunghiul ABC este echilateral. 7) Triunghiul neisoscel ABC şi triunghiul său cotangentic a b cτ τ τ sunt omologice, centrul de omologie fiind punctul lui Nagel al triunghiului ABC. Demonstraţie. Deoarece a b cA B C Nτ τ τ =I I ,unde N este punctul lui Nagel al

triunghiului ABC rezultă că triunghiurile ABC şi a b cτ τ τ sunt omologice (consecinţă a

teoremei lui Desargues). 8) Triunghiul median al triunghiului cotangentic al unui triunghi ABC este omologic cu triunghiul ABC . Demonstraţie. Fie 'A , 'B , 'C mijloacele laturilor

b cτ τ ,

a cτ τ , respectiv

a bτ τ ale

triunghiului cotangentic şi " 'A AA BC= ∩ , " 'B BB AC= ∩ , " 'C CC AB= ∩

(Fig. 379). Considerând ceviana "AA şi transversala b cτ τ avem:

' "1

' "c b

b c

A AAC A B

AB A A A C

τ ττ τ

⋅ ⋅ ⋅ = (vezi „Relaţia lui Van-

Aubel”), deci "

1 1"

−⋅ ⋅ ⋅ =

−p b b A B

c p c A C de unde

" ( )

" ( )

−=

−A B c p c

A C b p b. Analog,

" ( )

" ( )

B C a p a

B A c p c

−=

− şi

" ( )

" ( )

C A b p b

C B a p a

−=

−, de unde

" " "1

" " "

A B B C C A

A C B A C B⋅ ⋅ = şi din

reciproca teoremei lui Ceva rezultă că dreptele 'AA , 'BB şi 'CC sunt concurente, deci triunghiurile ABC

şi a b cτ τ τ sunt omologice.

9) Fie 'A , 'B , 'C punctele diametral opuse vârfurilor A,B,C ale triunghiului ABC în cercul circumscris. Dreptele '

aI A , '

bI B , '

cI C sunt respectiv perpendiculare pe laturile

triunghiului cotangentic al triunghiului ABC . Demonstraţie. Notăm cu

xz afixul punctului X .

Alegem un reper complex cu vârful în centrul cercului circumscris ( )O al triunghiului ABC ,

iar punctul A situat pe axa imaginară. Atunci

Az iα= , *α ∈ , 'Az iα= − . Din 'BA AB⊥ şi

'CA CA⊥ rezultă că există *,β γ ∈ astfel încât

'A B

A B

z zi

z zβ

−=

− şi 'A C

A C

z zi

z zγ

−=

−, de unde

2

2

1Bz i

αβα

β= −

+ şi

2

2

1Cz i

αγα

γ= −

+ (1).

A

B C

A" aτ

Fig. 379

A

B C

O

aI

bτ Re(z)

Fig. 380

Im(z)

Page 363: Barbu_TEOREME_FUNDAMENTALE(2008).pdf

370

Deoarece b

b

C p a

A p c

ττ

−=

− şi c

c

A p b

B p a

ττ

−=

− rezultă

( ) ( )− + −=

b

C Az p c z p a

zb

τ şi

( ) ( )c

A Bz p a z p bz

− + −= , deci

2

2 ( ) ( )

(1 )b

p c i c az

bbτ

αγ αγ− −

= ++ ⋅

şi

2

2 ( ) ( )

(1 )c

p b i b az

ccτ

αβ αβ− −

= ++ ⋅

(2). Fie a

T CI AB= ∩ . Din teorema bisectoarei exterioare

avem TA b

TB a= , a

a

CI BC

I T BT= . Astfel

acTB

b a=

−, a

a

CI b a

I T c

−= , de unde A B

T

az bzz

a b

+=

+ şi

( )

1a

C TC T

I

b az z

cz b a zczb a b c a

c

−+ ⋅ + −

= =− + −+

(3). Din relaţiile (1), (2) şi (3) rezultă 'a

b c

I Az z

iz zτ τ

θ−

= ⋅−

,

unde *θ ∈ , deci 'a b cI A τ τ⊥ . Analog se arată că ' ⊥

b a cI B τ τ şi ' ⊥

c b aI C τ τ .

III.6. Triunghiul antipodar

„Am fost un lucrător conştiincios, nu am fost superficial, am muncit până acum la bătrâneţe. Spiritul meu a fost geometric, m-a fermecat varietatea şi frumuseţea figurii.”

– Alexandru Miller179

Fie P un punct în planul unui triunghi ABC. Triunghiul ' ' 'A B C format de perpendicularele duse prin vârfurile A, B, C pe cevienele PA, PB respectiv PC, se numeşte triunghiul antipodar al punctului P (Fig. 381). Punctul P se numeşte punct antipodar. 1) Triunghiul ABC este triunghiul podar al punctului P în triunghiul ' ' 'A B C . 2) Triunghiul antipodar al ortocentrului H al triunghiului ABC este triunghiul anticomplementar. Demonstraţie. Vezi „Triunghiul anticomplementar”. 3) Triunghiul antipodar al centrului cercului înscris în triunghiul ABC este triunghiul antisuplementar corespunzător triunghiului ABC.

Demonstraţie. Vezi „Triunghiul antisuplementar”. 4) Triunghiul antipodar al centrului cercului circumscris O al triunghiului ABC este triunghiul tangenţial corespunzător triunghiului ABC. Demonstraţie. Vezi „Triunghiul tangenţial”.

179 Alexandru Miller (1879-1965) – matematician român , membru al Academiei Române

A'

B' C'

P

A

B C

Fig. 381

A"

B" C"

Page 364: Barbu_TEOREME_FUNDAMENTALE(2008).pdf

371

5) Triunghiul antipodar al unui punct P este ortologic cu triunghiul pedal al punctului P în raport cu triunghiul ABC. Demonstraţie. Fie " " "A B C triunghiul pedal corespunzător punctului P în raport cu triunghiul ABC (Fig. 381). Deoarece " ' ', " ' ', " ' 'A A B C B B A C C C A B⊥ ⊥ ⊥ şi

" " " A A B B C C P∩ ∩ = rezultă că triunghiurile ' ' 'A B C şi " " "A B C sunt ortologice.

6) Triunghiul antipodar unui punct P este ortologic cu triunghiul anticevian al punctului P. Demonstraţie. Triunghiul anticevian al punctului P este triunghiul ABC. Cum

' ', ' ', ' ', AP B C BP A C CP A B AP BP CP P⊥ ⊥ ⊥ ∩ ∩ = rezultă că triunghiurile ' ' 'A B C

şi ABC sunt ortologice. 7) Fie L, M, N punctele diametral opuse vârfurilor A, B, respectiv C ale unui triunghi ABC, în cercul circumscris acestuia. Triunghiul LMN este omologic cu triunghiul antipodar al oricărui punct P aflat în interiorul triunghiului ABC. Demonstraţie. Fie l, m , n dreptele ce trec prin A, B, C şi sunt perpendiculare pe dreptele PA, PB respectiv PC şi D, E, F al doilea punct de intersecţie dintre dreptele l, m, n şi cercul circumscris triunghiului ABC (Fig. 382). Deoarece ( ) 90m ADL = ° rezultă .PA DL

Dacă ,Q PO DL= ∩ din congruenţa

triunghiurilor APO şi LQO rezultă .PO OQ≡

Analog, se arată că dreptele EM, FN trec prin Q, de unde rezultă că

2QD QL QE QM QF QN p⋅ = ⋅ = ⋅ = relaţii ce

arată că punctele L, M şi N sunt polii dreptelor l, m şi n în raport cu cercul având centrul în Q şi raza p. 8) Triunghiul antipodar al unui punct P şi triunghiul podar al izogonalului său 'P sunt omotetice. Demonstraţie. Fie " " "A B C triunghiul podar al lui '.P Deoarece " "B C AP⊥ (vezi „Drepte izogonale”) şi ' 'B C AP⊥ rezultă ' ' " "B C B C (Fig.383 ). Analog, ' ' " "A B A B şi ' ' " ".A C A C Deoarece patrulaterul 'PBA C este inscriptibil rezultă

( ' ) 180 ( ) ( ) ( ) ( ' ) ( ' ")= ° − = + = + = m BA C m BPC m PBC m PCB m P BA m P CB

( " " ') ( ' " ") ( " " ").+ = m C A P m P A B m C A B Deci ' ' ' " " "C A B C A B≡ şi analog

' ' ' " " ",A B C A B C≡ de unde rezultă că triunghiurile ' ' 'A B C şi " " "A B C sunt asemenea. Deoarece triunghiurile ' ' 'A B C şi " " "A B C au laturile paralele şi sunt asemenea rezultă că ele sunt omotetice.

A

B C

O

Fig. 382

N M

D

P

Q

L

l

Page 365: Barbu_TEOREME_FUNDAMENTALE(2008).pdf

372

9) Triunghiurile antipodare ale punctelor lui Fermat corespunzătoare unui triunghi ABC sunt echilaterale. Demonstraţie. Fie 1F primul punct al lui Fermat - Toricelli. Deoarece

1 1 1( ) ( ) ( ) 120m AFB m BFC m CFA= = = ° (vezi „Punctele lui Fermat”), iar patrulaterele

1 'BF AC , 1 'CF BA şi 1 'AFCB sunt inscriptibile, rezultă că

( ' ) ( ' ) ( ' ) 60 ,m BA C m CB A m AC B= = = ° deci triunghiul ' ' 'A B C este echilateral. Analog

se arată că şi triunghiul antipodar al celui de-al doilea punct Fermat 2( )F este echilateral.

A

B C

P

P '

Fig. 383

A'

B' C'

A"

B" C"

A

B C

1F

A'

B'

C'

Fig. 384

120°

Page 366: Barbu_TEOREME_FUNDAMENTALE(2008).pdf

373

III.7. Triunghiul podar

„Matematica este ştiinţa despre raporturile între formule, lipsite de oricare conţinut.” - David Hilbert180

Fie P un punct interior triunghiului ABC. Se numeşte triunghi podar al punctului P, triunghiul ' ' 'A B C care are ca vârfuri picioarele perpendicularelor duse din P pe laturile triunghiului ABC (Fig. 385). Punctul P se numeşte punct podar. Cercul circumscris triunghiului podar al punctului P se numeşte cercul podar al punctului P. Dacă punctul P aparţine unei laturi a triunghiului, atunci dreapta care uneşte picioarele perpendicularelor din P pe celelalte două laturi ale triunghiului se numeşte dreaptă podară. 1) Punctul podar P aparţine cercurilor circumscrise patrulaterelor ' ', ' ',AB PC BC PA

' '.CA PB Demonstraţia este evidentă, deoarece patrulaterele ' ', ' ', ' 'AB PC BC PA CA PB sunt inscriptibile. 2) Dacă x, y, z sunt distanţele de la punctul podar P la vârfurile triunghiului ABC ,

atunci laturile triunghiului podar au lungimile ' , ' ,2 2

= =ax by

a bR R

'2

=cz

cR (a, b, c sunt

lungimile laturilor triunghiului ABC şi R raza cercului circumscris triunghiului ABC ). Demonstraţie. Teorema sinusurilor aplicată în triunghiurile ' 'AC B şi ABC ne dă:

' '

sin=

B CAP

A şi 2 ,

sin

aR

A= de unde rezultă ' ' .

2

APB C a

R= ⋅ Analog, ' '

2

BPC A b

R= ⋅ şi

' ' .2

CPA B c

R= ⋅

Observaţie: Dacă x y z= = atunci punctul P coincide cu centrul cercului circumscris

ABC şi deci .x R= 3) Triunghiul podar ' ' 'A B C al unui punct P şi triunghiul ABC sunt ortologice. Demonstraţia rezultă din definiţia triunghiurilor ortologice.

180David Hilbert (1962-1943) – matematician german, profesor la Universitatea din Göttingen, contribuţii

remarcabile în geometrie şi analiza matematică

A

B C

P

A'

B' C'

Fig. 385

Page 367: Barbu_TEOREME_FUNDAMENTALE(2008).pdf

374

4) Triunghiul podar al centrului cercului circumscris triunghiului ABC este triunghiul median al triunghiului ABC. Demonstraţia este evidentă. 5) Triunghiul podar al ortocentrului triunghiului ABC este triunghiul ortic al triunghiului ABC . Demonstraţia este evidentă. 6) Cercul lui Euler este cercul podar al ortocentrului triunghiului ABC (sau al centrului cercului circumscris triunghiului ABC). Demonstraţie. Vezi „Cercul lui Euler”.

7) Triunghiul podar al centrului cercului înscris este triunghiul de contact al triunghiului ABC . Demonstraţie. Vezi „Triunghiul de contact”. 8) Cercul podar al centrului cercului înscris într-un triunghi ABC este cercul înscris în triunghiul ABC. Demonstraţia este evidentă.

9) Triunghiurile podare ale punctelor Brocard ale triunghiului ABC sunt congruente. Demonstraţie. Vezi „Punctele lui Brocard”. 10) Triunghiurile podare ale punctelor izodinamice ale triunghiului ABC sunt echilaterale. Demonstraţie. Vezi „Punctele izodinamice”. 11) Punctul lui Lemoine al triunghiului ABC este centrul de greutate al propriului triunghi podar. Demonstraţie. Vezi „Punctul lui Lemoine”. 12) Teorema celor 6 puncte Fie 'P şi "P două puncte izogonale în raport cu triunghiul ABC şi ' ' 'A B C şi " " "A B C triunghiurile lor podare în raport cu triunghiul ABC. Punctele ', ', ', ", ", "A B C A B C sunt conciclice. Demonstraţie. Vezi „Drepte izogonale”. Observaţie: Centrul cercului pe care se găsesc punctele ', ', ', ", ", "A B C A B C este mijlocul segmentului ' "P P (vezi „Drepte izogonale”). 13) Fie P un punct nesituat pe cercul circumscris al unui triunghi ABC şi ' ' 'A B C triunghiul podar al lui P în raport cu triunghiul ABC. Perpendicularele din A, B, C pe

' ', ' 'B C A C respectiv ' 'A B sunt concurente într-un punct ',P izogonalul punctului P în raport cu triunghiul ABC. Demonstraţie. Vezi „Drepte izogonale”.

Page 368: Barbu_TEOREME_FUNDAMENTALE(2008).pdf

375

14) Teorema lui Oppenheim Al treilea triunghi podar este asemenea cu triunghiul original. Demonstraţie. Punctul P aparţine cercurilor circumscrise triunghiurilor 1 1,ABC

2 1 2 ,A BC 3 3 2 ,A B C 2 2 1A B C şi 3 2 3A B C (Fig. 386). Avem:

1 1 1 2 1( ) ( ) ( )= = = m C AP m C B P m A B P 2 2 3 2 3 3( ) ( ) ( ),= = m A C P m B C P m B A P

1 1 1 1 2( ) ( ) ( )= = = m PAB m PC B m PC A 2 2 2 3 3 3( ) ( ) ( )= = m PB A m PB C m PA C de unde

rezultă că 3 3 3( ) ( )= m BAC m B A C . Analog se arată că 3 3 3( ) ( )= m BCA m B C A , de

unde rezultă că triunghiurile ABC şi 3 3 3A B C sunt asemenea.

15) Triunghiul podar al unui punct P în raport cu un triunghi ABC este asemenea cu triunghiul circumpedal al punctului P. Demonstraţie. Vezi „Triunghiul circumpedal”. 16) Triunghiul podar al ortocentrului triunghiului ABC este omotetic cu triunghiul tangenţial al triunghiului ABC . Demonstraţie. Vezi „Triunghiul tangenţial”.

17) Aria triunghiului podar a b cP P P al unui

punct P în raport cu un triunghi ABC este

egalǎ cu

2 2

sin sin sin2

−⋅ ⋅ ⋅

R OPA B C , unde

R reprezintă lungimea razei cercului circumscris triunghiului ABC şi O centrul cercului circumscris acestui triunghi. Demonstraţie. Fie D al doilea punct de intersecţie dintre dreapta AP şi cercul circumscris triunghiului ABC. Avem:

( ) ( ) ( )c b

m DBC m DAC m PP P= = şi

( ) ( )a a c

m PBP m P P P= de unde rezultă cǎ

A

B C

P

aP

bP cP

Fig. 387

D

A

B C

P

A1

B1 C1

Fig. 386

B2

C2

A2

A3

B3 C3

Page 369: Barbu_TEOREME_FUNDAMENTALE(2008).pdf

376

( ) ( )a c b

m DBP m P P P= . Din teorema sinusurilor în triunghiul BPD avem:

sin sin sinPBD BDP ACB

PD BP BP= =

(1) (unde am folosit faptul cǎ BDP ACB≡ ). Deoarece

PC este diametru în cercul circumscris patrulaterului a b

PPCP avem sin= ⋅a bP P PC ACB şi

analog se obţin relaţiile sin= ⋅b cP P PA A , sin= ⋅

a cP P PB B (2). Din relaţiile (1) şi (2):

[ ]

s i n

2a b c

a c b c a c b

P P P

P P P P P P PA

⋅ ⋅= sau

[ ]

sin sin sin

2

⋅ ⋅ ⋅ ⋅= =

a b cP P P

PB PA A B DBPA

sin sin sin

2

⋅ ⋅ ⋅ ⋅PA PD A B C. Utilizând puterea punctului faţă de un cerc rezultă

2 2PA PD R OP⋅ = − şi atunci [ ]

2 2

sin sin sin2a b cP P P

R OPA A B C

−= ⋅ ⋅ ⋅ .

Observaţii:

1) Ţinând cont că [ ] 2 sin sin sinABC

A R A B C= ⋅ ⋅ ⋅ relaţia precedentă devine:

[ ] [ ]

2 2

24a b c ABCP P P

R OPA A

R

−= ⋅ .

2) Dacă P O≡ , atunci aria triunghiului podar al centrului cercului circumscris triunghiului

ABC este egală cu [ ] [ ]1

4a b c ABCO O OA A= ⋅ , ( a b cO O O este de fapt triunghiul median al

triunghiului ABC). 3) Dacă P G≡ , atunci aria triunghiului podar al centrului de greutate al triunghiului ABC

este egală cu [ ] [ ] [ ] [ ]

2 2 2

2 2 2 2 2

2 2 29

4 4 36a b c ABC ABC ABCG G G

a b cR OG a b c

A A A AR R R

+ +− + +

= ⋅ = ⋅ = ⋅ .

4) Dacă P H≡ , atunci aria triunghiului podar al ortocentrului triunghiului ABC devine:

[ ] [ ] [ ] [ ]

2 2 2 2 2 2 2 2 2 2 2

2 2 2

[9 ( )] 8 ( )]

4 4 4a b c ABC ABC ABCH H H

R OH R R a b c R a b cA A A A

R R R

− − − + + − + += ⋅ = ⋅ = ⋅

5) Dacă P I≡ , atunci aria triunghiului de contact este egală cu:

[ ] [ ] [ ] [ ]

2 2

2 2

2

24 4a b c ABC ABC ABCI I I

R OI Rr rA A A A

RR R

−= ⋅ = ⋅ = ⋅ .

6)Dacă P K≡ , atunci aria triunghiului podar al punctului simedian al triunghiului ABC este egală cu

[ ] [ ] [ ] [ ]

2 2 2

2 2 2 2 22 2 2 2

2 2 2 2 2 2 2

3

3( )

4 4 4 ( )a b c ABC ABC ABCK K K

a b c

R OK a b ca b cA A A A

R R R a b c

− + += ⋅ = ⋅ = ⋅

+ +.

7) Dacă notăm lungimile segmentelor aPP , bPP , cPP cu x , y respectiv z atunci

[ ] [ ] [ ] [ ]sin sin sin

2a b c b c c a a bP P P PP P PP P PP P

yz A xz B xy CA A A A

+ += + + = .

Page 370: Barbu_TEOREME_FUNDAMENTALE(2008).pdf

377

18) Fie P un punct ce variază pe un cerc C ce are centrul în punctul O - centrul cercului

circumscris unui triunghi ABC . Triunghiurile podare ale punctelor de pe centrul C, în

raport cu triunghiul ABC , au aceeaşi arie.

Demonstraţie. Deoarece [ ]

2 2

sin sin sin2a b cP P P

R OPA A B C

−= ⋅ ⋅ ⋅ , iar când P se plimbă pe

cercul C, segmentul OP are aceeaşi lungime fiind rază în cercul C, rezultă concluzia.

19) Dacă a b cP P P este triunghiul podar al punctului P în raport cu triunghiul ABC

atunci: a b b c a cP P P P P P

AB CP BC AP CA BP= =

⋅ ⋅ ⋅.

Demonstraţie. În cercul circumscris triunghiului b cAP P punctul P este punctul diametral

opus punctului A . Teorema sinusurilor ne dă: sinb cP P

APA= , de unde

sin2b c

BCP P AP A AP

R= = ⋅ şi de aici

1

2b cP P

BC AP R=

⋅ (unde R este raza cercului circumscris

triunghiului ABC ). Analog se arată că 1

2a b a cP P P P

AB CP CA BP R= =

⋅ ⋅.

20) Fie P un punct interior triunghiului ABC şi zyx ,, distanţele de la P la laturile

triunghiului BC , CA , respectiv AB. Notăm cu p perimetrul triunghiului podar al

punctului P. Atunci, ( ) cos .2

Cx y p+ ≤∑

Demonstraţie. Fie ',',' cba lungimile laturilor

triunghiului podar a b cP P P (Fig. 338). Avem: 2 2 2' 2 cos(180 )c x y xy C= + − ° − sau

2 2 2 2 2' 2 cos ( ) 4 cos2

Cc x y xy C x y xy= + + = − +

2 2 2' [ ( ) 4 ] c o s2

Cc x y x y≥ − + ⋅ sau

2 2 2' ( ) cos2

Cc x y≥ + de unde ' ( ) cos

2

Cc x y≥ +

cu egalitate dacă şi numai dacă x y= . Atunci

' ( ) cos2

Cp c x y= ≥ +∑ ∑ cu egalitate dacă şi

numai dacă x y z= = .

21) Triunghiul podar al punctului lui Lemoine al triunghiului automedian ABC este asemenea cu triunghiul ABC. Demonstraţie. Vezi „Triunghiul automedian”.

A

B C

P

aP

bP cP

Fig. 388

b ' c ' x

y z

a '

Page 371: Barbu_TEOREME_FUNDAMENTALE(2008).pdf

378

22) Proiecţia pe bisectoarea unghiului A a unui triunghi ABC a unui punct arbitrar P de pe cercul circumscris triunghiului BIC - unde I este centrul cercului înscris in triunghiul ABC - este centrul cercului circumscris triunghiului podar al punctului P în raport cu triunghiul ABC . Demonstraţie. Fie 'P simetricul lui P faţă de bisectoarea din A (Fig. 389). Deoarece centrul cercului circumscris triunghiului

BIC aparţine bisectoarei din A şi este mijlocul arcului BC al cercului circumscris triunghiului ABC (vezi „Cercuri

exînscrise”) rezultă că 'P aparţine cercului circumscris triunghiului, iar cum 'PBI P BI≡ şi 'PCI P CI≡ , rezultă că punctele P şi 'P sunt puncte izogonale. Proiecţiile punctelor izogonale P şi 'P pe laturile triunghiului ABC sunt conciclice pe un cerc cu centrul în mijlocul segmentului 'PP , deci proiecţia lui P pe bisectoarea din A este centrul cercului circumscris triunghiului podar al punctului P în raport cu triunghiul ABC . 23) Triunghiul podar al ortocentrului H al triunghiului ABC în raport cu triunghiul ortic al triunghiului ABC este omotetic cu triunghiul ABC , centrul de omotetie fiind punctul lui Gob. Demonstraţie. Vezi „Punctul lui Gob”. 24) Triunghiul podar al unui punct P de pe un cerc al lui Apollonius, în raport cu triunghiul ABC este isoscel. Demonstraţie. Vezi „Cercurile lui Apollonius”. 25) Fie ' ' 'A B C triunghiul podar al unui punct P în raport cu un triunghi ABC şi "A cel de-al patrulea vârf al paralelogramului ' ' ".B PC A Analog se determină punctele "B şi ".C Triunghiurile ABC şi " " "A B C sunt omologice, centrul de omologie fiind punctul izogonal conjugat al punctului P.

Demonstraţie. Arătăm că dreptele PA şi "AA sunt izogonale. Fie "X CA AC= ∩ şi

"Y BA AB= ∩ (Fig. 390). Deoarece

patrulaterul ' 'PB AC este inscriptibil, rezultă ' ' 'B C A B PA≡ şi " " ' .B AP B C P≡

Cum ( ' ) ( ' ) 90m B AP m B PA+ = ° şi

( ' ' ) ( " ') 90m B C A m A AC+ = ° rezultă că

' " 'PAB A AC≡ ( "A fiind ortocentrul triunghiului ' 'B C A ). Analog, se arată că dreptele PB şi "BB , respectiv PC şi "CC sunt izogonale, deci triunghiurile ABC şi " " "A B C sunt omologice, centrul de omologie fiind izogonalul lui P.

A

P P '

I

B C

Fig. 389

A

B C

P

A'

B' C'

Fig. 390

X Y

A"

Page 372: Barbu_TEOREME_FUNDAMENTALE(2008).pdf

379

26) Fie ' ' 'A B C triunghiul podar al unui punct P în raport cu un triunghi ABC. Notăm cu ad dreapta ce trece prin mijloacele segmentelor PA şi ' 'B C . Analog definim

dreptele ,b cd d . Dreptele , ,a b cd d d sunt concurente. Demonstraţie. Mijlocul segmentului PA este centrul cercului circumscris triunghiului

' ' 'A B C , deci dreapta ad este mediatoarea segmentului ' 'B C . Analog, ,b cd d sunt

mediatoarele segmentelor ' 'C A , respectiv ' 'A B , deci dreptele , ,a b cd d d sunt concurente

în centrul cercului circumscris triunghiului podar ' ' 'A B C .

III.8. Triunghiul tangenţial

„Totul ar fi trebuit sa fie sfere, dar n-a fost, n-a fost asa.

Totul ar fi trebuit sa fie linii, dar n-a fost, n-a fost asa.” Nichita Stănescu181

Triunghiul CBA TTT determinat de tangentele duse la cercul circumscris triunghiului ABC

în vârfurile acestuia se numeşte triunghiul tangenţial al triunghiului ABC .

1) Triunghiul tangenţial CBA TTT al triunghiului

ABC este triunghiul antipodar al centrului cercului circumscris O al triunghiului ABC în raport cu triunghiul ABC.

2) Centrul cercului circumscris O al triunghiului ABC reprezintă centrul cercului înscris în triunghiul tangenţial CBA TTT .

3) Dreptele BA BTAT , şi CCT sunt concurente.

Demonstraţie. Deoarece , ,≡ ≡A A B BT B T C T A T C

≡C CT A T B avem: 1=⋅⋅B

A

A

C

C

A

CT

CT

BT

BT

AT

AT şi din

reciproca teoremei lui Ceva rezultă că dreptele BA BTAT , şi CCT sunt concurente. Fie K

punctul de concurenţă al dreptelor , ,A B CAT BT CT (Fig. 391).

4) Dreptele , ,A B CAT BT CT sunt simedianele triunghiului ABC .

Demonstraţie. Fie ' ( ),= BCA pr A ' ( )=A AC AT pr T şi " ( )=

A AB AT pr T (Fig. 391). Cum

' 1( ') ( ) ( )

2= =A Am ABA m T CT m AC şi '( ' ) ( ) 90= = °

A Am AA B m CT T rezultă că triunghiurile

'ABA şi 'A AT CT sunt asemenea, deci

'

'=A A AT T T C

AA AB (1). Analog din asemănarea

triunghiurilor CAA' şi "A AT BT rezultă

"

'=A A AT T T B

AA AC (2). Împărţind relaţiile (1) şi (2)

181 Nichita Stănescu (1933 – 1983) – eseist, poet român, ales postum membru al Academiei Române

A

C O K

AT

BT CT

Fig. 391

B A' 'AT "

AT

Page 373: Barbu_TEOREME_FUNDAMENTALE(2008).pdf

380

membru cu membru rezultă '

"= ⋅ =A A A

AA A

T T T C CA CA

AB BT ABT T, unde am utilizat AA CTBT = , relaţie

ce arată că punctul AT aparţine simedianei din A a triunghiului ABC . Analog se arată că

BBT şi CCT sunt simediane.

Observaţii:

1) Punctul lui Lemoine K al triunghiului ABC este punctul de concurenţă al dreptelor

CBA CTBTAT ,, , adică punctul lui Gergonne al triunghiului tangenţial.

2) Punctul lui Lemoine este centrul de omologie dintre triunghiul ABC şi triunghiului său tangenţial. 5) Drepta lui Lemoine a triunghiului ABC este axa de omologie dintre triunghiul ABC şi triunghiul său tangenţial CBA TTT .

Demonstraţie. Fie 111 ,, CBA punctele de

instersecţie dintre tangentele la cercul circumscris ABC , vârfurile acestuia şi laturile opuse. Din teorema lui Lemoine rezultă că punctele 111 ,, CBA

sunt coliniare (punctele aparţin dreptei lui

Lemoine). Avem 2

1

1

=

AC

AB

CA

BA (1). Fie

A BC' AB T T= I , A CB' AC T T= I , B CA' BC T T= I .

Să arătăm că punctele ',',' CBA aparţin dreptei

11CA . Fie , ,= =I IA A B BK AT BC K BT CA

= IC CK CT AB , iar K punctul lui Lemoine al triunghiului ABC (Fig. 392). Deoarece

AAK este simediană în triunghiul ABC rezultă 2

=

A

A

BK AB

CK AC(2). Din relaţiile (1) şi (2)

rezultă 1

1

=A

A

BK BA

CK CA, adică punctele 1A şi AK sunt conjugate armonic faţă de punctele B şi

C. Analog, se arată că şi perechile de puncte 1( , ), ( , )B CB K C K sunt conjugate armonic faţă

de ( , )C A , respectiv ( , )A B , ceea ce demonstrează teorema.

6) Consecinţă: Triunghiul tangenţial CBA TTT este

triunghiul anticevian al punctului lui Lemoine corespunzător triunghiului ABC . 7) Laturile triunghiului ortic cba HHH al

triunghiului ABC sunt respectiv paralele cu laturile triunghiului tangenţial CBA TTT .

Demonstraţie. ( ) ( ) ( )= =C c bm T AB m AH H m ACB

rezultă că bcBC HHTT || . Analog baBA HHTT || şi

.|| caCA HHTT

A

B C

AT

BT

CT

1A

1B

AK

BK CK

Fig. 392

A

C aH

bH

H

AT

BT CT

Fig. 393

B

cH

Page 374: Barbu_TEOREME_FUNDAMENTALE(2008).pdf

381

8) Triunghiul ortic cba HHH al triunghiului ABC şi triunghiul tangenţial CBA TTT al

triunghiului ABC sunt omotetice.

Demonstraţie. Avem ≡ ≡C CT AB T BA ACB , deci ( ) 180 2 ( )= ° −Cm AT B m C . Dar

( ) 180 2 ( )= °−b c am H H H m C , deci ≡B C A b c aT T T H H H şi analog cbaCBA HHHTTT ˆˆ ≡ .

Atunci triunghiurile A B CT T T şi a b cH H H sunt asemenea şi deoarece triunghiurile au laturile

paralele (vezi proprietatea precedentă) rezultă că triunghiurile CBA TTT şi cba HHH sunt

omotetice. Observaţie: Centrul omotetiei dintre triunghiul ortic cba HHH şi triunghiul tangenţial

CBA TTT aparţine dreptei lui Euler a triunghiului ABC (vezi „Dreapta lui Euler”) şi se

numeşte punctul lui Gob. 9) Fie , ,A B C

∗ ∗ ∗ mijloacele laturilor , ,b c c aH H H H respectiv a bH H ale triunghiului ortic

al triunghiului ABC şi CBA TTT triunghiul său tangenţial. Punctele A, A∗ , AT ; B, B∗ ,

BT ; respectiv C, C∗ , CT sunt coliniare.

Demonstraţie. Deoarece A∗ este mijlocul antiparalelei b cH H la BC rezultă că aparţine

simedianei din A , adică dreptei AAT , de unde rezultă concluzia.

10) Triunghiul tangenţial CBA TTT şi triunghiul median al triunghiului ortic corespunzător unui triunghi ABC sunt omotetice, punctul lui Lemoine al triunghiului ABC fiind centrul de omotetie. Demonstraţia rezultă din proprietatea de mai sus.

11) Triunghiul tangenţial CBA TTT şi triunghiul median al triunghiului ortic corespunzător unui triunghi ABC sunt ortologice, centrele de ortologie fiind ortocentrul triunghiului tangenţial şi centrul cercului lui Euler al triunghiului ABC . Demonstraţie. Fie A B C∗ ∗ ∗ triunghiul median al triunghiului ortic cba HHH . Deoarece

9 b cO A H H∗ ⊥ şi bcBC HHTT || rezultă 9 C B

O A T T∗ ⊥ . Analog, se arată că 9 C AO B T T∗ ⊥ şi

9 A BO C T T∗ ⊥ , deci triunghiurile A B C∗ ∗ ∗ şi CBA TTT sunt ortologice, unul dintre centre

fiind centrul cercului lui Euler 9O . Deoarece A x B CT H T T⊥ şi B CT T B C∗ ∗

rezultă

A xT H B C∗ ∗⊥ (unde xH este ortocentrul triunghiului tangenţial), analog

B xT H A C∗ ∗⊥ ,

deci xH este al doilea centru de ortologie.

12) Aria triunghiului tangenţial este egal cu: [ ] [ ]

2 2 2

4

+ += +

A B C ABCT T T

a tgA b tgB c tgCA A ,

unde cba ,, sunt lungimile laturilor triunghiului ABC .

Demonstraţie. În triunghiul isoscel ABT C ( ( ) ( ) ( ))= =A Am T BC m T CB m A avem:

ACT

aA

2ˆcos = , de unde

2cos=A

aCT

A. Atunci,

[ ]

2 2sin sin

2 4cos 4

⋅ ⋅= = =

A

A A

T BC

BC CT BCT a A a tgAA

A. Analog, [ ]

2

4=

BT AC

b tgBA şi

Page 375: Barbu_TEOREME_FUNDAMENTALE(2008).pdf

382

[ ]

2

4=

CT BA

c tgCA , de unde [ ] [ ] [ ] [ ] [ ]= + + + =

A B C A B CT T T T AC T AC T BA ABCA A A A A

2 2 2

[ ]4

+ ++ ABC

a tgA b tgB c tgCA .

Observaţie: Dar triunghiul ABC este obtuzunghic, de exemplu ˆ( ) 90> °m A , atunci

[ ]

2

4= ⋅

AT BC

aA tgA .

13) Centrul cercului circumscris triunghiului tangenţial CBA TTT al triunghiului ABC

aparţine dreptei lui Euler a triunghiului .ABC

Demonstraţie. Dreapta determinată de centrul cercului circumscris, respectiv înscris într-un triunghi XYZ este dreapta lui Euler a triunghiului de contact al triunghiului XYZ (vezi „Dreapta lui Euler”). Cum triunghiul ABC este triunghiul de contact corespunzător triunghiului tangenţial rezultă că dreapta lui Euler a triunghiului ABC trece prin centrul cercului circumscris triunghiului tangenţial CBA TTT .

14) Fie O centrul cercului circumscris triunghiului ABC . Centrele CBA OOO ,, ale

cercurilor circumscrise triunghiurilor , ,BCO ACO respectiv BAO determină un triunghi omotetic cu triunghiul tangenţial CBA TTT al triunghiului ABC , centrul de

omotetie fiind punctul O . Demonstraţie. Deoarece BAOOOC ⊥ şi BATTOC ⊥ rezultă BABA TTOO || . Analog, CBCB TTOO || şi

ACAC TTOO || . Deoarece BCOOA ⊥ şi BCOTA ⊥ (triunghiul BCTA fiind isoscel)

rezultă că punctele AOO, şi AT sunt coliniare. Analog BOO, şi BT respectiv COO, şi

CT sunt coliniare. Atunci, triunghiurile CBA OOO şi CBA TTT sunt omotetice, centrul

omotetiei fiind punctul O .

A

B C

O

AO

BO

CO

Fig. 394

AT

BT

CT

Page 376: Barbu_TEOREME_FUNDAMENTALE(2008).pdf

383

15) Triunghiul tangenţial al triunghiului ABC şi triunghiul circumpedal al centrului de greutate al triunghiului ABC sunt omologice. Demonstraţie. Vezi „Punctul lui Exeter”. 16) Triunghiurile tangenţial şi extangenţial corespunzătoare unui triunghi ABC sunt

omotetice, raportul de omotetie fiind .2

R

rR +

Demonstraţie. Vezi „Triunghiul extangenţial”. 17) Fie CBA TTT triunghiul tangenţial, a b cN N N şi ' ' '

a b cN N N triunghiurile lui Napoleon

ale unui triunghi ABC. Punctele aN , 'a

N , AT sunt coliniare.

Demonstraţia este evidentă deoarece punctele aN , 'a

N , AT aparţin mediatoarei segmentului

BC. 18) Triunghiul tangenţial CBA TTT al triunghiului ABC şi triunghiul median

cba MMM al triunghiului ABC sunt omologice.

Demonstraţie. Deoarece ,≡A ABT CT ,≡B BAT CT ≡C CAT BT (Fig. 395) rezultă că

CcBbAa MMMMTM ,, sunt

mediatoarele laturilor triunghiului ABC , deci ele sunt concurente în centrul cercului circumscris O al triunghiului ABC , deci triunghiurile CBA TTT şi

cba MMM sunt omologice.

Observaţie: Centrul cercului circumscris triunghiului ABC este polul de omologie al triunghiului

CBA TTT şi cba MMM .

19) Axa de omologie dintre triunghiurile tangenţial şi median ale unui triunghi ABC este perpendiculară pe dreapta lui Euler a triunghiului ABC . Demonstraţie. Fie ''' CBA axa de omologie dintre triunghiurile

CBA TTT şi cba MMM (Fig. 395).

Deoarece BCMM cb || , atunci

' ≡ ≡ c bA AC ABC AM M , de unde

rezultă că triunghiurile 'bAM A şi

'cM AA sunt asemenea, deci

''

' '= b

c

A MAA

A M AA, egalitate

A

C aM

bM

C'

AT

BT

CT

Fig. 395

B

cM

B'

A'

Page 377: Barbu_TEOREME_FUNDAMENTALE(2008).pdf

384

echivalentă cu 2' ' '= ⋅b c

AA A M A M . Cum cMA' este secantă la cercul lui Euler, rezultă că

'A are aceeaşi putere faţă de cercul circumscris triunghiului ABC şi de cercul lui Euler al triunghiului ABC , deci 'A aparţine axei radicale a acestor cercuri. Analog se arată că 'B şi

'C aparţin acestei axe, deci axa de omologie este axa radicală a cercului circumscris triunghiului ABC şi a cercului lui Euler a triunghiului ABC . Deoarece axa radicală este perpendiculară pe linia centrelor 9OO , adică pe dreapta lui Euler, rezultă că axa de

omologie dintre triunghiurile CBA TTT şi cba MMM este perpendiculară pe dreapta lui

Euler a triunghiului ABC .

20) Prin inversiunea de centru O şi raport 2R , cercul circumscris triunghiului tangenţial CBA TTT se transformă în cercul lui Euler al triunghiului ABC .

Demonstraţie. Din asemănarea triunghiurilor aBOM şi OBTA rezultă:A

a

OT

R

R

OM= ,

adică 2ROTOM Aa =⋅ . Analog, se arată că 2ROTOMOTOM ccBb =⋅=⋅ , relaţii ce arată

că prin inversiunea J 2( , )O R cercul circumscris triunghiului tangenţial se transformă în

cercul circumscris triunghiului median, adică cercul lui Euler al triunghiului ABC .

21) Fie C cercul circumscris triunghiului ABC , ',',' CBA punctele de intersecţie dintre

înălţimile din BA, respectiv C şi laturile triunghiului ABC cu C, iar CBA TTT triunghiul

tangenţial al triunghiului ABC . Dreptele ',',' CTBTAT CBA sunt concurente într-un

punct ce aparţine dreptei lui Euler a triunghiului ABC .

Demonstraţie. Fie 111 ,, CBA punctele diametral opuse în C

ale punctelor BA, respectiv C , iar X punctul de

intersecţie dintre cercul AT -exînscris, corespunzător

triunghiului A B CT T T cu latura CBTT . Analog se definesc

prin punctele Y şi Z . Deoarece în triunghiul ABC , 'AA şi

1AA sunt izogonale (vezi „Drepte izogonale”) rezultă că

dreptele 'ATA şi 1ATA sunt izogonale conjugate în

triunghiul tangenţial. Deoarece AT este centrul de

asemănare dintre cercurile C şi cercul AT -exînscris, rezultă

că punctele AT , 1A şi X sunt coliniare. Deoarece dreptele

YTXT BA , şi ZTC sunt concurente în punctul lui Nagel al

triunghiului CBA TTT , rezultă că şi izogonalele lor - adică dreptele ',',' CTBTAT CBA sunt

concurente într-un punct S care este centrul de asemănare dintre cercurile C şi cercul

circumscris triunghiului CBA TTT , deci S aparţine dreptei lui Euler a triunghiului ABC .

A

B C

AT

BT

CT

X

1A

O

Fig. 396

A'

Page 378: Barbu_TEOREME_FUNDAMENTALE(2008).pdf

385

III.9. Triunghiul anticomplementar

„Geometrii se servesc de figuri vizibile şi fac judecăţi asupra acestora, deşi ei nu se gândesc la aceste figuri, ci la altele care se aseamănă, dar care nu pot fi sesizate decât cu mintea.”- Platon 182

Triunghiul anticomplementar (sau antimedian) al triunghiului ABC este triunghiul

''' CBA determinat de paralelele duse prin vârfurile triunghiului ABC la laturile opuse. Numim exmediană a unui triunghi o dreaptă ce trece prin vârful unui triunghi şi este paralelă cu latura opusă. Triunghiul anticomplementar este determinat de intersecţiile exmedianelor triunghiului ABC. Vârfurile triunghiului anticomplementar se numesc puncte exmediane. 1) Triunghiul ''' CBA este triunghiul anticomplementar al triunghiului ABC dacă triunghiul ABC este triunghiul său median.

2) Segmentele AB , BC , AC sunt liniile mijlocii ale triunghiului anticomplementar.

3) Dacă cba ,, sunt lungimile laturilor CABC, respectiv AB ale triunghiului ABC , atunci laturile triunghiului aticomplementar ''' CBA sunt egale cu 2a,2b,2c. 4) Centrul de greutate al triunghiului anticomplementar al triunghiului ABC este centru de greutate şi pentru triunghiul ABC . Demonstraţie. Fie G centrul de greutate al triunghiului ''' CBA şi cba MMM triunghiul

său median. Deoarece patrulaterul CABA' este paralelogram rezultă că diagonalele BC şi 'AA se înjumătăţesc, deci aAM este mediană în triunghiul ABC . Analog se arată că

bBM este mediană, deci G este centrul de greutate al triunghiul .ABC

Observaţie: Triunghiul anticomplementar ''' CBA este triunghiul anticevian al triunghiului ABC corespunzător centrului de greutate G al acestuia. 5) Aria triunghiului anticomplementar ''' CBA este egal cu: [ ] [ ]' ' '

4 .A B C ABC

A A=

182 Platon (428-348) – filosof, logician, matematician grec

A'

B' C'

C B

A

aM

bM cM G

Fig. 397

Page 379: Barbu_TEOREME_FUNDAMENTALE(2008).pdf

386

Demonstraţie. Deoarece patrulaterele ',' ABCBCABA şi BCAC ' sunt paralelograme,

rezultă că [ ] [ ] [ ] [ ]' ' '= = =ABC BCA ACB ABC

A A A A , de unde [ ] [ ]' ' ' 4 .= ⋅A B C ABC

A A

Cercul circumscris triunghiului anticomplementar se numeşte cerc anticomplementar. 6) Centrul cercului anticomplementar este ortocentrul triunghiului .ABC

Demonstraţie. Fie .BCAH a ⊥ Din ''|| CBBC rezultă că ''CBAH a ⊥ , cum A este

mijlocul segmentului ''CB obţinem că aAH este mediatoarea segmentului '.'CB Analog

se arată că înălţimea bBH este mediatoarea segmentului ''CA , deci punctul de concurenţă

al înălţimilor triunghiului ABC este centrul cercului circumscris triunghiului anticomplementar.

7) Raza cercului anticomplementar este egală cu dublul razei cercului circumscris triunghiului ABC . Demonstraţie.

Fie R raza cercului circumscris triunghiului ABC şi AR raza cercului circumscris

triunghiului anticomplementar ' ' '.A B C Atunci: [ ] [ ]' ' '

' ' ' 2 2 2

4 4 4

⋅ ⋅ ⋅ ⋅= = =

⋅ ⋅A

A B C ABC

a b c a b cR

A A

[ ]

2 24⋅ =

ABC

abcR

A.

8) Fie ' ' 'A B C triunghiul anticomplementar al triunghiului ABC şi ", ", "A B C simetricele punctelor A, B respectiv C faţă de ortocentrul H al triunghiului ABC. Dreptele

' ", ' ", ' "A A B B C C sunt concurente. Demonstraţie. Alegem un reper complex cu originea în centrul cercului circumscris (O) al triunghiului ABC. Notăm cu litere mici afixele punctelor corespunzătoare. Atunci

( ), ( ), ( ), ( ), ( ),O o A a B b C c H a b c+ + de unde rezultă " " "

2 2 2

a a b b c ch

+ + += = = şi de aici

" 2 2 , " 2 2 , " 2 2 .a a b c b a b c c a b c= + + = + + = + + Deoarece ' ' ' ' ' '

, ,2 2 2

c b a c a ba b c

+ + += = =

rezultă ' ' ',a b c a b c+ + = + + deci 2 ',a a b c a= + + − de unde 'a b c a= + − şi analog,

A'

B' C'

C B

A

aH

bH cH H

Fig. 398

A" B" C"

Page 380: Barbu_TEOREME_FUNDAMENTALE(2008).pdf

387

' , ' .b a c b c b a c= + − = + − Ecuaţia dreptei ' "A A este:

1

' ' 1 0

" " 1

z z

a a

a a

= sau

(2 ) ( 2 ) 3[( ) ( )] 0 (1).z a b c z b c a ba ab ca ac− − + + − + − + − = Analog ecuaţiile dreptelor

' "B B şi ' "C C sunt: ( ' ") : (2 ) ( 2 ) 3[( ) ( )] 0 (2)B B z b a c z a c b ab ba cb bc− − + + − + − + − = ,

respectiv ( ' ") : (2 ) ( 2 ) 3[( ) ( )] 0 (3).C C z c a b z a b c ac ca bc cb− − + + − + − + − = Sumând

ecuaţiile (1), (2) şi (3) obţinem o egalitate ceea ce arată că dreptele ' ", ' "A A B B şi ' "C C sunt concurente 9) Cercul circumscris triunghiului anticomplementar al triunghiului ABC este tangent cercurilor circumscrise triunghiurilor CHABHC, şi AHB, unde H este ortocentrul triunghiului .ABC

Demonstraţie. Deoarece patrulaterul CHBA' este inscriptibil, iar m(HBA ') 90= ° rezultă că 'HA este diametrul în cercul circumscris patrulaterului .'CHBA Cum 'HA este raza în cercul circumscris triunghiului anticomplementar rezultă cercul circumscris triunghiului BHC este tangent interior în 'A cercului anticomplementar. Analog se arată că cercurile circumscrise triunghiurilor CHA şi AHB sunt tangente interior cercului anticomplementar. 10) Punctul lui Lemoine al triunghiului anticomplementar al unui triunghi ABC este retrocentrul triunghiului .ABC Demonstratie. Vezi „Retrocentrul unui triunghi”. 11) Într-un triunghi ABC , dreapta ON , care uneşte centrul cercului circumscris cu punctul lui Nagel, al triunghiului ABC, trece prin punctul lui Feuerbach al triunghiului anticomplementar. Demonstraţie. Vezi „Punctul lui Nagel”. 12) Ortocentrul triunghiului anticomplementar al unui triunghi ABC este punctul lui Longchamps al triunghiului ABC. Demonstraţie. Vezi „Punctul lui Longchamps”. 13) Triunghiul antipodar al ortocentrului H al triunghiului ABC este triunghiul anticomplementar. Demonstraţia rezultă din cele de mai sus.

Page 381: Barbu_TEOREME_FUNDAMENTALE(2008).pdf

388

III.10. Triunghiul antisuplementar

„Definiţiile şi proprietăţile dreptei, ca şi dreptele paralele au ajuns să fie stâncile sau , ca să spunem altfel, scandalul elementelor de geometrie.” - D’Alembert183

Triunghiul antisuplementar a b cI I I al unui triunghi ABC este triunghiul determinat de

bisectoarele exterioare ale triunghiului ABC (Fig. 399).

1) Triunghiul a b cI I I având vârfurile în centrele cercurilor exînscrise este triunghiul

antisuplementar al triunghiului ABC . Demonstraţia este evidentă ţinând cont că punctele , ,a b cI I I aparţin bisectoarelor

exterioare ale triunghiului ABC . 2) Unghiurile triunghiului antisuplementar au măsurile egale cu

190 ( ),

2m A° −

190 ( ),

2m B° −

190 ( )

2m C° − .

183 Jean d’Alembert (1717-1783) – matematician şi fizician francez, contribuţii importante în algebră şi analiza matematică

A

B

C

Ia

Ib

Ic

Fig. 399

bτ cτ

I

aC

cC

bC

Page 382: Barbu_TEOREME_FUNDAMENTALE(2008).pdf

389

Demonstraţie. În triunghiul aBCI , ( ) 180 [ ( ) ( )]a a am BI C m I BC m I CB= ° − + =

1 1 1180 90 ( ) 90 ( ) 90 ( ).

2 2 2m B m C m A

° − ° − + ° − = ° − Analog,

1( ) 90 ( ),

2bm AI C m B= °− 1

( ) 90 ( ).2cm AI B m C= °−

3) Triunghiul antisuplementar corespunzător triunghiului ABC este triunghiul anticevian corespunzător centrului cercului înscris (I) în triunghiul ABC. Demonstraţia este evidentă. 4) Triunghiul ABC este triunghiul ortic al triunghiului antisuplementar. Demonstraţie. Deoarece , ,a b cAI BI CI sunt înălţimile triunghiului a b cI I I (vezi„Cercurile

exînscrise”) rezultă că triunghiul ABC este triunghiul ortic al triunghiului a b cI I I .

Consecinţe: i) Triunghiurile ABC şi antisuplementar sunt omologice. ii) Axa ortică a triunghiului antisuplementar a b cI I I al triunghiului ABC este axa ortică

a triunghiului ABC . 5) Cercul circumscris al unui triunghi ABC este cercul lui Euler al triunghiului antisuplementar al triunghiului ABC . Demonstraţie. Deoarece triunghiul ABC este triunghiul ortic al triunghiului a b cI I I rezultă

că cercul circumscris triunghiului ABC este cercul lui Euler al triunghiului a b cI I I .

6) Dreapta care uneşte centrele cercurilor înscris şi circumscris ale unui triunghi ABC este dreapta lui Euler a triunghiului antisuplementar. Demonstraţie. Centrul cercului înscris I al triunghiului ABC este ortocentrul triunghiului antisuplementar, iar centrul cercului circumscris (O) al triunghiului ABC este centrul cercului lui Euler al triunghiului antisuplementar, deci dreapta OI este dreapta lui Euler a triunghiului antisuplementar. Observaţie: Triunghiul antisuplementar a b cI I I şi triunghiul de contact a b cC C C au aceeaşi

dreaptă a lui Euler OI.

7) Dreptele , ,a b cOI OI OI sunt dreptele lui Euler ale triunghiului ,b c a cII I II I ,

respectiv a bII I .

Demonstraţia este analoagă celei precedente. 8) Triunghiul antisuplementar a b cI I I şi triunghiul cevian 1 2 3I I I al centrului cercului

înscris într-un triunghi ABC sunt ortologice, dreapta lui Euler a triunghiului a b cI I I

fiind axa de ortologie. Demonstraţie. Vezi „Dreapta lui Euler”.

Page 383: Barbu_TEOREME_FUNDAMENTALE(2008).pdf

390

9) Triunghiul antisuplementar a b cI I I şi triunghiul de contact a b cC C C al unui triunghi

ABC sunt omotetice. Demonstraţie. Deoarece b cC C AI⊥ şi b cI I AI⊥ rezultă că b c b cC C I I (Fig. 399).

Analog , a c a cC C I I , a b a bC C I I , de unde rezultă că triunghiurile a b cI I I şi a b cC C C sunt

omotetice. 10) Centrul cercului circumscris triunghiului antisuplementar coincide cu centrul cercului înscris în triunghiul extangenţial. Demonstraţie. Vezi „Triunghiul cotangentic”. 11) Laturile triunghiului median a b cM M M al unui triunghi ABC trec prin proiecţiile

vârfurilor triunghiului ABC pe laturile triunghiului antisuplementar a b cI I I .

Demonstraţie. Fie 1 2,B B proiecţiile lui B pe laturile c bI I

şi a bI I ale triunghiului antisuplementar şi

1 .= IP BB AC Triunghiul APB este isoscel deoarece

AB este bisectoare şi înălţime, deci 1B este mijlocul

segmentului PB. În triunghiul APB, 1cM B este linie

mijlocie, deci 1 cM B AP adică 1 c a cM B AC M M ,de

unde rezultă că 1 ∈ a cB M M analog se arată că

2 ∈ a cB M M .

12) Consecinţă: Laturile triunghiului median al triunghiului ortic trec prin proiecţiile picioarelor înălţimilor pe laturile triunghiului dat. Demonstraţia este evidentă ţinând cont că ABC este triunghiul ortic al triunghiului a b cI I I .

13) Centrul cercului circumscris triunghiului antisuplementar al unui triunghi ABC este simetricul cercului înscris în triunghiul ABC faţă de centrul cercului circumscris triunghiului ABC . Demonstraţie. Vezi „Triunghiul cotangentic”.

14) Fie a b cM M M triunghiul median şi a b cI I I triunghiul antisuplementar al

triunghiului ABC . Dreptele , ,a a b b c cI M I M I M sunt concurente în punctul lui Lemoine al triunghiului antisuplementar. Demonstraţie. Vezi „Cercurile exînscrise”. 15) Triunghiurile antisuplementar şi median ale triunghiului ABC sunt omoloage, centrul de omologie fiind punctul lui Lemoine al triunghiului antisuplementar. Demonstraţie. Proprietatea este o consecinţă a celei precedente. 16) Triunghiul antisuplementar a b cI I I şi triunghiul cevian al centrului cercului înscris într-un triunghi ABC sunt ortologice. Demonstraţie. Vezi „Dreapta lui Euler”.

A

B

C

P

1B

2B

aI

bI

cI bM

aM

Fig. 400

Page 384: Barbu_TEOREME_FUNDAMENTALE(2008).pdf

391

17) Aria triunghiului antisuplementar a b cI I I al triunghiului ABC este egală cu 2 pR , unde p este semiperimetrul triunghiului ABC şi R este raza cercului circumscris triunghiului ABC. Demonstraţie. Soluţia1. [ ] [ ] [ ] [ ] [ ]= + + + =

a b c a b cI I I I B C I A C I AB ABCA A A A A

[ ]2 2 2+ + + =a b cABC

ar br crA

[ ] [ ] [ ][ ] 2

2( ) 2( ) 2( )ABC ABC ABC

ABC

aA bA cAA pR

p a p b p c+ + + =

− − −.

Soluţia2. Triunghiul ABC este triunghiul ortic al triunghiului a b cI I I . Dar

1 1 1( ) 90 ( ), ( ) 90 ( ), ( ) 90 ( )

2 2 2a b cm BI C m BAC m AI C m ABC m BI A m BCA= − = − = −o o o

[ ][ ] [ ] [ ] [ ]2 cos 2 sin sin sin2 2 2 2a b c a b c a b c

ABC

ABC I I I a b c I I I I I I

AA B CA A I I I A A

pR= ⋅ ⋅ ⋅ = ⋅ = ⋅ , de unde

rezultă concluzia. 18) Între ariile triunghiurilor antisuplementar, cotangentic şi aria triunghiului de referinţă ABC există relaţia : 2

[ ] [ ] [ ]a b c a b cABC I I IA A Aτ τ τ= ⋅ .

Demonstraţie. 2 2

[ ] [ ] [ ][ ]

[ ][ ]2sin sin sin 2 sin sin sin

2 2 2 2 2 2

a b c

a b c

ABC ABC ABC

I I I

ABC

A A AA

A B C A B C AA τ τ τ

= = = .

19) Triunghiul antipodar al centrului cercului înscris în triunghiul ABC este triunghiul antisuplementar corespunzător triunghiului ABC. Demonstraţia este evidentă deoarece , ,b c c a a bAI I I BI I I CI I I⊥ ⊥ ⊥ .

20) Axa ortică a triunghiului a b cI I I este dreapta determinată de picioarele bisectoarelor exterioare ale triunghiului ABC. Demonstraţie. Evident, deoarece triunghiul ortic al triunghiului a b cI I I este triunghiul ABC

şi I este ortocentrul triunghiului a b cI I I .

Observaţie: Deoarece axa ortică a unui triunghi este perpendiculară pe dreapta lui Euler a triunghiului rezultă că dreapta care uneşte picioarele bisectoarelor exterioare ale unui triunghi ABC este perpendiculară pe OI.

Page 385: Barbu_TEOREME_FUNDAMENTALE(2008).pdf

392

A

B C

C'

C" B"

B'

A' A"

Q

P

Fig. 401

III.11. Triunghiul ciclocevian

„Matematica este alfabetul după care Dumnezeu a scris Universul.” - Galileo Galilei184

Fie cevienele ', ', 'AA BB CC şi P punctul lor de concurenţă. Cercul circumscris triunghiului ' ' 'A B C intersectează fiecare latură în două puncte (nu neapărat distincte):

', "; ', "A A B B şi ', "C C . 1) Dreptele ", "AA BB şi "CC sunt concurente.

Demonstraţie. Din teorema lui Carnot rezultă " ' " ' " '

1" ' " ' " '

A B A B B C B C C A C A

A C A C B A B A C B C B⋅ ⋅ ⋅ ⋅ ⋅ = , adică

" " " ' ' '1

" " " ' ' '

A B B C C A A B B C C A

A C B A C B A C B A C B

⋅ ⋅ ⋅ ⋅ ⋅ =

(1). Deoarece ', ', 'AA BB CC sunt concurente ,din

teorema lui Ceva rezultă ' ' '

1' ' '

A B B C C A

A C B A C B⋅ ⋅ = (2). Din relaţiile (1) şi (2) rezultă

'' '' ''1

'' '' ''

A B B C C A

A C B A C B⋅ ⋅ = şi din reciproca teoremei lui Ceva rezultă că dreptele ", "AA BB şi

"CC sunt concurente.

Observaţii:

1) Punctul Q de concurenţă al dreptelor ", "AA BB şi "CC se numeşte ciclocevianul punctului P . 2) Triunghiul " " "A B C se numeşte triunghiul ciclocevian al triunghiului ABC , corespunzător punctului P .

2) Ortocentrul H şi centrul de greutate G al unui triunghi sunt puncte cicloceviene.

Demonstraţie. Punctele , , ,a b cH H H , ,a b cM M M aparţin cercului lui Euler al triunghiului

ABC (vezi „Cercul lui Euler”) (Fig. 402).

184 Galileo Galilei (1564-1642) – matematician, fizician, astronom şi filosof italian

Page 386: Barbu_TEOREME_FUNDAMENTALE(2008).pdf

393

Observaţii:

1) Triunghiul median este triunghiul ciclocevian al triunghiului ABC corespunzător ortocentrului triunghiului ABC . 2) Triunghiul ortic este triunghiul ciclocevian al triunghiului ABC corespunzător centrului de greutate al triunghiului ABC .

3) Punctul lui Gergonne Γ al triunghiului ABC este propriul său punct ciclocevian.

Demonstraţie. Evident, deoarece cercul circumscris triunghiului de contact a b cC C C este

tangent laturilor triunghiului ABC (Fig. 403).

Observaţie. Triunghiul ciclocevian corespunzător punctului lui Gergonne este triunghiul de contact al triunghiului ABC .

III.12. Triunghiul I - pedal

„Aţi observat că cine este capabil la matematică are cunoştinţe perfecte domeniul tuturor ştiinţelor legate de studiereanaturii?”-Platon185

Triunghiul 1 2 3I I I determinat de picioarele bisectoarelor

interioare ale triunghiului ABC se numeşte triunghiul I – pedal. 1) Triunghiul 1 2 3I I I este triunghiul cevian al

triunghiului ABC corespunzător centrului cercului înscris în triunghiul ABC .

185 Platon (428-348) – filosof, logician, matematician grec

A

B C

Cc

Cb

Г

Ca

Fig. 403

A

B C

Hb

Mb

Ha Ma

Mc

Hc

H

G

Fig. 402

A

B C

I

1I

2I 3I

Fig. 404

Page 387: Barbu_TEOREME_FUNDAMENTALE(2008).pdf

394

2) Laturile triunghiului I – pedal, 1 2 3I I I , au lungimile 3 2( cos cos cos )

' ,( )( )

+ − + +=

+ +

abc A B Ca

a b a c

3 2(cos cos cos )' ,

( )( )

+ − +=

+ +

abc A B Cb

b c b a

3 2(cos cos cos )'

( )( )

+ + −=

+ +

abc A B Cc

c a c b, (unde am

notat cu ', ', 'a b c lungimile laturilor 2 3 3 1, ,I I I I respectiv 1 2I I şi cu a, b, c lungimile laturilor triunghiului ABC ).

Demonstraţie. Din teorema bisectoarei rezultă 2

2

,AI c

I C a= de unde 2 = +

bcAI

a c şi analog se

obţine 3 .bc

AIa b

=+

Teorema cosinusului aplicată în triunghiul 2 3AI I ne dă:

2 2 22 3 2 3 2 32 cos ,I I AI AI AI AI A= + − ⋅ ⋅ sau

2 2 2 2 2 2 22

2 2' 2

2( ) ( )

+ −= + − ⋅ ⋅ ⋅ =

+ ++ +b c b c bc bc b c a

aa c a b bca c a b

2 2 2 2 22 2 2 2 2

2 2 2 2[( ) ( ) ] [3 2( cos cos cos )]

( ) ( ) ( ) ( )+ + + − − + = ⋅ + − + +

+ + + +b c a b c

a b bc a c bc b c a A B Ca b a c a b a c

şi de aici rezultă: 3 2( cos cos cos )

' .( )( )

+ − + +=

+ +

abc A B Ca

a b a c Analog se determină şi

lungimile celorlalte două laturi.

3) Aria triunghiului I – pedal este egală cu 1 2 3[ ] [ ]

2.

( )( )( )= ⋅

+ + +I I I ABC

abcA A

a b b c c a

Demonstraţie. Din teorema bisectoarei avem: 1

1

,=BI c

I C b

2

2

,=CI a

I A c

3

3

.=AI b

I C a Atunci,

2 3[ ] 2 3 32

[ ] ( )( )

⋅= = ⋅ = ⋅ =

⋅ + + + +AI I

ABC

A AI AI AIAI c b bc

A AB AC AC AB a b a b a b a c (1), analog

1 3[ ]

[ ] ( )( )=

+ +BI I

ABC

A ac

A b a b c (2) şi 2 1[ ]

[ ] ( )( )=

+ +CI I

ABC

A ab

A c a c b (3), iar

1 2 3 2 3 1 3 2 1[ ] [ ] [ ] [ ] [ ]= − − −I I I ABC AI I BI I CI IA A A A A (4). Din relaţiile (1), (2), (3) şi (4) rezultă

1 2 3[ ] [ ]

2.

( )( )( )= ⋅

+ + +I I I ABC

abcA A

a b b c c a

4) Triunghiul cevian 1 2 3I I I al centrului cercului înscris într-un triunghi ABC şi

triunghiul antisuplementar a b cI I I şi sunt ortologice. Demonstraţie. Vezi „Dreapta lui Euler”.

Page 388: Barbu_TEOREME_FUNDAMENTALE(2008).pdf

395

5) Fie I centrul cercului înscris în triunghiul ABC, 1 2 3I I I triunghiul cevian al punctului

I, 'I simetricul lui I faţă de latura BC, 1 1 2 3 ' , ' .A AI BC A AI I I= ∩ = ∩ Dreptele 1'A A şi BC sunt perpendiculare. Demonstraţie.

Deoarece 'AA şi 1AI sunt bisectoare în triunghiurile 2 3AI I şi ABC rezultă:

1

2 2' cos , cos .

2 2 2

bc A bc AAA AI

a b c b c= =

+ + + Din teorema bisectoarei rezultă

1 1

AI c b c

II BI a

+= =

de unde 2

cos2

bc AAI

a= şi ' ' 2 cos .

2 (2 )

A a b cA I AI AA bc

a a b c

+ += − = ⋅

+ + Atunci,

1 1

' ',

AA IA

AI II= deci punctele 1, , ',A I I I sunt conjugate armonic, atunci 'A şi 1I sunt

picioarele bisectoarelor interioare, respectiv exterioare ale unghiului 1A al triunghiului

1 ,AA I deci 1' .A A BC⊥

6) Fie 1 2 3I I I triunghiul cevian al centrului cercului înscris (I) în triunghiul ABC, *I

simetricul punctului I faţă de *2 3 2 3, .I I D AI I I= ∩ Dreptele 1I D şi 2 3I I sunt

perpendiculare. Demonstraţie. Deoarece punctele 1, ', ,A A I I sunt conjugate armonic (cf. th. 5), atunci

considerând fasciculul 1( ; , , ', )D A I A I rezultă că 'DA este bisectoarea interioară a

unghiului IDA şi 1DI bisectoarea exterioară a unghiului ,IDA deci 1' .DA DI⊥

7) Fie 1 2 3I I I triunghiul I – cevian al triunghiului oarecare ABC, D piciorul înălţimii din

1I pe 2 3 , 'I I I simetricul lui I faţă de BC şi 1 ' .A AI BC= ∩ Punctele 1,A I şi D sunt

coliniare. Demonstraţie. Fie 1 2 3 2 3 ' , "A AI I I A I I BC= ∩ = ∩ şi " '.E AA AA= ∩ Deoarece "AA

este bisectoarea unghiului BAC (vezi „Dreapta antiortică”) rezultă că 1" .AA AI⊥ Din

1 1 2 3,EA BC I D I I⊥ ⊥ rezultă că 'A este ortocentrul triunghiului 1 " ,I A E deci 1" .A D I E⊥

Deoarece 1 "I D A D⊥ rezultă că punctele E, D şi 1I sunt coliniare. Fie 1 1 ' .Y DA A I= ∩

Deoarece 2'A I este bisectoarea interioară a unghiului 1 1,ADA I D bisectoarea exterioară a

A

B C

I

1I

2I 3I

Fig. 405

I '

I∗

1A A"

A'

E

D

Page 389: Barbu_TEOREME_FUNDAMENTALE(2008).pdf

396

unghiului 1ADA rezultă că punctele 1, , ',A J A I sunt conjugate armonic, deci 1 1

' '.

AA JA

AI JI=

Dar 1 1

' 'AA IA

AI IA= de unde

1 1

' 'JA IA

JI IA= şi cum 1, ( )I J AI∈ rezultă .I J≡ Deci, punctele 1,A I

şi D sunt coliniare.

Observaţie: Fie 2 .A AD BC= ∩ Analog se demonstrează că punctele 2 , , 'A I A sunt

coliniare. 8) Fie 1 2 3I I I triunghiul I cevian al triunghiului ABC, D piciorul înălţimii din 1I pe

latura 2 3I I şi 'I simetricul lui I faţă de BC. Dreptele 'AI şi AD sunt izogonale.

Demonstraţie. Punctele 1, ,A I D sunt coliniare (conform proprietăţii precedente) şi

2 , , 'A I A sunt coliniare. Din teorema lui Pappus aplicată patrulaterului 1 2'A A DA rezultă că

punctele 1 1", ,A I A şi 2A sunt conjugate armonic şi deoarece 1"AA AI⊥ rezultă că 1AI

este bisectoarea unghiului ' ,A AD deci dreptele 'AA şi AD sunt izogonale.

III.13. Triunghiuri altimediale

„Poetul nu este al sieşi ! Viaţa lui este un cântec

un plâns pe buzele fiecăruia dăruindu-se el se multiplică la infinit şi intră în toate chipurile, în toate

sufletele! Nu-l învăţaţi pe Poet pe dinafară

El se află înlăuntrul vostru, ascultaţi – l !” - Radu Cârneci186

Fie aM , bM , cM mijloacele laturilor triunghiului ABC şi , ,a b cH H H picioarele

înălţimilor sale. Triunghiurile a b cH M M , b c aH M M şi c a bH M M se numesc triunghiuri altimediale. 1) Triunghiurile altimediale

a b cH M M , b c aH M M şi

c a bH M M sunt congruente cu

triunghiul median a b cM M M .

Demonstraţie. În triunghiul dreptunghic aAH B , a cH M este mediana corespunzatoare

ipotenuzei,deci2c a a b

ABM H M M= = . Analog, ,

2a b c a

ACH M M M= = de unde rezultă că

a b c a b cH M M M M M≡ . Analog se arată că şi triunghiuri altimediale c a bH M M şi

b c aH M M sunt congruente cu triunghiul median a b cM M M .

186 Radu Cârneci186- (1928 - ) – poet român

Page 390: Barbu_TEOREME_FUNDAMENTALE(2008).pdf

397

2) Dacă , ,a b cG G G sunt centrele de greutate ale triunghiurilor altimediale a b cH M M ,

b c aH M M , respectiv c a bH M M , atunci dreptele a aM G , b bM G , c cM G sunt concurente în

punctul lui Lemoine (K) al triunghiului ABC . Demonstraţie. Fie M mijlocul segmentului b cM M şi ' .a a aA M G AH= ∩ Din teorema lui

Menelaus aplicată pentru transversala ( , , ')a aM B A în triunghiul aAH M obţinem:

'1

'a a a

a a a

MM G HA A

A H M A G M⋅ ⋅ = , de unde

' 12 1

' 2a

A A

A H⋅ ⋅ = , adică ' ' aA A A H≡ , deci 'A este

mijlocul înălţimii ' aA H , rezultă că ' aA H este o dreaptă Schwatt. Analog, se arată că

b bM G şi c cM G , sunt drepte Schwatt şi conform teoremei lui Schömilch, dreptele a aM G ,

b bM G , c cM G sunt concurente în punctul lui Lemoine (K) al triunghiului ABC .

3) Centrele cercurilor înscrise în triunghiurilor altimediale ale triunghiului ABC determină un triunghi omologic cu triunghiul ABC. Demonstraţie. Fie 1 2 3, ,I I I centrele

cercurilor înscrise în triunghiurile

a b cH M M , b c aH M M , c a bH M M ,

1 'A AI BC= I , 2 'B BI AC= I ,

3 'C CI AB= I , 1( ")cm M I A α= ,

1( ")bm M I A β= (Fig.407). Deoarece

c bM M BC rezultă "'

(1).' "

c

b

M ABA

A C M A=

Din teorema sinusurilor aplicată în triunghiurile 1"cM A I şi 1"bM A I

rezultă : 1" "

sin sin2

cM A A I

Bα= , respectiv

A

B C

bM cM

Fig. 407

aH A'

1I

A"

α β

A

B C aM

bM cM

Fig. 406

aG

M

aH

A'

Page 391: Barbu_TEOREME_FUNDAMENTALE(2008).pdf

398

1" "

sin sin2

bM A A I

Cβ= , de unde

sin" sin 2" sin sin

2

c

b

CM A

BM A

αβ

= ⋅ (2). Teorema sinusurilor aplicată în

triunghiurile 1cAM I şi 1b

AM I ne dă : 1

3sinsin 2

c

B

AM AI

α= , respectiv

1

3sinsin 2

b

C

AM AI

β= , de unde

3sinsin 2

3sin sin2

B

c

C b

αβ= ⋅ (3). Din relaţiile (1), (2) şi (3) rezultă

3sin sin' 2 2

3' sin sin2 2

B C

BA c

C BA C b= ⋅ ⋅ .

Analog,

3sin sin' 2 2

3' sin sin2 2

C A

CB a

A CB A c= ⋅ ⋅ şi

3sin sin' 2 2

3' sin sin2 2

A B

AC b

B AC B a= ⋅ ⋅ , de unde

' ' '1

' ' '

BA CB AC

A C B A C B⋅ ⋅ = şi din reciproca teoremei lui Ceva rezultă că dreptele 1 2 3, ,AI BI CI

sunt concurente, deci triunghiurile ABC şi 1 2 3I I I sunt omologice.

III.14. Triunghiurile lui Brocard. Cercul lui Brocard187

„Operele matematice robesc şi încântă tocmai ca operele pasiunii şi imaginației.” - Ion Barbu Fie K punctul lui Lemoine al triunghiului ABC . Paralele duse prin K la laturile , ,BC CA AB

intersectează mediatoarele acestor laturi în punctele 1 1, ,A B respectiv 1.C Triunghiul 1 1 1A B C

se numeşte primul triunghi al lui Brocard. Fie 2 2 2, ,A B C proiecţiile punctului O -centrul

cercului circumscris triunghiului ABC - pe simedianele duse din vârfurile , ,A B C .

Triunghiul 2 2 2A B C se numeşte al doilea triunghi al lui Brocard. Cercul având diametru

segmentul OK se numeşte cercul lui Brocard. Două triunghiuri care au acelaşi unghi Brocard se numesc echibrocardiene. 1) Cercul lui Brocard este circumscris triunghiurilor lui Brocard. Demonstraţie. Deoarece 1 1 1( ) ( ) ( ) 90m KAO m KBO m KC O= = = ° rezultă că punctele

1 1 1, ,A B C aparţin cercului lui Brocard. Analog pentru punctele 2 2 2, ,A B C .

2) Triunghiurile 1 1 1, ,ABC BC A CA B sunt isoscele şi asemenea.

Demonstraţie. Deoarece punctele 1 1 1, ,A B C aparţin mediatoarelor laturilor triunghiului

ABC , rezultă că triunghiurile 1 1 1, ,ABC BC A CA B sunt isoscele. Deoarece

1 1 1, ,a b c

AM B M C M sunt egale cu distanţele de la K la laturile , ,BC CA AB , rezultă

187 Henri Brocard (1845-1922) – matematician francez, contribuţii importante în geometrie

Page 392: Barbu_TEOREME_FUNDAMENTALE(2008).pdf

399

1 1 1a b cAM B M C M

a b c= = (unde , ,

a b cM M M sunt mijloacele laturilor , ,BC CA AB ), adică

1 1 1a b c

a b a

AM B M C M

BM CM AM= = şi cum 1 1( ) ( )

a bm BM A m CM B= = 1( ) 90

cm AM C = ° , rezultă că

triunghiurile 1aBM A ,

bCM A şi 1c

AM C sunt asemenea. Atunci, triunghiurile 1 ,BAC 1CB A

şi 1AC B sunt asemenea.

3) Dreptele 1 1,A B B C şi 1C A sunt concurente într-un punct Brocard.

Demonstraţie. În triunghiul 1 aBAM avem: 1 1

1( ) ,/ 2

a

a

a

AM KKtg A BM tg

BM BCω= = = (unde

1K este proiecţia lui K pe BC), deci 1 aA BM este egal cu unghiul lui Brocard ω , de unde

rezultă că 1 1 1, ,A B B C C A sunt ceviene ce determină unul din punctele lui Brocard.

4) Dreptele 1 1 1, ,AB BC CA sunt concurente într-un punct Brocard.

Demonstraţie: În triunghiul 1 bAB M avem: 1 2

1( )/ 2

b

b

b

B M KKtg B AM tg

AM ACω= = = (unde

2K este proiecţia lui K pe AC ), deci 1B AC este egal cu unghiul lui Brocard, adică

1 1 1, ,A B B C C A sunt ceviene ce determină unul din punctele lui Brocard.

Observaţie: Dacă 1 1 1 A B BC C AΩ = ∩ ∩ , atunci 1 1 1 ' A B BC C AΩ = ∩ ∩ ; dacă

1 1 1 A B BC C AΩ = ∩ ∩ atunci 1 1 1 ' .A B B C C AΩ = ∩ ∩

5) Dreptele 1 1 1, ,AA BB CC sunt concurente.

Demonstraţie. Fie 1 1 ' , 'B KA AC C KA AB= ∩ = ∩ , L mijlocul segmentului .OK

Deoarece punctele 'B şi 'C sunt puncte pe al doilea cerc al lui Lemoine (cu centrul în punctul L) rezultă că proiecţia lui L pe coarda ' 'B C este punctul "A - mijlocul acestui segment (1). În triunghiul dreptunghic 1KOA , 1( ( ) 90 )m KAO = ° , ''LA este linie mijlocie,

deci ''A este şi mijlocul segmentului 1KA (2). Din relaţiile (1) şi (2) obţinem că

1' 'C K A B= , deci dreapta 1AA este dreapta izotomică simedianei AK . Analog se arată că

A

B C

O

Fig. 408

1A

1B

1C

2B 2C

K 2A

aM

bM cM

Page 393: Barbu_TEOREME_FUNDAMENTALE(2008).pdf

400

1BB şi 1CC sunt izotomicele simedianelor concurente în punctul izotomic punctului lui

Lemoine "Ω . Observaţii:

i) Punctul de concurenţă al dreptelor 1 1 1, ,AA BB CC se numeşte al treilea punct al lui Brocard corespunzător triunghiului ABC. ii) Din cele prezentate mai sus se poate spune că primul triunghi al lui Brocard 1 1 1A B C este

triomologic cu triunghiul ABC , centrele de omologie fiind punctele lui Brocard , 'Ω Ω şi izotomicul punctului lui Lemoine (al treilea punct al lui Brocard "Ω ). 6) Primul triunghi Brocard 1 1 1A B C al triunghiului ABC este asemenea cu triunghiul

ABC . Demonstraţie. Avem ( ) 180 ( )m A B m BΩ = °− (vezi „Punctele lui Brocard”), deci

1 1 1 1 1( ) ( ) ( )m C A m C B A m BΩ = = (1); ( ) 180 ( ),m B C m CΩ = ° − de unde:

1 1 1 1 1( ) ( ) ( )m B A m BC A m CΩ = = (2). Din relaţiile (1) şi (2) rezultă că triunghiurile

ABC şi 1 1 1A B C sunt asemenea.

7) Primul triunghi al lui Brocard al unui triunghi ABC, triunghiul ' "ΩΩ Ω şi triunghiul ABC au acelaşi centru de greutate. Demonstraţie. Vezi „Triunghiuri omologice”. 8) Cercul lui Brocard trece prin punctele Brocard Ω şi 'Ω ale triunghiului ABC . Demonstraţie. Din proprietatea (2) rezultă că triunghiul 1 a

AM C şi 1 bB M A sunt asemenea,

deci 1 1 1 'a b

M AC M B A OB≡ ≡ Ω , adică patrulaterul 1 1 'AOBΩ este inscriptibil, deci 'Ω

aparţine cercului lui Brocard. Analog se arată că Ω aparţine cercului lui Brocard. 9) Punctele lui Brocard Ω şi 'Ω ale triunghiului ABC sunt simetrice faţă de diametrul OK .

Demonstraţie. Avem 1 2

1( ) ( ) ( )

2m OK m B K m KAΩ = Ω = Ω şi deoarece 1KB AC

rezultă 1( ) ( )m B K m CA ωΩ = Ω = , deci ( )m OK ωΩ = . Analog,

1( ' ) ( ' )m OK m AKΩ = Ω = ( ' )m CB ωΩ = , deci 'OK OKΩ ≡ Ω , adică punctele

Ω şi 'Ω sunt simetrice faţă de dreapta OK .

10) Raza cercului lui Brocard este egală cu 21 4sin

2cos

RRω

ωω

−= .

Demonstraţie. În triunghiul dreptunghic K OΩ avem: 21 4sin

2cos cos

O RKO Rω

ωω ω

Ω −= = = , de unde raza cercului Brocard este egală cu

21 4sin

2cos

RRω

ωω

−= .

Page 394: Barbu_TEOREME_FUNDAMENTALE(2008).pdf

401

11) Raza cercului lui Brocard este egală cu 2 2 2

22 2 2 2

1 3

2 ( )

a b cR R

a b cω = −

+ +

Demonstraţie. Deoarece 2 2 2

2 22 2 2 2

3

( )

a b cOK R

a b c= −

+ + (vezi „Punctul lui Lemoine”) rezultă

2 2 22

2 2 2 2

1 3

2 ( )

a b cR R

a b cω = −

+ +.

12) Fie H ortocentrul triunghiului ABC şi "Ω al treilea punct al lui Brocard al triunghiului ABC. Atunci, " .H OKΩ Demonstraţie. Fie 'Z OK= ΩΩ ∩ (Fig. 409). Deoarece G este centrul de greutate al

triunghiurilor ABC şi ' "ΩΩ Ω rezultă 1

2

OG

GH= şi

1

" 2

GZ

G=

Ω, de unde obţinem " .H OKΩ

13) Fie H ortocentrul triunghiului ABC şi "Ω al treilea punct al lui Brocard al

triunghiului ABC. Atunci, 2" 2 cos 1 4sinH R ω ωΩ = − . Demonstraţie. Fie 'Z OK= ΩΩ ∩ . Din triunghiul O ZΩ rezultă

2cos cos 1 4sinOZ O Rω ω ω= Ω = − , deci 2" 2 2 cos 1 4sinH OZ R ω ωΩ = = − (Cf. th. 12). 14) Ortocentrul primului triunghi Brocard aparţine dreptei ".HΩ Demonstraţie. Fie L centrul cercului lui Brocard şi 1 "H LG H= ∩ Ω . Deoarece L este

centrul cercului circumscris triunghiului 1 1 1A B C şi G centrul său de greutate rezultă că

dreapta LG este dreapta lui Euler a primului triunghi Brocard. Cum 1

1

2

OG LG

GH GH= = ,

rezultă că 1H este ortocentrul primului triunghi al lui Brocard 1 1 1A B C .

G

Ω

ω K

Z

Fig. 409

O

H "Ω

1H

L

Page 395: Barbu_TEOREME_FUNDAMENTALE(2008).pdf

402

15) Consecinţă: Patrulaterul 1KOH H este paralelogram.

Demonstraţie. Deoarece 1HH OK şi 1 2HH LO OK= = rezultă că 1KOH H este

paralelogram. 16) Consecinţă: Centrul cercului lui Euler al triunghiului ABC aparţine dreaptei 1KH . Demonstraţie. Deoarece într-un paralelogram diagonalele se înjumătăţesc rezultă concluzia. 17) Punctul lui Tarry, centrul de greutate şi centrul cercului lui Brocard corespunzător unui triunghi ABC sunt coliniare. Demonstraţie. Din asemănarea poligoanelor TACSB şi 1 1 1OAC KB (cf. th. 4 – „Punctul lui

Tarry”), punctele G,O,T se corespunde cu punctele G,L, respectiv O şi de aici rezultă OGT LGO≡ , deci punctele T,L,G sunt coliniare. 18) Fie L centrul cercului lui Brocard, G centrul de greutate, T punctul lui Tarry şi O centrul cercului circumscris unui triunghi ABC. Atunci, 2 .OG GL GT= ⋅ Demonstraţie. Deoarece punctele G,O,T respectiv G,L,O formează figuri omoloage,

rezultă GO GT

GL GO= , deci 2 .OG GL GT= ⋅

19) Fie L centrul cercului lui Brocard, G centrul de greutate, T punctul lui Tarry şi O

centrul cercului circumscris unui triunghi ABC. Atunci, 2

cos.

1 4sin

RGT GO

ω

ω= ⋅

Demonstraţie. Din asemănarea triunghiurilor GLO şi GOT rezultă RGO GL LO

GT GO OT R

ω= = =

(Fig. 409), deci 2

cos.

1 4sin

R RGT GO GO

ω

ω= ⋅ = ⋅

20) Al treilea punct Brocard "Ω aparţine dreptei ce uneşte punctele lui Steiner şi Tarry corespunzătoare unui triunghi ABC. Demonstraţie. Fie 1H ortocentrul primului triunghi al lui Brocard şi

'Z OK= ΩΩ ∩ (Fig.409). Atunci, 1 1"2

H H G

ZL GL

Ω= = , iar = − =ZL OZ OL

22 1 4 sin

cos 1 4 sin2 cos

−− − =

RR

ωω ω

ω

21 4sincos 2

2cos

R ωω

ω−

⋅ şi de aici 1 " 2H LZΩ = .

Avem: 1 2H T TG GL

LT GT LG

+=

−; din

2

2

GT R

GL Rω= rezultă

2 2

2

22 R RTG GL

GL R

ω

ω

++= şi

2 2

2

R RTG GL

GL R

ω

ω

−−= , de unde

2 21

2 2

22cos 2

R RH T

LT R R

ω

ω

ω+

= =−

. Dar 1 "2cos 2

H

LOω

Ω= , deci

1 1"H H T

LO LT

Ω= şi cum 1 "H LOΩ rezultă că "Ω aparţine dreptei TO.

Page 396: Barbu_TEOREME_FUNDAMENTALE(2008).pdf

403

21) Fie O centrul cercului circumscris triunghiului ABC şi "Ω al treilea punct al lui Brocard al triunghiului ABC. Atunci, " (2cos2 1).O R ωΩ = −

Demonstraţie. Din asemănarea triunghiurilor 1"H TΩ şi OLT rezultă

1"2cos 2

H TT

TO LTω

Ω= = şi de aici

"2cos 2 1

T TO

TOω

Ω −= − sau " (2cos2 1).O R ωΩ = −

22) Dreapta OΩ este tangentă cercului circumscris triunghiului "SΩΩ , unde S este punctul lui Steiner corespunzător triunghiului ABC. Demonstraţie. Din 2 2" (2cos2 1)O OS R OωΩ ⋅ = − = Ω rezultă concluzia.

23) Dreapta 'OΩ este tangentă cercului circumscris triunghiului ' "SΩ Ω , unde S este punctul lui Steiner corespunzător triunghiului ABC. Demonstraţie. Din 2 2" (2cos 2 1) 'O OS R OωΩ ⋅ = − = Ω rezultă concluzia.

24) Paralelele duse prin vârfurile unui triunghi la laturile respective ale primului triunghi Brocard sunt concurente într-un punct situat pe cercul circumscris triunghiului ABC. Demonstraţie. Vezi „Punctul lui Steiner”. 25) Punctele lui Steiner (S) şi Lemoine (K) al triunghiului ABC sunt puncte omoloage în triunghiul ABC, respectiv 1 1 1A B C - primul triunghi Brocard al triunghiului ABC. Demonstraţie. Vezi „Punctul lui Steiner”. 26) Perpendicularele duse din vârfurile triunghiului ABC pe laturile opuse ale primului triunghi Brocard corespunzător triunghiului ABC sunt concurente în punctul lui Tarry al triunghiului ABC Demonstraţie. Vezi „Punctul lui Tarry”. 27) Consecinţă: Perpendicularele duse din miloacele laturilor primului triunghi Brocard corespunzător triunghiului ABC pe laturile respective ale triunghiului ABC ale sunt concurente. Demonstraţia rezultă din proprietatea precedentă . 28) Punctele 2 2 2, ,A B C aparţin cercurilor circumscrise triunghiurilor , ,BOC COA AOB .

Demonstraţie. Deoarece două simediane exterioare şi o simediana interioară ale unui triunghi sunt concurente (vezi „Simediana exterioară”), atunci simediana AK şi tangentele în B, respectiv C la cercul circumscris triunghiului ABC sunt concurente în punctul

AT (Fig. 410). Deoarece

( ) 90A

m OCT = ° rezultă că A

OT este diametru în cercul

circumscris triunghiului BOC. Deoarece 2 AOA AT⊥ rezultă

2( ) 90A

m OA T = ° , adică 2A este punct pe cercul circumscris

triunghiului BOC . Analog se arată că 2B şi 2C sunt pe

cercurile circumscrise triunghiurilor COA, respectiv AOB.

Un cerc care trece prin două vârfuri ale unui triunghi şi este tangent la una din laturile triunghiului se numeşte cerc adjunct.

A

B

C

AT

O 2A

K

Fig. 410

Page 397: Barbu_TEOREME_FUNDAMENTALE(2008).pdf

404

29) Punctul 2A se află la intersecţia cercurilor adjuncte vârfului A .

Demonstraţie. Deoarece 2 A ABA T BCT BAC≡ ≡ rezultă că cercul circumscris

triunghiului 2BA A este tangent în A laturii AC . Analog, cercul circumscris triunghiului

2CA A este tangent în A laturii AB .

Observaţii: 1) Punctele 2B şi 2C se află la intersecţia cercurilor adjuncte vârfurilor B, respectiv C.

2) Vârfurile celui de-al doilea triunghi Brocard al triunghiului ABC sunt intersecţiile dintre cercurile adjuncte corespunzătoare vârfurilor , ,A B C . 30) Coordonatele unghiulare ale vârfurilor celui de al doilea triunghi Brocard 2 2 2A B C

sunt: (180 ( ), 2 ( ),180 ( )), ((180 ( ),180 ( ), 2 ( )),m A m A m A m B m B m B° − ° − ° − ° −

respectiv (2 ( ),180 ( ),180 ( ))m C m C m C° − ° − .

Demonstraţie. Avem 2 2( ) 180 ( ) 180 ( ),A

m BA A m BA T m A= °− = °−

2 2( ) 180 ( )A

m CA A m T AC= °− 180 ( )m A= °− , iar

2 2 2( ) 360 [ ( ) ( )] 2 ( ).m BA C m BA A m CA A m A= °− + = Analog se determină

coordonatele unghiulare ale vârfurilor 2B şi 2 .C

31) Cercul Brocard şi primul cerc Lemoine sunt concentrice. Demonstraţia este evidentă deoarece ambele cercuri au centrul în punctul L, mijlocul segmentului OK.

III.15. Triunghiul antiparalel determinat de o direcţie în raport cu un triunghi

„Mă stimez mai mult ca practicant al matematicilor şi prea puţin ca poet, şi numai atât cât poezia aminteşte de geometrie. Oricât ar părea de contradictorii aceşti doi termeni la prima vedere, există undeva, în domeniul înalt al geometriei, un loc luminos unde se întîlneşte cu poezia.” – Ion Barbu188

În triunghiul ABC , fie ', ', 'A B C proiecţiile punctelor , ,A B C pe o dreaptă oarecare d,

, , .D BC d E CA d F AB d= ∩ = ∩ = ∩ Fie M şi 'M , N şi 'N , P şi 'P proiecţiile

punctelor ', ',A B respectiv 'C pe laturile AC şi AB, BA şi BC, respectiv CB şi CA. Dreptele ', ', 'MM NN PP - antiparalele dreptei d faţă de laturile triunghiului ABC - determină

triunghiul XYZ care se numeşte triunghiul antiparalel al dreptei d în rapot ce triunghiul ABC. Dacă antiparalele ', ', 'MM NN PP sunt concurente atunci punctul Q de concurenţă al acestora se numeşte antipol. Evident există o unică dreaptă care este paralelă cu d şi conţine antipolul Q.

188 Ion Barbu (1895-1961) – matematician român, profesor la Universitatea din Bucureşti, contribuţii în algebră şi geometrie

Page 398: Barbu_TEOREME_FUNDAMENTALE(2008).pdf

405

1) Perpendicularele 'B N şi ' 'C P , 'C P şi ' 'A M , 'A M şi ' 'B N sunt respectiv concurente în punctele coliniare 1 1 1, ,A B C .

Demonstraţie. Fie ω ortopolul dreptei d faţă de triunghiul ABC . Atunci

' ' ' ' , ' ' ' ' , ' ' ' ' .A B N C P B C P A M C A M B Nω ω ω Deoarece patrulaterul

1' 'A BCω este paralelogram (având laturile opuse paralele două câte două), atunci punctul

1B este simetricul ortopolului ω faţă de mijlocul segmentului ' 'A C . Procedând analog

pentru punctele 1A şi 1C rezultă că punctele 1 1 1, ,A B C aparţin unei drepte 1d paralele cu

dreapta d.

Consecinţă: Dreapta d se situează la o egală distanţă de ortopolul său ω în raport cu triunghiul ABC şi cu dreapta 1d .

2) Triunghiul antiparalel XYZ este invers asemenea şi ortologic cu triunghiul ABC. Demonstraţie. Perpendicularele duse din A, B şi C pe antiparalelele ', 'MM NN respectiv

'PP sunt izogonalele perpendicularelor ', 'AA BB şi 'CC pe d, faţă de unghiurile triunghiului ABC (patrulaterele ' ' , ' ' , ' 'AM A M BN B N CP PC fiind inscriptibile). Cum

' ' 'AA BB CC rezultă că perpendicularele pe antiparalelele considerate sunt concurente. Deoarece antiparalelele determină triunghiul XYZ, rezultă că triunghiurile ABC şi XYZ sunt invers asemenea (după orientarea vârfurilor cu unghiuri egale ale celor două triunghiuri) şi în acelaşi timp ortologice. 3) Cercurile de diametre 1 1,AA BB şi 1CC trec prin câte un vârf al triunghiului

antiparalel XYZ, iar linia centrelor este o dreaptă ',d perpendiculară pe d.

Demonstraţie. Cercul de diametru 1BB conţine punctele 'M şi P. Dacă ' 'Y MM PP= ∩

şi cum PYZ ABC≡ rezultă că patrulaterul 'BPYM este inscriptibil, adică Y aparţine

A

B C 1A

1B

1C

A'

B'

C'

E

F

M

M'

N

N' P

P '

A"

B"

C"

X

Y Z

d 1d

Fig. 411

Page 399: Barbu_TEOREME_FUNDAMENTALE(2008).pdf

406

cercului de diametru 1BB . Analog, cercurile de diametre 1AA şi 1CC trec respectiv prin

punctele X şi Z. Evident, proiecţiile ", ", "A B C ale punctelor A, B, respectiv C pe dreapta

1d aparţin – câte unul – cercurilor considerate. Deoarece 1 1 1, ' ' , ' " ' "d d A C C A A A C C

rezultă 1 1" "C A AC≡ , adică segmentele 1 "A A şi 1 "C C au acelaşi mijloc. Analog, se arată

că mijlocul segmentului 1 "B B coincide cu mijlocul segmentului 1 "A A . Rezultă că linia

centrelor este o dreaptă 'd perpendiculară pe d, trecând prin mijlocul segmentelor

1 1", "A A B B şi 1 ".C C

III.16. Triunghi automedian „Există printre matematicieni o convingere intimă şi puternică, care-i susţine în cercetările lor abstracte, anume că niciuna dintre problemele lor nu pot rămâne fără răspuns.” – Gh. Ţiţeica189 Se notează cu a, b, c lungimile laturilor BC, AC, respectiv AB ale triunghiului ABC şi cu

am ,

bm ,

cm lungimile medianelor corespunzătoare acestora. Triunghiul ABC ( AB AC≠ )

se numeşte automedian dacă 2 2 22a b c= + sau 2 2 22b c a= + sau 2 2 22c a b= + . În cele ce

urmează vom considera cazul în care 2 2 22a b c= + . 1) Să se arate că un triunghi ABC este automedian dacă şi numai dacă =

b cbm cm .

Demonstraţie. Relaţia 2 2 22 = +a b c este echivalentă cu 2 2 2 2 2(2 )( ) 0− − − =a b c b c

(deoarece ≠b c ) sau 2 2 2 2 2 2 2 2(2 2 ) (2 2 )+ − = + −b a c b c a b c sau .=b c

bm cm

Observaţii:

i) Un triunghi este automedian dacă şi numai dacă .1/ 1/

=b cm m

b c

ii) Ţinând cont de proprietatea anterioară putem da următoarea definiţie: „ Un triunghi în care medianele şi laturile corespunzătoare sunt invers proporţionale se numeşte automedian.”

2) Triunghiul ABC este automedian dacă şi numai dacă 2 3.=am a

Demonstraţie. Din 2 2 22 = +a b c rezultă 2 2 2 22 2 3+ − =b c a a egalitate echivalentă cu

2 3.=am a

3) Dacă triunghiul ABC este automedian, atunci 2 3bm c= şi 2 3.

cm b=

Demonstraţie. Din formula medianei avem: 2 2 2 2 24 2( ) 3bm a c b c= + − = , de unde rezultă

2 3bm c= ; analog se arată că 2 3.

cm b=

189 Gheorghe Ţiţeica (1873-1939) – matematician român, profesor la Universitatea din Bucureşti, membru al

Academiei Române, contribuţii importante în geometrie

Page 400: Barbu_TEOREME_FUNDAMENTALE(2008).pdf

407

Observaţie: Din proprietăţile 2) şi 3) rezultă că într-un triunghi automedian este adevărată

relaţia: .a c b

a b c

m m m= =

4) Într-un triunghi automedian avem : 2 2 22

a b cm m m= + .

Demonstraţie. Deoarece într-un triunghi automedian sunt adevărate egalităţile

2 3bm c= , 2 3

cm b= şi 2 3

am a= , atunci condiţia 2 2 22a b c= + devine

2 2 22 a b cm m m= + .

5) Un triunghi automedian având lungimile laturilor numere întregi nu poate avea mediane de lungimi numere întregi.

Demonstraţie. Fie , , ,a b c∈ atunci 2

,3a c b

a b c

m m m= = = de unde rezultă că medianele au

valori iraţionale. 6) Un triunghi automedian având lungimile laturilor numere întregi nu poate avea aria egală cu un număr întreg. Demonstraţie. Din formula lui Heron rezultă: 2 2 2 2 2 2 2 4 4 416 2( )S a c b c a b a b c= + + − − − .

Utilizând faptul că 2 2 22a b c= + , relaţia de mai sus devine: 2 2 2 4 416 4 2 .S a b a b= − − Fără a restrânge generalitatea putem presupune că ( , ) 1a b = . Atunci, din relaţia precedentă rezultă

2 b , deci 1 12 , .b b b= ∈ Astfel, obţinem 2 2 2 4 41 18 8 8S a b a b= − − , de unde rezultă 2 a ,

contradicţie cu ( , ) 1a b = .

7) Triunghiul ABC este automedian dacă şi numai dacă 2ctgA ctgB ctgC= + .

Demonstraţie. Avem: 2 2 2cos ( )

sin

A b c a RctgA

A abc

+ − ⋅= = şi analoagele . Atunci, relaţia

2ctgA ctgB ctgC= + este echivalentă cu 2 2 2 2 2 2 2 2 22( ) ( ) ( )b c a a c b b c a+ − = + − + + − sau 2 2 22a b c= + .

8) Într-un triunghi automedian ABC este adevărată relaţia: 1

3tg tgAω = , unde ω este

unghiul lui Brocard al triunghiului ABC. Demonstraţie. Deoarece triunghiul ABC este automedian rezultă 2ctgA ctgB ctgC= + şi

cum ctg ctgA ctgB ctgCω = + + (vezi „Punctele lui Brocard”) avem : 3ctg ctgAω = sau

1

3tg tgAω = .

9) Într-un triunghi automedian ABC este adevărată relaţia 2

[ ] 2ABC

aA tgA= ⋅ .

Demonstraţie: Avem 2 2 22a b c= + şi 2 2

[ ] 2 2 2 24 2 22 ( )= = ⋅ = ⋅ =

⋅ ⋅ + −ABC

abc a abc a abcA

R R a R b c a

2

2 2 2

2

2 2

a a bc

R b c a⋅ ⋅ =

+ −

2 21sin

2 cos 2

a aA tgA

A⋅ ⋅ = ⋅

Page 401: Barbu_TEOREME_FUNDAMENTALE(2008).pdf

408

10) Într-un triunghi automedian ABC este adevărată relaţia ( )( ) 2( )a b c b c ar r r r r r rr− + = − .

Demonstraţie: Se utilizează egalitaţile: 2 2 22a b c= + , ( )a br r p p c= − , ( )( )

arr p b p c= − − şi

relaţiile analoage. 11) Un triunghi isoscel automedian este echilateral. Demonstraţie. Dacă, de exemplu, =a c rezultă 2 2 22 = +a b a , deci 2 2=a b , de unde =a b , adică triunghiul este echilateral. Analog se tratează şi celelalte cazuri.

12) Să se arate că triunghiul ABC este automedian dacă şi numai dacă centrul de greutate G al triunghiului ABC este mijlocul segmentului AD, unde D este punctul în care mediana corespunzătoare laturii BC intersectează cercul circumscris triunghiului ABC. Demonstraţie. Fie O centrul cercului circumscris triunghiului ABC şi AG GD≡ . Din puterea punctului G faţă de cercul circumscris triunghiului ABC , avem:

2 2 2 22 2 2 2 2 2

.3 9a

b c aR OG AG m

+ − − = = =

Utilizând relaţia

2 2 2 2 21( )

9OG R a b c= − + + obţinem: 2 2 22a b c= + , adică

triunghiul ABC este automedian (Fig. 412). Reciproc, dacă triunghiul ABC este

automedian rezultă 2 2 22a b c= + . Avem: 2 2R OG AG GD− = ⋅ sau

2 2 21 2( )

9 3 aa b c m GD+ + = ⋅ , deci

23 2

9 3 a

am GD= ⋅ , de unde rezultă

2 2 3 2.

2 3 33a

a

a a aGD m AG

m a= = = = =

13) Dacă

aAM şi

bBM sunt mediane în triunghiul automedian ABC, atunci

( ) ( )a b

m BAM m CBM= .

Demonstraţie. Avem 1( ) ( ) ( ) ( )

2a bm BAM m BCD m BD m CBM= = = , unde D este punctul

de intersecţie dintre mediana a

AM şi cercul circumscris triunghiului ABC.

Consecinţă: Într-un triunghi automedian ABC , b

BM CD , unde D este punctul de

intersecţie dintre mediana a

AM şi cercul circumscris triunghiului ABC.

14) Triunghiul ABC este automedian dacă şi numai dacă ortocentrul H al triunghiului ABC se proiectează pe mediana , ( )

a aAM M BC∈ , în centrul de greutate

(G) al triunghiului ABC . Demonstraţie. Fie O centrul cercului circumscris triunghiului ABC . Avem

22 2 ,

2a

aOM R

= −

A

B C

D

aM

O G

bM

Fig. 412

Page 402: Barbu_TEOREME_FUNDAMENTALE(2008).pdf

409

22 2 2 2 2 22 2 2 1 2( )

,9 3 36a a

a b c b c aOG R GM m

+ + + − = − = =

(1). Folosind ipoteza

2 2 22b c a+ = şi faptul că punctele H,G,O sunt coliniare rezultă: 2 2 2

2 2 2 2 2, , ,3 12 4a a

a a aOG R GM OM R= − = = − de unde

22 2 2 2 ,

4a a

aOG GM R OM+ = − = adică

triunghiul a

M GO este dreptunghic, deci H se proiectează pe medianaaAM în G. Reciproc,

dacă H se proiectează pe a

AM în G, atunci a

GM O este dreptunghic, adică 2 2 2OM GO GM= + (2). Din relaţiile (1) şi (2) rezultă 2 2 22 ,b c a+ = adică

triunghiul ABC este automedian.

15) Într-un triunghi automedian ABC ( 2 2 22 = +a b a ) dreapta lui Euler este perpendiculară pe mediana din A a triunghiului ABC. Demonstraţia rezultă din proprietatea precedentă, deoarece .

aHG AM⊥

16) În triunghiul ABC fie mediana

aAM , înălţimea

cCH , ( )

cH AB∈ şi G centrul de

greutate. Dacă triunghiul ABC este automedian, atunci cercul circumscris triunghiului

a cBM H conţine punctul G.

Demonstraţie. Cum triunghiul ABC este automedian

avem: 2 2 22a b c= + (Fig. 413). Din teorema lui Pitagora generalizată rezultă:

2 2 2 2 2 22

2 2 2c

b c a a a aAH

c c c

+ − −= = = . Atunci,

2

2c

aAB AH⋅ = . Dar, 22

3a aAG AM AM⋅ = ⋅ =

2 2 222 2( ) 1

3 4 2

b c aa

+ −⋅ = ⋅ , adică

c aAB AH AG AM⋅ = ⋅

relaţie care arată că punctele , , ,c a

B H G M sunt

conciclice. 17) În triunghiul ABC , fie ,

b cH H picioarele înălţimilor duse din B , respectiv C şi

aM piciorul medianei din A. Dacă triunghiul ABC este automedian, atunci centrul de

greutate al triunghiului ABC aparţine cercului circumscris triunghiului .a b

CM H

Demonstraţie. Dreptele b c

H H şi BC fiind antiparalele rezultă :

b c aAC AH AB AH AG AM⋅ = ⋅ = ⋅ (vezi aplicaţia precedentă) de unde rezultă concluzia.

Hb

A

Fig. 413

B

Hc

C

H G

aM

Page 403: Barbu_TEOREME_FUNDAMENTALE(2008).pdf

410

III.17. Triunghi circumpedal

„Geometria este arta de a raţiona corect pe figuri incorecte.” – Henri Poincaré190

Fie D un punct în planul triunghiului ABC. Numim triunghi circumpedal (sau metaarmonic) al punctului D în raport cu triunghiul ABC, triunghiul a cărui vârfuri sunt punctele de intersecţie ale cevianelor AD, BD şi CD cu cercul circumscris triunghiului ABC (Fig. 414). 1) Triunghiul circumpedal ' ' 'A B C şi triunghiul podar " " "A B C al unui punct D în raport cu un triunghi ABC sunt asemenea. Demonstraţie. Patrulaterele " ", " ", ' 'DA BC DA CB ABA B şi ' 'ACA C sunt inscriptibile (Fig.415), deci " " " ' ' 'D A B D C B C C A A A C≡ ≡ ≡ şi

" " " ' ' .DA C DBC B BA BA A≡ ≡ ≡ . Dar ( " ") ( " ") ( " " "),+ = m DA C m DA B m B A C

iar ( ' ' ) ( ' ' ) ( ' ' '),+ = m B A A m C A A m B A C adică ( " " ") ( ' ' ').= m B A C m B A C

Analog se arată că ( " " ") ( ' ' ')= m A B C m A B C adică triunghiurile ' ' 'A B C şi " " "A B C

sunt asemenea. 2) Fie ' ' 'A B C triunghiul circumpedal al unui punct D în raport cu un tringhi ABC, iar

" " "A B C triunghiul podar al acestui punct. Atunci: [ ]2

' ' ' 2 sin "sin "sin "A B C

A R A B C= .

Demonstraţie. Deoarece triunghiurile ABC şi ' ' 'A B C sunt asemenea (cf. th. (1)) rezultă ' ' 2 sin ' ' ' 2 sin " " ", ' ' 2 sin " " ", ' ' 2 sin " " "B C R B A C R B A C C A R A B C A B R A C B= = = =

(Fig. 415) Atunci, [ ]

2

' ' '

' ' ' 'sin ' ' '2 sin "sin "sin ''.

2A B C

B A A C B A CA R A B C

⋅= =

3) Fie triunghiul ABC şi ' ' 'A B C triunghiul circumpedal al centrului cercului înscris în triunghiul ABC. Atunci,

[ ]

[ ' ' ']

28sin sin sin

2 2 2= =ABC

A B C

A A B C r

A R (unde r este raza cercului

înscris, iar R raza cercului circumscris în triunghiul ABC). Demonstraţie. Punctele ', ', 'A B C sunt punctele de intersecţie dintre bisectoarele unghiurilor triunghiului ABC cu cercul

190 Henri Poincaré (1854 -1912) – matamatician şi fizician francez, contribuţii importante în toate ramurile

matematicii

A

C

A'

B '

D

Fig. 414

B

C' A

C A'

B '

C" B"

A"

D

Fig. 415

B

C'

A

C

A'

B '

I

Fig. 416

B

C'

Page 404: Barbu_TEOREME_FUNDAMENTALE(2008).pdf

411

circumscris. Deoarece 1

( ' ' ') [ ( ) ( )]2

= + m C A B m B m C şi analoagele, avem:

2 2[ ' ' '] 2 sin sin sin 2 cos cos cos .

2 2 2 2 2 2

+ + += =A B C

B C A C A B A B CA R R Cum

2 2[ ] 2 sin sin sin 16 sin sin sin cos cos cos

2 2 2 2 2 2= =ABC

A B C A B CA R A B C R rezultă

că [ ]

[ ' ' ']

28sin sin sin

2 2 2= =ABC

A B C

A A B C r

A R.

4) Fie triunghiul ABC, " " "A B C triunghiul circumpedal al ortocentrului triunghiului

ABC. Atunci [ " " "]

[ ]

2 "=A B C

ABC

A r

A R, unde "r este lungimea razei cercului înscris în triunghiul

" " ".A B C Demonstraţie. Avem: 2 2

[ " " "] 2 sin sin sin 16 sin sin sin cos cos cos ,= =A B C

A R A B C R A B C A B C deci

[ " " "]

[ ]

8cos cos cos .=A B C

ABC

AA B C

A Observând că " , " , "A A B B C C sunt bisectoarele triunghiului

" " "A B C din teorema precedentă rezultă [ " " "]

[ ]

2 "=A B C

ABC

A r

A R.

Consecinţă: 5) [ " " "] [ ] [ ' ' ']≤ ≤

A B C ABC A B CA A A , unde ' ' 'A B C şi " " "A B C reprezintă triunghiurile

circumpedale ale centrului cercului înscris, respectiv al ortocentrului triunghiului ABC.

Demonstraţie. Din inegalitatea lui Euler, 2r R≤ şi relaţiile [ ]

[ ' ' ']

2=ABC

A B C

A r

A R,

[ " " "]

[ ]

2 "=A B C

ABC

A r

A Robţinem

[ " " "] [ ] [ ' ' ']≤ ≤A B C ABC A B CA A A şi de asemenea inegalitatea remarcabilă într-un triunghi

1cos cos cos .

8A B C ≤

6) Triunghiul circumpedal ' ' 'A B C al ortocentrului H al unui triunghi ABC este omotetic cu triunghiul ortic a b cH H H al triunghiului ABC, centrul de omotetie fiind ortocentrul triunghiului ABC. Demonstraţie. Conform teoremei (1) triunghiurile a b cH H H şi ' ' 'A B C sunt asemenea şi

deoarece ' ' ' a b cH A H B H C H∩ ∩ = rezultă că triunghiurile a b cH H H şi ' ' 'A B C

omologice. Cum 'a aHH H A≡ , 'b bHH H B≡ , 'c cHH H C≡ rezultă că , ,a b b c c aH H H H H H

sunt linii mijlocii în triunghiurile ' ' , ' ' ,A B H B C H respectiv ' 'C A H rezultă

' ', ' ',a b b cH H A B H H B C respectiv ' 'c aH H C A , deci triunghiurile a b cH H H şi

' ' 'A B C sunt omotetice, centrul de omotetie fiind punctul H, iar raportul de omotetie fiind egal cu 2.

Page 405: Barbu_TEOREME_FUNDAMENTALE(2008).pdf

412

7) Fie ' ' 'A B C triunghiul circumpedal al centrului cercului înscris I în triunghiul ABC şi a b cC C C triunghiul său de contact. Atunci, dreptele ' , ' , 'a b cA C B C C C sunt

concurente. Demonstraţie. Fie M un punct pe dreapta IO astfel încât

.R

MO MIr

=uuuur uuur

Prin omotetia de centru M şi raport R

r cercul

C ( , )I r se transformă în cercul C ( , )O R (Fig. 417). Atunci I se

transformă în O şi dreapta aIC se transformă în dreapta paralelă

',OA de unde rezultă că punctul 'A este omoteticul punctului

aC prin omotetia considerată. Analog prin omotetia ,R

H Mr

se corespund punctele 'B şi bC respectiv 'C şi cC , deci

dreptele ' , ' , 'a b cA C B C C C sunt concurente în punctul M.

Observaţie: Avem: 2 2r R Rr

MIR r

−=

− şi

2 2.

R R RrMO

R r

−=

Demonstraţie: Deoarece 2, 2R

MO MI OI MO MI R Rrr

= = − = − (relaţia lui Euler)

avem: r

MI OIR r

=−

şi ,R

MO OIR r

=−

de unde rezultă concluzia.

8) Ortocentrul triunghiului circumpedal ' ' 'A B C al centrului cercului înscris I în triunghiul ABC este punctul I. Demonstraţie. Vezi „Cercul înscris”. 9) Fie ' ' 'A B C triunghiul circumpedal al centrului cercului înscris în triunghiul ABC şi

1 1 1, ,A B C picioarele bisectoarelor interioare ale triunghiului ABC. Atunci,

1 1 1

1 1 1 18.

' ' ' RA A B B C C+ + ≥

Demonstraţie. Din puterea punctului 1A faţă de cercul circumscris rezultă

1 1 1 1' ,A A A A A B AC⋅ = ⋅ de unde 2

1 2 2'

( )aa l

A Ab c a

=+ −

unde ( )12

cos .2a

bc Al AA

b c= ⋅ =

+ Atunci,

21

2 21

',

( )

A A a

AA b c a=

+ − de unde

2

1 ',

'

A A a

AA b c

= + deci 1' ' 2 .

a aA A AA R

b c b c= ≤

+ +

Avem: 1 1 1

1 1 1

' ' '+ + ≥

A A B B C C

1

2

+ + + + + ≥

b c c a a c

R a b c

1 6 18.

2 2

b a c a b c

a b a c c b RR R

+ + + + + ≥ =

A

C

A'

B '

M O

aC

I

Fig. 417

B

C'

Page 406: Barbu_TEOREME_FUNDAMENTALE(2008).pdf

413

10) Fie ' ' 'A B C triunghiul circumpedal al centrului cercului înscris I în triunghiul ABC. Atunci, 2 'IA IC r IB⋅ = ⋅ şi ' ' .IA IC R IB⋅ = ⋅ Demonstraţie. Deoarece 'B este centrul cercului circumscris triunghiului AIC (vezi

„Teorema lui Beltrami”-Cecuri exînscrise) rezultă 2 ' sin 2 'r

IA IB ACI IBIC

= ⋅ = ⋅ , deci

2 ' aIA IC r IB r II⋅ = ⋅ = ⋅ (unde aI este centrul cercului exînscris). Deoarece

( ') ( ') 180 ( )m IBC m BIC m BIC= = °− , ( ' ) ( )m BC I m BAC= rezultă

' ' sin sin ' sin

sin sin ' sin

IC BI IC BCI IBC BCI

BC BC BI BIC BC I BAC= ⋅ = ⋅ = şi cum 2 ' sinIB IA BCM= ⋅ obţinem

' ' .IA IC R IB⋅ = ⋅

11) Fie ' ' 'A B C triunghiul circumpedal al centrului cercului înscris (I) în triunghiul ABC în raport cu acest triunghi şi , ,a b cd d d distanţele dintre punctele ', ', 'A B C respectiv

la dreptele BC,CA,AB şi , ,a b cr r r razele cercurilor exînscrisecorespunzătoare

triunghiului ABC. Atunci, 1a b c

a b c

d d d

r r r+ + = .

Demonstraţie. Deorarece 'A este mijlocul arcului BC , avem ' 'A B A C≡ , de unde rezultă proiecţia lui 'A pe BC este

punctul aM , mijlocul segmentului BC (Fig. 418). Din

teorema sinusurilor rezultă ' 2 sin ,2

=A

A B R

2' ' sin ' ' sin 2 sin2 2

= = = =a a a

A AM A A B A BM A B R d . Cum

2a

Ar p tg= ⋅ avem:

2 sinsin cos

2 2a

a

d R A A R A

r p p= ⋅ = , de

unde:sin sin sin

1a b c

a b c

d d d R A R B R C

r r r p

+ ++ + = = .

12) Fie ' ' 'A B C triunghiul circumpedal al centrului cercului înscris I în triunghiul ABC, ' ' , ' ' , ' ' , ' ' , ' ' , ' ' .N A B BC M AC BC P A B AC S AC AB R BC AB Q BC AC= ∩ = ∩ = ∩ = ∩ = ∩ = ∩ Dreptele MQ,NR şi PS sunt concurente. Demonstraţie. Patrulaterul 'ABA C fiind inscriptibil, din teorema lui Ptolemeu obţinem ' ' 'AA BC BA AC A C AB⋅ = ⋅ + ⋅ şi

cum ' 'BA A C≡ rezultă '

'

AA AB AC

BA BC

+= (1) (Fig. 419). Dacă

'D AA BC= ∩ din teorema bisectoarei în triunghiul ABD,

obţinem: AI AB

ID BD= . Cum

BC ABBD

AB AC

⋅=

+ rezultă

AI AB AC

ID BC

+= . Din teorema bisectoarei pentru triunghiul

'AA B avem: '

'

AS AA AB AC

BS BA BC

+= = (din relaţia 1), de unde

A

C

A'

B '

ad aM

I

Fig. 418

B

C'

A

C

A'

B '

M N D

I

Fig. 419

B

C' P

Q R

S

Page 407: Barbu_TEOREME_FUNDAMENTALE(2008).pdf

414

AI AS

ID BS= , deci .SI BC Analog IP BC , deci .I SP∈ La fel se arată că punctul I aparţine

dreptelor RN şi MQ. 13) Triunghiul circumpedal ' ' 'A B C al punctului lui Lemoine al triunghiului ABC are aceleaşi simediane ca triunghiul ABC. Demonstraţie. Vezi „Triunghiuri cosimediane”. 14) Triunghiul circumpedal ' ' 'A B C al punctului lui Lemoine al triunghiului ABC are laturile proporţionale cu medianele triunghiului ABC. Demonstraţie. Vezi „Triunghiuri cosimediane”.

15) Triunghiul circumpedal al unui punct al lui Brocard al triunghiului ABC este congruent cu triunghiul ABC. Demonstraţie. Fie ' ' 'A B C triunghiul circumpedal al punctuluiΩ al lui Brocard al triunghiului ABC (Fig. 420).

Deoarece ( ' ' ) ( ' ) ( ' )m C A A m C CA m B BC= = şi

( ' ' ) ( ' )m B A A m B BA= rezultă că ( ' ' ') ( )m C A B m CBA= , deci

' 'B C AC≡ . Analog se arată că ' 'C A AB≡ şi ' 'A B BC≡ , deci triunghiurile ABC şi ' ' 'C A B sunt congruente.

16) Fie 1 1 1A B C triunghiul cicumpedal al centrului cercului circumscris (O) al unui

triunghi ABC în raport cu acest triunghi, a b cM M M triunghiul median al triunghiului

ABC. Dacă punctele , ,a b cS S S împart segmentele orientate 1 1 1, ,a b cAM B M C Muuuuur uuuuur uuuuur

în acelaşi

raport \1;4 / 3k ∈ , atunci dreptele , ,a b cAS BS CS sunt concurente într-un punct situat pe dreapta lui Euler a triunghiului ABC. Demonstraţie. Considerăm un reper cartezian cu originea în O (Fig. 421).

Avem: 1 1( )

1 1 2a

a

OA kOM kOS OA OB OC

k k

− = = + + − −

uuur uuuuuruuuur uuur uuur uuur

. Fie un punct Q pe dreapta aAS

astfel încât .a

QAl

QS= Avem

1

1 aOQ OA lOSl = − −

uuur uuur uuuur,

1 1( )

1 (1 )( 1) 2

kOQ OA OA OB OC

l l k

= ⋅ − + + − − −

uuur uuur uuur uuur uuur,

1

(1 )(1 ) 2(1 )(1 ) 2(1 )(1 )

l k lk lkOQ OA OB OC

l k l k l k

+ −= ⋅ + ⋅ + ⋅

− − − − − −

uuur uuur uuur uuur.

Determinăm pe \1l∈ astfel încât OQuuur

să aibă o scriere

simetrică în raport cu vectorii , ,OA OB OCuuur uuur uuur

.

Atunci, 2(1 )l k lk+ − = , sau 2 2

2

kl

k

−=

− (cu 1l = dacă şi numai dacă 4 / 3k = ). Pentru

2 2

2

kl

k

−=

− obţinem: ( )

3 4

kOQ OA OB OC

k= + +

uuur uuur uuur uuur. Considerăm ' bQ BS∈ , '' cQ CS∈ care

împart vectorii corespunzători în acelaşi raport l, rezultă ' ''OQ OQ OQ= =uuuur uuuur uuur

, adică

A

C

A'

B ' Ω

Fig. 420

B

C'

A

C

1A

O aM

Fig. 421

B aS

Page 408: Barbu_TEOREME_FUNDAMENTALE(2008).pdf

415

' ",Q Q Q≡ ≡ deci există un punct comun dreptelor , ,a b cAS BS CS dacă

3

OA OB OCOG

+ +=

uuur uuur uuuruuur

rezultă că punctele Q,O şi G sunt coliniare.

Observaţii:

1) Dacă4

3k = dreptele , ,a b cAS BS CS sunt paralele cu dreapta lui Euler.

2) Dacă 2k = , atunci a b cS S S H≡ ≡ ≡ şi Q este ortocentrul triunghiului.

III.18. Triunghiul simedian

„Natura vorbeşte în limba matematicii: literele sunt cercurile, triunghiurile şi alte figuri geometrice.” – Galileo Galilei191

Triunghiul a b cK K K determinat de intersecţiile simedianelor cu laturile triunghiului ABC se

numeşte triunghi simedian. 1) Dacă K este punctul lui Lemoine al triunghiului ABC şi a b cK K K triunghiul simedian

al acestuia, atunci 2 2

2a

AK b c

KK a

+= .

Demonstraţie. Din teorema lui Van-Aubel

rezultă: c b

a c b

AK AKAK

KK K B K C= + , adică

2 2 2 2

2 2 2a

AK b c b c

KK b a a

+= + =

Observaţie: Prin permutări circulare se obţin

relaţiile: 2 2

2b

BK a b

KK b

+= şi

2 2

2c

CK b a

KK c

+= .

2) Aria triunghiului simedian a b cK K K este egală cu:

2 2 2

[ ] [ ]2 2 2 2 2 2

2

( )( )( )a b cK K K ABC

a b cA A

a b b c c a= ⋅

+ + +.

Demonstraţie. Avem 2 2

[ ]

2 2 2 2[ ]

sin

sin ( )( )a bAK K b c b c

ABC

A AK AK A AK AK b c

A AB AC A AB AC a b a c

⋅ ⋅ ⋅ −= = =

⋅ ⋅ ⋅ + + (1).

Analog, 2 2

[ ]

2 2 2 2[ ] ( )( )

a cBK K

ABC

A a b

A a c b c=

+ + (2) şi

2 2[ ]

2 2 2 2[ ] ( )( )

a bCK K

ABC

A a c

A a b b c=

+ + (3), iar

191 Galileo Galilei (1564-1642) – matematician, fizician, astronom şi filosof italian

A

B C

aM A' aK

Fig. 422

bK

cK cM

bM

K G

Page 409: Barbu_TEOREME_FUNDAMENTALE(2008).pdf

416

[ ] [ ] [ ] [ ] [ ]a b c b c a c a bK K K ABC AK K K BK K K CA A A A A−= − − (4). Din relaţiile (1), (2), (3) şi (4) rezultă

2 2 2

[ ] [ ]2 2 2 2 2 2

2

( )( )( )a b cK K K ABC

a b cA A

a b b c c a= ⋅

+ + +.

3) Aria triunghiului simedian ABC este maximă, atunci când triunghiul ABC este echilateral.

Demonstraţie. [ ]a b cK K KA este maximă, atunci când raportul 2 2 2

2 2 2 2 2 2

2

( )( )( )

a b c

a b b c c a+ + + este

maxim. Utilizând inegalitatea mediilor 2

x yx y

+≥ ⋅ pentru , 0x y ≥ obţinem:

2 2 2 2 2 2

2 2 2 2 2 2 2 2 2

2 2 1

4( )( )( ) 8

a b c a b c

a b b c c a a b c≤ =

+ + +. Egalitatea se obţine pentru a b c= = , dacă

triunghiul ABC este echilateral şi a b cK K K este triunghiul median al triunghiului ABC.

4) Coordonatele triliniare ale vârfurilor triunghiului simedian a b cK K K sunt: Ka(0, b, c), Kb(a, 0, c), Kc(a, b, 0). Demonstraţie. Vezi [26].

III.19. Triunghiul 60°∆

„În zadar vor matematicienii să ascundă: ei nu demonstrează, ci combină şi numai izbindu-se dintr-o parte în alta ajung la adevăr.”- Evariste Galois192

Notăm cu 60°∆ un triunghi ABC care are un unghi de 60 , ( ( ) 60 ), , ,a b cm A H H H° = °

proiecţiile vârfurilor A, B, C pe laturi; 1 1 1, ,A B C picioarele bisectoarelor interioare,

, ,a b cM M M mijloacele laturilor triunghiului ,ABC O centrul cercului circumscris

triunghiului ABC (Fig. 423).

1) Într-un triunghi 60°∆ avem: ( ) 60 .c a bm H M H = °

Demonstraţie. Deoarece ,2c a b a

BCH M H M

≡ =

rezultă că triunghiul c aH M B este

isoscel, deci ( ) 180 2 ( )c am H M B m B= °− , dar cum ( ) 180 2 ( )b a cm H H H m B= °− , avem

că c a b a cH M B H H H≡ adică patrulaterul c a a bH H M H este inscriptibil,atunci

( ) ( )c a b c a bm H M H m H H H= . Dar ( ) 180 2 ( ) ( ) 60b a cm H H H m A m A= °− = = ° , deci

( ) 60 .c a bm H M H = °

192 Evariste Galois (1811 – 1832) – matematician francez, contribuţii remarcabile în algebră

Page 410: Barbu_TEOREME_FUNDAMENTALE(2008).pdf

417

2) Triunghiurile ,a b c b cM H H AM H şi b cAH M sunt echilaterale.

Demonstraţie. Deoarece ( ) 60 ,c a bm H M H = ° iar 2c a b a

BCH M H M

≡ =

rezultă că

triunghiul a b cM H H este echilateral. Analog pentru celelalte două triunghiuri.

3) Proiecţia unei laturi care formează unghiul de 60° pe cealaltă este egală cu jumătate din latura proiectată. Demonstraţie. Triunghiurile ,b cAM H b cAH M fiind echilaterale, rezultă

,2c b

ACAH AM= = .

2b c

ABAH AM= =

4) Lungimea segmentului AH (H este ortocentrul 60 ABC°∆ ) este egală cu lungimea razei cercului circumscris triunghiului ABC. Demonstraţie: Avem 2 cos 2 cos60 .AH R A R R= = ° = 5) Patrulaterul 1 1B IC A este inscriptibil (I este centrul cercului înscris în 60 ABC°∆ ).

Demonstraţie. Avem ( ) ( ) ( )1 1

1.

2m B IC m ABC ACB m A= + =

6) Lungimea segmentului AI este egală cu diametrul cercului înscris triunghiului ABC.

Demonstraţie. Din sin sin302

Ar AI AI= = ° rezultă 2 .AI r=

A

B C Ma

Mb Mc

O

H

O9

Ha

Hb

Hc

Fig. 423

A1

B1

C1

I

aO

Page 411: Barbu_TEOREME_FUNDAMENTALE(2008).pdf

418

7) Punctele B, C, O, H, I, aI aparţin unui cerc simetric cercului circumscris triunghiului

60 ABC°∆ în raport cu latura BC (unde aI este centrul cercului A – exînscris).

Demonstraţie. Centrul cercului circumscris triunghiului BIC - punctul aO − se află pe

mijlocul arcului BC al cercului circumscris triunghiului ABC şi trece prin punctul .aI

Raza acestui cerc este egală cu R – raza cercului circumscris triunghiului 60 ABC°∆ -

(triunghiul aO BC fiind echilateral). Avem: a a aO B O C O O= = = a a aO I O I R= = , deci O

este punct pe cercul cu centrul în aO şi raza R. Deoarece ( ) 2 ( ) 120= = ° m BOC m A şi

( ) ( ) 180 ( ) 120= = ° − = ° b cm BHC m H HH m A rezultă că ≡BOC BHC , deci

patrulaterul BHOC este inscriptibil, deci şi punctul H aparţine cercului pe care se află punctele B,C,O,I şi aI .

8) Puntele A, ,b cH H , H, 9O ( 9O este centrul cercului lui Euler al triunghiului 60 ABC°∆ ) aparţin unui cerc de rază R/2. Demonstraţie. Punctele A, ,b cH H ,H aparţin cercului cu centrul în punctul 1M - mijlocul

segmentului AH – şi de rază. Deoarece 1 9 2

RM O = (linie mijlocie în triunghiul OAH )

rezultă că 9O aparţine şi el cercului de mai sus.

9) Patrulaterul aAHO O este romb.

Demonstraţie. Avem: .a aAH HO O O OA R= = = = Diagonalele rombului sunt bisectoarea

unghiului A şi dreapta lui Euler a 60 ABC°∆ ( aO fiind mijlocul arcului BC al cercului

circumscris ABC ).

Fig. 424

A

B

C

I

O H

1A

cH

bH

aM

cM

aH

aO

aI

Page 412: Barbu_TEOREME_FUNDAMENTALE(2008).pdf

419

10) Centrul cercului lui Euler al triunghiului 60 ABC°∆ este mijlocul segmentului .aAO

Demonstraţia rezultă din proprietatea precedentă. 11) Bisectoarea exterioară a unghiului A şi dreapta lui Euler a triunghiului

60 ABC°∆ sunt perpendiculare. Demonstraţia rezultă din proprietăţile precedente. 12) Triunghiul aOI H este isoscel.

Demonstraţie. Deoarece aAI OH⊥ , punctele , ,a aI O I sunt coliniare rezultă .a aI O I H≡

13) Triunghiul OIH este isoscel.

Demonstraţie. Deoarece I este mijlocul arcului OH în cercul circumscris rezultă OI=IH. 14) În triunghiul 60 ABC°∆ sunt adevărate relaţiile:

i) 2 2 2a b c bc= + − ; ii) ( ) 3p R r= + ; iii) 4 ( )bc r R r= + ; iv) 2 1 , = +

a

rh r

Runde

=a ah AH ; v) [ ] [ ]4b cABC AH H

A A= .

Demonstraţie.

i) Din teorema cosinusului în triunghiul 60 ABC°∆ rezultă 2 2 2 2 22 cosa b c bc A b c bc= + − = + −

ii) DeoareceOI IH= rezultă 2 2 2( 2 ) 4 4 3R R r R Rr r p− = + + − , de unde ( ) 3p R r= + .

iii) Avem: [ ]3

( ) 32 4a

ABC

h a bcA pr r R r

⋅= = = = + , de unde 4 ( )bc r R r= + .

iv) Din 3 3 4 ( )

2 4 sin 2 2a

bc bc bc r R rh

a R A R R

+= = = = rezultă 2 1 .a

rh r

R

= +

v) Din cos2c

bAH b A= = , cos

2b

cAH c A= = rezultă [ ]

sin 3

2 16b c

b c

AH H

AH AH A bcA

⋅ ⋅= = ,

deci [ ] [ ]4b cABC AH H

A A= .

14) Într-un triunghi 60°∆ triunghiul ortic, triunghiul de contact, triunghiul tangenţial şi

triunghiul antisuplementar a b cI I I sunt triunghiuri 60°∆ .

Demonstraţie. Deoarece ( ) 180 2 ( ) ( ) 60b a cm H H H m A m A= °− = = ° rezultă că triunghiul

ortic este un triunghi 60°∆ . Fie a b cC C C triunghiul de contact al triunghiului ABC. Avem

1( ) ( ) ( ) ( ) ( ) [ ( ) ( )] 60

2= + = + = + = ° c a b c a b a c bm C C C m C C I m C C I m C BI m C CI m B m C

deci triunghiul a b cC C C este un triunghi 60°∆ . Deoarece triunghiurile ortic ( a b cH H H ) şi

tangenţial ( A B CT T T ) ale unui triunghi sunt omotetice, rezultă că

( ) 180 2 ( ) ( ) 60= °− = = °B A Cm T T T m A m A , deci A B CT T T este un triunghi 60°∆ În triunghiul

Page 413: Barbu_TEOREME_FUNDAMENTALE(2008).pdf

420

antisuplementar a b cI I I avem: ( ) 180 [ ( ) ( )]= °− + = b a c a am I I I m I BC m I CB

1 1180 90 ( ) 90 ( ) 60

2 2

° − ° − + ° − = °

m B m C deci a b cI I I este triunghi 60°∆ .

Observaţie: Triunghiul 60°∆ se mai numeşte triunghi semiregulat în unghiuri.

15) Fie triunghiul 60°∆ , H ortocentrul său. Mediatoarele segmentelor BH, CH intersectează dreptele AB, respectiv, AC în punctele M şi N. Punctele M, H şi N sunt coliniare. Demonstraţie. Fie 1M şi 1N mijloacele segmentelor BH, respectiv CH. Deoarece:

( ) 60 ,= ° cm BHH iar triunghiul MBH fiind isoscel rezultă

( ) ( ) 30= = ° m MBH m MHB , de unde ( ) 30= ° cm MHH , adică punctul M aparţine

bisectoarei unghiului cH HB . Analog se arată că punctul N aparţine bisectoarei unghiului

,bH HC de unde rezultă concluzia.

III.20. Triunghiul medianelor

„Geometria este arta de a judeca pe desene rău efectuate.” - Niels H. Abel193

1) Să se arate că se poate construi un triunghi având laturile de lungimi egale cu lungimile medianelor unui triunghi ABC . Demonstraţie. Fie aAM , bBM , cCM medianele

triunghiului ABC şi G centrul său de greutate. Prin punctele C şi cM ducem paralele la medianele bBM ,

respectiv aAM şi fie 'C punctul de intersecţie dintre

aceste paralele, iar 'aD AM CC= ∩ ,

'cE M C BC= ∩ (Fig. 425). Evident a aGM M D≡ , şi

deoarece 'cM C GD , avem 2

' 3c c

GC GD

MC MC= = , de unde

rezultă 3 3 3 2

'2 2 2 3c a aM C GD AG AM AM= = = ⋅ ⋅ = (1).

Deoarece aM este mijlocul segmentului GD şi

'cM C GD , rezultă că CE este mediană în triunghiul

'cM CC . Din 1

2c a

a

M G EM

GC M C= = rezultă că aM este centrul de greutate al triunghiului

'cM CC . Din congruenţa triunghiurilor aBGM şi aCDM rezultă că BG CG≡ , de unde

193 Niels H. Abel (1802-1829) – matematician norvegian, contribuţii fundamentale în algebră

aM

Fig. 425

A

B C

bM

cM

E

G

D C'

Page 414: Barbu_TEOREME_FUNDAMENTALE(2008).pdf

421

2 2'

3 3bMM CC= , adică 'bMM CC≡ (2). Din relaţiile (1) şi (2) rezultă că triunghiul

'cM CC are laturile de lungimi egale cu cele ale medianelor triunghiului ABC .

Observaţie : Triunghiul având laturile de lungimi egale cu lungimile medianelor unui triunghi ABC se numeşte triunghiul medianelor corespunzător triunghiului ABC . 2) Dacă a , b , c sunt lungimile laturilor triunghiului ABC , atunci medianele triunghiului medianelor corespunzător triunghiului ABC au lungimile egale cu : 3 / 4a , 3 / 4b , respectiv 3 / 4c .

Demonstraţie. Din aplicaţia precedentă rezultă 3 3

2 4aCE CM BC= = ,

' 3 3

2 4c c a cM M M M AC= = şi 3

' ''4

C C AB= , (unde 'c

M şi ''C sunt mijloacele laturilor

'CC , respectiv cM C ).

3) Unghiurile triunghiului medianelor au măsurile egale cu suplementele coordonatelor unghiulare ale centrului de greutate al triunghiului ABC . Demonstraţie. Deoarece ' bCC BM şi ' c aC M AM rezultă că

( ' ) ( ) 180 ( )c am M C D m M GB m AGB= = −o şi ( ' ) ( ) 180 ( )c am C M C m M GC m AGC= = −o ;

( ') 180 ( )cm M CC m BGC= −o .

III.21. Triunghiuri omologice Triunghiurile ABC şi ' ' 'A B C se numesc omologice dacă dreptele 'AA , 'BB şi 'CC sunt concurente. Punctul O de concurenţă al dreptelor 'AA , 'BB şi 'CC se numeşte centrul de omologie al triunghiurilor ABC şi

' ' 'A B C ; vârfurile A şi 'A , B şi 'B , C şi 'C se numesc omoloage, iar dreptele BC şi ' 'B C , CA şi ' 'C A , AB şi ' 'A B se numesc

omoloage. Fie ' '= ∩L BC B C ,

' '= ∩M AC A C şi ' '= ∩N AB A B .

Triunghiurile date sunt triomologice dacă admit trei centre de omologie. 1) Punctele de intersecţie ale dreptelor omologe, a două triunghiuri omologe coplanare, sunt coliniare. Demonstraţie. Vezi „Teorema lui Desargues”.

M

A

B

C

L

C'

B'

A' N

Fig. 426

O

Page 415: Barbu_TEOREME_FUNDAMENTALE(2008).pdf

422

Dreapta ce conţine punctele , ,L M N se numeşte axă de omologie. Dacă triunghiul ' ' 'A B C este înscris în triunghiul ABC, atunci dreapta LMN se numeşte polară triliniară,

iar O se numeşte pol triliniar.

Observaţie: Dreapta ce uneşte picioarele bisectoarelor exterioare este polara triliniară a centrului cercului înscris. Teorema lui Casey

2) Fie triunghiurile ABC, ' ' 'A B C , " " "A B C două câte două omologice, O centrul comun de omologie ' 'M AB A B= ∩ , ' ' 'M AC A C= ∩ , " "N AB A B= ∩ ,

' " "N AC A C= ∩ , ' ' " "P A B A B= ∩ , ' ' ' " ".P A C A C= ∩ Dreptele ', 'MM NN şi

'PP sunt concurente.

M

A

B

C

M'

C'

B'

A'

N

Fig. 427

O

A"

B"

C"

P

P '

N'

Page 416: Barbu_TEOREME_FUNDAMENTALE(2008).pdf

423

Demonstraţie. Arătăm că triunghiurile MNR şi ' ' 'M N P sunt omologice cu ajutorul reciprocei teoremei lui Desargues. Astfel, deoarece

' ' , ' ' ',MN M N A PM P M A∩ = ∩ = şi ' ' "NP N P A∩ = , iar punctele , ', "A A A

sunt coliniare, rezultă că dreptele ', 'MM NN şi 'PP sunt concurente. 3) Fie triunghiurile ABC şi ' ' 'A B C cu proprietatea că există punctele , ,L M N astfel încât ' '= ∩L BC B C , ' '= ∩M AC A C şi ' '= ∩N AB A B , iar dreptele 'AA şi 'BB

nu sunt paralele. Dacă punctele ,L M şi N sunt coliniare, atunci dreptele 'AA , 'BB şi 'CC sunt concurente.

Demonstraţie. Vezi „Teorema lui Desargues”.

III.22. Triunghiuri ortopolare „Moisil a fost mai mult decât un savant, a fost mai mulţi savanţi întruniţi în sesiune permanentă sau luându-şi locul unul altuia în cicluri succesive mari, reprezentate de temele fundamentale pe care le-a abordat. A fost până în ultimele zile deschizător de drumuri, inovator. În această aventură spirituală nu a admis dilentatismul superficial.” – Mircea Maliţa

Teorema lui Lalescu194 1) Fie ABC şi ' ' 'A B C două triunghiuri înscrise în acelaşi cerc. Dacă dreapta Simson a vârfului 'A în raport cu ABC este perpendiculară pe dreapta ' 'B C atunci : i) această proprietate este valabilă pentru toate vârfurile triunghiului ' ' 'A B C ; ii) Dreptele Simson ale vârfurilor triunghiului ABC în raport cu triunghiul ' ' 'A B C sunt perpendiculare pe laturile triunghiului .ABC

Demonstraţie. i) Fie N proiecţia lui 'A pe BC şi A" al doilea punct de intersecţie dintre

'A N cu cercul circumscris triunghiului .ABC Cum dreapta lui Simson ( A 's ) a

punctului 'A în raport cu triunghiul ABC este paralelă cu "AA , atunci pentru ca

' ' '⊥As B C trebuie ca ' ' "B C AA⊥ şi fie

' ' ",= ∩E B C AA ' ' .= ∩D B C BC

Patrulaterul "A EDN este inscriptibil şi atunci " ' ' ,AA A B DB≡ deci

( ') ( ' ) ( ')= +m AA m B B m CC . Egalitatea

precedentă este echivalentă cu: ( ') ( ' ) ( ') 0(mod360 )+ + = °m AA m B B m CC

(1) (unde am evaluat măsurile arcelor în sens trigonometric). Proprietatea i) este verificată pentru toate vârfurile triunghiului ' ' 'A B C deoarece permutarea ciclică a cuplurilor ( , '), ( , '), ( , ')A A B B C C nu modifică relaţia (1). ii)

Relaţia (1) nu se modifică dacă se inversează tripletele ( , , )A B C şi ( ', ', ')A B C , atunci

194 Traian Lalescu (1882-1929) – matematician român, contribuţii importante în geometrie

A B

C

A'

C'

B'

A" D

N

E

H

Fig. 428

Page 417: Barbu_TEOREME_FUNDAMENTALE(2008).pdf

424

dreptele lui Simson ale triunghiului ABC în raport cu triunghiul ' ' 'A B C sunt perpendiculare pe laturile triunghiului .ABC Triunghiurile ABC şi ' ' 'A B C cu proprietatea de mai sus se numesc triunghiuri S195 (sau ortopolare) unul în raport cu altul. 2) Triunghiurile ABC şi ' ' 'A B C sunt triunghiuri S dacă şi numai dacă

( ') ( ' ) ( ') 0(mod360 )+ + = °m AA m B B m CC .

Demonstraţia rezultă din teorema lui Lalescu. 3) Dacă ABC şi ' ' 'A B C sunt două triunghiuri S, atunci dreptele lui Simson ale vârfurilor triunghiului ' ' 'A B C în raport cu triunghiul ABC şi ale vârfurilor triunghiului ABC în raport cu triunghiul ' ' 'A B C trec prin acelaşi punct care se află la mijlocul segmentului ce uneşte ortocentrele celor două triunghiuri. Demonstraţie. Fie H şi 'H ortocentrele triunghiurilor ABC, respectiv ' ' 'A B C ; M, N şi P

mijloacele segmentelor ', ',HA HB respectiv '.HC Triunghiurile MNP şi ' ' 'A B C au

laturile paralele. Deoarece dreptele lui Simson ale punctelor ', ', 'A B C în raport cu triunghiul ABC sunt perpendiculare pe

' ', ' 'B C A C respectiv ' 'A B şi ele trec prin punctele M,N, respectiv P (vezi „Dreapta lui Simson”). Dreptele lui Simson ale punctelor

', ', 'A B C în raport cu triunghiul ABC sunt înălţimile triunghiului MNP, deci concurente în ortocentrul acestuia, punct care este mijlocul segmentului '.HH Schimbând rolurile triunghiurilor ABC şi ' ' 'A B C rezultă că dreptele Simson ale vârfurilor triunghiul

' ' 'A B C în raport cu triunghiul ABC sunt concurente tot în mijlocul segmentului '.HH

4) Consecinţă: Dacă H şi 'H sunt ortocentrele triunghiurilor S , ABC şi ' ' 'A B C ,

atunci triunghiul omotetic triunghiului ABC prin omotetia H ( ',1/ 2)H şi triunghiul

omotetic triunghiului ' ' 'A B C prin omotetia H ( ,1/ 2)H au acelaşi ortocentru în punctul

comun al dreptelor lui Simson. 5) Fie ( , )ABC αβγ şi ( , ' ' ')ABC α β γ două perechi de triunghiuri S înscrise în acelaşi

cerc. Triunghiurile αβγ şi ' ' 'α β γ sunt triunghiuri S unul faţă de celălalt.

Demonstraţie: Deoarece triunghiurile ABC şi αβγ sunt triunghiuri S rezultă :

( ) ( ) ( ) 0(mod360 ) (1)+ + = °m A m B m Cα β γ , iar din faptul că triunghiurile ABC şi

' ' 'α β γ sunt triunghiuri S rezultă ( ') ( ') ( ') 0(mod360 ) (2).+ + = °m A m B m Cα β γ Din (1) şi

2 Denumirea a fost dată de Traian Lalescu

B

A

C

C' A'A

B'

H

P M

N

A 's

Fig. 429

Page 418: Barbu_TEOREME_FUNDAMENTALE(2008).pdf

425

(2) rezultă ( ') ( ') ( ') 0(mod360 )+ + = °m m mαα ββ γγ , adică triunghirile αβγ şi ' ' 'α β γ

sunt triunghiuri S. 6) Consecinţă : În cercul circumscris unui triunghi ABC se pot înscrie o infinitate de triunghiuri S în raport cu triunghiul ABC . Observaţie: Toate aceste triunghiuri înscrise în cercul circumscris triunghiului ABC , împreună cu triunghiul ABC determină o familie de triunghiuri S . 7) Dreapta lui Simson a unui punct M în raport cu triunghiurile αβγ înscrise în acelaşi

cerc cu triunghiul ABC , faţă de care sunt triunghiuri S, păstrează o direcţie fixă. Demonstraţie. Considerăm o coardă NP în cercul circumscris triunghiului ABC, astfel încât triunghiul MNP este un triunghi S faţă de familia de triunghiuri considerată. Atunci, dreptele lui Simson ale punctului M în raport cu triunghiurile considerate sunt perpendiculare pe dreapta NP. Observaţie. Proprietatea precedentă poate fi reformulată astfel: dreptele lui Simson ale unui punct M în raport cu triunghiurile S sunt paralele între ele.

8) Două triunghiuri înscrise în acelaşi cerc care au un vârf comun şi laturile opuse paralele, sunt triunghiuri S. Demonstraţie. Fie A vârful comun triunghiurilor ABC şi ' '.AB C Deoarece

' 'B C BC rezultă ( ' ) ( ' )=m B B m C C , atunci ( ' ) ( ' ) 0+ =m B B m C C unde arcele 'B B şi

'C C au fost măsurate în sens trigonometric. Conform relaţiei (1) rezultă că triunghiurile ABC şi ' 'AB C sunt triunghiuri S. 9) Consecinţă: Dacă unuia dintre triunghiurile unei familii S deplasăm o latură paralelă cu ea însăşi, triunghiurile rămân, de asemenea, triunghiuri S . Observaţie: Din cele de mai sus se pot deduce diferite procedee de construcţie a unui triunghi S în raport cu un triunghi dat, când i se cunosc două vârfuri. Astfel, fie triunghiul ABC şi punctele 'B şi 'C pe cercul circumscris triunghiului ABC . i) Construim dreapta lui Simson 'Bd a punctului 'B în raport cu triunghiul ABC . Punctul

de intersecţie dintre perpendiculara dusă din 'C pe 'Bd şi cercul circumscris triunghiului

ABC este cel de-al treilea vârf al triunghiului cerut (Fig. 430). ii) Prin vârful A al triunghiului ABC construim o paralelă la dreapta ' 'B C care

intersectează din nou cercul în *A . Punctul de intersecţie dintre paralela dusă din ∗A la BC cu cercul circumscris triunghiului ABC este vârful 'A al triunghiului căutat (Fig. 431).

Page 419: Barbu_TEOREME_FUNDAMENTALE(2008).pdf

426

iii) Prin vârful A al triunghiului ABC construim perpendiculara pe coarda ' 'B C care

intersectează cercul circumscris triunghiului ABC în *A . Perpendiculara din *A pe BC intersectează cercul în 'A -punctul căutat (Fig. 432).

10) Triunghiurile ortic a b cH H H şi median a b cM M M ale unui triunghi sunt triunghiuri

S în cercul lui Euler al triunghiului ABC . Demonstraţie. Perpendiculara din punctul aM

pe latura b cH H a triunghiului ortic intersectează

a doua oară cercul lui Euler al triunghiului ABC în punctul eulerian 'A (mijlocul segmentului AH) (Fig. 433). Întrucât perpendiculara din 'A pe latura b cM M a triunghiului median cade în

punctul aH rezultă - conform observaţiilor

precedente - că triunghiurile median şi ortic ale triunghiului ABC sunt triunghiuri S în cercul lui Euler al triunghiului ABC . 11) Mijlocul segmentului ce uneşte ortocentrele triunghiurilor median şi ortic ale unui triunghi ABC este punctul lui Spieker al triunghiului ortic. Demonstraţie. Deoarece triunghiurile a b cH H H şi a b cM M M sunt triunghiuri S vom arăta

că punctul lui Spieker al triunghiului ortic este punctul de întâlnire al dreptelor lui Simson ale vârfurilor triunghiului a b cM M M în raport cu triunghiul a b cH H H (Fig.433). Segmentul

' aA M este diametru în cercul lui Euler , ' ⊥a b cA M H H şi fie 1 '= ∩b c aA H H A M , 1A

fiind mijlocul segmentului b cH H (vezi “Cercul lui Euler”), deci dreapta lui Simson aM

d a

punctului aM în raport cu triunghiul a b cH H H trece prin 1A . Deoarece a b cH H H şi

a b cM M M sunt triunghiuri S, rezultă că ⊥aM b cd M M , deci

aM ad AH⊥ . Fie 1B şi 1C

mijloacele segmentelor a cH H , respectiv a bH H . Cum ' aA H este bisectoarea unghiului

c a bH H H şi 1 1 a bB A H H , 1 1 a cAC H H rezultă că aM

d este bisectoarea unghiului

A' A

B C

B'

C'

Fig. 430

B 'd

A

B C

A'

B'

C'

A∗

Fig. 431

A

B C

A'

B'

C'

A∗ Fig. 432

A

B C aM

bM cM

aH

bH

cH

Fig. 433

A' 1A

1B

aMd

Page 420: Barbu_TEOREME_FUNDAMENTALE(2008).pdf

427

1 1 1B AC . Analog se arată că dreapta lui Simson bM

d a punctului bM este bisectoarea

unghiului 1 1 1A B C , deci punctul de concurenţă al dreptelor lui Simson - mijlocul

segmentului ce uneşte ortocentrele triunghiurilor S , a b cH H H şi a b cM M M - este punctul

lui Spieker al triunghiului ortic a b cH H H .

12) Triunghiul ' ' 'A B C , determinat de punctele euleriene ale triunghiului ABC şi triunghiul " " "A B C , având ca vârfuri punctele unde mediatoarele triunghiului ABC intersectează a doua oară cercul lui Euler al triunghiului ABC , sunt două triunghiuri S . Demonstraţie. Dacă a b cO O O este triunghiul lui Carnot al triunghiului ABC , atunci

triunghiul ' ' 'A B C este triunghiul median al triunghiului a b cO O O , iar triunghiul " " "A B C

este triunghiul ortic al aceluiaşi triunghi (vezi „Triunghiul Carnot”) şi conform proprietăţilor precedente, rezultă că triunghiurile ' ' 'A B C şi " " "A B C sunt triunghiuri S . 13) Triunghiurile ABC şi ' ' 'A B C sunt ortopolare dacă şi numai dacă ' ' 'abc a b c= (unde cu x am notat afixul punctului X). Demonstraţie. Fie P,Q,R picioarele perpendicularelor duse din punctul 'A pe dreptele BC,CA, respectiv AB. Lemă. Fie X un punct în interiorul unui triunghi ABC şi P proiecţia lui X pe BC. Afixul

punctului P este egal cu 2

1

2

bcp x x b c

R

= − + +

, unde cu , , ,a b c x am notat afixele

punctelor A,B,C, respectiv X, iar R este raza cercului circumscris triunghiului ABC.

Demonstraţie. Ecuaţiile dreptelor BC şi XP sunt: ( )( ) ( )( ) 0− − − − − =z b c b z b c b ,

respectiv ( )( ) ( )( ) 0− − + − − =z x c b z x c b . Deoarece punctul P aparţine dreptelor BC şi

XP rezultă (2 )( ) ( )( ) 0,− − − + − − =p b x c b b x c b de unde

2 2

1 1( ) ( )

2 2

− − = + + − = + − − −

c b c bp b x x b b x x b

R Rc b

c b

sau 2 2

1 1( )

2 2 = + − − = − ⋅ + +

bc bcp b x x b x x b c

R R.

Demonstraţia teoremei. Avem: 2 2

1 1' ' , ' ' ,

2 2 = − + + = − + +

bc cap a a b c q a a c a

R R

2

1' '

2 = − + +

abr a a a b

R. Dacă 'A nu coincide cu un vârf al triunghiului ABC, atunci

condiţia de perpendicularitate dintre PQ şi ' 'B C este echivalentă cu

( )( ' ') ( )( ' ') 0− − + − − =p q b c p q b c sau 2 2( )( ')( ' ') ( )( ')( ' ) 0− − − + − − ⋅ − =b a R ca b c b a R c a b c

sau mai departe ( )2 2 2 2 2 2

2 2' ' ' ( ) 0' ' '

− − − + − − − =

R R R R R RR a b c b a R c

b a c a b c, de

unde rezultă că ( ' ' ')( )( ')( ' ') 0,− − − − =abc a b c a b c a b c deci ' ' 'abc a b c= . Dacă 'A

coincide - de exemplu - cu vârful A, atunci dreapta lui Simson a lui 'A este perpendiculara din A pe BC. Dreptele 'A P şi ' 'B C sunt perpendiculare dacă şi numai dacă ' 'BC B C ,

adică ' '=bc b c sau ' ' '=abc a b c (deoarece '=a a ).

Page 421: Barbu_TEOREME_FUNDAMENTALE(2008).pdf

428

III.23. Triunghiuri ortologice

„Orice număr este zero înaintea infinitului.” – Victor Hugo196

Triunghiul ABC este ortologic cu triunghiul ' ' 'A B C dacă perpendicularele din A, B, C pe ' 'B C , ' 'C A respectiv ' 'A B sunt concurente. Punctul de concurenţă se numeşte centrul acestei ortologii. 1) Dacă triunghiul ABC este ortologic cu triunghiul ' ' 'A B C , atunci şi triunghiul

' ' 'A B C este ortologic cu triunghiul .ABC

Demonstraţie. Soluţia 1.Fie D, E, F proiecţiile punctelor A, B, şi C pe laturile ' 'B C ,

' 'C A , respectiv ' 'A B (Fig. 434). Avem: 2 2 2 2 2' ' ' '− = − =B A B D C A D C D A , 2 2 2 2 2' ' ' '− = − =C B C E A B A E B E

2 2 2 2 2' ' ' '− = − =A C A F B C B F CF . Din relaţiile precedente prin sumare obţinem:

2 2 2 2 2 2( ' ' ' ) ( ' ' ' )+ + − + + =B A C B A C B D C E A F 2 2 2 2 2 2( ' ' ' ) ( ' ' ' ) ( )+ + − + + ∗A B B C C A A E B F C D

Din teorema lui Carnot rezultă că 2 2 2 2 2 2' ' ' ' ' 'B D C E A F A E B F C D+ + = + + .

Relaţia ( )∗ devine: 2 2 2 2 2 2' ' ' ' ' '+ + = + +B A C B A C A B B C C A ( )∗∗ . Condiţia ca

triunghiul ABC să fie ortologic cu triunghiul ' ' 'A B C este dat de relaţia ( )∗∗ . Pentru ca

triunghiul ' ' 'A B C să fie ortologic cu triunghiul ABC este suficient să permutăm în relaţia ( )∗∗ rolurile triunghiului ABC cu cel al lui ' ' 'A B C , prin această permutare se obţine tot

relaţia ( )∗∗ , ceea ce arată că şi triunghiul ' ' 'A B C este ortologic cu triunghiul ABC .

Soluţia 2. Notăm cu P punctul de concurenţă al perpendicularelor din A, B, C pe laturile triunghiului ' ' 'A B C şi cu 'P punctul de intersecţie al perpendicularelor din 'A şi 'B pe

BC, respectiv AC. Avem: ' ' ' ' ' ', ' ' ' ' ' ', ' ' ' ' ' 'B C P C P B C A P A P C A B P B P A= − = − = −uuuuur uuuuur uuuuur uuuuur uuuuur uuuuur uuuuur uuuuur uuuuur

.

Folosind aceste relaţii obţinem 0 ' ' ' ' ' '= ⋅ + ⋅ + ⋅ =uuur uuuuur uuur uuuuur uuur uuuuurPA B C PB C A PC A B

' ' ' ' ' '⋅ + ⋅ + ⋅uuuuur uuur uuuuur uuur uuuuur uuurP C BA P B AC P A BC . Cum ' 'P B AC⊥ şi ' 'P A BC⊥ , avem ' ' 0P B AC⋅ =

uuuuur uuur ,

' ' 0P A BC⋅ =uuuuur uuur

, deci ' ' 0P C BA⋅ =uuuuur uuur

, de unde rezultă ' ' .P C AB⊥ Prin urmare perpendicularele din ', ', 'A B C pe laturile triunghiului ABC sunt concurente în 'P . Două triunghiuri care satisfac proprietăţile de mai sus se numesc ortologice. Fie P şi 'P cele două centre de ortologie ale triunghiurilor ABC şi ' ' 'A B C . Dacă centrele de ortologie P şi 'P coincid, atunci triunghiurile se numesc bilogice. 2) Două triunghiuri ABC şi ' ' 'A B C sunt ortologice dacă şi numai dacă este adevărată

relaţia: 2 2 2 2 2 2' ' ' ' ' '+ + = + +B A C B A C A B B C C A .

Demonstraţia rezultă de mai sus.

196 Victor Hugo (1802-1885) – scriitor francez

A

B C

A'

B' C' D

E F

Fig. 434

P ' P

Page 422: Barbu_TEOREME_FUNDAMENTALE(2008).pdf

429

3) Triunghiul ABC şi triunghiul podar al unui punct P sunt ortologice, centrele de ortologie fiind P şi izogonalul său 'P . Demonstraţie.

Triunghiul ' ' 'A B C este ortologic cu triunghiul ABC, centrul acestei ortologii fiind P. (Fig. 435). Conform teoremei de ortologie şi triunghiul ABC este ortologic cu triunghiul

' ' 'A B C , centrul acestei ortologii fie că este punctul '.P Patrulaterul ' 'BA PC fiind inscriptibil rezultă: ( ') 9 0 ( ' ' ) 9 0 ( ')= ° − = ° − = m ABP m A C B m BPA

( ') ( )m PBA m PBC= , deci 'ABP P BC≡ adică dreptele BP şi 'BP sunt

izogonale. Analog se arată că dreptele AP şi 'AP sunt izogonale, deci punctele P şi 'P sunt izogonale. 4) Triunghiul antipodar al unui punct P este ortologic cu triunghiul cevian al punctului P în raport cu triunghiul ABC. Demonstraţie. Vezi „Triunghiul antipodar”. 5) Triunghiul antisuplementar a b cI I I şi triunghiul cevian al centrului cercului înscris într-un triunghi ABC sunt ortologice. Demonstraţie. Vezi „Dreapta lui Euler”. 6) Dacă triunghiurile ABC şi ' ' 'A B C sunt ortologice având centrele de ortologie P şi

'P , atunci coordonatele baricentrice ale lui P în raport cu triunghiul ABC sunt egale cu coordonatele baricentrice ale lui 'P în raport cu triunghiul ' ' 'A B C . Demonstraţie. Deoarece ' ' ⊥A P BC , ' ' ⊥B P AC , ' ' ⊥C P AB rezultă: sin ' ' ' sin=B P C A , sin ' ' ' sin=P B C PAC şi sin ' ' ' sin=P C B PAB (Fig. 435). Din teorema

sinusurilor în triunghiul ' ' 'P B C rezultă

' ' ' 'sin ' ' ' sin ' ' '

=P B P C

P C B P B C şi

' ' ' 'sin sin

=P B P C

PAB PAC

de unde :

sin ' '' 'sin

=PAC P C

P BPAB (1). Atunci, [ ]

[ ]

sin sinsin sin

⋅ ⋅= = ⋅⋅ ⋅

PAC

PAB

A b PA PAC b PACcA c PA PAB PAB

(2).Din

relaţiile (1) şi (2) rezultă [ ]

[ ]:' ' ' ' =

PAC

PAB

Ab cP B P C A

(3). Analog se arată că [ ]

[ ]:' ' ' ' =

PBC

PCA

Aa bP A P B A

(4). Relaţiile (3) şi (4) dau coordonatele baricentrice ale punctului 'P în raport cu

A

B C

PP

P '

A'

B'

C'

Fig. 435

Page 423: Barbu_TEOREME_FUNDAMENTALE(2008).pdf

430

triunghiul ABC : [ ] [ ] [ ]' ' ' ' ' ' ' ' ': : ' ' ' ' sin : ' ' ' ' sin : ' ' ' ' sin = = ⋅ ⋅ ⋅ ⋅ ⋅ ⋅P B C P C A P A BA A A P B P C A P C P A B P A P B C

[ ] [ ] [ ]: : : :' ' ' ' ' ' = PBC PAC PAB

a b cA A A

P A P B P C.

Observaţie: Proprietatea de mai sus ne arată că dacă P este centrul de greutate al triunghiului ABC , atunci 'P este centrul de greutate al triunghiului ' ' 'A B C . 7) Triunghiul ortic a b cH H H al triunghiului nedreptunghic ABC şi triunghiului ABC

sunt ortologice, centrele de ortologie fiind ortocentrul şi centrul cercului circumscris triunghiului ABC . Demonstraţie. Perpendicularele din vârfurile A, B, C ale triunghiului nedreptunghic pe laturile ' 'B C , ' 'C A respectiv ' 'A B sunt concurente în centrul cercului circumscris triunghiului ABC (vezi „Triunghiul ortic”). 8) Fie a b cH H H triunghiul ortic al triunghiului ABC, ' ' ', ,

a b cH H H punctele diametral

opuse punctelor , ,a bH H respectiv cH în cercul lui Euler al triunghiului ABC.

Triunghiurile ABC şi ' ' 'a b cH H H sunt ortologice.

Demonstraţie. Deoarece triunghiurile a b cH H H şi ' ' 'a b cH H H sunt omotetice, rezultă -

conform proprietăţii precedente - ' 'c bOA H H⊥ , ' '

a cOB H H⊥ şi ' 'a bOC H H⊥ , adică

triunghiurile ABC şi ' ' 'a b cH H H sunt ortologice , centrul cercului circumscris triunghiului

ABC fiind unul dintre centrele de ortologie. 9) Două triunghiuri ABC şi ' ' 'A B C simetrice faţă de o dreaptă sunt ortologice. Demonstraţie. Deoarece ' ',≡AB BA ' '≡BC CB şi ' '≡CA AC , relaţia

2 2 2 2 2 2' ' ' ' ' '+ + = + +B A C B A C A B B C C A este evident adevărată, deci triunghiurile ABC şi ' ' 'A B C sunt ortologice. 10) Fie 'A , 'B , 'C simetricele vârfurilor A, B, C ale unui triunghi ABC faţă de dreapta lui Euler a triunghiului ABC . Triunghiurile ABC şi ' ' 'A B C sunt ortologice. Demonstraţia rezultă din proprietatea precedentă.

Page 424: Barbu_TEOREME_FUNDAMENTALE(2008).pdf

431

III.24. Triunghiuri paralogice

„Uşor nu e nici cântecul. Zi şi noapte – nimic nu-i uşor pe pământ

căci roua e sudoarea privighetorilor ce s-au ostenit toată noaptea cântând.”

Lucian Blaga 197 Fie , ,M N P punctele de intersecţie dintre dreapta d cu laturile , ,BC CA AB ale unui triunghi ABC . Punctele de intersecţie dintre perpendicularele ridicate în punctele , ,M N P pe dreptele , ,BC CA AB sunt vârfurile unui triunghi ' ' 'A B C numit triunghiul paralogic (ortoomologic) al triunghiului ABC . 1) Triunghiul paralogic ' ' 'A B C şi triunghiul ABC sunt asemenea. Demonstraţie. Patrulaterul 'PMB P este inscriptibil,deci ( ' ) ( )= m PB M m ABC ,

adică ' ' 'A B C ABC≡ . Patrulaterul 'APA N este inscriptibil, deci ( ' ' ) ( )=m B A N m PAN ,deci ' ' ' ≡B A C BAC

(2). Din relaţiile (1) şi (2) rezultă că triunghiurile ' ' 'A B C şi ABC sunt asemenea (Fig. 436). 2) Triunghiurile ' ' 'A B C şi ABC sunt omologice, dreapta d fiind axa de omologie. Demonstraţie.Deoarece ' '= ∩P AB A B ,

' ',= ∩M BC B C ' '= ∩N AC A C iar

punctele , ,M N P sunt coliniare, rezultă din teorema lui Desargues că dreptele ', ', 'AA BB CC sunt concurente, deci triunghiurile ' ' 'A B C şi ABC sunt omologice, dreapta d fiind axa de omologie 3) Centrul de omologie (τ ) dintre triunghiul ABC şi triunghiul paralogic ' ' 'A B C aparţine cercurilor circumscrise triunghiurilor ABC şi

' ' 'A B C . Demonstraţie. Avem ' ' '= ∩ ∩AA BB CCτ

(Fig. 437). Patrulaterele 'PB MB şi 'B MNC fiind inscriptibile: ' ' 'NM C NCC A C τ≡ ≡ şi 'PMB PB B≡ , iar cum PMB NMC≡ (unghiuri opuse la vârf) avem:

' ' ' '≡ C A A Bτ τ , adică patrulaterul ' ' 'A B Cτ este inscriptibil (1). Atunci

197 Lucian Blaga (1895-1961) - filozof, umanist, jurnalist, poet, dramaturg, traducător, profesor universitar şi

diplomat român, membru titular al Academiei Române

A

B C

P A'

B'

C'

Fig. 436

N

M

d

A

B C

A'

O

B'

C'

P

M

N

τ ζ

Fig. 437

T

T '

D

Page 425: Barbu_TEOREME_FUNDAMENTALE(2008).pdf

432

' ' ' ' 'A B A C B BCA B Aτ τ≡ ≡ ≡ , deci patrulaterul B CAτ este inscriptibil (2). Din relaţiile (1) şi (2) rezultă că τ aparţine cercurilor circumscrise triunghiurilor ABC şi

' ' 'A B C . 4) Cercurile circumscrise triunghiurilor ' ' 'A B C , ABC sunt ortogonale.

Demonstraţie. Fie C şi C’ cercurile circumscrise triunghiurilor ABC, respectiv ' ' 'A B C , iar

Tτ şi 'T τ tangentele în punctul τ (centrul omologiei) la C respectiv C’ (Fig. 437). Atunci

( ) ( ),= m T B m BAτ τ ( ' ) ( ' ' ) ( ' ') ( ' ' ) ( ' ')= + + = m PA m A B m T A m A B m T Bτ τ τ τ τ , de

unde rezultă că: ( ' ) 180 [ ( ) ( ' ')]= ° − + = m T T m T B m T Bτ τ τ

180 [ ( ) ( ' )] 180 90 90°− + = °− ° = ° m PA m PAτ τ , deci cercurile C şi C’ sunt ortogonale.

5) Dreptele lui Simson ale punctului τ faţă de triunghiurile ' ' 'A B C şi ABC sunt paralele cu dreapta d. Demonstraţie. Fie 1τ proiecţia lui τ pe BC şi D punctul de intersecţie dintre 1ττ cu cercul

circumscris triunghiului ABC . Dreapta lui Simson corespunzătoare punctului τ în raport cu triunghiul ABC este paralelă cu dreapta AB (vezi „Dreapta lui Simson”) arătăm că AD d . Patrulaterele ADBE şi 'BPB M fiind insciptibile, rezultă

1( ) ( ) 90 ( )= = °− =m DAB m D B m Bτ τ τ 90 ( ') ( )° − =m MPB m BPM , deci ≡DAP APM ,

adică DA PM . Atunci, dreapta lui Simson a punctului τ este în raport cu triunghiul ABC este paralelă cu dreapta d. Analog se arată că dreapta lui Simson a punctului τ este în raport cu triunghiul paralogic ' ' 'A B C este paralelă cu dreapta d. 6) Al doilea punct de intersecţie dintre cercurile circumscrise triunghiurilor ' ' 'A B C , ABC este centrul de similitudine al triunghiurilor ' ' 'A B C , ABC . Demonstraţie. Pentru ca punctul ζ să fie centrul de similitudine dintre triunghiurile

omologice şi asemenea ' ' 'A B C , ABC trebuie ca punctul ζ să fie pentru ambele

triunghiuri propriul său omolog, adică triunghiurile ACζ şi ' 'A Cζ trebuie să fie

asemenea. Avem: '≡ ≡ ≡ AC BC CT CAζ ζ ζ ζ şi

' ' ' ' '≡ ≡ ≡ A C ABC A B C A Cζ ζ , deci triunghiurile ACζ şi ' 'A Cζ sunt asemenea.

Observaţie: Analog se arată că triunghiurile ABζ şi ' 'A Bζ , respectiv BCζ şi ' 'B Cζ

sunt asemenea. 7) Dacă H şi 'H sunt ortocentrele triunghiurilor ABC şi ' ' 'A B C atunci axa de omologie trece prin mijlocul segmentului 'HH . Demonstraţie. Vezi „Teorema lui Sondat”.

Page 426: Barbu_TEOREME_FUNDAMENTALE(2008).pdf

433

III.25. Triunghiuri bilogice

„Crucea pusă lângă numele unora mulţi o iau drept plus.” - S. E. Lec198

Triunghiurile ABC şi ' ' 'A B C se numesc bilogice dacă sunt ortologice şi au aceleşi centru de ortologie. 1) Două triunghiuri bilogice sunt omologice. Demonstraţie. Fie ' ', ' ',= ∩ = ∩D AO B C E BO C A

' '= ∩F CO A B (Fig. 438). Atunci: ' ',⊥AD B C

' ', ' '⊥ ⊥BE A C CF A B şi ' ,⊥A O BC

' , ' .⊥ ⊥B O CA C O AB Fie " ' ',= ∩B BE A B

" ' '.= ∩C CF A C Deoarece O este ortocentrul

triunghiului ' " "A B C rezultă ' " ";A O B C⊥ cum 'A O BC⊥ rezultă " ".BC B C Atunci,

( " " ) ( )m B C F m BCF= (unghiuri corespondente) şi

cum ( " " ) ( " )m B C F m B EF= (deoarece patrulaterul

" "B C EF este inscriptibil) rezultă ( ) ( ),m BCF m BEF= deci patrulaterul BCEF este

inscriptibil. Analog se arată că punctele C, A, F, D respectiv A, B, D şi E sunt conciclice. Dacă ' ', ' ', ' 'X BC B C Y AC C A Z AB A B= ∩ = ∩ = ∩ atunci conform teoremei lui

Dergiades rezultă că punctele X, Y şi Z sunt coliniare; deci, conform reciprocei teoremei lui Desergues triunghiurile ABC şi ' ' 'A B C sunt omologice. 2) Fie ABC şi ' ' 'A B C două triunghiuri bilogice. Picioarele perpendicularelor duse din vârfurile B şi C pe laturile ' 'A C respectiv ' 'A B şi punctele B şi C sunt conciclice. Demonstraţia rezultă din teorema precedentă. 3) Fie XYZ axa de omologie dintre triunghiurile bilogice ABC şi ' ' 'A B C , iar O centrul comun de ortologie. Dreptele OX şi 'AA sunt perpendiculare. Demonstraţie. Fie ' ' , ' ' .D A O BC E B O AC= ∩ = ∩

Conform proprietăţii precedente punctele ', ', ', 'A B D E

aparţin unui cerc C 1.Din puterea punctului O faţă de

cercul C 1 rezultă ' ' ' 'OA OD OB OE⋅ = ⋅ (1).

Patrulaterul ' 'B CE F fiind inscriptibil ( ( ' ) ( ) 90 )= = ° m BE C m CFB rezultă ' 'OB OE OC OF⋅ = ⋅

(2). Dacă ' 'D AO B C= ∩ , atunci patrulaterul AFDC

este inscriptibil (conform proprietăţii precedente) de unde rezultă OC OF OA OD⋅ = ⋅ (3). Din relaţiile (1), (2) şi (3) rezultă ' ',OA OD OA OD⋅ = ⋅ adică punctele , ',A A D şi 'D sunt conciclice, de unde

( ' ') ( ')m AA D m ADD= (4). Fie OT tangenta în O la

cercul (C) circumscris patrulaterului ' ( ( ' ) ( ) 90 )= = ° OD DX m OD X m ODX (Fig. 439).

198 Stanislaw Lec (1909-1966) – poet polonez

A

B C

A'

B'

C' D

E F

Fig. 438

D'

P

E '

X

Z O

C" B"

O

X

D

D' T

A

A'

X'

Fig. 439

Page 427: Barbu_TEOREME_FUNDAMENTALE(2008).pdf

434

Atunci, ( ') ( ')m TOD m ODD= (5). Din relaţiile (4) şi (5) rezultă

( ' ) ( '),= m AA O m TOD deci ' .AA OT Cum OX OT⊥ (ca diametru în cercul C)

rezultă '.OX AA⊥

Observaţie: Analog se arată că 'OY BB⊥ şi '.OZ CC⊥ 4) Fie P centrul de omologie şi XYZ axa de omologie dintre triunghiurile bilogice ABC şi

' ' ',A B C iar O centrul comun de ortologie. Dreptele OP şi XZ sunt perpendiculare. Demonstraţie. Fie ' ' , ' ' , ' 'D A O BC E B O AC X OX AA= ∩ = ∩ = ∩ şi

' '.Y OY BB= ∩ Conform proprietăţii precedente ' ',XX AA⊥ deci patrulaterul

' ' 'A X D X este inscriptibil. Din puterea punctului O faţă de cercul circumscris patrulaterului ' ' 'A X D X rezultă ' ' 'OX OX OA OD⋅ = ⋅ (1). Punctele ', ', ', 'A B D E fiind conciclice – din puterea punctului O faţă de acest cerc – rezultă ' ' ' '⋅ = ⋅OA OD OB OE (2). Deoarece 'OY BB⊥ rezultă ( ' ) 90 ( ),= ° = m YY B m BEY deci punctele , ', ',B Y E Y sunt

conciclice, de unde ' ' 'OB OE OY OY⋅ = ⋅ (3). Din relaţiile (1), (2) şi (3) rezultă

' 'OX OX OY OY⋅ = ⋅ adică punctele , ',X X Y şi 'Y sunt conciclice. Dacă C’ este cercul de

diametru OP, atunci OP este perpendiculară pe tangenta OT în O la C’, iar OT XY

(analog cu demonstraţia din teorema precedentă) de unde .OP XY⊥

III.26. Triunghiuri biortologice. Triunghiuri triortologice

„Pitagora a sacrificat pe altarul lui Zeus o sută de boi şi acesta numai pentru un singur adevăr geometric. Dar dacă în zilele noastre am proceda în acelaşi fel, este puţin probabil că am putea găsi atâtea vite cornute pe întreg globul pământesc.” - M. V. Lomonosov199 Triunghiul ABC se numeşte biortologic cu triunghiul ' ' ',A B C dacă el este ortologic în acelaşi timp cu triunghiul ' ' 'A B C şi cu triunghiul ' ' '.B C A Triunghiul ABC se numeşte triortologic cu triunghiul ' ' ',A B C dacă triunghiul ABC este ortologic în acelaşi timp cu triunghiurile ' ' ', ' ' 'A B C B C A şi ' ' '.C A B Teorema lui Pantazi Dacă un triunghi ABC este ortologic în acelaşi timp cu triunghiurile ' ' ' A B C şi ' ' ',B C A atunci triunghiul ABC este ortologic şi cu triunghiul ' ' '.C A B

Demonstraţie. Fie 1 1 1 1 0D a x b y c= + + = ecuaţia dreptei BC şi '2 1 1 1' ' ' 0D a x b y c= + + =

ecuaţia dreptei ' '.B C Analog, se definesc ecuaţiile dreptelor ce conţin vârfurile triunghiurilor ABC şi ' ' 'A B C . Fie Aa, Bc, Cc perpendicularele duse din A,B,C pe

' ', ' ', respectiv ' '.B C A C A B Avem: 2 1 2 12 3

3 1 3 1

' '( ) : 0 ,

' '

a a b bA a D D

a a b b

+− =

+

3 2 3 23 1

1 2 1 2

' '( ) : 0,

' '

a a b bBb D D

a a b b

+− =

+ 1 3 1 3

1 22 3 2 3

' '( ) : 0.

' '

a a b bCc D D

a a b b

+− =

+ Condiţia ca triunghiurile

ABC şi ' ' 'A B C să fie ortologice este:

199 Mikhailo Lomonosov (1711-1765) – chimist rus

Page 428: Barbu_TEOREME_FUNDAMENTALE(2008).pdf

435

3 2 3 2 1 3 1 32 1 2 1

3 1 3 1 1 2 1 2 2 3 2 3

' ' ' '' '1

' ' ' ' ' '

a a b b a a b ba a b b

a a b b a a b b a a b b

+ ++⋅ ⋅ =

+ + +, egalitate echivalentă cu

1 2 1 2 2 3 2 3 3 1 3 1 1 3 1 3 2 1 2 1 3 2 3 2( ' ')( ' ')( ' ') ( ' ')( ' ')( ' ') (1)a a b b a a b b a a b b a a b b a a b b a a b b+ + + = + + + .

Ţinând seama de relaţia precedentă, pentru ca perpendicularele duse din ', ', 'A B C pe laturile triunghiului ABC să fie concurente, este necesar ca:

1 2 1 2 2 3 2 3 3 1 3 1 1 3 1 3 2 1 2 1 3 2 3 2( ' ' )( ' ' )( ' ' ) ( ' ' )( ' ' )( ' ' ) (2)a a b b a a b b a a b b a a b b a a b b a a b b+ + + = + + +

care este chiar relaţia (1). Deci, dacă perpendicularele duse din vârfurile

triunghiului ABC pe laturile triunghiului ' ' 'A B C sunt concurente, atunci şi

perpendicularele duse din vârfurile triunghiului ' ' 'A B C pe laturile triunghiului ABC

sunt concurente. Pentru ca triunghiurile ABC şi ' ' 'B C A să fie ortologice, trebuie ca:

1 3 1 3 2 1 2 1 3 2 3 2 1 1 1 1 2 2 2 2 3 3 3 3( ' ')( ' ')( ' ') ( ' ')( ' ')( ' ') (3)a a b b a a b b a a b b a a b b a a b b a a b b+ + + = + + +

Din relaţiile (1) şi (3) rezultă:

1 1 1 1 2 2 2 2 3 3 3 3 1 2 1 2 2 3 2 3 3 1 3 1( ' ')( ' ')( ' ') ( ' ')( ' ')( ' ')a a b b a a b b a a b b a a b b a a b b a a b b+ + + = + + +

relaţie echivalentă cu faptul că triunghiurile ABC şi ' ' 'C A B sunt ortologice. Deci, dacă două triunghiuri sunt biortologice, atunci ele sunt triortologice. Consecinţă: Dacă triunghiul ABC este ortologic cu triunghiurile

1 2 3 2 3 4, ...,M M M M M M respectiv 1 1,n nM M M− atunci triunghiul ABC este ortologic şi cu

triunghiul 1 2 .nM M M

III.27. Triunghiuri coparalele

„Expresia, conform căreia cel care nu cunoaşte sau îi este străină geometria nu are dreptul să între în şcoala filozofului, deloc nu înseamnă, că este necesar să fii matematician, pentru a deveni înţelept.” - J. W. von Goethe200

Triunghiurile ABC şi 1 1 1A B C se numesc coparalele dacă 1 1 1.AA BB CC

1) Dacă M este un punct ce aparţine laturii BC a triunghiului ABC, iar d este distanţa

dintre paralelele duse la AM prin B şi C, atunci [ ] .2ABC

AM dA

⋅=

Demonstraţie. Fie 'B şi 'C proiecţiile punctelor B şi C pe dreapta AM. Dacă

( )M BC∈ - Fig. 440 – atunci

[ ] [ ] [ ]' '

2 2

⋅ ⋅= + = + =

ABC ABM AMC

AM BB AM CCA A A

.2

⋅AM d

200J. W. von Goethe (1739-1832) – scriitor german

A

B CM

B’

C’

Fig. 440

Page 429: Barbu_TEOREME_FUNDAMENTALE(2008).pdf

436

Dacă \ ( )M BC BC∈ - fig. 441 şi fig. 442 – avem:

[ ] [ ] [ ]' '

.2 2 2ABC ABM AMC

AM BB AM CC AM dA A A

⋅ ⋅ ⋅= − = − =

2) Fie ABC şi 1 1 1A B C două triunghiuri coparalele. Dacă înălţimile vârfurilor A şi 1A ale

celor două triunghiuri sunt necongruente, atunci există punctele M BC∈ şi 1 1 1M BC∈

astfel încât 1 1

1 1

B MBM

BC BC= şi

[ ] [ ]1 1 1

1 1 .ABC A B C

AMAM

A A=

Demonstraţie. Evident cazurile 1AA BC şi 1 1 1AA BC

nu convin deoarece în fiecare dintre acestea rezultă că înălţimile din A şi 1A ar fi congruente. Fie

1M AA BC= ∩ şi 1 1 1 1.M AA BC= ∩ Astfel, din

1 1 1BB MM CC rezultă 1 1

1 1

,B MBM

BC BC= iar din

proprietatea precedentă avem [ ] 2ABC

AM dA

⋅= şi

[ ]1 1 1

1 1

2A B C

AM dA

⋅= (d fiind distanţa dintre 1BB şi 1CC ) de

unde se obţine: [ ] [ ]1 1 1

1 1 .ABC A B C

AMAM

A A=

3) Dacă triunghiurile ABC şi 1 1 1A B C sunt coparalele, a, b, c respectiv

1 1 1, ,a b c sunt lungimile laturilor lor, atunci

1 1 1

2 2 2 2 2 2 2 2 2 2 2 2 2 21 1 1 [ ] [ ]( ) ( ) ( ) 8( )ABC A B Cb c a a c a b b a b c c A A+ − ⋅ + + − ⋅ + + − ⋅ ≥ + ( )∗ .

Demonstraţie. Dacă înălţimile duse din vârfurile A şi 1A au lungimi diferite

1( )a ah h≠ atunci utilizând proprietatea precedentă fie 1 1

1 1

,B MBM

xBC BC

= = deci BM x BC= ⋅

şi 1 1 1 1.B M x B C= ⋅ Din teorema lui Stewart în triunghiul ABC rezultă : 2 2 2b BM c MC AM BC BM MC BC⋅ + ⋅ − ⋅ = ⋅ ⋅ , relaţie echivalentă cu

( ) ( )2 2 2 21 1 .AM x b x c x x a= ⋅ + − − − Analog se arată că :

B’

B C M

C’

A

Fig. 442

B’

B C

A C’

M

Fig. 441

A

B C

1A

1B 1C

1M

M

Fig. 443

Page 430: Barbu_TEOREME_FUNDAMENTALE(2008).pdf

437

( ) ( )2 2 2 21 1 1 1 11 1 .AM x b x c x x a= ⋅ + − − − Atunci din

[ ] [ ]1 1 1

221 1

2 2ABC A B C

AMAM

A A= şi relaţiile precedente

rezultă: [ ] [ ] [ ] [ ] [ ] [ ]1 1 1 1 1 1 1 1 1

2 2 2 2 22 2 2 2 221 1 1 1 1

2 2 2 2 2 20

ABC A B C ABC A B C ABC A B C

a a b c ca a b c cx x

A A A A A A

− +− + − ⋅ − − ⋅ + − =

(1) .

Ecuaţia (1) are rădăcini reale dacă şi numai dacă 0,∆ ≥ (unde ∆ este discriminantul

ecuaţiei (1) ). Utilizând relaţiile [ ]2 2 2 2 2 2 2( ) 4 16

ABCa b c a c A− + − = − şi

[ ]1 1 1

2 2 2 2 2 2 21 1 1 1 1( ) 4 16 , 0

A B Ca b c a c A− + − = − ∆ ≥ rezultă concluzia. Dacă

1,a ah h= atunci din

considerente de simetrie concluzia este adevărată şi în cazurile 1b bh h≠ şi

1.c ch h≠ În cazul

rămas de studiat - 1 1 1, ,a a b b c ch h h h h h= = = - cele două triunghiuri sunt congruente, iar

inegalitatea de demonstrat se verifică ca egalitate. 4) Două suprafeţe triunghiulare cu aceeaşi arie sunt coparalele. Demonstraţie. Dacă

1 1 1[ ] [ ]ABC A B CA A= atunci inegalitatea ( )∗ se transformă în inegalitatea lui

Pedoe: 1 1 1

2 2 2 2 2 2 2 2 2 2 2 21 1 1 [ ] [ ]( ) ( ) ( ) 16 )ABC A BCb c a a c a b b a b c c A A+ − ⋅ + + − ⋅ + + − ⋅ ≥ ⋅ ⋅ (vezi

„Teorema lui Pedoe”). 5) Două suprafeţe triunghiulare ABC şi 1 1 1A B C cu 1 1BC BC= sunt coparalele.

Demonstraţie. Utilizând faptul că 1a a= inegalitatea (1) devine: 2 2 2 2 21 1( ) 0b c b c− − + ≥ .

III.28. Triunghiuri înscrise

„Nimeni să nu intre aici dacă nu ştie geometrie.” - Platon201

Triunghiul ' ' 'A B C este înscris în triunghiul ABC, dacă pe fiecare latură a triunghiului ABC se află câte un singur vârf al triunghiului ' ' ', ( ' ( ), ' ( ), ' ( ))∈ ∈ ∈A B C A BC B AC C AB

Triunghiul ' ' 'A B C înscris în triunghiul ABC are laturile paralele cu trei ceviene

concurente ale triunghiului ABC dacă şi numai dacă: ' ' '

1BA CB AC

BC CA AB+ + = sau

' ' '1.

CA AB BC

BC AC AB+ + =

Demonstraţie. Fie 1 1 1, ,AA BB CC cevienele concurente. Laturile triunghiului ' ' 'A B C fiind

paralele cu cevienele ', ', 'AA BB CC rezultă că sunt posibile două ordonări:

i) 1 1 1' , ' , 'B A A C C B B A A C C B− − − − − − − − − (Fig. 444 )

ii) 1 1 1' , ' , 'B A A C C B B A A C C B− − − − − − − − − (Fig. 445).

201 Platon (428-348) – filosof, logician, mathematician grec

Page 431: Barbu_TEOREME_FUNDAMENTALE(2008).pdf

438

i) Din reciproca teoremei lui Ceva rezultă

1 1 1

1 1 1

1 AC B A C B

A B BC C A⋅ ⋅ = (1). Notăm cu a,b,c lungimile

laturilor BC, CA, respectiv AB şi cu ' ' ', , , , , (0,1).= = = ∈

BA CB ACx y z x y z

BC CA AB Din

teorema lui Thales rezultă 1

1

'' A AAC

AB A B= de unde

1 1

'1 1BA ax

zA B A B

= − = − adică 1

1a z

A B x

−= , deci

1

1

1AC x z

A B x

− −= (2). Analog, 1

1

1 (3)

B A x y

B C y

− −= şi

1

1

1 (4).

C B y z

C A z

− −= Fie 1 x y x t− − − = . Din

relaţiile (1), (2), (3) şi (4) rezultă 2[ ( ) ( )] 0t t x y z t xy yz zx+ + + ⋅ + + + = , adică

(1 )[(1 )(1 )(1 ) ] 0.x y z x y z xyz− − − − − − + =

Deoarece (1 )(1 )(1 ) 0x y z xyz− − − + > rezultă că

1x y z+ + = , relaţie echivalentă cu

' ' '1

BA CB AC

BC CA AB+ + = . Cazul ii) se tratează analog.

III.29. Triunghiuri înscrise izotomice „Matematica seamănă cu o moară: dacă veţi turna în ea boabe de grâu, veţi obţine făină, dar dacă veţi turna în ea tărâţe, tărâţe veţi obţine.” - Aldous Huxley202

Fie M şi 'M ; N şi 'N ; P şi 'P puncte izotomice pe laturile triunghiului ABC. Triunghiurile MNP şi ' ' 'M N P se numesc izotomice. 1) Triunghiurile de contact şi cel cotangentic corespunzătoare unui triunghi ABC sunt izotomice. Demonstraţie. Deoarece '= = −BM CM p b ,

'= = −CN AN p c şi '= = −AP BP p a (Fig. 446) rezultă

că triunghiurile de contact şi cotangentic ale triunghiului ABC sunt izotomice, unde a, b, c şi p lungimile laturilor şi semiperimetrul triunghiului ABC.

202Aldous Huxley (1894-1963) – scriitor englez

A

B C A'

B'

C'

1A

1B

1C

Fig. 444

A

B C A'

B'

C'

1A

1B 1C

Fig. 445

A

B C M

N P

P'

M'

N'

Fig. 446

Page 432: Barbu_TEOREME_FUNDAMENTALE(2008).pdf

439

2) Triunghiurile izotomice înscrise într-un triunghi au arii egale. Demonstraţie. Soluţia1. Fie ' , ' , 'x AP BP y AN CN z BM CM= = = = = = ,

', ',MM NNα β= = '.PPγ = Avem: [ ] [ ] [ ] [ ] [ ][ ]MNP ABC PAN BMP NCMA A A A A= − + + şi

[ ' ' '] [ ] [ ' '] [ ' '] [ ' '][ ]M N P A B C P A N B M P N C MA A A A A= − + + . Arătăm că

[ ] [ ] [ ] [ ' '] [ ' '] [ ' ']PAN BMP NCM P AN BM P N CMA A A A A A+ + = + + . Egalitatea precedentă este echivalentă

cu ( )( )sin sin ( ) sin sin ( )( )sin ( )sin

2 2 2 2 2 2

x y A xz B z y C xy A x z B z y Cγ β α γ α β+ + + + + ++ + = + + ,

adică: sin sin sin sin sin sin sin sinx B z B B z C x A y A A y Cα γ γα β β γ γβ α+ + + = + + + şi

utilizând teorema sinusurilor obţinem x b zb b z c x a ya a ycα γ γα β β γ γβ α+ + + = + + + .

Cum 2 , 2 , 2a z b y c xα β γ= + = + = + egalitatea precedentă devine evidentă, deci

[ ] [ ' ' '] .=MNP M N PA A

Soluţia 2. Fie MNP şi ' ' 'M N P două triunghiuri izotomice înscrise în triunghiul ABC şi

1=

−BM m

MC m,

1=

−CN n

NA n,

1=

−AP p

PA p. Afixele punctelor M, N şi P sunt:

(1 )= − ⋅ + ⋅M B Cz m z m z , (1 )= − ⋅ + ⋅

N C Az n z n z , (1 )= − ⋅ + ⋅

P A Bz p z p z .

Atunci, aria triunghiului MNP este egală cu

[ ] [ ]

1 1

1 [ (1 )(1 )(1 )] 1 [ (1 )(1 )(1 )]4 4

1 1

M M A A

MNP N N B B ABC

P P C C

z z z zi i

A z z mnp m n p z z mnp m n p A

z z z z

= ⋅ = + − − − ⋅ ⋅ = + − − − ⋅

( )∗ . Analog, ' 1

'

−=

BM m

M C m,

' 1

'

−=

CN n

N A n şi

' 1

'

−=

AP p

P B p, iar ' (1 )= − ⋅ + ⋅

M C Bz m z m z ,

' (1 )= − ⋅ + ⋅N A Cz n z n z , ' (1 )= − ⋅ + ⋅

P B Az p z p z . Aria triunghiului ' ' 'M N P se obţine

înlocuind în relaţia ( )∗ pe m, n şi p cu 1-m, 1-n respectiv 1-p, obţinându-se aceeaşi

expresie, deci [ ] [ ' ' '].=MNP M N P

A A

3) Centrul de greutate a două triunghiuri izotomice MNP şi ' ' 'M N P înscrise în triunghiul ABC sunt simetrice faţă de centrul de greutate al triunghiului ABC. Demonstraţie. Din aplicaţia precedentă rezultă că afixul centrului de greutate al triunghiului

MNP este 1

(1 ) (1 ) (1 )

3 3

+ + + − + + − + + −= =M N P A B C

G

z z z n p z p m z m n zz , iar afixul

centrului de greutate al triunghiului ' ' 'M N P este egal cu:

2

' ' ' (1 ) (1 ) (1 )

3 3

+ + − + + − + + − += =M N P A B C

G

z z z n p z p m z m n zz , de unde

1 2

32=

+ +=

+G G A B CG

z z z z zz , adică tocmai afixul centrului de greutate al triunghiului

ABC.

Consecinţă: Centrul de greutate al triunghiului ABC este mijlocul segmentului ce uneşte centrele de greutate al triunghiului de contact respectiv cotangentic corespunzătoare triunghiului ABC.

Page 433: Barbu_TEOREME_FUNDAMENTALE(2008).pdf

440

III.30. Triunghiuri pseudoisoscele „Multe capitole ale matematicii mi-au fost dragi. Matematica e una.” - Gr. Moisil203

Triunghiul ABC , în care bisectoarele exterioare ale unghiurilor B şi C sunt egale, triunghiul ABC nefiind isoscel, se numeşte pseudoisoscel. 1) Dacă în triunghiul pseudoisoscel ABC , aI ′ este simetricul punctului aI - centrul

cercului A -exînscris al triunghiului ABC faţă de mijlocul laturii BC, atunci centrul cercului înscris (I) al triunghiului ABC aparţine mediatoarei segmentului aAI ′ . Demonstraţie. Fie BE şi CD cele două bisectoare exterioare egale

( ,E AC D AB∈ ∈ ),(Fig. 447) ,b bAA CD A BC∈ , ,c cAA BE A BC∈ , iK , bK , cK

picioarele înălţimilor duse din aI , B respectiv C, în triunghiul

aBCI , , , b b b c c cK BK AA K CK AA′ ′= ∩ = ∩ ,bM AA BE= ∩ .cM AA CD′ = ∩

Avem: [ ] [ ] [ ] [ ] [ ]ABC BCE BAE CAD BCDA A A A A= − = − , [ ] .2

cDCE

BE CKA

⋅= Deoarece ,cAK BE′

rezultă că distanţa de la A la BE este egală cu ,c cK K ′ de unde [ ] 2c c

BAE

BE K KA

′⋅=

şi astfel: )

[ ](

2 2cc c c

ADC

BE CKBE CK K KA

′′ ⋅−= = (1). Analog

A

B C

D

E

M I

1C

1B

iK

bK

cK

aI

'aI

'cK

'bK

bA cA

Fig. 447

'iK

M' 1H

Page 434: Barbu_TEOREME_FUNDAMENTALE(2008).pdf

441

[ ] [ ] [ ] 2b

ABC CAD BCD

BD BKA A A

′⋅= − = (2). Din relaţiile (1) şi (2) rezultă b cBK CK′ ′= .

Unghiurile triunghiurilor aI BC au măsurile 1 1 1ˆ ˆˆ90 ( );90 ( );90 ( )2 2 2m A m B m C° − ° − ° − .

Atunci, 1 1ˆ ˆ( ) ( ) 9 0 (9 0 ( ) ) ( )2 2c a b am K C I m K B I m A m A= = ° − ° − = ,

1( ) ( ) ( )

2c bm K CM m K BM m BAC′ ′ ′= = , de unde rezultă că b cBK M CK M′ ′ ′∆ ≡ ∆ , şi de aici

BM CM ′≡ (3). Fie 1 1 1 1 11, , , , .b c aCC BE C AA BB DC B AA I BB CC′∈ ∈ = ∩

Patrulaterele a aBI CI ′ şi 1BH CI sunt paralelograme, ( unde I este centrul cercului înscris în

triunghiul ABC şi 1H ortocentrul triunghiului aBI C ). Deci aI ′ şi I sunt simetricele

punctelor aI şi 1H faţă de BC . Fie ,a iI K BC′ ′ ⊥ iK BC′ ∈ . Avem

1

' ' 1( ) ( ) ( ).

2ia am BI K m CI H m ACB′ = = Cum patrulaterul aBI CI este inscriptibil rezultă că

1( ) ( ) ( ).

2am BI I m BCI m ACB= = Dar 'a aBI I M AI≡ (ca unghiuri alterne interne), deci

1( ) ( ) ( )

2a a im M AI m BI K m ACB′ ′ ′= = (4). Din 1 1, 'aI C BM C A CM′ ≡ ≡ şi relaţia (3) rezultă

1 1 1aI C C A AB′ ≡ ≡ , adică patrulaterul 1 1aAB I C′ este romb. Deci,

1 1'

aaAI B I AB′≡ relaţie

care împreuna cu (4) dau a aIAI II A′ ′≡ , adică triunghiul aIAI ′ este isoscel, deci

1a aAI II I H′= = (5) şi punctele 1 1, ,C B I sunt coliniare deoarece dreapta 1IC este

perpendiculară pe segmentul ][ aIA ′ în mijlocul acestuia.

2) Consecinţă: Distanţele de la ortocentrul triunghiului aI BC la vârfurile acestuia sunt respectiv egale cu distanţele de la centrul cercului înscris I la vârfurile triunghiului ABC.

3) Triunghiul ortic al triunghiului aI BC este pseudoisoscel.

Demonstraţie. Triunghiul ortic b c iK K K al triunghiului aI BC are unghiurile de măsuri

1180 2 90 ( ) ( )

2m A m A

° − ° − =

respectiv ˆ( )m B , ˆ( )m C şi atunci triunghiurile b c iK K K

şi ABC sunt asemenea, deci triunghiul ortic al triunghiului aI BC este pseudoisoscel.

4) Într-un triunghi pseudoisoscel ABC este adevărată relaţia: aAI r r= − ( ar este raza cercului A-exînscris şi r raza cercului înscris în triunghiul ABC) . Demonstraţie. Avem rrKIKIIIAI aiiaa −=′−′′=′= (6).

203 Grigore Moisil (1906-1973) – matematician român, profesor la Universitatea din Iaşi, membru al Academiei Române

Page 435: Barbu_TEOREME_FUNDAMENTALE(2008).pdf

442

5) Într-un triunghi pseudoisoscel ABC este adevărată relaţia: a

a

BI a c

CI b a

−=

−.

Demonstraţie. Din asemănarea triunghiurilor cCA M ′ şi aCBI , respectiv a triunghiurilor

aBA M şi aBCI rezultă: a

c

CI CB

CM CA=′

şi a

b

BI BC

BM BA= de unde

'a c a bCI CA CM CB BM BC BI BA⋅ = ⋅ = ⋅ = ⋅ (unde am folosit relaţia (3)) şi de aici

a c

a b

BI CA

CI BA= (7). Dar, cAB BA c= = , deci cCA a c= − şi analog bBA b a= − . Astfel

relaţia (7) devine : a

a

BI a c

CI b a

−=

− (8).

6) Într-un triunghi pseudoisoscel ABC este adevărată relaţia: 2 .AI BI CI= ⋅

Demonstraţie. Din asemănarea triunghiurilor ABI şi 1 aBH I rezultă: 1 1a aBI I H BH

AB BI AI= = ;

dar 1a aI H II AI′= = şi 1 1BH CI= , deci aBI AI CI

AB BI AI= = (9). Analog, aCI AI BI

AC CI CI= =

(10), de unde rezultă 2 .AI BI CI= ⋅

7) Într-un triunghi pseudoisoscel ABC , 2sin sin sin2 2 2

A B C= ⋅ .

Demonstraţie. Înlocuim relaţia cunoscută 4 sin sin2 2

B CAI R= ⋅ şi analoagele în egalitatea

2AI BI CI= ⋅ şi va rezulta 2sin sin sin2 2 2

A B C= ⋅ .

8) Într-un triunghi pseudoisoscel ABC , 2a b cII II II= ⋅ .

Demonstraţie. Deoarece 4 sin , 4 sin , 4 sin2 2 2a b c

A B CII R II R II R= = = , egalitatea de

demonstrat este echivalentă cu 2sin sin sin2 2 2

A B C= ⋅ , demonstrată anterior.

9) Într-un triunghi pseudoisoscel ABC , a a

a

BI CIAI

AI

⋅= .

Demonstraţie. Din relaţiile (9) şi (10) rezultă a aBI CI b c⋅ = ⋅ . Cum în orice triunghi ABC

avem aAI AI b c⋅ = ⋅ rezultă a a aBI CI AI AI⋅ = ⋅ , deci a a

a

BI CIAI

AI

⋅= .

Page 436: Barbu_TEOREME_FUNDAMENTALE(2008).pdf

443

10) Într-un triunghi ABC pseudoisoscel, 3 2( ) 3 ( ) 0.a b c a abc bc b c− + + − + =

Demonstraţie. Din relaţiile (9) şi (10) rezultă a

a

CIBI AB b a c

CI BI AC a c b

−= ⋅ = ⋅

− (11). Din

asemănarea triunghiurilor iBIK ′ şi c cCK A′ , respectiv iCIK ′ şi b bBK A′ rezultă:

c b

i

CA BK

CI CK

′=

′. Cum b cBK CK′ ′= rezultă c i

c i

CA BKBI

CI BA CK= ⋅

′. Dar,

,i i i iBK CK p b CK BK p c′ ′= = − = = − (unde 2

a b cp

+ += ), deci:

BI a c p b

CI b a p c

− −= ⋅

− − (12).

Din relaţiile (11) si (12) rezultă legatura dintre laturile unui triunghi pseudoisoscel : a c p b c b a

b a p c b a c

− − −⋅ = ⋅

− − − adică 3 2( ) 3 ( ) 0a b c a abc bc b c− + + − + = (13).

Observaţie: Relaţia precedentă poate fi obţinută direct plecând de la egalitatea bisectoarelor exterioare CD şi BE.

11) Într-un triunghi pseudoisoscel ABC , 2 4 aa bc Rr+ = .

Demonstraţie. Relaţia 3 2( ) 3 ( ) 0a b c a abc bc b c− + + − + = este echivalentă cu

22abca bc

b c a= +

+ − şi cum

2( )a

abcRr

b c a=

+ − (vezi “Cercurile exînscrise”) rezultă

concluzia.

12) Într-un triunghi pseudoisoscel ABC ,1 1 1

0a b a c bc

+ + =− −

.

Demonstraţie. Relaţia 1 1 1

0a b a c bc

+ + =− −

este echivalentă cu

3 2( ) 3 ( ) 0.a b c a abc bc b c− + + − + =

13) Într-un triunghi pseudoisoscel ABC , 2( )a

a

r rII

r

−= .

Demonstraţie. Din asemănarea triunghiurilor AIR şi 'aAI R - unde R şi R’ sunt proiecţiile

punctelor I şi aI ′ pe latura AC – rezultă a a

AI r

AI r= ,iar de aici

a a

AI r

AI AI r r=

− − şi

utilizând relaţia (6) obţinem concluzia.

14) Consecinţă: Într-un triunghi pseudoisoscel ABC , ( )a a a

a

AI r r r rAI

r r

⋅ − ⋅= = .

Page 437: Barbu_TEOREME_FUNDAMENTALE(2008).pdf

444

III.31. Triunghiuri cosimediane „Nu s-ar putea oare reprezenta muzica drept matematica simţurilor şi matematica drept muzică a raţiunii? Căci muzicianul simte matematica,iar matematicianul concepe muzica. Muzica-i vis, matematica este viaţa practică.”- James Sylvester204

Fie ', ', 'A B C punctele de intresecţie ale simedianelor triunghiului ABC cu cercul circumscris triunghiului ABC. 1) Laturile triunghiului ' ' 'A B C sunt proporţionale cu medianele triunghiului ABC. Demonstraţie. Fie ', ', 'a b c lungimile laturilor triunghiului ' ' 'A B C . Avem:

1( ' ') ( ') ( ')

2m AA C m ACC m AC= = şi 1

( ' ') ( ') ( ')2

m AA B m ABB m AB= = de unde

( ' ' ') ( ) ( ) ( ) ( ) 180 ( ).m B A C m ABK m KCA m CBG m BCG m BGC= + = + = °−

Analog, ( ' ' ') 180 ( ), ( ' ' ') 180 ( ).m A B C m AGC m A C B m AGB= °− = ° − Teorema

sinusurilor aplicată în triunghiurile c

BCM şi BGC ne dă: / 2

sin sin=

cmc

GCB B şi

(2 / 3)

sin sin

⋅=

bm a

GCB BGC, de unde sin

4c

bcGCB

Rm= şi 3

sin8

c b

abcCGB

Rm m= . Atunci,

3' 2 sin(180 ) 2 sin

4b c

abca R BGC R BGC

m m= °− = = . Analog

3'

4a c

abcb

m m= şi

3'

4a b

abcc

m m= ,

deci ' ' '

.a b c

a b c

m m m= = Analog se arată că

' ' '

.a b c

a b c

m m m= =

2) Triunghiurile ABC şi ' ' 'A B C admit aceleaşi simediane ', ', '.AA BB CC .

Demonstraţie. Fie , ,a b cK K K intersecţiile dintre simediane cu laturile opuse, ", ", "A B C

intersecţiile simedianelor ', ', 'AA BB CC cu laturile ' ', ' ',B C C A respectiv ' '.A B

Avem: [ ' " ']

[] ' " ']

' " ' ' ''sin 3 3 sinsin : sin

" ' ' ' ' ''sin 4 4 sinA A C b

A A B a b a b c

A mC A A C AA abc abc

A B A A B A A m m m m m

α αα β

β β ⋅

= = = ⋅ ⋅ = ⋅ ⋅

204 James Sylvester (1814-1897) – matematician englez, profesor la Universitatea din Oxford, contribuţii importante în algebră

A

B C

K cK

A'

C'

B'

aK

bK

Fig. 448

A"

B" C"

Page 438: Barbu_TEOREME_FUNDAMENTALE(2008).pdf

445

(1), unde ( " ' '), ( " ' ')m A A C m A A Bα β= = şi 1( ' ) ( ' ) ,

2 = =

m C CA m C Aα

1( ') ( ') .

2 = =

m ABB m ABβ Teorema sinusurilor aplicată în triunghiul c

ACK şi b

ABK dă:

sin sin sin sin, (2).

c c b b

A A

AK CK AK BK

α β= = Deoarece

bBK şi

cCK sunt simediane rezultă:

2 2

, ,c b

c b

K A K Ab c

K B a K C a

= =

de unde rezultă 2 2

2 2 2 2, ,

c b

cb bcK A AK

a b a c= =

+ + iar

2 2 2 2

2 2, (3)b c

b c

acm abmBK CK

a c a b= =

+ +(vezi „Simediane”). Din realaţiile (2) şi (3) rezultă:

sin4

c

bc

Rmα = şi sin (4).

4b

bc

Rmβ = Din relatiile (1) şi (4) rezultă

2 2' " '

'' ' 'b

c

mC A b

A B m c

= =

(conform proprietăţii 1), deci "AA este simediană în triunghiul

' ' 'A B C . Analog se arată că "BB şi "CC sunt simediane în triunghiul ' ' 'A B C . Triunghiurile ABC şi ' ' 'A B C care admit aceleaşi simediane se numesc triunghiuri cosimediane. 3) Triunghiul circumpedal al punctului lui Lemoine al unui triunghi ABC şi triunghiul ABC sunt triunghiuri cosimediane. Demonstraţie. Proprietatea este o reformulare a teoremei (1). 4) Două triunghiuri cosimediane au acelaşi punct al lui Lemoine. Demonstraţia rezultă din faptul că triunghiurile admit aceleaşi simediane. 5) Două triunghiuri cosimediane sunt omologice, centrul de omologie fiind punctul lui Lemoine al triunghiului ABC. Demonstraţia este evidentă.

6) Cercurile lui Apollonis ale triunghiului ABC intersectează cercul circumscris al triunghiului ABC în vârfurile triunghiului cosimedian. Demonstraţie. Vezi „Cercurile lui Apollonius”. 7) Între unghiurile triunghiurilor cosimediane ABC şi ' ' 'A B C au loc relaţiile:

2' ' '

3ctgA ctgA ctgB ctgB ctgC ctgC ctgω+ = + = + = (ω este unghiul lui Brocard).

Demonstraţie. ' ( ) ( )ctgA ctgA ctgA ctg BGC ctgA ctg BGCπ+ = + − = − , G fiind centrul

de greutate al triunghiului ABC. Teorema cosinusului aplicată în triunghiul BGC dă: 2 2

2 2 2 2 22 cos

3 3 3 3b c b ca m m m m BGC

= + − ⋅ ⋅ ⋅

sau : 2 2 25

cos ,8

b c

b c aBGC

m m

+ −= de

Page 439: Barbu_TEOREME_FUNDAMENTALE(2008).pdf

446

unde 2 2 2 2 2 285 ( 5 )

8 3 3b c

b c

Rm mb c a b c a RctgBGC

m m abc abc

+ − + − ⋅= ⋅ = , iar

2 2 2b c a

ctgA Rabc

+ −= ⋅ .

Avem 2 2 2

2 2 2 2 2 25 2' ( )

3 3

R b c a RctgA ctgA b c a b c a

abc abc

+ −+ = + − − = ⋅ ⋅ + +

. Ţinând

cont că: [ ]

2 2 2 2 2 2

4ABC

b c a b c actg R

A abcω

+ + + += = ⋅

⋅ rezultă:

2' .

3ctgA ctgA ctgω+ = Analog se

arată că 2

' ' .3

ctgB ctgB ctgC ctgC ctgω+ = + =

8) Două triunghiuri cosimediane au acelaşi unghi Brocard. Demonstraţie. Fie 'ω unghiul lui Brocard al triughiului ' ' 'A B C . Atunci

2'

3ctgA ctgA ctgω+ = şi

2' ',

3ctgA ctgA ctgω+ = de unde 'ctg ctgω ω= şi cum

30 , ' 30ω ω≤ ° ≤ ° (vezi „Punctele lui Brocard”) rezultă '.ω ω= 9) Două triunghiuri cosimediane au acelaşi cerc Brocard. Demonstraţie. Cercul lui Brocard este cercul ce are ca diamtru segmentul determinat de centrul cercului circumscris triunghiului şi punctul lui Lemoine al aceluiaşi triunghi. Deoarece triunghiurile cosimediane au acelaşi cerc circumscris şi acelaşi punct al lui Lemoine rezultă că au şi acelaşi cerc Brocard. 10) Două triunghiuri cosimediane au aceleaşi puncte Brocard. Demonstraţie. Deoarece cercul lui Brocard trece prin punctele lui Brocard Ω şi 'Ω ale triunghiului ABC, Ω şi 'Ω sunt simetrice faţă de diametrul OK (vezi „Cercul lui Brocard”), iar triunghiurile cosimediane au acelaşi cerc Brocard, rezultă concluzia. 11) Două triunghiuri cosimediane au acelaşi al doilea triunghi Brocard. Demonstraţia este evidentă deoarece vârfurile celui de-al doilea triunghi Brocard sunt picioarele perpendicularelor duse din centrul cercului circumscris triunghiului de referinţă pe simedianele triunghiului.

Page 440: Barbu_TEOREME_FUNDAMENTALE(2008).pdf

447

III.32. Triunghiul celor trei imagini

„Matematica este o monstruoasă tautologie.” - Bernard Shaw205

Triunghiul * * *A B C ale cărui vârfuri sunt obţinute prin simetria vârfurilor triunghiului

ABC faţă de laturile opuse se numeşte triunghiul celor trei imagini al triunghiului ABC (Fig. 449). 1) Triunghiurile ABC şi * * *A B C sunt omologice centrul de omologie fiind ortocentrul triunghiului ABC. Demonstraţie. Deoarece dreptele * *,AA BB şi *CC sunt perpendiculare pe laturile BC, CA, respectiv AB rezultă că ele sunt concurente în ortocentrul triunghiului ABC, deci conform

teoremei lui Desargues triunghiurile ABC şi * * *A B C sunt omologice.

2) Centrul cercului circumscris triunghiului celor trei imagini * * *A B C corespunzător unui triunghi ABC este simetricul centrului cercului circumscris triunghiului ABC faţă de punctul lui Coşniţă. Demonstraţie. Vezi „Punctul lui Coşniţă”. 3) Cercurile circumscrise triunghiurilor * * * * * *, ,AB C BC A CA B trec prin inversul punctului lui Coşniţă faţă de cercul circumscris triunghiului ABC. Demonstraţie. Vezi „Punctul lui Coşniţă”.

4) Fie O centrul cercului circumscris triunghiului ABC şi * * *A B C triunghiul celor trei

imagini corespunzător triunghiului ABC. Cercurile circumscrise triunghiurilor AOA∗ , BOB∗ şi COC∗ se întâlnesc într-un punct care este inversul punctului izogonal conjugat al centrului cercului lui Euler al triunghiului ABC. Demonstraţie. Vezi „Teorema lui Musselman”.

205 Bernard Shaw (1856 -1950) – scriitor irlandez, laureat al Premiului Nobel pentru Literatură în 1925

B C

A

A∗

B∗

C∗

H

Fig. 449

Page 441: Barbu_TEOREME_FUNDAMENTALE(2008).pdf

448

Teorema lui Boutte

2) Fie G centrul de greutate al triunghiului ABC . Triunghiul celor trei imagini * * *A B C

este obţinut prin omotetia de centrul G şi raport 4 a triunghiului podar al centrului cercului lui Euler al triunghiului ABC.

Fie , ,a b cM M M mijloacele laturilor BC , CA respectiv AB şi 1 1 1, ,A B C mijloacele laturilor

,b c c aM M M M respectiv a bM M . Evident, triunghiul 1 1 1A B C se obţine din triunghiul ABC

prin omotetia de centru G şi raport 1/4. Fie X omoteticul lui ∗A prin H 14

( , )G (Fig. 450).

Arătăm că X este proiecţia centrului cercului lui Euler ( 9O ) al triunghiului ABC pe BC.

Cum 1 1

4∗= =

GA GX

GA GA din reciproca teoremei lui Thales rezultă că 1

∗A X AA . Dar

∗ ⊥b c

AA M M de unde rezultă că 1 b cA X M M⊥ , deci 1A X BC⊥ . Cu alte cuvinte

X BC∈ (Fig. 451). Evident , X este simetricul punctului 1A faţă de 1 1B C (care este linie

mijlocie în triunghiul a b cM M M ); cu

alte cuvinte punctul X aparţine cercului circumscris triunghiului a b cM M M ,

adică cercului lui Euler. Urmează că X este proiecţia lui 9O pe BC . Analog se

demonstrează că imaginile punctelor ∗B şi ∗C prin omotetia H 1

4( , )G sunt

proiecţiile lui 9O pe CA, respectiv AB.

Observaţie: Dacă 9 9 9a b cO O O este

triunghiul podar al centrului cercului lui Euler al triunghiului ABC, atunci

* * *

9 9 9

4.a b c

GA GB GC

GO GO GO= = =

B C

A

A∗

B∗

C∗

H

Fig. 450

9O

G O H G O

9O

A'

C

bM cM

A

B

H

1C

1B

1A

X

G

aM

O

Fig. 451

Page 442: Barbu_TEOREME_FUNDAMENTALE(2008).pdf

449

III.33. Triunghiuri izoliniare

„Cel mai fericit om este acela care face fericiţi un număr cât mai mare de oameni.” - Lev Tolstoi206

Fie două triunghiuri ABC şi ' ' 'A B C în acelaşi plan. Dacă paralelele duse prin , ,A B C la laturile ' 'B C , ' 'C A respectiv ' 'A B intersectează laturile opuse în trei puncte coliniare , ,α β γ atunci şi paralelele duse prin ', ', 'A B C la ,BC CA respectiv AB

întâlnesc laturile opuse ' ', ' 'B C C A şi ' 'A B în trei puncte coliniare ', ', '.α β γ

Fie ' ' '1 1 1A B C triunghiul determinat de dreptele ,A Bα β şi Cγ , iar 1 1 1A B C triunghiul

determinat de dreptele ' ', ' 'A Bα β şi ' 'C γ . Din asemănarea triunghiurilor ' ' 'A B α cu

1 'CB α , respectiv ' ' 'C Aα cu 1 'C Bα ( triunghiuri cu laturile paralele două câte două )

rezultă:1

' ' ' '

'

B A

B C

α αα α

= şi 1

' ' ' '

'

C A

C B

α α

α α= . Prin împărţirea relaţiilor precedente se obţine:

1

1

'' '

' ' '

B BB

C C C

α ααα α α

⋅=

⋅. Analog, se arată egalităţile: 1

1

'' '

' ' '

A AA

B B B

γ γγ

γ γ γ

⋅=

⋅ şi

1

1

'' '

' ' '

C CC

A A A

β ββ

β β β

⋅=

⋅, de unde rezultă că

' ' ' ' ' '

' ' ' ' ' '

B C A

C A B

α β γ

α β γ⋅ ⋅ =

' ' '1 1 1 1 1 1' ' '1 1 1

B C A B C A

C A B C A B

α β γ α β γ

α β γ α β γ⋅ ⋅ ⋅ ⋅ ⋅ = ⋅ =

(unde s-a aplicat teorema lui Menelaus

206 Lev Tolstoi (1828-1910) – scriitor rus

'1A

1A

A'

A

C

C' '1C

1C

B

B'

B'

'1B

α

α'

β

β'

γ'

γ

Fig. 452

Page 443: Barbu_TEOREME_FUNDAMENTALE(2008).pdf

450

triunghiurilor ' ' '1 1 1A B C şi ABC cu transversala αβγ ) şi din reciproca teoremei lui Menelaus

aplicată în triunghiul ' ' 'A B C rezultă că punctele ', ', 'α β γ sunt coliniare.

Observaţie: Triunghiurile ABC şi ' ' 'A B C ce îndeplinesc condiţiile de mai sus se numesc izoliniare.

III.34. Triunghiuri metaparalele

„Azi facem matematica ce va fi folosită mâine şi mai ales poimâine. Dacă n-am face-o azi, poimâine ar trebui s-o importăm.” – Gr. Moisil207

1) Fie triunghiurile ABC şi ' ' 'A B C . Dacă paralelele duse din A, B, C la ' 'B C , ' 'C A , respectiv ' 'A B sunt concurente într-un punct D, atunci paralelele duse din 'A , 'B , 'C la BC , CA , respectiv AB sunt concurente într-un punct 'D .

Demonstraţie.Fie 1 1( , )A x y ,2 2, )( yB x , 3 3( , )C x y , ' ' ' ' ' '

1 1 2 2 3 3'( , ), '( , ), '( , )A x y B x y C x y . Ecuaţiile

paralelelor duse din A, B, C la dreptele ' 'B C , ' 'C A , respectiv ' 'A B sunt:

1 13 2 3 2 3 2 3 2

, , , , , , , ,( ) ( ) ( ) ( )x y y y x x y y x xx y− − − = − − − ,

1 3 1 3 2 1 3 2 1 3

, , , , , , , ,( ) ( ) ( ) ( )x y y y x x y y y x xx− − − = − − −

2 1 2 1 3 2 1 3 2 1

, , , , , , , ,( ) ( ) ( ) ( )x y y y x x x y y y x x− − − = − − − . Condiţia de concurenţă este

echivalentă cu: 1 13 2 2 1 3 3 2 1 3 2 2 1 3 3 2 1

, , , , , , , , , , , ,( ) ( ) ( ) ( ) ( ) ( )y y y y x y y x x y x x y x xx x y− + − + − = − + − + − ,

adică ' ' '1 2 3

1 2 3

1 1 1

y y y

x x x

= ' ' '1 2 3

1 2 3

1 1 1

x x x

y y y

( )∗ . Simetria relaţiei precedente demonstrează

teorema . Observaţii:

1) Triunghiurile ABC şi ' ' 'A B C ce au proprietăţile de mai sus se numesc metaparalele sau paralelogice, iar punctele D şi 'D se numesc centre de metaparalelism.

207 Grigore Moisil (1906-1973) – matematician român, profesor la Universitatea din Iaşi, membru al Academiei Române

A

C'

B'

B A1

B1 C1

A'

C

D

D'

Fig. 453

Page 444: Barbu_TEOREME_FUNDAMENTALE(2008).pdf

451

2) Vom nota cu ABC ' ' 'A B C faptul că triunghiurile ABC şi ' ' 'A B C sunt metaparalele. 2) Dacă ABC ' ' 'A B C , ABC ' ' 'B C A , atunci ABC ' ' 'C B A .

Demonstraţie: Utilizând teorema precedentă relaţiile ABC ' ' 'A B C şi ABC

' ' 'B C A sunt echivalente cu: ' ' '1 2 3

1 2 3

1 1 1

y y y

x x x

= ' ' '1 2 3

1 2 3

1 1 1

x x x

y y y

, respectiv ' ' '2 3 1

1 2 3

1 1 1

y y y

x x x

=

' ' '2 3 1

1 2 3

1 1 1

x x x

y y y

. Din egalităţile precedente rezultă: ' ' '3 2 1

1 2 3

1 1 1

y y y

x x x

= ' ' '3 2 1

1 2 3

1 1 1

x x x

y y y

, adică ABC

' ' 'C B A .

3) Dacă ABC 1 2 3M M M , ABC 2 3 4M M M , ... , ABC 1 1n nM M M− , atunci

ABC 1 2n

M M M .

Demonstraţia proprietaţii se realizează utilizând relaţia ( )∗ .

III.35. Triunghiul pedal

„Cea mai mare dorinţă a mea este să comunic şi altora pasiunea mea pentru matematică.” – Miron Nicolescu208

Dacă M este un punct în planul triunghiului ABC, picioarele cevienelor AM, BM, CM sunt vârfurile unui triunghi 1 2 3M M M numit triunghiul pedal al lui M (Fig. 454). 1) Triunghiul pedal al centrului de greutate al unui triunghi ABC este triunghiul median corespunzător triunghiului ABC. Demonstraţie. Vezi „Triunghiul median”. 2) Triunghiul pedal al ortocentrului unui triunghi ABC este triunghiul ortic corespunzător triunghiului ABC. Demonstraţie. Vezi „ Triunghiul ortic”. 3) Triunghiul pedal al punctului lui Gergonne al unui triunghi ABC este triunghiul de contact corespunzător triunghiului ABC. Demonstraţie. Vezi „Punctul lui Gergonne”. 4) Triunghiul pedal al punctului lui Nagel al unui triunghi ABC este triunghiul cotangentic corespunzător triunghiului ABC. Demonstraţie. Vezi „Punctul lui Nagel”.

208 Miron Nicolescu (1903-1975) – matematician român, membru al Academiei Române, contribuţii în analiza matematică

A

B C 1M

3M

M

Fig. 454

2M

Page 445: Barbu_TEOREME_FUNDAMENTALE(2008).pdf

452

5) Fie ' ' 'A B C triunghiul pedal al centrului cercului circumscris triunghiului ABC.

Atunci, sin 2 sin 2 sin 2 sin 2 sin 2 sin 2

, , .' sin 2 ' sin 2 ' sin 2

AO B C BO C A CO A B

OA A OB B OC C

+ + += = =

Demonstraţie. Vezi „Centrul cercului circumscris unui triunghi”.

6) Dacă a b c

Ω Ω Ω este triunghiul pedal al primului punct al lui Brocard Ω

corespunzător unui triunghi ABC , ( ), ( ), ( ),a b c

BC CA ABΩ ∈ Ω ∈ Ω ∈ atunci 2

a

a

B c

C a

Ω = Ω ,

2

b

b

C a

A b

Ω = Ω şi

2

c

c

A b

B c

Ω = Ω .

Demonstraţie. Vezi „Punctele lui Brocard”.

7) Dacă ' ' 'a b cΩ Ω Ω este triunghiul pedal al celui de-al doilea punct Brocard, ' ( )

aBCΩ ∈ ,

' ( )b

CAΩ ∈ , ' ( )c

ABΩ ∈ , atunci 2'

'a

a

B a

cC

Ω = Ω ,

2'

'b

b

C b

aA

Ω = Ω ,

2'

'.c

c

A b

cB

Ω = Ω

Demonstraţie. Vezi „Punctele lui Brocard”.

8) Triunghiul pedal al retrocentului R al unui triunghi ABC este triunghiul podar al punctului lui Longchamps corespunzător triunghiului ABC. Demonstraţie. Vezi „Retrocentrul unui triunghi”. 9) Triunghiul antisuplementar

a b cI I I şi triunghiul pedal al centrului cercului înscris

într-un triunghi ABC sunt ortologice. Demonstraţie. Vezi „Dreapta lui Euler”. 10) În triunghiul ABC fie ' ' 'A B C triunghiul pedal al unui punct Q, astfel încât

'

'

BA p

A C n= ,

'

'

CB m

B A p= şi

'

'

AC n

C B m= . Dacă , ,

A B Cz z z sunt afixele punctelor A,B,C, atunci

afixul punctului Q este .A B C

Q

mz nz pzz

m n p

+ +=

+ +

Demonstraţie. Afixele punctelor ', ', 'A B C sunt 'B C

A

nz pzz

n p

+=

+, '

A C

B

mz pzz

m p

+=

+ şi

' .A B

C

mz nzz

m n

+=

+Fie punctul ' A B C

mz nz pzQ

m n p

+ + + +

. Utilizând condiţia de coliniaritate a

trei puncte ( ), ( )D E

D z E z şi ( )F

F z - anume:

1

1 0

1

D D

E E

F F

z z

z z

z z

= , rezultă fără dificultate că

punctele , ', 'A A Q ; , ', 'B B Q ; respectiv , ', 'C C Q sunt coliniare, iar cum punctul 'Q

aparţine celor trei ceviene ', ', 'AA BB CC rezultă că '.Q Q≡

Page 446: Barbu_TEOREME_FUNDAMENTALE(2008).pdf

453

11) În triunghiul ABC fie ' ' 'A B C triunghiul pedal al unui punct P, astfel încât '

'

BA

A Cν= ,

'

'

AB

B Cµ= şi

'

'

AC

C Bλ= . Atunci, pentru orice punct din planul triunghiului

ABC este adevărată relaţia: 1

.1 1 1

MP MA MB MCλ µ

λ µ λ µ λ µ= + +

+ + + + + +

uuur uuur uuur uuuur

Demonstraţie. Din teorema lui Ceva avem: 1

1λ νµ

⋅ ⋅ = , deci µ

νλ

= (1). Din teorema lui

Van – Aubel obţinem: '

PA

PAλ µ= + , de unde rezultă

1'

1 1MP MA MA

λ µλ µ λ µ

+= +

+ + + +

uuur uuur uuuur

(2). Din '

'

BA

A Cν= rezultă

1'

1 1MA MB MC

νν ν

= ++ +

uuuur uuur uuuur (3). Din relaţiile (1), (2) şi (3) rezultă

concluzia. Cazuri particulare:

1) Dacă P G≡ , atunci 1 1 1

3 3 3MG MA MB MC= + +uuuur uuur uuur uuuur

.

2) Dacă P I≡ , atunci a b c

MI MA MB MCa b c a b c a b c

= + ++ + + + + +

uuur uuur uuur uuuur.

3) Dacă P O≡ , atunci 1

(sin 2 sin 2 sin 2 )sin 2 sin 2 sin 2

MO A MA B MB C MCA B C

= ⋅ + ⋅ + ⋅+ +

uuuur uuur uuur uuuur.

4) Dacă P H≡ , atunci tgA tgB tgC

MH MA MB MCtgA tgB tgC tgA tgB tgC tgA tgB tgC

= + ++ + + + + +

uuuur uuur uuur uuuur (pentru un triunghi

nedreptunghic).

III.36. Triunghiul anticevian

„O teoremă de geometrie este o legătură organică între părţile unei figuri.” – Gh. Ţiţeica209

Fie P un punct din planul triunghiului ABC . Triunghiul anticevian ' ' 'A B C al triunghiului ABC corespunzător punctului P este obţinut astfel: ' ',A B C∈ ' ',B A C∈

' 'C B A∈ iar ' ' ' ∩ ∩ =AA BB CC P (Fig. 455 ). Punctul P se numeşte punct anticevian. 1) Triunghiul ABC este triunghiul cevian al punctului P în raport cu triunghiul

' ' 'A B C . 2) Triunghiul anticevian corespunzător centrului cercului înscris (I) în triunghi este triunghiul antisuplementar. Demonstraţie. Vezi „Triunghiul antisuplementar”.

209 Gheorghe Ţiţeica (1873-1939) – matematician român, profesor la Universitatea din Bucureşti, membru al

Academiei Române, contribuţii importante în geometrie

Page 447: Barbu_TEOREME_FUNDAMENTALE(2008).pdf

454

3) Triunghiul anticevian corespunzător centrului de greutate (G) este triunghiul anticomplementar. Demonstraţie.Vezi „Triunghiul anticomplementar”. 4) Triunghiul anticevian corespunzător punctului simedian (K) este triunghiul tangenţial. Demonstraţie. Vezi „Triunghiul tangenţial”. 5) Fie ' ' 'A B C triunghiul anticevian al unui punct P în raport cu un triunghi ABC. Bisectoarele unghiurilor triunghiului ' ' 'A B C intersectează laturile BC, CA şi AB în punctele ", ",A B respectiv ".C Dreptele ", "AA BB şi "CC sunt concurente. Demonstraţie. Din teorema

bisectoarei rezultă " '

," '

=BA BA

A C A C

" '

" '=

CB CB

B A B A şi

" '.

" '

AC AC

C B C B= Din

teorema lui Ceva aplicată în triunghiul anticevian ' ' 'A B C

rezultă ' ' '

1' ' '

AC BA CB

AB BC CA⋅ ⋅ = , de unde

se obţine: " " " ' ' '

1" " " ' ' '

BA CB AC BA CB AC

A C B A C B A C B A C B⋅ ⋅ = ⋅ ⋅ =

şi din reciproca teoremei lui Ceva aplicată în triunghiul ABC rezultă că dreptele ", "AA BB şi "CC sunt concurente.

A

B C

B'

C'P

A'

Fig. 455

A

B C

B'

C'P

A' Fig. 456

A"

B" C"

Page 448: Barbu_TEOREME_FUNDAMENTALE(2008).pdf

455

III.37. Triunghiuri ortogonale

„Am închis uşile ca să nu intre greşeala. Atunci adevărul m-a întrebat: pe unde voi intra eu?” - R. Tagore210

Două triunghiuri se numesc ortogonale dacă au laturile două câte două perpendiculare una pe alta. Să se construiască triunghiul ' ' 'A B C având vârfurile pe dreptele suport ale laturilor triunghiului ABC astfel încât cele două triunghiuri să fie ortogonale.

Demonstraţie. Presupunem că triunghiul ' ' 'A B C astfel încât ' ' , ' ' , ' ' ,A C AC A B AB B C BC⊥ ⊥ ⊥ ' , ' , ' .A AC B AB C BC∈ ∈ ∈ Construim

perpendicularele pe laturile triunghiului ABC în vârfurile acestui triunghi şi fie 1 1 1A BC

triunghiul obţinut prin intersecţiile acestor perpendiculare. Avem

1 1 1 1 1 1, , .AC AC A B AB BC BC⊥ ⊥ ⊥ Repetând procedeul de mai sus, obţinem triunghiul

2 2 2A B C cu 2 2 1 1 2 2 1 1 2 2 1 1, , .A C AC A B A B B C BC⊥ ⊥ ⊥ Se observă 2 2 2 2, , A C AC A B AB

2 2 .B C BC Se poate considera că triunghiul 2 2 2A B C se obţine din triunghiul ABC printr-o

omotetie. Fie O centrul acestei omotetii, centru care se află la intersecţia dreptelor

2 2 2, , .AA BB CC În mod analog se observă că 1 1 1A BC se poate obţine din ' ' 'A B C printr-o

omotetie şi deoarece 2 1

'OA OA

OA OA= rezultă că omotetia are acelaşi centru O. Din cele

prezentate rezultă procedeul de construcţie a două triunghiuri ortogonale: i) se construieşte triunghiul 1 1 1A BC ; ii) se construieşte triunghiul 2 2 2A B C ; iii) unim A cu 2A , B cu 2B şi se

determină centrul omotetiei O; iv) unim apoi O cu 1 1 1, ,A B C şi la intersecţia cu dreptele

suport ale laturilor ABC găsim punctele ', ', 'A B C care sunt vârfurile triunghiului căutat. Construcţia de mai sus este valabilă indiferent de natura triunghiului.

210 Rabindranath Tagore (1861-1941) –scriitor şi filosof indian, laureat al Premiului Nobel pentru Literatură

A

B C

A' B'

C'

1A

2A

1B

2B 1C 2C

O

Fig. 457

Page 449: Barbu_TEOREME_FUNDAMENTALE(2008).pdf

456

III.38. Triunghiul lui Carnot211 „Geometria este limbajul omului... de la naşterea sa, omul nu a acţionat decât pe fundamentul geometriei, pe care a pătruns-o cu atâta claritate încât putem admite că ea este aceea care ne condiţionează.” - Charles le Corbusier212

Fie H ortocentrul triunghiului ABC şi , ,a b cO O O centrele cercurilor circumscrise

triunghiurilor BCH, ACH, ABH. Triunghiul a b cO O O se numeşte triunghiul lui Carnot, iar

cercurile circumscrise riunghiurilor BHC, AHC, AHB se numesc cercurile lui Carnot - (Ca), (Cb), (Cc).

1) Cercurile circumscrise triunghiurilor BHC, AHC şi AHB sunt congruente cu cercul circumscris triunghiului ABC . Demonstraţie. Fie ', ', 'A B C mijloacele segmentelor AH, BH, CH, Ha , Hb , Hc picioarele înǎlţimilor triunghiului ABC , O centrul cercului circumscris triunghiului ABC şi '

aO

simetricul lui O faţǎ de BC. Atunci, 'a

AH OO≡ şi 'a

AH OO de unde

rezultǎ cǎ patrulaterul OOaHA este paralelogram. Deci, ' ( )≡ =

aHO OA R

(1) şi cum OCOaB este romb rezultă că ' ' ( )

a aO C O B OC OB R≡ ≡ ≡ = (2).

Din relaţiile (1) şi (2) rezultă că ' ' ' ( )a a aO C O B O H R≡ ≡ = , deci '

aO

este centrul cercului circumscris triunghiului BHC, adică '

a aO O≡ . Analog, bO şi

cO sunt simetricele lui O în raport cu

laturile AC, respectiv AB. Cercul circumscris triunghiului BHC este congruent cu cercul circumscris ABC. Analog, cercurile circumscrise triunghiurilor AHC şi AHB sunt congruente cu cercul circumscris triunghiului ABC.

Observaţii:

1) Laturile triunghiului lui Carnot conţin punctele lui Euler ( ', ', 'A B C ) ale triunghiului ABC. 2) Distanţele de la centrul cercului circumscris triunghiului ABC la centrele , ,

a b cO O O sunt

egale cu AH, BH, respectiv CH. 3) Centrele cercurilor lui Carnot sunt simetricele centrului cercului circumscris (O) al triunghiului ABC faţǎ de laturile triunghiului ABC.

2) Triunghiurile a b cO O O şi ABC sunt congruente.

211 Lazare Carnot (1753-1823) – matematician şi inginer francez 212 Charles le Corbusier (1877-1965) – arhitect, pictor francez de origine elveţiană

Ca

A

B

A '

C

aO

bO cO

O

H Cb

Cc

cH bH

B'

C '

Fig. 458

Page 450: Barbu_TEOREME_FUNDAMENTALE(2008).pdf

457

Demonstraţie. Cum a

BO CO şi b

AOCO sunt romburi rezultă:a b

BO OC AO şi

( )a b

BO OC AO R≡ ≡ = , deci patrulaterul a b

ABO O este paralelogram, adică .a b

AB O O≡

Analog, a c

AC O O≡ şi b c

BC O O≡ , triunghiurile a b cO O O şi ABC sunt congruente.

3) Triunghiurile

a b cO O O şi ABC sunt omotetice.

Demonstraţie: Fie 9O mijlocul segmentului OH. Atunci, triunghiul a b cO O O se obţine prin

omotetia de centru 9O şi raport -1, a triunghiului ABC.

4) Triunghiurile

a b cO O O şi ABC au acelaşi cerc al lui Euler şi aceeaşi dreaptă a lui

Euler. Demonstraţie. În triunghiul

a b cO O O ,O este ortocentrul său, iar H este centrul cercului

circumscris, deci are aceeaşi dreaptă Euler cu triunghiul ABC şi evident acelaşi cerc al lui Euler. 5) Cercurile lui Carnot sunt simetrice cercului circumscris triunghiului dat în raport cu laturile corespunzǎtoare. Demonstraţie. Deoarece ortocentrul H al triunghiului ABC aparţine cercului circumscris triunghiului BHC, simetricul sǎu faţa de latura BC, aparţine cercului circumscris triunghiului ABC, iar cercurile circumscrise triunghiurilor ABC şi BHC sunt congruente, rezultǎ cǎ cele douǎ cercuri sunt simetrice faţǎ de latura BC a triunghiului ABC. Analog, se aratǎ cǎ şi celelalte douǎ cercuri Carnot sunt simetrice faţǎ de laturile AC, respectiv AB. 6) Fie H ortocentrul unui triunghi ABC, (Ca), (Cb), (Cc) cercurile lui Carnot corespunzătoare triunghiului ABC, iar

bA şi

cA al doilea punct de intersecţie dintre

cercul (Ca) cu laturile AC respectiv AB. Analog se definesc punctele cB şi

aB , respectiv

aC şi

bC determinate de intersecţiile cercurilor (Cb), respectiv (Cc) cu laturile

corespunzǎtoare ale triunghiului ABC. Ortocentrul triunghiului ABC este centrul

cercurilor circumscrise triunghiurilor b c

AA A ,c a

BB B ,a b

CC C .

Demonstraţie.

A

B C

H

O

aH

bH

cH

aO

bA

cA

Ca

Fig. 459

Page 451: Barbu_TEOREME_FUNDAMENTALE(2008).pdf

458

Avem ( ) 90 ( ) 90 ( ) ( )= °− = ° − =c c a

m HAH m H HA m H HC m HCHa (1). În cercul (Ca)

1( ) ( ) ( )

2= =cm BA H m HCB m BH (2). Din relaţiile (1) şi (2) rezultǎ ≡

cHAB AA H , deci

triunghiul c

AHA este isoscel, de unde ≡c

AH HA (3). Analog se arată că ≡b

AH HA (4).

Din relatiile (3) şi (4) rezultă ≡ ≡c b

AH HA HA , deci H este centrul cercului circumscris

triunghiului b c

AA A . Analog , se aratǎ cǎ H este centrul cercului circumscris triunghiurilor

c aBB B şi

a bCC C .

7) Fie

aO centrul cercului Carnot (Ca) şi a b c

H H H triunghiul ortic al triunghiului ABC.

Atunci: ⊥a a b

BO H H şi ⊥a a c

CO H H .

Demostraţie. Dacă O este centrul cercului circumscris triunghiului ABC, atunci ⊥

a bOC H H (1) (vezi „Triunghiul ortic”) – Fig. 459. Patrulaterul

aBOCO fiind un romb -

datoritǎ faptului cǎ aO este simetricul lui O faţǎ de BC - rezultǎ

aBO OC (2). Din

relaţiile (1) şi (2) rezultǎ cǎ ⊥a a b

BO H H . Analog , se aratǎ ⊥a a c

CO H H .

Observaţie: Analog se aratǎ cǎ ⊥

b b aAO H H , ⊥

b b cCO H H şi ⊥

c c aAO H H ,

⊥c c b

BO H H .

8) Fie 1 1 1, ,A B C punctele de intersecţie dintre bisectoarele interioare ale triunghiurilor A,

B, respectiv C cu cercul circumscris triunghiului ABC. Triunghiul 1 2 3OO O , având

vârfurile în centrele cercurilor Carnot ale triunghiului 1 1 1A BC este omologic cu triunghiul ABC, centrul de omologie fiind un punct al lui Kariya al triunghiului ABC. Demonstraţie. Vezi „Punctul lui Kariya”. 9) Tangentele duse în ortocentrul H al unui triunghi ABC la cercurile lui Carnot (Ca), (Cb), (Cc) intersecteazǎ laturile BC, CA, respectiv AB în trei puncte coliniare. Demonstraţie. Fie ', ', 'A B C punctele determinate de intersecţiile dintre tangentele şi laturile triunghiului , iar ", ", " A B C al doilea punct de intersecţie dintre dreptele

, , AH BH CH cu cercul circumscris triunghiului ABC. Avem

A

B C

H

M

aO A'

A"

B"

C"

Fig. 460

Page 452: Barbu_TEOREME_FUNDAMENTALE(2008).pdf

459

1( ") ( " ") ( ")

2= = m BCC m BB C m BC şi cum 'A H este tangentă cercului (Ca) rezultǎ

( ' ) ( )= m A HB m HCB , de unde ' " "≡ A HB C B B , deci dreptele 'A H şi

" " B C sunt paralele. Fie " "= ∩M B C BC . Din asemǎnarea triunghiurilor ' A BH cu

"MBB şi 'A HC cu "MCC rezultǎ: '

' "=

A B BH

MA HB şi

'

' "=

A C HC

MA HC, de unde

' "

' "⋅=

A B BH HC

A C HC HB. Analog, se arată cǎ

' "

' "⋅=

B C CH HA

B A HA HC şi

' "

' "⋅=

C A AH HB

C B HB HA. Atunci

' ' '1

' ' '⋅ ⋅ =

A B B C C A

A C B A C B, deci punctele ', ', ' A B C sunt coliniare.

10) Fie

a b cC C C triunghiul de contact al triunghiului ABC. Ortocentrele triunghiurilor

b cAC C ,

a cBC C ,

a bCC C sunt centrele cercurilor Carnot ale triunghiului

a b cC C C .

Demonstraţie. Fie 'c cC C şi '

b bC C înǎlţimi în

triunghiul b c

AC C şi 1H ortocentrul

triunghiuluib c

AC C . Atunci 'c c bC C IC şi

'b b cC C IC , deci patrulaterul 1b c

IC H C este

paralelogram (Fig. 461). Cum c bIC IC≡

rezultǎ cǎ patrulaterul 1b cIC H C este romb,

deci 1H este simetricul centrului cercului

circumscris triunghiului de contact faţǎ de latura

b cC C , ceea ce aratǎ cǎ 1H este centrul

cercului Carnot corespunzǎtor laturii b cC C .

Analog se aratǎ cǎ ortocentrele triunghiurilor

a cBC C şi

a bCC C sunt centrele cercurilor

Carnot corespunzǎtoare laturilor a cC C

respectiv b aC C .

11) Laturile triunghiului Carnot trec prin punctele euleriene ',',' CBA ale triunghiului ABC .

Demonstraţia este evidentă din construcţie. 12) Triunghiul ''' CBA este triunghiul median al triunghiului lui Carnot cba OOO .

Demonstraţie. Deoarece HAAA ''≡ şi HOAOHOAO bbcc ≡≡≡ rezultă că patrulaterul

bcHOAO este romb, deci bc OAAO ''≡ , adică 'A este mijlocul segmentului bcOO .

Analog, se arată că 'B şi 'C sunt mijloacele segmentelor caOO şi baOO .

13) Triunghiul " " "A B C având ca vârfuri punctele unde mediatoarele laturilor triunghiului ABC intersectează a doua oară cercul lui Euler al triunghiului ABC este triunghiul ortic al triunghiului lui Carnot.

A

B C

I

aC

bC cC

'bC

'cC

Fig. 461

1H

Page 453: Barbu_TEOREME_FUNDAMENTALE(2008).pdf

460

Demonstraţie. Deoarece bcOOBC || şi "a

M A BC⊥ , iar " b cA O O∈ ( vezi „Cercul lui

Euler” ), rezultă că "aO A este înălţime în triunghiul lui Carnot. Analog se arată că "

bO B

şi "cO C sunt înălţimi în triunghiul cba OOO .

14) Triunghiul ''' CBA (determinat de punctele euleriene ale triunghiului ABC ) şi triunghiul " " "A B C ( având ca vârfuri punctele unde mediatoarele triunghiului ABC intersectează a doua oară cercul lui Euler al triunghiului ABC ) sunt două triunghiuri S . Demonstraţie. Vezi „Triunghiuri Ortopolare”.

15) Fie H ortocentrul unui triunghi ABC , cba HHH triunghiul ortic

1 a b c

H AH H H= I , 2 b a c

H BH H H= I , 3 c a b

H CH H H= I . Dreptele 2 3 3 1, ,H H H H

1 2H H sunt axele radicale dintre cercul lui Euler al triunghiului ABC şi respectiv

cercurile lui Carnot (Ca), (Cb), (Cc).. Demonstraţie. Vezi „Dreapta lui Euler”.

16) Triunghiul având vârfurile în centrele cercurilor Carnot şi triunghiurile având vârfurile în punctele unde înălţimea triunghiului dat intersectează cercul său circumscris sunt omologice, centrul de omologie aparţinând acestui cerc ciscumscris. Demonstraţie. Vezi „Punctul lui Kariya”. 17) Fie I centrul cercului înscris într-un triunghi ABC şi cba HHH triunghiul său

ortic. Centrele cercurilor înscrise în triunghiurile cbHAH , acHBH , baHCH sunt

centrele cercurilor Carnot corespunzătoare triunghiului de contact cba CCC al

triunghiului .ABC

Demonstraţie. Fie M proiecţia lui B pe bisectoarea CI şi 1

cA H M AI= I .

Avem: c b

AH H ACB≡ şi c

MH A MCB≡

1( ( ))

2m ACB= (deoarece patrulaterele

CBHH bc şi MCBH c sunt inscriptibile)

(Fig. 462) de unde rezultă că

1( ) ( )

c cm AH A m AH M= = 1

( ),2 c bm AH H

deci 1AHc este bisectoarea unghiului

c b

AH H . Cum 1AA este bisectoarea

unghiului A, rezultă că 1A este centrul

cercului înscris în triunghiul .bcHAH Dar

1 1 1( ) ( ) ( )

c cm IA M m A AH m A H A= + =

1 1ˆ ˆ( ) ( )2 2m A m C+ şi

1( ) ( )m MIA m MIA= = 1 1ˆ ˆ( ) ( ) ( ) ( )2 2

m IAC m ICA m A m C+ = + ,de unde

rezultă că 1 1.IAM MIA≡ Arătăm că punctul M aparţine dreptei .bcCC Fie

A

B

C aC

bH

bC cC

cH

M

I

1A

H

Fig. 462

Page 454: Barbu_TEOREME_FUNDAMENTALE(2008).pdf

461

1 .c b

M CI C C= I Avem: 1

1( ) [ ( ) ( )]

2m BIM m B m C= + (ca unghi exterior triunghiului

BIC ), 1

1 ˆ( ) 90 ( )2

m BIM m A= °− . Patrulaterul bc ACIC este inscriptibil, deci

1 ˆ( ) ( )2c bm IC C m A= , de unde

1

1 ˆ( ) 90 ( ).2cm BC M m A= ° + Astfel,

1( ) ( ) 180

cm BC M m MIB+ = ° , adică patrulaterul IMBCc 1 este inscriptibil, deci

1( ) ( ) 90

cm BM I m BC I= = ° , de unde rezultă că 1M este proiecţia lui B pe bisectoarea 1C ,

adică .1 MM ≡ Deoarece cbCCAI ⊥ , cbCCM ∈ şi triunghiul IMA1 este isoscel, rezultă

că punctele 1A şi I sunt simetrice faţă de latura bcCC , deci 1A este centrul cercului

Carnot corespunzător laturii bcCC a triunghiului de contact.

18) Fie cba OOO triunghiul lui Carnot al unui triunghi ABC şi a b c

H H H triunghiul ortic

al triunghiului ABC. Dreptele , ,a a b b c cO H O H O H sunt concurente.

Demonstraţie. Vezi „Punctul lui Hexyl”.

III.39. Triunghiul lui Lucas213 „Matematica reprezintă în sine o colecţie de rezultate, care pot fi aplicate la orice.” - Bertrand Russell214

Pe latura BC a triunghiului ABC în exterior se construieşte pătratul 1 1.BCC B Fie

1M AB BC= ∩ şi 1 .N AC BC= ∩ Fie P şi Q punctele de intersecţie dintre

perpendicularele ridicate din N şi M pe latura BC şi laturile AC, respectiv AB.

1) Patrulaterul MNPQ este pătrat. Demonstraţie. Din asemănările triunghiurilor AQM şi 1ABB ; APN şi 1ACC ; APN şi 1 1ABC rezultă:

1 1

,= =AM MQ AQ

AB BB AB 1 1

,= =AN NP AP

AC CC AC 1 1 1 1

= =MN AM AN

BC AB AC (1)

de unde 1 1

MQ NP

BB CC= şi cum 1 1BB CC≡ rezultă

.MQ NP≡ Deoarece MQ NP şi ,MQ BC NP BC⊥ ⊥

rezultă că patrulaterul MNPQ este dreptunghi. Atunci, PQ BC şi (2).PQ MN≡ Din relaţiile (1) şi (2) rezultă

1

AQ QP MN QM

AB BC BC BB= = = (şi cum 1BC BB≡ ), de unde

,MN QM≡ deci patrulaterul MNPQ este pătrat.

213 Francois Lucas (1842-1891) – mathematician francez, contribuţii în teoria numerelor 214 Bertrand Russell (1872 - 1970) – filosof, logician şi matematician englez, laureat al Premiului Nobel pentru literatură

A

B C M N

P Q

1B 1C

A'

Fig. 463

Page 455: Barbu_TEOREME_FUNDAMENTALE(2008).pdf

462

Observaţie: Cercul circumscris triunghiului APQ se numeşte A – Lucas. Analog, se definesc cercurile B – Lucas şi C – Lucas. Fie , ,

A B CL L L centrele cercurilor Lucas.

Triunghiul A B CL L L se numeşte triunghiul lui Lucas.

2) Fie a, b, c lungimile laturilor triunghiului ABC şi R raza cercului circumscris acestui

triunghi. Pătratul MNPQ are latura de lungime egală cu .2

1

a

aR

bc+

Demonstraţie. Fie 'A piciorul înălţimii duse din A. Din asemănarea triunghiurilor AQP şi

ABC rezultă: ,AQ AP QP

c b a= = de unde ,

c bAQ l AP l

a a= = , ( PQ l= ), iar din asemănarea

triunghiurilor BQM şi 'BAA rezultă ,'

BQ QM

AB AA= de unde

−=

a

cc l

la

c h şi de aici

⋅=

+a

a

a hl

a h. Dar

[ ]2 2,

4 2

ABC

a

A abc bch

a a R R

⋅ ⋅= = =

⋅ de unde rezultă .

21

al

aR

bc

=+

3) Razele cercurilor A – Lucas, B – Lucas şi C – Lucas sunt egale cu: 2

1=

+A

RR

aR

bc

,

21

B

RR

bR

ac

=+

, respectiv .2

1C

RR

cR

ab

=+

Demonstraţie. Deoarece triunghiurile APQ şi ABC sunt omotetice, centrul omotetiei fiind

punctul A şi raportul de omotetie fiind egal cu 12

1

l

aRa

bc

=+

(conform proprietăţii

precedente) rezultă că ,AR l

R a= de unde .

21

A

RR

aR

bc

=+

Analog se determină lungimile

celorlalte două raze.

4) Cercul circumscris triunghiului ABC şi cercurile lui Lucas sunt tangente interior. Demonstraţie. Deoarece triunghiurile AQP şi ABC sunt

omotetice, prin omotetia de centru A şi raport 12

1aR

bc+

rezultă că cercurile circumscrise acestor două triunghiuri se corespund prin omotetia considerată, deci cercurile sunt tangente interior.

A

B

C

O

P Q

AL

Fig. 464

Page 456: Barbu_TEOREME_FUNDAMENTALE(2008).pdf

463

Observaţii:

1) Raportul de omotetie poate fi considerat şi sub forma .AR

R

2) Analog se arată că cercurile B – Lucas şi C – Lucas sunt tangente interior cercului circumscris triunghiului ABC.

5) Cercurile lui Lucas sunt tangente două câte două. Demonstraţie. Avem: ,= − = −

B B BOL OB L B R R

= −C C

OL R R şi ( ) 2 ( ).= m BOC m BAC Aplicând

teorema cosinusului în triunghiul B C

OL L rezultă: 2 2 2 2 cos( ).B C B C B C B CL L OL OL OL OL L OL= + − ⋅ ⋅ Cum

,2

1= −

+B

ROL R

bR

ac

2

1= −

+C

ROL R

cR

ab

şi

2 2 2

2cos( ) cos 2 2cos 1 2 12B C

b c aL OL A A

bc

+ −= = − = −

rezultă 2 22 ( )

,( 2 )( 2 )

+ + ⋅= = +

+ +B C B C

R abc b R c R aL L R R

ac bR ab cR deci

cercurile B – Lucas şi C – Lucas sunt tangente. Analog, se arată că cercurile A – Lucas şi C – Lucas respectiv B – Lucas şi A – Lucas sunt tangente. 6) Laturile triunghiului Lucas au lungimile , , .A B B C C AR R R R R R+ + + Demonstraţia este o consecinţă a proprietăţii precedente. Fie , ,A B CT T T punctele de tangenţă dintre cercurile lui Lucas. Triunghiul A B CT T T se

numeşte triunghiul tangentelor Lucas. Cercul circumscris triunghiului A B CT T T se numeşte

cercul radical al cercurilor Lucas.

III.40. Triunghiul lui Fuhrmann „Dacă cineva vrea să determine cu un cuvânt laconic şi expresiv esenţa matematicii, acela trebuie să spună, că este o ştiinţă despre infinit.” - Henri Poincaré215

Fie C cercul circumscris triunghiului ABC . Triunghiul lui Fuhrmann al triunghiului ABC este triunghiul A B CF F F ale cǎrui vârfuri sunt simetricele mijloacelor arcelor

,BC AC respectiv AB , considerate în cercul C, faţă de laturile triunghiului ABC (Fig. 466). Cercul circumscris triunghiului lui Fuhrmann se numeşte cercul lui Fuhrmann. Centrul cercului lui Fuhrmann (F ) se numeşte punctul lui Fuhrmann (F). Fie ', ', 'A B C

mijloacele arcelor ,BC AC , respectiv AB şi ,a bM M , cM mijloacele laturilor triunghiului

ABC , iar O centrul cercului circumscris triunghiului ABC .

215 Henri Poincaré ( 1854 -1912) – matamatician şi fizician francez, contribuţii importante în toate ramurile matematicii

A

B

C

O

AL

BL

CL

AT

BT

CT

Fig. 465

Page 457: Barbu_TEOREME_FUNDAMENTALE(2008).pdf

464

1) Dreptele ' , ' , 'A B CA F B F C F sunt concurente în centrul cercului circumscris

triunghiului ABC . Demonstraţie. Deoarece 'OA BC⊥ rezultǎ cǎ şi 'AF A BC⊥ ,deci ' AO A F∈ . Analog

' BO B F∈ şi ' CO C F∈ .

2) Triunghiul ABC şi triunghiul Fuhrmann A B CF F F al triunghiului ABC sunt ortologice. Demonstraţie. Deoarece ' , 'A BF A BC F B AC⊥ ⊥ şi 'CF C AB⊥ iar ', ', 'A B CF A F B F C sunt

concurente în centrul cercului circumscris triunghiului ABC rezultă că triunghiul A B CF F F

este ortologic cu triunghiul ABC . 3) Perpendicularele duse din A, B şi C pe laturile B CF F , A CF F respectiv B AF F ale triunghiului Fuhrmann sunt concurente într-un punct P. Demonstraţie. Deoarece relaţia de ortologie dintre două triunghiuri este simetrică, rezultă (conform proprietăţii precedente) că triunghiul ABC este ortologic cu triunghiul Fuhrmann, deci perpendicularele duse din A, B şi C pe laturile B CF F , A CF F respectiv

B AF F sunt concurente într-un punct P.

4) Cercurile având centrele în vârfurile ,,A B CF F F şi trec prin punctele B şi C, C şi A, respectiv A şi B sunt concurente în punctul P. Demonstraţie. Fie I centrul cercului înscris în triunghiul ABC . Fie , ,a b cζ ζ ζ cercurile

circumscrise triunghiurilor BIC ,CIA respectiv AIB , iar ' ' ', ,a b cζ ζ ζ cercurile având centrele

în punctele ,,A B CF F F şi trec prin punctele B şi C, C şi A , respectiv A şi B (Fig. 467).

Conform teoremei lui Catalan, centrele cercurilor , ,a b cζ ζ ζ sunt mijloacele arcelor

,BC AC respectiv AB ale cercului circumscris triunghiului ABC . Cercurile aζ şi 'aζ

A

B C

A'

B'

C'

P

I O

FB

FC FA

Ma

Mb

Mc

Fig. 466

Page 458: Barbu_TEOREME_FUNDAMENTALE(2008).pdf

465

sunt simetrice faţă de BC; bζ şi 'bζ sunt simetrice faţă de CA; cζ şi '

cζ sunt simetrice faţă

de AB. Atunci conform teoremei lui Schoute cercurile ' ' ', ,a b cζ ζ ζ sunt concurente într-un

punct Q, linia centrelor fiind perpendiculară pe axa radicală a cercurilor; rezultă că , ,C B A C A BF F F F F F sunt mediatoarele segmentelor , ,AQ BQ CQ şi conform proprietăţii (3)

rezultă că punctele P şi Q coincid, deci cercurile ' ' ', ,a b cζ ζ ζ sunt concurente în punctul P.

5) Fie aI centrul cercului A -exînscris corespunzător triunghiului ABC , '

aI simetricul

lui aI faţă de BC . Dacă P este punctul de concurenţă al cercurilor ' ' ', ,

a b cζ ζ ζ (având

centrele în vârfurile triunghiului Fuhrmann şi trec prin punctele ( , )B C , ( , )C A

respectiv ( , )A B ), atunci punctele P, A şi 'aI sunt coliniare.

Demonstraţie. Fie 'A mijlocul arcului BC al cercului circumscris triunghiului ABC şi I centrul cercului înscris în triunghiul ABC . Punctele , , 'A I A şi aI sunt coliniare (vezi

“Cercurile exînscrise”) iar 'A este centrul cercului circumscris patrulaterului aBICI (adică

cercul aζ (Fig. 467). Cum cercurile

aζ şi 'aζ sunt simetrice faţă de latura

BC rezultă că punctul 'aI aparţine

cercului 'aζ . Fie AH şi CH

ortocentrele triunghiurilor IBC , respectiv IAB , iar ''A al doilea punct de intersecţie dintre cercul '

şi dreapta BC. Conform proprietăţii prin care simetricele ortocentrului unui triunghi faţă de laturile acestuia aparţin cercului circumscris triunghiului (vezi „Ortocentrul unui triunghi”) rezultă că simetricul lui

AH faţă de BC aparţine lui aζ şi

deci AH aparţine lui 'aζ . Analog

cH ∈ '

cζ . Evident I este ortocentrul

triunghiului cAH B . Simetricul său

faţă de AB aparţine cercului 'cζ .

Avem: ''≡ cA H B A A B

1( )

2 =

m APB (1), iar unghiurile

cAH B şi AIB sunt

suplementare ( I fiind ortocentrul lui cAH B ). Datorită simetriei rezultă '

a aI CB BCI≡ , iar în aζ avem: a aBCI BII≡

(2), de unde 'a a

BCI BII≡ . Cum ( ) ( ) 180cm AH B m AIB+ = ° şi

( ) ( ) 180am BIA m BII+ = ° rezultă ( ) ( )c am AH B m BII= , adică c aAH B BII≡ (3).

Din relaţiile (1), (2) şi (3) rezultă '"a

AA B BCI≡ , adică '"a

AA CI , relaţie care arată că

P≡Q

A

B C

C

ζa’

ζc’

ζb’

FA FC

FB

Fig. 467

I

A"

A'

AH 'aI

aI

1A aD aM

Page 459: Barbu_TEOREME_FUNDAMENTALE(2008).pdf

466

punctele P, A şi 'aI sunt coliniare (deoarece în '

cζ şi 'aζ avem

( ) 180 ( " )m BPA m BA A= °− = '180 ( ) ( )a a

m BCI m BPI° − = şi cum A şi 'aI se află în

acelaşi semiplan faţă de PB rezultă P, A şi 'aI coliniare).

Observaţie: Analog se arată că simetricele 'bI şi '

cI ale centrelor cercurilor B - exînscris,

respectiv C - exînscris faţă de CA şi AB sunt coliniare cu punctele P şi B, respectiv P şi C.

6) Teorema lui Stevanovic

Ortocentrul triunghiului lui Fuhrmann corespunzător triunghiului ABC este centrul cercului înscris în triunghiul ABC . Demonstraţie. Păstrând notaţiile de mai sus, fie 1 aA AI BC= ∩ , aM mijlocul laturii

BC şi ' a a aD I I BC= ∩ (Fig. 467 ). Datorită simetriei faţă de BC , rezultă că punctele

1A , AF şi 'aI sunt coliniare. Dar a aIM AD (vezi „Punctul lui Gergonne”- teorema lui

Poncelet) şi 'a A a a

M F D I . Din reciproca teoremei lui Desargues aplicată triunghiurilor

A aIF M şi 'a a

AI D având centrul de omologie 1A - rezultă 'a A

AI IF . Conform proprietăţii

(3) rezultă 'a B C

PI F F⊥ , adică A B CIF F F⊥ , analog B A CIF F F⊥ , deci I este ortocentrul

triunghiului lui Fuhrmann. 7) Dacă afixul cercului circumscris triunghiului ABC este în originea reperului complex , atunci afixul punctului lui Fuhrmann este egal cu

(2 ) (2 ) (2 )

2A B C

F

z p a z p b z p cz

p

− + − + −= .

Demonstraţie. Notăm cu Xz afixul punctului X . Afixul centrului cercului A - exînscris

este ( ) ( )[ ]

( )( )a

C A BI

c a b z b a az bzz

a b b c a

+ + − +=

+ + − (4) (vezi„Triunghiul cotangentic”), iar afixul

centrului cercului înscris în triunghiul ABC este A B CI

az bz czz

a b c

+ +=

+ + (2). Deoarece 'A

este mijlocul segmentului aII (vezi „Cercurile exînscrise”) rezultă ' 2aI I

A

z zz

+= (3). Cum

aM este mijlocul segmentului ' AA F rezultă '22

a

A A

I I

F M B CA

z zz z z z z

+= − = + − . Analog

se determină afixele punctelor BF şi CF , de unde rezultă afixul centrului de greutate al

triunghiului A B CF F F : 3

A B C

F

F F F

G

z z zz

+ += =

(4 ) (4 ) (4 )

6A B Cz p a z p b z p c

p

− + − + −= .

Deoarece I este ortocentrul triunghiului A B CF F F rezultă 2F FIG G F= (unde F este

centrul cercului lui Fuhrmann), deci 2

3F

I FG

z zz

+= , de unde

(2 ) (2 ) (2 )

2A B C

F

z p a z p b z p cz

p

− + − + −= .

Page 460: Barbu_TEOREME_FUNDAMENTALE(2008).pdf

467

8) Centrul cercului lui Fuhrmann corespunzător unui triunghi ABC este mijlocul segmentului HN , unde H şi N sunt ortocentrul, respectiv punctul lui Nagel al triunghiului ABC . Demonstraţie. Alegem un reper complex cu originea în centrul cercului circumscris triunghiului ABC .

Atunci, H A B Cz z z z= + + , (2 ) (2 ) (2 )

2A B C

F

z p a z p b z p cz

p

− + − + −= ,

( ) ( ) ( )A B CN

z p a z p b z p cz

p

− + − + −= . Deoarece

2H N

F

z zz

+= rezultă concluzia.

9) Punctul lui Nagel şi ortocentrul unui triunghi ABC aparţin cercului lui Fuhrmann corespunzător triunghiului ABC .

Demonstraţie. Deoarece AH F N F F Fz z z z z z− = − = − , adică AHF NF FF= = rezultă că

H şi N aparţin cercului lui Fuhrmann.

Observaţie: Punctele H şi N sunt diametral opuse în cercul lui Fuhrmann.

10) Raza cercului lui Fuhrmann corespunzător triunghiului ABC are lungimea egală cu lungimea segmentului OI , unde O şi I sunt centrele cercurilor circumscris, respectiv înscris în triunghiul ABC . Demonstraţie. Deoarece HN OI şi 2HN OI= (vezi„Punctul lui Nagel”), rezultă

1

2FR HN OI= = = 2 2R Rr− .

11) Consecinţă: Patrulaterul IONF este paralelogram. 12) Centrele cercurilor lui Euler ale triunghiului ABC şi triunghiului lui Fuhrmann coincid. Demonstraţie. Alegem un reper complex cu originea în centrul cercului circumscris triunghiului ABC , deci 0Oz = . Atunci, afixul centrului cercului lui Euler al triunghiului

ABC este 9 2 2

H O A B CO

z z z z zz

+ + += = . Afixul centrului lui Euler al triunghiului lui

Fuhrmann ste 9 2 2F

A B CI F

O

z z zz zz

+ ++= = .

13) Consecinţă: Raza cercului lui Euler a triunghiului lui Fuhrmann este egală cu jumătate din lungimea segmentului OI . 14) Fie A B CF F F triunghiul lui Fuhrmann corespunzător unui

triunghi ABC . Cercurile circumscrise triunghiurilor AF BC ,

BF CA , CF AB , A B CF F F sunt concurente în ortocentrul

triunghiului ABC . Demonstraţie. Fie 1H simetricul ortocentrului H faţă de

BC (Fig. 468); punctul 1H aparţine cercului circumscris

triunghiului ABC . Deoarece patrulaterele ABHF C şi 1 'BH A C

A

B C

H

aM

A'

AF

Fig. 468

Page 461: Barbu_TEOREME_FUNDAMENTALE(2008).pdf

468

sunt congruente, iar 'BHA C este inscriptibil rezultă că şi patrulaterul ABHF C este

inscriptibil, H aparţine cercului circumscris triunghiului AF BC . Analog se arată că H

aparţine cercurilor circumscrise triunghiurilor BF CA şi CF AB . Cum H aparţine şi

triunghiului A B CF F F , rezultă concluzia.

15) Simetricul centrului cercului circumscris unui triunghi ABC faţă de punctul lui Spieker al triunghiului ABC este punctul lui Furhmann. Demonstraţie. Vezi „Punctul lui Spieker”.

III.41. Triunghiul lui Lionnet

„Ştiinţele matematice, ştiinţele naturale şi ştiinţele umanitare pot fi numite, respectiv

şi ştiinţe supranaturale, ştiinţe naturale şi ştiinţe nenaturale.” - L. D. Landau216

1) Pe laturile unui triunghi ABC se construiesc în exterior triunghiurile ' , ' , 'A BC AB C ABC asemenea cu ' ' 'A B C . Cercurile circumscrise acestor triunghiuri au

un punct comun D. Demonstraţie. Din condiţia de asemănare rezultă

( ' ) ( ), ( ') ( ),= = m BA C m BAC m CBA m CBA

( ) ( ' )= m ACB m AC B . Fie D punctul de

intersecţie dintre cercurile circumscrise triunghiurilor 'ABC şi 'AB C (Fig. 470). Atunci, ( ) 180 ( ),= °− m ADB m C ( ) 180 ( )= °− m ADC m B

( ) 360 [180 ( ) 180 ( )]= °− °− + °− = m BDC m C m B

( ) ( ) 180 ( ) 180 ( ' )+ = °− = °− m B m C m A m BA C

deci punctul D aparţine cercului circumscris triunghiului 'BA C . Fie 1 2 3, ,L L L centrele cercurilor circumscrise triunghiurilor ' , ' , 'A BC AB C ABC .

Triunghiul 1 2 3L L L se numeşte triunghiul lui Lionnet corespunzător triunghiului ABC.

Cercul circumscris triunghiului 1 2 3L L L se numeşte cercul lui Lionnet.

2) Triunghiurile ' ' 'A B C şi 1 2 3L L L sunt omologice.

Demonstraţie. Deoarece ( ) 180 ( ), ( ') ( ') ( )= ° − = = m BDA m C m BDA m BCA m C

rezultă că ( ) ( ') 180+ = ° m BDA m BDA , deci punctele , , 'A D A sunt coliniare. Analog se

arată că , , 'B D B respectiv , , 'C D C sunt coliniare, deci D este centrul de omologie dintre triunghiurile ABC şi ' ' 'A B C .

216L. D. Landau (1908-1968) –fizician rus, laureat al Premiului Nobel pentru Fizică în anul 1962

G

F H N

pS

O I Fig. 469

A

B C

1L

2L

O

D

Fig. 470

3L

A'

B' C'

Page 462: Barbu_TEOREME_FUNDAMENTALE(2008).pdf

469

3) Triunghiurile ' ' 'A B C şi 1 2 3L L L sunt ortologice. Demonstraţie. Deoarece AD este axa radicală a cercurilor circumscrise triunghiurilor

'AB C şi 'ABC rezultă 2 3⊥AD L L deci 2 3' .⊥AA L L Analog, 1 3' ⊥BB L L şi 1 2' ⊥CC L L ,

iar cum ' ' ' ∩ ∩ =AA BB CC D , rezultă că D este un centru de ortologie , celălalt fiind

centrul cercului circumscris triunghiului ABC. 4) Triunghiului lui Lionnet 1 2 3L L L este asemenea cu triunghiul ' ' 'A B C .

Demonstraţie. Deoarece 1 3⊥BD L L , 1 2⊥CD L L rezultă că 2 1 3( ) 180 ( )= °− =m L L L m BDC

( ' ) ( ' ' ') ( )= =m BA C m B A C m BAC , şi analog 1 2 3( ) ( ' ' ')=m L L L m A B C , deci triunghiurile

' ' 'A B C şi 1 2 3L L L sunt asemenea.

5) Triunghiul lui Lionnet 1 2 3L L L este asemenea cu triunghiul .ABC

Demonstraţia rezultă din proprietatea precedentă. 6) Punctele D şi O sunt izogonale în triunghiul lui Lionnet, unde O este centrul cercului circumscris triunghiului ABC . Demonstraţie. Deoarece 1DL şi perpendiculara din D pe BC sunt drepte izogonale în raport

cu unghiul BDC , 1( ( ) 90 ( ))= °−m L DB m BCD , rezultă că 1DL şi 1LO sunt izogonale în

raport cu unghiul 3 1 2L L L , deoarece 1 3⊥BD L L , 1 2⊥CD L L .

III.42. Triunghiurile lui Morley217 „Arta de a rezolva probleme geometrice seamănă cu trucurile iluzioniştilor – uneori, chiar ştiind soluţia problemei, nu-i clar cum s-ar putea ajunge la ea.” - I. D. Novikov218

Teorema lui Morley

Trisectoarele unghiurilor unui triunghi se intersectează în trei puncte care sunt vârfurile unui triunghi echilateral. Demonstraţie. Fie triunghiul ABC şi L, M, N intersecţiile trisectoarelor (Fig.471). Avem:

1( ) ( ) ( ) ( )

3m BAN m NAM m MAC m A= = =

1( ) ( ),

3m NBA m B= ( ) 1

( )3

m MCA m C= .

Teorema sinusurilor în triunghiul ANB dă:

s i n s i n3 3

A N A B

B A B=

+ adică,

217 Frank Morley (1860-1937) – matematician englez, profesor la Universitatea Johns Hopkins, contribuţii in

algebră şi geometrie 218 Igor Dovikov (1935- ) – fizician rus

A

B C

1A

2A

1B

2B

1C

2C

L

M N

P

Q R

α α

α

β β β γ γ γ

Fig. 471

Page 463: Barbu_TEOREME_FUNDAMENTALE(2008).pdf

470

22 sin sin 3 4sinsin 2 sin sin 2 sin( )sin3 3 33 3 3

sin sin sinsin3 3 33

B C CB B B Rc R C R C

ANC C CC

π ππ

π π ππ

− − −− = = = = =

− − −−

2 2 2 2 1 2 22 sin 3 2 1 cos 2 sin 1 2cos 4 sin cos

3 3 3 3 3 2 3

− − − − − = + = + =

B C B C B CR R R

π π π

2 24 sin cos cos 8 sin cos cos 8 sin sin cos .

3 3 3 3 2 3 6 3 3 3 6 3

− = + = − − = −

B C B C C B C CR R R

π π π π π

Analog se obţine: 8 sin sin cos .3 3 6 3

B C BAM R

π = −

Teorema cosinusurilor în triunghiul

ANM ne dă: 2 2 2 2 cos3

AMN AM AN AM AN= + − ⋅ ⋅ şi utilizând relaţiile precedente

obţinem: 2 2 2 2 2 264 sin sin cos cos 2cos cos cos3 3 6 3 6 3 6 3 6 3 3

B C C B C B AMN R

π π π π = − + − − − − =

2 2 2

2 2 2 21 cos 1 cos

6 3 6 364 sin sin cos cos cos

3 3 2 2 3 3 3

+ − + − − − − = + − + =

C B

B C A B C B CR

π ππ

2 2 264 sin sin 1 cos cos cos cos cos3 3 3 3 3 3 3

B C B C B C A B C B CR

π π − − − − − − = + + − + =

2 2 2 264 sin sin 1 cos cos cos cos cos3 3 3 3 3 3 3

B C A B C A A B CR

− − = + − − =

2 2 2 264 sin sin sin .3 3 3

B C AR Aşadar, 8 sin sin sin .

3 3 3

A B CMN R= Simetria în A, B, C a

relaţiei precedente asigură valabilitatea enunţului. Observaţie: Triunghiul LMN determinat de trisectoarele interioare ale unghiurilor triunghiului ABC se numeşte triunghiul interior al lui Morley. Fie NML triunghiul lui Morley al triunghiului ABC şi

1 2 , ,A LM NB A NL MC= ∩ = ∩ 1 ,B MN LC= ∩ 2 B LM NA= ∩ , 1 ,C NL MA= ∩

2 , C MN LB P BN MC= ∩ = ∩ , 1 2 , , 'Q CL NA R AM LB A BB CC= ∩ = ∩ = ∩ ,

1 2 1 2 ' , 'B CC AA C AA BB= ∩ = ∩ , iar razele cercului circumscris triunghiurilor

' , ' , 'A BC AB C ABC le vom nota cu ' ' '1 2 3, ,R R R . Fie ( ) , ( ) ,= = m BAP m LBCα β

( ) =m MCA γ (Fig. 471).

Page 464: Barbu_TEOREME_FUNDAMENTALE(2008).pdf

471

1) Pentagoanele 1 2 1 2 1 2, ,AAMNA BB NLB CC LMC sunt inscripibile.

Demonstraţie. Deoarece MN PL⊥ şi PL este bisectoare în triunghiul BPC avem ( ) 60 2m BPC α= °+ şi de aici ( ) 30m LPC α= °+ . Dar

1 1( ) ( ) ( )m PA L m NPL m PLA α= − = şi analog 1( )m PA L α= , adică pentagonul

1 2AAMNA este inscriptibil. Analog, se arată că pentagoanele 1 2BB NLB respectiv

1 2CC LMC sunt inscriptibile.

2) Fie ' ' '

1 2 3, ,R R R razele cercurilor circumscrise pentagoanelor 1 2 1 2, ,AAMNA BB NLB

1 2CC LMC . Atunci, 31 2' ' '1 2 3

RR Ra b cR R R⋅ = ⋅ = ⋅ (unde a,b,c sunt lungimile laturilor

triunghiului ABC). Demonstraţie. Avem: 1 1( ) ( ) ( ) 60m B BC m B BL m LBC β= + = °+ (deoarece

1 ,B BL LNM≡ 1BLNB fiind patrulater inscriptibil). Analog, 2( ) 60m C CB γ= °+ , de

unde ( ' ) 180 (60 ) (60 )m BA C β γ α= °− ° + − °+ = . Teorema sinusurilor aplicată

triunghiurilor 'A BC , respectiv ANM ne dă: '1 12 , 2

sin sin

a NMR R

α α= = , de unde

1'1

R NM

aR= , adică 1

'1

RNM a

R= ⋅ . Analog, 32

' '2 3

,RR

NL b LM cR R

= ⋅ = ⋅ şi cum

NM NM LM= = rezultă concluzia. 3) Perechile de triunghiuri ( 'A BC ,ANM), ( 'B CA ,BLN), ( 'C AB ,CMN) sunt respectiv asemenea. Demonstraţie. Avem ( ' ) ( ) , ( ) 3 0m B AC m N AM m LPCα α= = = ° + .

Atunci, 1( ) ( ) ( ) 60 ( )m ANM m NQM m NMQ m B BCβ= + = °+ = , deci triunghiurile

'A BC şi ANM sunt asemenea . Analog se arată şi celelalte două asemănări. 4) Triunghiurile PQR şi LMN sunt omologice. Demonstraţie: Dreptele PL, MQ, respectiv NR sunt mediatoarele laturilor triunghiului echilateral LMN, deci sunt concurente, ceea ce arată că triunghiurile PQR şi LMN sunt omologice. 5) Laturile triunghiului ABC şi ale triunghiului Morley corespunzător LMN, sunt antiparalele în raport cu unghiurile sub care se văd laturile respective ale triunghiului ABC din vârfurile omoloage ale triunghiului LMN. Demonstraţie. Avem 1( ) ( )m NB L m LBC β= = , deci patrulaterul 1 2BBC C este

inscriptibil, adică dreptele 1 2 ,BC BC sunt antiparalele în raport cu unghiul BLC . Analog

se arată că 1 2C A este antiparalelă cu CA faţă de unghiul CMA şi 1 2A B este antiparalelă

cu AB faţă de unghiul .ANB 6) Dacă R este lungimea razei cercului circumscris triunghiului ABC , atunci lungimea laturii triunghiului lui Morley corespunzător este egală cu 8 sin sin sinR α β γ . Demonstraţie. Soluţia 1. O primă demonstraţie rezultă chiar din teorema lui Morley.

Page 465: Barbu_TEOREME_FUNDAMENTALE(2008).pdf

472

Soluţia 2. Avem ( ) 60 , ( ) 120m BNL m BLCα α= °+ = °+ , de unde sin sin

NL BL

β γ= ,

2 sin3

sin sin(120 ) sin(60 )

BL a R αγ α α= =

°+ °−, deci:

2 sin3 sin sin

sin(60 ) sin(60 )

RNL

α β γα α

= =°+ ⋅ °−

24 sin (3 4sin )sin sin

cos2 cos120

R α α β γα−

− °, de unde rezultă că 8 sin sin sinNL R α β γ= .

7) Trisectoarele unghiurilor exterioare ale unui triunghi ABC determină un triunghi echilateral. Demonstraţie.

Vom demonstra teorema considerând următoarele cazuri: a) triunghiul ABC este ascuţitunghic. Pe laturile triunghiului echilateral PQR construim triunghiurile isoscele ' , ' , 'P QR Q RP R PQ având unghiurile de la bază de măsură

60 ,60α β° + ° + respectiv 60 γ° + astfel încât 60α β γ+ + = ° şi

30 , 3 0 , 3 0 .α β γ< ° < ° < ° Intersecţiile laturilor triunghiurilor ' , ' , 'P QR Q RP R PQ

determină vârfurile unui triunghi ABC (Fig. 472). Arătăm că laturile celor trei triunghiuri isoscele sunt trisectoarele exterioare ale unghiurilor triunghiului ABC. Avem:

( ' ) 180 (120 2 ) 60 2 ,m PR Q γ γ= °− °+ = °− de unde rezultă că 1

( ' ) 30 ;2m PR Q γ= °−

( ) 60m ARB α β= °+ + = 120 90 (30 )γ γ° − = °+ ° − = 1

90 ( ' ).2m BR A° + Datorită simetriei şi

a ultimului rezultat avem că R este centrul cercului înscris în triunghiul ' .AR B Analog, se arată că Q şi P sunt centrele cercurilor înscrise în triunghiurile 'AQ C şi ' .BP C Atunci,

' ' ,BAR RAB R AT≡ ≡ adică RA şi 'R A sunt trisectoarele exterioare ale unghiului .BAC

Observaţii: Avem: 1

( ' ) 180 ( ' ) 180 (30 ) 1502

m R RQ m PR Q γ γ γ= °− − = °− °− − = ° ,

A

B

C

P

Q R

T

P '

Q' R '

Fig. 472

Page 466: Barbu_TEOREME_FUNDAMENTALE(2008).pdf

473

( ') 180 (30 ) (150 ) 60 ,m RAR γ β α= °− °− − °− = °− de unde 3 , 3A Bα β= = şi 3 .C γ=

Deoarece 30 , 30 , 30α β γ< ° < ° < ° rezultă că triunghiul ABC este ascuţitunghic.

b) Fie triunghiul echilateral PQR. Unghiurile de la baza triunghiurilor isoscele satisfac

relaţiile: 60 , 30 .α β γ α+ + = ° = ° Triunghiul 'BP C considerăm că are vârful 'P la infinit

(Fig. 473). Avem : 1

( ) 60 90 90 ( ' )2

m BPC m BP Cα β= °+ + = ° = ° + (unde

( ' ) 0m BP C = ° ), relaţie care arată că P este centrul cercului înscris în triunghiul ' .BP C

Analog ca în cazul precedent se arată că R şi Q sunt centrele cercurilor înscrise în triunghiurile 'AR B respectiv ' .AQ C Deoarece 3 3 30 90A α= = ⋅ ° = ° rezultă că triunghiul ABC este dreptunghic. c) Pe laturile triunghiului echilateral PQR construim triunghiurile isoscele

' , ' , 'PQ R Q RP R PQ care au unghiurile de la bază de măsuri 120 ,60 ,60α β γ° − ° + ° + ,

astfel încât 60 , 30 , 30α β γ α β γ+ + = ° > ° + < ° (Fig. 474) . Avem: ( ' ) 2 60 ,m RP Q α= − °

A

B C

P

Q R

P '

Q'

R '

Fig. 473

A

B C

P

Q R

P '

Q'

R '

Fig. 474

α

β γ

Page 467: Barbu_TEOREME_FUNDAMENTALE(2008).pdf

474

1( ' ) 30 ,

2m RP Q α= − ° ( ) 60 120= + °+ = °− =m BPC γ β α

190 ( 30 ) 90 ( ' )

2m BPCα°− − ° = °−

Ultima ecuaţie arată că punctul P este centrul cercului exînscris corespunzător punctului 'P al triunghiului ' .BP C Deci, laturile triunghiurilor isoscele sunt trisectoarele unghiurilor

exterioare ale triunghiului ABC.

Observaţie: Triunghiul PQR determinat de trisectoarele exterioare ale triunghiului ABC se numeşte triunghiul Morley exterior.

III.43. Triunghiul lui Grebe

„Două linii paralele se întâlnesc la infinit – cred şi ele în aceasta.” - S.Lec219

Pe laturile triunghiului ABC se construiesc în exterior pătratele C BBCA A , C AACB B şi

B AABC C . Fie ' C A A BA B B C C= ∩ , ' A B B CB C C A A= ∩ şi ' B C A CC A A B B= ∩ .

Triunghiul ' ' 'A B C se numeşte triunghiul lui Grebe (Fig. 475).

219 Stanislaw Lec (1909-1966) – poet polonez

A

B C

P aC

aM

aB

bC

cB

1O

2O 3O

aK

bK

cK

bA

K

A'

B' C' cA

Q R

Fig. 475

Page 468: Barbu_TEOREME_FUNDAMENTALE(2008).pdf

475

1) Dreptele ', 'AA BB şi 'CC sunt concurente în punctul lui Lemoine al triunghiului ABC .

Demonstraţie. Fie aM mijlocul laturii BC , a A AP AM B C= ∩ şi 'aK AA BC= ∩ .

Este cunoscut faptul că a A AAM B C⊥ (vezi „Triunghiurile Vecten”). Patrulaterul

'A AAB A C fiind inscriptibil (deoarece ( ') ( ') 180A Am AB A m AC A+ = ° ) rezultă

' A A AAA C C B A≡ (1). Cum ' 'AB A B rezultă ' ' 'a ABAK B A A C A A≡ ≡ (2).

Din relaţiile (1) şi (2) rezultă că A A aAB C BAK≡ (3). În triunghiul AAPB avem:

( ') ( ' ) ( ) 90A Am PAA m A AB m AB P+ + = ° (4). Din 'a aK AM PAA≡ (5)

(unghiuri opuse la vârf) şi ( ) 90Am B AC = ° rezultă

( ' ) ( ) ( ) 90A a a am A AB m K AM m M AC+ + = ° (6). Din relaţiile (4), (5) şi (6) rezultă

( )A Am AB C ≡ ( ),am M AC care împreună cu relaţia (3) dă ( ) ( )a am BAK m M AC= ,

relaţie care arată că aAK este simediană în triunghiul ABC . Deci, dreapta 'AA trece prin

punctul K al lui Lemoine al triunghiului ABC. Analog se arată că dreptele 'BB şi 'CC trec prin punctul K. 2) Triunghiurile ABC şi ' ' 'A B C sunt omotetice, centrul de omotetie fiind punctul lui Lemoine al triunghiului ABC . Demonstraţie. Deoarece ' 'AB A B , ' 'BC B C şi ' 'AC A C rezultă că triunghiurile ABC

şi ' ' 'A B C şi cum ' ' ' AA BB CC K∩ ∩ = rezultă că triunghiul ABC şi triunghiul lui Grebe

sunt omotetice, centrul omotetiei fiind punctul lui Lemoine al triunghiului ABC .

3) Consecinţă: Centrul cercului circumscris triunghiului Grebe aparţine axei Brocard a triunghiului ABC. Demonstraţie. Deoarece triunghiurile ABC şi ' ' 'A B C sunt omotetice, rezultă că prin omotetia considerată cercurile lor circumscrise se corespund, deci centrul cercului circumscris triunghiului Grebe este coliniar cu centrul cercului circumscris (O) al triunghiului ABC şi cu centrul omotetiei, punctul lui Lemoine (K). Cu alte cuvinte, centrul cercului circumscris triunghiului Grebe aparţine axei Brocard OK a triunghiului ABC. 4) Fie 1 2 3, ,O O O centrele circumscrise triunghiurilor A AAB C , B BBC A , respectiv

C CCA B . Triunghiurile 1 2 3OO O şi ABC sunt omotetice, centrul omotetiei fiind punctul

lui Lemoine al triunghiului ABC . Demonstraţie. Deoarece A aparţine cercului circumscris triunghiului A AAB C şi

( ')Am AB A rezultă că 1O se află la mijlocul segmentului 'AA . Analog, 2O şi 3O sunt

mijloacele segmentelor 'BB , respectiv 'CC . Astfel, 1 2OO , 2 3O O şi 3 1O O sunt linii

mijlocii în trapezele ' 'ABB A , ' 'BCC B respectiv ' 'ACC A , deci 1 2OO AB , 2 3O O BC

şi 3 1O O AC , iar cum 1 2 3 O A O B O C K∩ ∩ = (vezi proprietatea (2)) rezultă că

triunghiurile 1 2 3OO O şi ABC sunt omotetice, centrul omotetiei fiind punctul lui Lemoine

K. 5) Consecinţă: Triunghiurile 1 2 3OO O şi triunghiul lui Grebe al triunghiului ABC sunt

omotetice, centrul omotetiei fiind punctul lui Lemoine al triunghiului ABC . Demonstraţia rezultă din teoremele 2) şi 4), ţinând cont că relaţia de omotetie este tranzitivă.

Page 469: Barbu_TEOREME_FUNDAMENTALE(2008).pdf

476

6) Triunghiul lui Grebe ' ' 'A B C şi triunghiul ortic a b cH H H al triunghiului ABC sunt omologice. Demonstraţie. Fie " ' ' 'aA A H B C= I , " ' ' ',bB B H A C= I " ' ' 'cC C H A B= I ,

' 'R BC A B= I , ' 'Q BC A C= I . Avem: " '

' "= =a

a

RHA B

C A H Q

cossin

cossin

++= =

+ +

a

a

cc B

RB BH BbH C CQ

b CC

21 sin cos sin 2 sin 2

1 sin cos sin 2 sin 2

c B B C c B

b C C B b C

+ ⋅ + ⋅ ⋅ = + ⋅ +

. Analog se arată că: 2

" ' 2 sin 2

' " 2 sin 2

B C a C

A B c A

+ = +

şi 2

" ' 2 sin 2

' " 2 sin 2

C A b A

B C a B

+ = +

. Atunci, " ' " ' " '

1' " ' " ' "

A B B C C A

C A A B B C⋅ ⋅ = şi din reciproca teoremei lui

Ceva rezultă că dreptele ' , ' , 'a b cA H B H C H sunt concurente, deci triunghiurile ' ' 'A B C

şi a b cH H H sunt omologice.

III.44. Triunghiul lui Malfatti220 „Este suficient să arăţi, că un lucru oarecare este imposibil, că îndată se va găsi matematicianul care-l va face.” - W. W. Sawyer221 Trei cercuri aflate în interiorul unui triunghi ABC astfel încât fiecare este tangent la celelalte două şi la două laturi ale triunghiului se numesc cercuri Malfatti. Fie , ,Γ Γ ΓA B C

centrele cercurilor Malfatti şi , ,A B CT T T punctele de tangenţă dintre cercurile Malfatti (Fig.

476). Triunghiul Γ Γ ΓA B C se numeşte triunghiul Malfatti.

1) Dacă 1 2 3, ,r r r sunt razele cercurilor Malfatti, atunci laturile triunghiului Malfatti au

lungimile 1 2 2 3 3 1, ,r r r r r r+ + + .

220 Gian Francesco Malfatti (1731-1806) – matematician italian, profesor la Universitatea din Ferrara, contribuţii

în geometrie, algebră şi teoria probabilităţilor 221 Warwick Sawer (1911- ) –matematician englez, profesor la Universitatea din Toronto.

B

A

C

X

Fig. 476

I Z

Y

1A

2A

1B

1C

2C

ΓA

ΓB

ΓC

2B

AT

BT

CT

2r

3r 1r

Page 470: Barbu_TEOREME_FUNDAMENTALE(2008).pdf

477

2) Dreptele , ,A B C

A B CΓ Γ Γ sunt concurente în centrul cercului circumscris triunghiului ABC.

Demonstraţie. Deoarece cercurile sunt tangente la câte două laturi ale triunghiului rezultă că dreptele , ,A B CA B CΓ Γ Γ sunt bisectoarele unghiurilor triunghiului ABC, deci sunt concurente în centrul cercului circumscris triunghiului ABC. 3) Dreptele , ,Γ Γ Γ

A A B B C CT T T sunt concurente.

Demonstraţie. Deoarece 32 1

3 1 2

1Γ ΓΓ

⋅ ⋅ = ⋅ ⋅ =Γ Γ Γ

B C C AA B

A C B A C B

T T rT r r

T T T r r r, rezultă din reciproca teoremei

lui Ceva că dreptele , ,Γ Γ ΓA A B B C CT T T sunt concurente.

4) Razele cercurilor Malfatti au lungimile: 1

1 14 4

,21

4

B Ctg tg

rr

Atg

+ + = ⋅

+

2

1 14 4

,21

4

A Ctg tg

rr

Btg

+ + = ⋅

+ 3

1 14 4

214

+ + = ⋅

+

A Btg tg

rr

Ctg

.

Demonstraţie. Fie 1 2 1 2 1 2, ,= = =A A x B B y C C z . Din trapezul dreptunghic 1 2Γ ΓB CA A

rezultă 2 2 2

3 2 3 2( ) ( )+ − = +x r r r r , de unde 2

2 34=x r r şi analog 2

1 34=y r r , 2

1 24=z r r , iar de

aici 1 2 3, ,2 2 2

= = =yz xz xy

r r rx y z

. Fie , ,a b cC C C punctele de tangenţă ale cercului înscris în

triunghiul ABC cu laturile BC, CA, respectiv AB. Din 1 1 12( )+ + = + +a b c

x y z AC BC CC şi

1 1 1 2 ,+ = + =a c b b

BC C C BC C B y rezultă

1 2

− +=

a

x y zAC . Fie P proiecţia

punctului ΓB pe

aIC . Din triunghiul

dreptunghic ΓB

IP rezultă :

( )( )

22 2

2=

− −=

− + − +

r rB ry xztg

x y z y x y z Analog

se obţin egalităţile: 2

( )2

−=

− + +A rx yz

tgx x y z

şi

2

( )2

−=

+ −C rz xy

tgz x y z

. Folosind relaţiile precedente şi egalitatea

12 2 2 2 2 2

A B A C C Atg tg tg tg tg tg⋅ + ⋅ + = obţinem: ( ) ( )22 2 0+ + − + + + =r x y z r xy yz zx xyz

sau ( )( )

22

2

−−=

− + + −

r x rrx yz

x y z x r x, de unde

( )( )

2

2 2

−=

r x rAtg

x r x. Din egalitatea precedentă rezultă

ecuaţia 2 22 1 2 02 2

⋅ + − ⋅ − =

A Ax tg r tg x r care are singura soluţie acceptabilă:

1A 2A

P

I

Fig. 477

B C aC

Page 471: Barbu_TEOREME_FUNDAMENTALE(2008).pdf

478

14

Ax r tg

= +

. Analog, 14

By r tg

= +

şi 14

Cz r tg

= +

. Din relaţiile de mai sus

rezultă: 1 2 3

1 1 1 1 1 14 4 4 4 4 4

, ,2 2 21 1 1

4 4 4

+ + + + + + = ⋅ = ⋅ = ⋅

+ + +

B C C C A Btg tg tg tg tg tg

r r rr r r

A B Ctg tg tg

.

5) Dacă , ,

a b cr r r sunt razele cercurilor exînscrise corespunzătoare triunghiului ABC,

atunci: 2 3 3 1 1 2

1 2 3

1 23 1 1 2 2 3 2 3 3 1

, ,

2 1 2 1 2 1

2 1 2 1 2 1 2 1 2 1 2 1

− − −

− = − = − = − − − − − −

a b c

r r rr r rr rrr r r r r r

r r rr r r r rrr rr r r r r r r

.

Demonstraţie. Fie 14=

Atg t , 2 ,4

=B

tg t 34=

Ctg t . Din 1

4 4 4

A B Ctg + + =

rezultă

1 2 3 1 2 2 3 3 1 1 2 31 0− − − − − − + =t t t t t t t t t t t t , care împreună cu expresiile razelor scrise în

aplicaţia precedentă ne dau: 1

12 3

2 1 2

1− = ⋅

+t

r t rr r (*) şi analoagele. Dar,

2 22 3

2 3

( )( )1 1

2 2 4a

t tp B Cr r ctg ctg r r

p a t t

− −= ⋅ = ⋅ ⋅ = ⋅

−, de unde se obţine egalitatea

( )31 1 2

1 2 3

2 3 1 1 2 3

2(1 )(1 )

2 2 1 1 1 1 2a

tt t tr rr r t t

t t t t t t− = + + ⋅ ⋅ = ⋅ ⋅ ⋅

+ + + + şi utilizând relaţiile (*) rezultă

concluzia. 6) Raza cercului înscris în triunghiul ABC în funcţie de razele cercurilor Malfatti este

egală cu: 1 2 3

1 2 3 1 2 3

2=

+ + − + +

r r rr

r r r r r r .

Demonstraţie. Din expresiile razelor 1 2 3, ,r r r rezultă: 2 321,

4

r rAtg

r= − 1 32

1,4

r rBtg

r= −

1 221

4

r rCtg

r= − . Ţinând seama de egalitatea

4 4 2 4

A B Ctg tg

π + = −

rezultă

14 4 4 4 4 4

∑ + ∑ ⋅ = + ⋅ ⋅A A B A B C

t g t g t g t g t g t g adică

1 3 2 31 2 2 221 1 1

∑ − + ∑ − ⋅ − =

r r r rr r

r r r

2 3 1 31 2 2 221 1 1 1

+ − ⋅ − ⋅ −

r r rrrr

r r r, deci:

1 2 31 2 3 1 2 3 1 2 2 3 3 12

2 2( ) 0− + + + + + =

rr rrr r r r r rr r r r r

r r de unde obţinem

1 2 3 1 2 3

1 2 3

1

2

r r r r r r

r r r r

+ + ± + += . Deoarece

2 321

4= −

A r rtg

r, atunci

Page 472: Barbu_TEOREME_FUNDAMENTALE(2008).pdf

479

2 3 1 2 3

4

r r r r rAtg

r

+ ± + += şi deoarece 1

4

Atg ≤ rezultă 2 3 1 2 3

14

r r r r rAtg

r

+ −= şi

2 1 3 1 2 3

1 2 3

1

2

r r r r r r

r r r r

+ + − + += .

7) Fie X, Y şi Z mijloacele segmentelor 1 2 1 2, ,A A B B respectiv 1 2CC . Dreptele AX, BY, CZ sunt concurente. Demonstraţie.

Avem: 321 2(

1 11) ( ) ( ) ( )

2 2 2

= − + = + − − − = + −

rrBX a BA A C a p b p c a IB IC

r r sau

12 sin 4 sin cos sin cos

2 2 4 4 4sin sin2 2

r r A B C B CBX R A R

B C

+

= + − =

. Procedând analog se

obţine: 1 2

1( ) 4 sin sin cos cos

2 2 2 4 4

+= − + =

A B C B CXC a BA A C R , de unde

cos sin4 4 4

sin cos4 4 4

B C Ctg

BX

B C BXCtg

= = . Analog se arată că 4

4

Atg

CY

CYAtg

= şi 4

4

Btg

AZ

AZBtg

= . Deoarece

1⋅ ⋅ =BX CY AZ

XC YA ZB, din reciproca teoremei lui Ceva rezultă că dreptele AX, BY, CZ sunt

concurente. 8) Dacă X, Y şi Z sunt mijloacele segmentelor 1 2 1 2, ,A A B B respectiv 1 2CC atunci dreptele

,A BT X T Y şi CT Z sunt concurente în centrul cercului înscris în triunghiul lui Malfatti.

Demonstraţie. Tangenta comună cercurilor C 2( , )B rΓ şi C 3( , )C rΓ trece prin mijlocul

segmentului 1 2A A (adică prin punctul X ) şi totodată prin centrul cercului înscris Iµ în

triunghiul lui Malfatti, deoarece A B C

I T I T I Tµ µ µ≡ ≡ .

Observaţii:

1) Triunghiul A B CT T T este triunghiul de contact al triunghiului lui Malfatti.

2) Din proprietatea (3) rezultă că dreptele , ,Γ Γ ΓA A B B C CT T T sunt concurente în punctul lui

Gergonne al triunghiului lui Malfatti. 3) Punctul Iµ este centrul radical al cercurilor lui Malfatti.

9) Pentru orice punct M din planul triunghiului ABC sunt adevărate relaţiile:

1 21 2, ,a bA a B b

a a b b

r r r rr rM MA MI M MB MI

r r r r

− −Γ = + Γ = +

uuuuur uuur uuuur uuuuur uuur uuuur3 3−

Γ = +uuuuuur uuuur uuuur

cC c

c c

r r rM MC MI

r r,

(unde , ,a b cI I I sunt centrele cercurilor exînscrise corespunzătoare triunghiului ABC şi

, ,a br r respectiv cr razele lor).

Page 473: Barbu_TEOREME_FUNDAMENTALE(2008).pdf

480

Demonstraţie. Avem: 1 ,A

a a

A r

AI r

Γ= de unde 1

1

,A

A a a

A r

I r r

Γ=

Γ − deci

1 1 .aA a

a a

r r rM MA MI

r r

−Γ = +

uuuuur uuur uuuur Analog se arată celelalte egalităţi.

Observaţie: Ţinând cont că a

rpr

p a=

− putem scrie

1 1

1 1 1.A a

a

p aM MA MI

r r r rp

−Γ = − +

uuuuur uuur uuuur

10) Coordonatele baricentrice absolute ale centrelor cercurilor lui Malfatti corespunzătoare triunghiului ABC sunt:

1

1 12 ; ; ,A

a

rp a b cr r

Γ − − 2

1 1;2 ;B

b

a rp b cr r

Γ − −

, respectiv

3

1 1; ;2 .C

c

a b rp cr r

Γ − −

Demonstraţie. Deoarece coordonatele baricentrice absolute ale centrului cercului

A – exînscris sunt ; ;2( ) 2( ) 2( )a

a b cI

p a p a p a

− − − −

rezultă că pentru orice punct M este

adevărată relaţia:

1 1

1 1 1

2( ) 2( ) 2( )A

a

p a a b cM MA MA MB MC

r r r rp p a p a p a

− −⋅ Γ = − + + + − − −

uuuuur uuur uuur uuur uuuur sau

1 1

2 1 12 ,A

a

rpM rp a MA bMB cMC

r r r

Γ = − − + +

uuuuur uuur uuur uuuur de unde rezultă concluzia.

11) Coordonatele baricentrice absolute ale punctelor de tangenţă dintre cercurile lui

Malfatti sunt: 2 3 1 3

1 1 1 1 1 1 1 1; ; , ; ;A B

b c a c

a bT T

rp r r r r r r rp r r

− − − −

, respectiv

1 2

1 1 1 1; ; .C

a b

cT

r r r r rp

− −

Demonstraţie. Deoarece 2

3

B A

A C

r

r

Γ Γ=

Γ Γ rezultă că pentru orice punct M din planul triunghiului

ABC avem: 3 2

2 3

B C

A

r M r MM

r r

Γ + ΓΓ =

+

uuuuur uuuuuruuuuur

sau ţinând cont de proprietatea precedentă:

2

2 2 3

( ) 1 1 1 1bA

b b c

r r aM MA MB MC

r r rp r r r r

+Γ = + − + −

uuuuur uuur uuur uuuur de unde rezultă concluzia.

Page 474: Barbu_TEOREME_FUNDAMENTALE(2008).pdf

481

12) Dreptele , ,A B CAT BT CT sunt concurente.

Demonstraţie. Ecuaţia dreptei AAT în coordonate baricentrice este:

2 3

1 0 0 0

1 1 1 1

=

− −b c

x y z

a

rp r r r r

sau 3 2

1 1 1 10.

c b

y zr r r r

− − − =

Analog ecuaţiile dreptelor

BBT şi CCT sunt: 1 3

1 1 1 10

a c

x zr r r r

− − − =

respectiv

2 1

1 1 1 10.

b a

x yr r r r

− − − =

Deoarece

3 2

1 3

2 1

1 1 1 10

1 1 1 10 0

1 1 1 1 - 0

− −

− − =

− +

c b

a c

b a

r r r r

r r r r

r r r r

rezultă că dreptele ,A B

AT BT şi C

CT sunt

concurente.

Observaţii:

1) Punctul de concurenţă al dreptelor , ,A B C

AT BT CT se numeşte primul punct al lui

Malfatti 1( ).µ

2) Triunghiurile ABC şi A B CT T T sunt omologice, centrul de omologie fiind primul punct al

lui Malfatti. 13) Dreptele , ,

a a b b c cI T I T I T sunt concurente.

Demonstraţie. Ţinem cont de coordonatele baricentrice ale centrelor cercurilor exînscrise

, , , , , ,2( ) 2( ) 2( ) 2( ) 2( ) 2( )a b

a b c a b cI I

p a p a p a p a p a p a

−− − − − − − −

, ,2( ) 2( ) 2( )c

a b cI

p a p a p a

− − − −

şi scriem ecuaţiile dreptelor ,a a b bI T I T şi .

c cI T Utilizând

condiţia de concurenţă a trei drepte ( ) : 0i i i id a x b y c z+ + = 1,3i = şi anume

1 1 1

2 2 2

3 3 3

0

a b c

a b c

a b c

= rezultă concluzia.

Observaţie: Punctul de concurenţă al dreptelor , ,A B C

AT BT CT se numeşte al doilea punct

al lui Malfatti 2( ).µ

Page 475: Barbu_TEOREME_FUNDAMENTALE(2008).pdf

482

III.45. Triunghiul lui Schroeter222

„Atâtea clăile de fire stângi Găsi-vor gest închis sâ le rezume, Să nege, dreapta, linia ce frângi

Ochi în virgin triunghi tâiat spre lume?”

Ion Barbu223

Fie a b c

M M M şi a b c

H H H triunghiurile median, respectiv ortic ale unui triunghi

neisoscel şi nedreptunghic ABC, * * , ,= ∩ = ∩b c b c c a c a

A M M H H B M M H H * .= ∩

a b a bC M M H H Dreptele * * *, ,AA BB CC sunt paralele între ele şi perpendiculare

pe dreapta lui Euler a triunghiului ABC. Demonstraţie.

Fie 1C cercul lui Euler al triunghiului ABC,

2C cercul circumscris patrulaterului

a bH HH C

şi 3C cercul circumscris patrulaterului

a bM OM C (O este centrul cercului circumscris

triunghiului ABC), iar = ∩2 3

.D C C Evident, 1 2

, ,a bH H C C∈ ∩

1 3,a bM M C C∈ ∩ şi

∈ ∩2 3

, .C D C C Atunci ,a b a b

H H M M şi CD sunt concurente fiind axele radicale

corespunzătoare perechilor de cercuri considerate. Fie * .a b a b

C H H M M CD= ∩ ∩

( ) ( ) 180 ( )= = °− a a

m M OD m H HD m BCD şi cum a a

HH OM rezultă că punctele H,

O şi D sunt coliniare. Deoarece patrulaterul a

HH CD este inscriptibil rezultă

( ) 180 ( ) 90 ,= °− = ° a

m HDC m HH C deci *.HO CC⊥ Analog se arată că *AA HO⊥ şi * ,BB HO⊥ deci * * *.AA BB CC

Observaţie: Triunghiul * * *A B C se numeşte triunghiul lui Schroeter.

222 Heinrich Schroeter (1829-1892) – matematician german, contribuţii în geometrie 223Ion Barbu (1895-1961) – matematician român, profesor la Universitatea din Bucureşti, contribuţii în algebră şi

geometrie

A

B C aM

bM cM

aH

bH

cH

Fig. 478

O

H

*A

*B *C

D

Page 476: Barbu_TEOREME_FUNDAMENTALE(2008).pdf

483

III.46. Triunghiul lui Ţiţeica224

„Ţiţeica era plin de vioiciune, fericit să-mi vorbească despre căminul său, radiind, cu privirea sa luminoasă şi discretă, aceeaşi magnifică sănătate morală... Înţelegeam că în el se reuneau continuu preocuparea datoriei de împlinit şi o euforie izvorâtă din conştiinţa datoriei împlinite...” – Henri Lebesgue225

Teorema lui Ţiţeica

Fie cercurile ( ) ( ) ( )ROCROCROC ,,,,, 332211 astfel încât 1 2 3 H C C C= I I ,

1 2 ,A C C= I 2 3 ,B C C= I 3 1 C C C= I . Cercul circumscris triunghiului ABC este

congruent cu cercurile 21 ,CC şi 3C .

Demonstraţie.

Patrulaterul HAOO 23 este romb, deoarece HOAOHOAO 3223 ≡≡≡ . De asemenea

patrulaterul HBOO 31 este romb, deci 132 |||| BOHOAO şi deoarece 2 1( )AO BO R≡ =

rezultă că patrulaterul 21OABO este paralelogram, deci 21OOAB ≡ . Analog , 32OOBC ≡

şi 13OOCA ≡ , deci triunghiurile ABC şi 321 OOO sunt congruente, deci cercurile

circumscrise triunghiurilor ABC şi 321 OOO sunt congruente. Deoarece

1 2 3 ( )HO HO HO R≡ ≡ = rezultă că H este centrul cercului circumscris triunghiului

321 OOO , deci şi cercul circumscris triunghiului ABC are raza R .

Triunghiul 321 OOO se numeşte triunghiul lui Ţiţeica.

224 Gheorghe Ţiţeica (1873-1939) – matematician român, professor la Universitatea Bucureşti, membru al

Academiei Române 225 Henri Lebesgue (1875-1941) – matematician francez, contribuţii importante în analiza matematică

A

B C

A'

B' C'

H

1O

2O 3O

aH

bH cH

Fig. 479

Page 477: Barbu_TEOREME_FUNDAMENTALE(2008).pdf

484

1) Punctul H este centrul cercului circumscris triunghiului lui Ţiţeica 321 OOO .

Demonstraţia este evidentă întrucât 1 2 3 ( )HO HO HO R≡ ≡ = .

2) Punctul H este ortocentrul triunghiului ABC . Demonstraţie. Punctul 1O este simetricul lui 2O faţă de mijloacele segmentului HC , iar

ABOO ||21 , rezultă că ABCH ⊥ . Analog , ACBH ⊥ , deci H este ortocentrul

triunghiului ABC . Fie ',',' CBA punctele de intersecţie dintre 21 ,HOHO şi 3HO cu cercurile 21 ,CC

respectiv 3C .

3) Triunghiul ''' CBA este triunghiul anticomplementar al triunghiului ABC . Demonstraţie. Deoarece 1 1 2 2' 'HO O A HO O B R= = = = , rezultă că 21OO este linie

mijlocie în triunghiul ''BHA , deci ''||21 BAOO şi cum ABOO ||21 rezultă ' ' ||A B AB .

Deoarece 21OO este mediatoarea segmentului HC , rezultă că ' 'C A B∈ . Analog, punctele

',', CBA şi ', , 'A B C sunt coliniare şi ' ' || , ' ' ||B C BC A C AC , deci triunghiul ''' CBA este

triunghiul anticomplementar al triunghiului ABC . 4) Triunghiul lui Ţiţeica 321 OOO este omotetic cu triunghiul ABC , centrul de omotetie

fiind centrul cercului lui Euler al triunghiului ABC . Demonstraţie. Deoarece BCOOABOO ||,|| 3221 şi ACOO ||31 , iar triunghiul ABC şi

321 OOO sunt congruente, rezultă că triunghiurile sunt omotetice. Notăm cu litere mici

afixele punctelor corespunzătoare. Alegem un reper complex cu originea în centrul cercului circumscris triunghiului ABC , (0)O . Atunci, ( )H a b c+ + , iar centrul cercului lui Euler

9 2

a b cO

+ +

. Patrulaterele 123123 ,, HOCOHOBOHOAO fiind paralelograme rezultă

3 2 1 3, ,a h o o b h o o+ = + + = + 1 2c h o o+ = + , de unde 1 2 3 92( ) 4o o o a b c o+ + = + + = ,

( )1 9 9 94 2 2o o a o o a b c= − + = − = + , 2 92o o b a c= − = + , 3 92o o c a b= − = + . Deoarece

9

1 9

1a o

o o

−= − ∈

− rezultă ca punctele 9 1, ,A O O sunt coliniare. Analog, 9 2, ,B O O şi

9 3, ,C O O sunt coliniare, deci centrul de omotetie dintre triunghiurile ABC , 321 OOO este

centrul lui Euler. 5) Dreptele 32 ,, COBOAO sunt concurente în centrul cercului lui Euler al triunghiului

ABC . Demonstraţia rezultă din proprietatea precedentă. 6) Centrul cercului lui Euler al triunghiului ABC este mijlocul segmentelor

1 2 3, ,AO BO CO .

Page 478: Barbu_TEOREME_FUNDAMENTALE(2008).pdf

485

Demonstraţie. Din 9

1 9

1a o

o o

−= −

− rezultă 9 1 9a o o o− = − adică 9 9 1AO O O≡ . Analog

9 9 3BO O O≡ şi 9 9 1.CO O O≡

7) Triunghiul median al triunghiului Ţiţeica este omotetic cu triunghiul ABC , centrul de omotetie fiind ortocentrul triunghiului ABC . Demonstraţie. Dacă 111 CBA este triunghiul median al triunghiului 321 OOO atunci

ABOOBA |||| 2111 şi omoloagele, iar BHBAHA ∈∈ 11 , şi CHC ∈1 , deci triunghiurile

111 CBA şi ABC sunt omotetice, iar H este centrul de omotetie.

III.47. Triunghiurile lui Napoleon. Punctele lui Fermat226

„The last thing we want from you, general Lagrange, is a lesson in a geometry.” - Napoleon227

Pe laturile unui triunghi ABC se construiesc în exterior triunghiurile echilaterale 1A BC ,

1AB C şi 1ABC , cercurile lor circumscrise având centrele aN , bN şi cN . Triunghiul

a b cN N N se numeşte triunghiul exterior al lui Napoleon. Dacă triunghiurile echilaterale

2A BC , 2AB C şi 2ABC se construiesc în interiorul triunghiului ABC , atunci centrele lor

'a

N , 'bN şi '

cN sunt vârfurile unui triunghi numit triunghiul interior al lui Napoleon.

Triunghiul 1 1 1A B C se numeşte primul triunghi al lui Fermat, iar triunghiul 2 2 2A B C se

numeşte al doilea triunghi al lui Fermat.

1) Segmentele 1AA , 1BB , 1CC sunt congruente.

Demonstraţie. Din congruenţa triunghiurilor 1ABA şi 1C BC (deoarece 1AB BC≡ ,

226 Pierre de Fermat (1601-1665) – matematician francez, contribuţii în teoria probabilităţilor şi teoria numerelor 227 Napoleon Bonaparte (1769-1821) – cel mai important om politic şi militar după Revoluţia franceză, proclamat

în 1804 drept Împărat al francezilor

C B

A C1

A1

B1

F1

aN

bN cN

Fig. 480

Page 479: Barbu_TEOREME_FUNDAMENTALE(2008).pdf

486

1BA BC≡ şi 1 1ABA CBC≡ ) rezultă că 1AA ≡ 1CC (1). Din congruenţa triunghiurilor

1ACA şi 1BCB rezultă 1 1AA BB≡ (2). Din relaţiile (1) şi (2) rezultă 1 1 1AA BB CC≡ ≡ .

2) Cercurile circumscrise triunghiului 1A BC , 1ABC şi 1ABC au un punct comun.

Demonstraţie. Fie 1F al doilea punct de intersecţie dintre cercurile circumscrise

triunghiurilor 1BAC şi 1CB A . Avem

1 1( () 180 ) 180 60 120m BFC m BAC == − °− ° = °o ,

1 1( ) 180 ( ) 120m AFC m CB A == °− ° .

Atunci, 1 1 1( ) 360 [ ( ) ( )] 120m AF B m BFC m CF A= °− + = ° , deci

1 1( ) ( ) 180m AF B m AC B+ = ° ,

adică 1F aparţine şi cercului circumscris triunghiului 1ABC .

Observaţie: Punctul 1F se numeşte primul punct al lui Toricelli – Fermat.

3) Dreptele 1AA , 1BB şi 1CC sunt concurente în punctul 1F .

Demonstraţie. Deoarece 11 1 1( ( ( )

1) ) 602m BF A m BCA m BA == = o şi 1 1( ) 120m BF A = °

rezultă 1 1 1( () ) 180m AF B m BF A+ = ° , adică punctele A, 1F , 1A sunt coliniare, deci dreapta

1AA trece prin punctul 1F . Analog se arată că dreptele 1BB şi 1CC trec prin 1F .

4) Coordonatele unghiulare ale punctului

1F sunt egale cu 120° , dacă unghiurile triunghiului ABC au măsura mai mică de 120° . Demonstraţia rezultă din cele de mai sus.

5) Dacă ( ) 1 2 0m B A C > o , atunci

1 1( ) ( ) 60m AFC m BFA= = ° şi

1( ) 120m BFC = ° . Demonstraţie. În cercul circumscris patrulaterului 1 1ACB F avem:

1 1( () ) 60m AFC m AB C= = ° iar în cercul

circumscris patrulaterului 1 1ABC F avem

1 1( () ) 6 0m A F B m A C B= = ° şi

1 1 1( () 180 ) 120m BFC m BFC= °− = ° .

6) Triunghiul exterior al lui Napoleon este echilateral. Demonstraţie. Notăm cu , ,a b c lungimile laturilor BC, CA respectiv AB. Avem :

3

2 3 33 2c

c cAN = = ,

3

3b

bAN = , iar ( () ) 60

b cm N AN m BAC= + ° . Din teorema

cosinusului în triunghiul b cN AN avem

Fig. 481

A

B C

1A

1B

1C 1F

Page 480: Barbu_TEOREME_FUNDAMENTALE(2008).pdf

487

2 22 2 2 2 1 3

2 cos( 60 ) cos sin3 3 2 2

b c b c b c

b c bcN N AN AN AN AN A A A

+= + − ⋅ ⋅ + ° = − − , adică

[ ] [ ]

2 2 2 2 22 2 2 2

( )1 3 2 3

23 6 3 6 3b c ABC ABC

b c a b cN N b c a A A

+ + += − + − + ⋅ ⋅ = − ⋅ . Simetria

rezultatului precedent ne conduce la concluzia a b b c c a

N N N N N N= = , deci triunghiul

exterior al lui Napoleon este echilateral. 7) Cercurile circumscrise triunghiurilor 2A BC , 2AB C , 2ABC au un punct comun 2F .

Demonstraţie. Fie 2F al doilea punct de intersecţie

dintre cercurile circumscrise triunghiurilor 2BA C şi

2AB C . Atunci, 2 2(( ) ) 60m CF B m BA C= = ° ,

2 2( () ) 60m B F C m BAC= = ° de unde rezultă că punctele

2F , B şi 2B sunt coliniare. Analog se arată că punctele

2F , A şi 2A sunt coliniare. Atunci, 2( ) 120m BF A = ° ,

deci 2 2( () ) 180m BF A m BC A+ = ° , adică 2F aparţine

cercului circumscris triunghiului 2ABC .

Observaţie: Punctul 2F se numeşte al doilea punct Toricelli – Fermat.

8) Dreptele 2AA , 2BB şi 2CC sunt concurente în punctul 2F .

Demonstraţie. Din aplicaţia precedentă 2 2 2 = ∩F AA BB . Deoarece patrulaterul 2 2BC AF

este inscriptibil rezultă 2 2 2≡BC F BAF . Din congruenţa triunghiurilor 2BC C şi 2BAA

rezultă 2 2≡BC C BAA de unde

2 2 2 2 2( ( ( () ) ) ) 180+ = + = °m BC F m BC C m BAA m BAF , deci

punctele 2F , 2C , C sunt coliniare.

9)Segmentele 2AA , 2BB şi 2CC sunt congruente.

Demonstraţie: Din congruenţa triunghiurilor 2ACA şi 2B CB ( deoarece 2AC B C≡ ,

2A C BC≡ şi 2 2( () ) 60 ( )= = °−m ACA m BCB m C ) rezultă 2 2≡AA BB (1), iar congruenţa

triunghiurilor 2AA B şi 2CC B rezultă 2 2≡AA CC (2). Din relaţiile (1) şi (2) rezultă că

2 2 2 .≡ ≡AA BB CC

10) Triunghiul interior al lui Napoleon ' ' 'a b c

N N N este echilateral.

Demonstraţie. Avem ' 3

3=cc

AN , ' 3

3=

b

bAN iar ' '( ) ( ) 60= − °

b cm N AN m BAC . Din

teorema cosinusului în triunghiul ' 'b cN AN rezultă

2 2 2' ' 2

[ ]

2 3

6 3

+ += − ⋅

b c ABC

a b cN N A .

Simetria relaţiei precedente conduce la ' ' ' ' ' ' .= =a b b c c a

N N N N N N

A

B C

A2

B2

C2

F2

'aN

'bN

'cN

Fig. 482

Page 481: Barbu_TEOREME_FUNDAMENTALE(2008).pdf

488

Observaţie: Inegalitatea [ ]2 2 2 4 3 ⋅+ + ≥

ABCa b c A este echivalentă cu

2 2 2 2 2 2 2 2 2( ) ( ) ( ) 0a b b c c a− + − + − ≥ , evident adevărată (unde am folosit formulele lui

Heron în exprimarea ariei triunghiului ABC ).

11) Primul punct Fermat verifică egalitatea: 1 1 1 1 1 1 1 1 12( ).+ + = + +F A F B FC F A F B FC

Demonstraţie. Deoarece patrulaterul 1 1F BAC , 1 1FCB A şi 1 1F AC B sunt inscriptibile, din

relaţia lui Schooten rezultă 1 1 1 1+=F A F B FC , 1 1 1 1+=F B F A FC şi 1 1 1 1+=FC F A F B relaţii

care prin sumare dau 1 1 1 1 1 1 1 1 12( ).+ + = + +F A F B FC F A F B FC

12) Dreptele 1 aA N , 1 b

B N şi 1 cC N sunt concurente în centrul cercului circumscris

triunghiului ABC . Demonstraţia este evidentă deoarece 1 a

A N , 1 bB N şi 1 c

C N sunt mediatoarele laturilor BC ,

CA respectiv AC .

13)Triunghiurile ABC şi 1 1 1A B C sunt ortologice.

Demonstraţie. Deoarece 1 1 1, ,⊥ ⊥ ⊥a b c

A N B C B N A C C N A B şi

1 1 1 =I Ia b c

A N B N C N O rezultă că triunghiurile ABC şi 1 1 1A B C sunt ortologice, O fiind

un centru de ortologice. 14) Triunghiul exterior al lui Napoleon

a b cN N N şi triunghiul 1 1 1A B C sunt ortologice.

Demonstraţia rezultă de mai sus. 15) Dreptele ' ' '

2 2 2, ,a b c

A N B N C N sunt concurente în centrul cercului circumscris

triunghiului ABC . Demonstraţia este evidentă deoarece ' ' '

2 2 2, ,a b cA N B N C N sunt mediatoarele laturilor

triunghiului ABC .

16) Triunghiurile antipodare ale punctelor lui Fermat corespunzătoare unui triunghi ABC sunt echilaterale. Demonstraţie. Vezi „Triunghiul antipodar”.

17)Dreptele , ,a b c

AN BN CN sunt concurente.

Demonstraţie. Soluţia 1. Fie ' , ' , 'a b c

A AN BC B BN AC C CN BA= = =I I I .

Atunci: [ ]

[ ]

s in ( 3 0 )' s in ( 3 0 )

' s in ( 3 0 ) s in ( 3 0 )a

a

A B N a

A C N a

A A B B N BB A A B B

A C A A C C N C A C C

⋅ ⋅ + ° ⋅ + °= = =

⋅ ⋅ + ° ⋅ + °,

' sin( 30 )

' sin( 30 )

CB BC C

B A BA A

⋅ + °=

⋅ + °,

' sin( 30 )

' sin( 30 )

AC CA A

C B CB B

⋅ + °=

⋅ + °, de unde rezultă că

' ' '1

' ' '

BA CB AC

A C B A C B⋅ ⋅ = şi conform reciprocei teoremei lui Ceva rezultă că dreptele

, ,a b c

A N B N C N sunt concurente.

Soluţia 2. Notăm cu litere mici afixele punctelor corespunzătoare. Avem: 601 ( )CA B°= ℜ ,

601 ( )AB C°= ℜ , 60

1 ( )BC A°= ℜ , (unde prin 60 ( )X Y°ℜ am notat rotaţia de centru X şi unghi 60° a

Page 482: Barbu_TEOREME_FUNDAMENTALE(2008).pdf

489

punctului Y ). Atunci, 1 ( )a c b cω= + − , 1 ( )b a c aω= + − , 1 ( )c b a bω= + − , unde

cos sin3 3

iπ π

ω = + şi 1 2 ( )

3 3a

b c a b c b cn

ω+ + + + −= = (1),

1 2 ( )

3 3b

a c b c a c an

ω+ + + + −= = (2), 1 2 ( )

3 3c

a b c a b a bn

ω+ + + + −= = (3). Ecuaţiile

dreptelor , ,a b c

AN BN CN sunt: ( aAN ): ( ) ( ) 0a a a aa n z a n z an an− − − + − = (4),

( bBN ): ( ) ( ) 0b b a bb n z b n z bn bn− − − + − = (5),( cCN ): ( ) ( ) 0c c c cc n z c n z cn cn− − − + − =

(6). Sumând ecuaţiile (4), (5), (6) - ţinând seama de relaţiile (1), (2), (3) precum şi de:

1=ω+ω , 012 =+ω−ω , 13 −=ω - rezultă o identitate, ceea ce arată că dreptele sunt concurente.

18) Triunghiul exterior al lui Napoleon a b c

N N N şi triunghiul 1 1 1A B C sunt ortologice. Demonstraţie. Utilizând notaţiile din teorema precedentă rezultă

3 3 3( 2 )

3b c

c b i c a bn n

− + − +− = şi 1

2 3( ) 3( 2 ) 3 3

2 2 3

b a c i b c i c a b c ba a

i

− + + − − + + −− = = ,

de unde 1

3

3b cn n i

ia a

∗−= ∈

− , deci

b cN N 1AA⊥ . Analog se arată că

a bN N 1CC⊥ şi

a cN N 1BB⊥ şi cum 1 1 1 1 A A B B C C F=I I rezultă concluzia.

19) Triunghiul exterior al lui Napoleon

a b cN N N şi triunghiul antipodar al primului

punct al lui Fermat sunt omotetice. Demonstraţie. Fie " " "A B C triunghiul antipodar corespunzător punctului 1F . Deoarece

1 " "AF B C⊥ şi b cN N 1AF⊥ (cf. teoremei precedente) rezultă că

b cN N " "B C . Analog se

arată că b aN N " "B A şi

a cN N " "A C , rezultă că triunghiurile

a b cN N N şi " " "A B C sunt

omotetice.

20)Triunghiurile ABC şi 1 1 1A B C au acelaşi centru de greutate. Demonstraţie. Notăm cu litere mici afixele punctelor corespunzătoare. Avem:

21 0+ + =b c aε ε , 2

1 0+ + =c a bε ε , 21 0+ + =a b cε ε , unde

1 3

2

− +=

iε . Adunând

relaţiile precedente membru cu membru obţinem: 2

1 1 1(1 )( ) ( ),+ + + = − + +a b c a b cε ε adică 1 1 1+ + = + +a b c a b c , deci triunghiurile ABC şi

1 1 1A B C au acelaşi centru de greutate.

21)Triunghiurile ABC şi a b c

N N N au acelaşi centru de greutate. Demonstraţie. Notăm cu litere mici afixele punctelor corespunzătoare. Avem:

1

3a

b c an

+ += , 1 ,

3b

a c bn

+ += 1

3c

a b cn

+ += , de unde rezultă

1 1 12( )

3a b c

a b c a b cn n n

+ + + + ++ + = =

2( )

3

a b c a b c+ + + + += a b c+ + (unde am

utilizat proprietatea precedentă).

Page 483: Barbu_TEOREME_FUNDAMENTALE(2008).pdf

490

Observaţie: Din proprietăţile precedente rezultă că triunghiurile 1 1 1A B C şi a b c

N N N au

acelaşi centru de greutate.

22) Triunghiurile ABC şi ' ' 'a b cN N N au acelaşi centru de greutate.

Demonstraţie analoagă celei precedente.

23) Triunghiurile lui Napoleon interior şi exterior au acelaşi centru de greutate. Demonstraţia rezultă din proprietăţile precedente.

Observaţie: Din cele de mai sus rezultă că triunghiurile lui Napoleon şi triunghiurile lui Fermat au acelaşi centru de greutate. 24) Aria triunghiului Napoleon exterior

a b cN N N este egală cu:

2 2 2[ ] [ ]

1 3( )

2 24a b cN N N ABCA A a b c= ⋅ + ⋅ + + .

Demonstraţie. Avem 2 2 2 2

[ ] [ ]

3 2 3 3

4 3 6 4a b c

a b

N N N ABC

N N a b cA A

+ += = ⋅ + ⋅

, de unde

rezultă concluzia.

25) Aria triunghiului Napoleon interior ' ' 'a b cN N N este egală cu:

' ' '

2 2 2[ ][ ]

1 3( )

2 24a b cABCN N N

A A a b c=− ⋅ + ⋅ + + . Demonstraţie analoagă precedentei.

Observaţie: Din proprietăţile precedente rezultă ' ' '[ ] [ ][ ]a b c a b cN N N ABCN N N

A A A− = .

26) Fie a b cN N N şi ' ' '

a b cN N N triunghiurile lui Napoleon, CBA TTT triunghiul tangenţial

al unui triunghi ABC. Punctele aN , 'a

N , AT sunt coliniare.

Demonstraţia este evidentă deoarece punctele aN , 'a

N , AT aparţin mediatoarei segmentului

BC. Observaţie: Analog se arată că punctele bN , '

bN , BT , respectiv cN , 'cN , CT sunt coliniare.

27) Izogonalele punctelor 1F şi 2F ale lui Fermat sunt punctele izodinamice S şi 'S ale triunghiului ABC. Demonstraţie. Vezi „Puncte izodinamice”. 28) Triunghiul exterior al lui Napoleon a b cN N N este omotetic cu triunghiul podar

a b cS S S al primului punct izodinamic S al triunghiului ABC. Demonstraţie. Vezi „Puncte izodinamice”. 29) Triunghiul interior al lui Napoleon este omotetic cu triunghiul podar al celui de al doilea punct izodinamic al triunghiului ABC. Demonstraţie. Vezi „Puncte izodinamice”.

Page 484: Barbu_TEOREME_FUNDAMENTALE(2008).pdf

491

30) Fie 1 1 1A B C primul triunghi al lui Fermat al unui triunghi ABC şi punctele

1 1 1' ( ), ' ( ), ' ( )A AA B BB C CC∈ ∈ ∈ astfel încât 1 1' 2 ', ' 2 'A A AA B B BB= = şi 1' 2 '.C C CC=

Triunghiurile ' ' 'A B C şi al doilea triunghi al lui Napoleon ' ' 'a b cN N N sunt

congruente. Demonstraţie. Notăm cu litere mici afixele punctelor corespunzătoare. Atunci,

60 60 601 1 1( ), ( ), ( )° ° °= = =

C A BA R B B R C C R A (unde ( )

XR Yϕ înţelegem rotaţia de centru X şi

unghi ϕ a punctului Y), de unde 1 1( ), ( )a c b c b a c aθ θ= + − = + − şi 1 ( )c a a bθ= + − -

unde cos60 sin 60 .= ° + °iθ Deoarece 1' 2 'A A AA= rezultă ( ) 2

' .3

c b c aa

θ+ − += Analog,

( ) 2'

3

a c a bb

θ+ − += şi

( ) 2' .

3

b a b cc

θ+ − += Deoarece 2A este simetricul lui 1A faţă de

BC rezultă 2 1 ( )a b c a b b cθ= + − = − − de unde 2 2 ( )' .

3 3a

b c a b c b cn

θ+ + + − −= =

Analog, 2 ( )

'3b

c a c an

θ+ − −= şi

2 ( )' .

3c

a b a bn

θ+ − −= Deoarece

' ' ' ' , ' ' ' 'a b a ca b n n a c n n− = − − = − şi ' ' ' 'b cb c n n− = − rezultă că

' ' ' ', ' ' ' '≡ ≡a b a c

A B N N A C N N şi ' ' ' 'b cB C N N≡ adică triunghiurile ' ' 'A B C şi

' ' 'a b cN N N sunt congruente.

Observaţie: Deoarece triunghiul ' ' 'a b cN N N este echilateral rezultă că triunghiul ' ' 'A B C

este echilateral. 31) Triunghiul ' ' 'A B C şi al doilea triunghi al lui Napoleon au acelaşi centru de greutate. Demonstraţie. Deoarece ' ' ' ' ' 'a b ca b c n n n a b c+ + = + + = + + rezultă că centrele de greutate

ale triunghiurilor ' ' 'A B C şi ' ' 'a b cN N N coincid.

Observaţie: Centrul de greutate al triunghiului ' ' 'A B C coincide cu centrul de greutate (G) al triunghiului ABC. 32) Sunt adevărate relaţiile: 2 2 2' , ' , 'GA AA GB BB GC CC , ' ' 'GA GB GC≡ ≡ şi

2 2 2

1 1 1' , ' , ' .

3 3 3GA AA GB BB GC CC= = =

Demonstraţie. Din 2

' 1

3

g a

a a

−= − ∈

− rezultă că 2'GA AA

şi 2

1

3

g a

a a

−=

− adică 2

1' .

3GA AA= Deoarece

2 2 2AA BB CC≡ ≡ rezultă ' ' '.GA GB GC≡ ≡

33) Hexagonul ' ' ' ' ' 'c a bA N C N B N este regulat.

Demonstraţia este evidentă deoarece ' ' ' '.c cA N N C≡

A

B C

A2

1A

G A'

Fig. 483

A"

aN

Page 485: Barbu_TEOREME_FUNDAMENTALE(2008).pdf

492

Consecinţă: Punctele ', ', 'A B C aparţin cercului circumscris celui de-al doilea triunghi al lui Napoleon corespunzător triunghiului ABC. 34) Primul punct al lui Fermat ( 1F ) corespunzător unui triunghi ABC aparţine cercului circumscris al celui de-al doilea triunghi al lui Napoleon al triunghiului ABC. Demonstraţie. Din 1 1 1 1( ) ( ' ') 120= = ° m A F B m A F B şi ( ' ' ') 60= °m A C B rezultă

1( ' ') ( ' ' ') 180 ,+ = ° m A F B m A C B adică patrulaterul 1 ' ' 'F A C B este inscriptibil, deci 1F

aparţine cercului circumscris triunghiului ' ' 'A B C şi conform proprietăţii precedente – aparţine cercului circumscris triunghiului ' ' 'a b cN N N .

35) Al doilea punct al lui Fermat ( 2F ) aparţine cercului circumscris primului triunghi al lui Napoleon corespunzător unui triunghi ABC. Demonstraţia este analoagă cu precedenta. 36) Fie 2 2 2" ( ), " ( ), " ( )A AA B BB C CC∈ ∈ ∈ astfel încât 2 2" 2 ", " 2 ".A A AA B B BB= =

Punctele ", ", "A B C aparţin cercului circumscris primului triunghi al lui Napoleon. Demonstraţie. Patrulaterele ' ", ' ", ' "AA GA BB GB CC GC sunt paralelograme (deoarece

2

1' ", ' "

3GA AA GA AA AA= = ), de unde 1"GA AA şi 1

1" .

3GA AA= Analog,

1

1"

3GB BB= şi 1

1" .

3GC CC= Cum 1 1 1AA BB CC≡ ≡ rezultă că " " ".GA GB GC≡ ≡

Deoarece " ag a g n− = − rezultă că punctele ", ", "A B C aparţin cercului circumscris

triunghiului .a b cN N N

37) Triunghiul " " "A B C este echilateral şi congruent cu primul triunghi al lui Napoleon

a b cN N N corespunzător triunghiului ABC.

Demonstraţie. Se arată fără dificultate că " " a ba b n n− = − şi " " a ca c n n− = − adică " " a bA B N N≡ şi " " .a cA C N N≡

Page 486: Barbu_TEOREME_FUNDAMENTALE(2008).pdf

493

III.48. Triunghiurile lui Vecten

„Desenul corupe raţionamentul” – Ion Barbu

Pe laturile unui triunghi ABC se construiesc în exterior pătratele C BBCA A , C AACB B şi

B AABC C care au centrele AO , BO , respectiv CO . Triunghiul A B CO O O determinat de

centrele acestor pătrate se numeşte triunghiul Vecten exterior.

1) Dreptele CBA COBOAO ,, sunt concurente .

Demonstraţie. Triunghiurile dreptunghice isoscele BACO şi CABO sunt asemenea, atunci

C

B

AOAB

AC AO= , de unde B CAB AO AC AO⋅ = ⋅ şi de aici

sin sinB B C CAB AO BAO AC AO CAD⋅ ⋅ = ⋅ ⋅ (deoarece B CBAO CAO≡ ), deci

[ ] [ ]B CABO ACOA A= . Analog se arata că ][][ CC BAOBCO AA = şi ][][ BA CBOCAO AA = (1). Fie

BCAOA A I= 1 , ACBOB B I= 1 şi . 1 ABCOC C I= Atunci,

CA

BA

h

h

hAO

hAO

A

A

A

A

ACO

ABO

A

A

1

1

2

1

2

1

][

][==

⋅= (unde 1h şi 2h sunt lungimile înălţimilor duse din B şi C

pe latura AAO în triunghiurile AABO , respectiv AACO ). Analog, AB

CB

A

A

BAO

BCO

B

B

1

1

][

][= şi

A

B C

AC

AB

BC

CB

BA B' C' CA

Fig. 484

AO

BO

CO

aH

A'

Page 487: Barbu_TEOREME_FUNDAMENTALE(2008).pdf

494

.1

1

][

][

BC

AC

A

A

CBO

CAO

C

c = (2). Din relaţiile (1) şi (2) rezultă: 11

1

1

1

1

1 =⋅⋅BC

AC

AB

CB

CA

BA şi conform

reciprocei teoremei lui Ceva rezultă că dreptele BA BOAO , şi CCO sunt concurente.

Observaţie : Punctul de concurenţă al dreptelor AAO , BBO şi CCO se numeşte punctul

lui Vecten exterior 1( ).V

2) Sunt adevărate relaţiile: A B CAO O O⊥ , B C ABO O O⊥ , C A BCO O O⊥ .

Demonstraţie. Notăm cu litere mici afixele punctelor corespunzătoare. Prin rotaţia de

centru CO şi unghi 2

π, punctul B ajunge în A, deci 2 ( )

COA B

π

= ℜ , de unde

( )C Ca o b oω= + − , unde cos sin2 2

i iπ π

ω = + = , adică 1C

a ibo

i

−=

−. Analog , 2 ( )

AOB C

π

= ℜ

şi 2 ( )BO

C A

π

= ℜ , de unde 1A

b ico

i

−=

− şi

1B

c iao

i

−=

−. Atunci , *A

C B

a oi i

o o

−= − ∈ ⋅

− , adică

A B COO O O⊥ . Analog se arată că B C ABO O O⊥ şi C A BCO O O⊥ .

Observaţie: Ortocentrul triunghiului A B CO O O este punctul de intersecţie al

dreptelor AAO , BBO şi CCO .

3) Sunt adevărate relaţiile: A B CAO O O≡ , B C ABO O O≡ , C A BCO O O≡ .

Demonstraţie. Din A

C B

a oi

o o

−= −

− rezultă 1A

C B

a oi

o o

−= − =

−, adică A C Ba o o o− = − , de

unde A B CAO O O≡ . Analog se arată că B C ABO O O≡ şi C A BCO O O≡ .

Observaţie: Cu segmentele AAO , BBO şi CCO se poate construi un triunghi congruent cu

triunghiul A B CO O O .

4) Triunghiurile A AAB C , B BBA C , C CCA B şi ABC au aceeaşi arie.

Demonstraţie. Avem

[ ]sin

2A A

A A A AAB C

AC AB C ABA

⋅ ⋅= . Deci AAC AB c= = ,

AAB AC b= = şi sin( ) sin(360 180 ) sin(180 ) sinA AC AB A A A= °− ° − = ° − = , deci

[ ] [ ]sin

2A AAB C ABC

bc AA A= = . Analog se arată că [ ] [ ] [ ]A A C CAB C CA B ABCA A A= = .

Page 488: Barbu_TEOREME_FUNDAMENTALE(2008).pdf

495

5) Laturile triunghiului Vecten exterior au lungimile 2 2

[ ]'4

2ABC

b c Aa

+ += ,

2 2[ ]'

4

2ABC

a c Ab

+ += şi

2 2[ ]'

4

2ABC

b a Ac

+ += , unde 'a , 'b , 'c sunt lungimile laturilor

B CO O , A CO O , respectiv A BO O .

Demonstraţie. Din teorema cosinusului în triunghiul B CAO O obţinem

2 2 2 2 cosB C B C B C B CO O O A O A O A O A O AO= + − ⋅ ⋅ . Dar

2

2B

bO A = ,

2

2C

cO A = ,

( ) 90 ( )B Cm O AC m BAC= °+ ,deci 2 2

'2 sin2

2 2 2

b c bc Aa

⋅= + + ⋅

, adică

2 2[ ]'

4

2ABCb c A

a+ +

= . Analog, 2 2

[ ]'4

2ABCa c A

b+ +

= şi 2 2

[ ]'4

2ABCb a A

c+ +

= .

6) Este adevărată relaţia: 2 2 2 2 2 23( )

A A B B C CB C C A A B a b c+ + = + +

Demonstraţie. Teorema cosinusului în triunghiul A AAB C ne dă:

2 2 2 2 cosA C A AB C b c bc B AC= + − ⋅ , cos cos(180 )A AB AC A= ° − = 2 2 2

cos2

a b cA

bc

− −− = ,

de unde 2 2 2 2 2 2( )A AB C b c a b c= + − − − = 2 2 22 2b c a+ − . Analog, 2 2 2 22 2B BC A a c b= + − şi 2 2 2 22 2C CA B b a c= + − , de unde 2 2 2 2 2 23( )

A A B B C CB C C A A B a b c+ + = + + .

7) Dreptele CAA şi CBB sunt perpendiculare. Demonstraţie. Notăm cu litere mici afixele punctelor corespunzătoare . Avem :

2 ( )C CA B

π

= ℜ şi 2 ( )C CB A

π−

= ℜ , deci ( )Ca c i b c= + − şi ( )Cb c i a c= − − . Atunci:

[ ] *( )

( )C

C

i b c i a ca ai i

b b b c i a c

− − + −−= = − ∈ ⋅

− − + − , deci C CAA BB⊥ .

Observaţie: Analog se arată că B BAA CC⊥ şi A ABB CC⊥ .

8) Fie AG , BG , CG , centrele de greutate ale

triunghiurilor A AAB C , B BBC A respectiv

C CCA B . Dreptele AAG , BBG şi CCG sunt concurente în ortocentrul H al triunghiului ABC .

Demonstraţie. Fie 1B al patrulea vârf al

paralelogramului 1B BBC B A şi

1 BH BB AC= ∩ . Deoarece

( ) 180 ( )B Bm A BC m B= °− rezultă

1( ) ( )Bm BA B m B= . Atunci triunghiul

A

B C

AC

BC

1B

bH

CA BA

2B

BG

Fig. 485

Page 489: Barbu_TEOREME_FUNDAMENTALE(2008).pdf

496

ABC şi 1 BB A B sunt congruente ( 1( ) ( )Bm BA B m B= , 1B BAB BC B A≡ ≡ , BBC BA≡ ),

deci 1( ) ( )= = Bm BB A m BAC 1( )Bm B B C . Deoarece

1( ) ( ) 180 ( ) 90B bm BBC m ABH m CBA+ = °− = °

deci ( ) ( ) 90b bm BAH m ABH+ = ° , de unde rezultă că bBH AC⊥ (1). Deoarece într-un

paralelogram diagonalele se înjumatăţesc, rezultă că 2BB este mediană în triunghiul

B BBA C , deci 1BG B B∈ (2). Din (1) şi (2) rezultă că BG B AC⊥ . Analog, AAG BC⊥ şi

CCG AB⊥ , de unde rezultă că BBG , CCG sunt dreptele suport ale înalţimilor triunghiului

ABC , deci sunt concurente în ortocentrul triunghiului ABC . 9) Fie AH , BH , CH ortocentrele triunghiurilor A AAB C , B BBA C respectiv C CCA B .

Dreptele AAH , BBH şi CCH sunt concurente în centrul de greutate al triunghiului ABC.

Demonstraţie. Fie 1A cel de-al

patrulea vârf al paralelogramului

1A AC AB A , A AAP B C⊥ , ABQ AC⊥ ,

( , )A AP B CQ AC∈ ∈ şi = ∩aM AP BC

Cum ≡A A AB C Q AH B , AC A AB⊥

şi A AC A H Q⊥ rezultă AAB H Q ,

deci a aBAM AH Q≡ (alterne

interne). Deoarece 1 ≡ AAAC BCA

(vezi proprietatea anterioară) rezultă

2AC AA ABC≡ , deci triunghiurile

aABM şi 2AC AA sunt congruente, de

unde 2 11 1

2 2aBM AA AA BC= = = ,

deci aM e mijlocul segmentului BC ,

adică aAM este mediană în triunghiul ABC , adică G -centrul de greutate al triunghiului

ABC - aparţine dreptei aAH . Analog, BG BH∈ şi CG CH∈ .

Observaţie: Din congruenţa triunghiurilor aABM şi

2AC AA rezultă 21

2a A A AAM C A B C= = , deci

22 2A A aB C AM m= = . Analog 2B B bA C m= şi

2C C cA B m= (unde , ,a b cm m m sunt lungimile

medianelor triunghiului ABC ). Atunci 2 2 2 2 2 2 2 2 24( ) 3( )+ + = + + = + +A A B B C C a b cB C A C A B m m m a b c

Dacă pe laturile triunghiului ABC se construiesc spre

interior pătratele ' 'C BBCA A , ' '

C AACB B , ' 'A BBAC C având

centrele ' ',A BO O respectiv 'CO . Triunghiul ' ' '

A B CO O O se

numeşte triunghiul Vecten interior (Fig. 487).

A

B C

Q

P AC

aM

aH

AB

BC

CB

1A

2A

Fig. 486

A

B C

'AB

'CB

'BC

'AC

'CA

'BA

'AO

'BO

'CO

Fig. 487

Page 490: Barbu_TEOREME_FUNDAMENTALE(2008).pdf

497

10) Sunt adevărate relaţiile: ' ' 'A B CAO O O⊥ , ' ' '

B A CBO O O⊥ , ' ' 'C B ACO O O⊥ .

Demonstraţie. Notăm cu litere mici afixele punctelor corespunzătoare. Avem:

'2 ( )BO

A C

π

= ℜ , '2 ( )CO

B A

π

= ℜ , '2 ( )AO

C B

π

= ℜ , deci '

1B

a ico

i

−=

−, '

1C

b iao

i

−=

−, '

1A

c ibo

i

−=

−, de

unde '

*' '

( )A

B C

a o i a ai ic bi i

a ai ic bo o

− − + − −= = − ∈ ⋅

+ − −− , de unde rezultă ' ' '

A B CAO O O⊥ . Analog,

' ' 'B A CBO O O⊥ şi ' ' '

C B ACO O O⊥ .

11) Sunt adevărate relaţiile: ' ' 'A B CAO O O≡ , ' ' '

B A CBO O O≡ şi ' ' 'C B ACO O O≡ .

Demonstraţie. Din '

' 'A

B C

a oi

o o

−= −

− rezultă

'

' '1A

B C

a oi

o o

−= − =

−, de unde ' ' '

A B Ca o o o− = − ,

adică ' ' 'A B CAO O O≡ . Analog se arată că ' ' '

B A CBO O O≡ şi ' ' 'C B ACO O O≡ .

Observaţie: Cu lungimile segmentelor 'AAO , '

BBO , 'CCO se poate construi un triunghi

congruent cu triunghiul Vecten interior.

12) Dreptele 'AAO , '

BBO , 'CCO sunt concurente.

Demonstraţie. Dreptele 'AAO ,

'BBO ,

'CCO fiind perpendiculare pe dreptele ' '

B CO O , ' 'A CO O ,

' 'B AO O , sunt dreptele suport ale triunghiului ' ' '

A B CO O O , deci sunt concurente în ortocentrul

triunghiului Vecten interior.

Observaţie: Punctul de concurenţă al dreptelor ' 'B CO O , ' '

A CO O , ' 'B AO O se numeşte punctul

Vecten interior.

13) Triunghiurile 'BABA şi '

BBCC ; 'CBCB şi '

CACA , respectiv 'AABB şi '

AACC au aceeaşi arie. Demonstraţie. Notăm cu , ,a b c lungimile laturilor BC , CA respectiv AB . Din

' 'B BABA CBC≡ (unghiuri cu laturile perpendiculare două câte două) rezultă

' '

' ' ' '

[ ] [ ]

sin sin

2 2B B

B B B B

ABA CBC

AB BA ABA BA BC CBCA A

⋅ ⋅ ⋅ ⋅= = =

. Analog se arată că

' '[ ] [ ]C CBCB ACAA A= şi ' '[ ] [ ]A AABB ACC

A A= .

14) Dreptele CBB , BCC şi înalţimea din A a triunghiului ABC sunt concurente.

Demonstraţie. Fie aAH înălţimea din A , notăm 1( )am BAH α= , 2( )am H AC α= ,

1( )Cm CBB β= , 2( )Cm B BA β= , 1( )Bm ACC γ= , 2( )Bm C CB γ= . Din teorema

Page 491: Barbu_TEOREME_FUNDAMENTALE(2008).pdf

498

sinusurilor avem: 1sin sin(90 )C CCB BB

Cβ=

°+ şi

2sin sin(45 )C CAB BB

Aβ=

°+, de unde

1cos

sinC

b C

BBβ = şi 2

2 sin( 45 )sin

C

b A

BBβ

+ °= deci 1

2

sin cos

sin 2 sin( 45 )

C

A

ββ

=+ °

(1). Analog

se arată că 1

2

sin 2 sin( 45 )

sin cos

A

B

γγ

+ °= . Dar 1sin sin(90 ) cosB Bα = °− = şi

2sin sin(90 ) cosC Cα = °− = , de unde 1 1 1

2 2 2

sin sin sin1

sin sin sin

α β γα β γ

⋅ ⋅ = , şi din reciproca teoremei

lui Ceva sub formă trigonometrică rezultă concurenţa dreptelor CBB , BCC şi aAH .

Observaţie: Dacă triunghiul nu este ascuţitunghic, rezultatul se pastrează, dar calculele suferă unele modificari.

15) Fie ' A B A CA C C B B= ∩ , ' A B B CB C C A A= ∩ şi ' B C A CC A A B B= ∩ . Triunghiul ABC şi triunghiul ortic al triunghiului ABC sunt omologice. Demonstraţie. vezi „Triunghiul lui Grebe”. 16) Triunghiul Vecten exterior A B CO O O are aria egală cu

2 2 2

[ ] [ ] 8A B CO O O ABC

a b cA A

+ += + , unde a, b, c sunt lungimile laturilor BC, CA, AB.

Demonstraţie.

Alegem un sistem cartezian cu originea în centrul cercului circumscris triunghiului ABC. Notăm cu litere mici afixele punctelor corespunzătoare, iar prin ( )

XZ Yϕ= ℜ înţelegem că

punctul Z se obţine printr-o rotaţie de centru X şi unghi ϕ a punctului Y . Avem:

2 2( ), ( ),A BO O

B C C Aπ π

= ℜ = ℜ 2 ( )CO

A Bπ

= ℜ , de unde rezultă ,1A

b ico

i

−=

− ,

1B

c iao

i

−=

.1C

a ibo

i

−=

− Atunci: [ ]

1 1

1 14 8

1 1

− +

= = − + =

− +A B C

A A

O O O B B

C C

o o b ic b i ci i

A o o c ia c i a

o o a ib a ib

2[( ) ( )

8+ + − − − − + + +

ibc ca ab ac ba cb i aa bb cc ( )]i bc ca ab ac ba cb+ + + + + + (1) ,

[ ]

1

1 [ ]4 4

1

ABC

a ai i

A b b bc ca ab ac ba cb

c b

= = + + − − − (2) şi 2 2 2+ + =BC CA AB

2 2 2( )( ) ( )( ) ( )( )− + − + − = − − + − − + − − =c b a c b a c b c b a c a c b a b a

22[3 ( )],− + + + + +R bc ca ab ac ba cb deci 2 2 2

232

BC CA ABbc ca ab ac ba cb R

+ ++ + + + + = −

(3) ( unde a b c R= = = - raza cercului circumscris triunghiului ABC). Din relaţiile (1),

Page 492: Barbu_TEOREME_FUNDAMENTALE(2008).pdf

499

(2) şi (3) rezultă : 2[ ] [ ]

1[3 ( )]

4= + − + + + + + =

A B CO O O ABCA A R bc ca ab ac ba cb

2 2 2

[ ]

1

4 2

+ ++ ⋅ =

ABC

BC CA ABA

2 2 2

[ ] .8ABC

BC CA ABA

+ ++

17) Triunghiul Vecten interior ' ' 'A B CO O O are aria egală cu ' ' '

2 2 2

[ ][ ] 8A B CABCO O O

a b cA A

+ += − ,

unde a, b, c sunt lungimile laturilor BC, CA, AB. Demonstraţia este analoagă celei precedente.

III.49. Triunghiurile Sharygin

„…poezia nu este lacrima ea este insuşi plânsul

plânsul unui ochi neinventat lacrima ochiului celui care trebuie să fie frumos, lacrima celui care trebuie să fie fericit.”

Nichita Stănescu228 În triunghiul ABC fie ', ', 'A B C picioarele bisectoarelor interioare şi ", ", "A B C picioarele bisectoarelor exterioare. Punctele de intersecţie ale mediatoarelor segmentelor

', ', 'AA BB CC sunt vârfurile primului triunghi Sharygin. Mediatoarele segmentelor ", ", "AA BB CC se intersectează în trei puncte care sunt vârfurile celui de-al doilea triunghi

Sharygin.

1) Triunghiul de contact al triunghiului ABC şi primul triunghi Sharygin al triunghiului ABC sunt omotetice.

228Nichita Stănescu (1933 – 1983) – eseist, poet român, ales postum membru al Academiei Române

A

B C

aI

bI

cI

bC

aC

cC bS

aS

cS 1M

2M 3M

Fig. 488

Page 493: Barbu_TEOREME_FUNDAMENTALE(2008).pdf

500

Demonstraţie. Fie a b cS S S primul triunghi Sharygin al triunghiului ABC, a b cC C C triunghiul

de contact al triunghiului ABC. Deoarece triunghiul b cAC C este isoscel rezultă că

c bC C AI⊥ ; cum şi b cS S AI⊥ rezultă .c b c bC C S S Analog a b a bC C S S şi

(1).a c a cC C S S Fie 1 2 3, ,M M M mijloacele bisectoarelor ', ', '.AA BB CC Fie

1 = ∩a cA S S BC şi 2 .= ∩a bA S S BC Avem

1 2 1 2 2 1( ) 180 [ ( ) ( )]= ° − + = a a am A S A m S A A m S A A 1 2 3 2180 [ ( ) ( )]° − + = m BAM m M A C

1 1 1180 90 ( ) 90 ( ) [ ( ) ( )]

2 2 2m B m C m B m C

°− °− + °− = + şi

( ) ( ) ( )b a c b a c am CCC m CC I m CC I= + =

1( ) ( ) [ ( ) ( )],

2b cm C CI m C BI m C m B+ = + deci .b a c b a cS S S C C C≡ Analog ,

,a b c a b cS S S CCC≡ adică triunghiurile a b cS S S şi a b cC C C sunt asemenea (2). Din relaţiile

(1) şi (2) rezultă că triunghiurile a b cS S S şi a b cC C C sunt omotetice.

2) Consecinţă: Unghiurile primului triunghi Sharygin a b cS S S corespunzător

triunghiului ABC au măsurile egale cu 1 1

90 ( ), 90 ( )2 2

° − ° − m A m B , respectiv

190 ( ).

2° − m C

Demonstraţie. În proprietatea precedentă am arătat că

[ ]1 1 1( ) [ ( ) ( )] 180 ( ) 90 ( ).

2 2 2= + = °− = ° − c a bm S S S m B m C m A m A Analog se arată

că 1

( ) 90 ( )2

= °− a b cm S S S m B şi 1

( ) 90 ( ).2

= ° − a c bm S S S m C

3) Triunghiurile Sharygin sunt asemenea. Demonstraţie. Fie ' ' '

a b cS S S cel de-al doilea triunghi Sharygin, ", ", "A B C picioarele

bisectoarelor exterioare şi ' ' '1 2 3, ,M M M mijloacele segmentelor ",AA "BB respectiv ",CC

' ' ' ' , .b a b c

P S S BC R S S BC= ∩ = ∩ Avem: ' '1( " ) ( )bm A R S m M R B= =

1360 90 (180 ( )) 90 ( ) (3)

2m B m A

° − ° + ° − + ° −

. Din triunghiul 3PM C avem:

'3

1 1( ) 90 90 ( ) ( ) (4).

2 2m M PC m B m C

= ° − ° − =

Din relaţiile (3) şi (4) rezultă că

1( ' ) 1 8 0 [ ( ' ) ( ' ) ] 9 0 ( ) .

2= ° − + = ° − b b bm P S R m S P R m S R P m B

Atunci , ' ' ' 1( ) 90 ( )

2a b cm S S S m B= °− şi analog se arată că ' ' ' 1( ) 90 ( )

2b a cm S S S m A= °− ,

' ' ' 1( ) 90 ( ).

2a c bm S S S m C= ° − Din relaţiile de mai sus şi consecinţa precedentă rezultă că

triunghiurile lui Sharygin a b cS S S şi ' ' 'a b cS S S sunt asemenea.

Page 494: Barbu_TEOREME_FUNDAMENTALE(2008).pdf

501

Observaţie: Unghiurile celui de-al doilea triunghi Sharygin corespunzător triunghiului ABC

au măsurile egale cu 1 1 1

90 ( ),90 ( ),90 ( ).2 2 2m A m B m C° − ° − ° −

4) Primul triunghi Sharygin a b cS S S al triunghiului ABC şi triunghiul antisuplementar

a b cI I I al triunghiului ABC sunt omotetice.

Demonstraţie. Deoarece ( ) 90cm I AI = ° (Fig. 489) rezultă c bI I AI⊥ şi cum c bS S AI⊥

rezultă .c b c bI I S S Analog, a b a bI I S S şi (5).a c a cI I S S Cum

1( ) 90 ( ),

2am BI C m A= °− 1

( ) 90 ( )2bm CI A m ABC= °− şi

1( ) 90 ( )

2cm AI B m ACB= °− rezultă că triunghiurile a b cI I I şi a b cS S S sunt asemenea şi

utilizând relaţiile (5) rezultă că triunghiurile a b cI I I şi a b cS S S sunt omotetice.

5) Al doilea triunghi Sharygin ' ' '

a b cS S S al triunghiului ABC este asemenea şi omologic cu

triunghiul antisuplementar a b cI I I al triunghiului ABC.

A

B

C A"

B"

C"

'1M

'2M

'3M

P

R

'aS

'bS

'cS

Fig. 489

Page 495: Barbu_TEOREME_FUNDAMENTALE(2008).pdf

502

Demonstraţie. Deoarece ' ' ' 1( ) ( ) 90 ( )

2a a b cm BI C m S S S m A= = °− şi

' ' '( ) ( )b a b c

m CI A m S S S= = 1

90 ( )2m B° − rezultă că triunghiurile ' ' '

a b cS S S şi a b cI I I sunt

asemenea. Deoarece punctele ", ", "A B C sunt coliniare (ele aparţinând axei ortice a

triunghiului ABC), atunci ' ' '1 2 3, ,M M M sunt mijloacele patrulaterului complet

" " "B A BAC C şi conform teoremei lui Gauss sunt coliniare (aparţin dreptei lui Gauss).

Deoarece ' ' ' ' ' '1 2 ,

b c b c a c a cM I I S S M I I S S= ∩ = ∩ şi ' ' '

3 a b b a

M I I S S= ∩ din reciproca

teoremei lui Desarguss rezultă că dreptele ' ',a a b bI S I S şi '

c cI S sunt concurente, triunghiurile

' ' 'a b cS S S şi a b cI I I fiind astfel omologice.

6) Triunghiurile Sharygin ale triunghiului ABC sunt omologice cu triunghiul ABC, axa de omologie fiind dreapta lui Lemoine a triunghiului ABC. Demonstraţie. Fie ,b cX S S BA= ∩ ,b cY S S AC= ∩ 1 ,b cL S S BC= ∩ 'A piciorul

bisectoarei din A. Deoarece c bS S este mediatoarea segmentului 'AA şi 'AA este

bisectoarea unghiului BAC rezultă că patrulaterul 'AXA Y este romb. Din teorema lui

Menelaus pentru triunghiul ABC şi transversala 1, ,L X Y avem: 1

1

1,L B XA YC

LC XB YA⋅ ⋅ = unde

1

1

L B XB

LC YC= (deoarece XA YA≡ ) (6).

Deoarece 'XA AC rezultă ',

XB BA

AB BC=

adică 2ac c c

XBb c a b c

= ⋅ =+ +

(7) iar din

'YA AB rezultă ',

YC CA

YA BC=

2

(8).ab b b

YCb c a b c

= ⋅ =+ +

Din relaţiile (6),

(7) şi (8) rezultă 2

1

1

.L B c

LC b

=

Analog, dacă

2 a cL S S AC= ∩ , 3 a bL S S AB= ∩ rezultă 2

2

2

L C a

L A c

=

şi 2

3

3

,L A b

L B a

=

de unde

31 2

1 2 3

1L AL B L C

L C L A L B⋅ ⋅ = , iar din reciproca teoremei lui Menelaus rezultă că punctele 1 2,L L şi 3L

sunt coliniare deci triunghiurile a b cS S S şi ABC sunt omologice. Tangenta în A la cercul

circumscris triunghiului ABC intersectează dreapta BC într-un punct 1A care are

proprietatea 2

1

1

A B c

AC b

=

. Atunci, 1 1

1 1

A B L B

AC LC= de unde : 1 1

1 1 1 1

A B L B

AC A B LC L B=

− − adică

1 1A B L B

BC BC= egalitate echivalentă cu 1 1A B L B= şi deoarece 1A şi 1L sunt de aceeaşi parte a

A

B C A'

X Y

1L

cS bS

Fig. 490

Page 496: Barbu_TEOREME_FUNDAMENTALE(2008).pdf

503

lui B pe dreapta BC rezultă că punctele 1A şi 1L coincid, deci 1L aparţine dreptei lui

Lemoine a triunghiului ABC. Analog se arată că 2L şi 3L aparţin dreptei lui Lemoine, deci

axa de omologie dintre primul triunghi Sharygin şi triunghiul ABC este dreapta lui Lemoine a triunghiului ABC. Analog se arată că triunghiul ABC este omologic şi cu cel de-al doilea triunghi Sharygin.

Page 497: Barbu_TEOREME_FUNDAMENTALE(2008).pdf

5

CUPRINS

CAPITOLUL I – PUNCTE, DREPTE, CERCURI REMARCABILE

ASOCIATE UNUI TRIUNGHI I.1. Centrul de greutate al unui triunghi...............................................................................9

I.2. Centrul cercului circumscris unui triunghi..................................................................12

I.3. Cercul înscris într-un triunghi.....................................................................................16

I.4. Ortocentrul unui triunghi.............................................................................................22 I.5. Punctul lui Gergonne...................................................................................................28

I.6. Punctul lui Nagel. Dreapta lui Nagel...........................................................................36

I.7. Punctul lui Longchamps..............................................................................................48

I.8. Punctul lui Spieker......................................................................................................50

I.9. Punctul lui Brocard..................................................................................................... 53

I.10. Punctul lui Nobss. Dreapta lui Gergonne................................................................. 65 I.11. Punctele lui Soddy. Punctele lui Eppstein.................................................................67

I.12. Punctul lui Steiner.....................................................................................................76

I.13. Punctul lui Tarry........................................................................................................78

I.14. Puncte izodinamice....................................................................................................79

I.15. Punctul izogon........................................................................................................... 83 I.16. Puncte izotomice........................................................................................................86

I.17. Puncte izologice.........................................................................................................90

I.18. Retrocentrul unui triunghi..........................................................................................93

I.19. Punctul anti-Steiner....................................................................................................95

I.20. Punctul lui Bevan.......................................................................................................97

I.21. Punctul lui Exeter.................................................................................................... 101 I.22. Punctul lui Gob........................................................................................................ 102

I.23. Punctul lui Gray. Triunghiul lui Gray..................................................................... 103

I.24. Punctul lui Hexyl.................................................................................................... 107

I.25. Punctul lui Prasolov.................................................................................................108

I.26. Punctul lui Kariya....................................................................................................109

I.27. Punctul lui Schiffler.................................................................................................112 I.28. Punctul lui Weill......................................................................................................113

I.29. Punctele lui Pelletier................................................................................................114

I.30. Punctul lui Kenmotu................................................................................................116

I.31. Dreapta lui Euler. Cercul lui Euler..........................................................................118

I.32. Dreapta lui Gauss.................................................................................................... 134

I.33. Dreapta lui Brocard..................................................................................................136 I.34. Dreapta ortică...........................................................................................................137

I.35. Dreapta antiortică.....................................................................................................140

I.36. Dreapta lui Simson...................................................................................................141

I.37. Dreapta lui Lemoine.................................................................................................155

I.38. Transversala izotomică.............................................................................................157 I.39. Dreapta lui Steiner....................................................................................................158

I.40. Drepte izogonale. Puncte izogonale.........................................................................159

I.41. Izogonale exterioare.................................................................................................171

I.42. Dreptele lui Schwatt.................................................................................................172

I.43. Ortopolul unei drepte...............................................................................................173

I.44. Dreapta lui Aubert....................................................................................................180 I.45. Antibisectoarea.........................................................................................................181

Page 498: Barbu_TEOREME_FUNDAMENTALE(2008).pdf

6

I.46. Simediane. Punctul lui Lemoine...............................................................................184

I.47. Dreapta lui Housel....................................................................................................198

I.48. Simediana exterioară................................................................................................198

I.49. Cercuri exânscrise....................................................................................................200 I.50. Cercurile lui Lemoine...............................................................................................209

I.51. Cercul lui Taylor.......................................................................................................212

I.52. Cercul lui Tücker......................................................................................................216

I.53. Cercurile lui Lucas...................................................................................................218

I.54. Cercurile lui Apollonius...........................................................................................220

I.55. Cercurile adjuncte.....................................................................................................224 I.56. Cercul ortocentroidal................................................................................................227

I.57. Cercurile lui Neuberg................................................................................................230

I.58. Cercul lui Van Lamoen.............................................................................................233

I.59. Cercul lui Conway....................................................................................................235

I.60. Cercul lui Adams......................................................................................................238

CAPITOLUL II – TEOREME FUNDAMENTALE DIN GEOMETRIA

TRIUNGHIULUI II.1. Teorema bisectoarei interioare…………………………………………………….239

II.2. Teorema bisectoarei exterioare……………………………………………………240

II.3. Teorema lui Pitagora………………………………………………………………242

II.4. Teorema lui Pitagora generalizată…………………………………………………243

II.5. Teorema lui Stewart……………………………………………………………….245 II.6. Teorema sinusurilor……………………………………………………………….246

II.7. Teorema lui Ceva………………………………………………………………….247

II.8. Teorema lui Menelaus……………………………………………………………..248

II.9. Teorema transversalei……………………………………………………………..250

II.10. Teorema lui Leibniz……………………………………………………………...252

II.11. Teorema lui Toricelli-Fermat…………………………………………………….253 II.12. Teorema lui Feuerbach…………………………………………………………...256

II.13. Teorema lui Desargues…………………………………………………………...269

II.14. Teorema lui Döttl…………………………………………………………………271

II.15. Teorema lui Van Aubel…………………………………………………………...271

II.16. Teorema lui Descartes…………………………………………………………….276

II.17. Teorema lui Pompeiu……………………………………………………………..279 II.18. Teorema lui Erdös-Mordell……………………………………………………….280

II.19. Teoremele lui Fagnano……………………………………………………………286

II.20. Dreapta lui Droz-Farny…………………………………………………………...289

II.21. Teorema lui Steiner-Lehmus……………………………………………………...291

II.22. Teorema lui Barbilian…………………………………………………………….293

II.23. Teorema lui Bottema……………………………………………………………..294 II.24. Teorema lui Goormaghtigh……………………………………………………….295

II.25. Teorema lui Dergiades……………………………………………………………297

II.26. Teoremele lui Pappus……………………………………………………………..298

II.27. Teorema lui Salmon………………………………………………………………300

II.28. Teorema lui Pedoe………………………………………………………………...300 II.29. Teorema lui Simson generalizată…………………………………………………301

II.30. Teorema lui Sondat………………………………………………………………..301

II.31. Teorema lui Maxwell……………………………………………………………...304

II.32. Teorema trisecţiei…………………………………………………………………305

Page 499: Barbu_TEOREME_FUNDAMENTALE(2008).pdf

7

II.33. Teoremele lui Harcourt…………………………………………………………..306

II.34. Teorema lui Zaslavsky…………………………………………………………...307

II.35. Teorema lui Zajic………………………………………………………………...308

II.36. Teorema lui Viviani………………………………………………………………308 II.37. Teorema lui Veronese…………………………………………………………….309

II.38. Teorema lui Coşniţă………………………………………………………………310

II.39. Teorema lui Kiepert………………………………………………………………313

II.40. Teorema lui Gergonne……………………………………………………………314

II.41. Teorema lui Heron………………………………………………………………..314

II.42. Teorema lui Catalan………………………………………………………………315 II.43. Teorema lui Blanchet……………………………………………………………..315

II.44. Teorema lui Alasia………………………………………………………………..316

II.45. Teorema lui Ayme………………………………………………………………...317

II.46. Teorema lui Bobillier……………………………………………………………..318

II.47. Teorema lui Boutin………………………………………………………………..319

II.48. Teorema lui Cantor………………………………………………………………..320 II.49. Teorema lui Carnot………………………………………………………………..321

II.50. Teoremele lui Carnot……………………………………………………………...323

II.51. Teorema lui Casey………………………………………………………………...325

II.52. Teorema lui Clairaut………………………………………………………………328

II.53. Teorema lui Mathieu………………………………………………………………330 II.54. Teorema lui Miquel……………………………………………………………….331

II.55. Teorema lui Sawayama-Thebault…………………………………………………333

II.56. Teorema lui Schooten……………………………………………………………..336

II.57. Teorema lui Smarandache………………………………………………………...337

II.58. Teorema lui Snapper………………………………………………………………338

II.59. Teoerma lui Urquhart-Pedoe………………………………………………………339 II.60. Relaţii metrice în triunghiul dreptunghic………………………………………….340

II.61. Aria unui triunghi………………………………………………………………….341

CAPITOLUL III – TRIUNGHIURI REMARCABILE III.1. Triunghiul ortic……………………………………………………………………345

III.2. Triunghiul median…………………………………………….…………………...354

III.3. Triunghiul de contact…………………………………………….………………..358 III.4. Triunghiul extangenţial…………………………………………….……………..364

III.5. Triunghiul cotangentic…………………………………………….………………367

III.6. Triunghiul antipodar…………………………………………….…………………370

III.7. Triunghiul podar…………………………………………….…………………….373

III.8. Triunghiul tangenţial…………………………………………….………………..379

III.9. Triunghiul anticomplementar……………………………………….…………….385 III.10. Triunghiul antisuplementar………………………………………….…………...388

III.11. Triunghiul ciclocevian…………………………………………….……………..392

III.12. Triunghiul I-pedal…………………………………………….………………….393

III.13. Triunghiuri altimediale…………………………………………….…………….396

III.14. Triunghiurile lui Brocard. Cercul lui Brocard……………………….…………..398 III.15. Triunghiul antiparalel determinat de o direcţie în raport cu un triunghi…………404

III.16. Triunghiul automedian…………………………………………….……………..406

III.17. Triunghiul circumpedal…………………………………………….…………….410

III.18. Triunghiul simedian…………………………………………….………………..415

III.19. Triunghiul 060∆ …………………………………………….…………………...416

Page 500: Barbu_TEOREME_FUNDAMENTALE(2008).pdf

8

III.20. Triunghiul medianelor…………………………………………….…………….420

III.21. Triunghiuri omologice…………………………………………….…………….421

III.22. Triunghiuri ortopolare…………………………………………….…………….423

III.23. Triunghiuri ortologice…………………………………………….…………….428 III.24. Triunghiuri paralogice…………………………………………….……………431

III.25. Triunghiuri bilogice…………………………………………….………………433

III.26. Triunghiuri biortologice. Triunghiuri triortologice…………….……………….434

III.27. Triunghiuri coparalele…………………………………………….…………….435

III.28. Triunghiuri înscrise…………………………………………….……………….437

III.29. Triunghiuri înscrise izotomice…………………………………….……………438 III.30. Triunghiuri pseudoisoscele……………………………………….…………….440

III.31. Triunghiuri cosimediene……………………………………….……………….444

III.32. Triunghiul celor trei imagini……………………………………….……………447

III.33. Triunghiuri izoliniare……………………………………….…………………..449

III.34. Triunghiuri metaparalele……………………………………….……………….450

III.35. Triunghiul pedal……………………………………….………………………..451 III.36. Triunghiul anticevian……………………………………….…………………..453

III.37. Triunghiuri ortogonale……………………………………….…………………455

III.38. Triunghiul lui Carnot……………………………………….…………………..456

III.39. Triunghiul lui Lucas……………………………………….……………………461

III.40. Triunghiul lui Fuhrmann……………………………………….……………….463 III.41. Triunghiul lui Lionnet……………………………………….………………….468

III.42. Triunghiurle lui Morley……………………………………….………………...469

III.43. Triunghiul lui Grebe……………………………………….……………………474

III.44. Triunghiul lui Malfatti……………………………………….………………….476

III.45. Triunghiul lui Schroeter……………………………………….………………..482

III.46. Triunghiul lui Ţiţeica……………………………………….…………………..483 III.47. Triunghiurile lui Napoleon. Punctele lui Fermat……………….……………….485

III.48. Triunghiurile lui Vecten……………………………………….………………..493

III.49. Triunghiurile lui Saryghin……………………………………….………………499

Bibliografie……………………………………………………………………………..504